100% found this document useful (2 votes)
2K views539 pages

Hidden Facts in Mathematics

This document is a mathematics textbook by M. A. Otumudia, aimed at addressing challenges in understanding the New SSCE Mathematics syllabus. It includes a comprehensive collection of past examination questions, detailed explanations of mathematical concepts, and exercises for practice. The book covers various topics from fractions to calculus, providing answers and mathematical tables at the end.
Copyright
© © All Rights Reserved
We take content rights seriously. If you suspect this is your content, claim it here.
Available Formats
Download as PDF, TXT or read online on Scribd
100% found this document useful (2 votes)
2K views539 pages

Hidden Facts in Mathematics

This document is a mathematics textbook by M. A. Otumudia, aimed at addressing challenges in understanding the New SSCE Mathematics syllabus. It includes a comprehensive collection of past examination questions, detailed explanations of mathematical concepts, and exercises for practice. The book covers various topics from fractions to calculus, providing answers and mathematical tables at the end.
Copyright
© © All Rights Reserved
We take content rights seriously. If you suspect this is your content, claim it here.
Available Formats
Download as PDF, TXT or read online on Scribd
You are on page 1/ 539

HIDDEN FACTS

IN
NEW SSCE
MATHEMATICS
BY:
M. A. OTUMUDIA BSC MATHEMATICS EDUCATION UNI JOS

I
© OTUMUDIA PUBLISHERS LIMITED 2017
ISBN-978–978-958-649-3
All rights reserved. No part of this book may be reproduced, Stored in a retrieval system,
or transmitted in any form or by any means, recording, photocopying, or otherwise,
without the prior written permission of the publisher.

We are grateful as we acknowledge WAEC, NECO and NABTEB for being the source of questions
from past General Mathematics School Certificate Examination papers.

We are grateful to all those who contributed to the success of this book.

II
PUBLISHED BY:
OTUMUDIA PUBLISHERS LIMITED
NO 3 ADONOVWE STREET, OFF CENTRAL MOTOR PARK.
P. O BOX 540, UGHELLI NORTH,
DELTA STATE.
GSM NO. 08038633394, 08038633392
08022264074, 08022264072

ANNEX
NO 55B ISOKO ROAD
OPPOSITE ECOBANK
(ISOKO ROAD BRANCH)
UGHELLI

LAGOS STATE OFFICE:


28 AWOFESO STREET, BY MR.BIGGS, OFF SHEPELU STREET
PALM GROOVE BUS-STOP
SHOMOLU

OSUN STATE OFFICE :


REALITY HIGH SCHOOL
(OLUSESI MATHEMATICAL CENTRE)
KAYAFANDA AREA
BY AYESO POLICE STATION
ILESHA.

CONTACT HEAD OFFICE FOR INFORMATION ON LOCAL DISTRIBUTOR


AROUND YOU NOT MENTIONED ABOVE.

III
PREFACE
To boost the numerous effort by Mathematicians in addressing the perennial problems
of poor understanding and its attendant low grades in the subject, this book covers the
NEW SSCE syllabus directly including oversight areas.

A pool of past questions over the years on each topics were painstakingly collected ;
part of which were used as an additional examples to illustrate elaborately the required
concepts on each topics while similar ones were left as exercises.

Answers and mathematical tables are provided at the end of the text.

IV
Contents
Chapter one Chapter five
Fractions and approximation 1 Indices and logarithm 74
Fractions 1 Laws of indices 74
Word problems in fractions 4 Indices involving numbers and their powers 74
Decimal and significant figures 6 Indices in algebraic terms and algebraic powers 77
Percentage error 8 Indices involving equations 78
Indices involving simultaneous linear equations 80
Indices involving quadratic equations 81
Chapter two
Relationship between indices and logarithm 82
Commercial mathematics 10
Laws of logarithm 82
Ratio 10
Simplification in logarithm 83
Percentages 13
Change of base in logarithm 85
Loss 14
Substitution in logarithm 86
Cost 15
Working between indices and logarithm 88
Joint cases 16
Simple logarithmic equation 89
Commission, depreciation, exchange 17
Logarithmic simultaneous linear equations 90
Profit and loss 19
Logarithmic quadratic equation 91
Simple and compound interest 23
Problems on simple interest 23
Compound interest 26 Chapter six
Standard form & Logarithm II 92
Standard form 92
Chapter three Addition & subtraction in Standard form 92
Algebraic process 28 Multiplication & division in Standard form 93
Simple linear equation 28 Square root cases in Standard form 95
Expansion of Algebraic terms 32 Common logarithm 96
Coefficient of term in expansion 32 Logarithm of numbers greater than one 96
Factorization 33 Logarithm of numbers less than one 96
Difference of two squares expressions 34 Problems on logarithm II 97
Trinomial expression 35
Simplification of algebraic fractions 37
Substitution in algebraic expressions 42 Chapter seven
Undefined expressions 43 Number bases and surds 99
Expression equals to zero 44 Number bases 99
Change of subject formula 45 Conversion from base ten to others 99
Cases involving indices 45 Conversion of a given base to base ten 99
Cases involving non-indices 48 Conversion from base ten number plus decimal to others 99
Conversion of a given base number plus decimal to base ten 100
Chapter four Conversion from a non-base ten to another non-base ten 100
Simultaneous linear equations, Quadratics & Inequalities 50 Addition in bases 101
Simultaneous linear equations 50 Subtraction in bases 102
Uniform coefficient problems 50 Multiplication in bases 103
Non-uniform coefficient problems 51 Division in bases 104
Simultaneous linear equations in fractions 53 Problems of bases in equation form 104
Special cases in fractions 54 Problems of bases in simultaneous linear equations 106
Word problems 55 Miscellaneous problems 107
Quadratic equation I 55 Surds 109
Factorisation method 58 Addition, Multiplication & division in surd 109
Completing the squares method 62 Rationalization of surds 110
Making a given expression a perfect square 63 Simplification in surds 111
Formula method 63
Quadratic word problems 65 Chapter eight
Simultaneous linear & quadratic equations 65 Statistics I (Ungrouped data) 114
Inequalities I 66 Definition of terms 114
Inequality combination and ranges 69 Measure of central tendency 114
Inequalities involving quadratic expression 70 Arithmetic mean 114
Representation of inequality in a number line 70 Frequency mean 116
Single inequality number line 70 Median 117
Double inequalities number line 72 Frequency median 118
Mode 119

v
Measure of spread 119 Derived trig ratios 192
Range 119 Inverse or Arc trig ratios 196
Quartiles 120 Inverse trig ratios and equations 196
Mean deviation 120 Mathematical tables trig ratios problems 197
Variance and standard deviation 121 Trig ratios triangular problems 198
Miscellaneous cases on measure of spread & location 123 Right-angled triangular problems 201
Moving average 129
Chapter fourteen
Chapter nine Angles of elevation and depression 203
Sets 131 Single triangle cases involving angle of elevation 203
Types of sets 131 Single triangle cases involving angle of depression 207
Relationship between sets 132 Double triangles cases 208
Laws of sets 133
Problems on number sets 134 Chapter fifteen
Venn diagram 137 Bearing & Elements of plane geometry 211
Number Venn diagram 137 Elements of plane geometry 211
Venn diagram shading 138 Problems on angles formed between straight lines 212
Two subsets problems 139 Problems on angles formed between parallel lines 214
Three subsets problems 141 Problems on angles formed between lines in -
triangles, quadrilaterals 219
Chapter ten Bearing 223
Probability 145 Sine rule 223
Definition of basic terms 145 Cosine rule 224
General formula problems 145 Problems on sine and cosine triangles 225
Theoretical probability 146 Compass bearing 226
Empirical(experimental) probability 147 Three digit bearing 227
Addition rule 148 Relationship between three digits & compass bearing 227
Multiplication rule 150 Reversed bearing 228
General problems on probability 151 Problems on bearing I 229
Problems on bearing II 232
Chapter eleven
Sequence (AP & GP) and series 160 Chapter sixteen
Arithmetic progression (AP) 160 Latitude and longitude 238
nth term of an AP 160 Units of measurement 238
Terms formula for particular AP 162 Distance along great circles 239
Unknown a and d leading to equations 163 Distance along small circles 240
Sum of an AP 164 Joint cases on small and great circles 242
Geometric progression 167 Shortest distance between any two points-
nth term of a GP 167 on parallel of latitude 244
Unknown a and r leading to equations 168
Sum of a GP 169 Chapter seventeen
Sum to infinity 170 Statistics II (grouped data) 248
Problems on sequence 171 Definition of terms 248
Measure of location 248
Chapter twelve Measure of spread 250
Variation 173 Discrete & continuous data 253
Direct variation 173
Inverse variation 176 Chapter eighteen
Joint variation 178 Charts (Graphs) 253
Partial variation 180 Pie chart 253
Live diagrams in pie chart 255
Chapter thirteen Bar chart 257
Trigonometry 183 Live diagrams in bar chart 257
Special angles trigonometric ratios 184 Histogram 258
Substitution problems in special trig ratios 185 Cumulative frequency curve(O-give) 261
Trigonometric ratios of angles between 00 and 3600 187 Simultaneous linear equations graphical 265
Trigonometry of negative angles 189 Quadratic equations graphical 267
Identity 189 Trigonometric graph 283
Complementary angles 189 Sine graph 283
Problems on identities 190 Sine graph for amplitude multiples 283
Sine graph for periodicity multiples 283
vi
Cosine graph 284 Questions on triangles construction II 394
Cosine graph for amplitude multiples 284 Construction of triangles with ratio 395
Cosine graph for periodicity multiples 284 Construction of triangles with inscribe circle 395
Graph plotting in trigonometry 285 Construction of triangles with circum-circle 398
Linear inequalities graphs 289 Scaled, copied diagram construction 398
Quadrilateral construction 401
Chapter nineteen Triangle to quadrilateral construction 401
Plane and circle geometry 293 Trapezium construction 403
Plane geometry 293 Parallelogram construction 403
Congruent triangles 293 Rhombus construction 406
Similar triangles 294 Indirect quadrilateral construction 406
Intercept theorem 294
Mid point theorem 295 Chapter twenty three
Internal bisector of an angle of a triangle 301 Modulo Arithmetic 408
Polygons 301 Problems on modulo arithmetic 408
Problems on polygon 302
Other cases 307 Chapter twenty four
Circle geometry problems 310 Logic 412
Circle geometry problems II 324 Definition of terms in logic 412
Circle geometry problems III 329 Construction of truth table in logic 413
Logic and the use of Venn diagram 414
Chapter twenty
Plane mensuration 332 Chapter twenty five
Problems on Rectangle 332 Coordinate geometry 416
Problems on Square 336 Distance between two points 416
Problems on circle 337 Coordinate of mid – point 417
Problems on triangle 339 Gradient of a straight line 418
Problems on parallelogram 342 Equation of a straight line 419
Problems on trapezium 344 Equation of a straight line through two points 420
Problems on Rhombus 346 Parallel lines 421
Problems on length of an arc 347 Perpendicular lines 423
Problems on perimeter of a sector 350 Equation of a straight line through a given angle 424
Problems on area of a sector 351 Application of coordinate geometry and matrix to –
Problems on length of a chord 352 area of triangle & quadrilateral 426
Perimeter of segment of a circle 355
Area of segment of a circle 356 Chapter twenty six
Irregular plane shapes 358 Matrix 427
Definition of terms in matrix 427
Chapter twenty one Types of matrix 427
Solid mensuration 366 Addition and subtraction of matrices 428
Cylinder 366 Scalar multiplication in matrix 428
Surface area and volume of cylinder 366 Multiplication of matrix by a matrix 429
Hollowed cylinder, pipes and rings 369 Joint cases of addition, subtraction and multiplication 431
Cone 370 Transpose of a matrix 434
Problems on cones formed from a sector 370 Solving simultaneous linear equations with two –
Problems on cones 371 unknowns using 2×2 matrix 434
Problems on sphere and hemisphere 373 Determinant of matrix 436
Problems on triangular prism 375 2×2 matrix determinant 436
Problems on cube 377 Inverse of matrix ( 2×2 matrix) 437
Problems on cuboid 378 Solving simultaneous linear equations with –
Problems on pyramid 379 two unknowns using 2×2 inverse method 438
Problems on frustum 383 3×3 matrix determinant 439
Irregular solids 385
Chapter twenty seven
Chapter twenty two Vectors 441
Construction 389 Scalar, vector product 441
Foundation facts in construction 389 Addition of vectors 442
Questions on triangles construction 390 Sum, subtraction & scalar multiplication in vectors 442
Special angles and locus 392 Problems on parallel vectors 443
Parallel line & line ratio construction 394 Problems on projection, equal & angle b/w vectors 444

vii
Problems on magnitude, dot product of vectors 446 TABLES 512
Problems on perpendicular condition of vectors 446 ANSWERS 524
Problems on unit vectors 447
Problems on joint cases of vectors properties 448
Problems on position vectors 452
Problems on vectors and plane shapes 454
Problems on vectors and coordinate geometry 456

Chapter twenty eight


Mapping 457
Definition of terms in mapping 457
Types of mapping 457
Determination of mapping position 458
Value of a function 460
Composition of function 462
Inverse of a function 463
Joint cases 464

Chapter twenty nine


Remainder, factor theorems and partial fractions 466
Remainder and factor theorems 466
Remainder cases 467
Partial fractions 468
Cases with quadratic denominator that –
can be factorized 469
Cases with quadratic denominator that –
has repeated roots 471
Cases with quadratic denominator that –
cannot be factorized 471
Cases with degree of numerator greater –
than or equal to the denominator 473

Chapter thirty
Differentiation 474
Differentiation by first principle 474
Basic rules in differentiation 476
Chain rule problems 478
Product rule problems 480
Cases involving Chain and Product rules 482
Quotient rule problems 482
Cases involving Quotient and Chain rules 483
Application of differentiation (gradient) 484
Turning points (maximum & minimum) 485
The value of y for maximum and minimum points 487
Value of x, y for maximum and minimum points 489
Implicit functions 490
Implicit functions in gradient at a point 491
Comparing rates of change 492
Curve sketching 493
Chapter thirty one
Integration 495
Indefinite integral I 495
Indefinite integral II(change of variable) 496
Definite integral I 498
Definite integral II 499
Area under a curve 502
Area bounded between two curves 506
Volumes 511

viii
CHAPTER ONE 1999/3 (Nov)
3  1 1 
Fractions and Approximation Simplify 6   21  2 − 21
Fractions 4  3 4 
Fractions are of the form
a 3 1 4
b A 6 B4 C D
4 4 27
Where a is the numerator and b is the denominator (b  0)
Solution
Types of fraction are: First, we change mixed numbers to improper fractions.
Proper improper mixed numbers 27  64 9 
=   − 21
2 5 3 29 2 5 4  3 4 
, , 1 , 3
3 29 2 5 3 29 Applying BODMAS, bracket first. Inside the bracket,
The basic arithmetic rules of addition, subtraction, we apply BODMAS also and there multiplication 1st.
division and multiplication apply to them. If we are to 27
perform more than one operation of the aforementioned, =  (48 − 21)
then BODMAS pattern must be strictly adhered to. 4
27
2014/1 =  27
Simplify 10 2 − 6 2 + 3 4
5 3 27 1 1
A 6 415 B 6 1115 C 7 415 D 7 1115 =  = (C)
4 27 4
Solution
We change mixed numbers to improper fractions 2001/1a (Nov)
52 20 2 1 1 1
= − + 3 of 2  3 − 1
5 3 Simplify 1
3 4 2 4
LCM of the denominators 5 and 3 is 15
Solution
156 − 100 + 45
= First, we change mixed numbers to improper fractions.
15
101 = 5 of 9  7 − 5
= = 6 1115 (B) 3 4 2 4
15 Applying BODMAS, of 1st
5 9 7 5
2011/36 =   −
3 4 2 4
3  1 
Simplify 1 −  2 + 4 Next division, we change 
7
to 
2
4  3  2 7

A 3 512 B 2 7 12 C − 4 712 D 3 1 2 E − 5 512 = 15  2 − 5


4 7 4
Solution 15 5
First, we change mixed numbers to improper fractions. = −
14 4
= 7 −  7 + 4  Applying LCM of 14 and 4
4 3  −5
= 2  24 =
Applying BODMAS, bracket first 3 28
= 7 − 19
4 3 2004/2 Neco
Applying LCM of 4 and 3 i.e 12 3 3 2 1
Simplify 1 + 3  2  5
= 21 − 76 = − 55 = − 4 7
12 (C) 7 4 5 4
12 12
2010/8 A 1 37 B 2 49 C 3 7 D 3 3 7
18 1 E 5 14
1 1 3 1 Solution
Simplify 1 + 2  − First, we change mixed numbers to improper fractions.
2 3 4 2 10 15 12 21
= +  
A − 2 13 B − 2 4 C 2 8 D 2 34
1 1
7 4 5 4
Solution Applying BODMAS, division first, we change  21 to  4
We change mixed numbers to improper fractions. 4 21
3 7 3 1 10 15 12 4
= +  − = +  
2 3 4 2 7 4 5 21
Applying BODMAS, multiplication comes first Effecting multiplications first
3 7 1 10 12
=
2
+
4

2
= +
7 7
Applying LCM to the denominators
6 +7 − 2
10 + 12 22 1
= =
11
i.e 2 34 ( D ) = = = 3 (C)
4 4 7 7 7
1
2005/1 2006/1 Neco (Dec)
1 3 2 1 3 1 4
Simplify 15 − 6  2 + 5 Evaluate: 2  of 
2 4 3 2 4 3 5
A − 16 1 2 B − 2 12 C 2 1 2 D 16 1 2 A 1 7 25 B 25 C 1 2 D 7 25 E 7 32
32
Solution Solution
First, we change mixed numbers to improper fractions. Applying BODMAS, of first, while we change the mixed
31 27 8 number to improper fraction.
= −  +5 5 1 4
2 4 3 =  
Applying BODMAS, multiplication first. 2 4 5
31 Applying BODMAS, division 1st, we change  4 to  5
= − 18 + 5 5 4
2 5 1 5
31 5 =  
= − 13 = = 2 1
2 (C) 2 4 4
2 2
2005/23 Neco = 2532 ( B )
1 3 1 1 2007/9 Neco
Simplify 2 + 3  4 − 1 1 1  5 1
2 4 2 3 Simplify 2 + 1 of 1 − 
A 118 B 1 C 1 512 D 2 E 2 1 6 2 3  6 3
Solution A 5 34 B 4 12 C 3 2 3 D 3 12 E 2 34
First, we change mixed numbers to improper fractions. Solution
5 15 9 4 First, we change mixed numbers to improper fractions.
= +  −
2 4 2 3 5 4  11 1
= + of  − 
Applying BODMAS, division first, we change  9 to  2 2 3 6 3
2 9
Applying BODMAS, bracket first
5 15 2 4
= +  − 5 4  11 − 2 
2 4 9 3 = + of  
2 3  6 
Effecting the multiplication
5 5 4 5 4 9
= + − = + of
2 6 3 2 3 6
Since we are left with addition and subtraction, we Applying BODMAS, of first
apply LCM to the denominators 5 4 9
= +  Effecting the multiplications
15 + 5 − 8 12 2 3 6
= = = 2(D)
6 6 5 9
2005/ 1 (Nov)
= + 2 = = 4 12 ( B )
2 2
2  7 1  4 2009/1a (Nov)
Evaluate:  1  1 
7  8 14  7 Simplify, without using tables or calculator:
4 2 8 32 1 1 4
A B C D
3 1 −
7 7 49 343 3 4 9
Solution Solution
First, we change mixed numbers to improper fractions. We change mixed numbers to improper fractions.
2  15 15  4 10 5 4
  =  −
=  3 4 9
7 8 14  7
Applying BODMAS, division first, we change  5 to  4
Applying BODMAS, bracket first, 4 5
2  15 14  4 10 4 4
=    =  −
7 8 15  7 3 5 9
= 2  7  4 Working with multiplication first
7 4 7 8 4
Applying BODMAS, division first, we change  7 to  4
= −
4 7
3 9
2 4 4 24 − 4 20 2
=   = = i.e 2
7 7 7 9 9 9
32
= (D)
343
2
2009/1 Neco (Dec) We introduced bracket to separate numerator from
1 1 3 1 denominator. Applying BODMAS to the last bracket implies
Simplify 4  3   +  multiplication first
3 4 4 3
 45 + 28   13 3 
A 1213 B 1 112 C 1 19 D 1 512 E 1 4 9 =     − 
Solution  36  9 8
Change mixed numbers to improper fractions. 73  104 − 27 
=  
13 13 3 1 36  72 
=    + 
3 4 4 3 73  77 
Applying BODMAS, bracket first =  
36  72 
13 13 9 + 4 
=     Changing from division to multiplication
3 4  12  73  72  146 69
13 13 13 =   = i.e 1
=   36  77  77 77
3 4 12 2011/ 1a
Applying BODMAS, division first, we change  13 to  4 1 1 1
4 13 of 
13 4 13 13 Simplify 2 4 3
=   = = 149 ( E ) 1 3 1
3 13 12 9 − +
6 4 2
2012/5b Neco Solution
2 1 1 We solve numerator and denominator differently.
3 1 5 Applying BODMAS “of ” first
3 8  2
Simplify 1 1
2 1 7 
3 − 2 3 8 3
3 4 9 =
1 3 1
Solution − +
The whole problem is separated by  . Change mixed 6 4 2
numbers to improper fractions Applying LCM to the denominator and invert 1/3 to 3/1 to
11 9 11 change from  to  in numerator
 1 3 3
=
3 8  2 
11 9 34 8 1 8 3  1
− = = i.e  − 
2−9+6 1 8  12 
3 4 9 −
Next, we solve numerator and denominator differently 12 12
33 Changing from division to multiplication
8 11 9 3  12 
=   changed from division to  =  − 
44 − 27 2 34 8  1
12 9
= – = – 4 12
33 12 11  9 2
=  
8 17 2  34 2009/12 NABTEB (Dec)
Arrange 5/6, 2/3 and 3/8 in descending order of magnitude
Changing from division to multiplication
A 2/3, 3/8, and 5/6 B 3/8, 2/3, and 5/6 C 5/6, 2/3, and 3/8
33 12 2  34 D 5/6, 3/8, and 2/3
=   = 2
8 17 11  9 Solution
To know the value of their magnitude, we multiply through
2012/ 1a by the LCM of the dominators 6, 3, and 8
1 7 3 3 6 8
+1 2 1 2 8
Simplify
4 9 2 1 1 4
4 2 9 2 1 1 2
1 −2 
9 3 64 1 1 1 LCM is 3  2  2  2 = 24
Solution 5 2 3
The whole problem is separated by  . Change mixed
=  24 ,  24 ,  24
6 3 8
numbers to improper fractions = 20, 16 and 9
 5 7   13 8 9  They are in descending order already
=  +    −  
3 64 
5
/6, 2/3, and 3/8 ( C )
4 9 9
3
2008/2 Neco 2015/ 34 (Nov) Exercise 1.9
Arrange the following numbers in ascending order: 1 1 2 1 3
Simplify 3 − 4 + 2   3
1
8 , 17 , 1
4 , 1156 , 1756 8 4 3 6 4
1 , 17 , 1 , 1156 , 1756 1 , 17 , 1156 , 1 4 , 1756 4 13
A 8 4 B 8 A 1 B 5 C D 2
40 12 15 40
C
1
8 , 1156 , 1
7 , 1 4 , 1756 D 1 , 17 , 1156 , 1756 , 1
8 4 2015/1a Exercise 1.10
E 1
8 , 7,
1 1
4 , 17 ,
56
11
56 Without using mathematical tables or calculators
Solution simplify : 3 94  (5 13 − 2 34 ) + 5 10
9

We multiply through by the LCM of the denominators


8, 7, 4, 56, i.e 56 2014/5 Neco ( Nov ) Exercise 1.11
= 18  56, 17  56, 1 4  56, 1156  56 , 17 56  56 Simplify ( 43 + 32 )  3 12  3 14
= 7, 8, 14, 11, 17 A 2 39 B 2 3 39 C 2 1939 D 3 2 39 E 4 2 39
Arrange ascending order: 7, 8, 11, 14, 17
1
8 , 17 , 1156 , 1 4 , 1756 ( B) Word problems
2014/10
2009/4 (Nov) Exercise 1.1 Three quarter of a number added to two and a half of that
number gives 13. Find the number.
11 11 6 4
Simplify  −  A4 B5 C6 D7
8 7 5 5 Solution
37
A 269 B 1 11 C D 1 19 Let the number be x, we are to find x in: 3 x + 2 1 x = 13
280 269 56 37 4 2
2010/6b Neco Exercise 1.2 i.e x + 5 x = 13
3
4 2
2 13
4 −1 Multiply through by LCM 4 to clear fractions
Simplify 9 15 3x + 10x = 52
1 4 1 13x = 52
2 + of 2
5 7 3 x = 52 i.e 4
2011/ 4a Neco adjusted Exercise 1.3 13
4 2 1 2011/12b
Simplify 2 of  4
5 3 5 Ade received 3 of a sum of money, Nelly 1 of the
5 3
2009/1 Exercise 1.4
remainder while Austin took the rest. If Austin’s share is
 1 1  5 2 greater than Nelly’s share by N 3,000, how much did Ade
Simplify:  3 + 4    − 
 2 3  6 3 received?
A 1 14 B 8 12 C 35 D 47 Solution
Let the money be x
2007/ 6a Exercise 1.5
Ade’s share: 3 x
1 7 5
3 +
Simplify 12 8 Nelly’s share: 1 of  3 
1x − x  i.e 1
of  2 
 x = 2
x
1 1 3  5  3  5  15
2 −
4 6 3 1 
Austin’s share: 1x – x –  of (1x − 3 5 x)
2006/1a NATEB (Dec) Exercise 1.6 5 3 
1 2 2 4
1 i.e x – x = x
Simplify 4 5 15 15
1 The last statement is
2+ of 28
4 Austin’s share – Nelly’s share = 3000
2006/1a Exercise 1.7 4 2
x – x = 3000
2 13 15 15
4 − 1  4 − 2
Simplify 9 15   x = 3000
1 4 1  15 
2 + 2
5 7 3 2
x = 3000
15
2005/40 (Nov) Exercise 1.8 3000  15
1 1 3 x = i.e N 22,500
2 + −1 2
Simplify 2 3 4
1 1 1 Thus Ade’s share = 3 x
+1 −1 5
2 3 4
= 3  22,500 i.e N 13,500
7
A 13 B 12 C 1213 D 1 6 7
7
5
4
2010/ 2Neco 2004/6(c)
What fraction of 5 weeks is 5 days? A man left N5,720 to be shared among his son and three
A7 B1 C 1/6 D 1/7 E 1/10 daughters. Each daughter’s was ¾ of the son’s share. How
Solution much did the son receive?
One week is 7days Solution
5 week is 7  5 = 35 days Let the son’s share be P
Thus 5 days fraction of 5 weeks = 5 = 1
/7 ( D ) Each daughter share will be 3 p .
35 4
2008/ 12 Neco (Dec) The three daughter’s share will be 3 
3 p  i.e 9 p
A civil servant spent 2/7 of his monthly salary on clothes, 4  4
7
/20 of the remainder on feeding and save the rest. If he Sum of shares = total amount
spent N 7, 000.00 on feeding, find his monthly salary
A N28,000.00 B N32,000.00 C N40,000.00 But P + 9 p = 5720
4
D N 48,000.00 E N49,000.00 13
Solution p = 5720
4
Let the salary be x 5720  4
Clothes: 2 of x i.e 2 x P =
13
7 7
= N 1,760 son’s share
7  2  7 5 1
Feeding: of  x − x  i.e  x = x
20  7  20 7 4 2011/ 11 Neco
A poultry farmer discovered that 1/6 of the eggs laid by the
This step not necessary in this question birds in the farm were bad. Find the percentages that were good.
A 83.33% B 16.66% C 0.83% D 0.66% E 0.16%
2 1
Savings: x – x – x i.e  28 − 8 − 7  x = 13
x Solution
7 4  28  28 Bad eggs percentage = 1/6  100
If he spent N 7,000.00 on feeding, find his monthly… = 16.66%  16.67%
1 Thus good eggs percentage = 100 – 16.67
x = 7000, find x = 83.33% A
4
Alternatively
x = 7000  4
Good eggs percentage = 1 – 1/6 (100)
= N28, 000.00 ( A )
2007/ 7b Neco = 5  100
6
A quarter of farmland is too rough to use. If two-fifth of = 83.33%
the remainder is kept for poultry farms and this leaves 1 – 1/6 is used as the whole (Total) egg laid in the farm is in
221/2 acres for planting cash crops, what is the area of fraction term 1.
the farm?
Solution 2015/4a Exercise 1.13
Let the farmland be x By how much is the sum of 3 23 and 2 15 less than 7 ?
1
Rough part: x 2014/4 Neco ( Nov ) Exercise 1.14
4
What fraction must be added to the sum of 3 12 and 4
to give
1
?
2  1  2  3  3 5 2
Poultry farm: of  x − x  i.e  x = x A 19 B 7 C 5 D 4 E– 19
5  4  5  4  10 5 2 4 5 5

20 x − 5 x − 6 x 9
Cash crops: x – 1 x – 3 x i.e = x 2014/7a Neco ( Nov ) Exercise 1.15
4 10 20 20 Ade, Tayo and Uche jointly owned a business and shared
Next, cash crops part is 221/2 acres the profit such that Ade received 1/3, Tayo received 7/12 and
9 Uche received 1/12. If Ade received N 7,000.00 more than Uche,
Thus x = 221/2 find x what was the total profit and how much did each received?
20
9 45 2015/39 Exercise 1.16
x = A farmer uses 2/5 of his land to grow cassava, 1/3 of the
20 2
remainder for yam and the rest for maize. Find the part of
45  20
x = land used for maize
29 A 2/15 B 2/ 5 C 2/ 3 D 4/ 5
= 50 acres
2015/2 (Nov) Exercise 1.16b
2006/9 Exercise 1.12 A man spent 1 of his monthly salary on rent, 2 on food and 1
A baker used 40% of a 50kg bag of flour. If 1/8 of the 4 5 6
amount used was for cake, how many kilograms of flour on his children’s education. What fraction of his salary is left?
was used for cake? 7
A 11 B 13 C D 19
A 21/2 B 61/4 C 155/8 D 171/2 60 60 20 30
5
Approximation Solution
Decimal and Significant figures In significant figures, our concern is the first figure, not zero
These twin concepts are working tools for presenting or decimal point. So we take 247 for 3s.f and approximate
most answers in mathematics. The author deliberately 0.002473  0.00247 ( C )
fixed them together to avoid the usual confusion We round down by adding nothing 247
associated with them. Once the next digit to the one Since after 247 in 0.002473 we have 3 which is below 5
being considered is any of 5, 6, 7, 8, 9; we round up i.e.
adding 1 to our digit. 2005/3 (Nov)
On the other hand, if the digit is any of 0, 1, 2, 3, 4 we Which of these numbers is correct to three significant figures?
round down, i.e adding nothing to our digit. A 0.003 B 0.0034 C 0.012 D 3.04
Examples. Solution
Approximate the following 0.003 is correct to 3 d.p and correct to 1 s.f but not 3s.f
(a) 65009.269 to 0.0034 is correct to 4 d.p and correct to 2s.f
(i)1d.p (ii) 1s.f 0.012 is correct to 3 d.p and correct to 2 s.f
3.04 is correct to 3 s.f D. (also it is correct to 2 d.p)
(b) 24.543 to
(i) 2 d.p (ii) 2 s.f. 2014/34
Approximate 0.0033780 to 3 significant figures
(c) 0.001658 to A 338 B 0.338 C 0.00338 D 0.003
(i) 3d.p (ii) 3 s.f Solution
Analysis and Solution In significant figures, our concem is the first figure, not zero
or decimal point. So we take 337 for 3s.f and approximated
a. (i) 65009.269  65009.3 to 1d.p
there
Here, we considered the first digit after the decimal
0.0033780  0.00338 to 3s.f C
point. Since the number after it is 6, we rounded up by
We round up by adding 1
adding 1 to 2. The last sign is the approximation sign.

a. (ii) 65009.269  70000.000 to 1 s.f 2012/1


b. (i) 24.543  24.54 to 2 d.p Express 302.10495 correct to five significant figures
Here, the two digits after decimal point are five four A 302.10 B 302.11 C 302.15 D 302.1049
(Not fifty-four). The last of the two is 4 and next to it is Solution
3, thus, we round down. In significant figures, our concern is the first non zero figure,
so we take 302.10 for 5 s.f and approximate
b. (ii) 24.543  25. 000 to 2 s.f
we rounded up 302.10495  302.10 ( A )
We rounded down here, reason after the last 0 in
c. (i) 0.001658  0.002 to 3 d.p.
302.10495 is 4 which is not up to 5 and above
(ii) 0.001658  0.00166 to 3 s.f
2004/1 Neco
2010/ 13 Neco
Express the product of 0.007 and 0.057 to two significant
Express 0.00629946 to 3 significant figures
figures
A 0.000 B 0.006 C 0.006210
A 0.00040 B 0.00039 C 0.0040 D 0.0039 E 0.040
D 0.00629 E 0.00630
Solution
Solution
0.007  0.057 = 0.000399
Significant figures, our starting point is first non-zero
digit. Take 629 to approximate to 3s.f  0.00040 to 2 s.f (A)
0.00629946  0.00630 E
2006/1
Since the number after 629 is 9 we round up by adding
Evaluate (0.13)3 correct to three significant figures.
1 to 9 making 629 to be 630
A 0.00219 B 0.00220 C 0.00300 D 0.00390
Solution
2010/3 NABTEB (Dec)
Round off the following 51065 to 3 significant figures (0.13)3 = 0.0 0 2 1 9 7
A 51000 B 51100 C 51200 D 51300 Three significant figures: 1st 2nd 3rd 0
Solution Next to the last digit 9 is 7, so we round up by adding 1 to it.
In significant figures, our concern is the first figure, not 0.002197  0.00220 to 3s.f (B)
zero or decimal point. So we take 510 for 3s.f
51065  51100 ( B ) 2008/4 Neco (Dec)
We round up by adding 1 to 510 since after 510 we have 6 6.42 + 2.13
Evaluate , correct to two significant figures
4.1 − 2.85
A 6.9 B 6.8 C 5.9 D 5.8 E 5.4
2010/7 Solution
Correct 0.002473 to 3 significant figures Solving the numerator and denominator differently:
A 0.002 B 0.0024 C 0.00247 D 0.0025
6
6.42 + 2.13 8.55 2013/2 Neco
= Sum up the following numbers 0.032, 4.154, 6.0
4.1 − 2.85 1.25
and 0.3065, to two decimal places
= 6.8 4 A 10.00 B 10.40 C 10.49 D 10.50 E 11.00
Two significant figures: 1st 2nd 0 Solution
Next to the last digit 8 is 4, so we round down by 0 . 0 3 2 *
adding nothing to it. +4 . 1 5 4 *
Thus 6.84  6.8 to 2 s.f ( B ) 6 . 0 * * *
2008/4 NABTEB (Dec) 0 . 3 0 6 5
Express the difference between 1 7 and 17 10 . 4 9 2 5
25 300 In decimal place, we start counting after decimal point,
to 3 significant figures? whether it is zero or not so we take 10.49 for 2d.p and
A 1.22 B 1.23 C 0.12 D 0.13 approximate
Solution 10.4925  10.49 to 2d.p ( C )
7 – 17 = 32 – 17
1
25 300 25 300 2007/6 Neco
Changing to decimal 1 3 5 7
= 1.28 – 0.05667 Evaluate + + + , correct to three decimal
= 1.2 2 333 2 20 200 2000
1st 2nd 3rd 0 0 0 places.
Next to the last digit 2 is 3, so we round down by A 0.685 B 0.680 C 0.679 D 0.678 E 0.670
adding nothing to it. Solution
Thus 1.22333  1.22 to 3s.f A Since we are working in decimal places we change fractions
to decimals having our 3 d.p in mind
2011/1 Neco 1 3 5 7
Express the sum of 0.0714 and 0.0163 in two significant + + + = 0.5 + 0.15 + 0.025 + 0.0035
figures. 2 20 200 2000
A 0.080 B 0.086 C 0.087 D 0.088 E 0.090 = 0.6 7 8 5
Solution Three decimal place : 1st 2nd 3rd 0
0.0714 Next to the 3rd digit 8 is 5, so we round up by adding 1 to it.
+0.0163 0.6785  0.679 to 3d.p
0.0877
In significant figures, our concern is the first figure, not zero 2005/1 Neco
or decimal point. So we take 87 for 2 s.f and approximate Express 329,761 to the nearest thousand
0.0877  0.088 to 2s.f ( D ) A 300,000 B 320,000 C 329,000 D 329,700
We round up by adding 1 to 7 in 87 since the number E 330,000
after 87 is 7 Solution
The place value of thousand here
2008/5 NABTEB (Dec) 329, 761
Express 42.467 to 2 decimal places 000
A 424.67 B 42.50 C 42.47 D 42.46 is 329. The next number to it is 7. Thus we round up by
Solution adding 1
Decimal place, our starting point is the decimal point.
Immediately after decimal point, we start counting. So we 329,761  330,000 to the nearest thousand (E)
take .46 for approximation to 2 d.p
42.467  42.47 ( C ) 2005/ 2
Since the number after .46 is 7 we round up by adding 1 to 6 Find, correct to the nearest kobo, 85.23% of N 50
A N 43.00 B N 42.60 C N42.61 D N 42.62
2010/1 Neco Solution
Simplify 0.0589 + 7.382 – 0.7953 correct to 2 decimal places.
First convert N 50 to kobo
A 6.60 B 6.64 C 6.65 D 8.20 E 8.24
N 50 = 5000 kobo
Solution
0.0589 85.23
85.23% of N 50 =  500
7.382* 100
7.4409 = 4261.5 kobo
–0.7953 Divide by 100 to get N
6.6456 = N 42.615 kobo
In decimal places approximation, our concern is the  N 42.62 ( D )
decimal point. Immediately after decimal point, we start
counting. So we take .64 for approximation to 2 d.p
6.6456  6.65 to 2 d.p ( C )
Since the number after .64 is 5 we round up by adding 1
to 4 in .64 to become .65
7
2006/ 23 Percentage error
What is the place value of 9 in the number 3.0492? This is as a result of little degree of error allowed in
A 9 B 9 C 9 D 9 measurement
10000 1000 100 10 PE = wrong value – actual value x 100
Solution actual value
The decimal place value system of numbers is PE = Error x 100
1st 2nd 3rd 4th positions Exact
3 . 0 4 9 2 2008/11 Neco ( Dec)
Tenth hundredth thousandth A rope which is 4.85m long was measured by a boy as 4.95m.
9 What is his percentage error?
Thus the position of 9 is thousandth i.e (B)
1000 A 21/50% B 23/50% C 26/97%
3 6
2011/ 21 D 20 /50% E 20 /97%
What is the value of 3 in the number 42.7531? Solution
3
A 3 B C 3 D 3 error
 100
10000 1000 100 10 P.E =
exact
Solution
4.95 − 4.85
The decimal place value system of numbers is =  100
1st 2nd 3rd 4th positions 4.85
4 2 . 7 5 3 2 0.1
Tenth hundredth
thousandth =  100
Thus the position of 3 is thousandth i.e 3 (B) 4.85
1000 Working in fraction, since our options are in it
2015/29 Neco Exercise 1.17 10
The number 192039 was corrected to 192000 by a =
4.85
student. Which of the following can be the correct To clear decimal, we move decimal point twice at the
approximation of figures taken? denominator and do same at the numerator.
I. to the nearest hundred II. to the nearest thousand 1000 200
III. to 3 significant figures IV. to 4 significant figures = i.e = 26/97% ( C )
A. I and II only B. II and III only 485 97
C. III and IV only D. I, II, III & IV 2011/20
The length of a piece of stick is 1.75m. A girl measured it as
2014/1 (Nov) Exercise 1.18 1.80m. Find the percentage error
Correct 0.005854 to 2 significant figures 284 29 20
A. 0.0058 B. 0.0059 C. 0.0060 D. 0.0100 A % B % C 5% D %
7 7 7
Solution
2014/18 NABTEB ( Nov ) Exercise 1.19
error
Evaluate 0.8916  5.36 and round off to two significant P.E =  100
figure exact
A. 0.19 B. 0.18 C. 0.17 D. 0.16 1.80 − 1.75
=  100
1.75
2014/3 Neco Exercise 1.20
0.05
0.042  0.5 =  100
Evaluate and approximate your answer 1.75
0.04  0.14
We are working in fractions
to one decimal places 5 500 20
A. 0.38 B. 3.75 C. 3.80 D. 37.50 E. 375.00 = = i.e % (D)
1.75 175 7
2003/1 Neco ( Nov ) Exercise 1.21 To remove decimal, we move decimal point twice at the
Express the number 0.0099687 correct to four decimal denominator and do same at the numerator.
places 2004/10 Neco
A. 0.0100 B. 0.0099 C. 0.0090 D. 0.0010 E. 0.0009 A sales boy gave a change of N 75.00 to a buyer instead of
N80.00, calculate his percentage error, correct to one
2003/7 Neco ( Nov ) Exercise 1.22 decimal place.
The difference between 0.005348 correct to 3 A 6.0% B 6.2% C 6.3% D 6.6% E 6.7%
significant figures and 0.005384 correct to 4 decimal Solution
places is error
A. 0.0001 B. 0.0004 C. 0.0005 D. 0.00004 E. 0.00005 P.E =  100
exact
2003/6 Neco ( Nov ) Exercise 1.23 80 − 75 5
Correct 5.0962894 to three significant figures =  100 =  100
80 80
A. 5.96 B. 5.10 C. 5.09 D. 5.02 E. 5.00
= 6.25%  6.3% to 1 d.p
8
2007/ 3 Neco Let the actual value be x; PE is 5%
A boy when rounding up a number wrote 98 instead of 5 3.62 − x
980 (to 2 significant figures). What is the percentage =
error? 100 x
A 0% B 10% C 11.1% D 90% E 96% 5x = 100(3.62 – x)
Solution 5x = 362 – 100x
error 5x + 100x = 362
P.E =  100 105x = 362
exact
362
980 − 98 x = i.e 3.4476km  3.45km ( B )
=  100 105
980
882
=  100 = 90% (D)
2002/38 Neco ( Nov ) Exercise 1.24
980 During a chemistry practical, a student recorded his reading
2012/4 Neco as 18.13cm3 instead of 18.31cm3. Calculate his percentage
A man estimated his transport fare for a journey as error
N 210.00 instead of N 220.00. Find the percentage error A 9.8% B 1.8% C 0.98% D 0.18% E 0.098%
in his estimate, correct to 3 significant figures.
A 4.54% B 4.55% C 45.4% D 45.5% E 45.6% 2014/41 NABTEB ( Nov ) Exercise 1.25
Solution A string is 4.8m. A boy measured it to be 4.95m. Find the
error percentage error
P.E =  100 A 5/16% B 15/16% C 31/33% D 31/8%
exact
220 − 210 2013/7 Exercise 1.26
=  100
220 A sales boy gave a change of N 68 instead of N 72 .
10 Calculate his percentage error
=  100 A 4% B 55/9% C 515/17% D 7%
220
= 4.54545%  4.55% to 3 s.f 2004/44 Neco ( Nov ) Exercise 1.27
2005/21 Neco A man made a table with a rectangle top of dimension
A woman underestimated the cost of a pair of shoes and 36cm by 44cm instead of 37cm by 41cm. What is the
her percentage error was 183/4%. If the actual price of percentage error in the perimeter of the table correct to
the shoe was N 64.00, what was her underestimate? 1 decimal place?
A N12.00 B N 17.92 C N 52.00 D N52.48 E N 76.00 A 11.1% B 10.8% C 4.4% D 2.6% E 2.5%
Solution
Let the underestimation be x
Underestimation here implies wrong price is lower than
the actual price.
error
P.E =  100
exact
64 − x
183/4% =
64
75 1 64 − x
 =
4 100 64
75  64 = 400 (64 – x)
4800 = 25600 – 400x
400x = 25600 – 4800
x = 20800/400 i.e N 52.00 ( C )
2010/4 Neco
The distance between two points is measured to be
3.62km. If this is more than the actual distance and the
percentage error is calculated to be 5. What is the actual
distance?
A 3.18km B 3.45km C 8.62km D 3.80km E 3.81km
Solution
error
P.E =  100
exact
wrongvalue − actual
P. E =  100
actual value
9
CHAPTER TWO In division form
13 + x 4
Commercial mathematics =
RATIO 16 + x 5
This is the act of comparing like quantities such as 5(13 + x ) = 4(16 + x)
weights, lengths, money etc. 65 + 5x = 64 + 4x
The ratio m : n is represented in fraction as m 5x – 4x = 64 – 65
n x = –1
In ratios, m : n  n : m Our new ratio becomes
Similarly, the ratios m : n : r is expressed in fraction as 13 − 1 12
= i.e 4/5
m : n and m 16 − 1 15
n r r 2006/42 Neco
What number must be added to each term of the ratio
2009/ 11 Neco 11 : 15, so that it becomes the ratio 4 : 8?
If it costs three shop keepers A, B and C N 918,000.00 A–7 B 4 C 7 D 8 E 17
to purchase a common generator for their three shops Solution
and the amount was shared in the ration 1: 2: 3 Let the number be x
respectively. How much would shopkeeper B pay? 11 + x : 15 + x = 4 : 8
A N 316,000.00 B N 312,000.00 C N 309,000.00 In division form
D N 306,000.00 E N 303,000.00
11 + x 4
Solution =
Shop keeper B ratio is 2 15 + x 8
Sum of ratios 1 + 2 + 3 = 6 8(11 + x ) = 4(15 + x)
2 88 + 8x = 60 + 4x
Amount shop keeper B pays =  918000 8x – 4x = 60 – 88
6 28
= N 306, 000.00 ( D ) x = − = –7 (A)
4
2006/ 36
2007/ 2 Neco
A seller allows 20% discount for cash payment on the
A, Q, R, and S shared a sum of money in the ratio
marked price of his goods. What is the ratio of the cash
3 : 4 : 6 : 7. If S received N 600.00 more than Q,
payment to the marked price?
A1:4 B1:5 C3:4 D4:5 Calculate the amount of money that was shared
Solution A N 3,000.00 B N 4,000.00 C N 6,000.00
D N 8,000.00 E N 12,000.00
Cash payment : marked price
Solution
Discount (100 – 20) % : 100%
Sum of ratios: 3 + 4 + 6 + 7 = 20
80 : 100
Ratio of S is 7
8 : 10
Ratio of Q is 4
4 : 5 (D)
Let the amount of money shared be x
2012/3 Then 7 of x – 600 = 4 of x
In 1995, the enrolments of two schools X and Y were 20 20
1,050 and 1,190 respectively. Find the ratio of the 7x 4x
i.e − 600 =
enrolment of x to y. 20 20
A 50 : 11 B 15 : 17 C 13 : 55 D 12 :11 7x 4 x = 600

Solution 20 20
School X : School Y 3x
= 600
1,050 : 1, 190 20
Next we reduce the values to their lowest term possible 600  20
105 : 119 i.e division by 10 x =
3
15 : 17 i.e division by 7 = N 4,000.00 ( B )
2007/12b NABTEB
2014/3a Neco The monthly profit of a transport business was shared
What number must be added to each term of the ratio between two partners, a husband and wife in the
13: 16 so that it becomes the ratio 4: 5? ratio 7 : 5. If the wife received N15,000.00 less than the
Solution husband, find out how much the husband received.
Let the number be x Solution
Note the question says “what number” and not what Sum of ratios: 7 + 5 = 12
numbers as this will make us use x and y Ratio of wife is 5
13 + x : 16 + x = 4 : 5 Let the amount of money shared be x
and not 13 + x : 16 + y = 4 : 5 7
Then 5 of x + 15000 = of x
12 12
10
7 5x 7x
i.e 15000 = x− = 6300
12 12 100
2x 6300  100
15000 = x =
12 7
15000  12 = N 90,000
= x
2 2013/3a
Amount shared is N 90,000 The present ages of a father and his son are in the ratio
7 10 : 3. If the son is 15years old now, in how many years will
Husband share is =  90000 the ratio of their age be 2 : 1 ?
12 Solution
= N 52,500 At present
2006/10 Neco (Dec) Father : son
A sum of money is to be shared among Victor, Faith 10 : 3
and Debby in ratio 2 : 11/2 : 21/2 respectively. If Faith We let the father age be x
receives N 10,000.00, what is the sum of the money? x : 15
A N 13,333.00 B N 16,670.00 C N 30,000.00 Changing ratio to division form
D N 40,000.00 E N 66,000.00
10 x
Solution =
Let the sum of money shared be x 3 15
the given ratios has two fractions, making them whole 10  15
will help. x =
3
2 : 11/2 : 21/2 i.e 2 : 3/2 : 5/2 = 50 years
it becomes 4 : 3 : 5 (We multiplied through by LCM 2) At present we have
Sum of ratios: 4 + 3 + 5 = 12 Father : son
Faith ratio is 3 50 years : 15 years
Thus 3 of x = 10,000 In the years y, when their ratio is 2: 1
12 We have
3x = 10,000  12 50 + y : 15 + y
x = 10,000  12 2 : 1
3
50 + y 2
= N 40,000.00 ( D ) =
2005/ 6 Neco 15 + y 1
The ratio of Bimbo’s age to her mother is 5 : 7. What is 50 + y = 2(15 + y)
Bimbo’s age if her mother is 63 years old? 50 + y = 30 + 2y
A 8.5 years B 12 years C 35 years 50 – 30 = 2y – y
D 45 year E 52 years 20 = y
Solution
2007/8
Bimbo : mother
N140,000 is shared between Abu, Kayode and Uche. Abu
5 : 7
has twice as much as Kayode and Kayode has twice as much
x : 63
as Uche . What is Kayode’s share?
5 : 7 = x : 63
A N 80,000 B N 40,000 C N 20,000 D N10,000
5 x
= Solution
7 63 Abu : Kayode : Uche
5  63 A : K : U
x = A = 2K : K = 2U : U
7 Then changing all the ratios to one letter
= 45 years ( D ) 2(2u) : 2u : u
2007/15b NABTEB 4u : 2u : u
Obi and Audu own a shop. The ratio of Obi’s share to
Audu’s is 13: 7. Later Audu sells 1 of his share to Obi But 4u + 2u + u = 140,000
5 7u = 140000
for N 6,300.00. Find the value of the shop u = 140000
Solution 7
Let the value of the shop be x u = N 20,000
Obi : Audu Kayode’s share is 2u i.e 2  20,000 = N 40,000 ( B )
13 : 7
Sum of ratios is 13 + 7 = 20 2015/27
1  7  Ann scores 1 /2 as many marks as Brenda and Brenda scores
of  of x  = 6300 1
/2 as many marks as Catherine. If the sum of their marks is
5  20  112, how many marks did each student score?
11
Solution Solution
A (Ann) B (Brenda) C (Catherine) x a
1
/2 B = A B = 1/2 c c x : y = a : 2 implies =
y 2
Then changing all the ratios to one letter
1
/2 (1/2 c) 1
/2 c c
2x
y= ----- (1)
the result is a
1
/4 c 1
/2 c c y 3
y : z = 3: b implies =
z b
But 1/4 c + 1/2 c + c = 112
To clear fractions, multiply through by 4 LCM of the denominators
3z
y= ---- (2)
c + 2c + 4c = 4  112 b
7c = 4  112 Equating terms in y from (1) and (2)
c = 4  112 i.e 64 2 x 3z
=
7 a b
Marks are A B C 2bx = 3az
16 : 32 : 64
x 3a
=
z 2b
2007/ 37 Equation ratio x : z is 3a : 2b (D)
If the ratio x : y = 3 : 5 and y : z = 4 : 7,
find the ratio x : y : z 2014/ 30
A 15 : 28: 84 B 12 : 20 : 35 C 3 : 5 : 4 D 5 : 4 : 7 If x : y = 3 : 2 and y : z = 5 : 4, find the value of x in
Solution the ratio x : y : z
The target here is to make y uniform value similar to A 8 B 10 C 15 D 20
what we do when solving simultaneous linear equations Solution
x : y = 3 : 5 ------ (1) x 3
and y : z = 4 : 7 -------(2) x : y = 3 : 2 implies =
RHS only (1)  4 and (2)  5 will make y uniform y 2
x : y = 12 : 20 2x
y : z = 20 : 35 y= ----- (1)
3
Equating ratios x : y : z becomes 12 : 20 : 35
y 5
y : z = 5: 4 implies =
Alternatively z 4
The target here is to make y uniform value 5z
x 3 y= ----- (2)
x : y = 3 : 5 implies = 4
y 5 Equating terms in y from (1) and (2)
5x 2 x 5z
y= -----(1) =
3 3 4
y 4 8x = 15z
y : z = 4 : 7 implies =
z 7 x 15
=
4z z 8
y= -----(2) Equating ratio x : z is 15 : 18
7
Equation terms in y from (1) and (2) Thus x is 15 (C)
5x 4 z 2014/21 NABTEB ( Nov ) counter example
=
3 7 A sum of money is shared among ten students so that the
35x = 12z first gets N 80.00 , the next gets N 105.00, the next gets
x 12 the next gets N 130.00 and so on. How much does the tenth
= student get?
z 35 A N 280.00 B N 305.00 C N 330.00 D N 355.00
Equating ratios x : z i.e 12 : 35 Solution
Put x = 12 into (1) or z = 35 into (2) The sharing here followed a simple pattern
y = 5(12) i.e 20 80, 105, 130, …
3 Arithmetic Progression (A.P)
Thus x : y: z becomes 12: 20:35 (B) Whose first term a is 80, common difference d is 25
Tenth student’s share : T10 = 80 + (10 – 1 )25
2009/42 (Nov) = 80 + ( 9 ) 25
If x : y = a : 2 and y : z = 3 : b, find x : z = 80 + 225
A a : 2b B a : b C 2b : 3a D 3a : 2b = N 305.00 ( B )
12
Exercise 2.1 Percentages (%)
If A is twice as old as B and B is twice as old as C and The concept of percentage is as old as mathematics. It
the sum of their ages is 63 years, find ages A, B and C simply means expression out of 100. 25% means 25/100.
A lot of commercial activities are done in percentage:
Exercise 2.2 Commission, Discount, Depreciation, Profit and Loss,
If P : Q = 2 : 3 and Q : R = 4 : 7, find P : Q : R? Taxes etc

Exercise 2.3 2005/ 50


If x : y = 2 : 3 and y : z = 3: 4, find x : y : z The monthly salary of a man increased from N 2,700 to
N 3,200. Find the percentage increase.
2015/5 Neco Exercise 2.4 A 10% B 15% C 15.6% D 18.5%
A manufacturing company is owned by Dr. A and Solution
Engr. B and they agreed that their profit will be shared Percentage increase = 3200 − 2700  100
in the ratio 4 : 7. At the end of the year, Dr. A received 2700
N 7500.00 less than Engr. B. What is the net profit of 500
=  100 = 18.5% ( D )
the company for the year ended? 2700
A N 7,500 B N 10,000 C N 17,500 D N27,500 E N 52,500
2005/36
The salary of a man was increased in the ratio 40: 47.
2014/14a NABTEB ( Nov ) Exercise 2.5
Calculate the percentage increase in the salary
Three men provided capitals N 10,000.00 , N 25,000.00
A 85.1% B 58.5% C 17.5% D 14.9%
N 30,000.00 for a business on the understanding that the
Solution
share of the profit would be proportional to the capital
provided. If the profit is N 25,000.00, how much does Percentage increase = 47 − 40  100
40
each receive?
= 7  100 = 17.5% ( C )
40
2014/45 NABTEB ( Nov ) Exercise 2.6
2009/18 Neco (Dec)
Express ( 0.0425 : 2.5) as a fraction
When the salary of a man was increased by 20% he was
A. 17 B. 17 C. 17 D. 17 earning N 48,500.00. What would have been his salary if the
10,000 1000 250 100
increase had been by 26%
A N 60,140.00 B N 50,925.00 C N 50,900.00
2014/43 NABTEB ( Nov ) Exercise 2.7 D N 49,000.00 E N 48,750.00
If 3 children share N 10.50 among themselves in the Solution
ratio 6 : 7 : 8, how much is the largest share? Let the salary be x and in 100%
A N 3.00 B N 3.30 C N 4.00 D N 4.50 When increased by 20% it will be 120%
Thus 120% of x = 48,500
1976/10 (Nov) Exercise 2.8 120x = 48,500
Chijoke, Adeleke and Martins contributed N 8 000 , 100
N 6 000, N 4 000 respectively as capital to an 120x = 48,500  100
enterprise. Out of the total profits in a year, Martins 48,500  100
received N 500 for his services as a manager. Each of x =
120
them received 20% on the first N 3 000 of his capital
= 40416.67
and the remaining profit was shared in the ratio of the
remaining parts of their capitals. If at the end of the year 126
Next 126% of x =  40416.67
Martins received N 1 200 in all, find the total profit and 100
the amounts which Chijoke and Adeleke respectively (Increase is 100 + 26) = N 50, 925.00 ( B )
received 2005/ 7 (Nov)
If 3% of K = M, find 1% of K
2014/33 Neco ( Nov ) Exercise 2.9 M M 3M
In a certain wedding, the ratio of men to women is 2 : 7. A B C D 100M
3 300 100 3
If there are 124 men, how many women attended the
Solution
wedding?
A. 389 B. 399 C. 411 D. 434 E. 501 3% of K = M
3K
= M
2013/2b Exercise 2.9b 100
Salem, Sunday and Shaka shared a sum of N 31,100.00 3K = 100M
For every N 32.00 that Salem gets, Sunday gets 50kobo K = 100M
3
and for every N 34.00 Sunday gets, Shaka gets N 32.00.
Find Shaka’s share Thus 1% of K = 1 
100M
100 3
M
= (A)
3
13
2007/14b NABTEB
Discount A trader allows a retailer 20% trade discount and 5% for
2006/ 31 Neco cash payment. What will be the marked price of an article for
A buyer paid N 900.00 for an article over which he which a customer pays N 4,740.00?
enjoyed 10% cash discount. How much was the marked Solution
price? Let the marked price be x
A N 810.00 B N 900.00 C N 1,000.00 100 – (20 + 5)% of x = N 4,750
D N 1,100.00 E N 9,000.00 75x
= 47500
Solution 100
Let the marked price be x 75x = 47500  100
Allowing 10% discount on x gives N 900.00 x = 4750  100 = N 6,333.33
i.e (100 – 10)% of x = N 900 75
90x 2011/ 45
= 900
100 A shopkeeper allows a discount of 15% on the marked price
90x = 900  100 of a mobile phone. If a customer paid GHC 170.00 for
900  100 mobile phone, what was the marked price of the phone?
x = A GHC 144.50 B GHC 195 .40 C GHC 200.00
90 D GHC 255.00
= N 1,000.00 ( C ) Solution
Let the marked price be x
2009/ 8 Neco Discount allowed 15% i.e 85% of x = 170
After allowing a discount of 10% on an article, a seller 85
collected the sum of N2,700.00. Find the amount of the x = 170
discount paid 100
A N 27.00 B N 270.00 C N 290.00 85x = 170  100
D N 300.00 E N 2, 400.00 = 170  100 = GHC 200.00 (C)
Solution 85
Let the price of article before discount be x
Allowing 10% discount on x gives N 2,700 Loss
i.e (100 – 10)% of x = N 2700 2005/ 16 (Nov)
90x In a certain shipment, 2% of the crates were damaged. If the
= 2700 loss per damaged crate was D35 and the total loss due to
100
damages was D 700, how many crates were shipped?
90x = 2700  100
A. 2,000 B. 1,000 C. 200 D. 100
2700  100
x = Solution
90 Loss per damaged crate = D 35
= N 3,000 Total loss due to damage = D700
Amount of discount paid = N 3,000 – N 2700 700
= N 300.00 ( D ) Thus number of crates damaged = i.e 20 crates
35
Let the shipped crates be x
2005/ 11a (Nov)
But 2% of shipped crates (x) got damaged = 20 crates
A retailer allows a trade discount of 20% and a cash
2
discount of 5% on every article he sells. If a customer x = 20
pays N 1900 cash for an article, calculate the; 100
( i ) cash discount ( ii ) marked price x = 20  100
Solution 2
First, we find the original price of the item = 1,000 crates
Let it be x 2005 / 9 (Nov)
100 – (20 + 5) % of x = N 1900 A woman bought 600 oranges at N 1200. If 20% of the
75x orange got bad and the rest were sold at N25 for 12, find the loss
= 1900 A N50 B N200 C N 250 D N 288
100
75x = 1900  100 Solution
x = 1900  100 Cost per orange = 1200 i.e N 2.00
600
75
= N 2,533.33 20% of orange bad = 20  600 (oranges)
100
( i ) Cash discount = 5% of N 2,533.33 = 120 orange got bad
= 5  2,533.33 Oranges sold = 600 – 120 = 480
100
Amount of oranges sold = 480  25 = N 1,000
= N 126.67 12
( ii ) Marked price is N 2,533.33 Loss = N 1200 – N 1000
= N200 ( B )
14
Cost 2006/ 11 Neco (Dec)
2014/ 3b Neco A pair of shoes cost N 6,500.00 and 2 shirts cost N 4,500.00.
A businessman paid N25,000.00 for 12 machines. How much will 3 pairs of shoes and 3 shirts cost?
Find the: ( i ) cost of 10 machines of the same type A N 9,000.00 B N 13,500.00 C N 22,500.00
( ii ) Number of such machines N 175,000.00 can buy. D N 26,250.00 E N 33,000.00
Solution Solution
First we find the cost per machine. A pair of shoe costs N 6,500.00
If 12 machines cost N 25,000.00 2 shirts cost N 4,500.00 then 1 shirt cost 4500 = N 2,250.00
then one machine will cost 25000 = N 2083.33 2
12 Thus, cost of 3 pairs of shoe is 3  6500 = N19500
( i ) Cost of 10 machines of the same type Cost of 3 shirts is 3  2250 = N 6,750
= 10  N 2083.33 Combined cost N 26,250.00 ( D )
= N 20,833.30 2005/5
( ii ) Number of such machines N 175, 000.00 can buy Three bags and five cups cost N 800 altogether. If each of
= 175,000.00 the bags cost N150, how much does a cup cost?
2083.33 A N 50 B N 70 C N 100 D N 160
= 84 machines Solution
Since each (one) bag costs N150
2012/ 3a Neco 3 bags cost is 3  150 = N 450
The cost of telephone service per month is calculated on Cost of cups is N 800 – N 450 = N 350
three part rates, 350
N 14.00 per unit for the first 25 units, Cost of each cup is = N 70 ( B )
5
N 18.00 per unit for the next 50 units and 2006/ 37 (Nov)
N 22.00 per unit for the remainder. Find the difference in cost per week between employing
If the total amount paid is N 1,580.00, find the 16 men at a daily wage of D23.50 each and employing
remaining units 26 boys at a daily wage of D12 each.
Solution A D124 B D 392 C D 448 D D532
N 14.00 per unit for the 1st 25 units Solution
N 18.00 per unit for the next 50 units Daily cost of employing men is 16  D23.50 = 376
N 22.00 per unit for the remainder Weekly cost of employing men is 7  D376.00 = D2632.00
14  25 = N 350
Daily cost of employing boys is 26  D12 = D312.00
18  50 = N 900
Weekly cost of employing boys is 312  7 = D2184.00
= N 1,250
Difference in cost per week = 2632 – 2184
Remaining units amount is N 1,580 – N 1,250 = D448 ( D )
= N 330
Income
Thus Remaining units amount is 330 = 15 units
22 2010/ 4 Neco
2010/ 1a Neco Mr. Kolo is employed as a poultry farm manager on an
Find the difference of the cost of employing 15 females initial salary of N 300,000.00 per annum. His salary
at N 4,860.00 each and 10 males at N 7,050.00 each increases annually by N 4,000.00. Find the total money
Solution earned by him after working for five years.
First, we find both costs separately Solution
Female cost Male cost 1st year: N 300,000.00
15  4860 10  7050 2nd year: N 300,000.00 + N 4,000.00 = N 304,000
= N 72,900 = N 70,500 3rd year: N 304,000 + N 4,000 = N 308,000
4th year: N 308,000 + N 4,000 = N 312,000
Difference in cost = 72,900 – 70,500
5th year: N 312,000 + N 4,000 = N 316,000
= N 2,400
At the end of 5 years, total money earned
= 300,000 + 304,000 + 308,000 + 312,000 + 316,000 = N 1,540,000
1976/1b ( Nov) 2010/9 Neco
If 8kg of coffee costing 20.00 a kg is mixed with 12kg An employee earns N 450,000.00 per annum out of which he
of another kind costing 22.00 a kg, what is the cost of spends 12% on house rent. How much is left for other expenses?
the mixture per kg? A N 45,000.00 B N54,000.00 C N 396,000.00
Solution D N 449,912.00 E N 449,988.00
8kg of coffee at 20.00 per kg cost 8 × 20 = 160 Solution
12kg of 2nd coffee at 22.00 per kg cost 12 × 22 = 264 What is left for other expenses = annual salary – House rent
Total weight is 8kg + 12kg = 20kg First, we find the house rent i.e 12% of 450, 000
Cost of mixture per Kg = 160 + 264 = 12  450,000
20 100
= 424 i.e  21.20 = N 54,000
20 Amount left for expenses = N 450,000 – N 54,000= N 396,000.00

15
Joint cases 2010/6 Neco
2005/ 4(Nov) A married man with four children earns N 240,000.00 per
A sales clerk is on a monthly basic salary of N 7,500.00. annum. He claims a personal allowance of N 30,000.00 and
He is given a commission of 21/2 % of his monthly sales allowance of N15,000.0 for his wife and N 10,000.00 for each
and 3% of his basic salary as utility allowance. He pays child. What is his taxable income if the allowances are not
2% of his total income as income tax and 5 kobo in the taxable?
naira as cooperative society fund. A N210,000.00 B N 176,000.00 C N155,000.00
If his sales for a month is N 25,000.00, calculate his D N 85,000.00 E N 64,000.00
take - home pay for that month. Solution
Solution Allowance for 4 children = 4  10,000
Monthly sales is N 25,000.00, find take home pay. = N 40,000
Basic salary : N7, 500 Salary allowances
Commission: 21/2 of sales (N 25,000.00) Taxable income = 240,000 – (30,000 + 15,000 + 40,000)
5 1 = 240, 000 – 85, 000
i.e   25000 = 625
2 100 = N155,000.00 ( C )
Utility allowance: 3% of basic salary 2007/14a NABTEB
3 A married man with 5 children is on an annual salary of
i.e  7, 500 = 225 N 75, 000.00. The man is given tax relief as follows:
100
Total income for that month: N 7,500 + N 225 + N 625 = N 8350 Personal allowance of N 9,000.00
Next is Deductions Children allowance of N 1,500.00 per child for a
maximum of 4 children.
Tax : 2% of total income = 2  8350 i.e N167 Dependent relative allowance of 1/10th of his salary
100 Life insurance allowance of N 5,000.00
Cooperative society fund : 5kobo in the naira of his income If tax is paid at 10k in N on the 1st N 20,000.00 and
= 8350 i.e N1,670 15k in N on the remaining, calculate the amount of tax he pays:
5 Solution
Take - home pay = total income – deductions Annual salary is N 75,000.00
= 8350 – 167 – 1670 Tax Relief are: personal allowance N 9,000.00
= N 6,513 Children allowance 4  1500 is N 6,000
2012/ 6a Neco Dependent relative allowance 1/10  75000 is N 7,500
A man earns a total of N 24,000.00 per month. Some of Life insurance allowance is N 5,000
his monthly expenses are listed below: Total tax relief N 27, 500
House keeping N 6,400.00
Petrol and electricity N 1,700.50 Amount taxable is N 75,000 – N 27,500 = 47,500
Water rate N1,000.90 First tax is 10  20,000 = 200000 kobo i.e N 2,000.00
Insurance N 900.30 2nd tax is 15  27,500 = 412500 kobo i.e N 4,125.00
Income tax N 4,100.10 Total tax N 6,125.00
( i ) How much is paid on water rates per year? 1979/8
( ii ) Express the income tax paid as percentage of his In 1965, the amount of taxes collected in a country amounted to
Le20 000 per 100 persons. In 1975, the population increased by
total income.
8% and the revenue from taxation by 25%.
(iii) If petrol and electricity expenses are in the ratio of ( a ) Calculate, to 3 significant figures, the percentage
16 : 5, by how much is the amount expended on increase in taxation per person
petrol more than that of electricity. ( b ) If in 1976, the government decided to reduce taxation per
Solution person by 2%, calculate, correct to the nearest 10 Leones, the
( i ) Water rate per year = N 1,000.90  12 = N 12,010.80 revenue from taxation per 100 persons
i.e 12months Solution
4100.10 a. In 1965, 100 persons paid taxes of Le 20 000
( ii ) Income tax % =  100 = 17.08% 1
24000 1 person paid taxes of 100 × Le20 000 = Le200
Petrol : electricity 108
In 1975, 100 persons increased by 8% to ×100 persons
100
16 : 5
Total ratios = 16 + 5 i.e 21 = 108 persons
125
Amount difference between Petrol and electricity Le20,000 increased by 25% to ( 100 × 20,000) = Le25,000
= 11  1700.50 = N 890.74 108 persons paid taxes of Le 25,000
21 1
Alternatively 1 person paid taxes of 108 × Le25,000 = Le231.48
Total ratios = 16 + 5 i.e 21 Thus, increase in taxation per person is Le231.48 – Le200
Petrol amount electricity amount = Le31.48
16 5 Le 31.48
 1700.50  1700.50 The percentage increase in tax = Le 200 × 100% = 15.74%
21 21
= N1295.62 = N 404.88  15.7% to 3 s.f
Difference in amount = 1295.62 – 404.88 = N 890.74
16
b. In 1975, the taxation per person is Le 231.48 Hire purchase
In 1976, the taxation per person is reduced by 2% to 2004/ 4b (Nov)
98 An electrical shop sells a video recorder for D3,200 cash
100 × 231.48 = Le 226.85
price. Hire purchase deal is offered to Mr. Bah at an extra
The taxation per 100 persons = Le226.85 × 100 = Le22685
20% cost. He pays a deposit of D1,200 and Dx in 6 equal
 Le22690 to nearest 10 Leones
monthly installments. Calculate the value of x
Commission Solution
2009/3 Neco (Dec) Cash sales price is D3,200
A sales representative is entitled to a commission of 20
Hire purchase is 20% of 3,200 =  3,200
6.5% on every radio set sold. If his commission 100
amounts to N 1,250.00k per radio set sold, calculate the = D640
price of a radio set (to the nearest N) Since Mr. Bah bought it at Hire purchase
A N 19210.00 B N 19220.00 C N19229.00 Cost = 3,200 + 640
D N 19230.00 E N19231.00 = D 3,840
Solution He pays a deposit of D1,200 and left to pay Dx
Let the radio price be y Thus Dx = D3,840 – D1,200
Commission of 6.5 % on y gave N 1,250.00k = D 2, 640
i.e 6.5 % of y = 1250
6.5 = 1250 Depreciation
y
100 2008/ 7c
6.5y = 1250  100 A car costs N300,000.00. It depreciates by 25% in the first
y = 1250  100 year and 20% in the second year. Find its value after 2 years
6.5 Solution
= N 19,230.77k
 N 19,231.00 to the nearest N 1st year : 25% of N300,000.00 = 25  300,000 i.e N75,000
100
2005/3 Neco
Find the marked price of an article, if the N 75.50 Car value after 1st year = N 300,000.00 – N 75,000.00
commission allowed is 4.5% = N 225,000
A N 1,602.28 B N 1,650.00 C N1,653.28 20
D N 1,677.78 E N 6,899.00 2nd year : 20% N225,000 =  225,000
100
Solution = N 45,000
Let the marked price be y
Allowed 4.5% commission on y gives N 75.50 Car value after 2nd year = N 225 – N 45, 000
i.e 4.5 % of y = N 75.50 = N 180,000.00
4.5 y = 75.50
100 2006/ 25
4.5y = 75.50  100 A machine valued at N 20,000 depreciates by 10% every
y = 75.50  100 year. What will be the value of the machine at the end of two
4.5 years?
= N 1,677.78 ( D ) A N 16,200 B N 14,200 C N12,000 D N 8,000
2010/ 5 Neco Solution
A sales girl gets a commission of 8% of the value of the 1 year : 10% of N 20,000 = 10  20,000 i.e N 2,000
st

things she sells. Find her commission for selling 100


3 keyboards at N 25,000.00 each and 5 calculators at Machine value after 1st year = N 20,000 – N 2,000
N 600.00 each = N 18,000
A N 2,048.00 B N 3,200.00 C N5,740.80 10
2nd year: 10% of N 18,000 =  18,000 i.e N 1,800
D N 6,240.00 E N 9,750.00 100
Solution Machine value after 2nd year = N 18,000 – 1,800
Commission on keyboard 8% of N 25,000.00 = N 16, 200 ( A )
8
=  25,000 i.e N 2,000
100 Tax
Thus 3 keyboards commission = 3  2,000 i.e N 6,000 2005/ 40
What is the purchase tax, at 86 kobo in the naira on a jewelry
Commission on calculators 8% of N 600.00 valued at N 2,160.00?
A N 1,857.60 B N 2,759.50
= 8  600 i.e N 48.00
100 C N 3,759.50 D N 3,856.70
Thus 5 calculators commission = 5  48 i.e N 240 Solution
Total commission = N 6000 + N 240 Purchase tax is 2160  86 = 185760 kobo
= N 6240.00 ( D ) = N 1,857.60 ( A )

17
Exchange 2014/15b NABTEB ( Nov ) Exercise 2.15
2011/ 1 A man is allowed the following tax relief:
If N112.00 exchanges for D14.95, calculate the value of Personal allowance 121/2% of basic salary
D1.00 in naira Wife allowance 10 % of basic salary
A 0.13 B 7.49 C 8.00 D 13.00 Children allowance N 5,000.00 per child up to
Solution maximum of 4 children
If N112.00 exchanges for D14.95 Aged parent N10,000.00 for only one
112.00 Dependent relative 10% up to maximum of 2
Then D1.00 will be = 7.49 naira ( B )
14.95 relatives
2015/3 A married man with 5 children has an aged father and two
A trader bought an engine for $15,000.00 outside relatives. If the man’s basic salary is N 850,000.00 per
Nigeria. If the exchange rate is $0.075 to N1, how much annum and tax is calculated at the rate of 15% find the man’s
did the engine cost in naira? ( i ) taxable income per annum
A N 250,000.00 B N 200,000.00 ( ii ) monthly tax
C N150,000.00 D N 100,000.00 ( iii ) monthly pay
Solution
If the exchange rate is $0.075 to N1, 2014/31 ( Nov ) Exercise 2.16
15000 After allowing a discount of 15% on an article, a seller
Then $15,000 will be = N 200,000 ( B )
0.075 collected GHC 36,000.00 in cash. How much was the
discount?
1977/3 Exercise 2.10 A GHC 6,952.94 B GHC 6,502.94
A man made two investments, one at 7% and the other C GHC 6,352.94 D GHC 6,203.94
at 8% interest. The yield on the two investments was
Le222. If the total amount invested was Le3 000, how 2008/3 Neco Exercise 2.17
much was invested at each rate? If 15% of a sum of money is given as N 75,000.00, what is
1977/8 Exercise 2.11 the total sum?
The cost of a man’s holiday in 1974 was made up of A N 750,000.00 B N 500, 000.00 C N5,000,000.00
65% hotel bills, 20% traveling expenses and 15% D N 75,000,000.00 E N 750,000,000.00
incidentals. In 1975 the hotel bills had gone up by 40%
and the traveling expenses by 60%. He reduced his 2013/ 7 Neco Exercise 2.18
incidentals by 10% and thereby made a saving of 30 The marked price of an item is N 500.00. If a discount of
on the incidentals. By how much did the cost of his 10% is given, at what price is the item sold?
holiday in 1975 exceed that of 1974? A N 200.00 B N 250.00 C N 300.00
D N 450.00 E N 500.00
2014/15b NABTEB ( Nov ) Exercise 2.12
Given that the market exchange rate is N 200.00 to
1 British pounds ( £ ) , convert N 726,840.00 to £
2014/2 ( Nov ) Exercise 2.13
A man has a wife and 6 children and his total income in
a year was GHC 850. He was given the following tax
free allowances:
Personal GHC 120;
Wife GHC 30;
Children GHC 25 per child, for a maximum of 4 children
Medical GHC 40
The rest was taxed as follows:
First GHC 200 at 10%
Next GHC 200 at 15%
Next GHC 200 at 20%
Remainder at 25%
Calculate his : ( a ) taxable income; ( b ) monthly tax

2015/8b Neco Exercise 2.14


In 2001, the salary of Mr Edward was N 25,700.00 and
this was increased by 15% in 2002. If he paid 121/2% of
his salary as income tax, calculate, correct to 3
significant figures, the amount of tax paid in 2002
18
Profit and loss 2005/19 Neco
Any transaction is profitable if selling price is A man bought a certain number of articles, of which 10 cost
greater than cost price otherwise it is a loss N 15.00 and sold them at N 19.50 per dozen. Find the
Percentage profit = SP – CP x 100 percentage gain or loss.
CP A 7.7% B 8.3% C 23.1% D 30% E 92.9%
Where SP = Selling price Solution
CP = Cost price. If 10 articles cost N 15.00
% loss = CP – SP x 100 Then 1 article will cost 15 = N 1.50 CP
CP 10
Selling point
2007/ 10 Neco If 12 articles was sold for N 19.50
By selling a book at a loss of 19%, a man lost Then 1 article will be sold at 19.50 = N 1.625 SP
N11.40. Find the actual price paid for the book 12
A N 13.57 B N 20.63 C N 21.67 Since SP N 1.625 is greater than CP N 1.50, i.e % profit
D N 24.00 E N 60.00 SP − CP
% gain =  100
Solution CP
CP − SP 1.625 − 1.50
% loss =  100 =  100
CP 1.50
CP = ? % loss is 19% But CP – SP = N 11.40 = 8.3% (B)
Substituting
19 = 11.40  100 2006/ 11 Neco
CP Mohammed sold an article which originally cost
19CP = 1140 N 2,500.0 at the rate of N 2,050.00. Find the percentage loss.
CP = 1140 A 22% B 18% C 16% D 10% E 8%
19 Solution
= N 60. 00 ( E ) CP − SP
% loss =  100
2006/ 9 Neco CP
Ngozi sold an article for N 1,755.00 and made a profit % loss = ? ; CP = N 2,500.0; SP = N 2,050.00
of 35%. Find the cost price. Substituting
A N 2,370.00 B N 2,369.25 C N 1,300.00  2500 − 2050 
D N 614.25 E N 614.00 % loss =    100
 2500 
Solution
CP − SP = 450 i.e 18% (B)
% profit =  100 25
CP
2005/ 49
% profit = 35%; SP = N 1,755; CP = ?
Ladi sold a car for N 84,000 at a loss of 4%. How much did
Substituting Ladi buy the car?
1755 − CP A N 80,500 B N 80,640 C N 87,360 D N 87,500
35 =  100
CP Solution
35CP = 175500 – 100CP % loss = CP − SP  100
35CP + 100CP = 175500 CP
135CP = 175500 % loss = 4%; CP = ?; SP = N 84,000
CP = 175500 Substituting
135 4 = CP − 84,000  100
= N 1,300.00 ( C ) CP
4CP = (CP – 84,000)100
2008/31 NABTEB (Dec) 4CP = 100CP – 8,400,000
Joseph bought a car for N 200,000.00 and later sold it 8400 000 = 100CP – 4CP
for N 170, 000.00, what is his percentage loss? 8400000 = 96CP
A 117.6% B 115.6% C15.6% D 15 % 8400000
Solution = CP
96
% loss = CP − SP  100 = N 87,500 ( D )
CP
CP = N 200,000.00; SP = N 170,000.00; loss = ?
Substituting 2008/7
200000 − 170000 A trader bought 100 tubers of yam at 5 for N 350.00. She
% loss =  100 sold them in a set of 4 for N 290.00. Find her gain percent.
200000 A 3.6% B 3.5% C 3.4% D 2.5%
= 30000  100 Solution
200000
SP − CP
= 15 % ( D ) % gain =  100
CP
19
Cost price (CP) : 100  350 = N 7,000 2009/ 23 Neco (Dec)
5 By selling an article for N 600.00, a man made a profit of
Selling price (SP) : 100  290 = N 7,250 20%. What should be the selling price to make a profit of 30% ?
4 A N 550.00 B N 580.00 C N 630.00
Substituting D N 650.00 E N 780.00
% gain =  7250 − 7000   100 Solution
 7000  In the first case, lets us find CP
250 SP − CP
= % profit =  100
70 CP
= 3.57%  3.6% ( A ) % profit = 20%, SP = N 600.00, CP =?
2011/ 5b Neco Substituting
20 = 600 − CP  100
A dealer sells an article for N 560.00 and makes a profit
of 12%. Find the percentage of the profit he will realize, if CP
he sells the article for N 600.00 20CP = (600 – CP) 100
Solution 20CP = 60000 – 100CP
In the first case, lets us find CP 20P + 100CP = 60000
SP − CP 120CP = 60000
% profit =  100
CP 60000
CP = i.e N 500
% profit = 12%; SP = N 560.00; CP = ? 120
Substituting
560 − CP 2nd case: % profit = 30%, SP = ? , CP = N 500
12 =  100
CP Thus 30 = SP − 500  100
500
12CP = (560 – CP)100
12CP = 56000 – 100CP 30  5 = SP – 500
12CP + 100CP = 56000 150 = SP – 500,
112CP = 56000 SP = 500 + 150 i.e N 650 ( D )
56000
CP = i.e N 500.00 2004/7a Neco (Dec)
112
Victor sold a second – hand television to Funmi at a loss of
2nd case:
20%. Funmi then sold it to Usman for N5,200.00 making a
SP = N 600.00; CP = N 500.00; % profit = ?
loss of 15%. How much did Victor buy it?
% profit = 600 − 500  100 Solution
500 Funmi to Usman transaction at Loss %
100
=  100 = 20% % loss = CP − SP  100
500 CP
2011/3 Neco SP = N 5, 200.00 CP =?, % loss = 15%
An item was sold for N 650.00 with a profit of 30%. Substituting
If the profit had been 20%, what was the selling price? 15 = CP − 5200  100
A N 600.00 B N 550.00 C N 500.00 CP
D N 450.00 E N 400.00 15CP = (CP – 5200)100
Solution 15CP = 100CP – 520000
In the first case, lets us find CP 520000 = 100CP – 15CP
% profit = CP − SP  100 520000 = 85CP
CP CP = 520000 i.e N 6,117.65
% profit = 30%, SP = N 650.00, CP = ? 85
Substituting Victor to Funmi transaction at Loss %
30 = 650 − CP  100 % loss = CP − SP  100
CP CP
30CP = (650 – CP) 100
30CP = 65000 – 100CP % loss = 20%, CP =?, SP = N 6,117.65(Funmi to Usman cost price)
30P + 100CP = 65000 Substituting
130CP = 65000 CP − 6117.65
20 =  100
65000 CP
CP = i.e N 500.00
130 20CP = (CP – 6117.65) 100
2nd case: SP = ? ; % profit = 20%; CP = N 500.00 20CP = 100CP – 611765
SP − 500 611765 = 80CP
20 =  100
500 CP = 611765
80
10000 = (SP – 500) 100
100 = SP – 500, = N 7,647.06
SP = 100 + 500 i.e N 600.00 ( A )
20
2009/ 6b 4 = 6300 − SP  100
For musical shows, x children were present. There were 6300
60 more adults than children. An adult paid D5 and a 4  6300 = (6300 – SP)100
child D2. If a total of D1280 was collected, calculate 100 will cancel out
the: 4  63 = 6300 – SP
(i) value of x; 252 = 6300 – SP
(ii) ratio of the number of children to the number adults; SP = 6300 – 252
(iii) average amount paid per person = N 6,048.00
(iv) percentage profit it the organizers spent D720 Remainder of the items (98 – 42 – 35 = 21)
on the show. Cost price (CP) : 21180 = N 3,780.00
Solution SP = ?, % profit = 15%
( i ) Children Adult SP − 3780
x x + 60
15 =  100
3780
2x + 5(x + 60) = 1280 15  3780 = (SP – 3780) 100
2x + 5x + 300 = 1280 56700 = 100SP – 378000
7x = 1280 – 300 56700 + 378000 = 100SP
7x = 980 434700 = 100SP
x = 980 i.e. 140 434700
7 SP =
( ii ) x : x + 60 (where x is 140) 100
140 : 200 = N 4,347.00
i.e. 7 : 10 ( i ) Overall selling price:
9,072 + 6,048 + 4,347 = N 19,467.00
(iii) Average amount paid = Total amount collected ( ii ) Since overall CP = N 17,640.00 and SP = N 19,467
number of people i.e SP is bigger, we find % gain
= 1280 i.e D3.76
340
% gain = 19467 − 17640  100
17640
(iv) CP =D720; SP = D 1280; % profit = ?
= 1827  100
% profit = SP − CP  100 17640
CP
= 10.357%  10.36 % to 2d.p
1280 − 720
= 2008/12a NABTEB (Dec)
720 A trader sold an article for N 4,500.00 and made a profit of 15%.
560 How much must he sell it to make a loss of 71/2%?
=  100 = 77.78%
720 Solution
2007/ 13a NABTEB First, we find CP from the % profit
A trader bought 98 units of an article at N 180.00 each. % profit = SP − CP  100
He sold 42 of them at a profit of 20%, 35 at a loss of 4% CP
and the remainder at a profit of 15%. Find the overall % profit = 15%, SP = N 4,500.00, CP =?
( i ) selling profit to the nearest kobo and Substituting
( ii ) percentage gain or loss to 2 decimal places 15 = 4500 − CP  100
Solution CP
Total cost price : 98  180 = N 17,640.00 15CP = (4500 – CP)100
42 of the items 15CP = 450000 – 100CP
Cost price (CP): 42  180 = N 7,560.00 15CP + 100CP = 450000
SP = ?, % profit = 20% 115CP = 450000
SP − CP CP = 450000 i.e N 3,913.04
Thus % profit =  100 115
CP Next is the proposal for 71/2% loss
20 = SP − 7560  100 CP − SP
7560 % loss =  100
CP
20  7560 = (SP – 7560)100 (3913.04 − SP)
151200 = 100SP – 756000 71/2 =  100
3913.04
151200 + 756000 = 100SP
907200 = SP 15 391304 − 100SP
=
100 2 3913.04
SP = N 9,072.00 15  3913.04
= 391304 – 100SP
35 of the items 2
Cost price (CP): 35  180 = N 6,300.00 29347.8 = 391304 – 100SP
SP = ?; % Loss = 4% 100SP = 361956.2
% Loss = CP − SP  100 SP = 361956.2 = N 3,619.56
CP 100

21
2008/13 NABTEB (Dec) 2001/ 8b (Nov) special case
In the beginning of year 2007 a poultry attendant A man bought 250 electric lanterns in the United States of
bought 1300 day old birds at N150.00each. Later in that America (USA) for $5,000. He sold them in Lagos at N1,920
year she realized 7038 eggs which she sold at N450.00 each. At the time of his journey back to USA, the exchange
per crate. She spent N18,600.0 on feeding the birds and rate was N 96 to the dollar.
N 8,500.00 on drugs for the year. By the end of the (i) If by selling the lanterns at N 1,920 each, he made a profit
year, she sold the remaining 112 chickens at the rate of of 20%. Calculate the exchange rate at the time he bought the lanterns
N800.00 each. Find the percentage gain or loss (ii) How much in dollar will the sales amount to at the time
Solution of his journey back to USA?
Let us put the data in the picture of expenditure (cost) and sales Solution
Cost of day old birds: 1300  N150 = N195,000.00 (i) Bought 250 at $5,000
Sale of eggs: 7038 (Note 36 eggs in a crate) = 195.5 Sold N 1,920  250 = N 480,000
36 Return exchange rate N 96
Thus sales of eggs = 195.5  N 450 = N 87,975 Amount sales in Dollars $ 480,000 = $5,000
Cost of drugs: N 8,500 96
Sales of remaining chickens: 112  N 800 = N 89,600 % Profit = SP − CP  100
Total cost: 195,000 + 18,600 + 8,500 = N 222,100 CP
Total sales: 87,975 + 89,600 = N 177,575 20 = 480,000 − CP  100
Cost is greater than sales; hence % loss CP
20CP = (480000 – CP)100
% loss = CP − SP  100
CP 20CP + 100CP = 48000000
120CP = 48,000,000
= 222100 − 177575  100
222100 48,000,000
CP = = 400,000
= 44525 120
2221 Let exchange rate be x
= 20.05% Thus $ 5,000 = 400,000
2006/ 2a x
A dealer sold a car to a man and made a profit of 15%. 400,000
x =
The man then sold it to a woman for N120,175.00 at a 5000
loss of 5%. How much did the dealer buy the car? = N 80 per Dollar
Solution (ii) Sales at return to USA
Let us start with the man to the woman transactions, 250  N 1,920 = N 480,000
where we have data to work with. 480,000
In Dollars = = $ 5,000
% loss = CP − SP  100 96
CP 2015/9b
% loss = 5%, SP = N 120, 175.00, CP =? A trader bought 30 baskets of pawpaw and 100 baskets of
Substituting mangoes for N2,450.00. She sold the pawpaw at a profit of
5 = CP − 120175  100 40% and the mangoes at a profit of 30%. If her profit on the
CP entire transaction was N855.00, find the:
5CP = (CP – 120175)100 ( i ) cost price of a basket of pawpaw
5CP = 100 – 12017500 ( ii ) selling price of the 100 baskets of mangoes
12017500 = 100CP – 5CP Solution
12017500 = 95CP Let p be pawpaw and m be mangoes
CP = 12017500 i.e N126,500.00 From cost: 30p + 100m = 2450 --------( 1 )
95
From profit: 40p + 30m = 855 ----------( 2 )
The dealer to the man transaction (1) × 4 and (2) × 3 and subtract
% profit = SP − CP  100 120p + 400m = 9800
CP – (120p + 90m = 2565)
Here SP = N126,500 (i.e CP at man to woman transaction) 310m = 7235
% profit = 15%, CP = ?
Substituting m = 7235 i.e N23.34
310
15 = 126500 − CP  100 Substitute m value into (2)
CP 40p + 30(23.34) = 855
15CP = (126500 – CP)100 40p + 700.2 = 855
15CP = 12650000 – 100CP 40p = 855 – 700.2
15CP + 100CP = 12650000
115CP = 12650000 p = 154.8 i.e N3.87
40
12650000 ( i ) A basket of pawpaw cost N3.87
CP = i.e N 110, 000.00
115 ( ii ) % profit = SP − CP  100
CP
22
% profit is 30%; CP is 100(23.34 ) i.e N 2334; SP = ? SIMPLE INTEREST & COMPOUND INTEREST.
30 = SP − 2334  100 If a certain amount of money is to be paid (interest) per
2334 annum on a borrowed or deposited money (principal) on a
30 × 2334 = (SP – 2334)100 continuous basis as long as the transaction holds, then the
70020 = 100SP – 233400 interest is:
233400 + 70020 = 100SP Simple interest; if the interest to be paid in one year does
303420 not affect the principal for the next year calculation of
= SP
100 interest, i.e . the principal is fixed all through the transaction
N3,034.20 = SP Mathematically,
SI = PRT
2010/7a Exercise 2.19
100
Madam Kwakyewaa imported a quantity of frozen fish Where P is Principal; R is ratio and T is time
costing G H C 400.00. The goods attracted an import
Amount = P + interest
duty of 15% of its cost. She also paid a sales tax of 10
% of the total cost of the goods including the import Compound interest; if the interest to be paid in the 1st year
duty and then sold the goods for G H C 660.00. plus the principal forms the new principal for the next year.
Calculate her percentage profit. Thus the principal continues to increase as well as the
2010/3b Exercise 2.20 interest. Mathematically.
Bala sold an article for N6,900.00 and made a profit of CI = A – P
15%. If he sold it for N6,600.00 he would have made a: Where A is amount and P is principal
A profit of 13% B loss of 12%  R 
n

C profit of 10% D loss of 5% But A = P 1 +  −P


 100 
2012/4b Exercise 2.21
Where n is number of years and R is ratio
A sales man bought some plates at N 50.00 each. If he
sold all of them for N 600.00 and made a profit of 20%
Problems on simple interest
on the transaction, how many plates did he buy?
2009/5
2013/6b Exercise 2.22
Find the value to which N 3000.00 will amount in 5 years at
A man sold 100 articles at 25 for $66.00 and made a
6% per annum simple interest
gain of 32%. Calculate his gain or loss percent if he A N 3,900.00 B N 3,750.00 C N3,600.00 D N 3,300.00
sold them at 20 for $50.00 Solution
2009/10 Neco Exercise 2.23 Amount = Principal + interest
By selling 30 bananas for N 45.00, a trader makes a PTR
SI =
profit of 5%. What would be the percentage loss if he 100
sells the same 30 bananas for N 42.00? SI = 3000  5  6
A 2% B 3% C 5% D 6.7% E 7.14% 100
2014/6b Exercise 2.24 = N 900
A television set was marked for sale at G H C 760.00 in Thus amount = N 3000.00 + N 900
order to make a profit of 20%, the television set was = N 3,900.00 (A)
actually sold at a discount of 5%. Calculate, correct to 2 2013/ 6 Neco
significant figures, the actual percentage profit. What will be the simple interest on N 2500.00 for 2 years at
the rate of 5% per annum?
2006/4(Nov) Exercise 2.25 A N 250.00 B N 500.00 C N 625.00
A trader buys a bag of rice for N 3, 200.00 and sells it at D N 1250.00 E N 6250.00
a gain of 21/2 percent. What is the selling price? Solution
A N 3,120.00 B N 3,280.00 C N 3,360.00 D N 3,840.00
SI = PTR
2014/ 8 Neco Exercise 2.26 100
A trader sold an article for N 2,400.00 at a loss of 25%. Here P is N 2500; T is 2; R is 5
Calculate his cost price. Substituting
A N 3,000.00 B N 3,200.00 C N 3,500.00 SI = 2500  2  5
D N3,600.00 E N 4,000.00 100

2005/12a (old) Exercise 2.27 = N 250.00 ( A )


The marked price of radio set is D5,200.
(i) If this includes a 30% profit; calculate the cost price; 2007/ 36
Amadu bought this radio set and was offered a 5% discount; At what rate percent per annum will N 520.00 yield a simple
(ii) How much did he pay? interest of N 39.00 in three years?
(iii) What is the actual profit made by the seller? A 4% B 31/2% C 3% D 21/2%
Solution
2008/12 Neco Exercise 2.28 PTR
A pair of shoes was sold for N 2,250.00 at a loss of SI =
100
10%. What was the cost price? Here SI is N 39, P is N 520, T is 3, R is?
A N 750.00 B N 2,500.00 C N2,538.00 DN3,288.00 E N4,038.00
23
Substituting Solution
39 = 520  3  R SI = PTR
100 100
39  100 = 520  3  R Here SI is N 860,000; P is N 6,000,000; T is?; R is10
39  100 = R Substituting
520  3 6000000  T 10
860 000 =
21/2% = R ( D ) 100
2007/1 Neco 860 000  100 = 6 000 000  T  10
Calculate the rate percent per annum at which 860 000  100
= T
N5,000.00 doubles itself in 20years 6000 000  10
A 25% B 12% C 8% D 5% E 4% 1.43 = T i.e 1.4years ( A )
Solution
SI = PTR 2010/5 NABTEB (Dec)
100 Find the amount if simple interest is paid on N34320 in
Here P is N 5,000; T is 20; R is ? 5 years at 61/4 % per annum.
SI is N 5,000( i.e Double of N 5,000 minus N 5,000) A N 45045 B N 43043 C N 23595 D N 10725
Substituting Solution
5000  20  R Amount = Principal + (simple) interest
5000 =
100 SI = PTR
5000  100 = 5000  20  R 100
5000  100 Here SI is? P is N 34,320; T is 5; R is 61/4 i.e 25/4
= R Substituting
5000  20
SI = 34320  5  25
5% = R ( D ) 100  4
= N 10,725
2007/ 13b NABTEB
Thus, Amount = N 34320 + N 10,725
A simple interest on a sum of money invested at 4% for
= N 45,045 ( A )
4years was N 4,040.00. How much was invested?
Solution
2007/ 1b
SI = PTR A man invested N 20,000in bank A and N 25,000 in bank B
100
Here SI is N 4,040; P is?; T is 4; R is 4 at the beginning of a year. Bank A pays simple interest at a
Substituting rate of y% per annum and B pays 1.5y% per annum. If his
P  4 4 total interest at the end of the year from the two banks was
4,040 = N 4,600, find the value of y
100 Solution
4,040  100 = P  4  4 Bank A Bank B
4,040  100 = P P is N20,000 P is N25,000
4 4 R is y% R is 1.5y
N 25,250 = P Time T is 1year T is 1 year
(Reason beginning of year to the end implies one year)
2008/13 NABTEB(Dec) N 4,600 = SIA + SIB SI is given as PTR
How long will it take N2,600.00 to earn N 520.00 at 5% 100
per annum simple interest? 20000  1  y 25000  1  1.5 y
A 14 years B 10year C 8years D 4 years 4600 = +
100 100
Solution 4600 = 200y + 250  1.5y
SI = PTR 4600 = 200y + 375y
100
4600 = 575y
Here SI is N 520; P is N2,600; T is?; R is 5
4600
Substituting y =
575
520 = 2600  T  5
100 y = 8%
520  100 = 2600  T  5
520  100 = T
2008/ 5 Counter example
2600  5
p naira invested for 4years at r % simple interest per annum
4years = T (D) yields 0.36p naira interest. Find the value of r
2010/29 NABTEB (Dec) A 1 1/ 9 B 1 4/ 9 C 9 D 11
After how many years will 6 million naira yield an Solution
interest of N 860,000 at the rate of 10% per annum? PTR
SI =
A 1.4 years B 4 years, 8 months C 48 years D 96 years 100
24
p4r Solution
0.36p = First we find the simple interest
100
Our target is to make r subject of formula The deposit was kept in the bank for 3 months
0.36p  100 = 4pr 16 July to 9 October and not 4 months.
36 p SI = PTR
= r 100
4p 17
9= r(C) P is N 14, 600, T is 3 years, R is 81/2 i.e
12 2
Substituting
2005/45
A man took a loan of $P at the rate of 4% per annum SI = 14,600  3 17
100  12  2
simple interest. If at the end of 5years he paid back
$720, find the value of P? = N 310.25
A $600 B $556 C $500 D $456 Accrued Amount = principal + interest
Solution = N 14,600.00 + N 310.25
= N14,910.25
SI = PTR
100 2011/7a Neco
Recall that what he paid back is amount. A civil servant obtained a loan of N350,000.00. He is
Amount = Principal + interest expected to pay back the loan over 4years with a simple
720 = P + interest interest at a rate of 5 % per annum.
Thus interest is 720 – P i. Calculate the simple interest on the loan
Substituting ii. How much did he pay back?
P  54 iii. If this amount is paid back in monthly installments,
720 – P = how much did he pay back each month?
100
Solution
(720 – P) 100 = 20P
72000 – 100P = 20P (i) SI = PTR
100
72000 = 20P + 100P
Here P is N 350,000; T is 4; R is 5
72000 = 120P
72000 = P;
Substituting
120 SI = 350 000  4  5
P = $600 (A) 100
= N 70,000.00
2009/ 6 Neco (Dec) ( ii ) it refer to Amount = principal + interest
If the simple interest on N 4,500.00 for 3 years is = N 350,000 + N 70,000
N 540.00, find the simple interest on N 6,500.00 for 2 = N 420,000
years at the same rate. (iii) Time T is 4 years.
A N 390.00 B N 520.00 C N 540.00 We have 12 monthly installments payment
D N 590.00 E N 780.00
420,000
Solution =
We are to find rate from the first case of SI 4  12
SI = PTR = N 8,750 per month
100 2005/ 41 Neco Exercise 2.29
Here P is N 4,500; T is 3; R is ? SI is 540 Find the rate at which N 327.50 will yield N 78.60
Substituting in 6 years.
540 = 4500  3  R A 7% B 6% C 5% D 4% E 3%
100 2006/8 Neco (Dec) Exercise 2.30
540  100 = 4500  3  R If N10,000.00 is kept in a bank at the interest rate of 121/2%
540  100 = R per annum, how long will it take the money to yield an
4500  3 interest of N 2,500.00?
A 1 yr B 11/2 yrs C 2yrs D 21/2 yrs E 3 yrs
4% = R
In the second case: P is N 6,500, T is 2 and R is 4. 2007/5 Neco Exercise 2.31
Substituting Find the simple interest on N 2970.00 in 12 years at the rate
6500  2  4 of 6%
SI = = N 520 ( B ) A N 4125.00 B N 2238.40 C N 2138.40
100
D N 2069.20 E N 1069.50
2009/ 5 Neco 2010/8 Neco Exercise 2.32
A man deposited N14,600.00 in a Micro- finance Bank A man borrows N16,000.00 on condition that he pays back
on 16th July and on 9th of October the same year he N16,900.0 after 9months. At what rate percent per annum is
withdrew his money. Find the accrued amount if the interest charged?
bank pays an interest of 81/2% per annum A 2/15 % B 17/8 % C 21/2% D 55/8 % E 71/2%
25
2011/10 Neco Exercise 2.33 Note: The use of 1 for Time not 2 as the conventional
If N15,000.00 amount to N20,000.00 in two years at simple interest.
simple interest, what is the rate of interest?
A 121/2% B 162/3 % C 371/2 % D 662/3 % E 67 % Amount for 1st year = Principal + interest
( New principal ) = N 500 + N 30
2014/ 7 Neco Exercise 2.34 = N 530
Find the simple interest on N 700.00 for 9 years 2nd year interest
at 3% per annum. I2 = 530 x 6 x 1
A N 210.00 B N 189.00 C N 100.00 100
D N 76.00 E N 63.00 Note: the use of 1 year not 2 years.
2006/ 7 (Nov) Exercise 2.35 = 53 x 6
At what rate percent will N 4,800.00 amounts 10
to N 5,040.00 for 21/2 years simple interest? = 318
A 5% B 3% C 21/2% D 2% 10
= N 31.8.
2009/3(Nov) Exercise 2.36
Find the simple interest on N 5400.00 for Amount for 2nd years = N 530 + N 31.8
10 months at 5% per annum (Last principal) = N 561.8.
A N 207.00 B N 208.00 C N 225.00 D N 228.00 . . . Compound interest = Amount – initial principal
= N 561.8 – N 500
2001/8a (Nov) Exercise 2.37
= N 61.80 (C)
Find the simple interest on N 3000.00 in 31/2 years at
Alternatively
4% per annum. Give your answer to the nearest Naira.
1st year : Principal N 500
2012/7 Exercise 2.38 6% interest + 30 ( 6/100 x 500 )
If N 2,500.00 amounts to N 3,500.00 in 4 years at nd
2 year Principal N 530.00
simple interest, find the rate at which the interest was 6% interest + 31.80 ( 6/100 x 530 )
charged. N 561.80
A 5% B 7 1/2% C 8% D 10% Compound interest = N 561.80 – N 500
= N 61.80 (C)
Compound interest
It can be calculated in two ways: Formula method
Formula method Compound interest = amount – principal
We use the amount formula stated earlier
Compound interest = amount – principal R n
=P(1 + ) – P
=P 1 + R
n
– P 100
100 Here : P is 500 , R is 6 , n is 2 Substituting,
Where P is principal, R is rate and n is time
n Compound interest = 500 ( 1 + 6/100 )2 - 500
 R 
Note: A = P 1 +  =500 ( 1 + 0.06 )2 - 500
 100 
= 500 (1.06 )2 - 500
Without the use of formula (non use of amount formula) = 561.8 – 500
The use of simple interest calculation for each year = N 61.8 (C)
is vital here. Let us study the preceding examples
carefully. Example 2
Find the compound interest on N 400 for 2 years at 8% per
Example 1
annum.
The compound interest on N 500.00 for 2 years
A. N 32.00 B. N 34.56 C. N 66.56
at 6% per annum is ….
D. N 432.00 E. N 466.56
A. N 30.00 B. N 31.00 C. N 61.80
Solution
D. N 91.80 E. N 92.80.
Without use of the formula (non use of amount formula)
Solution Since rate for two years was mentioned in the question, we shall
Without the use of formula (non use of amount formula) have two different calculations.
We shall have two different calculations since two years Applying I = PRT
was mentioned 100
Apply the formula I = PRT , We have that 1St year interest
100 I1 = 400 x 8 x 1
1st year interest is initial “simple” interest from the 100
question. Please, note the use of Time 1 year and not 2 years
I1 = 500 x 6 x 1 =4x8
100 = N 32
=5 x 6 Amount for 1st year = N 400 + N32 = N 432
= N 30 (New Principal)
26
2nd year Interest: . . . Compound interest = Amount- initial principal
I2 = 432 x 8 x 1 = N 299.8274 - N 285.38
100 = N 14.4474
Please, note the use of T = 1 not 2  N 14.45 to the nearest
= 3456 = N 34.56 kobo
100
Amount for 2nd year = N 432 + N 34.56 = N 466.56 2008/7b
(Last Principal) A man inverts ƒ1500 for two years at compound interest.
. . . Compound interest = N 466.56 – N 400 After one year, his money amount to ƒ 1560. Find the;
= N 66.56 (C) (i) Rate of interest;
Alternatively, (ii) Interest for the second year.
1st year: Principal N 400 Solution
8% interest + 32 ( 8/100 x 400 ) Here we apply the non-formula method for compound interest
nd
2 year: Principal N 432
(i) I = PTR
8% interest + 34.56 ( 8/100 x 432 ) 100
N 466.56 I is ƒ 1560 – ƒ 1500 since interest is amount minus principal
Compound interest = N 466.56 – N 400
P is ƒ1500; R is ? T is 1
= N 66.56 ( C )
Substituting
Formula method 60 = 1500  R  1
Compound interest = amount – principal 100
R n 60 = 15R
=P(1 + ) – P 60
100 15 = R
Here : P is 400 , R is 8 , n is 2 Substituting, Thus R is 4%
8
Compound interest = 400 ( 1 + /100 ) - 400 2 ( ii ) 2nd year interest
=400 ( 1 + 0.08 )2 - 400 Already, we are told that amount is ƒ1560
= 400 (1.08 )2 - 400 1560  4  1
I2 =
= 466.56 – 400 100
= N 66.56 (C) = ƒ 62.40
If no restriction is given, we can use any of the 2011/ 4 Neco
methods in solving our compound interest Find the compound interest on N 225.00 invested for
problems but not both at the same time 20 years at 4% per annum.
A N 824.00 B N 718.00 C N 688.00
2004/6a (Nov) D N 652.00 E N 268.00
Find the compound interest (without use of formula) on Solution
N 285.38 in 2years at the rate of 21/2% per annum. Give Compound interest = amount – principal
your answer to the nearest kobo. n
 R 
Solution = P 1 +  −P
Here we are restricted to non formula method  100 
We shall have two different calculations since two years Here P is N 225; R is 4; n is 20
were mentioned Substituting
Applying the formula I = PTR , We have that 20

100 Compound interest = 225 1 + 4  − 225


 100 
1st year interest is initial “simple” interest from the
= 225 (1 + 0.04) − 225
20
question.
I1 = 285.38 × 1 × 2.5 = 225 (1.04) − 225
20
100
= N 7.1345 = 493.00 – 225
Note: The use of 1 for Time not 2 as in the conventional simple interest. = N 268.00 ( E )
2006/4a Neco
Amount for 1st year = Principal + interest By using formula, find the:
( New principal ) = N 285.38 + N 7.1345 ( i ) Simple interest;
= N 292.5145 ( ii ) Compound interest;
2nd year interest On N 2, 800.00 for 3 years at the rate of 4% per annum.
I2 = 292.5145 × 1 × 2.5 Solution
100 PRT
= N 7.3129 (i) I =
Note: the use of 1 year not 2 years. 100
2800  3  4
Amount for 2nd year = N 292.5145 + N 7.3129 = = N 336.00
(Last principal) = N 299.8274 100
27
( ii ) Compound interest = amount – principal CHAPTER THREE
n
 R  Algebraic Process
= P 1 +  −P Algebra is the aspect of mathematics that involves both
 100 
3
letters and numbers or letters only. Let’s consider some
= 2800 1 + 4  − 2800 examples below:
 100  (a) 2 + 3 – 16(5 + 4)
= 2800 (1 + 0.04) − 2800
3 (b) 92 + 26 – 1/4 + 1/20
(c) y+1
= 2800 (1.04) − 2800
3
(d) x+y=2
= 3149.62 – 2800 (e) x2 + 3x + 4
= N 349.62 (f) x2 – y2
2006/38 Neco The expression (a) and (b) are in numerical terms only.
Find the compound interest on N 450.00 in Hence, they cannot be called Algebra. Whereas (c) to (f) are
2 years at 5% algebraic expressions. The process of solving mathematical
A N 497.00 B N 496.13 C N 495.00 problems involving algebraic expression is called algebraic
D N 46.13 E N 45.00 process.
Solution Simple Equation (Linear)
Compound interest = amount – principal Simple linear equations are expressions whose highest
n degree of the unknown is one. Here we shall concern
 R  ourselves with linear equations in one variable.
= P 1 +  −P
 100  2006/8 (Nov)
Here P is N 450; R is 5; n is 2 Solve for x in the equation: 5(x – 7) – 7x = –3(4x – 5)
Substituting A 1/ 5 B 5/ 6 C2 D5
2
 5  Solution
Compound interest = 450 1 +  − 450 Opening brackets
 100 
5(x – 7) – 7x = –3(4x –5) becomes
= 450 (1 + 0.05) − 450
2
5x – 35 – 7x = – 12x + 15
Collect like terms together
= 450 (1.05) − 450
2
5x – 7x + 12x = 15 + 35
= 496.125 – 450 10x = 50
= N 46.13 x = 50/10 i.e 5 ( D )
2008/16 Neco
2005/ 40 Neco Exercise 2.39 If 5x – 3 = 4x – 7, what is the value of 6x?
Calculate the compound interest on N 2,000.00 for A 26 B6 C 4 D – 4 E – 24
2 years at 10% per annum. Solution
A N 420.00 B N 400.00 C N 320.00 First, we find x
D N 300.00 E N 200.00 5x – 3 = 4x – 7
2015/5b Neco Exercise 2.40 5x – 4x = 3 – 7
Calculate the compound interest on N 25,000.00 for x = –4
3 years at 12% per annum Thus, 6x = 6(– 4) i.e –24 ( E )
2014/12b NABTEB ( Nov ) Exercise 2.41 2013/ 18 Neco
Find the compound interest on N 450.00 in 3 years If 6x + 7 = 4x – 3, what is the value of 8x – 4?
at 4% per annum A – 44 B – 5 C – 1 D 36 E 44
Solution
First, we solve for x in 6x + 7 = 4x – 3
6x + 7 = 4x – 3
6x – 4x = – 7 – 3
2x = – 10
x = –10/2 i.e – 5
Substituting for x = – 5 in 8x – 4
8x – 4 = 8(–5) – 4
= – 40 – 4 i.e – 44 ( A )
2006/4a NABTEB (Nov)
Solve the equation: 3(2x – 7) = 2(x – 8)
Solution
Opening up brackets on both sides
6x – 21 = 2x – 16
6x – 2x = 21 – 16
4x = 5 Thus x = 5/4 i.e 11/4
28
2010/40 NABTEB (Nov) 3 2
− = 0
3x 2x 3 2( x − 2) 3(2 − x)
Simplify + =
5 15 5 3 2
=
9 9 7 8 2( x − 2) 3(2 − x)
A B C D
11 7 11 7 Cross multiply
Solution 9(2 – x) = 4(x – 2)
3x 2x 3 18 – 9x = 4x – 8
+ = 18 + 8 = 4x + 9x
5 15 5 26 = 13x
Multiply through by the LCM of 5 and 15 i.e 15 x = 26/13 i.e 2 ( A )
3(3x) + 2x = 3(3) 2008/39
9x + 2x = 9 1 2
11x = 9 If P = 1/2 and = , find the value of x
p −1 p+x
9
x = (A) A – 21/2 B –11/2 C 11/2 D 21/2
11 Solution
2006/23 Neco (Dec) 1 2
=
Find the value of t for which p −1 p+x
1 1 Substituting for P = 1/2
(t + 5) = (5t − 2)
3 4 1 2
=
4 3 4 3 4 1
2 −1 1
2 + x
A–2 B–1 C D1 E2
11 11 11 11 11 Cross multiply
1
Solution /2 + x = 2(– 1/2)
1
/2 + x = –1
1 1
(t + 5) = (5t − 2) x = – 1– 1/2
3 4 = –11/2 ( B )
Multiply through by 12, LCM of 3 and 4 2014 /6a
4(t + 5) = 3(5t – 2) 3 1

4t + 20 = 15t – 6 If = 3
, find p
20 + 6 = 15t – 4t 2p − 1
2
1
4 p + 1
26 = 11t Solution
4 First, we resolve the fraction inside fraction
26
t= /11 i.e 2 (E) 3 1
11
4 p −1 3
2007/14 Neco =
2 p+4
3b 1 3 b
Solve the equation + = + 4
9 2 4 4 4 p −1 1 p+4
A–3 B–2 C2 D3 E4 3 = 
Solution 2 3 4
Changing division to multiplication
3b 1 3 b
+ = + 2 1 4
9 2 4 4 3  = 
4 p −1 3 p+4
Collect like terms together
3b 3 1 6 4

b
= − =
9 4 4 2 4 p − 1 3( p + 4)
Multiply through by the LCM of 9, 4 and 2 i.e 36 Cross multiply to clear fraction
4(3b) – 9(b) = 9(3) – 18(1) 6  3(p + 4) = 4(4p – 1)
12b – 9b = 27 – 18 18p + 72 = 16p – 4
18p – 16p = – 72 – 4
3b = 9
2p = –76
b = 9/3 i.e 3 ( D ) p = – 76/ 2 i.e – 38
2009/2a (Nov)
2008/28 Neco (Dec)
4x − 1 3x − 1 5 − 2x
Solve the equation
3

2
= 0 Solve the equation − =
2( x − 2) 3(2 − x) 3 2 4
Solution
A2 B3 C6 D 13 E 26
4x − 1 3x − 1 5 − 2x
Solution − =
3 2 4
Multiply through by the LCM of 3, 2 and 4 i.e 12
29
4(4x – 1) – 6(3x – 1) = 3(5 – 2x) Multiply through by x (LCM)
16x – 4 – 18x + 6 = 15 – 6x 5 7
Collect like terms together x + x = x 4
16x – 18x + 6x = 15 + 4 – 6 x x
4x = 13 5 + 7 = 4x
x = 13/4 i.e 31/4 12 = 4x
12
2009/13 (Nov) / 4 = x i.e x = 3 ( E )
2 3 3
Solve for x if + = 2009/10b Neco (Dec) counter example
x 2x 2
2 3
7 3 6 7 Solve the equation − =1
A B C D x −1 x +1
6 7 7 3 Solution
Solution
Apply LCM method to the LHS
2 3 3 2( x + 1) − 3( x − 1)
+ = =1
x 2x 2 ( x − 1)( x + 1)
Find the LCM of x, 2x and 2
x x 2x 2 2 x + 2 − 3x + 3
=1
2 1 2 2 ( x − 1)( x + 1)
1 1 1 L C M of x, 2x and 2 is 2x − x+5
=1
Multiplying through by 2x ( x − 1)( x + 1)
2 3 3 Cross multiply
2x + 2x = 2x – x + 5 = (x – 1) (x +1)
x 2x 2 – x + 5 = x2 + x – x – 1
2(2) + 3 = 3x – x + 5 = x2 – 1
4+3 = 3x x +x–6=0
2
7= 3x Factorizing
x = 7/ (D)
3
x2 – 2x + 3x – 6 = 0
2008/27 NABTEB (Nov) x(x – 2) + 3(x – 2) = 0
5 8 (x – 2)(x + 3) = 0
Solve for x if + = 2 x – 2 = 0 or x + 3 = 0
m 3m
A 35/6 B 21/6 C 15/6 D 11/2 x = 2 or – 3
Solution
2005/31 Neco Special case
5 8
+ = 2 3y − 4x x
m 3m If = 5 , evaluate
4 y − 3x y
Find the LCM of 3m and m
m m 3m 17 16 16 14 3
A B C D E
3 1 3 11 11 17 17 4
1 1 L C M of m and 3m is 3m Solution
3y − 4x
Multiplying through by 3m =5
5 8 4 y − 3x
3m + 3m = 3m  2 Cross multiply
m 3m 3y – 4x = 5(4y – 3x)
3(5) + 8 = 3m(2)
3y – 4x = 20y – 15x
15 + 8 = 6m
23 = 6m
Collect like terms together
15x – 4x = 20y – 3y
23 11x = 17y
m= i.e 35/6 (A)
6 Divide through by y
2012/16 Neco 11x
5 7 = 17
If = 4 − , find the value of x y
x x x 17
A–1 B 1/3 C 1/2 D1 E3 = (A)
Solution y 11
5 7
= 4 −
x x
Collect like terms together
5 7
+ = 4
x x
30
Alternatively 25a 2 + 100a 2 20a 2
3y − 4x = 
=5 25 5
4 y − 3x 125a 2
5
Divide LHS through by y = 
25 20a 2
3y 4x
− 5
y y = i.e 11/4 ( C )
= 5 4
4y 3x
− Note: Author deliberately left 10a/5 unreduced to 2a as readers may
y y come across problems which will not reduce to whole number
x
3−4 2014/25 Neco
y
=5 1 1 1
x If + = , find y if x = 5
4−3 x y 3
y
A 61/2 B 71/2 C 81/2 D 91/3 E 91/2
Cross multiply Solution
x x 1 1 1
3–4 = 5(4 – 3 ) + =
y y x y 3
x x Substituting for x = 5
3–4 = 20 – 15
y y 1 1 1
+ =
Collect like terms together 5 y 3
x x Collect like terms together
15 –4 = 20 – 3
y y 1 1 1
= −
x y 3 5
11 = 17
y 1 5 − 3
=
x 17 y 15
= (A)
y 11 1 2
=
y 15
2006/27 Neco (Dec) 15 = 2y
2a − 3b a2 + b2 15
If = 4 , evaluate = y i.e y = 7 1/2 ( B )
3a − 2b 2ab 2
A3 B2 C 11/4 D 2/ 3 E 4/ 7 2009/1b
Solution If 9 is subtracted from half a certain number, the result is the
2a − 3b same as when 1/3 is added to the number. Find the number.
=4
3a − 2b Solution
Cross multiply Let the number be x
2a – 3b = 4(3a – 2b) 1st statement : 1/2x – 9
2a – 3b = 12a – 8b 2nd statement : 1/2x – 9 = x + 1/3
Collect like terms together Solving: 1/2x – 9 = x + 1/3
8b – 3b = 12a – 2a Multiply through by the LCM of 2 and 3 i.e 6
5b = 10a 3x – 54 = 6x + 2
10a Collect like terms together
b = 3x – 6x = 54 + 2
5
Substituting, we have – 3x = 56
2 56
 10a  x = i.e – 182/3
a + 2
 −3
a +b
2 2
 5 
=
2ab  10a  2006/7a Neco Exercise 3.1
2a    2 3
 5  Solve the equation: (3x − 5) − (2 x − 3) = 3
3 5
100a 2
a2 +
= 25 2009/35 Exercise 3.2
20a 2 2x + 1 3x − 1
Solve − = 0
5 6 4
A1 B 1/ 5 C – 1/ 5 D–1
31
2011/2 Exercise 3.3 Expansion of algebraic terms
Solve for x in the equation: 3 (2 x − 1) = 1 (5x − 3) The term expansion in algebraic mathematics is the same as
5 4 in arithmetic. It simply means opening up of barriers
A0 B1 C2 D3 (brackets). The expression
(t + s) (v + w)
2012/26 Neco Exercise 3.4 Means (t + s) x (v + w ). it can be expanded as:
Solve 2 x − 1 + 4 x + 1 = 3( x − 4) (t +s) (v + w) = t (v + w) + s (v + w)
3 5 4 = tv + tw + sv +sw
2008/46 NABTEB (Nov) Exercise 3.5 on the other hand, we can expand it as:
(t + s) (v + w) = (t + s ) v + (t + s) w
Solve for x if x = x + 2
4 6 3 = tv + sv + tw + sw
A3 B4 C6 D8
2005/30 Neco
2008/21 NABTEB (Nov) Exercise 3.6 Expand 2x(x – y) – 3y(4x + 3y)
m 2  2 A 2x2 – 14xy + 9y2 B 2x2 – 14xy – 9y2
Solve the equation  −   m +  C 2x2 – 10xy + 9y2 D 2x2 + 14xy – 9y2 E 2x2 – 10xy – 9y2
 2 3  3 Solution
A–6 B–5 C–4 D–3 2x(x – y) – 3y(4x + 3y) = 2x2 – 2xy – 12xy – 9y2
2013/4b Exercise 3.7 = 2x2 – 14xy – 9y2 ( B )
Solve the equation: ( x + 5) − (2 x − 1) = 5 2007/43
3 5 6 Expand the expression (3a – xy)(3a + xy)
2009/9 Exercise 3.8 A 9a2 – x2y2 B 9a2 + x2y2 C 9a2 – xy D 9a2 + x2y
If 3 2 Solution
− = 4 , solve for x
2x 3x (3a – xy)(3a + xy) = 3a(3a + xy) – xy(3a + xy)
4 = 9a2 + 3axy – 3axy – x2y2
A B 4 C 5 D 13
5 13 24 24 = 9a2 – x2y2 ( A )
2010 /16 Neco
2012/8 Neco Exercise 3.9 If x2 – 5x + c = (x – 8)( x + 3). Find the value of c
Solve for x in the equation: 1 + 2 = 1 A 24 B 5 C–5 D–9 E – 24
x 3x 3
Solution
A5 B4 C3 D1
x2 – 5x + c = (x – 8)( x + 3)
2015/2b Neco Exercise 3.10 Expanding RHS
1 2 = x(x + 3) – 8( x + 3)
Solve the equation : ( 2x + 1 ) – (x–2) = 3 = x2 + 3x – 8x – 24
3 5
x2 – 5x + c = x2 – 5x – 24
2015/7 Neco Exercise 3.11
Thus c = – 24 ( E )
If 12x – 5 = 9x – 72 , find 10x 2005/5b
A. – 5 B. – 1
2
C. 1
2
D. 5 E. 12 If ( m + 1 ) and ( m – 3 ) are factors of m2 – km + c ,
find the values of k and c
2014/8b Neco ( Nov) Exercise 3.12 Solution
2x + 5 3x + 2 1
Solve the equation : – = If (m + 1) and (m – 3) are factors of m2 – km + c, then
3 4 6
(m + 1)(m – 3) = m2 – km + c
m(m – 3) + 1(m – 3) = m2 – km + c
m2 – 3m + m – 3 = m2 – km + c
m2 – 2m – 3 = m2 – km + c
Thus – 3 = c and – 2m = – km
2= k
2011/17 Neco Exercise 3.13
What is the common factor of the expression
y2 + y and y2 – 1?
A 2y B y C y – 1 D y + 1 E 2y – 1

Coefficient of term in expansion


(a) Find the coefficient of w in the expansion of
(8w + 3 )(8w – 3)
Solution
(8w + 3)(8w – 3) = 8w (8w – 3) + 3 (8w - 3)
= 64w2 – 24w + 24w - 9
= 64w2 – 9
Hence the coefficient of w is zero.
32
(b) Find the coefficient of xy in the expansion Factorization
(2x – y + 4)(5x + 3y) The term factorization is the reverse of expansion, the
Solution subtopic preceding this. The process of factorization
Using the smaller bracket of the two terms to multiply, involves factoring out a common factor from a given
(2x – y + 4)(5x +3y ) = 5x(2x – y + 4 ) + 3y(2x – y + 4) expression. This is done based on the given problem.
= 10x2 – 5xy + 20x + 6xy – 3y2 + 12y Different strokes for different folks. Some methods are as
Collect like terms together shown next:
= 10x2 – 5xy + 6xy + 20x +12y – 3y2 One term factor
= 10x2 + xy + 20x + 12y – 3y2 When confronted with factorization problems of the forms
Thus the coefficient of xy is + 1 shown below, it is helpful if we factor out a letter or term
common to both sidesz
2007/15 Neco Examples
What is the coefficient of x in the expansion a.) ax + ay = a (x + y)
(2x – 3)(3x + 8)? b.) at2- a2t = at (t - a )
A – 25 B – 24 C6 D7 E 25 c.) xy2z3 – x2yz2 = xyz2 (yz – x )
Solution d.) s (t – 1 ) + 3 (1 – t) = s(t - 1) – 3 ( t - 1)
First, we expand as: = (t - 1)(s –3)
(2x – 3)(3x + 8) = 3x(2x – 3) + 8(2x – 3)
= 6x2 – 9x + 16x – 24 2011/28
= 6x2 + 7x – 24 Factorize the expression: am + bn – an – bm
Thus the coefficient of x is 7 ( D ) A (a – b)(m + n) B (a – b)(m – n)
C (a + b)(m – n) D (a + b)(m + n)
2012/35 Solution
Find the coefficient of m in the expansion of Grouping like terms together
am + bn – an – bm = am – bm + bn – an
m  2
 − 1 12   m +  = m(a – b) + n(b – a)
2  3 We will not get the desired result; so we factor – n instead of + n
A – /6
1
B – /2
1
C –1 D – 11/6 = m(a – b) – n(– b + a)
Solution Same as = m(a – b) – n(a – b)
m  2 m 3  2 = (m – n)(a – b) B
 − 1 12   m +  =  −   m + 
2  3  2 2  3 2010/2
Expanding, we have Factorize : x + y – ax – ay
m 3 2m 3 A(x – y)(1 – a) B(x + y)(1 + a) C(x + y)(1 – a) D(x – y)(1 + a)
= m − +  −  Solution
2 2 3 2 2 Group like terms together
2
m 3 m x + y – ax – ay = x – ax + y – ay
= − m + −1 = x(1 – a) + y(1 – a)
2 2 3
= (x + y)(1 – a) C
m  − 9m + 2m 
2
= +  −1 2005/11
2  6 
2
Factorize: x – ay + y – ax
m 7 A(1 – a)(x – y) B(1 + a)(x – y) C(1 + a)(x + y) D(1 – a)(x + y)
= − m −1
2 6 Solution
Thus the coefficient of m is – 7/6 i.e – 11/6 ( D ) Group like terms together
x – ay + y – ax = x – ax – ay + y
2008/24 Neco Exercise 3.14 = x(1 – a) – y(a – 1)
Find the coefficient of xy in the expansion of We will not get the desired result; so we factor + y instead of – y
(x – 4y)(3x + 2y) = x(1 – a) + y(– a + 1)
A 14 B 12 C 10 D – 10 E – 12 Same as = x(1 – a) + y(1 – a)
2009/21 Neco (Dec) Exercise 3.15 = (x + y)(1 – a) D
Find the coefficient of x in the expansion of 2005/45 (Nov)
(3x +2)(5x – 8)? Factorize: 3xg – hy + 3hg – xy
A – 16 B – 15 C – 14 D 1 E 14 A (3g + y)(x + h) B (3g – y)(x + h)
C (3x – y)(3g + h) D (x – h)(3g – y)
2013/ 27 Neco Exercise 3.16 Solution
In the expansion of (3x2–2x+7)(2x+3), what is the Grouping like terms together
coefficient of x2 3xg – hy + 3hg – xy = 3xg + 3hg – hy – xy
A–5 B3 C5 D6 E8 = 3g(x + h) – y(h + x)
Same as = 3g(x + h) – y(x + h)
= (3g – y) (x + h) B
33
2008/14 NABETEB (Nov) Exercise 3.17 2009/14 Exercise 3.20
Factorize ab – ad + cb – cd Factorize x2 – 2x – 3xy + 6y completely
A (a – c)(b – d) B (a – c)(b + d) A (x – 2)(3x – y) B (x – 3y)(x – 2)
C (a + c)(b – d) D (a + c)(b + d) C (x – 2)(x + 3y) D (3x + y)(x – 2)
2008/45 NABETEB (Nov) Exercise 3.18 2011/8 Exercise 3.21
Factorize the expression 4abxy – 4acxy + 4dxy One of the factors of (mn – nq – n2 + mq) is (m – n)
A 4y(ab – ac + d) B 4x(ab – ac + d) The other factor is
C 4xy(ab – ac + d) D 4xy(b – a + ad) A (n – q) B (q – n) C (n + q) D (q – m)
2014/3 NABTEB ( Nov) Exercise 3.22
2015/8 Exercise 3.19 If ( x – a ) is a factor of bx – ax + x2 – ab, what is the other factor?
Factorise completely 6ax – 12by – 9ay + 8bx A.( x + b ) B.( b – a ) C. ( x – a ) D. ( a + b )
A.(2a – 3b)(4x + 3y) B. (3a + 4b)(2x – 3y)
C.(3a – 4b)(2x + 3y) D.(2a + 3b)(4x –3y) 2014/1a ( Nov) Exercise 3.23
Factorise completely : m2 – 2mn + n2 – 9r2
2014/18
If (x – a) is a factor of bx – ax + x2 – ab, find the other factor Difference of two squares expressions
A (x + b) B (x – b) C (a + b) D ( a – b) These are expressions of the form
Solution x2 – y2 = (x – y ) (x + y)
Grouping like terms together 2014/53 Neco
bx – ax + x2 – ab = x2 – ax – ab + bx Factorize (25y2 – 4)
= x(x – a) – b( a – x) A (5y – 2)(5y + 2) B (2y – 5)(5y + 2)
Step cannot help us C (5y – 2)(5y – 2) D (5y + 2)(5y + 2) E (5y – 2)(2 – 5y)
= x(x – a) + b(– a + x) Solution
Same as = x(x – a) + b(x – a) Working with an eye of difference of two squares
= (x – a)(x + b) 25y2 – 4 = 52y2 – 22
Thus, the other factor is (x + b) A. = (5y)2 – 22
= (5y – 2)(5y + 2) A
2011/19 Neco 2012/25 Neco
Factorize 5xy + 90qy – 30y2 – 15xq Factorize 8ab2 – 32a2
A 5(y – 3q)(x – 8y) B 5(y – 3q)(x + 6y) A 4a(b – 2a)(b + 2a) B 4a(b – 2a)2
C 5(y + 3q)(x + 8y) D 5(y – 3q)(x – 6y) C 4a(b + 2a) 2
D 4a(b + 2a)3 E 4a(b – 2a)3
E 5(y + 3q)(x – 6y) Solution
Solution Working with an eye of difference of two squares
Group like terms together 8ab2 – 32a3 = 2a(4b2 – 16a2)
5xy + 90qy – 30y2 – 15xq = 90qy – 30y2 + 5xy – 15xq = 2a [(2b)2 – (4a)2]
= 30y(3q – y) + 5x(y – 3q) = 2a(2b – 4a)(2b + 4a)
Our purpose is not served in this step Factor out 2
Retaking the step = 30y(3q – y) – 5x(– y + 3q) = 4a(b – 2a)(b + 2a) A
Which is the same as = 30y(3q – y) – 5x(3q – y) 2009/22 Neco (Dec)
= (30y – 5x)(3q – y) Factorize (5d – 2e)2 – 9e2
The first bracket can be factorized as A (5d – 2e)(3d – 3e) B 5(d + e)(5d + e)
= 5(6y – x)(3q – y) D C 5(d – e)(5d + e) D (5d – 3e)(3d + 3e) E (5d – e)(3d – e)
Solution
2010/41 NABTEB (Nov) Working with an eye of difference of two squares
What result do you get when 6x2 + 18x + 12xy – 24x2y2
(5d – 2e)2 – 9e2 = (5d – 2e)2 – (3e)2
is factorized?
= [(5d – 2e) – 3e] [(5d – 2e) +3e]
A (6x + x)(3y + 2 – 4xy2) B 6x(x + 3 + 2y – 4xy2)
= (5d – 2e – 3e)(5d – 2e + 3e)
C (6x + 3y)(2y – 4x2y) D (6x + 2y – 4x)(3y – xy)
= (5d – 5e)(5d + e)
Solution
= 5(d – e)(5d + e) C
6x2 + 18x + 12xy – 24x2y2 = 6x(x + 3 + 2y – 4xy2) B
2007/16 Neco
If we try 6x2 + 18x + 12x – 24x2y2 Factorize 50k4 – 8b2
= 6x(x + 3) + 12y(x – 2x2y) dead end A (5k2 – 2b)(5k2 + 2b) B 2(5k2 – 2b)(5k2 + 2b)
C (5k2 – 2b)(5k2 – 2b) D 2[(5k2 + 2b)(5k2 + 2b)]
2009/11 (Nov) E 2[(5k – 2b)(5k + 2b)]
2 2
Factorize completely : x2 – x – ax + a Solution
A (x – a)(x – 1) B (x – a)(x + 1) Working with an eye of difference of two squares
C (x – a)(x – 1) D (x + a)(x + 1) 50k4 – 8b2 = 2(25 k4 – 4b2)
Solution = 2[(5k2)2 – (2b)2]
x2 – x – ax + a = x(x – 1) – a(x – 1) = 2[(5k2 – 2b) (5k2 + 2b)] E
= (x – a)(x – 1) A
34
2011/3b Neco The factors + 3 and – 1 satisfy our condition, Thus
Use the idea of difference of two squares to find the x2 + 2x – 3 = x2 + 3x – x –3
exact value of k in the equation : 24k = 1062 – 942 = x(x + 3 ) – 1 (x + 3)
Solution = (x + 3) (x – 1)
24k = (106 – 94)(106 + 94)
24k = ( 12 )(200)
(2) Factorize completely x2 – 4x – 12
24k = 2400
As usual we find the factors of C (- 12) that
k = 2400 When added will give b (- 4)
24
k = 100 Factors of –12 sum
12 and –1 11
2007/6a NABTEB 3 and – 4 -1
Factorize completely: (x2 + x)2 – (2x + 2)2 2 and – 6 -4
Solution The last factors 2 and – 6 satisfied our condition, thus
Applying difference of two squares x2 – 4x – 12 = x2 + 2x – 6x – 12
(x2 + x)2 – (2x + 2)2 = x(x + 2 ) – 6(x + 2)
= [(x2 + x) + (2x + 2)] [(x2 + x) – (2x +2)] = (x + 2 ) (x – 6)
= (x2 + x + 2x + 2)(x2 + x – 2x – 2) Note: To be objective minded, readers need not to go through long
= (x2 + 3x + 2)(x2 – x – 2) process of trial and error before arriving at the needed factors.
Factorizing each quadratic expression
2010/42 NABTEB (Nov)
= (x2 + 2x + x +2)(x2 – 2x + x – 2)
Factorize x2 + 7x + 12
= [x(x + 2) + 1( x + 2)] [x(x – 2) + 1(x – 2)]
A (x + 4)(x + 3) B (x – 7)(x – 7)
= [(x + 2)( x +1)] [(x – 2)(x + 1)]
C (x + 7)(x – 7) D (x – 49)(x – 49)
= (x + 2)( x +1)(x – 2)(x + 1)
Solution
Here the factors of 12 that when added gives +7 are 3 and 4
2006/10 (Nov) Exercise 3.24
x2 + 7x + 12 = x2 + 4x + 3x + 12
Factorize completely : 147x2 – 3y2
= x(x + 4) + 3(x + 4)
A 3x(49x – y) B 3xy(49x – y) C 3(7x – y)2
= (x + 4)(x + 3) A
D 3(7x + y)(7x – y)
2005/2a( i ) Neco Exercise 3.25 2015/32 Neco Exercise 3.28
Factorize 25a2 – 9b2 Factorise n2 + 24n – 81
1977/2a Exercise 3.26 A.(n – 3)(n + 27) B. (n + 3)(n – 27)
If 13V = 5732 – 5602 , find the value of V C.(n – 9)(n + 9) D.(n – 9)(n –9) E.(n – 1)(n + 81)

2014/39 ( Nov) Exercise 3.27


When the coefficient of x2 is not unity. (-1 inclusive)
Find the common factor of ( 9r2 – 16s2) and
Here we look for the factors of ac that when added gives us b.
( 12r + 16s)
Examples
A.4(3r + 4s) B.4(3r – 4s) C.(3r – 4s) D.(3r + 4s)
(1) Factorize 12 +11x - x2
2014/52 Neco Exercise 3.27b Here a ≠ 1 rather a = - 1
Simplify (4x + 1)2 – (3x – 1)2 Thus ac = (-1) x 12 i.e - 12 and b = +11
A 7x (x – 2) B 7x (x + 2) C – 7x (x – 2) Factors of – 12 that when added gives us +11
D – 7x (x + 2) E 49x are +12 and –1
12 + 11x – x2 = 12 + 12x – x – x2
= 12 (1 + x ) – x (1 + x )
= (1+ x ) (12 – x)
Trinomial expression (2) Factorize 2y2 + 11y +15
These types of expressions are always in three terms. Solution
The methods of solving the trinomials which are mostly ac = 30, b = 11, trying the factors of 30 that when added
quadratic (ax2 + bx + c ) in nature is in two ways: give us + 11, we have + 6 and + 5
2y2 + 11y +15 = 2y2 + 6y +5y +15
When coefficient x2 is unity (a = 1) = 2y (y + 3 ) +5 (y + 3 )
Examples = ( y + 3 ) (2y + 5 )
(1) Factorize x2 + 2x – 3
Solution (3) Factorize 5x2 +17xy + 6y2 special case
We find the factors of c (–3) that when added together Solution
gives us b (+ 2) i.e. comparing the given expression with Here we treat the equation which appears in square of two
ax2 + bx + c different functions as one square of function of x.
Factors of –3 sum Thus ac = 30 and b =17
- 3 and + 1 -2 The factors of +30 that when added will give + 17
+3 and – 1 +2 Are + 2 and +15. Thus,
35
5x2 +17xy + 6y2 = 5x2 + 15xy +2xy + 6y2 2013/16 Neco Exercise 3.31
= 5x (x + 3y) +2y (x + 3y) Factorize 2a2 + 7ab – 15b2
= (x + 3y) (5x + 2y) A (2ab + 3)(ab + 5) B (2ab + 3)(ab – 5)
C (5ab + 3)(2ab – 3) D (2a – 3b)(a + 5b)
2005/2a ii E (2a + 3b)(a – 5b)
Factorize 6 + x – 2x2
Solution 2012/7b factor theorem
Here ac is –12 and b is + 1, factor of –12 that when Given that x2 + bx + 18 is factorized as (x + 2)(x + c).
added gives +1 are –3 and 4 Find the value of c and b
6 + x – 2x2 = 6 – 3x + 4x – 2x2 Solution
= 3(2 – x) + 2x(2 – x) If (x + 2) and (x + c) are factor then
= (2 – x)(3 + 2x) x+2 = 0
x = –2
2010/23 Neco Put x = – 2 into original equation
Factorize the expression 42 – 15x – 3x2 x2 + bx + 18 = 0 becomes
A 3(x + 7)(x – 2) B 3(7 – x)(2 – x) 2
(– 2) + b(–2) + 18 = 0
C 3(x – 7)(2 – x) D 3(7 + x)(2 – x) E 3(7 + x)(2 + x) 4 – 2b +18 = 0
Solution 4 +18 = 2b
42 – 15x – 3x2 = 3(14 – 5x – x2) 22 2b
Here ac is – 14 and factor of – 14 that add up to give –5 =
2 2
are – 7 and 2
b = 11
3(14 – 5x – x2) = 3(14 – 7x + 2x – x2)
Substitute for b and factorize
= 3[7(2 – x) + x(2 – x)]
x2 + 11x + 18 = x2 + 2x + 9x + 18
= 3(2 – x)(7 + x) D
= x(x + 2) + 9(x + 2)
= (x + 2)(x + 9)
2007/41 Exercise 3.29
Thus c is 9
Which of the following is a factor of 2 – x – x2 ?
A1–x B1+x Cx–1 D2–x
2010/20 Neco factor theorem
2008/28 Neco Exercise 3.30 Find the value of P if y – 2 is a factor of y 2 – py – 10
Factorize 35 – 2x – x2 A–3 B–2 C2 D3 E5
A (5 + x)(7 + x) B (5 + x)(x – 7) C (5 – x)(7 + x) Solution
D (x – 5)(x + 7) E (x – 5)(x – 7) If (y – 2) is a factor, then
y–2 = 0
2006/22 Neco (Dec) Special case y = 2
Factorize 2t2 – ty – 3y2 Next we put y = 2 into y2 – py – 10
A (2t – 3y)(t – y) B (2t – 3y)(t + y) 22 – p(2) – 10 = 0
C (2t + 3y)(t – y) D (2t + 3y)(t + y) 4 – 2p – 10 = 0
E (2t + 3y)(t + y)2 4 – 10 = 2p
Solution – 6 = 2p
Here we treat the equation which appears in squares of p = – 6/2 i.e – 3 (A)
two different functions as one square.
Here ac is – 6 and b is –1, factors of – 6 that when 1992/15 PCE Exercise 3.32
added gives –1 are 2 and –3 If x – 2 is a factor of 2x2 – x + k then k is
2t 2 – ty – 3y2 = 2t2 + 2ty – 3ty – 3y2 A. 6 B. 2 C. – 3 D. – 6
= 2t(t + y) – 3y(t + y)
= (2t – 3y)(t + y) B
2008/12 special case
Factorize 5y2 + 2ay – 3a2
A (a – y)(5y – 3a) B (y – a)(5y – 3a)
C (y – a)(5y + 3a) D (y + a)(5y – 3a)
Solution
Here we treat the equation which appears in squares of two
different functions as one square. Coefficient of y2 is not
unity.
ac is –15 and b is 2 the factors of –15 that when added
gives 2 are 5 and – 3. Thus
5y2 + 2ay – 3a2 = 5y2 + 5ay – 3ay – 3a2
= 5y(y + a) – 3a(y + a)
= (5y – 3a)(y + a) D

36
Other simplification of algebraic terms 2014/57 Neco
2005/3b Neco Divide x2yz + xy2z by x2y – y3
Simplify 4{x – [2(3 – x) – 3(2x +1) + 1]} xy xz xy
Solution A B C
z ( xy − y 2 ) x− y x−z
Starting from the inner brackets
4{x – [2(3 – x) – 3(2x +1) + 1]} xz xy
D E
= 4{x – [6 – 2x – 6x – 3 +1]} x+ y x+z
= 4{x – [4 – 8x]} Solution
= 4(x – 4 + 8x) We factor out common terms in the numerator and
= 4x – 16 + 32x = 36x – 16 denominator
2008/20 Neco x 2 yz + xy 2 z xyz ( x + y )
Find the HCF of x2yz + xy2z and x2y – y3 =
x2 y − y3 y( x 2 − y 2 )
A x(x + z) B xy(x + y) C y(x + y)
D yz(z + x) E z(x + z) Difference of two squares at denominator
Solution xyz ( x + y )
=
HCF of x2yz + xy2z and x2y – y3 y ( x − y )( x + y )
First, we factor out common terms from each item xyz
x2 yz + xy2z = xyz(x + y) =
and x2y – y3 = y(x2 – y2) y( x − y)
= y(x – y)(x + y) =
xz
B
From our result; y(x + y) is the HCF x− y
2014/31 NABTEB 2008/25 NABTEB (Nov) Exercise 3.33
Find the least common multiples of x2y(x – y) Calculate the LCM of 2ax2y, 8a2xy3 and 6ax3y2
and xy2(x – y) A 16x2y3 B 18a2x3y C 24ax2y3 D 24a2x3y2
A.xy(x – y) B.x2y(x – y) C.x2y2(x – y) D.x2y2(x – y)2
Solution 2014/37 Exercise 3.34
(x – y) x2y(x – y) xy2(x – y) Subtract 1/2(a – b – c) from the sum of 1/2 (a – b + c)
xy x2y xy2 and 1/2(a + b – c)
x x y A 1/2(a + b + c) B 1/2(a – b – c)
y 1 y C 1/2(a – b + c) D 1/2(a + b – c)
1 1 2008/25 Neco Exercise 3.35
Thus LCM is (x – y) × xy × x × y = x2y2(x – y) Simplify 5x – [4y + 3{x – 2(y – x)}]
A 4x + 2y B 4x – 2y C 2(2x – y)
2014/ 24 Neco D – 2(2x – y) E – 4(x + y)
1
Simply [5 y − (2 + 3 y) + (7 y − 4)]
3 Simplification of algebraic fractions
A 6y + 4 B 3y + 2 C 6y + 2 D 3y – 2 E 9y – 6 Simplification of algebraic fractions is done by applying the
Solution same principles as in the case of fractional numbers only.
Simplifying from the inner bracket The basic rules of multiplication, addition and subtraction
1 1 also apply in this case. For instance, if we are asked to
[5 y − (2 + 3 y ) + (7 y − 4)] = [5 y − 2 − 3 y + 7 y − 4]
3 3 simplify the following :
1 1 81 21 1 81 21
= [9 y − 6] (1)   or
3 3 7 3 373
= 3y – 2 D 1 2
(2) +
4 3
2010/30 Neco 3 1
Find the sum of 25a – 15b + c, 13a – 10b + 4c (3) +
5 6
and a + 20b – c
A 12a – 5b + 5c B 12a + 5b – 5c C 13a + 5b + 4c The above problem (1) makes use of canceling out of factors
D 39a – 5b + 4c E 39a + 5b + 4c in numerator and denominator while (2) and (3) make use of
Solution L.C.M of the denominators,
Doing it arithmetically, we have
25a – 15b + c (a) Division and multiplication of algebraic fractions
+ 13a – 10b + 4c Here we apply the principles learnt in the preceding
a + 20b – c subtopics to factorize the numerator and the denominator
39a – 5b + 4c D “ separately” then we reduce the resulting terms where
possible; if they are not in their simplest form

37
2007/10a 2013/9
x −y 2 2
x2 − y2 ( x − y) 2
Simplify Simplify: 
3x + 3 y ( x + y) 2 (3x + 3 y )
Solution x− y 3
We simplify numerator and denominator differently A Bx+y C Dx–y
3 x− y
x2 − y2 ( x − y )( x + y ) Solution
=
3x + 3 y 3( x + y ) Let us take each terms differently as:
x− y x 2 − y 2 ( x − y ) 2 = ( x − y )( x + y ) ( x − y )( x − y )
=  
3 ( x + y ) 2 (3x + 3 y ) ( x + y)( x + y) 3( x + y)
2007/2a x − y ( x − y )( x − y )
= 
x − 8 x + 16
2
x+ y 3( x + y )
Simplify
x 2 − 7 x + 12 Changing from  to 
Solution x− y 3( x + y )
We factorize numerator and denominator differently = 
x + y ( x − y )( x − y )
x 2 − 8 x + 16 x 2 − 4 x − 4 x + 16 3
= = C
x 2 − 7 x + 12 x 2 − 3x − 4 x + 12 x− y
x( x − 4) − 4( x − 4)
=
x( x − 3) − 4( x − 3) 2008/15 Exercise 3.36
( x − 4)( x − 4) x−4 2 x 2 − 5 x − 12
= = Simplify:
( x − 4)( x − 3) x−3 4x 2 − 9
2008/9 x+4 x+4 x−4 x−4
A B C D
1 1 2x + 3 2x − 3 2x + 3 2x − 3
+
x y
Simplify : 2012/9 Exercise 3.37
x + y
54k 2 − 6
1 1 Simplify
A B Cx+y D xy 3k + 1
x+ y xy A 6(1 – 3k2) B 6(3k2 –1) C 6(3k –1) D 6(1 – 3k)
Solution
1 1 2014/2a Neco Exercise 3.38
+ a 2 − 4b 2
x y 1 1 Simplify
=  +   ( x + y ) a 2 − 5ab + 6b 2
x + y  x y 
Simplifying the first bracket 2014/45 ( Nov) Exercise 3.39
 y+ x m − n2 2

=    ( x + y ) Simplify :
 xy  n − m
Changing  to  A. m + n B. – m – n C. – m + n D. m – n
x+ y 1
=  (b) Addition and subtraction of algebraic fractions
xy x+ y
y + x is same as x + y just like 2 + 3 and 3 + 2 2006/43 (Nov)
1 x +1 3x − 1
= B
xy Express − as a single fraction.
2 3
2009/14 (Nov)
3x − 1 1 − 3x 5 − 3x 3x − 5
12ab − 4b 2 A B C D
Evaluate : 6 6 6 6
2b 2 − 6ab Solution
A. – 2 B. – 1 C. 1 D. 2 The LCM of the denominators 2 and 3 is 6
Solution
x +1 3x − 1 3( x + 1) − 2(3x − 1)
Simplifying numerator and denominator differently − =
12ab − 4b 2 4b(3a − b) 2 3 6
=
2b − 6ab
2
2b(b − 3a) 3x + 3 − 6 x + 2
=
To get a better result, we factor out –2b instead of 2b 6
4b(3a − b) 5 − 3x
= = –2 A = C
− 2b(3a − b) 6
38
2009/12 ( Nov ) 2005/5 (Nov)
t2
t 1 x 2
Simplify : − + Simplify : − , for x  2 and x  3
12 3 3 x−2 x−3
A 3(t – 2)2 B 1 3(t – 2)2 C t−2 D 1 (t – 2)2 A1 B 1 C − 2x D ( x − 1)( x − 4)
12 12 12 x−3 ( x − 3)( x − 2) ( x − 3)( x − 2)
Solution Solution
LCM of 12 and 3 is 12. Thus The LCM of the denominator x – 2 and x – 3 is (x – 2) (x – 3).
t2 t 1 t 2 − 4t + 4 x 2 x( x − 3) − 2( x − 2)
− + = − =
12 3 3 12 x−2 x−3 ( x − 2)( x − 3)
t − 2t − 2t + 4
2
x − 3x − 2 x + 4
2
= =
12 ( x − 2)( x − 3)
t (t − 2) − 2(t − 2)
= x 2 − 5x + 4
12 =
( x − 2)( x − 3)
(t − 2)(t − 2)
= Factorizing the numerator
12 x 2 − x − 4x + 4
(t − 2) 2 =
= D ( x − 2)( x − 3)
12
x( x − 1) − 4( x − 1)
2012/20 Neco =
2x + 1 3x − 7 5 ( x − 2)( x − 3)
Simplify − − ( x − 1)( x − 4)
2 9 18 = D
A 2x + 1 B 2x + 6 C 2x + 1 ( x − 3)( x − 2)
2 x + 18 2x + 3 2005/38
D E 2 3
3 3 Simplify −
Solution 3xy 4 yz
The LCM of denominators 2, 9 and 18 is 18 2− x 2 z − 3x 4 z − 3x 8z − 9 x
A B C D
2 x + 1 3x − 7 5 9(2 x + 1) − 2(3x − 7) − 5 12 xyz 12 xyz 12 xyz 12 xyz
− − =
2 9 18 18 Solution
18 x + 9 − 6 x + 14 − 5 The LCM of the denominators 3xy and 4yz is
= 3 3xy 4yz
18 2 xy 4yz
12 x + 18 2 xy 2yz
= x xy yz
18
y y yz
6(2 x + 3) z 1 z
=
18 1 1 LCM 322  x  y  z = 12xyz
2x + 3
= E 2 3 4 z (2) − 3(3x)
3 − =
2007/46 3xy 4 yz 12 xyz
4 2+ x 8z − 9 x
Simplify : − = D
2x x 12 xyz
2− x 2005/1a
A –1 B – 2x C 2x D 1 2 3
x Express as a single fraction, − + 2
Solution y +1 y −1 y −1
LCM of 2x and x is 2x. Thus Solution
4 2+ x 4 − 2(2 + x) The LCM of the denominators y + 1, y – 1 and y2 – 1
− =
is y2 – 1 since y2 – 1 = y2 – 12 i.e (y – 1) (y + 1)
2x x 2x
4 − 4 − 2x 1 2 3 1( y −) − 2( y + 1) + 3
= Thus − + 2 =
2x y +1 y −1 y −1 ( y + 1)( y − 1)
− 2x y −1− 2y − 2 + 3
= =
2x ( y + 1)( y − 1)
= –1 A y
= −
( y + 1)( y − 1)
39
2005/32 Neco 2008/16 Neco (Dec)
 2x   1 2 x+4
Simplify  2 −  +  3 +  Evaluate 2 + −
 3y   y x 2
3y − 2x + 3 9x − 2x + 1 15 y − 2 x − 3 2 − x2 8 − x2 4 − x2 4 + 8x − x 2
A B C A B C D
3y 3y 2
y 2x 2x 2x 2x
15 y + 2 x + 3 8 − 4x − x 2
15 y − 2 x + 3 E
D E 2x
3y 3y 2
Solution
Solution 2 x+4
First we simplify each bracket 2+ −
 2x   1  6 y − 2x   3 y + 1  x 2
 2 −  +  3 +  =   +   LCM of the denominators x and 2 is 2x
 3y   y  3y   y  2 x+4 2(2 x) + 2(2) − x( x + 4)
The LCM of the denominator 3y and y is 3y 2+ − =
x 2 2x
6 y − 2 x + 3(3 y + 1)
= 4x + 4 − x 2 − 4x
3y =
2x
6 y − 2x + 9 y + 3
= 4− x 2

3y = C
2x
15 y − 2 x + 3 2011/31
= D
3y m (m − 1) (m − 2)
Simplify: + − when n  o
n 5n 10n
2006/18 Neco (Dec)
m−3 11m m +1 11m + 4
4 3 p A B C D
If − is expressed as 10n 10n 10n 10n
x−5 x−6 ( x − 5)( x − 6)
Solution
then P is equal to LCM of n, 5n and 10n is 2  5  n i.e 10n
A x+9 B x+5 C x–9 D x – 39 n n 5n 10n
Solution 5 1 5 10
The LCM of the denominators x – 5 and x – 6 is 2 1 1 2
(x – 5)(x – 6). Thus 1 1 1 LCM = 2 × 5 × n
4 3 4( x − 6) − 3( x − 5)
− =
x−5 x−6 ( x − 5)( x − 6) m (m − 1) (m − 2) 10m + 2(m − 1) − 1(m − 2)
+ − =
4 x − 24 − 3x + 15 n 5n 10n 10n
= 10m + 2m − 2 − m + 2)
( x − 5)( x − 6) =
x−9 10n
= 11m
( x − 5)( x − 6) = B
Hence P is x – 9 C 10n
But if we are asked to solve and not simplify
2007/17 Neco m (m − 1) (m − 2)
m−n n−m + − = 0
Express as a single fraction : − −1 n 5n 10n
m n We multiply through by the LCM 10n
m + n − mn
2 2
n − m − mn 2 2 10m + 2(m – 1) – 1 (m – 2) = 0
A B 10m + 2m – 2 – m + 2 = 0
mn mn Explanation only; not a good example in this direction
m + n + mn
2 2
m − n 2 − mn
2
m+n
C D E 2012/28 Neco
mn mn mn
2a a
Solution Simplify +
LCM of m and n is mn. Thus b −1 b+2
m−n n−m n(m − n) − m(n − m) − mn 3a(b + 1) a(b + 1) 3b(a + 1)
− − 1= A B C
m n mn (b − 1)(b + 2) (b − 1)(b + 2) (b − 1)(b + 2)
mn − n 2 − mn + m 2 − mn ab (b + 1) 3b(a − 1)
= D E
mn (b − 1)(b + 2) (b − 1)(b + 2)
m − n − mn
2 2
Solution
= D The LCM of denominators b – 1 and b + 2 is
mn
(b – 1)(b + 2). Thus
40
2a a 2a(b + 2) + a(b − 1) Solution
+ = The LCM of x and y is xy. Thus
b −1 b+2 (b − 1)(b + 2) x+ y x− y y ( x + y ) − x( x − y )
2ab + 4a + ab − a − =
= x y xy
(b − 1)(b + 2)
xy + y 2 − x 2 + xy
3ab + 3a =
= xy
(b − 1)(b + 2)
3a(b + 1) 2 xy + y 2 − x 2
= A = C
(b − 1)(b + 2) xy
2013/41 2005/39 ( Old) Exercise 3.40
2 1 3 2
Express − as a simple fraction Simplify −
x+3 x−2 x+2 2x − 3
x−7 x −1 x−2 4x − 1 4x − 5
A 2 B 2 C 2 A B
x + x−6 x + x−6 x + x−6 ( x − 2)(2 x − 3) ( x + 2)(2 x − 3)
x+7 4 x − 11 4 x − 13
D 2 C D
x + x−6 ( x + 2)(2 x − 3) ( x + 2)(2 x − 3)
Solution 2006/4 Exercise 3.41
The LCM of denominators x + 3 and x – 2 is x−4 x−3
(x + 3)(x – 2). Thus Simplify: −
2 1 4 6
2( x − 2) − 1( x + 3)
− = x − 18 x−6 x − 18 x−6
x+3 x−2 ( x + 3)( x − 2) A B C D
12 12 24 24
2x − 4 − x − 3
= 2010/23 Exercise 3.42
x ( x − 2) + 3( x − 2)
2 2
x − 7 Simplify : −
= 2+ x 2− x
x − 2 x + 3x − 6
2
4 8 4x 8 − 4x
x−7 A B C– D
= 2 A 4− x 2
4− x 2
4 − x2 4 − x2
x + x−6
2014/36 2010/24 Neco Exercise 3.43
2 3 p 2 1
If − is equal to , find P Write − as a single fraction
x−3 x−2 ( x − 3)( x − 2) 2+ x 2− x
A – x – 5 B – (x + 3) C 5x – 13 D 5 – x 6 − 3x 4 − 3x 2 − 3x
Solution A B C
(2 − x) 2
2− x 2
(4 − x) 2
The LCM of denominators x – 3 and x – 2 is
(x – 3) (x – 2). Thus 2 − 3x 2 − 3x
D E
2 3 2( x − 2) − 3( x − 3) 4− x 2
4 + x2
− =
x−3 x−2 ( x − 3)( x − 2) 2009/7 Neco (Dec) Exercise 3.44
2 x − 4 − 3x + 9 1 2
= Simplify − 2
( x − 3)( x − 2) x −1 x −1
5− x 1 1 −1 1 1
= A B C 2 D E
( x − 3)( x − 2) x −1 x +1 x −1 x2 −1 x2 +1
Thus P is 5 – x D 2012/34 Exercise 3.45
2013/19 Neco x− y
x+ y x− y Express 3 −   as a single fraction
Simplify −  y 
x y 3 xy x − 4y 4y + x 4y − x
A B C D
2 xy + y + x
2 2
2 xy − y + x
2 2
y y y y
A B
xy xy 2011/16 Neco Exercise 3.46
2 xy + y − x
2 2
xy + y + x
2 2 5 6
C D Simplify −
xy xy x −1 x−2
− ( x + 4) ( x + 4)
E x y+ y −x
2 2 2
A 2 B 2
xy x − 3x + 2 x − 3x + 2
41
( x + 4) ( x − 4) ( x − 4) p+ q 3u + 1
C D E 2 Thus =  −1
x − 3x + 2
2
x − 3x + 2
2
x − 3x + 2 p−q 1−u
2014/30 Neco Exercise 3.47
3u + 1 3u + 1
= =
1 2 (1 − u )  − 1 u −1
Simplify −
x −1 x+2 2008/2b
−x x x2 − y2
A B Given that P = 2
( x − 1)( x + 2) ( x − 1)( x + 2) x + xy
4− x 4+ x x−4 ( i ) Express P in its simplest form;
C D E ( ii ) Find the value of P if x = – 4 and y = – 6
( x − 1)( x + 2) ( x − 1)( x + 2) ( x − 1)( x + 2)
Solution
2015/51 Neco Exercise 3.48 x2 − y2 ( x + y )( x − y )
3x + 2 x − 1 5 (i) =
Write – – as a single fraction x 2 + xy x( x + y )
4 4 12 x−y
3x + 2 x − 1 x − 1 =
A. B. C. x
4 3 5 ( ii ) substituting for x and y
3x + 2 3x + 2 x−y − 4 − (−6)
D. E. =
6 12 x −4
2014/8 NABTEB (Nov) Exercise 3.49 −4 + 6
=
a 2a a −4
Simplify + –
4 3 12 2 1
= = −
2a a 5a a −4 2
A. B. C. D.
3 4 6 12
2015/1a Neco
Substitution in algebraic expressions If the numbers x, y, z are in the ratio 6 : 5 : 8,
There are two types of substitutions in algebraic 12 x − 9 z
find
expressions: 4y + z
(a) When the value to be substituted is given Solution
(b) Solving for the value to be substituted under x = 6, y = 5, z = 8
given conditions Substituting
12 x − 9 z 12(6) − 9(8)
(a) When the value to be substituted is given =
4y + z 4(5) + 8
2006/4b 72 − 72 0
2u 1+ u = = =0
If p = and q = , 20 + 8 8
1−u 1−u 2007/11
p+ q If x = 3, y = 2 and z = 4, what is the value of 3x2 – 2y + z?
express in terms of u A. 17 B. 27 C. 35 D. 71
p−q
Solution
Solution 3x2 – 2y + z = 3(32) – 2(2) + 4
2u 1+ u = 3×9– 4 + 4
p+q = +
1−u 1−u = 27 B
2u + 1 + u 3u + 1
= = 2006/6a Neco
1−u 1−u 3m − 2 x +1
If x = , express in terms of m
2u 1+ u m −1 2x − 1
p– q = – Solution
1−u 1−u
x +1
2u − (1 + u ) 2u − 1 − u = (x +1)  (2x – 1)
= = 2x − 1
1−u 1−u Substituting x
u −1 − (1 − u )  3m − 2    3m − 2  
= = = –1 =  + 1  2  − 1
1−u 1−u  m −1    m −1  

42
 3m − 2 + m − 1   6m − 4  2005/ 47
=   − 1 2x − 2
 m −1   m −1  For what value of x is the expression not
( x − 2)( x + 3)
 4m − 3   6m − 4 − 1(m − 1) 
=   defined?
 m − 1   m −1 
A x = 4 or – 2 B x = 2 or – 3
 4m − 3   6 m − 4 − m + 1  C x = 3 or – 2 D x = 2 or – 4
=  
 m − 1   
m −1 Solution
The expression cannot reduce further
 4m − 3   5m − 3  So for it to be undefined
=   
 m −1   m −1  (x – 2)(x + 3) = 0
Changing  to  x – 2 = 0 or x + 3 = 0
x = 2 or –3 B
4m − 3 m − 1 4m − 3
=  = 2014/29 Neco
m − 1 5m − 3 5m − 3
Find the value of x for which the fraction 2 x 2 + y is
2 2

2015/38 Neco Exercise 3.50 x −4


a + b
3 undefined.
Evaluate   for a = -7 , b = 3 Solution
a − b Ax=5 Bx=4 Cx=3 Dx=2 E x=1
Solution
A. − 125
8
B. − 104 C. 8
125 D. 16
125 E. 2
5 It is clear that the expression will not reduce further hence
we impose the undefined condition
2008/11 Exercise 3.51
x2 – 4 = 0
x 2 y − 2 xy x2 – 22 = 0
Given that x = 2 and y = − 1
4 , evaluate
5 (x – 2)(x + 2) = 0
A. 0 B. 15 C. 1 D. 2 x – 2 = 0 or x + 2 = 0
x = 2 or – 2 D
2006/17 Neco (Dec) Exercise 3.51b

Evaluate x 2 + x − 2 when x = –1
2 2006/5
2x + x − 3 2x
Given that y = 1 – , find the value of x for which y
A2 B1 C – /2 1
D–1 E–2 4x − 3
2014/6b Neco ( Nov) Exercise 3.52 is undefined
3x − 2 2z + 3 A3 B 3/4 C – 3/4 D–3
Given that Z = , express in terms of x Solution
2x + 3 3z − 2
For any expression of the form a ( x) to be undefined
b( x )
b(x) = 0; here the expression is not yet in full fraction a ( x) ,
(b) Solving for the value to be substituted b( x )
under the conditions: making it full fraction
(i) undefined expressions 2x 4x − 3 − 2x
1 – =
(ii) expression equals zero 4x − 3 4x − 3
2x − 3
Undefined expressions =
a( x)
4x − 3
For any expression of the form to be undefined Since the expression cannot reduce further: we apply the
b( x ) undefined condition
then b (x) = 0 4x – 3 = 0
4x = 3
2005/33 Neco x = 3/4 B
x +1
Find the value of x for which the fraction is 2006/20
2x − 1 cr − px
undefined. Given that y = , then the value of y is undefined if
aq − bp
A–2 B – 1/2 C0 D 1/ 2 E 2
Solution A cr = px B cr > px C aq = bp D aq< bp
The expression cannot reduce further E aq > bp
x +1 Solution
is undefined if 2x – 1 = 0 Since the given expression cannot reduce further. It is
2x − 1 undefined if
2x = 1 aq – bp = 0
x = 1/ 2 D aq = bp C
43
2012/1a Neco Good case (ii) Expression equals zero
Find the value of x for which the fraction For any algebraic expression a (x) = 0 Then a(x) = 0
x −9 2
b (x)
is undefined. Examples
2x − 7x + 3
2

Solution (1) For what value (s) of x is the expression


First, we simplify the expression to see whether it will x+3 equals zero?
reduce further x2 + 10x – 25
Solution
x2 − 9 x 2 − 32
= Since the numerator cannot be factorised further, we impose
2x 2 − 7x + 3 2x 2 − 6x − x + 3 the condition
( x − 3) ( x + 3 x+3 = 0
=
2 x( x − 3) − 1( x − 3) x2 + 10x – 25
( x − 3) ( x + 3) if x + 3 = 0
= x = –3
(2 x − 1) ( x − 3)
x+3 (2) For what value (s) of x is the expression
= x2 – 2x – 15 equals zero?
2x − 1
x2 – 25
Next, we impose the undefined condition First, we factorize the numerator x2 – 2x – 15
2x – 1 = 0 = x2 + 3x – 5x – 15
2x = 1 thus x = 1/2 = x (x +3) – 5(x + 3)
= (x +3 )(x –5 ) Thus
2007/ 10 Exercise 3.53 x2 – 2x – 15 = (x +3)(x – 5)
2x − 1 x2 – 25 (x –5)(x +5)
For what value of x is the expression not
x+3 = x+3
defined? x+5
A3 B2 C 1/2 D–3 Imposing the given condition on the reduced form
x+3 = 0
2008/21 Neco (Dec) Exercise 3.54 x = -3
2x + 5
Find the values of x for which is not defined. 2006/11a Neco ( Dec)
4x 2 − 9 Find the value of x for which the expression
A x = 3/2 or – 3/2 B x = 2/3 or – 2/3 C x = 2/5 or – 2/5 2 x 2 + 9 x − 11
D x = 5/2 or 3/2 E x = 5/2 or – 3/2 is zero
6 − 4x
2009/10 Neco (Dec) Exercise 3.55 Solution
2x + 1 For any algebraic expression a( x) = 0 then a(x) = 0
For what value of x is undefined? b( x )
12 − 5 x − 3x 2 First, we simplify the expression to see whether it will
A x = 5 or 1 B x = 3 or – 3/4 C x = – 3 or 4/3 reduce
D x = –5 or 1 E x = 5 or 3/4 2 x 2 + 9 x − 11 2 x 2 + 11x − 2 x − 11
=
2012/24 Neco Exercise 3.56 6 − 4x 6 − 4x
x2 +1 x(2 x + 11) − 1 (2 x + 11)
For what values of x is not defined? =
x2 −1 6 − 4x
A x = –1 or 1 B x = 1/2 or 0 C x = 1/2 or 2 ( x − 1)(2 x + 11) ( x − 1)(2 x + 11)
D x = – 1/2 or 2 E x = 0 or –2 = =
6 − 4x 2(3 − 2 x)
It can not reduce further; hence we impose the condition of equals zero
2x2 + 9x – 11 = 0
i.e (x – 1) (2x + 11) = 0
x – 1 = 0 or 2x + 11 = 0
x = 1 or 2x = – 11
x = 1 or – 11/2

2014/44 Neco ( Nov) Exercise 3.57


x 2 − 9 x + 14
For what values of x is the fraction equals to
x2 − x − 2
zero?
A. 2 or -1 B. -2 or -7 C. 2 or -1 D. -2 or 1 E. 2 or 7

44
Change of subject of the Formula 2005/ 2a
The term change of subject of the formula implies t−3
transformation of a given formula to make another ( i ) Make t the subject of relation k = m
quantity (usually a letter) the subject of the equation. t−9
This is other than the one in that position. ( ii ) Find the value of t, if k = 2 and m = 1
In doing it, basic mathematical rules must not be Solution
violated. t −3
k = m
Some Basic Guides t−9
(a) Remove square or square root if present Since T is under square root, we take square of both sides
(b) Clear fractions, clear bracket if they exist t − 3
(c) Collect like terms together involving the required subject. k2 = m2  
(d) If necessary factorize the term on each side of t − 9
the equation m 2 t − 3m 2
(e) Isolate the required subject usually by division k2 =
by a common factor t −9
Note Cross multiply to clear fraction
The above stated guides are no strict rules to be k2(t – 9) = m2t – 3m2
followed when solving any problem under this topic; Collect terms in t together
rather, they are guides which readers will find useful k2t – m2t = 9k2 – 3m2
t(k2 – m2) = 9k2 – 3m2
Cases involving indices(powers) 9k 2 − 3m 2
2005/4 (Nov) t =
k 2 − m2
k 2 + x2 ( ii ) If k = 2 and m = 1 then
Make x the subject of the relation y =
t 9(2 2 ) − 3(12 )
t =
A x = yt − k 2 B x = ( yt ) 2 − k 2 2 2 −12
C x = (yt)2 – k2 D x = (yt – k)2 36 − 3 33
= = = 11
Solution 4 −1 3
k 2 + x2 2005/34 Neco
y =
t 1 1 C2 
If A = B +   , make C the subject of the formula
First we clear fraction 2 8  B 
yt = k + x
2 2
A  2 2BCA − B B  2 2(2 A − B
Since x is under square root, we take square of both
sides. C  2 B( 2 A − B ) D  2 2 B( 2 A − B) E  2 B( 2 A + B )
( yt )2 = k2 + x2 Solution
Next, we isolate x 1 1 C2 
( yt )2 – k2 = x2 A = B +  
x is squared , so we take square root 2 8  B 
First, we clear fraction, LCM of 8 and 2 is 8, so we multiply through by 8
( yt ) 2 − k 2 = x B
C2
8A = 4B +
2005/1c Neco B
If 4a3 = c – b2, find the value of b Isolate term in C
when a = – 3 and c = 24 (correct to 2 d.p) C2
8A – 4B =
Solution B
4a3 = c – b2 B(8A – 4B) = C2
First we make b2 the subject of the formula 8AB – 4B2 = C 2
b2 = c – 4a 3 Since C is squared, we take square root
b is under square, we take square root of both sides
 8 AB − 4B 2 = C
b = c − 4a
3

Substituting for the given values of a and c


 4 B( 2 A − B = C
= 24 − 4(−3) 3 i.e  2 B(2 A − B) = C

= 24 − 4(−27)
= 24 + 108
= 132 = 11.489  11.49 to 2 dp.
45
2006/6a (Nov) 2014/12b ( Nov )
2
 r  u
( i ) Make r the subject of the relation Q = P 1 + Given the relation T =
 100  1
f + 1
g
( ii ) Find, correct to 3 s.f, the value of r when ( i ) make g the subject of the relation
Q = 625 and P = 225
( ii ) find g when T = 3, f = 4 and u = 5
Solution
2
Solution
 r 
Q = P 1 + u
100    T = 1
+ 1
f g
2
Q  r 
Since g is under square root, we take square of both sides
= 1 + 
P  100  u
Since r’s bracket is squared, we take square root to free it T2 =
1
f + 1
g
Q r Cross multiply to clear fraction
= 1 +
P 100 T2 ( 1
f + 1
g
)= u
Q r Next we isolate g,
−1 =
P 100 1 1 u
+ =
Multiply through by 100 to clear fraction f g T2
 Q  1 u 1
100  −1 = r = –
 P  g T2 f
Apply LCM of fraction to the RHS
 625 
r = 100  −1 1 fu − T 2
 225  =
g T2 f
 25  Just invert both sides
= 100  −1
 15  T2 f
g =
 10  fu − T 2
= 100   = 66.666  66.7 to 3 s.f
 15  ( ii ) Substituting for the given values
2014/7b Neco ( Dec ) 32  4
g =
 A  n 
1
4  5 − 32
Given the formula 100   − 1 = r
94
 P   = = 36 = 3 311
( i ) Make A the subject of the formula 20 − 9 11
( ii ) Find the value of A when P = 7808, n = 3 and r = 25 2008/34
1
Solution
If P =  Q( R − T )  , make T the subject of the relation
3
 
1

100  A  − 1 = r  15 
 
n

 P   R + p3 R − 15 p 3
1
A T= B T=
 An r 15Q Q
  – 1 =
15 p 3 15R − Q
P 100 C T=R– D T=
1 Q P3
 An r
  = + 1 Solution
P 100 1

Since A is under root n, we take power of n of both sides  Q( R − T )  3


P =  
A  r 
n
 15 
=  + 1 Since T’s bracket is cube – root, we raise both sides to power 3
P  100 
Q(R − T )
 r 
n P3 =
A = P 1 +  15
 100  Cross multiply to clear fraction
( ii ) Substituting for the given values 15P3 = Q(R – T)
3 15P3 = QR – QT
 25  Next, isolate term in t
A = 7808 1 + 
 100  QT = QR – 15P3
= 7808( 1 + 0.25)3 QR − 15P 3 15 P 3
T = i.e R – (C)
= 15250 Q Q
46
2013 /3b Neco 2007/18 Neco Exercise 3.58
1
x n
 2p + q  2 If z = − , make y the subject of the formula.
Given that h =   , express p m y
 p − 3q 
in terms of q and h x − z 2m z2 mn
Ay= By=x– Cy=
Solution mn mn x − z 2m
1
mn z2
 2p + q 2 Dy= E y= – x
h =   , nz 2 − x mn
 p − 3q 
Since p is under square root, we take square of both sides 2007/6b Exercise 3.59
2p + q m n2
h2 = If P = − ,
p − 3q 2 5m
Next, we clear fraction ( i ) Make n the subject of the relation;
h2(p – 3q) = 2p + q ( ii ) Find, correct to 3s.f, the value of n when P = 14 and m = – 8
h2p – 3h2q = 2p + q
Collect terms in p together 2010/29 Exercise 3.60
h2p – 2p = 3h2q + q
p(h2 – 2) = 3h2q + q 2( x 2 + m )
If y = , make x the subject of the formula
q(3h 2 + 1) 3N
p =
h2 − 2 9 y 2 N 2 − 2m 9 y 2 N 2 + 2m
A B
2006/12 Neco (Dec) 2 2
If v2 = u2 + 2as, what is/are the value(s) of u 9 y N − 4m
2 2
9 y N 2 + 4m
2

when v = 7, a = 10 and s = 2? C D
A–3 B3 C 3 D– 9 E 9
2 2
Solution
2014/14 Exercise 3.61
First, we make u the subject of the formula
v2 = u2 + 2as m 2
Make U the subject of the formula E = (v − u 2 )
v – 2as = u2
2 2g
Since u is squared, we take square root of both sides 2 Eg v2 2 Eg
v − 2as = u
2 A u= v2 − B u = −
m m 4
Substituting
2 Eg 2v 2 Eg
u = 7 − 2  10  2
2 Cu = v − D u =
m m
= 49 − 40
2015/34 Exercise 3.62
= 9 i.e  3 (C)
Tk − H
Make k the subject of the relation T =
2006/6 Neco (Dec) k − H
a2  b2 − c H (T 2 − 1) HT
If S = , find s when A. k = B. k =
2a T2 − T ( T −1) 2
a = 2, b = – 2 and c = – 5
H (T 2 + 1) H ( T − 1)
A 7 or 1 B 7 or − 1 C. k = D. k =
4 4 4 4 T T
C − 7 or − 1 − 7 or − 4 E – 7 or – 4
4 4 D 4 2012/23 Neco Exercise 3.63
t
Solution  r 
Substituting for the given values Make r the subject of the relation V = P1 + 
 100 
2 2  (−2) 2 − (−5)
S =  v  v v
2 2 A 100  t − 1  Bt − 1 C t
 p  p p
4 4+5
=  v 
4 D t
v
+2 E 100  t −1
4 9 43 p  p 
= = = 7 or 1 ( A )
4 4 4 4

47
Cases involving non - indices ( No powers) 2005/6b(Nov)
1 1 1
2013/17 Neco If = + and b + c – k = 0, express a in terms of b and k only
Make t the subject of the formula p = (t – 1)ax – at a b c
p − ax p + ax p + at Solution
A B C p + ax D
ax − a ax − a at We are to eliminate c form 1 = 1 + 1
E pxa + t a b c
Solution From b + c – k = 0, we make c the subject formula
p = (t – 1)ax – at c = k–b
Open up bracket containing t 1 1 1
Thus = + becomes
p = axt – ax – at a b c
Collect terms in t together 1 1 1
p + ax = axt – at = +
Factor out t at RHS a b k −b
p + ax = t(ax – a) Next, we make a the subject of the formula,
p + ax clear fractions on RHS
= t B 1 k −b+b
ax − a =
a b ( k − b)
2001/1b (Nov) 1 k
2d − 3c m =
If = , express d in term of c, m and n a b ( k − b)
5d + c n b(k – b) = ak
Solution b( k − b)
We are asked to make d the subject of the formula. = a
Cross multiply to clear fractions k
n(2d – 3c) = m(5d + c)
Opening brackets, we have 2009/8 Neco (Dec)
2dn – 3cn = 5dm + cm r p r
Make r the formula in the expression + =
Collect terms in d together y x x
2dn – 5dm = cm + 3cn yp yp p
Factor out d A r= B r= C r=
d(2n – 5m) = cm + 3cn x− y y−x y ( y − x)
cm + 3cn xy x+ y
D r= E r=
d = p p
2n − 5m
Solution
2005/9 r p r
1 1 1 + =
If = + , p = 2 and q = 4 find the value of r y x x
p q r 5 7
Collect terms in r together
4 3 4 17 p r r
A B C D = −
17 4 3 4 x x y
Solution Applying LCM method to RHS
In indices, we are told that 1 1 = a i.e 1  1/a = 1 
a
1 = p ry − rx
a =
a x xy
Thus substituting for the given values Isolate terms in r
1 1 1 pxy
= + becomes = ry − rx
p q r x
5 7 1 5 4 1 5 7 1 py = r(y – x)
= + and not = + not = +   yp
2 4 r 2 7 r 2 4 r = r B
No substituting in r y−x
5 7 1
− = 2006/46(Nov)
2 4 r
x+ y
10 − 7 1 Given that p = , make y the subject of the relation
= x− y
4 r
3 1 x( p − 1) x( p + 1) x+P p−x
= A B C D
4 r p +1 p −1 x −1 p +1
4
Thus 3r = 4 and r = /3 (C)

48
Solution ( b ) Substituting for the given values
x+ y 21 − 21(−3) − 7(4)(−2)
p = A =
x− y 5 4
First we clear fractions 21 + 63 + 56 140
p(x – y) = x + y A = = =7
px – py = x + y
20 20
Collect terms in y together 2011/50
px – x = y + py MN
Factor y in the RHS If E = and E = 75, M = 120, N = 5000, find S
S+N
px – x = y(1 + p)
A 1000 B 2000 C 3000 D 4000
px − x x( p − 1)
= y i.e =y Solution
1+ p 1+ p First, we make S subject of the formula
2007/ 42 MN
a + bc E = becomes
Make W the subject of the relation =g S+N
wd + f E(S + N) = MN
A a + bc − fg B a − bc + fg C a + bc − f D ES + EN = MN
dg dg dg ES = MN – EN
a + bc − dg MN − EN
S=
fg E
Solution N (M − E )
a + bc =
=g E
wd + f Substituting for the given values
Cross multiply to clear fraction 5000(120 − 75)
g(wd + f) = a + bc =
75
Opening bracket, we have
gwd + gf = a + bc 5000  45
= = 3000 C
Isolate term in w 75
gwd = a + bc – gf
w = a + bc − fg A 2008/19 Neco (Dec)
dg V T
Make S the subject of the formula =
2006/2 Neco (Dec) S +T S
Make ‘A’ the subject of the equation T2 T T
A S= B S= C S=
x 5  V +T V −T V +T
(a) y = 1 −  B + A
3 7  T2 VT 2
D S= E S=
(b)Hence, find the value of A if y = –3, x = 4 and B= –2 V −T V +T
Solution Solution
x 5  V T
(a) y = 1 − B + A =
3 7  S +T S
Isolate term in A Cross multiply to clear fractions
x 5  VS = T(S + T )
 B + A = 1 – y VS = TS + T2
3 7 
Collect terms in S together
Multiply through by 3 to clear fraction outside the bracket VS – TS = T2
 5  S(V – T) = T2
x B + A  = 3(1 − y )
 7  T2
S = (D)
5 V −T
xB + Ax = 3 – 3y
7
Isolate term in A 2008/21 Neco Exercise 3.64
5 If A = 12 h(a + b), make ‘a’ the subject of the formula
Ax = 3 – 3y – xB
7 2 A − 2bh 2 A − bh 2 A − bh
A B C
Multiply through by 7 to clear fraction b h b
5Ax = 21 – 21y – 7xB A − 2bh 2A − h
21 − 21y − 7 xB D E
A = h bh
5x
49
2010/29 Neco Exercise 3.65 CHAPTER FOUR
p−q Equations
Make p the subject of the formula h =
pt − qr Simultaneous linear equations
Simultaneous linear equation as the name implies, is a pair of
hr − q h(qr − 1) q(hr − 1)
A B C linear equation (highest degree of the unknown is one)
ht − 1 ht − r ht − 1 occurring together. Though, in this case there are two different
q(hr − h) q ( r − h) unknowns unlike in the simple equation, which has only one
D E unknown. Unless otherwise stated, we can apply either
rt − h ht + r
elimination or substitution method in solving any given
2011/21 Neco Exercise 3.66 simultaneous linear equation. Though a third method of
a b a solving exists; this is graphical solution. It is left for
An expression is given as + = discussion in a later chapter.
x y y
Make a the subject of the formula Uniform coefficient problems
bx x bx Examples
A B C (1) Solve the equations
y ( y − x) y − bx x− y x + 2y = 5 and x + 3y = 8
x bx Solution
D E
bx − y y−x Applying substitution method
2012/11 Exercise 3.67 x + 2y = 5 ---- (1)
x + 3y = 8 --- (2)
3p s
Make p the subject of the relation : q = + From (1) x = 5 – 2y
r 2 Substitute x value into (2) and not into (1)
2q − rs x + 3y = 8 will become
A p= B p = 2qr – sr – 3 (5 – 2y) + 3y = 8
6
5 – 2y + 3y = 8
2qr − s 2qr − rs 3y – 2y = 8 –5
C p= D p=
6 6 y=3
2014/51 Neco Exercise 3.68 Substitute y value into (1)
2uv − r x + 2y = 5 will become
Make u the subject of the formula S = x + 2(3) = 5
u−2 x+6 =5
2s − r 2s + r 2s + 1 x=5–6
A u= B u= C u=
s − 2v s + 2v s − 2v x = -1
s + 2v s − 2v Solution set is x = -1 and y =3
D u= E u=
2s − r 2s − r Applying elimination method to the same problem
x + 3y = 8 ------- (1)
2015/26 Neco Exercise 3.69 -(x + 2y = 5) ------- (2)
1 1 1 y=3
If = + , make v the subject of the formula i.e equation (1) - (2)
f u v The equations are arranged to place the one with bigger values on
fu u top to enable us subtract the smaller one from it.
A. v = B. v = Substitute y value into (1)
f −u f −u
x + 3y = 8 will become
fu + fv u − f fu x + 3 (3) = 8
C. v = D. v = E v =
u fu u− f x + 9 =8
2015/11a Exercise 3.70 x=8–9
Make m the subject of the relation x = -1
mt Solution set is x = -1 and y =3
h = Note: Students are to use any of the methods and not
d( m + p )
the two at the same time on a particular question
2014/42 ( Nov) Exercise 3.71
1 1 2014/29 NABTEB
Given that = t – , make n the subject of the Solve for x and y, if x + y = 3 and 3x – y = 1
m n A 1, 2 B 2, 1 C 3, 1 D 1, 3
formula Solution
t −1 m x + y = 3
A. n = B. n =
m t −1 + ( 3x – y = 1 )
mt − 1 m 4x = 4
C. n = D. n = x = 1
m mt − 1
50
Substitute x = 1 into x + y = 3 2011/14 Neco
1 + y = 3 Solve the simultaneous equations
y = 3 – 1 3x – y = 7 and 2x = 3 – y
y = 2 A (–1, 2) B (– 2, – 1) C (– 2, 1)
x, y i.e 1, 2 A D (2, 1) E (2, – 1)
Solution
2011/7 2x + y = 3
Solve the equations: 3x – 2y = 7 + ( 3x – y = 7 )
x + 2y = –3 5x = 10
A x = 1, y = – 2 B x = 1, y = 3 x = 2
C x = –2, y = – 1 D x = 4, y = – 3 Substitute for x = 2 into 2x + y = 3
Solution 2(2) + y = 3
3x – 2y = 7 4+y = 3
+ ( x + 2y = –3 ) y = 3–4
4x = 4 y = –1
x = 1 Solution in coordinate form (x, y) is (2, –1) E
Substitute x = 1 into 3x – 2y = 7
3(1) – 2y = 7 2007/20 Neco Exercise 4.1
3 – 2y = 7 Find the valve of x – y, if 2x + 2y = 16 and 8x – 2y = 44
– 2y = 7 – 3 A 2 B 4 C 6 D 8 E 10
–2y = 4 2014/24 NABTEB (Nov) Exercise 4.2
y = –2 Find the value of 2x – y. If x + y = 8 and 4x – y = 22
Thus x = 1, y = – 2 (A) A2 B4 C6 D 10
2014/8 ( Nov ) Exercise 4.3
2013/13 If 3x – y = 5 and 2x + y = 15 , evaluate x2 + 2y
If 12 x + 2 y = 3 and 32 x + 2 y = 1 , A 29 B 30 C 35 D 42
find (x + y) Exercise 4.4
A3 B2 C1 D0 Find the value of x – y, If 2x + y = 8 and 4x – y = 22
Solution A2 B5 C7 D 3
First, we solve for x and y
1
2 x + 2y = 3
–( 3
2 x + 2y = 1) Non - uniform coefficient problems
Examples
–x = 2
x = –2 (1) Solve the equations
Substitute for x = –2 into 1
x + 2y = 3 3p + 2q = 7 and 4p + 3q = 10
2
Solution
1
(–2) + 2y = 3
2 Applying substitution method
–1 + 2y = 3 4p + 3q = 10 ----------(1)
2y = 3 + 1 3p + 2q = 7 -----------(2)
2y = 4 from (2) 3p = 7 – 2q
y=2 p = 7 – 2q
Thus x + y = –2 + 2 i.e 0 ( D ) 3
Substitute p value into (1) not (2)
2006/28 4p + 3q = 10
If x + y = 12 and 3x – y = 20, find the value of 4 ( 7 – 2q ) + 3q = 10
2x – y 3
A8 B 10 C 12 D 15 ( 28 – 8q ) + 3q = 10
Solution 3
x + y = 12 Multiply through by 3 to clear fractions.
+ ( 3x – y = 20 ) 28 – 8q + 9q = 30
4x = 32 9q – 8q = 30 – 28
x = 8 q=2
Substitute for x = 8 into x + y = 12 Substitute q value into equation (2)
8 + y = 12 3p + 2q = 7 will become
y = 12 – 8 3p + 2 (2) = 7
y = 4 3p + 4 = 7
Thus, 2x – y = 2(8) – 4 3p = 7 – 4
= 16 – 4 3p = 3
= 12 ( C ) p = 1 (p, q ) format i.e (1, 2)
51
Applying Elimination method to the same problem. (2)  2 and subtract
4p + 3q = 10 -------- (1) 2x + 4y = 14
3p + 2q = 7--------- (2) – ( 2x + y = –1 )
To eliminate q , let us exchange their coefficients i.e 3y = 15
(2) x 3 and (1) x 2 y = 5
9p + 6q = 21 Substitute for y = 5 into (2)
- ( 8p + 6q = 20) x + 2y = 7 becomes
p = 1 x + 2(5) = 7
Substitute p value into (2) x + 10 = 7
3p + 2q = 7 will become x = 7 – 10
3(1) + 2q = 7 x = –3
3 + 2q = 7 Thus, x – y = – 3 – 5
2q = 7- 3 = –8 (D)
2q = 4
q=4 =2 2009/ 12 Neco (Dec)
2 Solve 5d = 2e – 14
Solution set in ordered pair means (x, y ) format i.e (1,2) 5e = d + 12
A e = – 2, d = –2 B e = – 2, d = 2
2009/9 (Nov) C e = 2, d = – 2 D e = 1, d = – 1 E e = –1, d = 1
Find the value of x in the simultaneous equations: Solution
2x + y = 4 ; Rearranging the equation, we have
x – 2y = 2 5d – 2e = – 14 ---- (1)
A6 B4 C3 D2 – d + 5e = 12 ------ (2)
Solution (2)  5 and add
2x + y = 4 ------ (1) 5d – 2e = – 14
x – 2y = 2 ------ (2) + ( –5d + 25e = 60 )
(1)  2 and add to eliminate y
4x + 2y = 8 23e = 46
+ ( x – 2y = 2 ) e = 2
Substitute for e = 2 into (2)
5x = 10 – d + 5e = 12 becomes
x = 2 (D) – d + 5(2) = 12
– d = 12 – 10
2007/12 –d = 2
Solve the simultaneous equations: d = – 2 Option C
x + y = 2 and 3x – 2y = 1
A x = 2, y = 1 B x = 1, y = 1 2006/29 Neco Exercise 4.5
C x = 1, y = 2 D x = –1, y = 1 Solve the simultaneous equations
Solution 2x = 10 – y
x + y = 2 ----- (1) 3x – 2y = 1
3x – 2y = 1 ----- (2) A x = 3, y = 4 B x = 4, y = 3
(1)  3 and subtract to eliminate x C x = 4, y = 1/2 D x = 5, y = –21 E x = 10, y = 1
3x + 3y = 6
– ( 3x – 2y = 1 ) 2010/17 NABTEB (Nov) Exercise 4.6
Solve for x and y in the equations
5y = 5 2x + 5y = 1
y = 1 3x – 2y = 30
Substitute for y = 1 into ( 1 ) A x = 7, y = – 2 B x = 8, y = – 3
x + y = 2 becomes C x = 6, y = 3 D x = 8, y = 3
x + 1 =2
x = 2–1 2005/21 Exercise 4.7
x = 1 Solve for y in the equations:
Thus x = 1, y = 1 ( B ) 2x – 3y = 22
3x + 2y = 7
2005/18 A–5 B–4 C4 D5
If 2x + y = –1 and x + 2y = 7, 2009/8 Exercise 4.8
find the value of x – y Solve the following simultaneous equations:
A8 B2 C–2 D–8 2x + 3y = 7
Solution x + 5y = 0
First, we solve for x and y A x = 5, y = – 1 B x = 1/5, y = –1
2x + y = –1 ----- (1) C x = – /5, y = 1 D x = – 5, y = 1
1

x + 2y = 7 ------ (2)
52
2006/9 (Nov) Exercise 4.9 Changing unknown, we have
Solve the simultaneous equations: 2p + 3q = 12 ----------- (a)
2x – 3y = 5 + (5p – 3q = 9) ------------(b)
4x + 5y = –1 7p = 21
A x = 4, y = 5 B x = 1, y = –1 p = 21/7 i.e p = 3
C x = – 1, y = –1 D x = 1, y = 0 Substitute p value into (a)
2007/7c Exercise 4.10 2p + 3q = 12 will become
Solve the equations: 2(3) + 3q = 12
3x – 2y = 21 6 + 3q = 12
4x + 5y = 5 3q = 12 – 6 i.e 6
q = 6/3 i.e q = 2
2015/37 Neco Exercise 4.11
If 2x + 3y = 19 and 8x = 11 + y, by how much is
p = 1 /y , q = 1 /x
7x greater than 2y? 3 = 1 /y , 2 = 1 /x
A1 B 22 C 32 D 42 E 52 y = 1 /3 , x = ½

Simultaneous linear equation in fractions. 2014/3a


Examples Solve the simultaneous equations:
1 1 1 1
(1) Solve the Simultaneous equations + = 5, − = 1
x - y =1 and x + y = 1 x y y x
2 6 3 2 5 Solution
Solution 1 1
Let p = and q =
x – y = 1 ------ (1) x y
2 6 p + q = 5 and q – p = 1
x + y = 1 ------ (2) Solving the resulting equations
3 2 5 p + q = 5
To clear fractions multiply through by their LCM +(– p + q = 1)
In equation (1) the LCM of the denominators 2,6 and 1
is 6, while the LCM of 3, 2 and 5 in (2) is 30 2q = 6
q = 3
Thus, (1) x 6 and (2) x 30 Substitute q = 3 into p + q = 5
3x – y = 6 (a) from ( 1 ) p + 3 = 5
10x + 15y = 6 (b) from ( 2 ) p = 5–3
Applying substitution method, we have from (a) p = 2
y = 3x – 6 Changing back to our original variables
Substitute y value into (b)
1 1
10x + 15y = 6 will become p = and q =
10x + 15 (3x – 6) = 6 x y
10x + 45x – 90 = 6 1 1
55x = 6 + 90 2 = and 3 =
55x = 96 x y
x = 96 2x = 1 and 3y = 1
55 =141/55 1 1
Substitute x value into (a) x = and y =
2 3
3x – y = 6 will become
3( 96 ) – y = 6 2009/9b Neco (Nov)
55 Solve the simultaneous equations
288 – y = 6 3 4 1 2 5
55 y = 288 – 6 i.e y = - 42 − = , − = 1
55 55 c d 3 c d
(2) Solve the equations 1 1
Hint: put x = and y =
2 + 3 = 12 and 5 - 3 = 9 c d
y x y x Solution
Solution Here the examiner gave us a guide to follow
We change the unknown to another, which will be 3 4 1 2 5
reconverted after solving. − = and − = 1 becomes
Let p = 1 and q = 1 then c d 3 c d
y x 1  1  1 1  1 
3  − 4  = and 2  − 5  = 1
2(1/y) + 3( 1/x) = 12 ----(1) c d  3 c d 
5(1/y) – 3(1/x) = 9 -----(2)

53
1 1 Note that 0.4 is the same as 4
10 i.e 2
5
Substituting for x = and y =
c d 2 1 
3x – 4y = 13 ------ (1)
x +  x = 3
5 2 
2x – 5y = 1 ------ (2) 1
x + 5 x = 3
Multiply (i) through by 3 to clear fraction
9x – 12y = 1 ---- (3) 6x = 3
Working with equations (2) and (3) 5
(3)  2, (2)  9 and subtract 6x = 15
18x – 24y = 2 x = 15/6 i.e 5
/2
– ( 18x – 45y = 9 ) 15 5
Next, y =   =
21y = – 7 22 4
y = – 7 −1 5 5 10 + 5 15
21 i.e 3 Thus x + y = + = i.e i.e 33/4 ( C )
Substitute for y = − 13 into ( 2 ) 2 4 4 4
2x – 5y = 1 becomes 2014/36 ( Nov) Exercise 4.12
2x – 5( − 13 ) = 1 1 1
If 2 p + q = 1 and p – 2 q = 7, find ( p + q )
5
2x + 3 = 1 A. -8 B. – 4 C. 4 D. 8
2x = 1 – 53 2015/43 Exercise 4.13
2x = – 2/3 1 1
Find the value of p if 4 p + 3q =10 and 2p – 3 q = 7
x = – 2/3  2 i.e – 2/3  1/2 = –1/3 A. 4 B. 3 C. – 3 D. – 4
Next, we change c and d into the original equation
x = 1/c i.e c = 1/x . Special cases
 − 1  −3 2012/12
= 1   i.e 1   Thus c = – 3
 3   1  If x + y = 2y – x + 1 = 5, find the value of x
A3 B2 C1 D–1
1 1
y= ,d = Thus d = – 3 Solution
d y x + y = 2y – x + 1 = 5
First, we rearrange the equation as
2006/3a x + y = 2y – x + 1 and 2y – x + 1 = 5
Solve for x and y in the following equation
9
Collect like terms together
2x – y = 2 x + x + y – 2y = 1 and 2y – x = 5 – 1
x + 4y = 0 2x – y = 1 and 2y – x = 4
Solution Solving the resulting equations
9
2x – y = 2 ---- (1) 2x – y = 1 -------- (1)
– x + 2y = 4 ------ (2)
x + 4y = 0 ------ (2)
From (2) x = – 4y, next substitute x = – 4y into (1) (1)  2 and add to eliminate y
9 4x – 2y = 2
2(– 4y) – y = 2 + (– x + 2y = 4 )
– 8y – y = 2
9 3x = 6
Multiply through by 2 to clear fraction x = 2 (B)
– 16y – 2y = 9 2005/35 Neco
– 18y = 9 Solve the equations:
y =
9
i.e – 1/2 x + y + 7 = 2x – 3y + 5 = 3x – 4y
−18
A x = 6, y = 1 B x = 1, y = 6
Next, x = – 4y becomes C x = –1, y = – 6 D x = – 1, y = 6 E x = – 6, y = 1
x = – 4(– 1/2) Solution
= 2
Presenting the problem properly
2012/33 x + y + 7 = 2x – 3y + 5 = 3x – 4y is the same as
1
If x + 0.4y = 3 and y = 2 x, find the value of (x + y) x + y + 7 = 2x – 3y + 5 and 2x – 3y + 5 = 3x – 4y
Collect like terms together
A 11/4 B 21/2 C 33/4 D5 x – 2x + y + 3y = 5 – 7 and 2x – 3x –3y + 4y = – 5
Solution
4y – x = – 2 and y – x = – 5
First, we find value of x and y
Solving the resulting simultaneous equations
x + 0.4y = 3 ----- (1)
1 4y – x = – 2
y = 2 x ------- (2) – ( y – x = –5 )
1
Substitute y = 2 x into (1) and work in fraction as shown in the 3y = 3
given options y = 1
54
Substitute for y = 1 into y – x = –5 6s + 9t = 93
1–x = –5 – ( 6s + 4t = 58 )
–x = –5 – 1 5t = 35
–x = –6 t = 7
Thus x = 6 and y = 1 ( A ) Substituting t value into (1)
2s + 3t = 31 becomes
2005/2b 2s + 3(7) = 31
The line y = 3x + 5 and y = – 4x – 1 intersect at 2s + 21 = 31
a point k. find the coordinates of k 2s = 31 – 21
Solution 2s = 10
y = 3x + 5 and y = – 4x – 1 s = 10/2 i.e 5
We rearrange the two equations and solve Yaw pay GH  5.00 for singlet and GH  7.00 for shirt.
y – 3x = 5
– ( y + 4x = –1 ) 2009/12
– 7x = 6 Five bottles of fanta and two packets of biscuits cost
x = – 6/7 GH  6.00. Three bottles of fanta and four packets of
Substitute x value into y – 3x = 5 biscuits cost GH  5.00. Find the cost of a bottle of fanta
y – 3(– 6/7) = 5 A GH  0.50 B GH  1.00 C GH  1.50 D GH  5.00
y + 187 = 5 Solution
18 5f + 2b = 6 --- (1)
y = 5– 7 3f + 4b = 5 ---- (2)
17
y = 7 (1)  2 and subtract
Coordinates of k are (– 6/7, 17
/ 7) 10f + 4b = 12
– ( 3f + 4b = 5 )
2008/8 Exercise 4.14 7f = 7
If p – 2g + 1 = g + 3p and p – 2 = 0, find g f = 1
A–2 B–1 C1 D2 Thus cost of a bottle of fanta is GH  1.00 ( B )

Word problems
Quadratic equation I
2005/ 39 (Nov) A quadratic function (equation) is noted by the highest power
The sum of two numbers is 31 while their positive of two among others. Its general form is
difference is 13. Find their product ax2 + bx + c = 0 where a ≠ 0.
A 99 B 117 C 198 D 286 If we allow a case where a = 0 then ax2 = 0 thus, reducing the
Solution equation to a linear one since the next power of x is one i.e bx.
Let the two number be x and y Other constants b, and c can take the value of zero.
x + y = 31 Examples
x – y = 13 (i) x2 + x + 10 = 0 ( a = 1,b = 1, C = 10)
Find xy, first we solve for x and y (ii) 7x2 – x - 13 = 0 (a = 7, b = -1 C = -13)
x + y = 31 (iii) 19x2 + 3 = 0 (a = 19, b = 0, c = 3)
+ ( x – y = 13 ) (iv) 14x – x2 = 0 (a = - 1, b = 14, c = 0)
2x = 44 (v) x2 = 0 ( a = 1, b = 0, c = 0)
x = 22
Substitute for x = 22 into x + y = 31 Comparison between quadratic and other relations
22 + y = 31 Examples
y = 31 – 22 (i) y = 2x + 1
y = 9 (ii) y = 5x3 + x2 – 16x + 4
Product xy = 22  9 i.e 198 ( C ) (iii) y = 19 – 2x – 3x2
Analysis
Example (1) has the highest degree of one thus; it will have
2009/ 2b (Nov) only one solution.
From a shop, Kofi bought 2 singlets and 3 shirts for Example (ii) has the highest degree of three thus; it will have
GH  31.00. While Kwasi bought 3singlets and 2 shirts three solutions.
for GH  29.00.How much will Yaw pay for one Example (iii) has the highest power of two thus, it will have
singlet and one shirt he bought from the same shop? two solutions. These solutions are called roots of a quadratic
Solution relation, they are also known as the zeros of the relation
2s + 3t = 31 ------- (1) because when such roots are substituted, the whole equation
3s + 2t = 29 ------- (2) becomes zero.
(1)  3 and (2)  2 then subtract
55
2012/21 Neco Solution
Which of the following is NOT a quadratic expression? Applying the formula:
A 3x2 – 2x B x(x – 3) C x2 – 5 x2 – (sum of roots) x + product of roots = 0
D 5x(x – 2) E 5(x – 1) 3 11
Solution Sum of roots: = − + 7 i.e
Expanding the unexpanded options i.e B, D and E
2 2
B. x(x – 3) = x2 – 3x 3 − 21
Product of roots = −  7 i.e
D. 5x(x – 2) = 5x2 – 10x 2 2
E. 5(x – 1) = 5x – 10 Substituting, we have
(E) has the highest power of 1; it is not a quadratic expression.
11  − 21 
x2 – x + = 0
2  2 
To derive a quadratic equation with known roots.
Given any two roots of a quadratic equation we can 11 21
x2 – x – = 0
derive the required equation 2 2
Examples Multiply through by 2 to clear fractions
1. If 2 and +3 are the roots of a quadratic equation, then 2x2 – 11x – 21 = 0 C
the equation is ?
Solution 2009/2a Neco
Let x be our variable (unknown) The roots of the equation ax2 + bx + c = 0 are
x = 2 or 3 x = – 2/3 and 3/2 . Find the value of a, b and c
x = 2 or x = 3 Solution
x – 2 = 0 or x – 3 = 0 Applying the formula:
(x – 2)(x – 3) = 0 x2 – (sum of roots)x + product of roots = 0
x(x – 3) – 2(x – 3 ) = 0
x2 – 3x – 2x + 6 = 0 Sum of roots = – 2/3 + 3/2 i.e −4 + 9 = 5/6
6
x2 – 5x + 6 = 0 Product of roots = – 2/3  3/2 i.e – 1
The method of solving e.g.1 can be generalized as : Substituting
let a and b be the roots of any quadratic equation. Then, x2 – 5/6x + (–1) = 0
x = a or x = b Multiply through by 6 to clear fraction
x – a = 0 or x – b = 0 6x2 – 5x – 6 = 0
(x – a ) (x – b ) = 0 Comparing the result with ax2 + bx + c = 0
x ( x – b) – a( x – b) = 0 a = 6, b = – 5 and c = – 6
x2 – bx – ax + ab = 0
x2 – (b + a) x + ab = 0
x2 – (a + b) x + ab = 0 ---------* 2010/45 NABTEB (Nov)
a + b implies sum of roots Given that x = 3/2 and x = – 6, construct a quadratic equation
ab implies product of roots for the above roots
we can restate * as A x2 + 12x – 9 = 0 B x2 + 12x – 18 = 0
x2 – ( sum of roots)x + product of roots = 0 C x – /2 x – 9 = 0
2 9
D 2x2 – 9x – 18 = 0
Students can always quote any of the two formulae Solution
above depending on ability to recall. Applying the formula: x2 – (a +b)x + ab = 0
a + b = 3/2 – 6 i.e –9/2 ; ab = – 3/2  (6) i.e – 9
2008/40 Substituting
Find the quadratic equation whose roots are c and – c x2 – (–9/2)x + (–9) = 0
A x2 + c2 = 0 B x2 – c2 = 0 x2 + 9/2x – 9 = 0
2 2
C x + 2cx + c = 0 D x2 – 2cx + c2 = 0 Multiply through by 2 to clear fraction
Solution 2x2 + 9x – 18 = 0 D
Applying the formula :
x2 – (sum of roots)x + product of roots = 0 2009/27 Neco (Dec)
Sum of roots: c + (– c) = 0 Find the quadratic equation whose roots are 1/2 and – 2/3
Product of roots: c  (– c) = – c2 A 6x2 – x + 2 = 0 B 6x2 – x – 2 = 0
Substituting, we have C 6x2 + x – 2 = 0 D 2x2 – 3x – 5 = 0
x2 – (0)x + (– c2) = 0 E 6x – 7x – 2 = 0
2

x2 – c2 = 0 B Solution
Applying the formula: x2 – (a +b)x + ab = 0
2008/27 Neco 1  2 1 2 3−4
Construct a quadratic equation whose roots are – 3/2 and 7.
a+b= +  −  i.e − = i.e –1/6
2  3 2 3 6
A 3x2 + 11x + 21 = 0 B 2x2 + 11x + 21 = 0
C 2x2 – 11x – 21 = 0 D 3x2 – 11x – 21 = 0 1  2
ab =   −  i.e – 1/3
E x2 – 11x – 21 = 0 2  3
56
Substituting 2014/45 Neco (Nov)
x2 – (– 1/6)x + (–1/3) = 0 Find the quadratic equation whose roots are – 3/2 and 1/4.
x2 + 1/6 x – 1/3 = 0 A 8x2 + 10x = 3 B 8x2 – 10x = 3
Multiply through by the LCM of 6 and 3 i.e 6 to clear C 8x + 10x = –3 D 8x2 – 10x = –3 E 8x2 + 10x = 5
2

fractions 6x2 + x – 2 = 0 C Solution


Applying the formula : x2 – (a + b)x + ab = 0
2009/20 Neco (Dec)
− 6 +1
Given that x = ½ and – ½ are the roots of the equation a + b = – 3/ 2  1/ 4 = i.e – 5/4 ; ab = – 3/2  1/4 = – 3/8
px2 + qx + r = 0, find p, q and r. 4
A p = 4, q = 0, r = –1 B p = 4,q = 1, r = 0 Substituting, we have
C p = 1, q = 4, r = 0 D p = 1, q = 0, r = – 1  5  3
x2 –  −  x +  −  = 0
E p = 1, q = 4, r = – 1  4  8
Solution
Applying the formula : x2 – (a +b)x + ab = 0 x2 + 5 x – 3 = 0
4 8
1  1 Multiply through by LCM of 4 and 8 i.e 8 to clear fractions
a+b = +  −  i.e 0
2  2 8x2 + 10x – 3 = 0
8x2 + 10x = 3 ( A)
1  1
ab =   −  i.e – 1/4 2010/45 Counter example
2  2 If the sum of the roots of the equation (x – p)(2x + 1) = 0
Substituting, we have is 2, find the value of P
x – ox + (– 1/4) = 0
2 A 1 1/ 2 B 1/ 2 C –1/2 D –11/2
x2 – 1/4 = 0 Solution
Multiply through by 4 First, we expand as
4x2 – 1 = 0 (x – p)(2x + 1) = 0
Comparing with px2 + qx + r = 0 x(2x + 1) – p(2x + 1) = 0
p = 4, q = 0, r = –1 A 2x2 + x – 2px – p = 0
2x2 + (1 – 2p)x – p = 0
2011/23 Neco Comparing this with the general formula sum and product of root
Find the quadratic equation whose roots are 1/2 and 2/3. x2 – (sum of root)x + product of roots = 0
A 6x2 + 7x – 2 = 0 B 6x2 + 7x + 2 = 0 The question’s statement implies that:
C 2 – 7x – 6x = 0
2
D 6x2 – 7x – 2 = 0 –(1 – 2p) = 2
E 6x – 7x + 2 = 0
2 –1 + 2p = 2
Solution 2p = 2 + 1
Applying the formula: x2 – (a +b)x + ab = 0 2p = 3
1 2 3+ 4 7 p = 3/2 Thus p = 11/2 A
a+b = +   i.e =
2 3 6 6 2009/34 (Nov) Exercise 4.15
1 2 Find the equation whose roots are – 3 and 5
ab =    i.e 1/3 A x2 + 2x – 15 = 0 B x2 – 2x – 15 = 0
2 3 C x + 8x – 15 = 0
2
D x2 – 8x – 15 = 0
Substituting, we have
2006/11 (Nov) Exercise 4.16
x2 – 7/6x + 1/3 = 0 Find the equation whose roots are –5 and –3
Multiply through by the LCM of 6 and 3 i.e 6 to clear fractions
6x2 – 7x + 2 = 0 E A x2 + 2x – 15 = 0 B x2 + 8x + 15 = 0
C x – 8x + 15 = 0
2
D x2 – 8x – 15 = 0
2014/42 (Nov) Neco
2014/41 NABTEB Exercise 4.17
Construct a quadratic equation whose are –1/2 and 21/2.
Find the equation whose roots are 2 and 1/2
A 4x2 + 8x + 5 = 0 B 4x2 + 8x – 5 = 0
A x2 – 5x + 2 = 0 B 2x2 – 5x – 2 = 0
C 4x – 8x – 5 = 0
2
D 4x2 – 8x + 5 = 0
C x – 2x + 5 = 0
2
D 2x2 – 5x + 2 = 0
E 4x2 – 8x = 0
Solution 2011/16 Exercise 4.18
Applying the formula: Form the equation whose roots are x = 1/2 and – 2/3
x2 – (sum of roots)x + product of roots = 0 A 6x2 – x + 2 = 0 B 6x2 – x – 2 = 0
Sum of roots = – 1/2 + 21/2 i.e 2
2
C 6x + x + 2 = 0 D 6x2 + x – 2 = 0
Product of roots = – 1/2  21/2 i.e – 1/2  5/2 = – 5/4 2005/24 Exercise 4.19
Substituting, we have Find the equation whose roots are – 1/2 and 3/2
x2 – 2x +  − 5  = 0 A 2x2 – 4x – 3 = 0 B 4x2 – 4x – 3 = 0
 4 C 4x – 6x – 3 = 0
2
D 2x2 – 6x – 3 = 0
x – 2x – 5 = 0
2 2005/5 Exercise 4.20
4 Find the equation whose roots are 2 and – 31/2
Multiply through by 4 to clear fraction A 2x2 + 3x + 14 = 0 B 2x2 + 5x + 7 = 0
4x2 – 8x – 5 = 0 C C 2x + 5x – 7 = 0
2
D 2x2 + 3x – 14 = 0
57
Methods for solving quadratic equations Example 3
Quadratic equation can be solved, (finding the roots) Solve d2 + 10d – 24 = 0
by the following methods: Solution
(i) Factorization c = -24 and b = 10
(ii) Completing the Squares Factors of – 24 Added values
(iii) Formula (i) -24 and 1 -23
(iv) Graphical (ii) + 8 and – 3 +5
We shall discuss the first three methods under this (iii) +3 and – 8 -5
chapter. Meanwhile, we will treat the graphical solution (iv) -12 and + 2 -10
under graph & charts. (v) 12 and – 2 +10
Factor (v) meets our need; we replace +10d with +12d – 2d
Factorization method d2 + 10d – 24 = 0 becomes
Our discussion shall be based on the general form of a d + 12d – 2d – 24 = 0
2
quadratic equation i.e ax2 + bx + c = 0. Thus when the d(d + 12) – 2(d +12) = 0
author mention “a” he is referring to the coefficient of (d + 12)(d – 2) = 0
x2 or any other variable as the case maybe. Similarly “b” d + 12 = 0 or d – 2 = 0
implies coefficient of x and “c” refers to the constant. d = –12 or 2

When a is unity (+1) 2010/21 NABTEB (Nov)


Examples Factorize the following equation x2 – 8x + 16 = 0
(i) x2 + 5x + 6 = 0 A x = 4, 4 B x = – 3, 4 C x = – 3, 4
(ii) k2 + 7k + 10 = 0 D x = – 4, – 4
(iii) d2 + 10d – 24 = 0 Solution
(iv) h2 + 8h + 16 = 0 x2 – 8x + 16 = 0
Quadratic problems like the ones stated above are Here coefficient of x2 is unity. C is 16 and b is –8
factorized and solved by finding two factors of c that Factors of c that when added give us b are – 4 and – 4
when added will give b x2 – 8x + 16 = 0 becomes
x – 4x – 4x + 16 = 0
2
Example 1. x(x – 4) – 4(x – 4) = 0
Solve by factorization : x2 + 5x + 6 = 0 (x – 4)(x – 4) = 0
Solution x – 4 = 0 or x – 4 = 0
C = + 6 and b = + 5 x = 4 twice ( A)
Factors of + 6 Added values
(i) + 6 and +1 +7 2011/ 39
(ii) + 2 and + 3 +5 Find the smaller value of x that satisfies the equation:
(iii) - 2 and – 3 -5 x2 + 7x + 10 = 0
Out of the stated factors of 6; only (ii) fits b which is+ 5 A–5 B–2 C2 D5
We now replace + 5x by 2x + 3x in the equation. Thus. Solution
x2 + 5x + 6 = 0 becomes x2 + 7x + 10 = 0
2
x + 2x + 3x + 6 = 0 Factors of c (10) that when added give b (+7) are 2 and 5.
x(x +2) + 3(x + 2) = 0 Next, we replace + 7x with + 2x + 5x
(x + 3)(x + 2 ) = 0 x2 + 7x + 10 = 0 becomes
x + 3 = 0 or x + 2 = 0 x2 + 2x + 5x + 10 = 0
x = – 3 or – 2 x(x + 2) + 5(x + 2) = 0
(x + 2)(x + 5) = 0
Example 2 x + 2 = 0 or x + 5 = 0
Solve k2 + 7k + 10 = 0 x = – 2 or – 5
Solution Smaller value of x is – 5 A
c = +10 and b = +7 Note with negative numbers the “so called” bigger numbers are smaller in
value than the smaller number, so – 100 is smaller than – 10
Factors of +10 Added values
(i) + 10 and + 1 + 11 2009/19 Neco (Dec)
(ii) + 5 and + 2 +7 Solve the equation 6x + 16 = x2
The last one fits in. Thus, we replace +7k with +5k +2k
A – 2, 8 B 2, 4 C 2, 6 D 2, 8 E 2, – 8
k2 + 7k +10 = 0 Solution
2
k +5k + 2k + 10 = 0 6x + 16 = x2
k(k + 5) + 2(k + 5) = 0 Rearranging
(k +2)(k +5) = 0 x2 – 6x – 16 = 0
k + 2 = 0 or k + 5 = 0
Next, we find the factors of c i.e – 16 and
k = – 2 or – 5 when added give b i.e – 6 , they are 2 and – 8
Then we replace – 6x by – 8x + 2x
58
x2 – 6x – 16 = 0 becomes Example 3
x – 8x +2x – 16 = 0
2
Solve by factorization 7w2 + 10w = –3
x(x – 8) + 2(x – 8) = 0 Solution
(x – 8)(x + 2) = 0 Rearranging, 7w2 + 10w + 3 = 0
x – 8 = 0 or x + 2 = 0 a = + 7 and c = + 3, thus ac = + 21 and b = + 10
x = 8 or – 2 A Factors of + 21 Added up
1993/10 Exercise 4.21 (i) + 21 and 1 + 22
Solve the equation x2 – 2x – 3 = 0 (ii) – 3 and – 7 - 10
A.(-3, 1) B.(-1, -3 ) C.(3, 1) D.(-3, 0 ) E.(- 1, 3) (iii) + 3 and + 7 + 10
VTR – 11/17 NTI TCII Factors (iii) meet our need; we replace +10w with +3w +7w
Exercise 4.22 7w2 + 10w + 3 = 0 becomes
2
7w + 3w + 7w + 3 = 0
Find the sum of the roots of the quadratic equation
w(7w + 3) + 1(7w +3) = 0
x2 + x – 9 = 3
(7w + 3)(w + 1) = 0
A.- 12 B. – 7 C. -1 D. 1
7w + 3 = 0 or w + 1 = 0
1995/14 Exercise 4.23 7w = –3 or w = –1
What is the smaller value of x for which x2 – 3x + 2 = 0 w = – 3/7 or –1
A. 1 B.2 C. 3 D. 4 E.5 2009/37
2001/91 Neco Exercise 4.24 If c and k are the roots of 6 – x – x2 = 0, find c + k
Solve the equation x2 – 3x – 10 = 0 A2 B1 C–1 D–3
A – 2 or – 5 B – 3 or -10 C5 or – 2 D2 or 5 E 3 or 10 Solution
6 – x – x2 = 0
When a is not Unity ( - 1 inclusive) Here the coefficient of x2 is –1 i.e not unity
Examples: ac i.e – 1  6 is – 6 and b is – 1
(i) 3 – 2x – x2 = 0 Factors of ac that when added gives b are 2 and – 3
(ii) 9 + 8t – t2 = 0 Next, we replace – x by + 2x – 3x
(iii) 7w2 + 10w = - 3 6 – x – x2 = 0 becomes
Quadratic problems like the ones listed above are 6 + 2x – 3x – x2 = 0
solved by finding the factors of the product of a and c 2(3 + x) – x(3 + x) = 0
(ac) that will Sum up to give b. (3 + x)(2 – x) = 0
3 + x = 0 or 2 – x = 0
Example 1.
x = – 3 or 2 i.e c and k
Solve by factorization method; 3 – 2x – x2 = 0
Thus c + k = – 3 + 2
Solution
= –1 C
a = -1 and c = 3 thus ac = -3 and b = - 2
1989/24 Exercise 4.25
Factors of – 3 Added up
Solve the following equation 6x2 – 7x – 5 = 0
(i) 3 and – 1 +2
(ii) – 3 and +1 -2 A. x = ½ or x = -2 1/2 B. x = 1/3 or x = 21/2
Factors (ii) meet our need ; thus, we replace – 2x C. x = 1 2/3 or x = -1/2 D. x = -12/3 or x = ½
with –3x + x E. x = 5/6 or x = -1
3 – 2x – x2 = 0 becomes 1990/24 Exercise 4.26
3 – 3x + x – x2 = 0 Solve the equation 2a2 – 3a – 27 = 0
3(1 – x ) + x(1 – x) = 0 A.3/2, 9 B. –2/3, 9 C. 3, 9/2 D. –3, -9/2 E. –3, 9/2
(1 – x )(3 + x) = 0
1995/17 Exercise 4.27
1– x = 0 or 3 + x = 0
Solve the equation 3x2 + 25x – 18 = 0
x = 1 or – 3
Example 2. A. –3,2 B. – 2, 3 C. – 2,9 D. – 9, 2/3 E. –2/3, 9
Solve by factorization method 9 + 8t – t2 = 0 2004/20 Exercise 4.28
Solution Solve the equation 10 – 3x – x2 = 0
a = –1 and c = 9 thus ac = – 9 and b = +8 A x = 2 or –5 B x = -2 or 5 C x = -1 or 10
Factors of – 9 Added up D x = 2 or 5
(i) +3 and – 3 0 2004/25 Exercise 4.29
(ii) – 9 and +1 -8 Find the sum of the roots of the equation
(iii) 9 and – 1 +8 2x2 + 3x – 9 = 0
Factors (iii) suit us. Thus, we replace + 8t with + 9t – t A. – 18 B.- 6 C. 9/2 D. - 3/2
9 + 8t – t2 = 0 becomes
9 + 9t – t – t2 = 0 1998/8 Exercise 4.30
9(1 + t) – t(1 + t) = 0 Solve the equation 5x2 – 4x - 1 = 0
(1 + t)(9 – t) = 0 A. –1, 1/5 B. –1, -1/5 C. –1, 1/5 D. 1, -1/5 E. – 1, 5
1 + t = 0 or 9 – t = 0
t = – 1 or 9
59
When b = 0 (4) Solve the equation 16k2 = 49
Examples Solution
(i) x2 – 4 = 0 16k2 = 49
(ii) c2 = 9 Rearranging, we have
(iii) 4q2 =36 16k2 – 49 = 0
(iv) 16k2 = 49 Expressing in difference of two squares format.
(v) 2b2 = 2 42k2 – 72 = 0
Quadratic problems like the ones shown above are (4k)2 – 72 = 0
solved by either applying our knowledge of (4k – 7) (4k + 7) = 0
“difference of two squares” i.e. 4k – 7 = 0 or 4k + 7 = 0
x2 – y2 factorizes into (x – y) (x + y ) 4k = 7 or 4k = -7
or making the unknown the subject of formula k = 7/4 or –7/4
k = 13/4 or –13/4
(1) Solve the equation x2 – 4 = 0
Alternatively
Solution
16k2 = 49
To achieve a difference of two squares status we
k2 = 49
observe that
16
x2 – 4 = 0
k=+ 49
x2 – 22 = 0
16
(x – 2) (x + 2) = 0
x – 2 = 0 or x + 2 = 0 = + /4 or – 7/4
7

x = + 2 or – 2
Alternatively (5) Solve the equation 2b2 = 2
x2 - 4 = 0 Solution
x2 = 4 2b2 = 2
x = + √4 i.e +2 or -2 b2 = 2/2
b2 = 1
(2) Solve the equation c2 = 9 b= +√1
Solution = + 1 or – 1
C2 = 9
Rearranging, we have,
C2 – 9 = 0 When C = 0
Expressing in difference of two squares format Examples
C2 – 32 = 0 (i) x2 – 2x = 0
(C – 3) (C + 3) = 0 (ii) 2f2 + 3f = 0
C- 3 = 0 or C + 3 = 0 (iii) 5m2 = 10m
C = + 3 or – 3 (iv) 16z2 = 15z
Alternatively Cases of Quadratic problems similar to the examples above
C2 = 9 are solved by factoring out “one” of the variable (unknown)
C = + 9 and equating the result to zero. Of course the unknown
= + 3 or –3 factored out is equal to zero at the end of the day.
(3) Solve the equation 4q2 = 36
Solution (1) Solve x2 – 2x = 0
4q2 = 36 Solution
Rearranging, we have x2 – 2x = 0
4q2 – 36 = 0 Factoring out x, we have
Expressing in difference of two squares format. x(x – 2) = 0
22q2 - 62 = 0 x = 0 or x – 2 = 0
(2q)2 – 62 = 0 x = 0 or 2
(2q – 6) (2q + 6) = 0
2q – 6 = 0 or 2q + 6 = 0 (2) Solve the equation 2f2 + 3f = 0
2q = 6 or 2q = - 6 Solution
q = 6/2 or – 6/2 2f2 + 3f = 0
q = 3 or – 3 Factoring out f,
f(2f + 3) = 0
Alternatively
f = 0 or 2f + 3 = 0
4q2 = 36
f = 0 or 2f = - 3
q2 = 36/4
f = 0 or -3/2
q = +√9 = + 3 or – 3

60
(3) Solve the equation 5m2 = 10m x2 – 5x + 6 – 12 = 0
Solution x2 – 5x – 6 = 0
5m2 = 10m Factorizing
Rearranging x2 – 6x + x – 6 = 0
5m2 – 10m = 0 x(x – 6) + 1(x – 6) = 0
5m(m – 2) = 0 (x – 6)(x + 1) = 0
5m = 0 or m – 2 = 0 x – 6 = 0 or x + 1 = 0
m = 0/5 or m = 2 x = 6 or –1
m = 0 or 2
2014/42
(4) Solve the equation 6z2 = 15z Solve for x, if (3x + 2)(2x – 7) = 0
Solution A 2/3 or – 7/2 B – 2/3 or 7/2 C 3 or 2
6z2 = 15z D 2 or – 7
Rearranging Solution
6z2 – 15z = 0 The given problem is already factorized by presentation
3z(2z – 5) = 0 (3x + 2)(2x – 7) = 0
3z = 0 or 2z – 5 = 0 3x + 2 = 0 or 2x – 7 = 0
z = 0/3 or 2z = 5 3x = – 2 or 2x = 7
z = 0 or 5/2 x = – 2/3 or 7/2 ( B )
2014/40 NABTEB Exercise 4.31
Solve for x, if x2 – 2x = 0 2009/10b
Ax=0 B x = 2 C x = 0 or 2 D x = – 2 or + 2 2 3
Solve the equation − =1
2000/37 Exercise 4.32 x −1 x +1
Solve the equation 3y2 = 27y Solution
A.y = 0 or 3 B. y = 0 or 9 C. y = - 3 or 3 Applying LCM of x – 1 and x + 1 i.e (x – 1)(x + 1) to LHS
D.y = 3 or 9 2( x + 1) − 3( x − 1)
= 1
1994/23 Exercise 4.33 ( x − 1)( x + 1)
Which of the following is a root of the equation 2 x + 2 − 3x + 3
x2 + 6x = 0 = 1
A. 0 B. 1 C. 2. D. 3 E. it does not have any root. ( x − 1)( x + 1)
5 – x = (x – 1)(x + 1)
Expanding the RHS
Miscellaneous cases 5 – x = x(x – 1) + 1(x – 1)
2014/32 NABTEB 5 – x = x2 – x + x – 1
x+3 3x 5 – x = x2 – 1
Solve the equation = Rearranging
2x − 3 4x − 6
A –6, 3/2 B 6, – 3/2 C 6, 2/3 D 6, 3/2 x2 + x – 6 = 0
Solution Factorizing
x2 – 2x + 3x – 6 = 0
x+3 3x
= x(x – 2) + 3(x – 2) = 0
2x − 3 4x − 6 x – 2 = 0 or x + 3 = 0
Cross multiply x = 2 or – 3
(x + 3)(4x – 6) = 3x(2x – 3) 2014/40 (Nov) Neco
x(4x – 6) + 3(4x – 6) = 3x(2x – 3) If 4 is a root of the quadratic equation
4x2 – 6x + 12x – 18 = 6x2 – 9x x2 + kx + 17 = 0, find value of k
6x2 – 4x2 – 9x + 6x – 12x + 18 = 0 A – 33/4 B 4/33 C4 D 33/4 E 17
2x2 – 15x + 18 = 0 Solution
Factorizing If 4 is a root of the given quadratic equation, then we
2x2 – 12x – 3x + 18 = 0 substitute 4 for x in it
2x(x – 6) – 3(x – 6) = 0 x2 + kx + 17 = 0 becomes
(x – 6)(2x – 3) = 0 2
4 + k (4) + 17 = 0
x – 6 = 0 or 2x – 3 = 0 16 + 4k + 17 = 0
x = 6 or 2x = 3 4k = – 33
x = 6 or 3/2 ( D ) k = – 33/4 (A)
2014/10a 2005/26
Solve: (x – 2)(x – 3) = 12 Given that one of the roots of the equation
Solution 2x2 + (k + 2)x + k = 0 is 2, find the value of k
First, we expand LHS A–4 B–2 C–1 D – 1/ 4
x(x – 3) – 2(x – 3) = 12 Solution
x2 – 3x – 2x + 6 = 12 Since 2 is a root of the equation, we can substitute 2 for x as

61
2(22) + (k + 2)2 + k =0 = 29
8 + 2k + 4 + k = 0 4
3k + 12 = 0 Take the square root of both sides
3k = –12 x – 5/2 =  29
k = –12/3 i.e – 4 A 4
x – 5/ 2 =  29
Completing the Squares method 2
This method is applied where factorization method is not x = 5  29
possible. It derives the way of application from its name 2 2
“completing the squares”. It seeks to make the given x = 5 + 29 or 5 - 29
equation a perfect square, which can be factorized. 2 2 2 2
Examples
1.Solve the equation 2x2 + 3x – 8 = 0 by completing the = 10.4 or – 0.4 = 5.2 or – 0.2 to 1 dp.
squares method 2 2
Solution
2x2 + 3x – 8 = 0 2001/6a (Nov)
2
Make the coefficient of x unity i.e dividing through by 2 Solve the equation 2x2 + 7x + 2 = 0, by method of
x2 + 3/2x – 4 = 0 completing the square. Give answer correct to 3 decimal
Carry the constant (- 4) to the RHS places.
x2 + 3/2 x = 4 Solution
Add the square of half of the coefficient of x to both 2x2 + 7x + 2 = 0
Make the coefficient of x2 unity here divide through by 2
sides of the equation i.e (1/2 of 3/2)2 becomes ( 3/4 )2
7
x2 + 3/2x + ( 3/4 )2 = 4 + (3/4)2 x2 + x +1= 0
2
On the LHS, we take two terms
Carry the constant 1 to the RHS
under squares while we perform
7
arithmetic operation on RHS x2 + x = −1
(x + ¾)2 = 4 + 9/16 2
(x + ¾)2 = 73 Add the square of half of the coefficient of x to both sides of
16 the equation i.e (1/2 of 7/2)2 becomes (7/4)2
2 2
Take the square root of both sides 7 7 7
x + x +   = −1 +  
2
x + ¾ =  73 2 4 4
16 On the LHS, we take the two terms under squares while we
x + ¾ =  73 perform arithmetic operation on the RHS
2
4  7 49
x = – 3/4  73  x +  = −1 +
 4 16
4 2
= – /4 + 73 or – 3/4 – 73  7 33
x +  =
3

4 4  4 16
= – 3 + 8. 5 or – 3 – 8. 5 Take the square root of both sides
4 4 4 4 7 33
= 5.5 or – 11.5 ; x + = 
4 16
4 4
= 1.4 or – 2.9 to 1 d.p 7 33
x + = 
4 4
Example 2
Solve the equation x2 – 5x – 1 = 0 7
33
x= − 
by completing the square method, leaving your roots 44
correct to 1 decimal place.
7 33 7 33
Solution =− + or − −
The coefficient of x2 is already unity. 4 4 4 4
x2 – 5x – 1 = 0 7 5.7446 7 5.7446
Carry the constant (–1) to the RHS = − + or − −
4 4 4 4
x2 – 5x = 1
Add the square of half the coefficient of x to both sides.
− 1.2554 − 12.7446
= or
Here the coefficient of x is (-5). i.e ( 1/2 of – 5 )2 = (- 5/2 )2 4 4
x2 – 5x + (- 5/2 )2 = 1 + (- 5/2 )2 = – 0. 31385 or – 3.18615
The two terms under square on the RHS are x and – 5/2  – 0.314 or – 3.186 to 3d.p
(x – 5/2 )2 = 1 + 25/4
62
Making a given expression a perfect square 2006/20
Some quadratic expressions just require a certain number What must be added to x2 – 3x to make it a perfect square?
to be added to make it a perfect square. 9 9
We can do this by applying our knowledge of completing A B C6 D9
4 2
the square method.
Solution
Examples
We observe that the coefficient of x is – 3
(1) What must be added to make the expression
x2 + 6x a perfect square? Then square of ½ the coefficient of x is
2 2
Solution 1   − 3 9
First step we check whether the coefficient of x2 is  of − 3  i.e   = A
2   2  4
unity, if not we make it unity by dividing through by the Therefore for x2 – 3x to be a perfect square, 9/4 is to be added
coefficient of x2.
2nd step. We take square of half the coefficient of x 2008/25 Neco (Dec)
i.e (1/2 of + 6)2 = (+3 )2 i.e 9 What must be added to x2 + 5x to make it a perfect square?
Thus, adding +9 and not +3 to make the expression a
5 5 25
perfect square. A 10x B x C D E 25
2 2 4
2. Find the value of t such that the given expression is Solution
a perfect square. y2 + t y + 9/4 We observe that the coefficient of x is 5
Solution Thus square of ½ the coefficient of x is
This is a reverse process of the steps taking in the 2 2
preceding example. 1  5 25
 of 5  i.e   = D
Starting with last operation, which is squaring. 2  2 4
To remove square we take square root of the constant. Therefore for x2 + 5x to be a perfect square, 25/4 is be added
i.e 9/4 =  3/2 ignore minus values.
= + 3/2 2008/41 NABTEB (Nov) Exercise 4.34
Next operation to be removed is “1/2 of ”. What value of m makes x2 + 8x + m perfect square?
To do this, we multiply the new constant by 2 A – 16 B – 8 C8 D 16
i.e 2 x 3/2 = 3 Thus t = 3 2014/7b ( Nov ) Exercise 4.35
When k is added to the expression y2 – 12y , the expression
2010/48 counter example
becomes ( y + p )2 . Find the values of p and k
16
If x + kx +
2
is a perfect square, find the value of x 2005/33 (Nov) Exercise 4.36
9 Find the value of t that will make x2 – 40x + t
A 8/ 3 B 7/ 3 C 5/ 3 D 2/ 3 a perfect square
Solution A 900 B 400 C 200 D 100
This is a reverse process of the steps taken in the
preceding example.
Starting with the last operation, which is squaring
To remove square, we take square root of 16/9 Formula Method
16 4 Given any quadratic equation x2 + bx + c = 0 the roots
i.e =  ignore minus value here are gotten by
9 3
= + 4/3
x=-b  b2 – 4ac
Next operation to be remove is “ 1/2 of ”
2a
To do this, we multiply the new constant by 2 i.e
Where a, b and c are constants. The general formula
4 8 (Almighty formula) as it is called can be used to solve any
2 =
3 3 quadratic equation.
Thus k is 8/3 (A) It is derived from the general form of a quadratic equation:
ax2 + bx + c = 0; by completing the square method as:
2008/17 Neco ax2 + bx + c = 0
What must be added to expression v2 – 18v to make Divide through by a to make the coefficient of x2 unity
it a perfect square? x2 + bx + c = 0
A – 81 B – 18 C 9 D 36 E 81 a a
Solution x2 + b x = – c
We observe that the coefficient of v is – 18 a a
thus square of ½ the coefficient of v is Add the square of ½ coefficient of x to both sides
2 2
1   − 18  i.e (1/2 of b/a )2 = ( b/2a )2
 of − 18  i.e   = 81
2   2  x2 + b/a x + b 2
= -c + b 2
Therefore for v2 – 18v to be a perfect square, 2a a 2a
81 is be added E
63
= - c + b2
a 4a2 =2 24
2
= - 4ac + b2 = 2 + 4.899 or 2 – 4.899
4a2 2 2
x+b 2
= b – 4ac
2
= 6.899 or – 2.899
2a 4a2 2 2
Take square root of both sides
= 3.449 or – 1.449
x+b=  b2 - 4ac  3.45 or – 1.45 to 2 d.p
2a 4a2
2008/7a
Solve, correct to two decimal places,
= b2 – 4ac
the equation 4x2 = 11x + 21
2a
Solution
x=-b  b2 – 4ac Rearranging the equation
2a 2a 4x2 – 11x – 21 = 0
No factor of ac (4 × – 21) that will give b (–11)
Thus the equation is not factorizable
x=-b  b2 – 4ac
Applying the general formula for ax2 + bx + c = 0
2a
Examples − b  b 2 − 4ac
1. Solve the equation 2x2 + 7x – 5 = 0 correct to 1 dp
x =
2a
Solution Here a is 4, b is – 11 and c is – 21
Mere observation shows that the given equation cannot
be factorized. Thus we apply the general formula − (−11)  (−11) 2 − 4  4  (−21)
=
x=-b b2 – 4ac 24
2a 11  121 + 336 11  457
= =
Comparing the given equation with ax2 + bx + c = 0 8 8
We have; a = 2, b = 7, c = - 5 11  21.378
Substituting =
8
= -7  72 – 4 x 2 x (-5) 11+ 21.378 11 − 21.378
2 x2 = or
8 8
32.378 − 10.378
= -7  49 + 40 = or
8 8
4 = 4.05 or –1.30 to 2d.p
= -7  89
4 2006/21 Neco (Dec) Exercise 4.37
= -7  9.4 Which of the following are roots of the quadratic equation
2x2 – 8x + 5 = 0?
4
= -7 + 9.4 or - 7 - 9 .4 6 6 6
A 2 B– 2 C– 4
4 4 2 2 4
= 0.6 or – 4.1 to 1dp
6 6
D– 6 E– 8
4 4
2. Solve the equation of t2 – 2 t – 5 = 0 correct to 2 d.p 2014/13(Nov) Exercise 4.38
Solution
Solve the equation : x2 – 132 x + 15
2 = 0
Comparing the given equation with the general form
5 3
at2 + bt + c = 0; a = 1, b = - 2, c = -5 A. x = – 1 or 2 B. x = – 2 or 5
Substituting, 1 5 3
C. x = – 2 or 2 D. x = 2 or 5
t = - (-2)  ( - 2)2 – 4 x 1 x (-5)

2 x 1

=+2  4 + 20

2
64
2015/11 Exercise 4.40
Word problems Adding 42 to a given positive number gives the same result
2009/47 (Nov) as squaring the number. Find the number
If 3 times a certain number is subtracted from twice the A. 14 B. 13 C. 7 D. 6
square of the number, the result is 5. What are the
possible values of the number? Simultaneous Linear and Quadratic equations
A – 5, – 1 B –1, 21/2 C –1, – 21/2 D 5, – 1 The approach given to problems of this nature, is to make
Solution one of the unknown in the linear equation the subject of
Let the number be x formula then substitute the value into the quadratic equation
2(x2) – 3x = 5 or factoring the quadratic equation which has unknown of
i .e 2x2 – 3x – 5 = 0 the form x2 – y 2 (difference of two squares) and then
Coefficient of x2 is not unity. substituting into the linear equation
Here ac, i.e 2 × (– 5) is – 10 and b is – 3 Examples:
Factors of ac that when added gives b are 2 and – 5 1. Solve the equations
Replace – 3x by + 2x – 5x x – y = 2 and x2 + y2 = 52
2x2 – 3x – 5 = 0 becomes Solution
2x + 2x – 5x – 5 = 0
2
x – y = 2 ------ (1)
2x(x + 1) – 5(x + 1) = 0 x2 + y2 = 52 ------ (2)
(x +1)(2x – 5) = 0 From (1) x – y = 2
x + 1 = 0 or 2x – 5 = 0 x=y+2
x = –1 or 2x = 5 Substitute x value into (2)
x = –1 or 5/2 i.e –1, 21/2 B x2 + y2 = 52 will become
( y + 2)2 + y2 = 52
1995/7c y2 + 4y + 4 + y2 = 52
The product of two consecutive positive odd numbers is 2y + 4y + 4 – 52 = 0
2
195. By constructing a quadratic equation and solving it 2y2 + 4y – 48 = 0
find the two numbers. Reducing the terms, we have
Solution y2 + 2y – 24 = 0
Let the two consecutive odd numbers be y and y + 2 Factorizing,
y(y + 2) = 195 y2 + 6y – 4y – 24 = 0
y2 + 2y = 195 y (y + 6) – 4 ( y + 6) = 0
y + 2y – 195 = 0
2
(y + 6) (y – 4 ) = 0
Factorising, we have y + 6 = 0 or y – 4 = 0
y2 – 13y + 15y – 195 = 0 y = - 6 or + 4
y(y – 13 ) + 15(y –13 ) = 0 Substitute y values into (1)
(y – 13) (y + 15 ) = 0 x – y = 2 will become
y – 13 = 0 or y + 15 = 0 x - (- 6) = 2 or x - 4 = 2
y = 13 or -15 x + 6 = 2 or x - 4 = 2
Since the numbers are positive we disregard –15.Thus, x= 2 - 6 or x = 2 + 6
y = 13 and y +2 = 13 + 2 i.e 15 x = - 4 or 8
The solution sets are
2003/40 NABTEB ( -4, - 6) and ( 8, 4) in (x, y ) format
Find two consecutive numbers whose product is 156.
A 12 and 13 B 15 and 16 2. Solve the equation
C – 12 and 13 D – 13 and 12 p + q = 3, and p2 – q2 = 15
Let the two consecutive numbers be x and x +1 Solution
Translating: x (x + 1) = 156 p + q = 3 - ---- (1)
i .e x2 + x = 156 p2 – q2 =15 --- (2)
x2 + x – 156 = 0 The LHS of (2) can be factorized as :
Factorising (p + q ) (p – q) = 15
x2 + 13x – 12x – 156 = 0 Substituting the value of p + q = 3
x ( x + 13) – 12(x + 13) = 0 3( p – q) = 15
( x + 13 ) (x – 12) = 0 Divide both sides by 3
x + 13 = 0 or x –12 = 0 p - q = 5 ---- (3 )
x = -13 or 12 (D) p + q = 3 ---- (1)
2p = 8
2014/41 Neco ( Nov) Exercise 4.39 p = 8/2 = 4
The sum of two consecutive odd numbers is – 4. Find Substitute for p value in (1)
the two numbers p + q = 3 will become
A. 3 and –1 B. –3 and –1 C. 3 and 1 4 + q = 3 q = 3 – 4 = -1
D. 3 and 4 E.–3 and – 4
65
Other Examples Solution
3. If y = x2 – 4x – 10 and y = 2 y = x2 + x – 2 ------- (1)
Find the values of x that satisfies both relations y=x+1 -------- (2)
Solution From (2) substitute y = x + 1 into (1)
Since y = x2 – 4x – 10 and y = 2 then y = x2 + x – 2 will become
y  x2 – 4x – 10 = 2 x + 1 = x2 + x – 2
x2 – 4x – 10 – 2 = 0 x2 + x – 2 – x –1 = 0
x2 – 4x – 12 = 0 x2 - 3 = 0 (B)
Factorising
x2 – 6x + 2x – 12 = 0 2002/1 Neco Exercise 4.41
x(x – 6) + 2(x – 6) = 0 If y = 3x2 – 5x – 2 at what values of x is y = – 4?
(x – 6)( x + 2 ) = 0 2003/9 Neco Exercise 4.42
x = 6 or – 2 If x – y = 3 and x2 – y2 = 0, find the values of x and y
2001/22 respectively.
The graph of the curve y = 2x2 – 5x – 1 A. 1/2, - 1/2 B. 1/2 , 1/2 C. –3/2, -3
/2
and a straight line PQ were drawn to D.- 3/2, 3/2 E.3/2, - 3/2
solve the equation 2x2 – 5x + 2 = 0 what is
the equation of the line PQ?
A. y = – 1 B. y = 1 C. y = 3 D. y = –3
Solution
Inequalities I
It is not always that equality sign serves our purposes.
Let y = 2x2 – 5x – 1 ------- (1)
Statements such as Lagos is more populated than Abuja,
2x2 – 5x + 2 = 0 ------- (2)
N200 is not equal to N500 will not fit with equality. Thus,
Then (2) – (1) our use of inequality sign.
Will yield the required equation. < “ is less than ‘’
y = 2x2 – 5x + 2 – (2x2 – 5x – 1) > “is greater than’’
= 2x2 – 5x + 2 – 2x2 + 5x + 1. > “ is greater than or equal to”
y = 3 (C)  “ is less than or equal to ”
VTR – 13/3A NTI TCII
Find the value of x and y such that y = ½ (x2 – 3) < ( less than ) and > ( greater than)
and x + y = 6. are called strict inequalities
Solution Examples
y = ½ (x – 3) ----- (1)
2 (1) x  3 means x could be any of 4,5,6 …
x + y = 6 ---- (2) But x cannot be 3
From (2) y = 6 – x
Substitute y value into (1) (2) y < 5 means y can take
y = ½ (x2 – 3) will become values 4,3,2,1,0, – 1 … but not 5
6 – x = ½ (x2 – 3)
Multiply through by 2 to clear fraction > (greater than or equal to ) and < ( less than or equal to )
2(6 – x) = x2 – 3 are called weak inequalities.
12 – 2x = x2 – 3 the word “at most” is used for greater than or equal to.
i.e. x2 + 2x – 15 = 0 Examples
Factorizing 1. t  10 means it can take values 10, 11, 12 …
x2 + 5x – 3x – 15 = 0 2. s  17 means it accepts values as from 17, 16, 15 …
x(x + 5) – 3(x +5) = 0
(x + 5)(x – 3) = 0 Linear inequalities in one variable
x + 5 = 0 or x – 3 = 0 Basic operational principles of linear equations applies to
x = – 5 or 3 linear inequalities except when we multiply or divide both
Substitute x value into (2) sides of the equation by a negative number – then we
x + y = 6 will become reverse the inequality symbol concerned
–5 + y = 6 or 3 + y = 6 Examples
y = 6 + 5 or y = 6 – 3 1. Solve 3x – 4 > 8
y = 11 or 3 Solution
The solution Sets: (– 5, 11) and ( 3, 3 ) in (x, y) format. 3x – 4 > 8
3x > 8 + 4
2002/32 NABTEB 3x > 8 + 4
Write down the equation whose roots are the points of 3x > 12
intersection of the graphs of y = x2 + x – 2 and y = x + 1 Divide both sides by 3
A.x2 + 3 = 0 B.x2 – 3 = 0 C. x2 – 1 = 0 x > 12/3
D. x + 2x – 3 = 0
2
x>4

66
2. For what values of x is When x = -1
½ (x + 3) < 3 x + 3  4x
Solution -1 + 3  4 (-1)
½ ( x + 3) < 3 2-4 Is it true?
Multiply both sides by 2 Please adhere strictly to sign changes when dividing or
x+3<6 multiplying both sides of inequality equation by a negative value
x<6–3
2014/20 NABTEB (Nov)
x<3
Find the range of values of x for which 3(x + 8) < 7x
3. Solve the inequality 6x – 14 ≥ 13x Ax>–6 Bx<–6 Cx>2 Dx>6
Solution Solution
6x – 14 ≥13x 3(x + 8) < 7x
Collect like terms together 3x + 24 < 7x
6x – 13x > 14 Collect like terms together
– 7x > 14 3x – 7x < – 24
Divide both sides by the coefficient of x i.e – 7 – 4x < – 24
-7x ≤ 14 Divide both sides by – 4 and reverse the inequality sign
-7 -7 − 4x − 24
Notice the inequality symbol changes from > to < 
−4 4
x < - 2
x > –6 (A)
4. Solve the inequality x + 3 < 4x 2007/44
Solution For what range of values of x is 4x – 3(2x – 1) > 1 ?
x + 3 < 4x Ax>–1 Bx>1 Cx<1 Dx<–1
Collect like terms together Solution
x – 4x < - 3 4x – 3(2x – 1) > 1
-3x < - 3 4x – 6x + 3 > 1
Divide both sides by the coefficient of x i.e –3 Collect like terms together
-3x  - 3 4x – 6x > 1 – 3
-3 -3 –2x > – 2
x1 Divide both sides by the coefficient of x i.e – 2 and
reverse the inequality sign
Checking − 2x −2
Lets try the values of x within the solution range: 1, 2, 3… 
−2 −2
When x =1 x < 1 C
x + 3 < 4x will be 2007/27 Neco
1 + 3 < 4(1) true Solve the inequality: x – 4(x + 2) > 8 + 5x
Ax>0 Bx<0 Cx>–2 Dx<–2 Ex>2
when x = 2 Solution
x + 3 < 4x will be x – 4(x + 2) > 8 + 5x
2 + 3 < 4(2) true x – 4x – 8 > 8 + 5x
5<8 Collect like terms together
x – 4x – 5x > 8 + 8
when x = 3
– 8x > 16
3 + 3  4(3) Divide both side by – 8 and reverse the inequality sign
6 < 12
− 8x 16
Let’s now try the non-reversed inequality. i.e 
−8 −8
-3x  -3
x < –2 (D)
Divide both sides by –3 but do not reverse the sign as
2009/36 (Nov)
some students will argue – that both sides are negative.
-3x  -3 Solve the inequality: 13 (2x – 1) < 5
-3 -3 Ax<–5 Bx<7 Cx>8 Dx<8
x<1 Solution
1
Lets try the values of x within the solution range: 3 (2x – 1) < 5
1, 0, -1, -2… Multiply through by 3 to clear fraction
when x = 1 2x – 1 < 15
x + 3  4x 2x < 15 + 1
4 = 4 true 2x < 16
when x = 0 2x 16

x + 3  4x 2 2
0+30 Is it true? x < 8 (D)
67
2005/50 Neco Solution
x x 2 1 – 2x < −
1
Solve the inequality, − 
6 2 3 3
Ax<1 Bx–2 Cx<–2 Dx2 Ex–2 Multiply through by 3 to clear fraction
Solution 3 – 6x < – 1– 3
– 6x < – 1
x x 2
−  – 6x < – 4
6 2 3 Divide both sides by – 6 and reverse the inequality sign
Multiply through by the LCM of the denominators 6, 2 − 6x −4
and 3 i.e 6 to clear fractions 
−6 −6
x – 3x  2(2) 2
x – 3x  4 x > (C)
3
– 2x  4 2008/42
Divide both sides by – 2 and reverse the inequality sign
1
− 2x 4 For what range of values of x is x + 2  2 x − 1 ?
 2
−2 −2 Ax2 Bx 2 Cx 3 Dx 3
x  –2 (E) Solution
1
2009/1a Neco x + 2  2x − 1
2
Solve the inequalities given below
Multiply through by 2 to clear fraction
1 1
(3x − 4) − (2 + 3x)  x + 5 x + 4  4x – 2
2 3 Collect like terms together
Solution x – 4x  – 2 – 4
1 1 – 3x  – 6
(3x − 4) − (2 + 3x)  x + 5
2 3 Divide both side by – 3 and reverse the inequality sign
Multiply through by the LCM of the denominators − 3x −6
2 and 3 i.e 6 to clear fractions 
−3 −3
3(3x – 4) – 2(2 + 3x)  6(x + 5)
x  2 (A)
9x – 12 – 4 – 6x  6x + 30 2008/ 2a
Collect like terms together 2 1
9x – 6x – 6x  30 + 12 + 4 Solve the inequality: ( x − 2) − ( x + 5)  0
5 6
– 3x  46
Solution
Divide both side by – 3 and reverse the inequality sign
2 1
− 3x 46 ( x − 2) − ( x + 5)  0
 5 6
−3 −3
Multiply through by the LCM of the denominators
x  – 151/3 5 and 6 i.e 30 to clear fraction
2008/ 27 Neco (Dec)
6  2(x – 2) – 5(x + 5)  0
y
Solve the inequality (y – 3) < 12x – 24 – 5x – 25  0
3 Collect like terms together
Ay<2 B y < 3.5 C y < 4.5 D y > 4.5 E y > 6 12x – 5x  24 + 25
Solution
7x  49
y
(y – 3) < 7x 49
3 
7 7
Multiply through by 3 to clear fraction
x  7
3y – 9 < y
2012/14
Collect like terms together
−m 5 5m 7
3y – y < 9 Solve the inequality: −  −
2y < 9 2 4 12 6
2y 9 5 5 −1 −1
 Am Bm Cm Dx
2 2 4 4 11 11
y < 4.5 ( C ) Solution
−m 5 5m 7
−  −
2008/41 2 4 12 6
1 Multiply through by the LCM of the denominators
Solve the inequality 1 – 2x < −
3 2, 4, 12 and 6 i.e 12 to clear fractions
2 2 2 2 – 6m – 3(5)  5m – 2(7)
Ax< Bx<– Cx> Dx>– – 6m – 15  5m – 14
3 3 3 3

68
Collect like terms together Inequality combination and ranges
– 6m – 5m  – 14 + 15 Two inequalities about a particular variable can be combined
– 11m  1 as follows:
Divide both sides by – 11 and reverse the inequality sign 1. If – 4 < x and x < 2, combine the two inequalities
− 11m 1 and give the range of x to whole numbers

− 11 − 11 Solution
−1 By observation we know that x is between - 4 and + 2,
m  (C)
11 thus combining
-4<x<2
2009/ 43 (Nov) Range of x = -3, -2, -1, 0, 1
The time spent in answering a question was recorded The two limits (- 4 and 2 ) are not included because of the
by 3 boys as follows: x = 1 min 9 sec ; y = 11/9 min and strict inequalities at both ends
z = 1.19min. Arrange these times in order starting with
the least 2. Given that x > 8 and 20 > x, combine the two
Ax<y<z By<x<z Cy<z<x Dz<y<x inequalities and list their ranges that are whole
Solution numbers only
x = 1 min 9 sec Solution
y = 11/9 min i.e 1min 6.67 sec (1/9 of 60sec ) i . Combination
z = 1.19 min i.e 1 min 11.4 sec (0.19  60sec) 20 > x and x > 8
Smallest is y, next x, then z Combined as
y<x<z B 20 > x > 8
ii. Range = { 19,18 … 11,10,9}
2010/16 Exercise 4.43 Both extreme values of 20 and 8 are not included because of
Solve the inequality: 3(x + 1)  5(x + 2) + 15 the strict inequalities
A x  – 14 B x  – 14 C x  – 11 D x  – 11
3. If t ≤ 3 and 0 < t
2012/22 Neco Exercise 4.44 the result of combining the two inequalities is:
Solve the inequality: 4(2 – 3x) < 2 – 2 x Solution
3 0 < t ≤ 3 and
9
Ax  – Bx 9 Cx< 9 Range = { 1, 2, 3} in whole numbers only.
17 17 17
0 is not included because of the strict inequality at that end
9 17
Dx > Ex> but 3 is included as a result of the weak inequality at that
17 9
limit.
2012/4b Neco Exercise 4.45
4. If r < - 5 and – 9 < r
1 1
Solve the inequality ( x + 8) − (2 x − 4)  1 the result of combining the two inequalities is:
3 7 Solution
2013/10 Exercise 4.46 -9<r<-5
2x − 5 Range (whole numbers) = {-9, -8,…-3 , – 4}
Solve the inequality:  (2 − x) -9 is included because of the weak inequality there whereas
2
– 5 is not, due to the strict inequality
Ax>0 Bx<0 C x > 2 1/ 2 D x < 21/4
2013/2a Exercise 4.47 2009/36
Solve: 7x + 4 < 12 (4x + 3) If x is a positive integer, list the values of x which satisfy the
equations 3x – 4  6 and x – 1 > 0
2014/23 Neco Exercise 4.48 A {1, 2, 3} B {2, 3} C {2, 3, 4} D {2, 3, 4, 5}
Find the solution set of the inequality Solution
x 3 − 2x x +1 Solving for x values
+ 
3 5 5 3x – 4  6 and x – 1 > 0
A x  11/ 2 B x  11/3 Cx1 3x  6 + 4 and x >1
Dx –1 E x  –11/2 3x  10 and x >1
3x 10
2015/2a Exercise 4.49  and x>1
Solve the inequality : 4 + 34 (x + 2)  3
8 x +1 3 3
x  10/3 i.e 3 (integer)
2015/4b Neco Exercise 4.50 Range x >1 but x  3 = {2, 3}
5x 1 x 7
Solve the inequality : −  +
8 6 3 24 2005/19
Find the range of values of x for which
2x – 1  3 and 2 – x  5
A–3x1 B–2x3 C–3x4 D–3x2
69
Solution 2000/21 UME
Solving both inequalities at the same time Solve the inequality 2 – x > x2
2x – 1  3 and 2 – x  5 A. x < – 2 or x > 1 B. x > 2 or x < – 1
2x  3 + 1 and –x  5–2 C. –1 < x < 2 D. – 2 < x < 1
2x  4 and –x  3 Solution
Here we reverse inequality 2–x>x 2

2x 4 −x 3 Rearranging
 and 
2 2 −1 −1 2 – x – x2 > 0
x  2 and x  –3 Factorizing the LHS
Combing both results 2 – 2x + x – x2 > 0
x  – 3 is same as – 3  x 2 (1– x) + x(1– x) > 0
–3x2 (D) (1– x)(2 + x) > 0
By quadratic principle
2011/6b Exercise 4.51
1– x > 0 or 2 + x > 0
If 2 + x < 6 and 7 + x  4, what is the range of x
– x > –1 or x > – 2
satisfying both inequalities
x < 1 or x > – 2
Inequalities involving quadratic expression Combining the two equalities from the smallest – 2
Here the basic operational principle of solving quadratic –2<x<1 (D)
equations by method of factorization is mostly applied. Note that:
x > – 2 means – 2 < x to enable us combine the two inequalities
1993/20 UME 2006/33 UME Exercise 4.52
Solve the inequality y2 – 3y > 18
If y = x -x-12, find the range of values of x for which y  0.
2
A. – 3 < y < 6 B. y < –3 or y > 6
A. x  – 3 or x  4 B. x < – 3 or x > 4.
C. y > – 3 or y > 6 D. y < –3 or y < 6
Solution C. – 3  x  4 D. – 3 < x  4
y2 – 3y > 18 2007/19 UME Exercise 4.53
Rearranging The solution of the quadratic inequality ( x2 + x – 12 )  0 is
y2 – 3y – 18 > 0 A. x  – 3 or x  4 B. x  3 or x  – 4
Factorising the resulting quadratic expression on C. x  3 or x  – 4 D. x  3 or x  – 4
the LHS, 2015/35 Neco Exercise 4.54
y2 – 6y + 3y – 18 > 0 y − 1 6
y (y – 6) + 3 ( y – 6) > 0 Solve the inequality 
2 y
(y – 6) (y + 3) > 0
By quadratic principle A. y  -3 B. y  4 C. y  -3 or y  4
y – 6 > 0 or y + 3 > 0 D. -3  y  4 E. -4  y  3
y > 6 or y > – 3 (C)

1997/18 UME Representation of inequality in a number line


Find the range of values of x which satisfies the O – open dot is used to represent strict inequalities
inequality 12x2 < x + 1 i.e < or > while,
A–1<x< 1 B. 1 < x < 1 – thick dot represents weak inequality
4 3 4 3 i.e  or 
C. 1<x<1 D.–1 < x < – 1 Single inequality number line
4 3 4 3 2005/9b
Solution Solve the following inequality : 2(x + 3) – 3(x – 1)  12
12x2 < x + 1 Represent your answer on the number line
Rearranging Solution
12x2 – x – 1 < 0 2(x + 3) – 3(x – 1)  12
Factorizing the LHS 2x + 6 – 3x + 3  12
12x2 – 4x + 3x –1 < 0 2x – 3x  12 – 6 – 3
4x (3x –1) + 1 (3x – 1) < 0 –x  3
( 3x – 1)(4x + 1) < 0 Divide both sides by –1 and reverse the inequality sign
By quadratic format −x 3

3x – 1 < 0 or 4x + 1 < 0 −1 −1
3x < 1 or 4x < – 1 x  –3
x < 1 or x < – 1 The line starts at –3 with thick dot ( weak inequality there) and
3 4 moves forward
The values show that x is between – 1/4 and 1/3 . Thus
– 1/4 < x < 1/3 A.
–5 –4 –3 –2 –1 0 1 2
70
2005/9a 3x + 4 < 2(x + 3)
Find the smallest integer that satisfies the inequality 3x + 4 < 2x + 6
x + 8 < 4x – 15 Collect like terms together
Solution 3x – 2x < 6 – 4
First we solve for the range of x x <2
x + 8 < 4x – 15 The line starts at 2 with open dot and goes backward (D)
Collect like terms together
x – 4x < – 15 – 8 2014/21 Neco
– 3x < – 23 Which of the following inequalities is represented by the
Divide both sides by –3 and reverse the inequality sign number line ?
− 3x − 23
 x
−3 −3 – 3 – 2 –1 0 1 2 3
x > 7.67
The smallest integer that satisfies the inequality is 8. A. x > – 2 B. x ≥ – 2 C. x ≤ 2 D. x < 2 E. x = 2
The number line will explain it better Solution
The starting point is –2 and it is open dot i.e strict inequality.
Since the arrow is pointing forward from –2 to positive axis
x it implies greater than
6 7 8 9 10 11
Thus x > – 2 (A)
So the first whole number (integer) on the path of the arrow is 8
2014/10c NABTEB
2006/14
3 9
Represent the solution of the inequality Solve the inequality : (x–3) ≥ and represent your
x−2 x+3 4 2
 on the number line answer in a number line
2 4
Solution
A B 3 9
0 7 0 7
(x–3) ≥
4 2
Multiply through by the LCM of 4 and 2 i.e 4 to clear fractions
C D 3( x – 3 ) ≥ 2 × 9
0 7 0 7 3x – 9 ≥ 18
Solution 3x ≥ 18 + 9
x−2 x +3 3x ≥ 27

2 4 3x 27
Multiply through by the LCM of 4 and 2 i.e 4 to clear 
3 3
fractions
x ≥ 9
2(x – 2) ≥ x +3
The number line is .
2x – 4 ≥ x +3 0 8 9 10
2x – x ≥ 3 +4
x ≥ 7 (D) 2005/46 Exercise 4.55 option D
Represent the solution of 3x + 15 > 0 on a number line
2009/15 A.
Which of the following illustrates the solution of
( 3x + 4 ) –6 –5 –4 –3 –2 –1 0 1
 x +3 ? B.
2
A.
–1 0 1 2 3 4 5 6
–3 –2 –1 0 1 2 3 4 C.
B. –1 0 1 2 3 4 5 6
D.
–3 –2 –1 0 1 2 3 4
C. –6 –5 –4 –3 –2 –1 0 1

–3 –2 –1 0 1 2 3 4 2006/5b Neco (Dec) Exercise 4.56


( i ) Find the range of values of x for which
D.
2x – 3 > 7x + 12
–3 –2 –1 0 1 2 3 4 ( ii ) Represent your answer on a number line
Solution 2014/10a (Nov) Exercise 4.57
( 3x + 4 ) 1 5
First we solve  x +3 ( i ) Solve the inequality : x − ( x + 2)  1 + x
2 2 6
Multiply through by 2 to clear fraction ( ii ) Illustrate the solution on a number line
71
2006/41 ( Nov ) Exercise 4.58 Solution
Which of the following represents the solution of the The range is from – 5 to 3
1 3x Thick dot at – 5; reason – weak inequality of 
inequality x −  on the number line ?
4 2 Open dot at 3; reason – strict inequality of <
A B
-2 -1 0 -2 -1 0
-5 -4 -3 –2 –1 0 1 2 3 4
C D 2011/48
-1 0 1 -1 0 1 Illustrate the inequality –1< 3x + 5 < 14 on a number line
2006/7 Neco ( Nov ) Exercise 4.59 A.
x
– 2 –1 0 1 2 3 4 5 6
–3 –2 –1 0 1 2 3 B.
The number line above can be represented by which one
of the following inequalities? – 2 –1 0 1 2 3 4 5 6
A. x < – 3 B. x < 3 C. x > – 3 D. x ≥ 3 E. x > 3 C.
2013/12 Exercise 4.60
– 2 –1 0 1 2 3 4 5 6
Which of the following lines represents the solution of
the inequality 7x < 9x – 4 ? D.

A B – 2 –1 0 1 2 3 4 5 6
-2 0 2 -2 0 2
Solution
C D –1 < 3x + 5 < 14
-2 0 2 -2 0 2 First, we solve for the range
Applying the split method, we have
– 1 < 3x + 5 and 3x + 5 < 14
Double inequalities number line Collect like terms together
2006/7a – 1 – 5 < 3x and 3x < 14 – 5
( i ) If 4x < 2 + 3x and x – 8 < 3x, what range of – 6 < 3x and 3x < 9
values of x satisfies both inequalities? 6 3x 3x 9
( ii ) Represent your result in ( i ) on the number line −  and 
3 3 3 3
Solution
–2 < x and x < 3
4x < 2 + 3x and x – 8 < 3x
Thus combining the result, we have – 2 < x < 3
4x – 3x < 2 and x – 3x < 8
Open dot at – 2 and open dot at 3 ( B )
x<2 and – 2x < 8
Divide both sides by – 2 and
2014/7
reverse inequality Which of the following number lines represents the
− 2x

8 solution to the inequality : – 9  2 x – 7 < 5
3
−2 −2
x > –4 A B
Combining the two ranges –3 0 18 –3 0 18
–4<x<2
( ii ) C D
–3 0 18 –3 0 18
–4 0 2 Solution
First, we solve for the range
2009/22 NABTEB (Nov) Applying the split method
State the range of values of x for the graph below:
– 9  2 x – 7 < 5 will become
3
–9 2 x–7 and 2
x–7<5
–1 0 +4 3 3
2 2
A either x < – 1 or x  4 B either x < – 1 or x > 4 –9+7 x and x<5+7
3 3
C either x < – 1 or x  4 D either x  1 or x  – 4
–2 2 x and 2
x < 12
Solution 3 3
LHS is x < – 1 (open dot and arrow backward) – 6  2x and 2x < 36
RHS is x > 4 (open dot and arrow forward) Option B –3 x and x < 18
Then combining the results, we have
2009/9 Neco (Nov)
– 3  x < 18 option D Thick dot at –3 and open dot at 18
Show on the number line the inequalities – 5  x < 3

72
2014/9 2006/14 Neco Exercise 4.62
Given that x > y and 3< y, which of the following What is the solution of the inequality represented on the number
is / are true? line below?
I y > 3 II x < 3 III x > y > 3
A I only B I and II only C I and III only D I, II and III only x
Solution –3 –2 –1 0 1 2 3
From the statement A–2x 3 B–2<x3 C–2>x>3
x > y, D–2x<3 E–2<x <3
x > 3 and y > 3
2008/15 Neco Exercise 4.63
Conveniently, we can take y > 3, then
The number line below represents :
Combining x > y > 3. Thus option C x
2014/27 Neco
Which of the following number lines correctly –5 – 4 – 3 – 2 –1 0 1 2 3
represents the inequality – 4  x  3?
A0 x<–5 B0<x<–5 C–5 x<2
A.
D –5 < x  2 E–5<x<2
–4 –3 –2 –1 0 1 2 3 4 5 2008/43 Exercise 4.64
B. Which of the number lines below best represent the
inequality –3  x < 4?
–4 –3 –2 –1 0 1 2 3 4 5 A.
C.
– 3 –2 –1 0 1 2 3 4
–4 –3 –2 –1 0 1 2 3 4 5 B.
D.
– 3 –2 –1 0 1 2 3 4
–4 –3 –2 –1 0 1 2 3 4 5 C.
E.
– 3 –2 –1 0 1 2 3 4
–4 –3 –2 –1 0 1 2 3 4 5 D.
Solution
–4x3 – 3 –2 –1 0 1 2 3 4
The line starts at – 4 and ends at 3. 2011/22 Neco Exercise 4.65
Both ends are weak inequalities i.e thick dots The number line below represents
Option D

2012/17 Neco –2
–3 –1 0 1 2 3 4 5
The number line below represents A–3x<5 B–3<x5 C – 3< x < 5
D–3x5 E–3x5
x 2012/10 Exercise 4.66
– 3 –2 –1 0 1 2 Represent the inequality – 7 < 4x + 9  14 on a number line
A–2>x0 B–2x>0 C–2<x<0 A.
D–2<x 0 E–2x<0
Solution –6 –5 –4 –3 –2 –1 0 1 2 3
Thick dot at – 2 shows weak inequality B.
Open dot at 0 shows strict inequality
The range is from – 2 to 0 –6 –5 –4 –3 –2 –1 0 1 2 3
Thus – 2  x < 0 ( E ) C.

2005/25 Neco Exercise 4.61 –6 –5 –4 –3 –2 –1 0 1 2 3


D.
x
–6 –5 –4 –3 –2 –1 0 1 2 3
–5 – 4 – 3 – 2 –1 0 1 2 3 4
2014/36 Neco ( Nov) Exercise 4.67
Which of the equations represent the number line
above?
A–4x3 B – 4< x < 3 C–4x<3 –6 –5 – 4 –3 –2 –1 0 1 2 3
Dx3 E–4<x3
Which of the following is illustrated in the diagram above, if
x varies over the set of real numbers?
A. – 5< x < 1 B. – 5< x  1 C. – 5 x  1 D – 5  x <1 E –5x

73
CHAPTER FIVE ii. 125
2
3
= ( 125 )
3
2

Indices and logarithm


The given examples on simple application were not fully
Indices solved; it is intentionally done to allow for step by step
Laws of indices understanding of the laws
1. xa × xb = xa + b but xa × yb  xa + b
Examples on simple application Indices involving numbers and their powers
i. 35 × 31 = 35 + 1 i.e 36 but 35 × 21  35 + 1 2015/33 Neco
ii. 72 × 72 = 72 + 2 i.e 74 but 72 × 82  72 + 2 −1
1 2
Simplify  
4
2. xa  xb = xa – b
A. − 2
1
B. 14 C. 1
D.2 E.4
OR xa = xa – b 2
Solution
xb
Applying indices law
Examples on simple application −1
i. 24  23 = 24 – 3 i.e 21 1
= (4 ) 2
2 1
 
ii. 53  51 = 53 – 1 i.e 52 4
= 2 D
iii. 34 = 34 – 2 i.e 32
32 2008/8 NABTEB (Nov)
2

3. x0 = 1 Find the value of 8 3

Examples on simple application A 1 /2 B 1/4 C 1/8 D 1/16


i. 20 = 1 Solution
ii. 100 = 1 Applying the law of indices
2
iii. (42)0 = 1 − 1
8 3
= 2
–a 8 3
4. x = 1 and
xa 1
= 2
−1 −a
 y x 1 1
  = Also   −1
= x most common x =
a
or (2 3 ) 3
x y  x x
−a
1 1
 y x
a = 2 i.e (B)
  =   2 4
x  y 2011/12 Neco
Examples on simple application 1

i. 10– 2 = 12 Evaluate (0.008) 3

10
A 0.15 B 0.8 C 0.5 D 0.2 E 0.1
ii. ( 54 )−2 = ( 54 )2 Solution
(1001 )−2
1
iii. = 1002  8 3
1

(0.008) =   3
 1000
5. ( xa )b = xab We raise items inside the bracket to power 3
Examples on simple application to cancel root 3 outside
i. ( 23 )2 = 23×2 i.e 26 1
 23  3
ii. ( 52 )2 = 52×2 i.e 54 =  3 
 10 
3
6. x a = x
1
a
 2 3 2
=  = i.e 0.2 D
Examples on simple application
1
 10  10
i. 16 2
= 16
1
3
2008/3 NABTEB (Nov)
ii. 27 3
= 27 4
1
4 Simplify (−8) 3
ii. 10000 4
= 10000
A – 32 B – 16 C 16 D 32

( x) b
Solution
=
b
7. x a a The power 4 which is even will cancel the minus sign
4 4 4

i. 4
3
2
= ( 4) 3
(−8) 3 = ( 8 ) 3 = (2 3 ) 3 = 24 i.e 16 ( C )
74
2014/7 NABTEB (Nov) Next, we seek to express the bracket item in power 3 that
 1

6 will cancel roots 3 outside it
The value of  4  is 4 2
  5  3 3
  = 2   
3 
1
A /8 B4 C6 D8 4 
Solution 3
2
2
6 5 3 5
 1
 6 3 3
= 2  = 2 
4 4  =4 4
= 4 2 2
= (2 ) 2
= 23 =8 D 4 4
 
  25 16 32
= 2 = 2  = i.e 17/25
16 25 25
2014/8 NABTEB
2
− 2006/1b (Nov)
Simplify 125 3
 15
Without using mathematical tables or calculator
A 1/25 B 3/25 C 1/5 D 3/5 1

2

2
−4
Solution Simplify : 16 2  8  3 3  81 3
2
− 15 Solution
125 3
 15 = 2 Express the items in 3 and 2 with their power
125 3 1

2

2 1

2

2

Raise denominator in power 3 16 2  8 − 4  3 3  81 3 = (2 4 ) 2  (2 3 ) − 4  3 3  (3 4 ) 3

15 15 −
2

8
= = = 2 2  2 − 12  3 3
3 3

3
2
(5) 2 Applying laws of indices
(5 ) 3
2  8
− − − 
3 5 3 2 − 12
= = D =2 3 3  3
5  5 5 2 8
− +
− 10
2008/1 Neco (Nov) =2 3 3 3
1 1
6
Simplify 1253  49 2  10− 1` =2 − 10
3 3

A 1/35 B 2/35 C 1/14 D 31/2 E 41/3 =2


− 10
 32
Solution
1 1 9 9
1 1 = i.e
(5 )  (7 )
3 3 2 2
2 10
1024
125  49  10 − 1` =
3 2
10
5  7 7 2014/10 Neco
= = i.e 31/2 D 1 2
10 2
9  27
2 3

2005/1b( i ) Neco Evaluate 2


3
− 64 3
1 4
Simplify   + 5(9 0 ) A 111/6 B 1 3/ 4 C 111/16 D 17/21 E 113/63
 16  Solution
Solution We should clear root 2 by raising item in it to power 2
Applying indices laws similarly; cube-root is cleared by raising its item to power 3
3 3 1 2 1 2

1 9 2  27 3 (3 )  (3 )
+ 5(9 ) = (16) + 5  1
4 4 2 2 3 3
0
  =
 16  2
3
2

Express 16 in power that will cancel root 4 64 3


(4 ) 3

3
3 3 2 27
= (2 ) + 5
4 4 = = /16 i.e 111/16 (C)
42
= 23 + 5 2008/3 Neco (Nov)
= 8 + 5 i.e 13
2 0  3− 2
2005/2a Evaluate 27  1
2 −

 64  3 4 2
Evaluate 2    A 48 B 12 C6 D 3/ 2 E 1/ 3
 125 
Solution
Solution
Express 27 in 3 and its powers, 4 in 2 and its power
Applying the laws of indices for minus power
2 2 2 0  3− 2 33  2 0  3 − 2
 64 

3  125  3 27  1
= 1
2   = 2  − −
 125   64  4 2
(2 2 ) 2

75
33  2 0  3 − 2 1 1 1
=
=   + 
25 2 32  2  64  3
2 −1   
 16   162   27 
Applying indices laws
Express all items in the power that cancel their roots. i.e
= 33–2  2 0+1 power 2 cancel square root, power 3 cancel cube root
= 3 1  21 1 1 1
=6 (C)  52  2  16  2  2  4 3  3
=  2  +
    3 
2008/1 Neco 4   81  2  3 
25  4− 2 Second bracket item has to be reduced before complying
Evaluate
2 −3  2 6 2 1 3 1
5  42  2 4
=   +      =
A9 B8 C3 D E 8/ 1/ 5 2 16 2 4 3
9 4 +  2  
Express all items in 3 and its powers 4  81  3 4 9  3
( − 2)
25  4− 2 25  2 2 = 5 + 4  4
= 4 9 3
2 −3  2 6 2 −3  2 6 Applying BODMAS to the fractions, division 1st
25  2− 4 5 4 3 5 1 15 + 4 19
= −3 = +  = + = = = 17
2  26 4 9 4 4 3 12 12 12
25 − 4 21
= −3 + 6 = 3 = 2 – 2 = 1/4 ( E ) 2013/13 Neco Exercise 5.1
2 2 1
2006/3 −2
2 1 Simplify (343 ) 3

25  25 3 6
A 7–2 B 7 –1 C 70 D 71 E 72
Simplify 7 1
− 2012/11 Neco Exercise 5.2
1 6  1 6
     −
1

5 5 27 2

1 1
Simplify
A 25 B1 C /5 D /25 2

Solution 81 3
Express all items in 5 and its powers 6

6

25 25 −2
E  
3
2 1 2 1 A 3 25 B3 25
C3 6
D 36
(5 2 )  (5 ) 2 6 4
25 3  25 6 3

7 1
= 7 1 2008/2 Neco (Nov) Exercise 5.3
− − 3
−1 −1 6
1 6 1 6 (5 ) 6
 (5 )
     Simplify 16 4  2 4  32 − 1`
5 5 A0 B2 C4 D 10 E 20
4 2

5 53 6
2010/6 NABTEB (Nov) Exercise 5.4
= 7 1 1
− −
5 5 6 6  4  2
Simplify  
Applying indices laws  25 
4 2

8−2 6 A 3 1/ 2 B 2 1/ 2 C 4 1/ 2 D 6 1/ 3
5 3 6 5 6
5 6
= = 7 −1
= 6
= 1 B
7  1
+ − 
2013/10 Neco Exercise 5.5
5 6  6 5 6
5 6
3 4  27 − 3
2006/34 Neco Evaluate
1 1
3− 4

9  32  2  27  3 A 1/ 3 B3 C9 D 18 E 81
Evaluate: 1 +     
16  162   64 
7 1 7
2007/4 Neco Exercise 5.6
A /12 B 1 /2 C 1 /12 D 191/108 E 27/27 1 2

Solution 16 4
+ 27 3
By proper presentation Evaluate 3

1 1
9  32  27  2

3
4 2

1 +    = A 1 3/ 8 B 21/4 C 51/2 D 9 E 76
16  162   64 
1 1 1
− 2012/39 Exercise 5.7
 25  2  32  2  27  3
  +    −
1 1
 16   162   64   27  3  4  2
−A A Solve:     
 y x  125  9
By laws of indices   =   10 9
x  y A /9 B /10 C 2/5 D 12/125
76
Indices in algebraic terms and algebraic powers 2014/ 11c NABTEB (Nov) Adjusted
2010/18 Neco 3x −1  9 x + 1  27 − x
Simplify
Simplify 27a
3 −6 815x − 3
Solution
A 3a B 3a2 C 3a– 2 D 9a – 2 E 9a2
The lowest base here is 3 and we can change 9, 27 and 81 to
Solution
3 and its power
Express 27 in 3 and its powers
1 3x −1  9 x + 1  27 − x 3x −1  32 ( x + 1)  (33 ) − x
3 −6 3 −6 3 =
27a = (3 a ) 815x − 3 3 4 ( 5 x − 3)
3 −6
3x −1  32 x + 2  3− 3 x
= 3 a 3 3
= 3a– 2 (C) =
320x −12
2008/10
Applying law of indices to the powers at numerator
Simplify: 3 27 x 3 y 9 3x −1+ ( 2 x + 2) + ( −3x )
A 9xy3 B 3xy6 C 3xy 3
D 9y 3 =
320x −12
Solution
1 3x −1+ 2 x + 2 −3x
=
3 3 3 3 9
27 x y = (3 x y ) 3
9
320x −12
3 3 9 31
= = 31  3 – (20x – 12)
= 33 x 3 y 3 3 20 x − 12

= 3xy3 (C ) = 31  3 – 20x + 12
2014/35 31 + 12 3 13
= =
82  4n +1 3 20x 3 20x
Simplify :
2 2n  16
A 16 B8 C4 D1 2008/14 Neco (Nov) Exercise 5.8
Solution Simplify 56a– 6  7a– 4
1 A 8a– 10 B 8a– 8 C 8a– 2 D 8a2 A 8a10
8 4
2 n +1
8  4 2 n +1
 2
=  
 2  16
2n
2 2n  16  2007/7 Exercise 5.9
Our target will be to make each item that cannot cancel 3x 3
Simplify:
out in powers of 2 (3 x) 3
1
8  4  4
2 n 1
 2 A 1 B 1 /3 C 1 /9 D 1/27
=  
 2 4
2n 2
 2012/7 f/maths Exercise 5.10
1 3n + 1
x
 8 2  2 2n  2 2  2 Simplify
=   2n + 5 2
x x 2n − 3
 2 4
2n 2

A. 0 B. - 12 C. 1 D. 10
2 will cancel out, while power ½ will cancel out all power 2
2n

8  2
= = 4 (C) 2014/11c NABTEB ( Nov ) Exercise 5.11
4
3 x − 2  9 x + 1  27 − x
Simplify
2014/3 Neco ( Nov ) 815 x − 3
1
−1
Simplify: (27 x  64 x )  (3x)
3 6 3

A 3x B 4x C 4x2 D 4x3 E 12x 3


Solution
1
1
(27 x  64 x )
3 6 3
(27 x 3  64 x 6 )  (3x) −1 =3
3x
We seek to raise items under power 1/3 to power 3
1

(3 x  4 x ) 3 3 3 6 3
=
3x
3x  4 x 2
= = 4x2 (C)
3x
Note that : power 6  3 is 2
77
Indices involving equations 2006/21 Neco
1
There are cases where the resulting indices of a given Solve the equation: 3 2 x − 1 =
problem become equation. These equations are 27 x
sometimes: A–3 B – 1 C – 1 /5 D 1/5 E3
- simple linear or simultaneous linear Solution
- quadratic in nature 1
Whatever the equation may be, we are required to solve 32x −1 =
27 x
accordingly.
Express RHS in 3 and its power to align with LHS
Simple linear cases 1
32x −1 =
2005/12 3 3x
Solve the equation 27 = 85 – x 32x −1 = 3 – 3x
A 5 B 8 C 3 D 15 Since they are in the same base, we equate powers
8 3 2 4
2x – 1 = – 3x
Solution 2x + 3x = 1
27 = 8 5 – x 5x = 1
Express RHS in 2 and its power x = 1/ 5 ( D )
27 = 2 3(5 – x)
2006/19 Neco (Nov)
Since they are in the same base, we equate powers
7 = 3(5 – x) 5 x −3
If = 1 , find the value of x
7 = 15 – 3x 125 3x − 4
3x = 15 – 7 A 1 1/ 8 B 8/ 9 C 7/ 8 D 5/ 8 E0
x = 8 (B) Solution
3
5 x −3
=1
2007/21 Neco 125 3x − 4
If 27x – 2 = 729, find x We cross multiply
A3 B4 C5 D6 E7
5x – 3 = 125 3x – 4
Solution
Next, we express RHS in 5 and its power to be like LHS
27x – 2 = 729
5x – 3 = 5 3(3x – 4)
Our target will be to raise both sides to same base
Since they are in the same base, we equate powers
number and its power.
x – 3 = 3(3x – 4)
33(x – 2) = 36 x – 3 = 9x – 12
Since they are in the same base, we equate powers 12 – 3 = 9x – x
3(x – 2) = 6 9 = 8x
x–2 = 2 9
/8 = x i.e 11/8 (A)
x = 2+2
x = 4 (B) 2006/3a NABTEB (Nov)
3x
8x
2005/ 14 If 4 2 = , find x
1
2 4
 1 3 Solution
If 3 =  y
 , find y
 27  3x
8x
A2 B 1 /2 C – 1/2 D–2 4 2
=
4
Solution Proper presentation shows
2 1
1
 1  3 3x
8 2
x
3 = 
y
4 2
=
 27  4
Express RHS in 3 and its power to align with LHS Express all items in 2 and its power for uniformity
1 2 1
−3 x
3 = (3 )
y 3 3x
(2 3 ) 2
1
(2 ) 2 2
=
22
3 = 3 –2
y
3x
Since they are in the same base, we equate powers 2 3x
=2 2
 2 −2
1 3x
= –2 −2
y 2 3x
=2 2

1 = –2y Since they are in the same base, we equate powers


– 1/ 2 = y (C) 3x
3x = −2
2
78
3x Applying the law of indices
3x − = −2 21 – y + y – 1 – (y + 2) = 26 – 9y
2
21 – y + y – 1 – y – 2 = 26 – 9y
6 x − 3x
= −2 2 – y – 2 = 26 – 9y
2 Equating powers
3x = – 4 – y – 2 = 6 – 9y
x = – 4/3 9y – y = 6 + 2
2008/7 Neco (Nov) 8y = 8
m+2 y = 1-
4 9
If   = , find the value of m 2010/2b Neco
3 16 x 3 3
A–4 B–2 C0 D2 E4 If 8 = 2  4 , find x
2 8 4

Solution Solution
m+2 3 3
4
x
9
  = 82 = 28  4 4
3 16 Express all items in 2 and its power
Our target is to make RHS similar to LHS x
3  3 3
2 
m+2
4 3 2 2 2
=2 28 4

  = 3x 3 6
3 42 2 2
=2 28 4
m+ 2 2
4 3 Applying the law of indices to RHS power items
  =   3x 3 6 3 + 12
3 4 2 2 = 28
+
4
i.e 2 8
We are near success, next invert ¾ to 4/3 and 3x 15
reflect it in the power 2 2
=2 8
m+2 −2
4 4 Since they are in the same base we equate powers
  =   3x 15
3 3 =
2 8
Since they are in the same base, we equate powers
15  2
m+2 = –2 3x =
8
m = –2–2
Multiply both sides by 1/3 to get x
m = –4 (A)
1 15  2 1
2008/30 Counter example 3x × = 
3 8 3
If 2n = 128, find the value of (2n – 1) (5n – 2)
x = 5/4
A 5(106) B 2(106) C 5(105) D 2(105)
Solution
2011/46
First, we solve for n from
2n = 128 If 27x = 9y, find the value of x/y
A 1/ 3 B 2/ 3 C 11/2 D3
Express the right hand side in 2 and its powers
Solution
2n = 27
Equating powers
27x = 9y
Raise both sides to 3 and its powers
n = 7
Next, (2 n–1
) (5 n–2
) = 7–1
(2 ) (5 )7–2 33x = 32y
Now they are in the same base, we equate powers
= (26) (55) 3x = 2y
= 2  25  55 To get x/y, divide both sides, by y
= 2(2  5)5 3x 2y
= 2(105) D =
y y
2009/1b
3x
21 − y  2 y −1 = 2
If y +2
= 8 2 − 3y , find y y
2 Next, we divide both sides by coefficient of x/y i.e 3
Solution x 2
21 − y  2 y −1 = (B)
= 8 2 − 3y y 3
2 y +2
Express RHS in 2 and its powers 2014/6a NABTEB (Nov)
2 1− y
2 y −1 Solve the equation: 16 × 8x = 32 × 42x – 1
y +2
= 2 3( 2 − 3y ) Solution
2
16 × 8x = 32 × 42x – 1
Here we cannot work in 4 or 8 or 16 and their powers but in
2 and its power
79
24 × (23)x = 25 × (2 2)2x – 1 2010/18 Exercise 5.16
24 × 23x = 25 × 24x – 2 5n + 3
Applying law of indices to the powers Given that 2n − 3
= 5 0 , find n
25
2 4 + 3x = 2 4x – 2 + 5 An=1 Bn=2 Cn=3 Dn=5
24 + 3x = 24x + 3
Now they are in the same base, we equate powers 2010/12 Neco Exercise 5.17
4 + 3x = 4x + 3 Find the value of x, given that
1
4 – 3 = 4x – 3x of 92x = 27x
3
1= x
2014/1 Neco A–2 B–1 C0 D1 E2
1 2009/28 Neco (Nov) Exercise 5.18
If 4m = , find the value of 2m2
64 1
Solve the equation 32x – 1 =
A – 18 B 16 C 18 D 32 E 256 243
Solution A–2 B2 C3 D4 E5
1
4m = 2009/2 NABTEB (Nov) Exercise 5.19
64 Given 5 (x + 1) = 125, find the value of x
We should raise both sides to 4 and its powers A6 B3 C2 D1
1
4m = 3 2009/31 Exercise 5.20
4 1
4 = 4– 3
m If 9
2x
= (27 x ) , find x
Same base achieved, next we equate powers 3
m = –3 A 2 B1 C–1 D–2
Thus 2m2 = 2(– 3) 2
= 2× 9 Simultaneous linear in nature
= 18 ( C ) 2008/6a
2014/ 1a NABTEB (Nov) 1
1
n If 2 x + y = 16 and 4x – y
= , find the value of x and y
Find n in the equation 36 ×   = 6  216
n −1 32
6
  Solution
Solution 1
2x + y = 16 and 4x – y =
1
n
n −1
32
36 ×   = 6  216 Express both equations in 2 and its power
6
1
We raise items on both sides to 6 and its power 2x + y = 2 4 and 22 (x – y) = 5
62 × 6– n = 61 × (63)n – 1 2
2x – 2y
Applying law of indices to the powers 2 x+y 4
= 2 and 2 = 2–5
62 – n = 61 × 63n – 3 Same base achieved, next we equate powers
62 – n = 63n – 3 + 1 x + y = 4 -------- (1)
2x – 2y = –5 ------- (2)
62 – n = 63n – 2
From (1) y = 4 – x , substituting into (2)
Same base achieved, next we equate powers
2x – 2y = –5 becomes
2 – n = 3n – 2
2x – 2(4 – x) = –5
2 + 2 = 3n + n
2x – 8 + 2x = –5
4 = 4n
4x = –5 + 8
1 = n
4x = 3
2013/2 Exercise 5.12 x = 3/4
Substitute x = 3/4 into (1)
If 9(2 – x) = 3, find x
A1 B 3/ 2 C 2 D 5/ 2 ¾ + y= 4
Thus y = 4 – 3/ 4 i.e 13/4
2014/11 NABTEB (Nov) Exercise 5.13
If 9x – 1 = 27, the value of x is 2011/2b
A 21/2 B 21/4 C2 D3 If 9 (1 – x) = 27y and x – y = –11/2, find the value of x + y
2012/13 Neco Exercise 5.14 Solution
Find the value of x in the equation 4x = 32 9(1 – x) = 27y
A 1.5 B 2.0 C 2.5 D 4.0 E 8.0 Raise both sides to 3 and its power
32(1 – x) = 33y
2009/10 (Nov) Exercise 5.15
Same base achieved, next we equate powers
If 9(2x – 1) = 27, find x 2(1 – x) = 3y
A5 B3 C 5/ 4 D 4/ 5 2 – 2x = 3y

80
2 = 3y + 2x Raise the RHS to a number in power of 2 to align with LHS
i.e 3y + 2x = 2 ----- (1) 32
Also given: x – y = – 11/2  – 3/2 ------ (2) (x + 1)2 =
52
Multiply (2) through by 2 to clear fractions 2
2x – 2y = – 3 ------ (3) 3
( x + 1) 2 =  
– (2x + 3y = 2) 5
– 5y = – 5 Since they are in the same power, we can equate base
y = 1 x + 1 = 53 or x + 1 =  53
Substituting for y = 1 into 2x + 3y = 2
2x + 3(1) = 2 x = 3
5 –1 or x = –1  53
2x = 2 – 3 = – 2 /5 = – 2/5 or – 8/5
2x = – 1 2011/4 f/maths
x = – 1/2 Thus x + y = – 1/2 + 1 i.e 1/2 Solve: 2(2y + 2) – 9(2 y ) = –2
1996/5 PCE Exercise 5.21 Solution
If 5x– y = 125 and 34y – x = 243, Find the value of 2x – 2y Applying law of indices
A.225/3 B.21/3 C.2 –1/3 D.2 –25/3 2(2y + 2) – 9(2 y ) = –2 becomes
2 × 22 – 9(2 y ) = –2
2y
2014/1b ( Nov ) Exercise 5.22a i.e 4( 2y )2 – 9(2 y ) = –2
Solve simultaneously, the equations
Let 2y = p
1
5x – 4y = 6 and 33( y − x ) = 4p2 – 9p = –2
27 i.e 4p – 9p + 2 = 0
2
1978/11a Exercise 5.22b Factorizing : 4p2 – 8p – p + 2 = 0
27 x 4p(p – 2) – 1( p – 2 ) = 0
Calculate the values of x and y if = 9
81x + 2 y (4p – 1)( p – 2 ) = 0
and x + 4y = 0 4p – 1 = 0 or p – 2 = 0
4p = 1 or p =2
Quadratic in nature 1
p = 4 or p = 2
2009/4 Neco (Nov) Changing back to our original variable
x 27 x
2
p = 14 implies 2y = 14 or p = 2 implies 2y = 2
Solve the equation 3 =
9 i.e 2y = 2– 2 or 2 y = 21
A – 1, 2 B 1, 2 C 1, – 2 D –1, – 2 E 0, 3 y = – 2 or 1
Solution
2013/2a Neco f/maths Exercise 5.23
2 27 x Solve 52x + 4 × 5 x + 1 – 125 = 0
3x =
9 1997/6 PCE Exercise 5.24
Express the RHS in 3 and its power Solve for x in the equation 32x – 12(3x) + 27 = 0
2 33x A.3, 9 B.2, 1 C. –3, 9 D. –1, 2
3x = 2
3 1999/9a (Nov) f/maths Exercise 5.25
Applying the law of indices Solve the equation:
2 32x + 2 – 10(3x + 1) + 9 = 0
3 x = 33x – 2
Same base achieved, next we equate powers
x2 = 3x – 2 Other cases
x – 3x + 2 = 0
2
2012/10 Neco
Factorizing 1 1

x2 – 2x – x + 2 = 0 Given x 3 = (0.25) 6 , find x


x(x – 2) – 1(x – 2) = 0 A (0.25)3 B 0.05 C 0.50 D 2.00 E 5.00
(x – 2)(x – 1) = 0 Solution
x – 2 = 0 or x – 1 = 0 1 1

x = 2 or 1 B x 3 = (0.25) 6
2009/11 Neco (Nov) Note that 0.25 is same as 25/100
1
9 1
Solve (x + 1)2 =  52 6
25 x3 =  2 

A – 2/5, 8/5 B 2/5, – 8/5 C – 2/5, – 8/5  10 
D – 2/5, – 5/8 E 2/5, 5/8 1
2
1
1
 5 6  5 3
Solution x3 =   i.e x 3 =  
9  10   10 
(x + 1)2 =
25 The same powers achieved, next we equate bases
Thus x = 5/10 i.e 0.5 ( C )
81
2005/1a 2009/13 Exercise 5.26
If 2 x + 2 (x – 1 ) = 48, find the value of x If (2x + 3)3 = 125, find the value of x
Solution A1 B2 C3 D4
2x + 2(x – 1) = 48 2006/6 (Nov) Exercise 5.27
Factor out 2x at the RHS 1
2x(1 + 2– 1) = 48 Find x if 5 x 3 =
25
2x(1 + 1/2) = 48 A 1/25 B 1/5 C1 D5
2x( 32 ) = 48
2006/7b (Nov) Exercise 5.28
2
Multiply both sides by to clear fraction 8
3
Solve the equation (3x – 1)3 =
x
2 = 48 × 2 27
3
2x = 16 × 2
Theory of Logarithm
Express the RHS in 2 and it powers
This aspect of logarithm deals with applying the relationship
2 x = 25 between indices & logarithm and laws of logarithm to solve
Same base achieved, next we equate powers problems. Here, we will not use log tables.
x = 5
2005/29 Neco Relationship between Indices and
1 x Logarithm
Solve the equation 0.2 =
25 If bx = N then ,
A–5 B–2 C1 D2 E5 x = log bN
Solution Note: Students should observe that
1 both indices and logarithm make use
0.2x =
25 of the same base when we are
We raise both sides to the same base converting from one form to another.
x
 2 1
  = Examples on simple applications
 10  25
( i ) Given that 102 = 100,
It can’t work, but /10 is 1/5
2
x
Write the expression in logarithm form.
1 1 Solution
  =
5 25 2 = log10 100
Ok now, we can express them in 5 and its power
( ii ) If 26 = 64,
5–x = 5–2 Write the expression in logarithmic form.
Same base achieved, next we equate powers Solution
–x = –2 6 = log2 64
x = 2 (D)
( iii ) Express 4x = 256 in logarithmic form.
2014/31 Neco (Nov) Solution
3
x
Solve for x if 3x = 81 5 4 = 256
x = log4 256
A. 3 B. 9 C. 27 D. 81 E. 243
Solution ( iv ) Write in logarithmic form 7–2 = 1
3 49
3x = 81
5
Solution
Divide both sides by 3 –2 = log7 1/49
3

x 5 = 27 Laws of Logarithm
To get x we raise both sides to the opposite of it power (1) Log b (MN) = log b M + log bN
5
 3 3 5 Examples on simple applications
x5  = (27) 3
  a) Log630 = log6 (6 x 5)
  = log66 + log65
Recall that 27 is 3 3
3
5 b) log272 = log2 (8 x 9)
x = (3 ) 3 = log28 + log 29
= 35 i.e 243 ( E )
c) log39 + log327 = log3 (9 x 27)
= log3243
and not log3 9 + log3 27 =Log3 (9 + 27)
82
(i) log10 100 = log10102 2009/30 NABTEB (Nov)
= 2log1010 Evaluate log a a
3

A0 B1 C3 Da
(ii) log7343 = log773
Solution
= 3 log 77
Applying law of logarithm
(iii) ½ log5 25 log a a3 = 3log a a
= log5 251/2 = 3 (C)
(2) log b ( M/N ) = log bM – log bN 2008/5 Neco (Nov)
Without using tables, evaluate 2 + log 1020 – log 102
Examples on simple applications A2 B3 C4 D5 E6
(a) log10 ( 100/3 ) = log10 100 – log103 Solution
Since the log here is log 10, we express 2 in log 10
(b) log6 36 = log636 – log6 17 2 + log 1020 – log 102 = log 10102 + log 1020 – log 102
17
102  20
(c) log3 81 – log3 9 = log3 (81/9) = log 10
2
(3) logb Nk = k log b N = log 10 102 × 10
Special cases under (3) = log 10 103
(i) logbb = logbb1 = 3log1010
= 1Logbb = 1 = 3 (B)
i.e logbb = 1 2009/29 Neco (Nov)
Stated as; Any logarithm to its base is one. Simplify log x8 – log x2 + log x 4
Examples A log x2 B 2log x2 C 4log x2 D 8logx2 E 16log x2
(a) log1010 = 1 Solution
(b) log33 = 1 Applying logarithm laws
(c) log66 = 1 8 4
log x8 – log x2 + log x 4 = logx
2
(ii) log b1= logbb0 (Since b0 = 1) = log x 16
= 0 logbb
= log x 24
=0 i.e log b 1 = 0
= 4log x2 ( C )
Stated as: Log of one to any base is zero 2005/ 1b (ii) Neco
Note:
The examples given under relationship between indices Simplify: log 10 1000
& logarithm were not fully solved; this was deliberately Solution
( )
1
done to allow for step-by-step comprehension of the
log 10 (1000) 2 = log 10 10
1 3 2
stated concepts (principles). Examples of fully solved
3
problems applying the earlier stated principles are given
= log 10 10 2
below:
3
= 2 log 1010
Simplification in logarithm = 3
i.e 11/2
2
2009/7 NABTEB (Nov)
Simplify log 4 + log 8 2006/4 Neco (Nov)
A log32 B log 12 C log 23 D log 1/2 Evaluate log1045 + log109–1 – log102–1 without using table
Solution A 10 B5 C2 D1 E 1/ 2
Applying laws of logarithm Solution
log 4 + log 8 = log 4  8 Applying the laws of logarithm
= log 32 (A) 45  19
log1045 + log109 – log102
–1 –1
= log 10
2006/4a NABTEB (Nov) 1
2
Simplify log 354 + log315 – log 310 By simple arithmetic, we have
Solution 45  2
Applying laws of logarithm = log 10
9
log 354 + log315 – log 310 = log3 54  15
10 Some terms will cancel out
= log3 81 = log1010
Next, express 81 in powers of 3 = 1 (D)
= log3 34
= 4log3 3 2005/4
= 4 Simplify the expression log1018 – log102.88 + log1016
A 31.12 B 3.112 C2 D1
83
Solution 2011/7 Neco
Applying the laws of logarithm  125  5
18 16 Simplify log 10   − 2 log 10  
log1018 – log102.88 + log1016 = log10  8  4
2.88
A 125 B 16 C 10 D4 E1
288
= log10 Solution
2.88 By laws of logarithm
Move decimal point forward twice in denominator and numerator 2
28800  125  5 125 5
= log10 log 10   − 2 log 10   = log 10 − log 10  
288  8  4 8  4
= log10 100  125 25 
= log10 102 = log 10   
= 2log 1010  8 16 
=2 (C) Changing division to multiplication
 125 16 
2008/32 Counter example = log10   
 8 25 
log 8
Simplify Some terms will cancel out
log 4 − log 2
= log1010
1
A 2/ 3 B 2 log2 C 3/ 2 D log 2 = 1 (E)
Solution
The problem at hand seems not to follow our normal pattern 2009/14 Neco (Nov)
log 8 log 8 Simplify: log10 65 + log10 2 − log10 13
=
log 4 − log 2 log 2 2 − log 2 A1 B 1/ 5 C 1/4 D 1/2 E 1/ 3
Solution
log 8 Applying logarithm laws
=
2 log 2 − log 2
65  2
log 8 log10 65 + log10 2 − log10 13 = log 10
= 13
log 2
1
Simplify 65 into 13  5
log 8 2
13  5  2
Next, numerator = = log 10
log 2 13
= log 10 5  2
1 3

log( 2 3 ) 2 log 2 2
= = 1

log 2 log 2 = log 10 10 i.e log10 10 2


1 1
3
2
log 2 = 2 log1010 = 2 D
=
log 2
3 2014/4a (Nov)
= /2 (C)
Without using tables or calculator, evaluate:
2010/11 Neco
log 3 − log 2  75  5  100 
log10   − 2 log10   + log10  
Simplify  10  9  243 
log 9 − log 4
Solution
A – 2.0 B – 0.5 C 0.0 D 0.5 E 2.0 Applying logarithm laws
Solution
 75  5  100 
Applying laws of logarithm log10   − 2 log10   + log10  
log 3 − log 2 log 32  10  9  243 
= 2
log 9 − log 4 log 94  75  5  100 
= log10   − log10   + log10  
Proper observation shows the denominator is a multiple  10  9  243 
of numerator  75 5
2
100 
log 32 = log10      
=  10 9 243
log ( 32 ) 2
Changing  to × in the bracket, we have
log 3
 75 9  9 100 
= 2
= log10    
2 log 3
2  10 5  5 243 
= 1/2 i.e 0.5 (D) = log10 10
=1

84
2008/5 Neco (Nov) Change of base
Without using tables evaluate 2 + log 1020 – log 102 If any given base is not conducive or good enough to
A2 B3 C4 D5 E6 work with , we can change base
Solution
Since the log here is log 10, we express 2 in log 10 logbN = logcN
2 + log 1020 – log 102 = log 10102 + log 1020 – log 102 logcb
102  20 Here the new base is not common to the numbers given but it suits
= log 10 our work.
2 There are cases where the number whose logarithm we are looking
2
= log 10 10 × 10 for will be used :
= log 10 103 logbc = logcC = 1
= 3log1010 logcb logcb
= 3 (B)
2008/ 6 Neco
2011/17 Exercise 5.29 Find the value of log 3
81
log 27
Simplify A 81 B 16 C8 D3 E 3
log 81 Solution
A3 B2 C 3/2 D 3/ 4 Here we change base 3 to 3 as shown below:
2006/1 (Nov) Exercise 5.30 log 3 81
log 27 log 3
81 =
Simplify log 3 3
log 9
log 3 3 4
1
A 1 /2 B3 C log 3 D log ()
3
2
= 1
log 3 3 2
2012/6 f/maths Exercise 5.31
Evaluate: log 10 (13 + 14 ) + 2 log 10 2 + log 10 ( 73 )
4 log 3 3 1 2
= = 4 2 =4 1 i.e 8 ( C )
2 log 3 3
1
A. –3 B. 0 C. 5/6 D. 1
2005/1a Exercise 5.32 2005/9 Neco
1
Evaluate without using the mathematical table or Calculate the value of log2x if logx 4 = 2

calculator, log 10 30 – log 10 6 + log 10 2 A1 B2 C3 D4 E8


Solution
We will change base but first we align logx4 to log2x
logx 4 = 12
logx22 = 12
2log x2 = ½
Multiply through by ½
1
log x2 = 4
log x x
Now, we can change base as : log2 x =
log x 2
1
=
log x 2
1 1
Substituting for log x2 value = i.e 1  4
1
4
4
= 1 1 = 4 D
2013/11 Neco
Simplify the expression log 43  log 34 + log 232
A1 B2 C3 D6 E7
Solution
We change base in log 34 to align with log 43
log 4 4
log 43  log 34 + log 232 = log43  + log 225
log 4 3
1
= log43  + 5log 22
log 4 3
= 1 + 5 = 6 (D)
85
2012/11a Neco Substituting for the given value
Given that log 327 = log 1 64 + n log 3 1
81
= 3(0.477)
4 = 1.431  1.4 to 2 s.f (B)
find the value of n
Solution 2009/1 NABTEB (Nov)
A look at the problem shows that log 1 64 will be an If log2 = 0.301, what is log64?
4 A 1.506 B 1.606 C 1.706 D 1.806
obstacle, so we change its base Solution
log 1 64 = log 4−1 64 We express log64 in log2 and its multiples
4 log64 = log26
log 2 64 = 6log2
= and our working proper Substituting for the given value
log 2 4 −1
= 6(0.301)
n
log 2 2 6 1 = 1.806 (D)
log 3 3 =3
−2
+ log 3  
log 2 2  81 
2006/10b NABTEB (Nov)
6 log 2 2 Given that log10 2 = 0.3010 and log 107 = 0.8451,
3 log 3 3 = + log 3 3−4n
− 2 log 2 2 evaluate, without the use of table log10 3.92
Solution
3 = −62 + – 4n log33 Our target will be to express log10 3.92 in log10 2 and log 107
3 = – 3 – 4n with its multiples.
4n = –3–3 log10 3.92 = log 10 100
392

n = – 6 /4
= log 10 392 – log 10 100
Note: 392 is 28  14 i.e 2  2  7  2  7
2015/34 Neco Exercise 5.33
For better working 8  49
Evaluate log 3 5 125
= log 10 ( 8  49) – log10 102
A. 3 B. 5 C. 6 D. 8 E. 9 = log 10 8 + log 10 49 – log10 102
2015/1 Neco Exercise 5.34 = log 10 23 + log 10 72 – log10 102
Evaluate log327 + log93 – log322 = 3log 10 2 + 2log 10 7 – 2log10 10
A. –1/5 B. 1/2 C. 3 D. 33/10 E. 31/2 Substituting for the given values
2015/8c Neco Exercise 5.35 = 3(0.3010) + 2(0.8451) – 2
Without using mathematical tables or calculator, = 0.903 + 1.6902 – 2
evaluate: log16 0.25 = 2.5932 – 2 = 0.5932
2015/4 f/maths Exercise 5.36 2006/7b Neco
If log3x = log93, find the value of x If log 2 = 0.3010 and log 7 = 0.8451, evaluate,
1 1 1
A. 3
2
B. 3 2 C. 3 3 D. 2 3 without using mathematical table or calculator
( i ) log 0.2 ( ii ) log 352
2013/3 Neco f/maths Exercise 5.37
Solution
Find without using tables, the value of
( i ) Our target will be to express 0.2 in the form of 2 or 7
 8  and their multiples
log 2    log 1 10
3 27
  100
2
log 0.2 = log
A. –21/2 B. –11/2 C. –2/3 D. 11/2 E. 21/2 10
= log2 – log10
Substituting for log 2, Note log10 is 1
Substitution in Logarithm = 0.3010 – 1
Some logarithmic problems involve substituting for a = – 0.6990
2
given log value. By this, we are then constrained or ( ii ) log 35 = 2log35
limited to solving the problem by simplifying to get Expressing 35 in 2 and 7
either exactly the given log value or its multiples. = 2[log (5  7) ]
= 2[log 5 + log 7]
2014/5 NABTEB (Nov) 10
If log 103 = 0.477, evaluate log1027 correct to 2 s.f We don’t have 5, but 5 is
2
without using log tables 10
A 0.55 B 1.4 C 1.43 D 1.8 = 2 [log + log 7]
2
Solution = 2[log 10 – log 2 + log 7]
Our target will be to express log 1027 in log 103 and Substituting
its multiples = 2 [1 – 0.3010 + 0.8451]
log1027 = log1033 = 2 [1.5441]
= 3log103 = 3.0882
86
2006/12 Neco special case Solution
If 10n = 0.63 and log1063 = 1.8, find the value of n  28 
A – 2.63 B – 1.80 C – 0.20 D 0.20 E 2.63 log   = log28 – log3
 3 
Solution
Next, we express 28 in 2, 3 and 7
Here we are dealing with indices and logarithm
10n = 0.63 = log(4  7) – log3
Changing from indices to logarithm = log22 + log7 – log3
log 100.63 = n = 2log2 + log7 – log3
What we were given is log 1063, so we convert 0.63 to 63 Substituting for the given values
63 = 2x + z – y (B)
log 100.63 = log10
100
= log1063 – log10100 2013/1b
= log10 63 – log10102 i.e log1063 – 2log10 10 Given that log 102 = 0.3010 and log103 = 0.4771 evaluate
Substituting correct to 2 significant figures and without using table or
= 1.8 – 2 calculator, log10 1.125.
= – 0.2 ( C ) Solution
2008/1a Our target will be to express 1.125 in terms of 2 and 3
Without using calculator or table, find the value of 1125
log10 1.125 = log 10
log 3.6 given that log 2 = 0.3010, log 3 = 0.4771 100
and log 5 = 0.6990 By log law of division
Solution = log101125 – log10100
Our target will be to express 3.6 in 2, 3 and 5 and their multiples But 1125 is 5  225
36 is 5  3  75
log 3.6 = log
10
is 5  3  3  25
= log 36 – log 10 i.e 53  32
= log (4  9) – log 10 Case of 3 is ok, but we have 5 in place of 2;
= log 4 + log 9 – log 10 10
= log 22 + log 32 – log 10 the way out is as 5
2
= 2log 2 + 2log3 – log 10 = log10 (53  32) – log10102
Substituting for the given values = log10 53 + log1032 – log10102
= 2(0.3010) + 2(0.4771) – 1 = 3log10 5 + 2log103 – 2log1010
= 0.602 + 0.9542 – 1 10
= 1.5562 – 1 = 3[ log10 ] + 2log103 – 2log1010
2
= 0.5562 = 3log10 10 – 3 log 102 + 2log10 3 – 2
2009/6a Substituting for the given values
If log5 = 0.6990, log 7 = 0.8451 and log8 = 0.9031, = 3 – 3(0.3010) + 2(0.4771) – 2
 35  49  = 3 – 2 – 0.903 + 0.9542
evaluate log  
 40  56  = 1 – 0. 0512
Solution = 0.9488
First, we apply simple arithmetic to simplify the bracket
as 2006/13a Exercise 5.38
If log102 = 0.3010 and log107 = 0.8451, evaluate without
 35  49   40  using logarithm table or calculator, log10 35
log   = log  35  49  
 40  56   56 
 56 
= log  35  49  
 40 
= log (7  49  7)
= log 74
= 4log 7
Substituting
= 4(0.8451)
= 3.3804

2009/3 Counter example


If log 2 = x, log 3 = y and log 7 = z,
 28 
find in terms of x, y and z, the value of log  
 3 
A 2x + y – z B 2x + z – y C x + y – 2z
Dx+z–y
87
Relationship between indices and logarithm Now they are in the same base, we equate powers
2014/7 NABTEB 3(x + 2) = 5
If log n 1 = x, what is the value of x? 3x + 6 = 5
A–2 B–1 C0 D1 3x = 5 – 6
Solution 3x = –1
log n1 = x x = – 1/3 (B)
Changing from log to indices
nx = 1 2005/28 Neco
By law of indices Solve the equation log 2 x2 = – 6
x = 0 (C) A – 32 B – 12 C 1/64 D 1/ 8 E8
Since only n0 = 1 by indices Solution
log 2 x2 = – 6
2014/28 NABTEB Changing from logarithm to indices
If log ax = P, express x in terms of a and p x 2 = 2– 6
Next, we seek to align RHS power to LHS
A x = a + p B x = a/p C x = p D x = ap
x2 = (2–3)2
Solution
Since they are in the same power, we equate terms
log ax = P x = 2–3
Changing from log to indices x = 1/8 ( D )
ap = x ( D )
2005/2 (Nov)
2013/13 Neco Given that 10x = 3, express log10300 in terms of x
If log 216 = 4x – 1, what is the value of x? A 2x Bx+2 C 2x + 1 D x – 2
A 3/ 4 B 4/5 C1 D 5/4 E 4/3 Solution
Solution
10x = 3
log 216 = 4x – 1
Changing from indices to logarithm
Changing from logarithm to indices
log10 3 = x
24x – 1 = 16 Next, we seek to express log10300 in form of log103
Express the RHS in 2 and its powers
log10300 = log10 (3  100)
24x – 1 = 24 By law of log multiplication
Since they are in same base, we can equate powers = log103 + log 10100
4x – 1 = 4 = log103 + log 10102
4x = 4 + 1 = log103 + 2log 1010
4x = 5 Substituting for x
x = 5/4 ( D ) = x + 2 B
2005/24 2009/32 (Nov) Exercise 5.39
If log 9x = 1.5, find x If log 2 p = 3, find the value of p
A 36 B 27 C 24.5 D 13.5 A3 B6 C8 D9
Solution
log 9x = 1.5 2015/6 Exercise 5.40
Changing logarithm to indices
Given that logx64 = 3, evaluate xlog28
91.5 = x A. 6 B. 9 C. 12 D. 24
Next, we express 1.5 in fractional form 1.5 is 15/10 i.e 3/2
3
Thus 9 2 = x
(3 )
3
2 2
= x
3
3 = x
i.e 27 = x (B)

2006/34 (Nov)
Solve for x in the equation: log832 = x + 2
11 1 11
A B − C–1 D−
3 3 3
Solution
log832 = x + 2
Change from log to indices
8x + 2 = 32
Expressing both sides in base2 and its powers
23(x + 2) = 25
88
Logarithmic equations 2012/7c Neco
At times, questions in logarithm involve equations, If log2 x = 3 + log2 y , find the value of
x
which may be simple linear, simultaneous linear y
equations or quadratic equation in nature. Solution
Which ever pattern the questions follows; we solve Transforming 3 to log2
using the knowledge of the method of solving such log2 x = 3 + log2 y becomes
equation. log2 x = log223 + log2 y
Collect unknown terms together
Simple equation log2 x – log2 y = log28
2014/1b NABTEB ( Nov ) Applying laws of logarithm
Solve log3(2x + 1) – log3(x – 3) = 2 x
log2 = log28
Solution y
Express the RHS in log3 form x
Equating terms = 8
log3(2x + 1) – log3(x – 3) = log332 y
Applying laws of logarithm 2007/7b
2x + 1 If log 10y + 3log10x = 2, express y in terms of x
log3 = log3 3 2
x−3 Solution
Equating terms Since the LHS is in base 10, we align the RHS as well
2x + 1 log 10y + 3log10x = log10102
= 32
x−3 log10y + log10x 3 = log 10 102
Cross multiply Applying the law of addition in log
2x + 1 = 9(x – 3) log10 y × x3 = log 10102
2x + 1 = 9x – 27 Since both sides are in the same base 10, we equate terms
1 + 27 = 9x – 2x yx3 = 102
28 = 7x Making y the subject of formula
28 7x 100
= y = 3
7 7 x
4 = x 2011/1a Neco Counter example
2014/ 23 Neco Given that log(y – 2)3 = log 64, without using
If log3(2x – 1 ) – log 32 = 1, find x mathematical tables, find y
A 4 1/ 2 B 3 1/ 2 C 21/2 D 11/2 E1 Solution
Solution log(y – 2) = log 64
3

log3(2x – 1 ) – log 32 = 1 we can equate terms directly here


Applying laws of logarithm (y – 2)3 = 64
2x + 1 Next, we express the RHS in power 3 to align with LHS
log3 = log3 3
2 (y – 2)3 = 43
Equating terms Since they are in the same power, we can equate terms
2x + 1 y–2 = 4
= 3 y = 4+2
2
y = 6
2x – 1 = 2 × 3
2009/1b Neco (Nov)
2x = 6 + 1
Find x if log3 (3x – 2 ) – log3 (x + 2) = 2
7
x = i.e 31/2 ( B ) Solution
2 Applying laws of logarithm
2007/2b log3 (3x – 2 ) – log3 (x + 2) = 2 becomes
If 4log a + 5log a – 6log a = log 8, what is a? log3 (3x – 2 ) – log3 (x + 2) = log332
Solution 3x − 2
Applying laws of logarithm i.e log 3 = log332
4log a + 5log a – 6log a = log 8 becomes x+2
log a 4 + log a5 – log a6 = log 8 Equating terms
 a4  a5  3x − 2
= 32
log  6
 = log 8
 x+2
 a  Cross multiply
Some terms will cancel out 3x – 2 = 9(x + 2)
log a3 = log 8 3x – 2 = 9x + 18
Equating terms 3x – 9x = 18 + 2
a3 = 8 or a =
3
8 – 6x = 20
a3 = 23 20 10
x = i.e −
a = 2 −6 3
89
2007/11 Neco Since they are in the same base, we equate terms as:
Solve the equation 3log 3x – log 3x = 2 9 = a2
1 32 = a2
A B 2 C 3 D3 E9 3 = a (A)
2
Solution
3log 3x – log 3x = 2
2005/ 9a Exercise 5.41
Expressing both sides in log3, we have
log 3x3 – log 3x = log332 If log10 Q − log10 12 (Q −1) = 1 , find the value of Q
Applying the laws of logarithm
x3 2009/3 (Nov) Exercise 5.42
log3 = log3 3 2 Given that log x = 2log3 + log6 – log2,
x find the value of x
Since they are in the same base, we equate terms
A 9 B 18 C 27 D 81
x3
= 32
x 2012/9 Neco Exercise 5.43
i.e x2 = 32 Solve the equation log10x – log 10(2x – 3.8) = 1
Since they are in the same power, we equate terms A0 B–1 C–2 D1 E2
x = 3 ( D)
2005/34 (Nov) Exercise 5.44
2006/55 Neco Solve for x in the equation:
Solve the equation: 6
log 2x + log 3 = 2 log  
3 5
log10 (6a + 3) + log   = log10 (2a − 1) 6 39 39 1
10  7  A B C D
25 25 50 5
A–4 B – /2
1
C 1/ 2 D 2 E4
Solution
2012/32 Exercise 5.45
3
log10 (6a + 3) + log   = log10 (2a − 1) Which of these statement about y = 8 m is correct?
10  7  1
Since they are in the same base, we apply the law of A log y = log 8 × log m B log y = 3log 2 × 2 log m
logarithm as: C log y = 3log 2 – 1
2 log m D log y = 3log 2 + 1
2 log m
3
log10 (6a + 3)  = log10 (2a − 1)
7
Since they are in the same base, we equate terms as: Simultaneous linear equations in nature
3
(6a + 3) × = 2a – 1 1997/8
7 Solve the simultaneous equations
Cross multiply Log10 x + log10y = 4
(6a + 3)  3 = 7(2a – 1) Log10x + 2log10y = 3
18a + 9 = 14a – 7 Solution
18a –14a = – 9 – 7 Expressing both sides of the equations in logarithm to base 10
4a = – 16 Log10x + log10y = log10104 ---- (1)
16 Log10x + log10y2 = log10 103 ----- (2)
a = – From (1)
4
= –4 (A) Log10 (xy) = Log10104
xy = 104 ---- (a)
2006/ 31 counter example Also from (2)
If log a 270 – log a10 + loga 13 = 2, Log10 (xy2) = Log10103
what is the value of a? xy2 = 103
A3 B4 C5 D6 i.e (xy) y = 1000
Solution Substituting for xy value from (a)
(104)y = 1000
log a 270 – log a10 + loga 13 = 2
y = 1000 = 1 = 10- 1
Since the LHS is all in the same base a, we do same to 10000 10
the RHS Substituting y values into (a)
log a 270 – log a10 + loga 13 = loga a2 xy = 104
Applying the laws of logarithm x(10- 1 ) = 104
1 x = 104 x 101
270 x = 105 = 100,000
loga 3 = log a2
a Solution set is : x = 105 and y = 10 - 1
10
log a9 = loga a2
90
2006/2 f/maths − 1  1 − 4  2  (−9)
If 22x – 3y = 32 and log y x = 2, x =
2 2
find the values of x and y
Solution − 1  1 + 72
=
Converting logarithm to indices 4
Log yx = 2 becomes − 1  73
y2 = x ----------------------(1) =
4
Note that 22x – 3y = 32 is same as
− 1  8.544
22x – 3y = 25 =
Equating powers 4
2x – 3y = 5 ----------------(2) − 1 + 8.544 − 1 − 8.544
= or
Substituting for x = y2 4 4
2y2 – 3y = 5 7.544 − 9.544
= or
Rearranging 4 4
2y2 – 3y – 5 = 0 = 1.886 or – 2.386
Factorizing 1993/6 UME
2y2 + 2y – 5y – 5 = 0 If 2log3 y + log3 x2 = 4, then y is
2y(y + 1) –5(y + 1) = 0 A. 4 – log3x2 B. 4 C. 2 D.  9
(y + 1)(2y – 5) = 0 2 log3x2 x x
y + 1 = 0 or 2y – 5 = 0 Solution
y = –1 or 2y = 5
Applying the laws of logarithm,
y = –1 or 5/2
log3y2 + log3 x2 = 4
when y = –1; x = y2 becomes x =1
Since they are in the same base
when y = 5/2; x = y2 becomes x = 25/4
log3 ( y2 × x2 ) = 4
Converting to indices
2013/13 Neco f/maths Exercise 5.46
Solve the equations: 34 = y2 x2
If log3 ( p + q ) = 1 and log2 ( 2p – q ) = 2 34 = y2
find the value of pq x2
A. 21/3 B. 15/9 C. 11/2 D. 2/3 E. 3/7 32 2
= y2
x
2010/9a Neco f/maths Exercise 5.47
2
Solve the equations: y =  9 =  9 (D)
log8 (4 + x ) + log8 y = 2 x x
log4 (x + 52 ) – log4 y = 2 2009/6 Neco f/maths
Solve the equations:
2009/2 f/maths Exercise 5.48
log (x2 + 6x ) = 1.4314
Solve the simultaneous equations A. –9 or 3 B. 9 or –3 C. –9 or 5 D. 5 or 9 E. 9 or 7
log2x – log2y = 2 , log2( x – y ) = 3 Solution
log (x2 + 6x ) = 1.4314
Quadratic equation in nature Changing from logarithm to indices
2009/8b Neco x2 + 6x = 10 1.4314
Given that log2(2x – 1) + log2 (x +1) = 3, find x Actually the RHS is the antilog of 1.4314
Solution x2 + 6x = 27
Applying laws of logarithm x + 6x – 27 = 0
2

log2(2x – 1) + log2 (x +1) = 3 becomes factorizing: x2 + 9x –3x – 27 = 0


log2(2x – 1) + log2 (x +1) = log2 23 x(x + 9) –3( x + 9) = 0
log2(2x – 1)(x + 1) = log28 (x + 9)( x –3) = 0
Equating terms x + 9 = 0 or x –3 = 0
(2x – 1)(x + 1) = 8 x = –9 or 3 A.
Expanding the LHS 2013/18 Neco f/maths Exercise 5.49
2x(x + 1) – 1(x + 1) = 8 Solve the logarithmic equation :
2x2 + 2x – x – 1 = 8 log5 (x2 – 9 ) = 0
2x2 + x – 9 = 0 A.  3 B.  7 C.  10 D. 3 E.  9
Factorizing, it is not factorizable 2014/1 f/maths Exercise 5.50
Applying the general formula Solve (log2m)2 – log2m3 = 10
− b  b 2 − 4ac 1976/8 ii Exercise 5.51
x =
2a Without using tables, find the value of n in
Here a is 2, b is 1 and c is – 9 . Substituting 2 log( n + 3 ) = log4

91
CHAPTER SIX 2014/3 NABTEB Exercise 6.1
Evaluate 64.25  10 – 3
Standard form & Logarithm II A 0.06425 B 0.6425 C 6.425 D 64.25
Standard Form
2009/8 NABTEB (Nov) Exercise 6.2
This topic like most topics in mathematics is a bit
Express 0.000502 in standard form
confusing to students due to lack of basic understanding.
A 5.02  10 4 B 5.02  10 3
Standard form is the expression of numbers in scientific
form to reduce the stress and space of writing very large C 5.02  10 –4
D 5.02  10 – 3
or very small numbers. 2010/14 Neco Exercise 6.3
It is written in the form A × 10n. Where A is a number Which of the following has the same value as 0.0162560 ?
that can be any of 1,2,3,4,5,6,7,8,9. and n can be a A 1.6256  10 2 B 1.6256  10 1 C 1.6256  10 0
positive or negative integer. D 1.6256  10 –2
E 1.6256  10 – 3
Examples;
(a) Express the following in standard form
Operations with standard form
( i ) 1,267,000,000 ( ii ) 0.000 000 00156
Solution Addition
( i ) 1,267,000,000 = 1 267 000 000. 0 Example
We have only made the decimal point clearer, then start i. Express the sum of 6.03 × 106 and 2.17 105 in
counting backward (reverse) till you get to the space standard form
before the first digit – place your decimal there and note solution
the number of digits or places jumped. Firstly, we factor out the lowest power of 10 i.e 105
= 1.267 × 109 Thus 6.03 x 106 + 2.17 x 105
( ii ) Similarly, we move forward till the space between = (6.03 × 101 + 2.17) × 105
first digit and second one. Hence = (60.3 + 2.17) ×105
0.000 000 00156 = 1.56 × 10- 9 = 62.47 × 105
(b) Express the following in standard form Converting to standard form , we have
( i ) 5 ( ii ) 50 ( iii ) 500 ( iv ) 5000 = 6.247 × 105 × 101
Solution = 6.247 × 106
( i ) 5 is the same as 4.0
5 = 5.0 × 100 is the same as 5.0 since 100 = 1
Subtraction
( ii ) 50 = 5.0 × 101 Example
( iii ) 500 = 5.0 × 102 Without using a calculator, find the difference between
( iv ) 5000 = 5.0 × 103 9.5 × 107 and 3.08 × 106 leaving your answer in standard form
Solution
2009/1 (Nov) We factor out the lowest power of 10 i.e 106
Express 0.00347 in the standard form Thus, 9.5 × 107 – 3.08 × 106
A 3.47  10 – 3 B 3.47  10 – 2 C 3.47  10 2 D 3.47  10 3 = (9.5 × 101 – 3.08) × 106
Solution = (95.00 – 3.08) × 106
0.00347 = 3.47  10 – 3 (A) = 91. 92 × 106
Converting to standard form, we have
2012/1 Neco = 9.192 × 106 × 101
Express 0.0462 in standard form = 9.192 × 107
A 0.462  10– 1 B 0.462  10– 2 C 4.62  10– 1
D 4.62  10– 2 E 4.62  10– 3 2002/2c Exercise 6.3b
Solution Without using calculator, find the difference between
0.0462 = 4.62  10– 2 ( D ) 3.4 × 109 and 2.2 × 108. Leave your answer in standard form
2010/27 NABTEB (Nov)
Express 0.0000043169 in standard form to three
significant figures
A 4.3169  10 – 5 B 4.3169  10 – 6
C 4.317  10 – 6 D 4.31  10 – 4
Solution
First, we express in standard form
0.0000043169 = 4.3169  10– 6
Next, approximation to 3 s.f
= 4.32  10 – 6
Note that approximation to 3 decimal places is 4.317 10 – 6

92
Multiplication Converting to standard form
2014/2 NABTEB = 1.125  103  10– 7
Simplify 0.0285  0.267, leaving the answer in standard form = 1.125  103 + (–7)
A 7.6095  10 – 3 B 7.6095  10 – 2 = 1.125 103 – 7
C 7.6095  10 – 1 D 7.6095  10 0 = 1.125 10– 4
Solution 2014/4 Neco
First, we express them in whole numbers without decimals
Express 0.17  0.17 in standard form
0.0285  0.267 = 285  10– 4  267  10 – 3
A 2.89  10– 4 B 2.89  10– 2 C 2.89  10 – 1
= 285  267  10 – 4  10 – 3
D 2.89  10 1 E 2.89  10 4
= 76095  10 – 4 + (– 3)
Solution
= 76095  10 – 7 We express them in whole numbers without decimals
Converting to standard form
0.17  0.17 = 1710 – 2  17 10 – 2
= 7.6095  10 – 7  10 4
= 17  17  10 – 2  10 – 2
= 7.6095  10 – 7 + 4
= 289  10 – 2 + (– 2)
= 7.6095  10 – 3 A
= 289  10 – 4
Converting to standard form
2006/ 3 (Nov)
= 2.89  10 2  10– 4
Express the product of 1.04 and 0.08 in the form
= 2.89  10 2 – 4
a  10n, where 1<a<10 and n an integer
= 2.89  10 – 2 ( B)
A 8.32  10 2 B 8.32  10 1 C 8.32  10–1
D 8.32  10 –2
2010/1 Exercise 6.4
Solution
Simplify 0.000215  0.000028 and express your answer in
We are simply asked to work in standard form
the standard form
expressing the item in whole numbers without decimals
A 6.03 10 9 B 6.0210 9 C 6.03 10 – 9 D 6.02 10 – 9
1.04  0.08 = 104 10– 2  8  10–2
= 104  8  10 – 2 – 2 2014/2 NABTEB (Nov) Exercise 6.5
= 832  10– 4 Express the product of 0.06 and 0.09 in standard form
Converting to standard form A 5.4  10– 3 B 5.4  10– 2 C 5.4  10– 1 D 5.4  10 2
= 8.32  10– 4  10 2 1976/4a ( Nov ) Exercise 6.6
= 8.32  10– 4 + 2 Multiply 7.37  109 by 3.02  10– 7 , giving your answer in
= 8.32  10 – 2 (D) the form A  10n where n is an integer and A is a number
between 1 and 10 to 3 significant figures
2006/2 (Nov)
Evaluate (4.5  10– 2)2 leave your answer in the
standard form Division
A 2.025 10– 4 B 2.025 10– 3 C 9.0 10– 4 D 9.0 10– 3
Solution 2007/3
–2 2 –2 –2 Simplify (0.3  105)  (0.4  107), leaving your answer in
(4.5  10 ) = 4.5 10  4.5  10
the standard form
Expressing them in whole numbers
A 7.5  10 – 4 B 7.5  10 – 3 C 7.5  10 – 2 D 7.5  10 – 1
= 45 10 – 2  10 – 1  45  10 – 2  10 – 1
Solution
= 45  45  10 – 2 – 1 – 2 – 1 We express terms in whole numbers without decimals
= 2025  10– 6
3  105  10−1
Converting to standard form (0.3  10 )  (0.4  10 ) =
5 7

= 2.025  103  10– 6 4  107  10−1


–1
= 2.025 10– 3 (B) 10 will cancel out
3 105
= 
2011/2 Neco 4 107
Express the product of 0.0045 and 0.025 in standard = 0.75  10 5 – 7
form
= 0.75  10– 2
A 1.125  10 – 1 B 1.125  10 – 3 C 1.8  101
Converting to standard form
D 1.125  10 2 E 1.8  10 2
= 7.5  10–2  10–1
Solution
First, we express them in whole numbers without decimals
= 7.5  10– 3 (B)
0.0045 0.025 = 45  10– 4  25  10– 3
2007/1a
= 45  25  10 – 4  10 – 3
Evaluate, without using mathematical tables or calculator
= 1125  10– 4 + (– 3)
(3.69  10 5)  (1.64  10 – 3), leaving your answer in the
= 1125  10 – 4 – 3 standard form
= 1125  10– 7

93
Solution 2006/2 Neco (Nov)
We express them in whole numbers without decimals Evaluate 2.25  10– 2  0.225  10– 3  22.5  102
369 105  10− 2 A 2.25  10– 3 B 2.25  10– 2 C 2.25  10– 1
(3.69  10 5)  (1.64  10– 3) = D 2.25  10 2 E 2.25  10 5
164 10− 3  10− 2
10– 2 will cancel out Solution
We express them in whole numbers without decimals
369 105
=  2.25  10– 2  0.225  10– 3 × 22.5  102
164 10− 3 225 10− 2  10− 2  225 102  10−1
= 2.25  105– (–3) =
225 10−3  10−3
= 2.25  108
2013/1 Neco 225 225 10− 2 − 2 + 2 −1
=
Evaluate 0.009  0.012, leave your answer in the 225 10−3 − 3
standard form Some terms will cancel out
A 7.5 10– 2 B 7.5 10– 1 C 7.5 10 0 225 10− 3
D 7.5 10 1 E 7.5 10 2 = 
1 10− 6
Solution Power –6 goes to numerator
We express them in whole numbers without decimals
= 225  10– 3 – (–6)
0.009 9  10− 3 = 225  10 – 3 + 6
0.009  0.012 is the same as =
0.012 12  10− 3 = 225  10 3
9 Converting to standard form
=  10− 3 − ( −3)
12 = 2.25  10 3  10 2
3 = 2.25  10 5 ( E )
=  10− 3 + 3
4
= 0.75 2009/8b Neco (Nov)
Converting to standard form 0.0125  0.625
Simplify without using calculator or tables,
= 7.5  10 – 1 B 0.075  0.25
2002/3 NABTEB Exercise 6.7 correct to 2 decimal places
Evaluate 0.9687 leaving your answer in standard form Solution
0.001 First, we express in whole numbers without decimals
A. 9.687 × 10– 4 B. 9.687 × 10–1 0.0125  0.625 125 10− 4  625 10−3
C. 9. 687 × 10 2 D. 9.687 × 10 3 =
0.075  0.25 75  10− 3.  25  10− 2
2004/4 Neco Exercise 6.8
Express the quotient of 0.422 and 0.004in standard form. 125 625 10− 4 +( −3)
2 1 0 = 
A. 1.055 × 10 B.10. 55 × 10 C.105.5 × 10 75  25 10− 3 + ( −2)
D. 10.55 × 101 E. 1.055 × 10-2 Some terms will cancel out
125 10−7
= 
Multiplication and division joint cases 3 10− 5
2014/1a 125
=  10− 7 −( −5)
Without using tables or calculator, 3
0.6  32  0.004 Minus power from denominator becomes plus
simplify
1.2  0.008 0.16 125
=  10− 7 + 5
leaving your answer in the standard form (scientific notation) 3
Solution 125
First, we express in whole numbers without decimals =  10− 2
3
0.6  32  0.004 6  10−1  32  4  10− 3 = 41.667  10 – 2
=
1.2  0.008 0.16 12  10−1  8  10− 3  16  10−2 Here we are not working in standard form
Some terms will cancel out = 0.41667
1 10− 4  0.42 to 2d.p
= 
2 10− 6
1
Power –6 goes to numerator =  10− 4 − ( − 6) 2012/8 Neco
2 0.0324  0.0064
= 0.5  10– 4 + 6 Simplify and give your answer in
0.048  0.12
= 0.5  102
Converting to standard form standard form.
= 5.0  10– 1  102 A 36  10 – 2 B 3.6  10 – 2 C 3.6  10 2
= 5.0  101 D 0.36  10 1 E 0.36  10 – 1

94
Solution 18  10− 2  125  10−1
0.18  12.5
First, we express in whole numbers without decimals =
0.05  0.2 5  10− 2  2  10−1
0.0324  0.0064 324 10− 4  6410 −4
=
0.048  0.12 48  10−3 1210−2 18  125 10− 2 −1
=
324 6410 − 4 + ( −4) 5  2  10− 2 −1
= Some terms will cancel out as square cannot be applied
48  1210− 3 + ( −2)
directly at this level
36 10 − 8
=  = 9  25  10− 3 −( − 3)
1 10 −5 2 2 −3+3 2 2
= 3  5  10 i.e 3  5
= 36  10– 8 – (–5)
= 36  10– 8 + 5 =35
= 15 (not in standard form)
= 36  10– 3
Converting to standard form = 1.5  101 (in standard form)
= 3.6  101  10– 3
2009/3a
= 3.6  10– 3 + 1 i.e 3.6  10– 2 (B)
0.14  3.2
Without using table or calculator evaluates:
2009/34 NABTEB (Nov) Exercise 6.9 0.0028 90
0.00275 0.008 Solution
Simplify and give your answer in
0.0025 0.09 We express given numbers in whole without decimals
standard form.
0.14  3.2 14  10− 2  32  10−1
A 9.7 10– 2 B 9.7  10– 1 C 9.7  10– 3 D 9.7  10 1 =
0.0028 90 28  10− 4  9  101
2014/2a NABTEB (Nov) Exercise 6.10
4.56  3.6 14  32  10− 2 −1
Evaluate , leave your answer in standard form =
0.12 28  9  10− 4 +1

1977/2b Exercise 6.11 16  10− 3


=
pq 3 9  10− 3
Evaluate , if p = 108, q = 10– 4 and
r2 16 16
=  10−3−( −3) =  10−3+3
r = 0.000 01, expressing your answer in powers of 10 9 9
16 4
= =
9 3
Cases involving square root = 1.333
2008/1a Exercise 6.12
2009/6 Without using tables or calculators, find the value of
Express the square root of 0.000144 in the standard form
A 1.2  10 – 4 B 1.2  10 – 3 C 1.2  10 – 2 D 1.2  10 – 1 0.00196 2.25
, in standard form
Solution 62.5
We express in numbers without decimal
1 2006/2a Neco Exercise 6.13
−6
0.000144 = (144  10 ) 2
Simplify without using mathematical tables or calculator
1
−6 2.401 105
= (12  10 ) 2
2

0.0049 10− 2
Taking the square root of the numbers and powers as
2 6

= 10
12 2 2

= 12  10 – 3
Converting to standard form
= 1.2  101  10 – 3
= 1.2  10 – 2 ( C )

2005/6a
Without using Mathematical tables or calculator
0.18  12.5
evaluate:
0.05  0.2
Solution
We express in numbers without decimals
95
Common Logarithm E.g (3) With the aid of tables evaluate (42.95)3
Logarithm to base 10 is known as the common Solution
logarithm. There are other bases of logarithm e.g base e No Log
(natural, Naperian base). (42.95) 3
1.6330 × 3
The logarithm of a number consists of two parts called 79250.0 4.8990
the Characteristics and the mantissa. Note; Since antilog xtics is 4, movement 5 steps from the
Characteristics is a whole number while Mantissa is a imaginary decimal point before the first digit i.e 7
decimal. To get the characteristics of any given number, Hence 7925 = 79250.0
express the number in standard form, the index is the E.g (4) Evaluate the 3 66.32 using tables
characteristics. Solution
E.g, Study carefully the table below: No Log
Numbers Standard Logarithms Answers 1/3
form Xtics mantissa (66.32) 1.8216 ÷ 3
(a) 5 5.0 x 100 0 .6990 Log of 5 4.048 0.6072
= 0.6990 Note
(b) 60 6.0 x 101 1 .7782 Log of 60 Log problems involving roots, we divide the log value
= 1.7782 by the equivalent root e.g
(c) 8156 8.156 x 103 3 .9115 Logof 8156
=3.9115 , divide by 2
(d) 8.2 x 10 –3 .9138 Log of 0.0082
0.0082 -3 or 3 = 3.9138
3
, divide by 3
-1
(e) 0.2 2.0 x 10 .3010 Log of 0.2
- 1 or 1 = 1.3010
4 , divide by 4 etc

You will agree with the author that your ability to find
the characteristics of any given problem depends on
basic knowledge of standard form Logarithm of numbers less than one
E.g 1 Evaluate 0.5624  0.0378
Logarithm of numbers greater than one
No Log
Multiplication (Addition of log values)
0. 5624 1.7500 Addition –1 + 1 – 2 = - 2
E.g. (1) Using log table, evaluate 69.24  8.31
0.0378 2.5775
No Log 0.02125 2.3275
69.24 1.8403
8.31 0.9196  0.5624  0.0378 = 0.02125
575.3 2.7599

Analysis E.g 2 Evaluate 0.003512  0.6207


69.24 = 6.92  101 No Log
8.31 = 8.31  100 0.003512 3.5455 Subtraction
Check 69 under 2 difference of 4 0.6207 1.7929
i.e 8401 + difference (2) = 8403
Check 83 under 1 0.005657 3.7526
add up log values  0.003512  0.6207 = 0.005657
Check the Antilog of 75 under 9, diff. of 9
5741 + 12 = 5753 E.g. (3) Evaluate (0.07392)4
Antilog xtics 2
Movement 3 steps from the imaginary decimal before No log
the first digit i.e 5 (0.07392)4 2.8628  4
Eg (2) 0.00002826 5.4512
Using logarithm table, evaluate 7031  4.911.
Solution  (0.07392)4 = 0.00002826
No Log Multiply the xtics and the mantissa separately and add the
7031 3.8471 result i.e 0.8628  4 = 3.4512
4.911 0.6912 and 2  4 = 8
1432 3.1559
Thus 8 + 3.4512 = 5.4512
Division in log, we subtract the log values and
find anti log.
96
Solution
E.g (4) Evaluate 4 0.00692 To get a value, we check the reverse of the log value

No Log of a to base 10 i.e antilog of 2 .4472 implies .44 under 7


(0.00692) 1/4 3.8401  4 is expressed as: difference 2 then we move two steps backward

( 4 +1.8401)  4 Antilog of 2 .4472 = 2799
1
0.2884 1 . 4600 2800

 (90.00692) = 0.2884 1/4 Apply the place of decimal 2 to it
Think of a number to be added to 4 to give 3 on 2800 = 0.0280 ( C )
the xtic side only i.e 4 + 1 = 3
Then divide 4 by 4 and 1.8401 by 4. 2014/45
3
If log 5.957 = 0.7750, find log 0.0005957
− − − −
A 4 .1986 B 2 .9250 C 1 .5917 D 1 .2853
Problems on logarithm II
Solution
2011/1b No log


Given that x = 10 1 .6741
, without calculator, 0.0005957 4 .7750  3 is expressed as:
find the value of x −
Solution 6 + 2 .7750  3
− −
x = 10 1 .6741 2 .9250 (B)
1 −
2007/7a
x = 10 1 .6741
2
With the aid of four – figure logarithm table
We take the log10 of both sides
evaluate ( 0.004592 )3
1

1
log 10 x 2
= log 10 10 1 .6741 Solution
− No log
1
log10 x = 1 .6741 Log 1010 −
( 0.004592 )
2 1
− 3 3 .6620  3
1
log 10x = 1 .6741
2 −
Multiply through by 2 to clear fraction 0.1663 1.2207

log10 x = [2  0.6741 + 2  1 ] 2005/3a (Nov)
= 1.3482 + (–2)
− 0.0751
log10x = 1 .3482 Use logarithm table to evaluate

0.00784
Take antilog of 1 .3482 to get x Solution
x = 0.2229 No log

2007/34 0.0751 2 .8756
− Subtraction
Find the value of a if log10a + log10a2 = 0.9030 0.00784 3 .8943
A 4.0 B 2.0 C 1.6 D 0.0
Solution ( 00.00784
.0751 )
1
2
0 .9813  2
log10a + log10a2 = 0.9030
3.095 0.4907
Applying the law of addition log
log10 a  a2 = 0.9030
2005/10b Neco
Changing from log to indices
3 0.5915
100. 9030 = a3 Use logarithm table to evaluate
Antilog of .90 under 3 is 7998 with characteristic of 0 is 17.8 2
decimal place of one step forward and after 7. Solution
7.998 = a3 No log
3
7.998 = a −
3 0.5915 1 .7720  3 is express as:
thus a = 1.9998
− −
 2.0 B 3  3 + 2.7720  3 = 1 .9240
17.82 1.2504  2 2.5008
2005/ 15


If log10 = 2 .4472 , find a
a 0.002650 3 .4232
A 2.800 B 0.280 C 0.028 D 0.0028
97
2007/1 NABTEB 2006/12 Neco (Nov)
Use logarithm table to evaluate Use logarithm table to evaluate
0.784 3  23.67 1 (0.2514) 2  2.005 1
4 = A 4
1
= A 2

3.479 (0.3927) 3

Solution Solution
No log No log
− −
− −
0.784 3
1 .8943 3 = 2.6829 + (3) (0.2514)2 1 .4004  2 0.8008 + (2)
− − − −
(say 3  0.8943 and 3  1 ) 1 .6829 (Say 2  0.4004 and 2  1 ) 2 .8008
23.67 1.3742 1.3742
2.005 0.3021 0.3021
1.0571 − −
3.479 0.5414 0.5414 Numerator 1 .1029 1 .1029
0.5157 −
(0.3927)
1

1
3 1 .5941 3 is expressed as:
A 4
0.5157  4 − −
1.346 0.1289 3 + 2 .5941 3 1 .8647

1 .8647
2011/8 Neco −
Denominator 1 .2382
Use four figure tables table to evaluate
1 −
2
 1
 A 2
1 .2382  2
 22.5 3  33.5  −
 5  = A2 expressed as 2  2 and 1.2382  2
 0.00523  −
  0.4160 1 .6191
Solution
No log 2009/9 Neco
1
22.5 3
1.3522  3 0.4507 846.2 2  54.36  10 − 3
33.5 1.5250 1.5250 Use logarithm table to evaluate 1
Numerator 1.9757 1.9757 462.4 3

( 0.00523 ) Solution
1
5 3 .7185  5 is expressed as:
− − − Note that: 54.36  10− 3 is the same as 0.05436
5 + 2 .7185  5 1.5437 1.5437 No log
2
A 2.43202 846.22 2.9275  2 5.8550
− −
0.05436 2 .7353 2 1 .3677
73110.0 4.8640
Numerator 5.2227 5.2227
1
462.4 3
2.6650  3 0.8883
2008/6a NABTEB (Nov) 21600.0 4.3344
Use logarithm table to evaluate
117.5  2.56 1
2014/8 Neco Exercise 6.14
3 = A 3

5.96  39 3 0.5915  (392.8) 3


Solution Use logarithm table to evaluate
39.436  (12.03) 3
No log
117.5 2.0701 2014/6a Neco( Nov ) Exercise 6.15
2.56 0.4082 3.267
Numerator 2.4783 2.4783 Use mathematical tables to evaluate
0.483  0.385
5.96 0.7752
39 1.5911 1978/11b Exercise 6.16
Denominator 2.3663 2.3663 If a = 5.732, b = 0.2795 and c = 378.4, use logarithm tables
2 3
A
1
3
0.1120  3 to evaluate a b  c correct to 3 significant figures
1.090 0.0373 2006/10a NABTEB (Nov) Exercise 6.17
Use logarithm table to evaluate
(3.415) 4  28.91
, correct to 3 significant figures
0.267
98
CHAPTER SEVEN Conversion of a given base to base 10
This is done by method of expansion in the powers of “old”
Number bases & Surds base as shown below
Number bases 2013/3 Neco
Just as we use fuel gauge to measure petrol, baskets to Convert 11011two to denary
measure garri; similarly we measure (count) numbers in A 27 ten B 29 ten C 49 ten D 51ten E 53 ten
bases. The common one is base 10 (denary/decimal Solution
system). Others are base 5 (quinary system), 11011two = 1  24 + 1  23 + 0  22 + 1  21 + 1  20
base 8 (octal system ) etc = 16 + 8 + 0 + 2 + 1
Conversion from base ten to others = 27ten (A)
This is done by repeated division and we take our result
of the remainders from bottom to top 2010/1 NABTEB (Nov)
Convert 112three to base 10
2010/3 Neco
A 14 B13 C 12 D 10
Convert 29ten to a number in base two
Solution
A 1011two B 1101two C 1111two
Converting from other bases to base ten
D 10111two E 11101two
Solution 112three = 1  32 + 1  31 + 2  30
2 29 = 9+3 + 2
2 14 + 1 = 14ten ( A )
2 7 + 0
2 3 + 1 2010/26 NABTEB (Nov)
2 1 + 1 Express 556seven as a denary number
0 + 1 A 245 ten B 286 ten C 450 ten D 550 ten
29ten = 11101 two ( E ) Solution
2011/9 Neco Converting from other bases to base ten (denary), expand in
Convert 59 ten to base two the powers of the old base
A 1111011two B 1110111two C 111011two 556 seven = 5  72 + 5  71 + 6  70
D 110111 two E 11110two = 5  49 + 35 + 6
Solution = 245 + 35 + 6
2 59 = 286 ten
2 29 + 1
2 14 + 1 Conversion from base ten numbers plus decimal
2 7 + 0 to others
2 3 + 1 2006/5 Neco
2 1 + 1 Express 12.625ten in base two
0 + 1 A 101.101 B 101.110 C 1100.011
59ten = 111011 two D 1100.101 E 1100.110
2014/1 NABTEB Solution
Express 12ten in binary We solve in two parts
A 1000 B 1001 C 1011 D 1100 First, convert whole number 12ten to base two
Solution 2 12
Converting from base ten to 2, long division 2 6 + 0
2 12 2 3 + 0
2 6 + 0 2 1 + 1
2 3 + 0 0 + 1
2 1 + 1 12ten = 1100two
0 + 1 Convert fraction part ∙625ten to base two
Multiplication by 2 three times to show 3 places of decimal
12ten = 11002 ( D )
0 ∙ 625  2
2012/4 Exercise 7.1 1 ∙ 25  2  2  0.25
Convert 3510 to a number in base 2 0 ∙ 50  2  2  0.50
A 1011 B 10011 C 100011 D 11001 1∙0
2005/1a Neco Exercise 7.2 0.625ten = 101two
Express 3710 in base two
Thus 12.625ten = 1100.101two (D)
Exercise 7.3
Express 15ten in binary
A 1001 B 1110 C 1011 D 1111

99
Example CTD2 Conversion of a given base number with decimal
Convert 87.6510 to a number in base 8 to base 10
Solution 2011/8 Neco
We solve in two parts Express 101.01two to denary
First convert whole number part 87ten to base eight. A 3.150 B 5.250 C 7.125 D 7.150 E 7.250
8 87 Solution
8 10 + 7 101.01two = 122 + 021 + 120 + 02– 1 + 12– 2
8 1 + 2 1
0 + 1 = 4+0+1+0+
4
87ten = 127eight 1
Next, we convert 0∙6510 to base eight =5 ten i.e 5.25 (B)
4
Multiplication by 8 two times to show 2 places of 1975/2a
decimal Express the binary number 101.101 as a number in base ten
0 ∙ 65  8 Solution
5∙2  8 101.101two = 122 + 021 + 120 + 12–1 +02–2 +12–3
1∙6 =4+0+1+
1
+0+ 1
0.65ten = 0∙51eight 2 8
Thus 87.6510 = 127.51 eight = 4 + 1 + 0.5 + 0 + 0.125
= 5.625ten
Example CTD3 Example CBD3
Convert 0.5410 to a number in base three correct to Convert 11011.11 2 to a denary number
5 decimal places Solution
Solution 11011.11 2 = 124+123+022+121 +120 +12–1 +12–2
We continue multiplying by 3 till we set five decimal 1 1
= 116 + 18 + 0 + 12 +11+ 1 + 1
places 2 4
0 ∙ 53  3 1 1
= 16 + 8 + 0 + 2 + 1 + +
1 ∙ 59  3  0∙59  3 2 4
1 ∙ 77  3  0∙77  3 = 27 + 2 + 1 = 27 3 or 27.75 ten
ten
2 ∙ 31  3  0∙31  3 4 4
0 ∙ 93  3  0∙93  3 Exercise 7.8
2 ∙ 79 stop Convert 302.213 to base ten
0∙5410 = 0∙11202three
2009/13 Neco (Nov)
Conversion from a non base ten to another
3
Convert   ten to binary non base ten number
8 2008/6
A 11 two B 0.111two C 0.11two Convert 425 to a base three numeral
D 0.011two E 0.0011two A 2013 B 2103 C 2113 D 2223
Solution Solution
Convert   ten to binary implies 0.375ten to base 2
3 First, we convert 425 to base ten
8 425 = 4  5 1 + 2  5 0
We multiply by 2 three times to show three places of decimal = 20 + 2
0 ∙ 375  2 = 22 ten
0 ∙ 75  2  2  0∙75 Next, we convert 22ten to base 3
1 ∙ 50  2  2  0∙50 3 22
1∙0 3 7 + 1
0 ∙ 375ten = 0∙011two ( D) 3 2 + 1
0 + 2
Exercise 7.4 22 ten = 211 3 ( C )
Convert 0.3710 to base eight; leaves your answer in 3d.p
Exercise 7.5 VTR – 11/7 NTI TCII
Convert 127.3510 to base three Express 132six as a number in base FIVE
Exercise 7.6 A.211 B.210 C.201 D.112 E.102
Convert 0.41ten to base two; leaves your answer in 3d.p Solution
Exercise 7.7 First; we convert to base ten
Convert 144.41ten to base four; leave your answer in 132six = 1 x 62 + 3 x 61 + 2 x 60
2d.p = 62 + 3 x 6 + 2 x 1
= 36 + 18 + 2
= 56 ten

100
2ndly; we convert 56ten to base five 2009/49 (Nov)
5 56 Find the missing number in the following addition of
5 11 + 1 numbers in base two
5 2 +1 1 0 1 1 1two
0 +2 i.e 211five + * * * * *
Thus 132six = 211five (A) 1 0 0 0 0 0two
A 1001two B 1100two C 1101two D 1011two
2003/3b Neco Solution
Convert 223four to a number in base three. By simple algebra 2 + x = 5
Solution Implies x = 5 – 2
First; we convert to base ten So in actual sense, we are subtracting as:
223four = 2 x 42 + 2 x 41 + 3 x 40 1 0 0 0 0 0 two
= 2 x 16 + 2 x 4 + 3 x 1 – 1 0 1 1 1 two
= 32 + 8 + 3 0 0 1 0 0 1 two (A)
= 43ten Explanation
2ndly; we convert 43ten to base three The carry – carry operation results to
3 43 0 1 1 1 1 2 two
3 14 + 1 – 1 0 1 1 1 two
3 4+2 0 0 1 0 0 1 two
3 1+1
0+1 i.e 1121 three 2013/3
Thus, 223four = 1121three In what number base is the addition
465 + 24 + 225 = 1050?
Exercise 7.9 A ten B nine C eight D seven
Convert 241 in base 5 to base 8 Solution
A. 718 B. 1078 C. 1768 D. 2418 4 6 5
2 4
Addition in bases 2 2 5
2014/22 Neco 1 0 5 0 seven ( D )
Find the missing numbers, if the following addition is in Explanation
base six 5 + 4 +5 is 14 i.e 20
3 2 4 Write 0 carry 2
* * * 10 + 2 is 12 i.e 15 in base seven
10 0 1 Write 5 carry1
4 + 2 is 6 plus 1 i.e 7 which is 10 in base seven written out in full
A 534 B 432 C 323 D 234 E 233
Solution 2009/4 NABTEB (Nov)
In actual sense, the presentation is: If 345 + 72 + 124 = 552, what is the base used?
1 0 0 1 six A9 B8 C7 D6
– 3 2 4 six Solution
2 3 3 six (E) First, we arrange the numbers in the old fashion arithmetic style
Analysis 3 4 5
(Note any borrowed number is six) 7 2
After the carry – carry operations the new look is: +1 2 4
0 5 5 7 six 5 5 2
3 2 4 six
What base was 5 + 2 + 4 = 11 in base ten becomes 2 or 12
2 3 3 six
Base 9 is suspect i.e write 2 carry 1
Confirm from next line
2014/31 Neco
4 + (7 + 2) plus carried 1 in base 9 is 15 write 5; carry 1
Express 213four + 202 four in denary
Last line follows: 3 + 1 plus carried 1 is 5
A 72 ten B 73 ten C 78 ten D 102 ten E 415 ten
Solution 2005/3 Exercise 7.10
First, we do the addition in base four In what base is 234 + 141 = 405?
2 1 3 A5 B6 C7 D8
+ 2 0 2 2014/3 ( Nov) Exercise 7.10b
10 2 1 four Find the sum of 3035 and 1045
Next, we convert 1021four to base ten A 4125 B 4025 C 2445 D 1445
1021four = 1  43 + 0  42 + 2  41 + 1  40 2015/2 Exercise 7.10c
= 64 + 0 + 8 + 1 The sum of 110112, 111112 and 10002 is 10m10n02
= 73ten (B) Find the values of m and n
A m = 0, n = 0 B m= 1, n = 0 C m= 0, n = 1 D m= 1, n = 1
101
Subtraction in bases 2008/2 NABTEB (Nov)
2004/1a NABTEB Find the difference between 42 nine and 111five
( ii ) Evaluate: 110136 – 25346, in base 6 A7 B9 C 10 D 12
Solution Solution
1 1 0 1 3 six Since they are not in the same base, we convert both to base
– 2 5 3 4 six ten to reflect the options given, otherwise we would have
4 0 3 5 six converted only 42 nine to base five and subtract.
Explanation 42 nine = 4  91 + 2  90
The carry – carry operations result is = 36 + 2
0 6 5 6 9 = 38ten
– 2 5 3 4 111five = 1  52 + 1  51 + 1  50
4 0 3 5 six = 25 + 5 + 1
= 31ten
2003/3b ( i ) NABTEB
Thus their difference: 38 – 31 = 7 (A)
Given P = 242 five and Q = 14 five
Calculate ( i ) P – Q
2008/1 NABTEB (Nov)
Solution
The difference between 10010 two and 1101two is
P – Q is the same as
A 11 B 101 C 110 D 111
2 4 2 five
Solution
– 1 4 five
Since they are in the same base, we can subtract directly
2 2 3 five
1 0 0 1 0
Explanation 1 1 0 1
The carried or borrow operation result is   1 0 1 (B)
2 3 7 five
– 1 4 five Analysis
2 2 3 five 0 – 1 cannot, carry 1 to zero makes it 2 & 2 – 1 = 1.
2nd column becomes zero since we carried 1 from there 0 – 0 = 0
2006/2 3rd column: 0 – 1 cannot, carry 1 from forth column; none
Simplify: 11011 two – 1101 two there. Carry 1 from the 5th column place at 4th column 2
A 101000two B 1100two C 1110 two D 1011two carry 1 from there to 3rd column i.e 2,
Solution 2 – 1 = 1.
1 1 0 1 1two At 4th column: 1 left there – 1 = 0
– 1 1 0 1 two At 5th column: 0 left there
1 1 1 0 two ( C )
Explanation 2002/19 (Nov) Exercise 7.11
The carried – carry operation result is If 5 1 2 m
0 2 2 1 1 – 3 5 4 m
– 1 1 0 1 1 2 5 m ,
1 1 1 0 two find the number base
A six B seven C eight E nine
2014/34 Neco (Nov)
2010/43 Exercise 7.12
Subtract 2255six from 20035six
The subtraction below is in base seven.
A 1334six B 13340six C 22334 six
Find the missing number
D 23340six E 31340six
5 1 6 2
Solution
– 2 6 4 4
2 0 0 3 5six
2 * 1 5
– 2 2 5 5six
1 3 3 4 0six A2 B3 C4 D5
Explanation
5–5=0
3 – 5 cannot,
Carry one from zero, can’t, move to 2, carry 1 (6) drop
at zero. Carry 1 there and leave 5 behind drop 1 at zero
to get six. Carry 1 (6) from there and leave 5 behind.
Thus the new look is
1 5 5 9 5 six
– 2 2 5 5 six
1 3 3 4 0 six (B)
Note that this analysis step is for explanation only and
not part of the working
102
Multiplication in bases Solution
1975/2c 2 0 3 four
Calculate (212)three  (201) three giving your answer as × 2 0 3 four
a number in base three 1 2 2 1
Solution 0 0 0 *
2 1 2 1 0 1 2 *
 2 0 1 1 0 3 0 2 1 four (D)
2 1 2 Explanation
0 0 0 * 3  3 = 9 i.e 21 in base four (two four + 1)
1 2 0 1 * Write 1 carry 2
1 2 1 0 1 2 three 3  0 = 0 + carried 2 i.e 2
3  2 = 6 i.e 12 written in full
Here 2  2 is 4 i.e. 10
2007/1 Also 2 + 2 is 4 i.e. 10
Evaluate (111 two)2 and leave your answer in base 2
A 111001two B 110001two C 101001two D 10010two 2004/1a NABTEB
Solution (i) Evaluate: 2136 × 246
(111 two)2 is same as: Solution
1 1 1 two 2 1 3 six
× 1 1 1 two × 2 4six
1 1 1 1 3 0 0
1 1 1 * 4 3 0 *
1 1 1 * 1 0 0 0 0 six
1 1 0 0 0 1 two
Explanation
Note: 2 is 10, write zero carry one 4  3 = 12 i.e 20 in base six
4 is 20 write zero carry two Write 0 carry 2
3 is 11 written out in full; it is the last operation 4  1 = 4 plus 2 i.e 6, which is 10 in base six
Write 0 carried 1
2005/1 NABTEB 4  2 = 8 i.e 12 plus 1 carried 13 write in full
Find the value of 1111  1001 in base two 2  3 = 6 i.e 10 in base six
A 1100 B 10101 C 101011 D 10000111 Write 0, carry 1
Solution 3 + 3 = 6 i.e. 10 in base six.
1 1 1 1
× 1 0 0 1 2007/2a NABTEB
1 1 1 1 Find the product of 3246 and 156
0 0 0 0 * Solution
0 0 0 0 * Since they are in the same base we multiply directly bearing
1 1 1 1 * in mind that we are working in base 6
10 0 0 0 1 1 1two (D) 3 2 4
 1 5
2003/3b ( ii ) NABTEB 2 5 1 2
Given P = 242 five and Q = 14 five +3 2 4 *
Calculate ( ii ) PQ 10 1 5 2
Solution Explanation
PQ is 2 4 2 five 5  4 is 32, write 2, carry 3
× 1 4 five (it is 5  4 = 20, how many 6 in 20 is 3, balance 2)
2 1 2 3 five
5  2 is 10 + 3 carried is 13 i.e 21 in base six language write
2 4 2 * 1, carry 2
1 0 0 4 3 five
5  3 is 15 + 2 carried is 17 i.e 25 in base six language write
Note: 11 is 21 in base five it in full
Explanation 1  4 = 4 in base six
4  2 = 8 i.e 13 in base five 1  2 = 2 in base six
Write 3 carry 1 1  3 = 3 in base six
4  4 is 16 plus carried 1 is 17 i.e 32, Note: 5 + 2 = 7 i.e 11 in base six;
Write 2 carry 3 2 + 3 + 1 = 6 i.e 10 base six

2004 /1 NABTEB 2014/42 NABTEB (Nov)


Evaluate (203 four)2 in base four Find (1012)2, expressing the answer in base 2
A 2030 B 10012 C 12002 D 103021 A 11001 B 10010 C 11101 D 10101
103
Solution 100two = 1  22 + 0  21 + 0  2 0
(1012) = 1012  1012
2
= 4+0+0
First, we convert 1012 to base ten = 4 ten
1012 = 1  22 + 0  21 + 1  20 Performing the division: 52 = 13
= 4 + 0 + 1 4
= 5 ten Next, we convert 13ten to base two
But (1012)2 = 5  5 = 25 ten 2 13
Converting 25 ten to base 2, by division 2 6+1
2 25 2 3+0
2 12 + 1 2 1+1
2 6 + 0 0+1 13ten = 1101two (C)
2 3 + 0
2 1 + 1
0 + 1
(1012)2 = 11001 two A

2009/35 NABTEB (Nov) Exercise 7.13


Evaluate 1001two  101two
A 100111 two B 101101two C 11001 two
D 111001 two

Problems of bases in equation form

1975/2b
If (23)n = (1111)two, find n
Division in bases Solution
1998/35 (Nov) We convert both sides to base ten
Evaluate 11110two  110two 23n = 1111two becomes
A 11two B 100two C 101two 2  n1 + 3  n0 = 1  23 + 1  22 + 1  21 + 1  20
D 110two E 111two 2n + 3 = 8 + 4 + 2 + 1
Solution 2n + 3 = 15
First, convert all numbers to base ten 2n = 15 – 3
11110 two = 1  24 + 1  23 + 1  2 2 + 1  2 1 + 0  2 0 2n = 12
= 16 + 8 + 4 + 2 +0 n=6
= 30ten
2005/2b NABTEB( Nov)
110two = 1  22 + 1  21 + 0  2 0 If 345 = 23x, find x
= 4+2+0 Solution
= 6ten We convert both sides to base ten
30 345 = 23x becomes
Performing the division: =5
6 3  51 + 4  50 = 2  x1 + 3  x0
Next, we convert 5ten to base 2 15 + 4 = 2x + 3
2 5 19 – 3 = 2x
2 2+1 16 = 2x
2 1+0 Thus x = 8
0+1
5ten = 101two ( C ) 2005/1 NABTEB (Nov)
Find x if 21x = 710
2010/15 Neco A2 B3 C5 D 11
Evaluate (110100)two  (100)two Solution
A 10011two B 1010two C 1101two We convert the LHS to base ten
D 1100two E 101two 21 x = 710 becomes
Solution 2  x1 + 1  x0 = 7
First, convert all numbers to base ten 2x + 1 = 7
110100 two = 125+124+023+122+021+020 2x = 7 – 1
= 32 + 16 + 0 + 4 + 0 + 0 2x = 6
= 52ten x=3 (B)

104
2014/5a Neco 2014/2b
If 244n = 1022four, find n If 124n = 232five, find n
Solution Solution
We convert both sides to base ten We convert both sides to base ten
2n2 + 4n1 + 4 n0 = 143 + 042 + 241 + 240 124n = 232five becomes
2n2 + 4n + 4 = 64 + 0 + 8 + 2 1  n + 2  n + 4  n0 = 2  52 + 3  51 + 2  50
2 1

2n2 + 4n + 4 = 74 n2 + 2n + 4 = 50 + 15 + 2
Divide through by 2 n + 2n + 4 – 67 = 0
2

n2 + 2n + 2 = 37 n2 + 2n – 63 = 0
n2 + 2n – 35 = 0 Factorizing the quadratic equation
Factorizing the quadratic equation n2 + 9n – 7n – 63 = 0
n2 + 7n – 5n – 35 = 0 n(n + 9) – 7(n + 9) = 0
n(n + 7) – 5(n + 7) = 0 (n – 7)(n + 9) = 0
(n – 5)(n + 7) = 0 n – 7 = 0 or n + 9 = 0
n – 5 = 0 or n + 7 = 0 n = 7 accepted
n = 5 or –7
We accept n = 5 since we are dealing with base. 2014 /9a (Nov)
Find the value of n, if
2006/3 Neco (Nov) 44n + 55n = 121n
Find the value of x, if 10001two = 101x Solution
Ax=1 Bx=2 Cx=3 Dx=4 Ex=5 (44n + 55n) operation cannot be done since the base n is unknown
Solution We convert both sides to base ten.
We convert both sides to base ten 44n + 55n = 121n becomes
10001two = 101x becomes 4 n + 4 n + 5 n1 + 5 n0 = 1 n2 + 2 n1 +1 n0
1 0

12 + 02 + 02 + 021 +120 = 1x2 +0x1 +1x0


4 3 2
4n + 4 + 5n + 5 = n2 + 2n + 1
16 + 0 + 0 + 0 + 1 = x2 + 0 + 1 9n + 9 = n2 + 2n + 1
16 + 1 = x2 + 1 n + 2n – 9n + 1 – 9 = 0
2

16 = x2 n 2 – 7n – 8 = 0
It follows that x =  16 Factorizing the quadratic equation
n2 – 8n + n – 8 = 0
= 4 n(n – 8) + 1(n – 8) = 0
We take x = + 4 (D) (n – 8)(n + 1) = 0
n – 8 = 0 or n + 1 = 0
2006/2b Neco n = 8 accepted
Find n, if five times 43n is equal to 311n
Solution 1978/1b
5( 43n ) = 311n If 52x – 24x = 25x calculate the value of x
We convert both sides to base ten Solution
5(4  n1 + 3  n0) = 3  n2 + 1  n1 + 1  n0 52x – 24x = 25x
5(4n + 3) = 3n2 + n + 1 (52x – 24x ) operation cannot be done since the base x is unknown
20n + 15 = 3n2 + n + 1 We convert both sides to base ten.
3n + n – 20n + 1 – 15 = 0
2
52x – 24x = 25x becomes
3n2 – 19n – 14 = 0 5  x1 + 2  x0 – (2  x1 + 4  x0) = 2  x1 + 5  x0
Factorizing the quadratic equation 5x + 2 – (2x + 4) = 2x + 5
3n2 – 21n + 2n – 14 = 0 5x + 2 – 2x – 4 = 2x + 5
3n(n – 7) + 2(n – 7) = 0 5x – 2x – 2x = 5 + 4 – 2
(3n + 2)(n – 7) = 0 x=7
3n + 2 = 0 or n – 7 = 0 2014/2b NABTEB (Nov) Exercise 7.14
n = 7 accepted Find the value of n in the equation 173n = 81
2014/2 2008/1b NABTEB (Nov) Exercise 7.15
If 23x = 325 , find the value of x If (23)x = (1111)two, find x
A7 B6 C5 D4
Solution 2011/38 Exercise 7.16
We convert both sides to base ten Given that 124x = 7(14x), find the value of x
23 x = 325 becomes A 12 B 11 C9 D8
2  x1 + 3  x0 = 3  51 + 2  50
2x + 3 = 15 + 2
2x = 17 – 3
2x = 14
x=7 (A)
105
2006/5 (Nov) Next, we convert 13 ten to base two
If 243X5 = 366, find x 2 13
A0 B1 C2 D3 2 6 + 1
Solution 2 3 + 0
Convert the LHS to base ten 2 1 + 1
243X5 = 366 becomes 0 + 1
2  53 + 4  52 + 3  51 + x  50 = 366 13 ten = 1101two
250 + 100 + 15 + x = 366
365 + x = 366 2008/1b Exercise 7.17
x = 366 – 365 y 11
x = 1 (B) If all number in the equation = are in base
y + 101 10010
1999/6 (Nov) two, solve for y
Given that X = 111101two, find x
A 29 B 61 C 62 D 63
Solution
We convert the RHS to base ten
111101two = 125 + 124 + 123 + 122 + 021 + 120
= 32 + 16 + 8 + 4 + 0 + 1
= 61 ( B )

2007/5a Neco
If ( ytwo )2 = 2211three – 220four, find the value of y.
Solution
( ytwo )2 = 2211three – 220four,
The RHS cannot be done since they are of different bases
Convert both sides to base ten. Problems of bases in equation form that are
(y two) 2 = 2211three – 220 four becomes simultaneous linear equations in nature
(y20)2 = 233+232 +131+130 – (242+241+040)
(y 1)2 = 54 + 18 + 3 + 1 – (32 + 8 + 0) 2006/2 NABTEB (Nov)
y2 = 76 – 40 Find the value of x and y in the following equations:
y2 = 36 32x + 51y = 1010
y2 = 62 23 x + 42y = 710
y = 6ten Solution
We may convert 6ten to base 2 First, we convert the LHS of both equations to base 10,
2 6 since their RHS is in base 10 already
2 3 + 0 32x + 51y = 1010 becomes
2 1 + 1 [3  x + 2  x ] + [5  y + 1  y0] = 10
1 0 1

0 +1 3x + 2 + 5y + 1 = 10
6ten = 110 two 3x + 5y = 7 ------- (1)
Also 23x + 42y = 710 becomes
1979/2b [2  x1 + 3  x0] + [4  y1 + 2  y0] = 7
Solve for x in the following base two equation. 2x + 3 + 4y + 2 = 7
Leaving your answer in base two 2x + 4y = 7 – 5
11(1 + x) 2x + 4y = 2 ------ (2)
10x – = 101 Solving the resulting simultaneous linear equations; from 2
10
2x = 2 – 4y
Solution
x = 1 – 2y
First, we convert all in terms from base two to base ten
Substitute this into (1)
10x two = [1  21 + 0  20]x = 2xten 3x + 5y = 7 becomes
11two = 1  21 + 1  20 = 3ten 3(1 – 2y) + 5y = 7
10two = 1  21 + 0  20 = 2ten 3 – 6y + 5y = 7
101two = 1  22 + 0  21 + 1  20 = 5ten 3–y=7
11(1 + x) 3(1 + x) 3–7=y
10x – = 101 becomes 2x – =5 –4= y
10 2 Substitute y value into (2)
Multiply the resultant equation through by 2 to clear 2x + 4x = 2 becomes
fraction 2x + 4(– 4) = 2
4x – 3(1 + x) = 10 2x – 16 = 2
4x – 3 – 3x = 10 2x = 18
x = 10 + 3 i.e x = 13 x=9
106
2009/6b Neco 2005/10a Neco
Solve the simultaneous equations: The following simultaneous equations are given in base two:
11x – 10y = 101two ------ (1) 11x + 10y = 1100
10 x + 11y = 1001two ----- (2) 101x – y = 111
Solution Find the value of x and y, leaving your answers in base two
First, we convert both sides of the two equations to base 10 Solution
11x – 10y = 101two becomes First, we convert both sides of the two equations to base 10
[1  x + 1  x ] – [1  y + 0  y0] =
1 0 1 11x + 10y = 1100 becomes
1  22 + 0  21 + 1 20 [1  x1 + 1  x0] + [1  y1 + 0  y0] =
x+1–y=5 1  23 + 1  22 + 0  21 + 0  20
x–y=5–1 x+1+y+0=8+4+0+0
x – y = 4 ----- (1) x + y = 11 ----- (1)
Also 10x + 11y = 1001 becomes Also 101x – y = 111 becomes
[1  x1 + 0  x0] + [1  y1 + 1  y0] = [1  x2 + 0  x1 + 1  x0] – y = 1 22 +121 +120
1  23 + 0  22 + 0  21 + 1  20 x2 + 0 + 1 – y = 4 + 2 + 1
x + 0 + y +1 = 8 + 0 + 0 + 1 x2 – y = 7 – 1
x+y+1=9 x2 – y = 6 ------ (2)
x + y = 8 ----- (2) From (1) y = 11 – x
Solving the resulting simultaneous linear equations Substitute it into (2)
x–y = 4 x2 – (11 – x) = 6
+( x+y = 8) x2 – 11 + x = 6
i.e x2 + x – 17 = 0
2x = 12 Readers to complete the solving
x = 6
Substitute x = 6 into (1) Exercise 7.18
x – y = 4 becomes Find the number base x and y in the simultaneous equations
6–y=4 25x – 23y = 610
6 – 4 = y thus y = 2 34x + 32y = 3610
2014/10b Neco (Nov) 2014/10b Neco ( Nov ) Exercise 7.19
Find x and y in the simultaneous equation: Find x and y in the simultaneous equations:
35x + 50y = 5810
35x + 50y = 5810
51x + 34y = 5610 51x + 34y = 5610
Solution
First, we convert the LHS of both equations to base 10,
since their RHS is in base 10 already
35x + 50y = 5810 becomes
[3  x1 + 5  x0 ] + [5  y1 + 0  y0 ] = 58
3x + 5 + 5y + 0 = 58
3x + 5y = 58 – 5 Miscellaneous cases
3x + 5y = 53 ------- (1) 2006/1 NABTEB (Nov)
Also 51x + 34y = 5610 becomes Which of the following is the greatest?
[5  x1 + 1  x0] + [3  y1 + 4 y0] = 56 A 27 nine B 65 seven C 121 eight D 431five
5x + 1 + 3y + 4 = 56 Solution
5x + 3y = 56 – 5 First, we convert all the options to base ten
5x + 3y = 51 ------ (2) 27nine = 2  91 + 7  90
Solving the resulting simultaneous linear equations = 18 + 7
multiply (1)  5 and (2)  3 and subtract = 25 ten
15x + 25y = 265
– ( 15x + 9y = 153 ) 65 seven = 6  71 + 5  70
= 42 + 5
16y = 112 = 47 ten
y=7
Substitute this into (1) 121eight = 1  82 + 2  81 + 1  80
3x + 5y = 53 becomes = 64 + 16 + 1
3x + 5(7) = 53 = 81 ten
3x + 35 = 53 431five = 4  52 + 3  51 + 1  50
3x = 53 – 35 = 100 + 15 + 1
3x = 18 = 116 ten
x = 18/3 The greatest is 431five ( D )
x = 6

107
2005/4 (Nov) Explanation
Arrange 3910; 678; 2013 in ascending order of magnitude After the carry – carry operation the new look is:
A 3910; 678; 2013 B 678; 3910; 2013 0 2 2 1 1 2
C 3910; 2013; 678 D 2013; 3910; 678; 1 1 0 0 1
Solution 1 1 1 1 1 two
First, we convert all numbers to base ten
3910 is already in base ten as 39ten 2005/22 Neco
If 101two  11two + Ptwo = 10101two, find P
678 = 6  8 1 + 7  8 0 A 1101two B 1011two C 111two D 110two E 101two
= 48 + 7 i.e 55ten Solution
2013 = 2  3 2 + 0  3 1 + 1  3 0 First, we perform the multiplication by BODMAS
= 18 + 0 + 1 i.e 19ten 101two  11two is
Ascending order of magnitude: 19, 39, 55 1 0 1
2013; 3910; 678; ( D )  1 1
1999/5 (Nov) 1 0 1
The sum of three numbers in base two is 11101. If the 1 0 1 *
first two numbers are 1011 and 1101, find the third 1 1 1 1 two
number. 101two  11two + P two = 10101two becomes
A 11two B 101two C 1011two D 1111two Ptwo = 10101 two – (101two  11two)
Solution = 10101 – 1111 written out as
We are simply asked to solve 11101 – (1011 + 1101) 1 0 1 0 1
First, 1 0 1 1 – 1 1 1 1
+ 1 1 0 1 0 1 1 0 (D)
1 1 0 0 0 two 2014/1 NABTEB Exercise 7.20
What is the place value of 2 in 243 base five?
Next, 1 1 1 0 1 A2 B 10 C 50 D 200
– 1 1 0 0 0
2002/1a NABTEB Exercise 7.21
0 0 1 0 1two
Copy and complete the table in base 5
2009/6a (Nov) x 2 3 4
By how much is 110002 greater than or less than 2 4
1112  112? (Leaves your answer in base 2) 3 22
Solution 4 22
First, we multiply 1112  112 2003/1 NABTEB Exercise 7.22
1 1 12 The place value of 3 in 5324six
× 1 12 A 18 B 108 C 180 D 1204
1 1 1
Exercise 7.23
1 1 1 *
The total of four numbers is 1214five. What is their average
1 0 1 0 1 two
expressed in base five?
Next, we do the subtraction; A. 303.10 B. 242.4 C. 141 D. 114
Note the order as 11000 – 10101 i.e 2009/4 Exercise 7.24
1 1 0 0 0 Arrange the following number in descending order of
1 0 1 0 1 magnitude: 22three, 34five, 21six
0 0 0 1 1 two A 21six, 22three, 34five B 21six, 34five, 22three
C 22three, 34five, 21six D 34five, 21six, 22three
2007/7 Neco
Solve 11101two + 11011two – 11001two
A 11001two B 11011two C 11100two
D 11110two E 11111two
Solution
First is addition
1 1 1 0 1
+ 1 1 0 1 1
1 1 1 0 0 0 two
Next: subtraction
1 1 1 0 0 0
– 1 1 0 0 1
1 1 1 1 1 two (E)
108
SURD 2009/1a
Irrational or non rational numbers are numbers whose Given that ( 3 − 5 2 ) ( 3 + 2 ) = a + b 6 find a and b
value have no ending Solution
Examples are 2 , 3 , 5 e.t.c the square root of all We simply open up the bracket as:
non perfect squares are irrational likewise their ( 3 − 5 2) ( 3 + 2) = 3 3 + 2 − 5 2 3 + 2 ( ) ( )
multiples and fractions. Also  is an example of
irrational numbers. we refer to the square root of
= 3 3 + 3 2 − 5 2  3 − 5 2  2
non-perfect square as SURD. = 3+ 6 – 5 6 – (5  2)
Surd has it own basic rule of operation with addition, 6 – 5 6 – 10
= 3 +
subtraction, division and multiplication.
Addition of surds = –7– 4 6
Example = – (7 + 4 6 )
Simplify: 2 + 3 2014/6
Solution Simplify: 12 48 − 3( )
2 + 3 = 2 + 3 A 18 B 16 C 14 D 12
Solution
But 2 + 3 ≠ 2 + 3 or 5 First, we open the bracket using multiplication rule in surds
Generally 12 ( 48 − 3 ) = 12  48 – 12  3
a + b ≠ a+b
= 12  12  4 – 433
Also a - b ≠ a −b
= 12 2  2 2 − 2 2  32
Multiplication of surds = 12  2 – 2  3
a × b = ab = 24 – 6
This true for all surds = 18 ( A )

Examples 2006/ 12a (Nov) Exercise 7.25


a. simplify 2 × 6 Simplify 3 (2 2 + 3 ) + 2 ( 3 − 2 ) , leaving the answer in
Solution surd form
2 × 6 = 2 6
= 12
Division in surds
= 4 3 = 4 × 3 =2 3 m
m
=
60 n n
b.Simplify
Solution This true for all surds
60 = 415
Examples
= 2 15 35
1. Simplify
5
c.Simplify 3 × 3
Solution Solution
35 35
3 × 3 = 33 =
5 5
= 9
=3 = 7
d. Simplify 80 + 20 - 45 17
2. Simplify
Solution 4
We bring out the perfect square factors in the given surd as: Solution
80 + 20 - 45 = 16 5 + 4 5 - 9 5 17 17 17
= =
= 16  5 + 4  5 - 9  5 4 4 2

=4 5 +2 5 - 3 5
= (4 + 2 -3) 5
=3 5

Generally, a  a = a
109
Rationalization of surds Rationalization of surd by Conjugate
Examples Surd Conjugate their product
1 2+ 3 2- 3 22 – 3
1. Rationalize
3
Solution 2- 3 2+ 3 22 – 3
1 1 3 Generally
=  a + b a- b a2 – b
3 3 3
The last term is a general expression for Surd and their
1  3 3 conjugates. Also from the table above we have seen that the
= =
3 3 3 product of a surd and its conjugate is not a surd since
Recall that a  a = a (a + b ) (a - b ) = a (a - b)+ b (a - b)
To rationalize, when the denominator is a single surd we = a2 - a b + a b - b  b
multiply the numerator and denominator by the surd in the = a2 – b
denominator.
2006/4b
10 2
2. Simplify 1+ 2
5 Simplify:
2− 3
Solution
Solution
10 2 10 2 5 We rationalize as:
= 
5 5 5 1+ 2 (1 + 3 ) (2 + 3 )
=
2 5 2− 3 (2 − 3 ) (2 + 3 )
= 10 = 2 10
5
1(2 + 3 ) + 3 (2 + 3 )
6 3 =
3. Simplify 2 3 - + 22 − 3
3 27
2+ 3+2 3 +3
Solution =
4−3
6 3 6 3 3 27
2 3 - + = 2 3 - + = 5+3 3
3 27 3 3 27  27
6 3 3 27 2008/9
=2 3 - + 7− 3
3 27 Rationalize the expression:
13 − 3
9 3
= 2 3 - 2 3 + 44 + 3 3 44 + 2 3 44 − 3 3
9 A B C
83 83 83
3 3
= = 1
3 3 44 − 4 3 44 − 2 3
9 D E
83 83
2006/ 1b
Solution
7 5
By rationalizing the denominator, simplify: 7− 3 (7 − 3 ) (13 + 3 )
7 =
13 − 3 (13 − 3 ) (13 + 3)
(leave your answer in surd form)
Solution 7(13 + 3 ) − 3 (13 + 3 )
=
7 5 7 5 7 132 − 3
=
7 7 7 91 + 7 3 − 13 3 − 3
=
7 35 169 − 3
=
7 88 − 6 3
=
= 35 166
2009/ 2 (Nov) Exercise 7.26 Divide through by 2
6 44 − 3 3
Simplify = (C)
3 83
2010/8b Neco
2 3
A B 3 C 2 3 D6 3 1 1
3 Without using tables, find the value of −
11 − 2 11 + 2
2006/ 38(Nov) Exercise 7.27
5
Solution
Rationalize and leave your answer in surd form Applying LCM to the denominators
3 1 1 11 + 2 − ( 11 − 2)
5 3 3 3 5 3 9 3 − =
A B C D 11 − 2 11 + 2 ( 11 − 2)( 11 + 2)
3 5 9 5
110
Applying the conjugation of surd to the denominator Simplification in surds
11 + 2 − 11 + 2 Generally, in solving a given question, sometimes we
= 2 employ one or more of the principles treated so far.
11 + 2 11 − 2 11 − 2
4 4 2012/2
= i.e
11 − 4 7 3 5  4 6
Simplify
2013/4 Neco 2 2 3 3
1
Rationalizing A 2 B 5 C 2 2 D 2 5
3− 6
Solution
3 6 3+ 6
A B C 6 D 12 E6 Applying multiplication rule in surd
6 3
3 5  4 6 3  4 5 6
Solution =
1 1( 3 + 6 ) 2 2 3 3 2  3 23
=
3− 6 (3 − 6 ) (3 + 6 ) 12 3 0
= i.e
3+ 6 6 6
= Applying division rule in surd
32 − 6
3+ 6 30
= (B) = 2
3 6
= 2 5 (D)
2006/5b NABTEB
2 2008/ 26 Neco (Dec)
Rationalize 2
4+3 2  2− 3
Simplify  

Solution  2 + 3 
Applying conjugate rule
2 2(4 − 3 2 ) A – 19 – 20 6 B – 19 + 20 6 C 19 – 20 6
=
4+3 2 (4 + 3 2 ) (4 − 3 2 ) D 19 + 20 6 E 49 – 20 6
 8−6 2  Solution
=   2
2
 4 − 3  3(2)   2− 3 2
  = ( 2 − 3 ) by law of indices
 2+ 3
8−6 2   ( 2 + 3)2
=
16 − 18 ( 2 − 3) ( 2 − 3)
8−6 2 =
= ( 2 + 3 )( 2 + 3)
−2
Opening up the brackets
= –4+3 2
2 ( 2 − 3) − 3 ( 2 − 3)
=
2 ( 2 + 3) + 3 ( 2 + 3)
2012/2 Neco Exercise 7.28 2− 6 − 6 +3
Which of the following represents the =
2+ 6 + 6 +3
conjugate of 3 + 2 ?
5−2 6
3− 2 =
A 2− 3 B C 3− 2 5+2 6
3+ 2
Rationalizing
D 3+ 2 E 3+ 2
(5 − 2 6 ) (5 − 2 6 )
3− 2 =
(5 + 2 6 ) (5 − 2 6 )
Exercise 7.29
3− 2 2 25 − 10 6 − 10 6 + (4  6)
Simplify : =
3 + 2 2 25 − 10 6 + 10 6 − (4  6)

A. 1 B. 17– 2 C. 17 + 12 2 D. 17 – 12 2 25 − 20 6 + 24
=
25 − 24
= 49 – 20 6 (E)
2011/32
If 72 + 32 − 3 18 = x 8 , find the value of x
A1 B 3/ 4 C 1/2 D 1/4
Solution
Simplifying the surds
111
72 + 32 − 3 18 = x 8 becomes 2005/6b
8 − 4 18
98 + 48 − 3 9 2 = x 8 Simplify: leaving your answer in the form
50
3 8 + 2 8 − 3 3 2 = x 8
a + b n where a and b are rational numbers and n is an integer.
5 8 −9 2 = x 8 Solution
We simplify both sides to the lowest surd of 2 First, we simplify the surds to their lowest terms
5 4 2 − 9 2 = x 4 2 8 − 4 18 8 − 4 9 2
=
5  2 2 – 9 2 = 2x 2 50 25  2
10 2 – 9 2 = 2x 2 8 − 43 2 8 − 12 2
= =
2 = 2x 2 5 2 5 2
Divide both sides by coefficient of x Rationalizing
2 2x 2 (8 − 12 2 )(5 2 )
= =
2 2 2 2 5 2 5 2
1
/2 = x (C)
40 2 − 60  2
2014/2a =
25  2
Simplify 3 75 − 12 + 108 , leaving the answer in surd
40 2 − 120
form (radicals) =
Solution 50
120 40
3 75 − 12 + 108 = 3 3  25 − 4  3 + 4  3  9 Keeping to the given instruction = − + 2
50 50
2 2 2 2
= 3 3 5 − 3 2 + 2  3 3 12 4
= − + 2
5 5
= 35 3 –2 3 +23 3
2005/32
= 15 3 – 2 3 + 6 3 6
Simplify − 3
= 19 3 48
2001/2a (Nov) − 3 2 3 3 3
Without using mathematical tables or calculator, simplify A B C D2 3
2 3 2
2 3 (3 3 − 2) given that 3 = 1.732 Solution
Solution Using the lowest surd 3 as standard
Expanding the bracket 6 6
− 3 = − 3
2 3 (3 3 − 2) = 6  3 – 4 3 48 16  3
6
= 18 – 4 3 = − 3
4 3
Substituting for 3
3
= 18 – 4(1.732) = − 3
= 18 – 6.928 2 3
= 11.072 32 3
2005/6a (Nov) Rationalizing = − 3
2 3 2 3
1 1
Simplify 5 + − 6 3
5 125 = − 3
43
Solution
1 1 1 1 3 − 3
5 + − = 5 + − = − 3 = (A)
2 2
5 125 5 25  5
2005/29(Nov)
1 1
= 5+ − 63 − 28
5 5 5 Simplify:
7
5 5 5
Rationalizing = 5 + − A 7 B2 7 C1 D5
5 25  5
Solution
5 5 63 − 28 97 − 47
= 5+ − =
5 25
7 7
Applying LCM to the denominators
3 7 −2 7
25 5 + 5 5 − 5 =
= 7
25
29 5 7
= = = 1 (C)
25 7
112
2006/49 = 4 2 −3 2 + 2
20 2
Simplify:
5 28 − 2 63 = 2 2 −3 2 + 2
5 7 5 5 7 5 7 7 =0 (A)
A B C D 2009/1b (Nov)
7 7 5 5
Solution Simplify without using tables or calculator
20 20 2 + 96 − 4( 6 − 1) 2 and express your answer in the form
=
5 28 − 2 63 5 47 − 2 97 m + n 6 where m and n are real numbers.
20 Solution
=
5 2 7 − 2 3 7 First we treat the bracket items
20 20 5 2 + 96 − 4( 6 − 1) 2 = 2 + 16  6 − 4( 6 − 1)( 6 − 1)
= = =
10 7 − 6 7 4 7 7
= 2 + 4 6 − 4(6 − 6 − 6 + 1)
5 7
Rationalizing = = 2 + 4 6 − 4(7 − 2 6 )
7 7
= 2 + 4 6 − 28 + 8 6
5 7
= (A) = – 26 + 12 6
7
2006/3b Neco (Dec) 2010/2a Neco
1 1 12
Simplify: + 3 2 − Express 243 + as a single surd.
2 3 2 3
Solution 13  12 
Hence, find the value of  243 + 
First, we rationalizing the fractional surds  
3 3
1 1 Solution
+ 3 2 −
2 3 2 Applying LCM to denominator
1 2 1 3 2 12 243  3 + 12
= + 3 2 − 243 + =
2  2 3 2 3 2 3 3
2 3 2 81 3  3 + 12
= + 3 2 − =
2 9 2 3
2 2 (9  3) + 12 27 + 12 39
= + 3 2 − = = i.e
2 6 3 3 3
Applying LCM to the denominators 13  12  13  39  507
Thus  243 +  =   = = 169
3 2 + 18 2 − 2 20 2 10 2   3  3 3
3 3
= = =
6 6 3
2010/20 Exercise 7.30
2007/5 6 3
Simplify: 2 3 − +
Simplify 3 45 − 12 5 + 16 20 , leaving the answer in 3 27
surd form
A1 B1 3 C 2 3 −52 D 6 3 − 17
A 29 5 B 14 15 C 12 15 D 11 5 3 3
Solution 2008/4 Exercise 7.31
10
Using the lowest surd 5 as standard Simplify: 50 +
2
3 45 − 12 5 + 16 20 = 3 9  5 − 12 5 +16 4  5
A 10 B 10 2 C 20 D 20 2
= 3  3 5 − 12 5 +16  2 5
2014/4 ( Nov) Exercise 7.32
= 9 5 − 12 5 + 32 5
If 2 2 + 125 – 45 + 4 = a + b c ,
= 29 5 ( A ) evaluate (2a – b)
2009/ 44 A. 8 B. 4 C. 2 D. 0
1
Simplify: 32 − 18 + 2 2015/32 Exercise 7.33
2
A0 B 2 C2 2 D4 2 2 + 3
If is simplified as m + n 6 ,
Solution 3
Using the lowest surd 2 as standard find the value of (m + n)
1 1 A. 1/3 B. 2/3 C. 12/3 D. 11/3
32 − 18 + 2 = 16  2 − 9  2 + 2
2 2

113
CHAPTER EIGHT E.g 3 The ages of 25 students in JSS I of Ibru college
Agbarha – Otor is given as follows;
Statistics I (ungrouped data)
7 8 10 15 8 9 7
Statistics is the science of collecting, organizing and
8 8 9 10 12 7 13
analyzing data for any given purpose. It helps us to
14 8 8 9 10 15 8
reduce large and scattered data to an understandable
7 10 11 11
level, thereby enabling us to make decisions in the face
arrange the above data in a frequency table.
of uncertainty. At one time or the other in our day to
Solution
day activities, we must have engaged statistical values
Arranging the above data in a frequency table.
in taking decision.
Ages Tally No of students ( freq.)
For instance, Zino’s parent wants to send Zino to a
7 IIII 4
college in Garki – Abuja. You will agree with them that
8 IIII II 7
there are so many schools to choose from. Through
9 III 3
enquiry about various schools’ qualities such as :
10 IIII 4
(i) School performance in JSCE & SSCE
11 II 2
(ii) Qualification of teaching staff
12 I 1
(iii) School location
13 I 1
(iv) Fees charged,
14 I 1
They will make a good decision on the best school that
15 II 2
suit them. By the above act, they have engaged in the
Total = 25
statistical activities of collecting, organizing and analysis
of data for the purpose of sending their child to school.
The centrality and spread of any given data are of great
Population
importance to statisticians. These two mentioned concepts
This is the totality of observation with which we are
enable us to make appropriate decision.
concerned. The whole Secondary Schools in Garki-
Abuja forms the population of the above example.
Measure of central tendency
Sample (measure of location)
It is basically the subset of population. You will agree Measure of central tendency can be thought of as a measure
with the author that it will not be Naira (Penny) wise for which gives the location of the “center” of the data. The most
Zino’s parents to check on all the schools in Garki – commonly used ones are the mean, median and mode.
Abuja. Rather they will check on a few, say about 3 to 8
schools (sample) among the over 50 secondary schools MEAN
The mean (arithmetic mean) denoted by x ( x bar) of a set of
Raw data
n numbers x1, x2, … xn is
They are collected data which have not been organized
x = x 1 + x2 + … + x n
numerically, i.e they have not been arranged either in
n
ascending or descending order.
x = x (where  is summation)
E.g The scores of 10 students who sat for a
n
mathematics test, are: 3, 5, 10, 0, 1,8, 10, 4, 2, 6.
2005/28
Find the mean of the numbers 1, 3, 4, 8, 8, 4 and 7
ARRAY A4 B5 C6 D7
This is the arrangement of raw numerical data in Solution
ascending or descending order of magnitude. From the x
Mean =
data supplied above: n
Ascending order: 0, 1, 2, 3, 4, 5, 6, 8, 10, 10 1+ 3 + 4 + 8 + 8 + 4 + 7
=
Descending order: 10, 10, 8, 6, 5, 4, 3, 2, 1,0. 7
35
= i.e 5 ( B )
Frequency & Tally 7
Frequency: this is the number of occurrences of any
value in a given data while tally is the stroke 2014/28 NABTEB
representation of frequency where: Calculate the mean of 1, 0, 1, 2, 3, 2, 0, 2, 3, 2
I represents 1 A 1.6 B 1.7 C 1.8 D 2.0
II represents 2 Solution
III represents 3 x
IIII represents 4 Mean =
n
IIII represents 5 1+ 0 +1+ 2 + 3 + 2 + 0 + 2 + 3 + 2
IIII I represents 6 =
10
IIII II represents 7 and so on. 16
= i.e 1.6 (A)
10

114
2009/ 37 NABTEB (Nov) 2008/59 Neco
The weight of 5 girls in kg are 48, x, 52, 50 and The ages to Abu, Segun, Kofi and Funmi are 17years,
(2x – 5). If their average weight is 47 kg. Find the (2x – 13) years, 14 years and 16 years respectively. What is
weight of the girl that has the heaviest weight the value of x if their mean age is 17.5 years?
A 85kg B 65kg C 55kg D 50 kg A 18 B 23 C 25 D 36 E 70
Solution Solution
sum of weight x
Average weight = Mean =
No of girls n
47 = 48 + x + 52 + 50 + (2 x − 5) 17.5 = 17 + (2 x − 13) + 14 + 16
5 4
47  5 = 150 + x + 2x – 5 17.5  4 = 47 + 2x – 13
235 = 145 + 3x 70 = 34 + 2x
235 – 145 = 3x 70 – 34 = 2x
90 = 3x 36 = 2x
30 = x 18 = x (A)
Next, we list the weight: 48, 30, 52, 50, 55
Thus the heaviest weight is 55kg ( C ) 2008/53 Neco
Twenty girls and y boys sat for an examination.
2004/15a NABTEB The mean marks obtained by the girls and boys were
The average cost of 20 articles is N 213.00; the average 62 and 57 respectively. If the total scores for both girls and
cost of the first 9 is N 115.00.Find the average cost of boys was 2950, find y
the remaining articles A 51 B 48 C 30 D 25 E 18
Solution Solution
x Let the twenty girls total score be x
Mean = Also
n Already, we are given y boys (not total score of boys)
x x x
213 = 115 = From Mean =
20 9 n
x = 20  213 x = 9  115 Girls Boys
= 4260 = 1035 x 2950 − x
62 = 57 =
Average cost of (20 – 9 article) 20 y
4260 − 1035 62  20 = x substituting for x = 1240
=
20 − 9 2950 − 1240
3225 1240 = x 57 =
= i.e 293.18 y
11
57y = 1710
2006/53 Neco (Nov)
y = 30 (C)
The mean age of 10 students is 10 years 6 months.
2011/54 Neco
If the age of two of the students, 12 years 10 months
The average of 10 boys was 12 years. A boy of 15 years was
and 13 years 6 months are subtracted from the ages of
replaced with that of 5 years. Find the new average age of
the ten students, what will be the mean age of the
the boys.
remaining 8 students?
A 7 years B 8 years C 9 years
A 4 years 1 months B 9years 6 months
D 10 years E 11 years
C 9 years 10 months D 12years 3 months
Solution
E 13 years 2months
Let the sum of the 10 boys’ ages be y, then
Solution
x
x Mean = here x is y
Mean = n
n
x i.e 12 = y
10.5 = 10
10 12  10 = y
10.5  10 = x i.e 105 y = 120
Sum ages of two students = 1210/12 + 13 6/12 But If 15 years boy was replaced by 5 years
= 25 + (0.83 + 0.5) Then the new y = 120 – 15 + 5
= 26.33years = 110
Then 26.33 years is subtracted from 105 year Our n i.e numbers of boys remain 10
and 2 is subtracted from 10 students
105 − 26.33 New average = 110 = 11 years (E)
Mean (8students) = 10
10 − 2 2007/52 Neco
78.67 The mean of a set of numbers 68, 65, x, 69, 77, 48, 64
=
8 is 67 find x
= 9.83 i.e 9 years 10 months ( C ) A 78 B 68 C 67 D 66 E 62
115
Solution Solution
x  fx
Mean = Mean =
n f
67 = 68 + 65 + x + 69 + 77 + 48 + 64 Table for mean showing x, f and fx
7
Scores(x) Frequency (f) fx
67  7 = 391 + x
469 = 391 + x 1 3 3
469 – 391 = x 2 4 8
78 = x (A) 3 3 9
4 6 24
2013/58 Neco Exercise 8.1 5 5 25
If the mean of 6, 8, 9, x, 5, 7 is 8. 6 4 24
What is the value of x?
7 3 21
A6 B8 C9 D 10 E 13
8 2 16
2007/30 Exercise 8.2 f = 30 fx = 130
Divide the sum of 8, 6, 7, 2, 0, 4, 7, 2, 3 by their mean
Substituting
A9 B8 C7 D6
Mean = 130 i.e 4.3 (C)
2008/39 NABTEB (Nov) Exercise 8.3 30
Calculate the mean of the distribution: 4, 3, 6, 8, 9, 3, 9
2011/59 Neco
A6 B7 C8 D9
The table below shows the scores of certain number of
students in a class.
Scores 2 4 5 6 7
No of students 5 3 6 4 2
59. Find the mean scores
Mean of frequency distribution A 4.8 B 4.5 C 4.0 D 3.8 E 3.5
For any given data say x1, x2, …, xk that occur f1, f2, Solution
…,fk times respectively, then the mean  fx
Mean =
x = f1x1 + f2x2 + … + fk xk f
f1 + f2 + …+ fk Table for mean showing x, f and fx
= fx Scores (x) Frequency (f) fx
f 2 5 10
2008/12c NABTEB (Nov) 4 3 12
Calculate the mean of the distribution of the test scores 5 6 30
below: 6 4 24
Scores 30 35 40 45 50 7 2 14
No of student 3 21 46 18 12 f = 20 fx = 90
Solution Substituting
 fx Mean =
90
i.e 4.5 (B)
Mean = 20
f
2005/44 (Nov)
Table for mean showing x, f and fx
The mean of the frequency distribution shown is 1.5. Find
Scores(x) Frequency(f) fx the value of p
30 3 90
x - value 0 p 3
35 21 735
40 46 1840 Frequency 2 3 1
45 18 810 Solution
50 12 600  fx
Mean =
f = 100 fx = 4075 f
Substituting Table for mean showing x, f and fx
Mean = 4075 i.e 40.75 x - value frequency(f) fx
100
0 2 0
p 3 3p
2005/49 Neco
3 1 3
Find the mean of the distribution
f = 6 fx = 3 + 3p
Scores 1 2 3 4 5 6 7 8
Frequency 3 4 3 6 5 4 3 2 Substituting
3 + 3p
1.5 =
A3 B 3.5 C 4.3 D 4.5 E5 6

116
1.5  6 = 3 + 3p Median
9 = 3 + 3p The median of a set of numbers arranged in an array is the
9 – 3 = 3p middle number (if their total number is odd) or the arithmetic
6 = 3p mean of the two middle numbers (if the number of the given
Thus, p = 2 (D) values is even). Median divides the data into two equal parts.

2010/58 Neco E.g 1 Find the median of the following


The table shows the distribution of scores of students in (a) 6, 1, 0, 3, 10, 2, 5
a test. If the mean score is 3.5, find the value of x (b) 5, 7, 11, 9, 15, 12, 18, 6
Scores 1 2 3 4 5 6 Solution
Arranging the value in an array, we have:
Frequency 1 4 x 6 2 2
(a) 0, 1, 2, 3, 5, 6, 10
A1 B2 C3 D4 E5 The given values is odd, thus median = 3 (middle)
Solution (b) 5, 6, 7, 9, 11, 12, 15, 18
 fx The given values is even, thus
Mean = Median = 9 + 11 Arithmetic mean of the two
f
Table for mean showing x, f and fx 2 middle values
= 10
Scores (x) Frequency (f) fx
1995/48
1 1 1
Find the median of the following numbers:
2 4 8
2.64, 2.50, 2.72, 2.91 and 2.35
3 x 3x
A2.91 B 2.71 C 2.64 D 2.50 E 2.35
4 6 24 Solution
5 2 10 First, we arrange the given data in ascending order: 2.35,
6 2 12 2.50, 2.64, 2.72, and 2.91
f = 15 + x fx = 55 + 3x Since there are 5 numbers (odd)
Substituting Median = 2.64 (middle number)
55 + 3x 2011/60 Neco
3.5 =
15 + x Calculate the median of the following set of numbers:
3.5(15 + x) = 55 + 3x 3, 2, 6, 8, 9, 6, 8, 12, 11, 12
52.5 + 3.5x = 55 + 3x A9 B8 C7 D6 E3
3.5x – 3x = 55 – 52.5 Solution
0.5x = 2.5 First, we arrange the given data in ascending order:
Thus, x = 5 (E) 2, 3, 6, 6, 8, 8, 9, 11, 12, 12
Since there are 10 numbers (even)
2008/57 Neco Exercise 8.4 8+8
Median = (Arithmetic mean of the two middle values)
Scores 0 1 2 3 4 5 2
No of students y 12 10 6 5 8 =8
The table above shows the test scores of students in a 2009/29
class. If the average score is 2.2, find the value of y. What is the median of the following scores
A 13 B 11 C 10 D9 E7 22, 41, 35, 63, 82, 74?
A 82 B 52 C 49 D 22
2013/57 Neco Exercise 8.5 Solution
The table below shows the distribution of some scores. First, we arrange the given data in ascending order:
Find the mean 22, 35, 41, 63, 74, 82
Scores 1 2 3 4 5 6 Since there are 6 scores (even)
41 + 63
Frequency 3 3 0 4 5 5 Median = (Arithmetic mean of the two middle values)
2
A 1.0 B 1.1 C 1.5 D 4.0 E 5.5 = 52 (B)

2007/53 Neco Exercise 8.6


Find the median of 16, 20, 8, 14, 12, 16, 10, 12, 7, 16
A 12 B 13 C 14 D 15 E 16

2014/28 Neco (Nov) Exercise 8.7


Calculate the median of the following set of numbers:
6, 8, 3, 2, 10, 6, 11, 9, 12, 4
A3 B6 C7 D8 E9

117
Median (frequency distribution) (a) Median working
f is 2 + 5 +13 +11 + 9 +10 i.e 50 which is even
To get median for frequency data, we apply the idea of
odd and even numbers to get the median class or Median class =  f
2
column among other columns or classes. f is another 50
important term for us to know, which is “ Summation = i.e 25th class
2
of frequency”. The two ways of getting our median Adding frequency cumulatively till we get to 25th class:
class or column are: 2 + 5 +13 +11 i.e 4th column,
(a) Median class (column) = f if f is even Thus, median = 4
2
(b) Median class (column) = f + 1 if f is odd 2007/12a NABTEB
2 The distribution of the daily wages in N 100.00 of some
After we have gotten our median class, then keep workers on a farm is as given below:
adding the frequency of the given data till we get to the Wages 2 3 4 5 6 8 10
class or column where our number falls in – that No of workers 2 4 10 11 15 10 3
position is our median. Examples on this will help us (a) How many workers are on the farm?
better (b) Calculate the ( ii ) median wage
2005/12 (Nov) Solution
A number of families were asked how many children (a) f = 2 + 4 + 10 + 11 + 15 + 10 + 3
they had. The results are as follows: = 55
No of children 1 2 3 4 5 6 (ii) Median wage working
Frequency 10 15 8 6 4 7 f is 55 which is odd
(a) Calculate the: ( ii ) median Median class =  f + 1
Solution 2
55 + 1
f = 10 + 15 + 8 + 6 + 4 + 7 =
2
= 50 is even 56
= i.e 28th class
Median class =  f 2
2
50
Adding frequency cumulatively to 28th class:
= i.e 25th class 2 + 4 +10 +11 +15 i.e 5th column
2
Thus median = 6
Adding frequency cumulatively till we get to 25th class:
= N 600.00 since wages is in N 100.00
10 + 15 i.e 2nd column
Thus median = 2

2011/11
The table shows the scores obtained when a fair die was
thrown a number of times
Scores 1 2 3 4 5 6
Frequency 2 5 x 11 9 10
If the probability of obtaining a 3 is 0.26,
(a) find the median
Solution
First, we find x by principle of probability
freq (3)
Prob of 3 =
f
x
0.26 =
2 + 5 + x + 11 + 9 + 10
x
0.26 =
37 + x
0.26(37 + x) = x
9.62 + 0.26x = x
9.62 = 1x – 0.26x
9.62 = 0.74x
9.62
= x
0.74
x = 13

118
Mode Measures of Spread (variation/dispersion)
The mode of a set of numbers is that value which occurs This is the degree to which data are scattered (dispersed) about
with greatest frequency. i.e. it is the most common value. its average value. The several measures of spread are the
The mode may not exist or it may not be unique (when range, mean deviation, inter quartile range, variance and
we have more than one mode) standard deviation

2009/29 NABTEB (Nov) 2014/30 Neco (Nov)


What is the mode of the scores below: Which of the following is not a measure of dispersion?
1, 2, 3, 4, 4, 5, 5, 5, 4, 2, 2, 3, 4, 5, 5, 6 A interquartile range B mean C mean deviation
A1 B3 C4 D5 D range E standard deviation
Solution Solution
Mode = 5 it occurred most (D) Mean is not a measure of dispersion (B)
2009/54 Neco (Nov) 2002/26 ( Nov) Exercise 8.9
The data below are the ages of a set of pupils Which of the following is / are not the measure(s) of
7, 9, 6, 10, 8, 8, 9, 11, 8, 7, 9, 6, 8, 9, 10, 7, 8, 7, 9, 8. dispersion?
What is the mode?
I Range
A 10 B9 C8 D7 E6 II Variance
Solution III mode
Mode = number that occurred most A 1 only B. II only C. III only D. 1 and II only.
=8 (C)
2009 /28 NABTEB (Nov) Exercise 8.8 Range
A type of average that shows the most popular item is This is the simplest measure of dispersion and it is defined as
A mean B median C mode D variances the difference between the largest and smallest numbers in the
set. Please Note that range is a poor measure of spread as it
uses only two extreme values.
2008/50 Neco
The ages of 10 students in a class are:
15, 16, 15.5, 17, 14.9, 14.5, 14.1, 15.1, 15.2, 14.8
Find the range of their ages.
A 6.1 B 4.8 C 2.9 D 2.1 E 1.9
Mode of frequency distribution Solution
Range = biggest number – smallest number
2009/16 NABTEB (Nov) = 17 – 14.1
Determine the mode of the distribution below = 2.9 ( C )
x 2 6 10
2013/54 Neco
f 5 10 9
A group of numbers are written in ascending order of
A5 B6 C9 D 10 magnitude as (x – 2), 8, (5 +x), 12, (x +14). Find the range
Solution of the number.
Mode = number that occurred most A 12 B 14 C 16 D 18 E 20
= 6 ( B ) it has highest frequency of 10 Solution
Range = biggest number – smallest number
= (x + 14) – (x – 2)
In ascending order, the first number is the smallest while the
last one is the biggest
= x + 14 – x + 2
= 16 (C)
2015/60 Neco
The table below shows the frequencies (f) of some children
of age x years.
x 1 2 3 5 6 7 8
f 3 4 5 7 6 5 4
Find the range of their ages
A1 B3 C4 D7 E8
Range = biggest number – smallest number
=8–1
=7 (D)

119
Interquartile and semi-interquartile range 2014/11b (Nov) Exercise 8.10
Quartiles The scores of students were recorded as follows
This is the division of the cumulative frequency 4 2 1 6 5
Into 4 portions. 3 5 6 1 2
(Lower quartiles ) Q1 = ¼ of cum freq, 1 5 5 6 3
4 3 5 1 5
(Middle quartile) Q2 = 2/4 of cum freq ( i ) Construct a frequency distribution table
(Upper quartile) Q3 = ¾ of cum. freq. ( ii ) Calculate the inter-quartile range
Inter quartile range = Q3 – Q1
Semi – inter quartile range = Q3 – Q1 2015/27 Exercise 8.11
2 The scores of twenty students in a test are as follows:
2007/55 Neco 44, 47, 48, 49, 50, 51, 52, 53, 53, 54, 58, 59, 60,
The marks of a certain number of students in a class are 61, 63, 65, 67, 70, 73, 75. Find the third quartile.
6, 7, 8, 12, 15, 8, 9, 5, 28, 15, 17, 21. A.62 B.63 C.64 D.65
What is the semi – interquartile range?
A4 B5 C 6.5 D8 E 15
Solution
First, we arrange the data in the table form (group) Mean deviation (M.D)
Marks 5 6 7 8 9 12 15 17 21 28 The mean deviation of a set of n numbers x1, x2 ,… xn is
Freq 1 1 1 2 1 1 2 1 1 1 defined as:
Cum freq 1 2 3 5 6 7 9 10 11 12
M.D = x – x
Q3 − Q1 n
Semi – interquartile range =
2 Where x is the mean of the numbers and  x - x is the
3
4
 12 − 1
4
 12 absolute value of the deviation of x from the mean ( x )
= and n is the number of digits given
2
9th position − 3rd position
= 2010/57 Neco
2
15 − 7 Find the mean deviation of 2, 4, 6, 5 and 3
= A 1.2 B 2.3 C 4.0 D 5.2 E 10
2
8 Solution
= i.e 4 ( A )
2  x− x
2013/51 Neco Mean deviation = But x is mean
n
The table below gives the shoe sizes of 12 students in a
2+4+6+5+3
class and here mean = i.e 4
5
Size 4 6 7 9 14
Mean deviation table
No of students 3 2 4 2 1 x x − x x−x
Find the semi – interquartile range
2 –2 2
A 0.7 B 1.3 C 1.5 D 1.8 E 2.8
Solution 4 0 0
Q3 − Q1 6 2 2
Semi – interquartile range = 5 1 1
2
3 1 3 –1 1
of cummulative freq − of cum. freq
= 4 4  x−x = 6
2
6
M.D = i.e 1.2 (A)
Size No of students (f) Cumulative freq 5
4 3 3
6 2 3+2=5 2002/ 38 (Nov)
7 4 5+4=9 Calculate the mean deviation of the numbers
9 2 9 + 2 = 11 0, –1, –3, 4, 5, 1
14 1 11 + 1 = 12 A0 B2 C 7/ 3 D 2 4/ 5
Solution
3
 12 − 1
 12  x− x
Semi – interquartile range = 4 4
Mean deviation = But x is mean
2 n
9th position − 3rd position 0 + (−1) + (−3) + 4 + 5 + 1 6
= and here mean = = i.e 1
2 6 6
7−4
= = 3/2 i.e 1.5 (C)
2

120
Mean deviation table Variance & Standard deviation
x x − x x−x The variance of a set of numbers x1, x2 … xk is given as
V =  ( x – x )2
0 –1 1 n
–1 –2 2 Where V is the variance and (x – x )2 is the square of the
–3 –4 4 deviation from the mean while standard deviation is the
4 3 3 square root of variance.
5 4 4
1 0 0 S.D =  (x – x )2
n
 x − x = 14 If the numbers x1, x2 ….xk occur with frequency
f1 , f2 …, fk respectively, then,
M.D = 14 i.e 2.33 Variance V =  f (x – x )2
6
2008/52 Neco Exercise 8.12 f
Find the mean deviation of 20, 25, 21, 27, 28, 29, to the
nearest whole numbers S.D = f (x – x )2
A2 B3 C4 D5 E6 f
1999/41 (Nov) Exercise 8.13 Eg1. Find the variance and standard deviation of the set
Calculate the mean deviation of the numbers 6, 7, 8, 9, 10 of numbers, 2, 5, 6, 3 and 4
A0 B 1.2 C 1.5 D4 Solution
Variance =  (x – x )2
2013/56 Neco Exercise 8.14
n
Find the mean deviation of the numbers 2, 3, 5, 6
But mean = 20 =4
A 1.0 B 1.2 C 1.4 D 1.5 E 1.6
5
x x-x (x – x )2
Mean Deviation (frequency Distribution) 2 -2 4
If x1, x2 ,… xk occur with frequency f1, f2 ,… fk 5 1 1
then the mean deviation is given by 6 2 4
f x− x 3 -1 1
Mean deviation = where x is mean 4 0 0
f
 (x – x )2 = 10
2014/11 Var = 10 = 2
Scores 1 2 3 4 5 6 5
Frequency 2 5 13 11 9 10 S.D =  (x – x )2 = 2 = 1.4
n
The table shows the distribution of outcomes when a die Eg 2. Calculate the variance and standard deviation of
is thrown 50 times. Calculate the: (a) mean deviation the frequency distribution below.
Solution x 1 2 3 4 5
f x− x f 2 1 2 1 2
Mean deviation = where x is mean
f Solution
 fx 1 2 + 2  5 + 3 13 + 4 11 + 5  9 + 6 10 Variance = f (x – x )2
x = =
f 50 f
200 But mean = 2 x 1 + 1 x 2 + 2 x 3 + 1 x 4 + 2 x 5
= i.e 4 2+1+2+1+2
50
Mean deviation table = 24 = 3
8
x x − x x−x f f x−x x x – x (x – x )2 f f (x – x )2
1 –3 3 2 6 1 -2 4 2 8
2 -1 1 1 1
2 –2 2 5 10
3 0 0 2 0
3 –1 1 13 13 4 1 1 1 1
4 0 0 11 0 5 2 4 2 8
5 1 1 9 9 8 18
6 2 2 10 20 V = f (x – x)2
f x − x = 58 f
= 18/8
58 = 2.25
M.D = i.e 1.16
50 and S.D is square root of variance 2.25 = 1.5
121
2002/3 Solution
The variance of given distribution is 25. what is the
standard deviation? SD =
(
 x−x )
2
But x is mean
A. 125 B. 75 C. 25 D. 5 n
Solution and here mean x = 6 + 0 + 4 + 3 + 2 i.e 3
Standard deviation is the square root of variance. 5
Table for SD
Thus SD = var iance
SD = 25 = 5 (D)
x x − x (x − x ) 2

6 3 9
2013/55 Neco 0 –3 9
Calculate the variance of the following set of numbers 4 1 1
below: 5, 11, 13, 14, 17 3 0 0
A 5.3 B 7.3 C 8.0 D 10.3 E 16.0 2 –1 1
( )
Solution 2
 x−x
Variance
− x
= 
(x )
2
But x is mean =20
n
here mean x = 5 + 11 + 13 + 14 + 17 i.e 12 SD =
20
= 2 (A)
5 5
Table for variance
x x − x (x − x ) 2
2013/50 Neco
Calculate the standard deviation of the following set of
5 –7 49
numbers: 2, 3, 4, 4, 5, 6
11 –1 1
A 1.29 B 1.35 C 2.04 D 2.50 E 3.56
13 1 1
Solution
14 2 4
17 5 25 SD =
 x−x ( )
2
But x is mean
 x−x( ) = 80
2 n
2+3+ 4+ 4+5+ 6
here mean x = i.e 4
Variance = 80 i.e 16.0 (E) 6
5 Table for SD

2012/27
x x − x (x − x ) 2

Given that the mean of the scores 15, 21, 17, 26, 18 and 2 –2 4
29 is 21, calculate the standard deviation of the scores 3 –1 1
A 10 B4 C5 D 30 4 0 0
Solution 4 0 0
5 1 1
SD =
 x−x( )
2
But x is mean i.e 21 given 6 2 4
n
Table for SD
(
 x−x ) = 10
2

x x − x (x − x ) 2

SD =
10
= 1.29 (A)
15 –6 36 6
21 0 0 2011/11b
17 –4 16 The table shows the scores obtained when a fair die was
26 5 25 thrown a number of times
18 –3 9
Scores 1 2 3 4 5 6
29 8 64
Frequency 2 5 x 11 9 10
(
 x−x ) = 150
2
If the probability of obtaining a 3 is 0.26,
(b) standard deviation of the distribution
150 Solution
SD =
6 First, we find x by principle of probability
freq (3)
= 25 = 5 ( C ) Prob of 3 =
f
2014/19 NABTEB (Nov) x
0.26 =
The standard deviation of the numbers 6, 0, 4, 3 and 2 is 2 + 5 + x + 11 + 9 + 10
A2 B1 C 1.5 D 2.2 x
0.26 =
37 + x
122
0.26(37 + x) = x Miscellaneous cases involving measures of
9.62 + 0.26x = x spread & location
9.62 = 1x – 0.26x
9.62 = 0.74x 2009/55 Neco (Nov)
9.62 If the mean and median of 170, 230, y, 215 and 235 are 210
= x
0.74 and x respectively, find the values of x and y
x = 13 A 215 and 215 B 215 and 210 C 215 and 200
D 210 and 200 E 210 and 215

(b) S.D =
(
f x−x )2 Solution
f Mean =  x
n

170 + 230 + y + 215 + 235
But x = 1 2 + 2  5 + 3 13 + 4 11 + 5  9 + 6 10 210 =
2 + 5 + 13 + 11 + 9 + 10 5
200 210  5 = 850 + y
= 2 + 10 + 39 + 44 + 45 + 60 = i.e 4 1050 = 850 + y
50 50
Table for SD 1050 – 850 = y
200 = y
x x − x (x − x ) 2 f (
f x−x )
2
Median’s working
1 –3 9 2 18 Arranging the data in ascending order:
2 –2 4 5 20 170, 200, 215, 230, 235 (odd)
3 –1 1 13 13 Median = 215 i.e x
4 0 0 11 0 Thus, x and y are 215 and 200 (C)
5 1 1 9 9
6 2 4 10 40 2009/53 Neco (Nov)
f = − If the mean of 2, 1, 2, 3, 1, 3, 4, 4, 5, 5 subtracted from the
f ( x − x)
2
50 range the result is
=100 A1 B2 C3 D4 E5
Solution
100
SD = Mean =  x
50 n
2 +1+ 2 + 3 +1+ 3 + 4 + 4 + 5 + 5
= 2 = 1.4 = = 30 i.e 3
10 10
2005/44 Neco Exercise 8.15
Range = 5 – 1 i.e 4
Calculate the standard deviation of 5, 5, 6, 6, 8
A 8.0 B 6.0 C 3.3 D 1.2 E 1.1 Thus, range – mean = 4 – 3
= 1 (A)
2006/51 Neco Exercise 8.16 2014/32 to 34 Neco (Nov)
Find the standard deviation of 3, 4, 5, 7, 6 The percentage scores of ten students in an examination are
A 1.41 B 2.00 C 4.00 D 4.47 E 5.00 20, 60, 50, 80, 70, 90, 20, 20, 60, and 30
2007/54 Neco Exercise 8.17 Use the information to answer questions 32 to 34
Find the standard deviation of 2, 5, 9, 2, 7 32. Find the mean score
(correct to 2 d.p) A 30 B 50 C 60 D 70 E 80
A 5.80 B 3.41 C 2.76 D 1.80 E 1.34 Solution
Mean = 
2009/56 Neco (Nov) Exercise 8.18 x
Calculate the standard deviation of the following set of n
numbers: 4, 5, 6, 7, 8 20 + 60 + 50 + 80 + 70 + 90 + 20 + 20 + 60 + 30
=
A 1.4 B 2.0 C 2.5 D 2.9 E 6.0 10
500
= i.e 50 (B)
10
33. What is the median score?
A 50 B 55 C 60 D 70 E 80
Solution
First, we arrange the given data in ascending order:
20, 20, 20, 30, 50, 60, 60, 70, 80, 90
Since there are 10 scores (even)
Median = 50 + 60 (Arithmetic mean of the two middle values)
2
110
= i.e 55 ( B )
2

123
34. What is the modal score? 57. Calculate the standard deviation of the data,
A 20 B 30 C 35 D 50 E 55 correct to one decimal place.
Solution A 2.7 B 3.8 C 5.4 D 7.2 E 8.1
Mode = 20 ( A ) the number that occurred most Solution

SD =
(
 x−x ) 2
But x is mean
2008/51 Neco n
Marks 1 2 3 4 5 6 The mean is gotten as 29 from question 55
Frequency 8 11 13 14 9 5 Table for standard deviation
What is the sum of the median and mode of the above
x x − x x− x
2
( )
distribution? 22 –7 49
A5 B7 C8 D9 E 10 27 –2 4
Solution
34 5 25
Median’s working for frequency data
20 –9 81
f is 8 + 11 + 13 + 14 + 9 + 5 i.e 60
42 13 169
Median class =  f
2 (
 x−x ) = 328
2

60
= i.e 30th class 328
2 SD = = 65.6  8.1 to 1 d.p ( E )
Adding frequency from the beginning till we get to the 5
30th class: 8 + 11 + 13 i.e 3rd column 2011/55 – 56 Neco
Thus median = 3 mark Given the following scores: 2, 3, 4, 5, 6, use the information
Mode’s working to answer questions 55 and 56
Mode = 4 (has highest frequency) 55. Find the standard deviation
A 2.0 B 1.4 C 1.3 D 1.2 E 1.1
Thus, sum of median and mode = 3 + 4 Solution

2014/55 to 57
=7 (B)
SD =
(
 x−x ) 2
But x = 2 + 3 + 4 + 5 + 6 = 4
n 5
Use the set of data below to answer questions 55 to 57.
22, 27, 34, 20, 42 Table for standard deviation

55. Find the mean of the data


x x − x x− x ( )2

A 22 B 24 C 27 D 29 E 36 2 –2 4
Solution 3 –1 1
Mean =  x 4 0 0
n 5 1 1
22 + 27 + 34 + 20 + 42
= 6 2 4
5
=
145
i.e 29 ( D )  x−x ( ) = 10
2

5
10
56. Calculate the mean deviation SD = = 2 i.e 1.4 (B)
A 4.6 B 6.0 C 7.2 D 9.0 E 11.0 5
Solution 56. Find the variance
A 1.20 B 1.44 C 1.69 D 1.96 E 2.00
 x− x
Solution
Mean deviation = But x is mean
n
SD = Variance
The mean is gotten as 29 from question 55
Thus, Variance = (standard deviation)2
Table for mean deviation
= ( 2 )2 = 2 ( E )
x x − x x−x 2012/2 Neco
22 –7 7 A survey was carried out to investigate the number of eggs
27 –2 2 in some bird’s nests. The table below shows the findings:
34 5 5 No of eggs frequency
2 11
20 –9 9
3 12
42 13 13 4 25
Σ x − x = 36 5 30
6 12
7 10
Mean deviation = 36 i.e 7.2 (C) (a) Calculate the mean number of eggs in the nests
5
(b) Calculate the mean deviation
124
(a) Solution 2014/ 9b NABTEB (Nov)
 fx The figure below shows the number of goals scored by a
Mean =
f group of players in a match
Table for mean showing x, f and fx No of goals 0 1 2 3 4 5 6
No of eggs (x) Frequency (f) fx Frequency 26 29 14 15 5 6 3
2 11 22 Calculate the: ( i ) mean ( ii ) mode
3 12 36 ( iii ) standard deviation of the number of goals
4 25 100 Solution
5 30 150  fx
(i) Mean =
6 12 72 f
7 10 70 Table for mean showing x, f and fx
 f =100  fx = 450 No of goals (x) Frequency (f) fx
0 26 0
Mean = 450 i.e 4.5 1 29 29
100
2 14 28
b. Solution 3 15 45
f x− x 4 5 20
Mean deviation = where x is mean 4.5 5 6 30
f 6 3 18
Mean deviation table  f = 98  fx = 170
x x − x x−x f f x−x Substituting
2 – 2.5 2.5 11 27.5 Mean = 170 = 1.73  2 goals
3 – 1.5 1.5 12 18 98
4 – 0.5 0.5 25 12.5 (There is nothing like 1.73 goals)
5 0.5 0.5 30 15
6 1.5 1.5 12 18 ( ii ) Mode = 1
7 2.5 2.5 10 25
f = 100 f x − x = 116
( iii ) S.D =
f x−x ( ) 2

f
Mean deviation = 116 i.e 1.16 Table for SD
( ) ( )
100
x x − x x− x
2 f 2
f x−x
2010/59 Neco 0 –2 4 26 104
The table gives the scores of a group of students in a 1 –1 1 29 29
mathematics test. Use the information for question 59 2 0 0 14 0
Scores 2 3 4 5 6 8 3 1 1 15 15
No of students 3 4 2 7 2 2 4 2 4 5 20
5 3 9 6 54
59. What is the sum of the mode and the median? 6 4 16 3 48
A5 B7 C 10 D 12 E 20 f = −
f ( x − x) =
2
Solution 98
Mode = 5 (it has the highest frequency) 270
Median’s working
f is 20 even 270
SD = = 2.755 = 1.66
Median class =  f 98
2 We left it in decimals since it is a deviation of the number of
20
= i.e 10th class goals and not number of goals.
2
Adding frequency from the beginning till we get 2005/12 (Nov)
to 10th class: 3 + 4 + 2 + 7 i.e 4th column A number of families were asked how many children they
Thus, median = 5 had. The results are as follows:
Sum of mode and median = 5 + 5 No of children 1 2 3 4 5 6
= 10 (C) Frequency 10 15 8 6 4 7
(a) Calculate the: ( i ) mean
( b ) Find the standard deviation of the distribution
correct to two decimal places

125
Solution 2010/5 Neco
 fx The table below shows the distribution of scores in
(i) Mean =
f percentage obtained by 20 students in a class test.
Table for mean showing x, f and fx Scores 20 25 30 35 40 45 50
No of children (x) f fx No of students 5 3 1 6 2 1 2
1 10 10 b. ( i ) Find the mode ( ii ) median of the distribution
2 15 30 c. Calculate the mean score
3 8 24 Solution
4 6 24 b. ( i ) Mode = 35 (it has the highest frequency)
5 4 20
6 7 42 ( ii ) Median’s working
 f = 50  fx = 150 f is 20 which is even
Median class =  f
Mean = 150 i.e 3 2
50 20
= i.e 10th class
( b ) SD =
(
f x−x )2 −
but x = 3
2
Adding frequency cumulatively till we get to 10th class:
f 5 + 3 + 1 + 6 i.e 4th column
Table for SD Thus median = 35

x
x − x (
x− x
2
) f
f x−x ( )
2

C. Mean’s working
1 –2 4 10 40  fx
Mean =
2 –1 1 15 15 f
3 0 0 8 0 Table for mean showing x, f and fx
4 1 1 6 6 Score (x) Frequency (f) fx
5 2 4 4 16 20 5 100
6 3 9 7 63 25 3 75
f = −
30 1 30
f ( x − x) =
2
50 35 6 210
140
40 2 80
140 45 1 45
SD =
50 50 2 100
= 2.8  1.67 to 2dp f = 20 fx = 640
Substituting
2014/38 – 40 NABTEB (Nov) 640
Mean = i.e 32
The data below shows the frequency distribution of 20
marks scored by a group of students in a class test.
Use the information to answer questions 38 – 40 2006/10
The following table shows the distribution of test scores in a class
Score 2 3 4 5 6
Frequency 2 4 5 3 1 Scores 1 2 3 4 5 7 8 9 10
No of pupils 1 1 5 3 k2 +1 6 2 3 4
38. How many students took the test?
A 13 B 14 C 15 D 18 (a) If the mean score of the class is 6, find the
Solution ( i ) value of k ( ii ) median score
f = 2 + 4 + 5 + 3 + 1 Solution
= 15 (C)  fx
( i ) Mean =
f
39. What is the modal mark?
6 = 1  1 + 2  1 + 3  5 + 4  3 + 5(k 2 + 1) + 7  6 + 8  2 + 9  3 + 10  4
A2 B3 C4 D5
Solution 1 + 1 + 5 + 3 + (k 2 + 1) + 6 + 2 + 3 + 4
2
Mode = 4 ( C ) 6 = 1 + 2 + 15 + 12 + 5k + 52 + 42 + 16 + 27 + 40
25 + k + 1
40. Find the mean mark
6 = 160 + 5k2
2
A2 B1 C 3.8 D 1.3
Solution 26 + k
 fx 6(26 + k2) = 160 + 5k2
Mean = 156 + 6k2 = 160 + 5k2
f
2  2 + 3  4 + 4  5 + 5  3 + 6 1 6k2 – 5k2 = 160 – 156
= k2 = 4
2 + 4 + 5 + 3 +1
4 + 12 + 20 + 15 + 6 57 k=2
= = i.e 3.8 (C) Thus frequency k2 + 1 is 22 + 1 = 5
15 15
126
(ii) Median’s score working Table for mean showing x, f and fx
f is 30 which is even Mass(kg) x No of students (f) fx
Median class =  f 55 4 220
2 56 7 392
30 57 12 684
= i.e 15th class
2 58 10 580
Adding frequency cumulatively till we get to 10th class: 59 8 472
1 + 1 + 5 + 3 + 5 i.e 5th column 60 9 540
Thus median = 5 f = 50 fx = 2888

Mean = 2888
2006/5a Neco (Nov) 50
Age 11 12 13 14 15 = 57.76  58 to the nearest kg ( C )
Frequency 5 2 x–1 x+1 3
58. The median of the distribution is
The table above gives the frequency distribution ages of
A 59kg B 58kg C 57kg D 56kg E 55 kg
students in a class. If the mean age is 13 years, find the:
Solution
( i ) value of x ( ii )median age and ( iii )modal age
Solution f is 50 which is even
Median class = 
f
 fx
( i ) Mean = 2
f 50
11  5 + 12  2 + 13( x − 1) + 14( x + 1) + 15  3 = i.e 25th class
13 = 2
5 + 2 + ( x − 1) + ( x + 1) + 3 Adding frequency from the beginning till we get to
55 + 24 + 13x − 13 + 14 x + 14 + 45 25th class: 4 + 7 + 12 +10 i.e 4th column
13 =
10 + x − 1 + x + 1 Thus, median = 58kg (B)
125 + 27 x
13 =
10 + 2 x 2005/48 Neco
13(10 + 2x) = 125 + 27x The table below shows the scores of a set of students in a
130 + 26x = 125 + 27x mathematics test. If x is the mode and y is the median, find
130 – 125 = 27x – 26x (x, y)
5=x Marks 4 5 6 7 8
Thus x – 1 is 4 and x + 1 is 6 Frequency 5 9 8 6 2

The new table A (5, 5) B (6, 5) C (5, 6) D (8, 9) E (9, 8)


Solution
Age 11 12 13 14 15
Mode = 5 (it has the highest frequency)
Frequency 5 2 4 6 3
Median’s working
( ii ) Median age
f is 5 + 9 + 8 + 6 + 2 is 30 which is even
f is 5 + 2 + 4 + 6 + 3 i.e 20 even
Median class = 
f
Median class = 
f
2
2 30
20 = i.e 15th class
= i.e 10th class 2
2
Adding frequency cumulatively to the 15th class:
Adding frequency from the beginning till we get to
5 + 9 + 8 i.e 3rd column
10th class: 5 + 2 + 4 i.e 3rd column
Thus median = 6
Thus median = 13 age
(x, y) is mode, median i.e (5, 6) C.
(iii) Modal age is 14
2007/12a NABTEB
2006/57 – 58 Neco (Nov)
The distribution of the daily wages in N 100.00 of some
Mass (kg) 55 56 57 58 59 60 workers on a farm is as given below:
No of students 4 7 12 10 8 9
Wages 2 3 4 5 6 8 10
The table above shows the masses to the nearest No of workers 2 4 10 11 15 10 3
kilogram of 50 students in a class
(a) How many workers are on the farm?
57. What is the mean mass to the nearest kilogram? (b) Calculate the ( i ) mean wage
A 56 B 57 C 58 60 ( iii ) Modal wage.
Solution Solution
 fx (a) f = 2 + 4 + 10 + 11 + 15 + 10 + 3
Mean =
f = 55

127
 fx 2005/47 Neco Exercise 8.20
(b) Mean =
f Find the sum of the median and range of the following
numbers 16, 13, 10, 11, 26, 9, 8, 38 and 14.
= 2  2 + 3  4 + 4 10 + 5 11 + 6 15 + 8 10 + 10  3 A 49 B 43 C 30 D 17 E 13
2 + 4 + 10 + 11 + 15 + 10 + 3
4 + 12 + 40 + 55 + 90 + 80 + 30
=
55 2005/8 Exercise 8.21
311 In a family survey, the numbers of children in some family
=
55 are 4, 3, 1, 2, 4, 5, 1, 4, 3. By how much does the mode
= 5.65 = N 565.00 (wages in N 100.00) exceed the mean?
A0 B1 C2 D5
(iii) Modal wage is 6 = N600(wages in N 100.00)
2006/12a NABTEB (Nov) Exercise 8.22
2008/53 Neco (Nov) Below are amounts of money given to 15 students as gifts in
Considering this set of numbers 3, 3, 4, 5, 6, 6, 6, 7. a school, in naira?
The modes, median and mean are respectively 2 3 7 5 3 9 5 6,
A 5, 5.5, 6 B 5.5, 6, 5 C 6, 5, 5.5 4 5 6 6 7 5 9
D 5.5, 5, 6 E 6, 5.5, 5 Calculate the ( i ) mean ( ii ) mode and
Solution ( iii ) median, to the nearest ten kobo
Mode = 6
2006/54 -56 Neco (Nov) Exercise 8.23
Median’s working For a class of 30 students, the scores a mathematics test out
The given data is already in ascending order: of 10 marks are as follows:
3, 3, 4, 5, 6, 6, 6, 7 (8 of them even) 4 5 7 2 3 6 5 5 8 9
5 4 2 3 7 9 8 7 7 7
Median = 5 + 6 (arithmetic mean of the two middle values)
2 3 4 5 5 2 3 6 7 7 2
= 5.5
54. What is the mode of the scores?
Mean’s working A3 B4 C5 D6 E7
x
Mean = 55. What is the median of the score?
n
A3 B4 C5 D6 E
= 3+3+ 4+5+6+6+6+7 = 40
i.e 5
8 8
56. What is the range of the distribution?
Mode, median and mean: 6, 5.5, 5 (E) A2 B7 C8 D9 E 10

2008/35 NABTEB (Nov) 2015/16 Neco Exercise 8.24


The following are scores of 20 students in a Find the product of the median and the mode in the table
mathematics test: 10, 2, 6, 5, 4, 4, 5, 6, 6, 4, 6, 1, 8, 9, 7, below:
7, 6, 6, 5, 6. Calculate the median and the mode of this Score 0 1 2 3 4 5
distribution Frequency 2 4 4 9 6 8
A (6, 6) B (6, 5) C (6, 7) D (5, 6) A.6 B.8 C.9 D.12 E.15
Solution
Median’s working 2015/20 to 22 Neco Exercise 8.25
First, we arrange the given data in ascending order Use the table below to answer questions 20 to 22
1, 2, 4, 4, 4, 5, 5, 5, 6, 6, 6, 6, 6, 6, 6, 7, 7, 8, 9, 10 Score 1 2 3 6 8 9
Since there are 20 numbers (even) Frequency 4 3 5 7 2 1
6+6 20. Find the mean score
Median = (arithmetic mean of the two middle values)
2 A 3.8 B 4.0 C 4.2 D 4.4 E 4.6
= 6
21. Find the median score
Mode = 6 A2 B3 C7 D8 E9
Thus median, mode is (6, 6) A.
21. What is the modal score?
A5 B6 C7 D8 E9
2002/12a NABTEB Exercise 8.19
The profits made by a hawker in 5 consecutive days of
selling are: N 8, N15, N20, N12, N15.
( i ) What is the range of the profits?
( ii ) Calculate, the mean, the median and the standard
deviation of the profits
128
Moving average July sales + Aug sales + Sept sales
October prediction =
This is a tool in statistics used to predict or forecast 3
future results using past records. Any forecast is the 370 + 360 + 410
average of the preceding n months’ actual records =
3
1140
= = 380
Example MA1 3
The number of books in thousands sold by OPL Sales
Representative in the year 2014 is given in the table Aug sales + Sept sales + Oct sales
below: November prediction =
3
Months Jan Feb Mar Apr May Jun = 360 + 410 + 450
Books sold 450 440 460 410 380 400 3
1220
July Aug Sep Oct Nov Dec = = 407
3
370 360 410 450 470 490
Calculate three months moving averages Sept sales + Oct sales + Nov sales
December prediction =
Solution 3

( i ) April prediction =
Jan sale + Feb sales + Mar sales = 410 + 450 + 470
3 3
1330
450 + 440 + 460 = = 443
= 3
3
1350
= = 450 Example MA2
3
The number of books in thousands sold by OPL sales
Feb sales + Mar sales + Apr sales representative in the year 2014 is given in the table below:
May prediction =
3 Months Jan Feb Mar Apr May Jun
440 + 460 + 410 Books sold 450 440 460 410 380 400
=
3
1310 July Aug Sep Oct Nov Dec
= = 437
3 370 360 410 450 470 490
Mar sales + Apr sales + May sales Calculate the six monthly - moving averages
June prediction = Solution
3
460 + 410 + 380 July prediction
=
3 = Jan sale + Feb sales + Mar sales + Apil sales + May sales + Jun sales
1250 6
= = 417 450 + 440 + 460 + 410 + 380 + 400
3 =
6
Apr sales + May sales + Jun sales =
2540
= 423
July prediction = 6
3
410 + 380 + 400
= August prediction
3
1190 = Feb sales + Mar sales + Apil sales + May sales + Jun sales + July sales
= = 397 6
3
440 + 460 + 410 + 380 + 400 + 370
=
May sales + Jun sales + July sales 6
August prediction = 2460
3 = = 410
380 + 400 + 370 6
=
3
1150 September prediction
= = 383 = Mar sales + Apil sales + May sales + Jun sales + July sale + Aug sales =
3 6
460 + 410 + 380 + 400 + 370 + 360
Jun sales + July sales + Aug sales
September prediction = 6
3 2380
400 + 370 + 360 = = 397
= 6
3
1130
= = 377 October prediction
3 =
April sales + May sales + Jun sales + July sales + Aug sales + Sept sales
6

129
= 410 + 380 + 400 + 370 + 360 + 410 August prediction =
May + June + July
6 3
= 2330 = 388 =
40 + 52 + 48
6 3
140
November prediction = = = 47
3
May sales + Jun sales + July sales + Aug sales + Sept sale + Oct sales
6
June + July + August
= 380 + 400 + 370 + 360 + 410 + 450 September prediction =
3
6
= 2370 = 395 = 52 + 48 + 50
6 3
150
= = 50
December prediction = 3
Jun sales + July sales + Aug sales + Sept sale + Oct sales + Nov sales
6 July + August + Septeber
October prediction =
= 400 + 370 + 360 + 410 + 450 + 470 3
6 48 + 50 + 55
=
2460 3
= = 410 153
6 = = 51
3

2005/13 NABTEB August + Sept + Oct


November prediction =
The prices of kerosene per litre on the first week of each 3
of the 12 months of the year are as given in the table 50 + 55 + 43
=
below 3
Months Jan Feb Mar Apr May Jun 148
= = 49
Prices(N) 18 21 25 30 40 52 3

July Aug Sep Oct Nov Dec Sept sales + Oct sales + Nov sales
48 50 55 43 26 18 December prediction =
3
Calculate the three – monthly moving averages for the =
55 + 43 + 26
period 3
Solution 124
= = 41
Jan + Feb + March 3
April prediction =
3
18 + 21 + 25 2008/12b NABTEB (Nov)
= Calculation the 4 – point moving averages for the
3
64 distribution: 17, 14, 11, 18, 15, 14, 13, 14
= = 21 Solution
3
Feb + March + Apirl Point 15 prediction = 17 + 14 + 11 + 18 = 60 = 15
May prediction = 4 4
3
= 21 + 25 + 30 Point 14 prediction =
14 + 11 + 18 + 15
=
58
= 14.5
3 4 4
76
= = 25
3 11 + 18 + 15 + 14 58
Point 13 prediction = = = 14.5
March + April + May 4 4
June prediction =
3 18 + 15 + 14 + 13 60
25 + 30 + 40
Point 14 prediction = = = 15
= 4 4
3
95 2002/15b NABTEB Exercise 8.26
= = 32
3 Calculate the 4 – point moving average for the distribution:
Apirl + May + June 25, 30, 32, 28, 27, 20, 16, 18 to two decimals
July prediction =
3
30 + 40 + 52 2004/12b NABTEB (Nov) Exercise 8.27
=
3 Calculate the 5 – point moving average for the distribution:
=
122
= 41 32, 31, 27, 28, 20, 18 and 16
3

130
CHAPTER NINE Subsets
SETS This is the child or children of a given universal set
Set is the collection of objects. (Objects mean numbers, E.g. 1  = {a, b, c, d, e}
things, places etc). Generally speaking, the above X = {a, b, c}
definition is acceptable but for precision; Y = {a, k, c, d}
* set is the collection of well defined objects. X is a subset of  denoted by  or  i.e. x  
Also, we say that the universal set  contains x i.e.
Eg Suppose a new teacher is posted to your school to   x. While y   and   y means y is not a subset of
teach mathematics in SSII; he goes to the library to get
 and  does not contain y respectively.
some textbooks. Below is the Librarian response to his
question. Mathematics teacher: Sir, where can I get SSII
Proper and improper subsets
maths texts?
Any universal set is an improper subset of itself.
Librarian response; (I) You will get some in the
Also the empty set is an improper subset of all sets but every
mathematics shelves
other subset originating from the universal set is a proper
(II) You will get some in shelf
subset.
2 of the mathematics shelves.
Of course, both responses are correct but statement II is Power sets
precise. We usually denote sets by Capital letters while This is the collection of all the subsets of a given set
their members called ELEMENTS by lower case say s, denoted by 2s
letters. Elements(members) of any set are usually E.g list all the subsets of
enclosed in a curly bracket and frequently we may write A = { 1, 2, 3, }
for short. The power sets = 2s
{x : x satisfies a certain condition } or = 23
{ x/x satisfies a certain condition} = 8 (eight subsets)
They are: Ø, { 1, 2, 3}, { 1}, { 2 } , {3 }, {1,2 },
Where the signs / or : means “such that” {1, 3 }, { 2, 3}
Universal sets
This is the mother set from which other subsets emerge Equal sets
Two or more sets are equal if they contain the same
or relate and it is denoted by  or U
elements, order of arrangement immaterial
E.g. 1  = {All SSIII students of Esi College, Warri} E.g If X = {a , b, c, d}
These students include: Y = {b, d, a, c, d}
A = All science students Z = {first four letters of the alphabet}
B = All social science students K = {a, b, c, d, e, f }
C = All arts students We say that X = Y i.e. They contain the same elements when
E.g. 2  ={x : x is the set of all workers Living in Abuja} counted once;letter d repetition in Y is immaterial.
Then the subsets will include: Also X = Z but X ≠ K, since k contains e and f which are
S = {All civil servants} not elements of X
T = {All taxi drivers}
V = {All private sector workers} Equivalent sets
Two or more sets are equivalent when they contain equal
Singleton Sets elements denoted by ≡.
These are single element (member) sets E.g Which of the sets listed below are equivalent.
E.g. 1. A = {x : x is the principal of Ibru college A = {1, 2, 3, 4}
Agbarha-Otor} B = {a, c, d, e}
C = {a, b, b, e, c, d}
E.g. 2 B = {x : x is the president of Nigeria} A ≡ B since one element of A can pair with only
You will agree with the assertion that there is only one element of B without remainder. But A and B
one principal in a college and one president in a are not equal. Neither A nor B is equivalent to C.
country at a given time. This is so because C has a sixth element, which can
not be paired. Though B is equal to C when the elements of
Null or empty set C are counted once.
This type of set contains no element. It is the subset of
every set and is denoted by  or { } Other set Notations
E.g. W = {y : y is the set of a JSS one student who “Element of’’ is denoted by 
is a principal} E.g 1. Y = {x : x is between 10 and 15}
=  Then 11Y, 12 Y and 13 Y but any number outside the
Note : The set J = {0} is not an empty set because range is not an element of Y. Denoted by . For instance
zero is an element. 30  Y means that 30 is not an element of Y.

131
Cardinality of set: refers to the number of elements in a 2008/13 Neco (Dec)
given set - Say X and it is denoted by n(X). Given a universal set U = {2, 3, 4, 5, 6, 7, 8, 9, 10} and the
E.g 1 If X = {i, j, k, l} subsets A = {3, 5, 6, 7, 10}, B = {2, 5, 8, 9, 10}
Then the Cardinality of X is n(X) = 4 Find A1B1
A {4} B {5} C {3, 4} D {4, 5} E {5, 10}
E.g 2 If B = {a, b, c, d, e, f} then find n(B) Solution
Solution A1 = {2, 4, 8, 9} and B1 = {3, 4, 6, 7}
n(B) = 6 A1 B1 = {4} A.

2010/15 Finite set


Given that T = {x : – 2< x  9} where x is an integer. These are countable sets.
What is n(T)? E.g. 1 T = {All the State Governors in Nigeria}
A 9 B 10 C 11 D 12 E.g. 2 X = {All the students who registered for
Solution NECO exams in 2002}
n(T) Implies number of elements in set T Even a primary school pupil can easily count all the
First, we list the members of set T governors in the 36 states of Nigeria. The example 2 is more
T = {–1, 0, 1, 2, 3, 4, 5, 6, 8, 9} detailed because NECO headquarters has the total figure of
– 2 was not included because of the strict inequality there students who sat for the exam in the year 2002.
9 was included because of the weak inequality there
n(T) = 11 C.
Infinite set
These are uncountable sets
Union
The union of two or more sets is the combination of E.g.1 D = {x : x  ℕ}
elements of the sets without repetition. It is denoted by ={All natural or counting number}
 and not U. E.g 2. F = {x: x  ℤ}
Mathematically if A and B are two sets then, = {Set of all integers}
A  B = {x / x  A or x B} = {…, - 2, - 1, 0, 1, 2, 3, …}
E.g 1 If A = {1, 2, 3, 4, 5, 6}
B = { 5, 2, 2, 7,1, 3, 2} 2014/14 Neco ( Dec ) Exercise 9.0
then find A  B. Which of the following is an example of an infinite set?
A { Even numbers less than 20 } B { Multiples of three }
Solution
C { Number of lecturers in Nigerian Universities }
A  B = {1, 2, 3, 4, 5, 6, 7} D { Number of Local Governments in Nigeria}
E { Prime numbers greater than two but less than 97 }
E.g 2 List the elements of X Y
If X = {a, b, c, d} and Y = {a, d, e, a, f }
Disjoint set
Solution
Two sets A and B are said to be disjoint when they have no
X Y = {a, b, c, d , e, f }
element in common i.e. A  B = 
Intersection Non disjoint set
The intersection of two or more sets has a result of a set Two sets C and D are non-disjoint when they have at least
that contains only the common elements in them. It is one element in common
denoted by  and not n. i.e. CD ≠ 
If A and B are two subsets, mathematically
Examples
A  B = {x: x A and xB}
Given that X = {a, b, c, d}, Y = {a, b} and Z = {c, d}
E.g 1. The sets of two groups of students who took part Then find the pair of sets that are:
in the finals of shell and chevron Scholarship test are (a) Non – disjoint ( b) disjoint.
given as. Solution
S = {Ada, Olu, Akpos} (a) To establish non-disjoint. We find intersection.
C = {Ejiro, Akpos, Iluwa} X  Y = {a, b}
Find S  C Thus X and Y are non-disjoint
Solution
(b) Y  Z =  thus,
S C = {Akpos} Y and Z are disjoint.
Complement of set 2009/7 Exercise 9.1
The complement of any set say K is the set that has Two sets are disjoint if
every element in the universal set except those of K. A They are both empty
it is denoted by K1 or Kc B Their union is an empty set
i.e. U – K = Kc C Their intersection is an empty set
D one of them is a subset of the other
132
Range Sets Odd numbers
Here elements of inequality are employed to define the Odd numbers are numbers not divisible by 2.
given sets. They are strict inequalities < (less than) and Example:
> (greater than) & the weak inequalities < ( less than or List the elements of the set
equal to ) and > (greater than or equal to). R = {k : k is an odd number between 2 and 11}
Any set defined by strict inequalities does not accept Solution
numbers at the boundaries (limits). Similarly, any set The word “between’’ here means 2 and 11 are not inclusive.
defined by weak inequalities accepts numbers at the R = {3, 5, 7, 9}
boundaries (limits). Note: 1 is an odd number
Examples Multiples
List the elements of the sets below: The Multiple of any number is the set, which contains the
(i) G = {x : 1< x < 5} product of the given number and 1, 2, 3, … except when
(ii) L = {y : 16 < y  19} limits are given. Alternatively the multiples of any given
number, say 4 are those numbers that can be divided by 4
(iii) M = {a : 25  a <30} without remainder.
(iv) N = {b : 3  b  6} Example: List the elements of the set
Solution J = {t: 1 < t < 10 and t is the multiple of 3}
(i) G = {2, 3, 4} Solution
Reasons: Both boundaries are strict inequalities hence 1 The range {1< t < 10} = {1, 2, 3, 4, 5, 6, 7, 8, 9, 10}
and 5 are not included in the set G. The multiples of 3 here are {3, 6, 9,}
Therefore J = {3, 6, 9}
(ii) L = {17, 18, 19}
Reason : 16 is not included because the left half Odd-prime Numbers
inequality is strict, while 19 is included because the These are numbers that have the odd numbers properties and
right half inequality is weak. are further subjected to the prime number properties.
(iii) M ={25, 26, 27, 28, 29} Example:
Reason: 25 is included because the left half inequality is If U = {x: 1 < x < 10} and
weak while 30 is not because the right half inequality is Y = {x: x is odd – prime}
strict. Then list the elements of Y
Y odd = {1, 3, 5, 7, 9 }
(iv) N = {3, 4, 5, 6, } Y odd – prime = {3, 5, 7}
Reason: 3 and 6 are included because of the weak
inequalities at both ends. DIVISIBILITY
This is the other term for multiple. Also not divisible and not
a multiple are the same.
Number sets
This type of set deals with properties of the numbers LAWS OF SET
such as prime, odd, even, multiples, odd- prime and
1. IDEMPOTENT LAW
divisibility. For the purpose of emphasis the author will
A A = A; A A = A
briefly define and give examples of the above named
2. ASSOCIATIVE LAWS
properties.
A (BC) = (AB) C
Prime numbers A (BC) = (AB) C
3. COMMUTATIVE LAW
A number is said to be prime if it has only two factors;
one and itself.
AB = BA; AB = BA
Example:
List the elements of the set 4. DISTRIBUTIVE LAW
A = {z : z is a prime number from 1 to 10} A  (BC) = (AB)  (AC)
A = {2, 3, 5, 7} A  (BC) = ( A B)  (AC)
Note: 1 is not a prime number because it has only one factor. 5. DE-MORGAN’S LAW
(AB)1 = A1B1
Even numbers (A B)1 = A1B1
These are numbers divisible by 2 without remainder. 6. COMPLEMENT LAWS
Example: (A1)1 = A ; U1 = 
Given that W = {x: x is even number from 5 to 15}: AA1 =  ; AA1 =  ;  1 = 
list the elements of W 7. IDENTITY LAWS
Solution
W = {6, 8, 10 , 12, 14} A = A ; A  = A
A =  ; A = 
133
Problems on number sets Solution
P = {2, 3, 5}
1999/ 1 (Nov)
Q = {2, 3}
If ξ = {1, 2, 3, 4, 5}, P = {2, 3, 5}, Q = {1, 2, 3}
Note: Factors of 6 are 1, 2, 3, 6
Find the complement of PQ Prime factors of 6 are 2, 3
A {2, 3, 5} B {2, 4, 5} C {1, 4, 5} D {1, 3, 5}
PQ = {2, 3}
Solution
(PQ) 1 = {–1, 0, 1, 4, 5, 6} C
PQ = {2, 3}
(PQ)1 = {1, 4, 5} C. 2004/ 26
Given that  = {1, 2, 3,…,10}, P = {x : x is prime} and
1998/ 13 (Nov) Q = { y : y is odd}, find P1Q
If U = {Positive numbers less than 20} A {2} B {1, 9} C{ 3, 5, 7} D{4, 6, 8, 10}
P = {multiples of 4}, Q = {multiples of 6},find PQ. Solution
A {4, 6, 8, 12, 16, 18} B {6, 12, 18}  = {1, 2, 3, 4, 5, 6, 7, 8, 9, 10}
C {4, 6} D {4, 6, 12} E {12} Applying the definition for prime numbers between 1 and 10
Solution P = {2, 3, 5, 7}
U = {1, 2, 3, 4, 5, 6, … 19} P1 = {1, 4, 6, 8, 9, 10}
P = {4, 8, 12, 16} and Q = {6, 12, 18} Applying the definition of odd numbers between 1 and 10
PQ = {12} E. Q = {1, 3, 5, 7, 9}
2007/ 12 Neco P1 Q = {1, 9} B.
Let the universal set U be the set of integers 2005/ 34
U = { x : 0 < x  10}. What is the complement of the P = {3, 9, 11, 13} and Q = {3, 7, 9, 15} are subsets of the
Set B = { x : x  U, x is not divisible by 3 } ? universal set  = {1, 3, 7, 9, 11, 13, 15}. Find P1Q1
A {4} B {4, 8} C {9, 6, 3} A {3, 9} B {5, 7, 15} C {1} D {1, 11}
D {4, 8, 12, 16} E {1, 2, 3, 5, 6, 7, 9} Solution
Solution P1 = {1, 7, 15}
Listing their elements Q1 = {1, 11, 13}
U = {1, 2, 3, 4, 5, 6, 7, 8, 9, 10} P1Q1 = {1} C.
B = {1, 2, 4, 5, 7, 8, 10}
B1 = {3, 6, 9} C. 2009/7 (Nov)
The sets P = {2, 5, 8} and Q = {2, 3, 5, 7} are subsets of the
1999/ 2 (Nov)
universal set  = (1, 2, 3, 4, 5, 6, 7, 8, 9, 10}. Find P1Q
Given that X = {e, f, g}, Y = {e, f, h} and Z = {g, h, k}.
A {3} B {2, 5} C {3, 7} D {2, 3, 5, 7}
How many elements are there in XYZ?
Solution
A3 B5 C7 D9
P1 = {1, 3, 4, 5, 6, 7, 9, 10}
Solution
Q = {2, 3, 5, 7}
We are to find n(XYZ)
P1Q = {3, 7} C
Firstly, XYZ = {e, f, g, h, k}
n(XYZ) = 5 2005/35
P = {3, 9, 11, 13} and Q = {3, 7, 9, 15} are subsets of the
2003/8a
universal set  = {1, 3, 7, 9, 11, 13, 15}.Find P1 Q1
A = {1, 2, 5, 7} and B = {1, 3, 6, 7} are subsets of the
A {3, 9} B {5, 7, 15} C {1} D {1, 11}
universal set U = {1, 2, 3, 4,…,10}.
Solution
Find ( i ) A1 ( ii ) (AB)1 (iii) (AB)1 P1 = {1, 7, 15} Q1 = {1, 11, 13}
(iv) the subsets of B each of which has three elements
P1 Q1 = {1} C.
Solution
U = {1, 2, 3, 4, 5, 6, 7, 8, 9, 10} 2006/42
A = {1, 2, 5, 7} If P = {1, 3, 5, 7, 9} and Q = {2, 4, 6, 8, 10} are subset
( i ) A1 = {3, 4, 6, 8, 9, 10} of a universal set U = {1, 2, 3, 4, 5, 6, 7, 8, 9, 10},
( ii ) AB = {1, 7} what are the elements of P1 Q1 ?
(AB)1 = {2, 3, 4, 5, 6, 8, 9, 10} A {1, 3, 5, 7, 9} B {2, 4, 6, 8, 10} C 
( iii ) AB = {1, 2, 3, 5, 6, 7} D {1, 2, 6, 8, 10}
(AB)1 = {4, 8, 9, 10} Solution
( iv ) {1, 3, 6}, {1, 3, 7}, {3, 6, 7}, {1, 6, 7} P1 = {2, 4, 6, 8, 10}
Q1 = (1, 3, 5, 7, 9}
2004/ 25(Nov) P1 Q1 =  C.
Given that  = {–1, 0, 1, 2, 3, 4, 5, 6}, where
P = {Prime numbers} and Q = {Prime factors of 6} are 2007/ 10a NABTEB
subsets of . Find (PQ)1 . If U = {1, 2, 3, …,10} and A = {4, 6, 8, 10}
A {2, 3} B {–1, 0, 1, 2, 3} C {–1, 0, 1, 4, 5, 6} B = {1, 4, 5, 11} and C = {4, 5, 11, 12}.
D {–1, 1, 2, 3} Find C1  (AB)

134
Solution 2010/4b Neco
Starting with the bracket items Let U = {3, 6, 9, … , 45}
AB = {4} A = {y : y < 22} B = {y : y is a factor of 45}
C1 = {1, 2, 3, 6, 7, 8, 9, 10} Where y U. Find : ( i ) n(U) ( ii ) AB , ( iii ) (AB)
C1  (AB) = {1, 2, 3, 4, 6, 7, 8, 9, 10} Solution
( i ) n(U); here we apply nth term formula for AP,
2008/3 since elements of U follows a simple AP.
Let P, Q, R, S be subsets of R defined by the following sets: Tn = a + (n – 1)d
P = {x : (x – 2)(x + 3)(x – 4)(x + 5) = 0} 45 = 3 + (n –1)3
Q = {x : (x + 1)(x – 1)(x – 2) = 0} 45 – 3 = (n –1)3
R = {x : (x – 5)(x + 2)(x + 1) = 0} 42 = (n –1)3
S = {x : (x – 1)(x + 2)(x – 3) = 0} 42 = n – 1
3
Find: ( i ) PQ ( ii ) PQR ( iii ) RS
Solution 14 + 1 = n
First, we list the elements of each set. n = 15
Applying the principle of solving quadratic equations Thus n(U) = 15
Set P Alternatively
(x – 2)(x + 3)(x – 4)(x + 5) = 0 ( i ) By listing
x – 2 = 0 or x + 3 = 0 or x – 4 = 0 or x + 5 = 0 U = {3, 6, 9, 12, 15, 18, 21, 24, 27, 30, 33, 36, 39, 42, 45}
x = 2 or – 3 or 4 or –5 Thus, n(U) = 15
Listing the elements in order of magnitude ( ii ) AB = ?
P = {– 5, – 3, 2, 4} First, we list elements of A, B
Set Q A = {3, 6, 9, 12, 15, 18, 21}
(x + 1)(x – 1)(x – 2) = 0 B = {1, 3, 5, 9, 15, 45} Applying simple definition of factors
x + 1 = 0 or x – 1 = 0 or x – 2 = 0 AB = {1, 3, 5, 6, 9, 12, 15, 18, 21, 45}.
x = –1 or 1 or 2 AB = {3, 9, 15}
Listing the elements in order of magnitude
Q = {– 1, 1, 2} 2010/13
Set R Given that P = {x : 1  x  6} and Q = {x : 2 < x < 10}
(x – 5)(x + 2)(x + 1) = 0 where x is an integer. Find n(PQ)
x – 5 = 0 or x + 2 = 0 or x + 1= 0 A 4 B6 C8 D 10
x = 5 or –2 or –1 Solution
Listing the elements in order of magnitude n(PQ) implies number of elements in PQ
R = {–2, –1, 5} First, we list the elements of set P, set Q then PQ.
Set S P = {1, 2, 3, 4, 5, 6}
(x – 1)(x + 2)(x – 3) = 0 1 and 6 were included because of the weak inequalities there
x – 1 = 0 or x + 2 = 0 or x – 3 = 0
Q = {3, 4, 5, 6, 8, 9}
x = 1 or –2 or 3 2 and 10 were NOT included because of the strict inequalities there.
Listing the elements in order of magnitude PQ = {3, 4, 5, 6}
S = {– 2, 1, 3}
Thus, n(PQ) = 4 A.
( i ) PQ = {–5, –3, –1, 1, 2, 4}
( ii ) PQR = {–5, –3, –2, –1, 1, 2, 4, 5} 2011 1b Neco
( iii ) R  S = {– 2} If U = {x : the solution set of the inequalities 2x – 3 < 27 and
2x – 1 2 + 3x}
2009/ 16 Neco (Dec) P is a subset of U such that P = {Prime numbers}
Given that U is the universal set defined as List the elements in : ( i ) U ( ii ) P
U = {x : 1  x  12 }, P and Q are subsets such that Solution
P = {x : x < 12} and Q = {x : x is an even number}. ( i ) We get elements of U by solving the two inequalities
Find P1 Q (where P1 = complement of P) 2x – 3 < 27 2x – 1  2 + 3x
A { } B {12} C {1, 2} D {2, 4, 6, 8, 12} 2x < 27 + 3 2x – 3x  2 + 1
E {2, 3, 5, 7, 9, 11} 2x < 30 – 1x  3
Solution 2x 30 −1x 3
 
Listing the elements of the universal set and the subsets 2 2 −1 −1
U = {1, 2, 3, 4, 5, 6, 7, 8, 9, 10, 11, 12} x < 15 x  –3
1 and 12 were included because of the weak inequality there. U = {–3, –2, –1, 0, 1, 2, 3, 4, 5, …, 13, 14}
P = {1, 2, 3, 4, 5, 6, 7, 8, 9, 10, 11}
12 was NOT included because of the strict inequality there.
( ii ) P = {2, 3, 5, 7, 11, 13}
Q = {2, 4, 6, 8, 10, 12}
P1 = {12}
Thus, P1 Q = {12} B.

135
2012 / 13 – 14 Neco x(x – 9) + 3(x – 9) = 0
Given that U = {x : –3< x  15} (x + 3)(x – 9) = 0
A = {x : 3  x < 15}, B = {x : –1< x< 8} x + 3 = 0 or x – 9 = 0
x = – 3 or 9 D.
13. Find (AB) 1 i.e {x : x = – 3, x = 9)
A {– 2, –1, 0, 15} B {–2, –1, 1, 15} C {– 2, –1, 15}
D {0, 1, 2, 15} E {1, 2, 15} 2009/18 NATEB Exercise 9.2
Solution If ξ = {1, 2, 3, …15, 16} and T = {3, 6, 9, 12, 15}
Listing elements S = {1, 4, 9, 16}. Then (TS)1 is
U = (x : – 3 < x  15} A {1, 3, 4, 6, 9, 10, 15, 16} B {2, 3, 4, … 15, 16}
= {–2, –1, 0, 1, 2, 3, 4, 5, 6, 7, 8, 9, 10, 11, 12, 13, 14, 15} C {2, 5, 7, 8, 10, 11, 13, 14} D {3, 4, 6, 9, 10, 15, 16}
–3 was omitted because of the strict inequalities there
15 was included because of the weak inequalities there 2010/1 Exercise 9.3
A = {2, 4, 6, 8}, B = {2, 3, 7, 9} and
A = {x : 3  x < 15} C = {x : 3 < x < 9} are subsets of the universal set
= {3, 4, 5, 6, 7, 8, 9, 10, 11, 12, 13, 14} U = {2, 3, 4, 5, 6, 7, 8, 9} Find (a) A(B1 C 1);
3 was included because of the weak inequalities there
15 was NOT included because of the strict inequalities
(b) (AB)(BC)
2014/ 11 Exercise 9.4
B = {x : –1< x < 8} If X = {0, 2, 4, 6}, Y = {1, 2, 3, 4} and Z = {1, 3}
= {0, 1, 2, 3, 4, 5, 6, 7} are subsets of U = {x : 0  x  6}, find x(Y1 Z)
–1and 8 were NOT included because of the strict inequality there
A {0, 2, 6} B {1, 3} C {0, 6} D { }
AB = {0, 1, 2, 3, 4, 5, 6, 7, 8, 9, 10, 11, 12, 13, 14} 2008/4 Neco Exercise 9.5
(AB)1 = {– 2, –1, 15} C. Given that : P = { x : (x – 2)(x + 3)(x – 4)(x + 5) = 0 }.
The elements in P are
14. Find A1B A (– 5, –2, 3, 4} B {–5, –3, 2, 4}
A {–1, 0, 1, 15} B {–2, –1, 0, 1, 2} C {2, 4, 6} C {–5, –3, –2, 4} D{– 4, –3, – 2, 5} E {2, 3, 4, 5}
D {0, 1, 2} E {0, 1, 2, 15} 2006/17 Neco Exercise 9.6
Solution If  = {K, S, A, P, M, E, C}, P = {S, P, M, E}
A = {3, 4, 5, 6, 7, 8, 9, 10, 11, 12, 13, 14}
A1 = {–2, –1, 0, 1, 2, 15} and Q = {K, A, P, C}, R = {A, P, M, C}
B = {0, 1, 2, 3, 4, 5, 6, 7} Find Q1 (P R)
A1 B = {0, 1, 2} D. A {k, a, m, e} B {k, s, m, c} C {s, a, p, m
D {s, a, e, c} E {s, p, m, e}
2012/37 2008/2b NATEB (Dec) Exercise 9.7
If P = {Prime factors of 210} and If U = {1, 2, 3, 4, 5, 6, 7, 8, 9, 10} A = {2, 3, 5, 8},
Q = {Prime numbers less than 10} B = {1, 3, 7, 8, 9}. Write down the sets
Find PQ ( i ) AB ( ii ) A1 B1
A = {1, 2, 3,} B = {2, 3, 5} C = {1, 3, 5, 7} 2014/4b (Nov) Exercise 9.8
D = {2, 3, 5, 7} Given that X = {x : 10  x < 15} and
Solution Y = { even numbers < 18} are subsets of
Using the elementary way of listing elements of prime factors
U = { 10, 11, 12, …, 20 }, find : ( i ) XY ( ii ) n(X1Y)
2 210
3 105 2009/45 NATEB Exercise 9.9
5 35 Given that ξ = {1, 2, 3, 4 …10}, A = {a : a is even) and
7 7 B = {b : b is prime}, then A1 B is
1 A {5} B {2, 8} C {3, 5, 7} D 
Thus P = {2, 3, 5, 7} and 2012/ 6 Neco Exercise 9.10
Q = {2, 3, 5, 7} If P and Q are subsets of a universal set U,
PQ = {2, 3, 5, 7, } D. evaluate P1(QP)
2014/ 12 A BP C Q D P1 Q E P1 Q
Find the truth set of the equation x2 = 3(2x + 9)
A {x : x = 3, x = 9} B {x : x = –3, x = –9}
C {x : x = 3, x = –9} D {x : x = –3, x = 9}
Solution
We simply solve the resulting quadratic equation
x2 = 3(2x + 9)
x2 = 6x + 27
x – 6x – 27 = 0
2

Factorizing
x2 – 9x + 3x – 27 = 0

136
Venn diagram 2005/13b
This is a representation of set in shapes whereby subsets Three sets P, Q and R are defined as follows:
are represented by circles while rectangle enclosing the P = {1, 2, 5}, Q = {2, 5}, R = {2, 4, 6} and the universal
circle(s) represents the universal set. set  = {1, 2, 3, 4, 5, 6}
( i ) Draw a Venn diagram to represent the information
Number Venn diagram ( ii ) List the elements of P1, (P R)1 (PQ)1 and
This is a representation of a given set of numbers or (PQ)(QR)
set’s solution in Venn diagram. Solution
2013/5 ( i ) PQR = {2}, PQR1 = {5},
P Q PQ1 R = { }, P1 QR = { }
a, b f U
e, g P Q
k, n m, d 5
1
s, r, t 2
R
The diagram above consists of the sets P, Q and R. 4, 6
Find RQ R 3
A {m, d} B {d, e, n} C {d, e, g, m} 1
( ii ) P = {3, 4, 6}
D {e, g, k, m} E {k, n, e, g} (PR)1 (PQ)1 starting from the brackets
Solution PR = {1, 2, 4, 5, 6} PQ = {1, 2, 5}
RQ = {d, e, g, m} C. (PR)1 = {3} (PQ)1 = {3, 4, 6}
On the other hand, if we are asked to Thus (PR)1 (PQ)1 = {3}
find RQP1 = RQ only
= {d, m} (PQ)(QR) starting from the brackets
PQ = {2, 5} QR = {2}
2000/ 3b
Thus (PQ)(QR) = {2, 5}
P
8
2 1998/1
0
3
A, B and C are subsets of the universal set  such that:
7
1
4  = {0, 1, 2, 3, … , 12};
6 Q
5
9 A = {x : 0  x  7}; B = {4, 6, 8, 10, 12};
C = {1< y < 8} where y is a prime number;
The Venn diagram above represents a universal set ξ of ( a ) Draw a Venn diagram to illustrate the information
integers and its subsets P and Q. given above.
List the elements of the following sets
( b ) Find: ( i ) (BC)1; ( ii ) A1 BC
( i ) PQ; ( ii ) PQ; ( iii ) (PQ)1 ( iv ) (PQ)1 Solution
Solution
 = {0, 1, 2, 3, 4, 5, 6, 7, 8, 9, 10, 11, 12}
( i ) PQ = {0, 2, 8, 7, 3, 6, 4}
A = {0, 1, 2, 3, 4, 5, 6, 7}
( ii ) PQ = {3, 7} B = {4, 6, 8, 10, 12}
( iii ) (PQ)1 = {1, 5, 9} C = {2, 3, 5, 7}
( iv ) (PQ)1 = {0, 2, 8, 6, 4, 1, 5, 9} A1 BC = { }, ABC1 = {4, 6}
2005/ 27 – 28 (Nov) AB1 C = {2, 3, 5, 7}
Use the Venn diagram to answer questions 27 and 28 (a)
A U U
A
X Y B C B C
a f, g i 0, 1
b b 4, 2, 3 8, 10, 4, 2, 3
c 6 5, 7 6 5, 7
k 12

e, j
Z 9, 11
First filling of intersection elements
27. What is X1 Y1 Z?
A {a, d} B {b, i} C {e, j} D { f, g } ( b ) ( i ) BC = {2, 3, 4, 5, 6, 7, 8, 9, 10, 12}
Solution (BC)1 = {0, 1, 9,11}
X1 Y1 Z is elements of Z only = {e, j} C. ( ii ) A1 BC = { }
28. What is X YZ1?
A {a, b) B {e, j} C {b, i} D {f, g}
Solution
X YZ1 = X Y only
= {f, g} D.
137
2009/17 Neco (Dec) Exercise 9.11 (ii) Independent subsets
In the Venn diagram below, what is (QR)?
P
A BUA B
Q
1
2 3
4
8 6

5
R AB shaded AB shaded
A {2} B {4, 6} C {2, 3, 5, 8} A B A B
D {1, 2, 3, 5, 8} E {1, 2, 3, 4, 5, 8}
2005/ 1b Exercise 9.12
 = {1, 2, 3, 4, 5, …,10}, A = {1, 2, 3, 4, 5}
B = {2, 3, 5} and C = {6, 8, 10}
U (AB)1 shaded (AB)1 shaded
A
1
B A B A B
5

C
6, 10
9

( i ) Given that the Venn diagram represents the sets A1B shaded AB1 shaded
above, copy and till in the elements
( ii ) Find AC (iii) Find AB1 (iii) Three subsets
A B A B

C C
ABC shaded A BC shaded

A B A B
Venn diagram shading
This is the representation of set relationships such as
intersections, union and complements by appropriate
C C
shading in Venn diagrams
AB1C shaded ABC1 shaded
One subset
U A B A B
U A
A

A1 shaded A shaded C C

A1BC shaded AB1C1 shaded


Two subsets
(i) When A  B (A is a subset of B)
A B A
B B
B A
A
A1 Shaded AB shaded C C

A1BC1 shaded A1B1C shaded


A B
AB A
B

AB shaded
C C
(A BC) 1 shaded (BC) A1C shaded

138
2009/ 6 (Nov) 2014/36 Neco (Dec) Exercise 9.17
P The shaded region in the diagram below represents

A B
Q R

P, Q and R are three intersecting sets in the universal set


. Which of the following represents the shaded A AB B A1B1 C (AB)1
portion? D AB E A1B
A (PQ)R B (PQ)1R C (RQ)P1
D (PQ) Q
1 2015/6 Neco Exercise 9.18
Solution Which of the following is illustrated by the shaded portion in
The shaded portion is (RQ)P1 the Venn diagram below?
A B
2009/48 Exercise 9.13
Which shaded region in the following diagrams
represents (PQ)R ?
P Q P Q
C
A B
R R A (AC)B1 B (AC)B1 C (AC)B1
P D (AC)B1 E (AC)(BC)
P Q Q
C D 2015/42 Exercise 9.19
P Q
R R

2005/18 Neco Exercise 9.14


A B
R
Describe the shaded portion in the diagram.
A P1QR1 B (PR)1Q C P1QR D (PQ)1R

Two subsets problems


C For any two subsets A and B not disjoint i.e A B ≠ 
In the Venn diagram below, the area shaded represents n(AB) = n(A) + n(B) – n(A B)
A A1 (BC) B A(B1 C) C A(BC1) 1998/12 ( Nov )
D A1 (BC) E A(B1 C) M and N are two sets such that n(M) = 10, n(N) = 7 and
2015/4 (Nov) Exercise 9.15 n(MN) = 13. Find n(MN)
A 16 B6 C4 D3 E1
Solution
S
R Applying two subsets formula
n(MN) = n(M) + n(N) – n(MN)
13 = 10 + 7 – n(MN)
13 = 17 – n(MN)
n(MN) = 17 – 13 i.e 4 C.
In the Venn diagram, R is a subset of S. Find the set
which represents the shaded portion 2009/4a
A R1 B S C R1S D R1S Out of 30 Candidates applying for a post, 17 have degrees,
15 diplomas and 4 neither degree nor diploma. How many of
2006/46 UME Exercise 9.16 them have both?
If E  G  U, where U is the universal set, then the set, Solution
then the shaded Venn diagram representing U–E or EC is G – degree, P – diploma and x – both
.........................
..............
.............. ...........
...........
......................... .........................
......................... U30
..............
..............
U ...........
...........
......................... .........................
U
.........................
G G17 P15
.............. ...........
......................... .........................
.........................
A .............. ...........
.........................
..............
E ........... C .........................
.........................
.............. ...........
G
.........................
.............. ........... .........................
E
.........................
.........................
.........................
.............. ...........
......................... .........................
......................... 17 - x x 15 - x

.........................
.............. ...........
......................... .........................
.............. ...........
U .........................
.........................
....
.............. ...........
......
.........................
E
........... …
……
.........................

U
......................... 4
B ......
..............
......
.........................
....
..............
G........... D .........................

… …
.........................
.........................
G 17 – x + x + 15 – x + 4 = 30
.........................
.............. ...........
E
.........................
.........................
.........................
..............
.............. ...........
...........
......................... .........................
......................... 36 – x = 30
36 – 30 = x thus x = 6
139
2007/ 5b Neco 2012/ 7 Neco
In class of 64 students, each student offers either From the Venn diagram below, find the value of x
Physics or Mathematics or both. If 50 students offer U = 8x
P Q
Mathematics and the number of students offering
Mathematics only is twice the number of students 3x - 1 x 5x - 2
offering Physics only, how many students offer both
subjects?
2
Solution
M – Mathematic n(M) = 50 A1 B2 C3 D4 E5
P – Physics n(P) = Solution
n(U) = 64 We equate everything in the box to the universal set
U64
P M50 3x – 1 + x + 5x – 2 + 2 = 8x
9x – 1 = 8x
2y y
9x – 8x = 1
x = 1 (A)

Let n(PM) = x then y = 50 – x 2012/ 6b


Re - labeling our diagram, we have A number of tourists were interviewed on their choice of
U64
means of travel. Two – thirds said that they traveled by road,
P M50 13 4
by air and by both air and road. If 20 tourists did
30 15
2(50 - x) x 50 - x
not travel by either air or road,
( i ) represent the information on a Venn diagram;
( ii ) how many tourists
2(50 – x) + x + 50 – x = 64 (a) were interviewed;
100 – 2x + 50 = 64 (b) Travelled by air only?
150 – 64 = 2x Solution
86 = 2x R – Road
x = 43 A – Air
Let the number of tourists be x
2006/ 4a 4 2 13
n(AR) = x , n(R) = x , n(A) = x
In a class of 45 students, 32 offered Physics (P), 28 15 3 30
offered Government (G) and12 did not offer any of the (i)
two subjects. A 13 x R 2 x
U=x
( i ) Draw the Venn diagram to represent the 30 3

information 4 x
( ii ) How many students offered both Physics and 15
Government?
20
( iii ) What is n(PG)?
Solution Resulting diagram
U=x
( i ) Let n(PG) = x A 13 x R 2 x
30
U45 3
P32 G 28
13 x 4 x 4 x 2 x 4 x
30 15 15
32 - x x 28 - x 3 15

12 20

( ii ) 32 – x + x + 28 – x + 12 = 45 ( ii ) a.
72 – x = 45  13 4  4 2 4 
 x − x + x +  x − x  + 20 = x
72 – 45 = x  30 15  15  3 15 
x = 27
5 4 6
x + x + x + 20 = x
( iii ) n(PG) = 32 – x + x + 28 – x 30 15 15
= 60 – x Multiply through by LCM of 15 and 30 to clear fractions
= 60 – 27 5x + 8x + 12x + 600 = 30x
= 33 30x – 25x = 600
5x = 600
x = 600 5
x = 120 tourists

140
(b) n (A) only Three subsets problems
A R For any three subsets A, B and C not disjoint i.e
ABC  
n(ABC) = n(A) + n(B) + n(C) – n(AB) – n(AC)
 13 4  – n(BC) + n(ABC)
=  x − x
 30 15  If the given three-subset problem is disjoint apply only the
Venn diagram approach. Otherwise both methods can be
5
= x used.
30
5 2013/ 4 Neco
=  120 = 20 tourists.
30 In a class of 55 students, 21 study Physics, 24 study
2008/33 Geography and 23 study Economics, 6 study both Physics
Every staff in an office owns either a Mercedes and /or and Geography, 8 study both Geography and Economics and
a Toyota car. 20 own Mercedes, 15 own Toyota and 5 study both Economics and Physics. If x study all the 3
5 own both. How many staffers are there in the office? subjects and 2x study none of the three subjects. Find
A 25 B 30 C 35 D 45 ( i ) the value of x,
Solution ( ii ) the number of students that study physics only
M – Mercedes (iii) the number of students that study only two subjects.
T – Toyota Solution
n(M) = 20, n(T) = 15, n(MT) = 5, n(U) = x P – Physics n(P) = 21
U= x G – Geography n(G) = 24
M 20 T15 E – Economics n(E) = 23
n(PG) = 6, n(GE) = 8, n(EP) = 5
20 - 5 5 15 - 5
n(PG E) = x, n(PGE)1 = 2x , n(U) = 55
U55
P21 G24
6-x
20 – 5 + 5 + 15 – 5 = x x
30 = x (B) 5- x 8-x

2014/6a ( Nov ) Exercise 9.20 2x E 23


1
In a class of 52 students, 16 are Science students. If 3
Here n(PGE) = n(U) – n(PGE)1
of the boys and 14 of the girls are Science students,
how many boys are in the class? n(U) – n(PGE)1 = n(P) + n(G) + n(E) – n(PG)
– n(GE) – n(EP) + n(PG E)
2011/ 2 Neco Exercise 9.21 55 – 2x = 21 + 24 + 23 – 6 – 8 – 5 + x
Among 40 applicants for a sales manager’s post, 26 55 – 2x = 68 – 19 + x
have B.Sc degree and 18 have NCE, 6 applicants have 55 – 49 = 2x + x
no certificate. Use a Venn diagram to illustrate the 6 = 3x
information. How many applicants have: Thus, x = 2
( i ) both B.Sc and NCE; ( ii ) B.Sc only
( iii ) NCE only? ( ii ) n(P) only = 21 – (6 – x + x + 5 – x)
= 21 – (11– x)
2009/ 5 Neco (Dec) Exercise 9.22 Substituting for the value of x
In a class of 25 pupils, 12 offer Physics and 18 offer = 21 – (11– 2)
Chemistry. If every pupil offers at least one of the two, = 12
how many offer both subjects?
A5 B6 C 7 D 12 E 13 ( iii ) Two subjects only = 6 – x + 5 – x + 8 – x
= 19 – 3x
2012/10 UTME Exercise 9.23 = 19 – 6
In a class of 46 students, 22 play football and 26 play = 13
volleyball. If 3 students play both games, how many Example TSS 2
play neither? In a survey, out of 100 out–patients who reported at a clinic
A1 B2 C3 D4 in a week it was found out that 70 complained of fever, 50
2009/11 PCE Exercise 9.24 had stomach ache and 30 had injuries. All the 100 patients
In a class of 40 students, 30 students take Economics had at least one of the complaints and 44 had exactly two of
and 20 take Accounting. If 8 students take neither the complaints. How many patients had all the three
Economics nor Accounting, find the number of students complaints?
who take Economics but not Accounting. Solution
A 18 B 12 C8 D2 This questions calls for application of our knowledge of
solving three subsets formula/Venn diagram problems
let the first letters represents each complaint
141
n(FS) = x n(FI) = y n(SI) = z ( i ) all the three dances?
and n(FSI) = p ( ii ) exactly two of the dances?
( iii ) exactly one of the dances?
F70 U100 ( iv ) Gwape dance only?
S50 Solution
x-p
E – Efik n(E) = 189
P Y – Yoruba n(Y) = 152
y-p z-p G – Gwape n(G) = 130
n(EY) = 64, n(EG) = 72, n(YG) = 56
I30 n(E YG) = x ,
But x – p + y – p + z – p = x + y + z – 3p n(EYG) = n(U) since each viewer like at least one of the
and x + y + z is 44 three dances.
Thus x – p + y – p + z – p = 44 – 3p (a)
E Y U300
189 152
From the formula method
n(FSI) = n(F) + n(S) + n(I) – n(FI) – n(FS) – 64-x
n(SI) + n(FSI) x
72-x 56-x
= n(F) + n(S) + n(I) – [ n(FI) + n(FS) + n(SI) ] +
n(FSI) G130
100 = 70 + 50 + 30 – [ 44 + 3P] + P
100 = 150 – 44 – 3P + P (b) i. Applying the formula for three subsets problem
100 = 106 – 2P n(EYG) = n(E) + n(Y) + n(G) – n(EY) – n(EG)
2P = 106 – 100 – n(YG) + n(EYG)
2P = 6 Thus P = 6/2 i.e 3 Substituting
300 = 189 + 152 + 130 – 64 – 72 – 56 + x
2007/ 38 – 39 300 = 471 – 192 + x
Rice
Plantain 300 = 279 + x
300 – 279 = x
6
5 8 x = 21
x
4 2 (ii) Exactly two of the dances
5 = 64 – x + 72 – x + 56 – x
Yam
= 64 – 21 + 72 – 21 + 56 – 21
The Venn diagram shows the choice of food of a = 43 + 51 + 35
number of visitors to a canteen. Use the information to = 129
answer questions 38 and 39 (iii) Exactly one of the dances
= 189 – (64 – x + x + 72 – x) + 130 – (72 – x + x + 56 – x)
38. If there were 35 visitors in all, find the value of x + 152 – (64 – x + x + 56 – x)
A5 B4 C3 D2 = 189 – (136 – 21) + 130 – (128 – 21) + 152 – (120 – 21)
Solution = 189 – 115 + 130 – 107 + 152 – 99
5 + 6 + x + 4 + 5 + 2 + 8 = 35 = 74 + 23 + 53
30 + x = 35 = 150
x = 35 – 30 i.e 5 (A)
39. How many people took at-least two kinds of food (iv) Gwape dance only
A 10 B 12 C 15 D 17 = 130 – (72 – x + x + 56 – x)
Solution = 130 – (128 – 21)
At-least two kinds of food implies = 130 – 107
two kinds of food plus three kind of food. = 23
= 4+6+2+x
= 12 + 5 2011/ 6
= 17 (D) In a class of 40 students, 18 passed Mathematics, 19 passed
2010/9 Neco Accounts, 16 passed Economics, 5 Mathematics and
In a survey of 300 viewers who watched 2009 National Accounts only, 6 Mathematics only, 9 Accounts only ,
Festival of Art and Culture, it was found that 189 liked 2 Accounts and Economics only. If each student offered
Efik dances, 152 liked Yoruba dances and 130 liked at least one of the subjects,
Gwape dances and each liked at least one of the three (a) how many students failed in all the subjects?
dances. If 64 liked Efik and Yoruba dances, 72 liked (b) find the percentage number who failed in at least
Ekik and Gwape dances and 56 liked Yoruba and one of Economics and Mathematics;
Gwape dances. (c) calculate the probability that a student selected at
(a) Draw a Venn diagram to illustrate the information random failed in accounts
(b) How many viewers liked
142
Solution 45 + 25x – 5x = 125
n(MAE) = x, n(ME) only = y, n(E) only = z 20x = 125 – 45
n(MAE)1 = f 20x = 80
Drawing its Venn diagram x= 80 i.e 4
U40 20
M 18 A19 (ii) n(PQR1) refers to P and Q circle without R
5 9
P Q
6
x
y 2

z
R
E 16
f
n(PQR1) = 16 – 2x + 5x + 6 + x
Substituting for x = 4
Solving for the unknowns = 16 – 2(4) + 5(4) +6 + 4
5 + 9 + 2 + x = 19 subset Accounts = 16 – 8 + 20 + 6 + 4
x = 19 – 16 = 38
x = 3 1991/3
6 + 5 + x + y = 18 subset Mathematics In a certain class, 22 pupils take one or more of chemistry,
Substituting for x value Economics and Government. 12 take Economics (E), 8 take
6 + 5 + 3 + y = 18 Government (G) and 7 take Chemistry (C) nobody takes
y = 18 – 14 Economics and Chemistry and 4 pupils take Economics and
= 4 Government.
(a), (i) Using set notation and the letters indicated above,
y + x + 2 + z = 16 subset Economics write down two statements in the last sentence
Substituting for x and y values (ii) Draw a Venn diagram to illustrate the information
4 + 3 + 2 + z = 16 (b). How many pupils take
z = 16 – 9 i.) Both Chemistry and Government
=7 ii.) Government only?
(a) 6 + 5 + x + y + z + 2 + 9 + f = 40 Solution
Substituting for x, y and z values a ( i ) n(EC) = 0 or EC = 
6 + 5 + 3 + 4 + 7 + 2 + 9 + f = 40 n(EG) = 4
f = 40 – 36 U22
= 4 students failed E 12 G8
(b) 100 – ( %pass in at least one of Econs and Maths)
 y+x  12 - 4 4 X 7-x
= 100 –   100 
 40  8-4 - x
 7  C7
= 100 –   100 
 40  The acceptable diagram is as shown above and NOT
= 100 – 17. 5 i.e 82.5 G E C G E C
E G
(C) Pr(failed Accts) = 1 – Pr(Pass Accts)
19
= 1– C
40
21 Because econs and chem. are disjoint
= b(i.) 12 – 4 + 4 + 8 – 4 – x + x + 7 – x = 22
40
2014/ 9a 23 – x = 22
U 23 – 22 = x
P Q
x=1
16 - 2x 5x 6+ x n(CG) = 1
4x (ii) n(G)only = 8 – 4 – x
8x 7x
= 4 – 1 i.e 3
19 - 3x
R 2008/8 NABTEB
4
In a Technical College, 115 students sat for Federal Craft
In the Venn diagram, P, Q and R are subsets of the Certificate Examination ( FCCE ), 69 of them passed Physics,
universal set U. If n(U) = 125, find: 70 passed Technical Drawing and 80 passed Mathematics. Of
( i ) the value of x ; ( ii ) n(PQR1) these, 44 passed both Physics and Mathematics and 45 passed
Solution Technical Drawing and Mathematics. Given that 14 of them passed
n(U) = 125 implies everything inside the box equals 125 all the three subjects and 5 failed the three subjects, find the
16 – 2x + 5x + 4x + 8x + 19 – 3x + 7x + 6 + x + 4 = 125 (i) number of students who passed only Physics?
(ii) number of students who passed only one of the three subjects ?
4 is bolden because it is not in any circle but in the
(iii) number of students who passed only two of the three subjects?
universal sets. ( iv) probability of the number that passed only mathematics?
143
Solution 2007/10 PCE Exercise 9.27
P – Physics n(P) = 69 H I
U
T – Technical drawing n(T) = 70 8-x
6+x 2+ x
M – Mathematics n(M) = 80 x
n(PM) = 44 , n(TM) = 45, n(PMT) = 14 11-x 6-x
n(PT) only = x – 14, n(PMT)1 = 5 4+x
U115 Y
P69 T70
From the diagram above if the universal set is 40, find x.
x-14 A1 B2 C3 D4
14 2005/23 UME Exercise 9.28
44-14 45-14
Football Tennis
5
5 M 80 2 9
4
7 3
Since n(PMT) = n(U) – n(PMT)1
5
n(PMT) = n(P) + n(M) + n(T) – n(PM) – 5 Athletics
n(PT) – n(TM) + n(PMT)
115 – 5 = 69 + 80 + 70 – 44 – x – 45 + 14 The Venn diagram above shows a class of 40 students with the
110 = 144 – x games they play. How many of the students play two games only?
A 16 B4 C 19 D 15
x = 144 – 110
x = 34 2011/ 44 Exercise 9.29
S (16)
( i ) n(P)only = 69 – [ 44 – 14 + 14 + ( x – 14 ) ] 6
= 69 – [ 30 + 14 + (34 – 14)]
3
= 69 – [ 30 + 14 + 20 ] 2
3
=5 4 7
( ii ) n(P)only + n(T)only + n(M)only R(16) D(17)
n(T)only = 70 – [ 45 – 14 + 14 + ( x – 14 ) ] 5

= 70 – [ 45 – 14 + 14 + 20 ] The Venn diagram shows the number of students in a class who


=5 like reading (R), dancing (D) and swinging (S). How many
Also n(M)only = 80 – [ 44 – 14 + 14 + 45 – 14 ] students like dancing and swimming?
= 5 A7 B 9 C 11 D 13
Thus n(P)only + n(T)only + n(M)only = 5+5+5 i.e 15 2001/ 4 (Nov) Exercise 9.30
T R
5
( iii ) n(PT)only + n(TM)only + n(PM)only 2 7
= 44 – 14 + 45 – 14 + x – 14 4
3
6
= 30 + 31 + 20
1
= 81 V
5 1
( iv ) Pr(Maths)only = i.e The Venn diagram shows the number of trades that sell radio (R),
115 23 television (T) and video (V) in a market
( a ) How many of the traders sell:
Exercise 9.25 (i) both radio and television? (ii) at least two of the items?
In a Science Class, 18 students read Physics, 25 read
Mathematics, 23 read Chemistry, 9 read Physics and 2005/10 Neco Exercise 9.31
Mathematics, 10 read Mathematics and Chemistry and 6 read In a quiz competition of 60 competitors, 24drank Coca-cola, 25
Physics and Chemistry. If there are 50 students altogether and drank Fanta while 22 drank Maltina, 6 drank Coca- cola and
5 students did not read any of the three subjects. How many Maltina, 8drank Coca-cola and Fanta and 5 drank Fanta and
students read Maltina. If x number of competitors drank the three types of soft
( i ) all three subjects ( ii ) only Mathematics drinks and 5 did not drink any of them,
( iii ) Chemistry but not Mathematics ( a ) Draw a Venn-diagram to illustrate this piece of information
( iv ) Physics and Chemistry but not Mathematics ( b ) How many drank:
( i ) the three types of soft drink? ( ii ) Coca-cola only
Exercise 9.26 ( iii ) Fanta only? ( vi ) Maltina only?
A sample poll of 128 voters revealed the following
information in favour of three candidates A, B and C of 2014/ 10a Neco (Nov) Exercise 9.32
a certain party in Agbarha – Otor. 25 were in favour of In a science class of 120 students, 56 passed Physics, 61 passed
Chemistry, 60 passed biology, 20 passed both Physics and
A alone. 66 favoured B, 18 favoured C alone. 9 were
Chemistry, 24 passed both Physics and Biology, 26 passed both
sitting on the fence. 39 were in favour of exactly two Chemistry and Biology and 5 passed none of the three subjects.
candidates. 72 were in favour of exactly one candidate. Draw a Venn diagram to illustrate the information.
30 were in favour of candidate A and atleast one other Find the number of students who passed:
candidate. How many people were in favour of ( i ) all the three subjects, (ii) Physics only,
( i ) both A and C ( ii ) B and C only ( iii ) A or B (iii) Chemistry only.
144
CHAPTER TEN …At most,…
The given number or less. For example; If Tosin asked his
Probability classmate to lend him at most N 200.00. The classmate can
This is a branch of mathematical statistics basically as well give him N5, or N 10. 00, N 190. 00; as any of
concerned with drawing conclusions or inferences from these amounts will solve his problem provided the money is
occurrences or experiments involving uncertainties. For not above N 200.00.
any conclusion or inference made to be reasonably
accurate an understanding of some probability theories …, At least,…
is essential. The given number or more. If the “at most” is replaced by
What does statements such as “Nigeria will probably “at least” in the previous example then the mentioned
win the football match against Norway”, You will amount of N 10 .00 or N 190.00 or N 190.99 will not take
probably pass SSCE”, There is a 30% chance that this care of Tosin’s demand except N 200. 00 and above.
job will not be finished on time”.
In each case we are expressing the outcome of which General formula
we are not certain but because of part information or The probability of an event happening can be given a
from understanding of the structure or the experiment or numerical value x ; where
event bring about some degree of confidence in the x = number of required outcomes
validity of statement. Chances, possibly, likely, Success, number of possible outcomes
failure etc are examples of words used in probability
(a) When x = 1
statements.
If the probability of any event is1, then it is sure or
Definitions of basic terms used in probability certain to occur.
Sample space (b) When x = 0
A set whose elements represent all possible outcomes If the value of x is 0 then the event is impossible.
of an experiment usually denoted by S It will never occur.
Eg 1 A single throw of a die
S = {1, 2, 3, 4, 6} (c) The range of solution to probability problem is
between 0 and 1 i.e 0  Pr (x)  1
Eg 2 The tossing of a coin once
The implication is that we do not have negative answers nor
S = {H, T}
values greater than 1
Eg 3 The tossing of two coins
S = {HH, HT, HT, TT} (d) For any event, the probability of success plus failure:
Pr (Success) + Pr (failure) = 1
Eg 4 The launching of a missile from a Pr (x) + Pr ( x ) = 1
Submarine. S = {Success, failure}
(e) The probability of failure or non - occurrence or
Event not happening :
Any Subset of the Sample space Pr ( x ) = 1 – Pr( x )
Eg 5 Consider the experiment of tossing two coins
S = {HH, HT, TH, TT}
General formula problems in probability
Then the event (E) that head appears on the first coin is
E = {HH, HT} 2009/15NATEB (Nov)
A box contains 9 blue, 6 red and 10 white beads. If a bead is
Outcome taken at random, from the box, find the probability that it is
It refers to all possible distinct results of a trial. white
Considering the examples given so far under basic A 1/ 5 B 2/5 C 3/ 5 D 4/ 5
definitions. Solution
Examples Outcomes Pr(white bead) = Number of white beads
1 6 Total number of beads
2 2 10 2
3 4 = i.e /5 ( B )
25
4 2 2006/50 Neco
The concept of outcomes is similar to cardinality in set A bag contains 3 black, 4 yellow and 7 red balls. A ball is
i.e n (A) or n (B) depending on the given set. picked from the bag at random. What is the probability that
it is white?
A1 B 7/14 C 4/14 D 3/ 4 E 0
Experiment Solution
This is the process through which outcomes are Pr(white) = number of white balls
generated. The acts of throwing a die, tossing a coin, Total balls
e.t .c are examples of experiment. Experiment can also 0
be called trial. = i.e 0 ( E )
14
it is zero since there is no white ball in the bag.
145
2009/39 NATEB (Nov) 2006/7
A letter is chosen at random from the letters NIGERIA. A fair coin is tossed three times. Find the probability of
What is the probability that it is an ‘I’ getting two heads and one tail
A 1/ 7 B 2 /7 C 3/7 D 4/ 7 A 1/ 2 B 3/8 C 1/ 4 D 1/ 8
Solution Solution
Pr(letter ‘I’) = Number of letter ‘I’ Recall the formula for power set 2s
Total number of letters in Nigeria Here element of the sample space is 23 = 8
2 {HHH, HHT, HTH, THH, TTH, THT, HTT, TTT}
= (B)
7 3
Pr(Two heads and one tail) = (B)
2012/ 59 Neco 8
A box contains the following set of balls numbered 2006/5
3, 5, 7, 9, 11, 13, 3, 2, 3, 5, 8, 10. If a ball is picked at Two fair dice are tossed together once.
random, what is the probability that it is numbered 3? (a) Draw the sample space for the possible outcomes
A 1/12 B 1/6 C 1/ 4 D 2/ 3 E 5/12 (b) Find the probability of getting a total;
Solution ( i ) of 7 or 8 ( ii ) less than 4
Pr(3) = f(3) Solution
Σf (a) Total 1 2 3 4 5 6
3 1 1 2 3 4 5 6 7
= i.e (C)
12 4 2 3 4 5 6 7 8
3 4 5 6 7 8 9
2013/60 Neco
4 5 6 7 8 9 10
What is the probability of winning the first prize by
5 6 7 8 9 10 11
Dele who purchased 20 tickets in a lottery when 1000
6 7 8 9 10 11 12
tickets were sold?
A 1/100 B 1/50 C 1/25 D 1/ 8 E 1/ 2 11
(b) i. Pr(Total 7 or 8) =
Solution 36
20 3 1
Pr(winning) = ii. Pr(Total less than 4 ) = i.e
1000 36 12
1
= /50 ( B ) 2008/54 Neco (Dec)
A number is chosen at random from the set {1, 2, …,10}.
What is the probability that it is a prime number?
Theoretical probability A 0.3 B 0.4 C 0.5 D 0.6 E 0.7
Theoretical probability is the probability calculated Solution
without the experiment being performed, i.e using only First, we list out the members of each set
information that is known about the physical situation μ = {1, 2, 3, 4, 5, 6, 7, 8, 9, 10} ten of them
EXAMPLES Prime numbers = {2, 3, 5, 7} four of them
1. Two numbers are chosen at random from 1, 3, 6. 4
Pr(Prime number) = i.e 0.4 (B)
Find the probability that the sum of the two is not odd. 10
Solution
Arranging the given values in a tabular form, we have 2008/54 Neco (Dec) Adjusted
+ 1 3 6. A number is chosen at random from the set {1, 2, …,10}.
1 2 4 7 What is the probability that it is an odd prime number?
3 4 6 9 A 0.3 B 0.4 C 0.5 D 0.6 E 0.7
6 7 9 12 Solution
First, we list out the members of each set
Pr (not odd) = 1 – Pr (odd) μ = {1, 2, 3, 4, 5, 6, 7, 8, 9, 10} ten of them
4 Odd - Prime numbers = {3, 5, 7} three of them
=1– i.e 5/9
9 3
Pr(Prime number) = i.e 0.3 (A)
2. Find the probability that a number selected at random 10
from 41 to 56 is a multiple of 9. 2008/60 Neco (Dec)
Solution A die and a coin were thrown once, what is the probability of
Listing the numbers: 41, 42, 43, 44, 45, 46, 47, 48, 49, getting a head and six?
50, 51, 52, 53, 54, 55, 56. A 1/12 B 1/ 6 C 1/2 D 2/ 3 E1
Total numbers = 16 i.e. (56 – 41) + 1 Solution
The multiples of 9 are two i.e. 45 & 54 The sample space is as shown below:
Hence Pr (multiple of 9) = 2/16 i.e 1/8 Die
1 2 3 4 5 6
H H1 H2 H3 H4 H5 H6
coin T T1 T2 T3 T4 T5 T6
146
Solution
Total sample space : 12
(a) Pr (N 4) = f (N 4)
Head and six: 1
1
f
Pr (H6) = (A) Here f is the total number of students
12
= 5
Alternatively: Applying multiplication rule 16
Coin Die (a) Pr (N2) = f (N 2)
Sample space {H, T} {1, 2, 3, 4, 5, 6} f
1 1 =3
Pr(H) = Pr(a six) =
2 6 16
1 1
Thus Pr(a head and six) =  2006/60 Neco ( Dec )
2 6
1
Mass (kg) 55 56 57 58 59 60
= (A) No of students 4 7 12 10 8 9
12
The table above shows the masses to the nearest kilogram of
Empirical (experimental) probability 50 students in a class. If a student is chosen at random, what
Empirical (Experimental) probability is calculated using is the probability that he has mass of at least 58kg?
the results of an experiment that has been performed a A1/5 B 17/50 C 23/50 D 27/50 E 33/50
number of times. Solution
f (58kg ) + f (59kg ) + f (60kg )
In Experimental probability Pr(at least 58kg) =
Pr (E) = f (E) f
f 10 + 8 + 9
=
Where f(E) is the number of times the event E occurred 50
f is the total frequency = 27
/50 ( D )
EXAMPLES 2009/3a(iii)
1. Some families were measured by the number of The table shows the number of children per family in a
children they have; to give the following frequency community.
distribution No. of children 0 1 2 3 4 5
No of children 0 1 2 3 4 5 or more No. of families 3 5 7 4 3 2
No of families 12 28 22 8 2 2 Find the probability that a family has at least 2 children
Use this information to calculate the probability that Solution
another family of this type will have Pr(at least 2 children)
(a) 2 (b) 3 or more (c) less than 2 children = Pr(2 children) + Pr(3 children) + Pr(4 children) + Pr(5 children)
Solution f (2) f (3) f (4) f (5)
= + + +
Here, f = 74 f f f f
(a) Pr (2 children) = f (2) children =7 + 4 + 3 + 2
f 24 24 24 24
= 22 = 11 = 16
74 37 24
(b) Pr (3 or more) = f (3) + f (4) + f (5) =2
f 3
Pr (3 or more) = 8 + 2 + 2
74 2010/19
= 12 = 6 Number of pets 0 1 2 3 4
74 37 Number of students 8 4 5 10 3
The table shows the number of pets kept by 30 students in a
(c) Pr (less than 2 children) = f (1) + f (0) class. If a student is picked at random from the class, what is
f the probability that he/she kept more than one pets?
= 28 + 12 A 1/ 5 B 2 /5 C 3/5 D 4/5
74 Solution
= 40 = 20 Pr (more than one pet) = f(2) + f(3) + f(4)
74 37 f
2. The table below shows the amount of money, which = 5 + 10 + 3
some students possess in their wallet at a given time. 30
Amount in N 1 2 3 4 5 6 18 3
No of students 1 3 2 5 1 4 = i.e (C)
30 5
If a student is chosen at random from the group,
what is the probability that the student has
(a)N4 (b)N2
147
Addition Rule Solution
Under this subheading addition rule we have twin Die sample space = {1, 2, 3, 4, 5, 6} i.e 6 elements
concepts namely mutually Exclusive (M.E) and non- Pr(2 or 6) = Pr(2) + Pr(6)
mutually Exclusive Events. The words or & either are 1 1
= +
used in both cases. 6 6
(a) Mutually Exclusive events (M.E) = 2/6 i.e 1
/3 (C)
Two events A and B are M.E if the occurrence of any
one of them excludes the other. 2003/37 NABTEB
Mathematically Two dice are thrown at the same time. What is the
Pr(A or B) = Pr(AB) recall that probability that the sum will be 7 or 11 ?
Pr(AB) = Pr (A) + Pr (B) – Pr(AB) A.1/9 B.2/9 C.1/3 D.4/9
Solution
But Pr(AB) =  Since nothing is common to both The sample space is as shown below :
sum 1 2 3 4 5 6
Hence Pr (AB) = Pr (A) + Pr (B) M. E 1 2 3 4 5 6 7
Diagrammatically, M.E events A & B are represented below.
2 3 4 5 6 7 8
A B 3 4 5 6 7 8 9
4 5 6 7 8 9 10
5 6 7 8 9 10 11
6 7 8 9 10 11 12

Examples of M.E Events Pr( sum 7 or 11) = Pr (sum 7) + Pr(sum 11)


(a) The events of going to School by foot and by car = 6/36 + 2/36
are M. E; This is so because no single student can go to = 8/36
school by foot and at the same time by car . Impossible! = 2/9 (B)
(b) The events of cooking a particular pot of soup using
stove and gas cooker at the same time are M. E 2003/40 Neco
In a basket of fruits there are 6 grapes, 11 bananas and 13
(c) The events of the current principal of the oranges. If one fruit is chosen at random, what is the
Government college Ughelli to be serving and be probability that the fruit is either a grape or banana ?
retired at the same time are M.E
A.17/30 B.11/30 C.6/30 D.5/30 E11/150
Solution
E.g. 1. A haulage contractor has 3 type A, 2 type B and
Total fruit 6 + 11 + 13 i.e 30
7 type C lorries available for deliveries, all of which
Pr( a grape or banana) = Pr (grape) + Pr ( banana)
are used frequently. What is the probability that a lorry
delivering a load will be of type A or C? = 6/30 + 11/30
Solution = 17/30 ( A)
If the contractor decides to use lorry A for the supply,
automatically he cannot use lorry type C; thus, M. E 2004/35
Pr (A or C) = Pr (A) + Pr (C) The table below gives the marks scored by a group of
=3 + 7 students in a test. Use the table to answer question 35
12 12 35. What is the probability of selecting a student from the
= 10 = 5 group that scored 2 or 3 ?
12 6 Mark 0 1 2 3 4 5
Frequency 1 2 7 5 4 3
2009/59 NATEB (Nov)
A fair die is rolled once. What is the probability of A.1/11 B.5/22 C.7/22 D.6/11
obtaining either a 2 or a 5? Solution
A 1/36 B 1 /6 C 1/ 3 D 1/ 2 E 2/3 Pr( mark 2 or 3) = Pr( mark 2) + Pr( mark 3)
Solution = f(2) + f(3)
Die sample space = {1, 2, 3, 4, 5, 6,} f f
Pr(either a 2 or a 5) = Pr(2) + Pr(5) =7 + 5
Nothing is common 22 22
= 1/6 + 1/6 = 12 =6
= 2/6 i.e. 1/3 ( C ) 22 11 (D)

2012/50 Neco VTR – 12/ 6a


A fair die is rolled once. What is the probability A number is chosen at random from the set 15, 16, 17,… 32.
of obtaining 2 or 6? What is the probability that the chosen number is
A 1/36 B 1/6 C 1/ 3 D 1/2 E 2/ 3 (i) a multiple of 7?
148
(ii) a prime number? 5 4 3
= + −
(iii) a prime or a multiple of 7? 10 10 10
Solution 5+ 4−3
Sample space : {15, 16, 17, 18, 19, 20, 21, 22, 23, 24, =
10
25, 26, 27, 28, 29, 30, 31 32} 18 of them 6
= /10 i.e 3/5 ( D )
Multiple of 7 : { 21, 28} 2 of them
Prime number: {17, 19, 23, 29, 31} 5 of them
(i) Pr (multiple of 7) = 2/18 i.e 1/9 2011/6
If a number is chosen at random from the set{x : 4  x 15},
(ii) Pr (prime number) = 5/18 find the probability that it is a multiple of 3 or a multiple of 4
(iii) Pr (prime or multiple of 7 ) = 2/18 + 5/18 A 1/12 B 5/12 C 1/2 D 11/12
= 7/18 Solution
First, we list the mother set and their subsets
X = {4, 5, 6, 7, 8, 9, 10, 11, 12, 13, 14, 15}
Non – mutually Exclusive events Multiples of 3 = {6, 9, 12, 15} four of them
Two events A and B are non – mutually exclusive, if Multiples of 4 = {4, 8, 12} three of them
both events share something in common i.e. AB   AB = {12} one item
Mathematically, Pr (A or B) = Pr (AB), Pr(multiple of 3 or a multiple of 4)
Pr (AB) = Pr (A) + Pr (B) – Pr (AB) = Pr(Multiple 3) + Pr(Multiple 4) – Pr(common item)
Diagrammatically, non – M. E events A and B are 4 3 1
= + −
represented below. 12 12 12
6 1
A B = i.e (C)
12 2

2008/ 60
A class of 15 students offers either Physics or Chemistry or
both. If 11 offer Physics and 9 offer Chemistry. What is the
E.g If an event A implies obtaining prime number and probability that a student chosen at random offers both
event B implies obtaining an odd number in a single Physics and Chemistry?
throw of a die. Find the probability of events A or B A 1/ 4 B 1/3 C 3/ 5 D 3/ 4 E 1
Solution Solution
S = {1,2, 3, 4, 5, 6} Apply set formula for two subsets
A = {2, 3, 5} and B = {1, 3, 5}, also AB = {3, 5} n(PC) = n(P) + n(C) – n(PC)
Pr (A) = 3/6 Pr (B) = 3/6 Pr (AB) = 2/6 15 = 11 + 9 – n(PC)
n(PC) = 20 – 15
Thus Pr ( A or B) = Pr (A) + Pr (B) - Pr (AB) =5
= 3/ 6 + 3/ 6 -2/ 6 5 1
Thus Pr (PC) = i.e (B)
= 3+3–2 15 3
6
= 4/ 6 = 2 / 3 VTR- 12 /45
Note There are twelve cards numbered 1 to 12. A card is selected
When any of the word or & either is used, students at random. What is the probability that it is either even or a
should not jump into solving for addition. Rather, they perfect square ?
should first identify the class of the problem A.1/8 B.1/4 C.7/12 D.2/3 E.3/4
(i.e. M. E or non – M. E ). Solution
Sample space: {1, 2, 3, 4, 5, 6. 7, 8, 9, 10, 11, 12}
2008/ 54 Neco 12 of them
Bello chooses a number randomly from 1 to 10. Even cards : {2, 4, 6, 8, 10, 12,} 6 of them
What is the probability that it is either odd or prime? Perfect square cards: {4, 9,} 2 of them
A 1/10 B 2/ 5 C 1/2 D 3/5 E 9/10 EP = {4} 1 of them
Solution Pr( even or perfect square) = Pr(E) + Pr(P) – Pr(EP )
The word or here tells us of addition but first we check
= 6/12 + 2/12 – 1/12
for common terms
=6+2–1
μ = {1, 2, 3, 4, 5, 6, 7, 8, 9, 10} Ten of them 12 = 7/12 (C)
Odd = {1, 3, 5, 7, 9} five of them
Prime = {2, 3, 5, 7} four of them
Odd and Prime = {3, 5, 7} three of them
Pr(either odd or prime) = Pr(odd) + Pr(prime) – Pr (odd and prime)

149
Multiplication rule Solution
Under multiplication rule there is also twin concepts of “and” shows multiplication rule. The probability of scoring
independent and dependent events. The words “and” any number on one die is independent of scoring a number
or “both” is used in both cases on the second die. Thus
First die 2nd die
1
Independent events Pr(2) = /6 Pr(5) = 1/6
Two events A and B are said to be independent if the
occurrence or non – occurrence of event A does not Pr (2 and 5) = Pr(2) x Pr(5)
affect the probability of event B = 1/6 x 1/6
Mathematically. = 1/36 (A)
Pr (A and B) = Pr (A) x Pr (B)
i.e. Pr (AB) = Pr (A) x Pr (B) 2003/11a NABTEB
A bag contains 3 black balls and 2 red balls. A ball is picked
Examples at random from the bag and then replaced. A second ball is
1. Three balls are drawn successively from a box chosen at random, What is the probability that
containing 8 red balls, 6 white balls and 4 black balls. (i) they are both black ?
Find the probability that they are drawn in order red, (ii) one is black and the other is red ?
white and black if each ball is: Solution
(i) Replaced. The words “both” in (i) “and” in (ii) shows multiplication
Solution rule.
You will agree with the author that once a ball is (i) Pr(BB) = Pr (B) x Pr (B)
removed and replaced it does not affect the 2nd draw. = 3/5 x 3/5
Hence this is an independent event. = 9/25
Pr (RWB) = Pr (R) x Pr (W) x Pr (B)
(ii) Pr (B and R) = Pr(B) xPr (R)
= 8 6 4
= 3/5 x 2/5
18 18 18
= 8 = 6/25
243
2004/5(a) (Nov)
2008/ 38 NATEB (Nov) counter e.g A class consists of 30 boys and 20 girls. 60% of the boys and
A president and a secretary are to be chosen for a club 40% of the girls can swim. A boy and a girl are chosen at
from a group of 4 girls and 6 boys. What is the random, find the probability that both of them can swim
probability that both are girls? Solution
A 4/25 B 2/15 C 1/ 3 D 2/ 5 The word “Both” reminds us of multiplication rule ; while
Solution the boy’s ability to swim is independent of the girl’s ability.
Pr(President and Secretary) = Pr(girl president) × Pr(girl secretary)
Thus, Pr (B and G) = Pr (B) xPr(G)
4 3
=  Pr (swim) = Number of swim able pupils
10 9
Total number of pupils
2
= ( B ) dependent event Boys (swim ) = 60 x 30 = 18
15
100
Total boys = 30
2008/ 36 NATEB (Nov) Pr (swim able boys) = 18
A bag contains 8 red balls and 12 white balls of the 30
same size. If a ball is picked at random from the bag Girls (swim) = 40 x 20 = 8
and replaced and a second picked. What is the 100
probability that they are of the same colour? Total girls = 20
A 4/25 B 9/25 C 13
/25 D 17/25 Pr(swim able girls) = 8
Solution 20
Pr(same colour) = Pr(RR) or Pr(WW) Therefore, Pr(B and G) = 18 x 8 = 6
8 8 12 12 30 20 25
=  + 
20 20 20 20 Alternatively
Replaced here implies independent event The ability of the boy to swim is independent of the girls
hence the first ball picking did not affect the 2nd pick ability.
13 Pr( Boys and girls swimming) = Pr (B) x Pr (G)
= (C)
25 = 60 x 40
100 100
VTR – 11/42 NTI TCII =3 x2 = 6
A pair of fair dice with each face numbered 1 to 6 are 5 5 25
thrown once. Find the probability of scoring a 2 on one
die and a 5 on the other
A.1/36 B.1/18 C.7/36 D.1/3 E.2/3
150
Dependent events 2012/ 28
Two events A and B are said to be dependent, if the A bag contains 4 red and 6 black balls of the same size. If
occurrence of event A affect the occurrence of B. then the balls are shuffle briskly and two balls are drawn one after
Pr (A and B) = Pr (A) x Pr (B / A) the other without replacement, find the probability of
i.e Pr (AB) = Pr (A) x Pr (B / A) picking balls of different colours.
The Pr (B / A) reads probability of B given that A has A 8/15 B 13/25 C 11/15 D 13/15
occurred. Though there are cases where the candidate Solution
Drawing without replacement is a dependent event
will represent the above by Pr (AB) = Pr (A) x Pr (B)
Pr(Different colours) = Pr(R) Pr(B) or Pr(B) Pr(R)
and still apply the sense of dependence in the working.
=    +   
4 6 6 4
It is acceptable at O/L.
Examples 10 9 10 9
a. From e. g 1 under independent events lets impose a Once the first ball is picked, it will affect the total number of
second condition i.e balls in the 2nd draw
(ii) not replaced. 4 4
= +
Solution 15 15
If each ball drawn is not replaced then the number left = 8/15 (A)
inside the box must be affected; hence this case must be
dependent. 2003/38 Neco
Pr (RWB) = Pr (R) x Pr (W/R) x Pr (B/RW) A packet contains 4 red, 5 blue and 6 black identical biros.
= 8 x 6 x 4 Two biros are picked at random without replacement; find
18 17 16 the probability of picking a red and a black biro.
8+6+4 7+6+4 7 +5 + 4 A.73/105 B.2/3 C.4/35 D.8/35 E.8/125
= 2 Solution
51 Total biros: 4 + 5 + 6 i.e 15
Analysis Pr ( R and B) = Pr(R) x Pr (B/R)
In the first draw, the total balls and the number of red = 4/15 x 6/14
balls is not affected. In the 2nd draw for white, the total The second draw is affected in the total because of the NO replacement
balls has reduced as a result of the non – replacement of = 4/5 x 1/7
the first drawn ball but the number of white balls is not = 4/35 (C)
affected.In the 3rd draw, the total is affected because of
the 1st and 2nddrawn balls that are not replaced, though
the number of black balls is not affected.
b. A science class consisting of 2 boys and 8 girls all
participated in an end of year quiz. Find the probability
of choosing both slot A and B by girls from the group to
represent the college at the local government level.
Solution General problems on probability
The word “both” reminds us of multiplication rule. 2000/11a (Nov)
Let G1 – Girls choose the first slot Chinedu and Kareen take part in a test. The probability that
Let G2 – Girls choose the 2nd slot Chinedu pass the test is 1/3 and the probability that kareen
Then pass is 4/5. Calculate the probability that Chinedu or Kareen
Pr (G1 and G2) = Pr (G1) x Pr (G2/G1) passes the test.
= 8 X 7 = 0.62 Solution
10 9 Pr(Chinedu or Kareen pass) =
Once a girl has chosen the first slot, there are now Pr(Chinedu pass)Pr(Kareen fail) or Pr(Chinedu fail) Pr(Kareen pass)
7 girls left and the total number of the group reduces to 9. 1  4  1 4
=  1 −  + 1 −  
2012/56 Neco 3  5  3 5
If 2 balls are drawn from a bag containing 5 blue and 1 1 2 4
=  + 
10 red balls without replacement, find the probability 3 5 3 5
that the balls drawn are blue. 1 8
= +
A 2/21 B 1/9 C 8/21 D 13/21 E 2/3 15 15
Solution 9 3
= or
Pr(B1B2) = Pr(B1)  Pr(B2 / B1) 15 5
5 4 2003/ 24
=  Out of 60 members of an Association, 15 are Doctors and 9
15 14
After the first draw without replacement; the next draw Lawyers. If a member is selected at random from the
for blue will have 4 blue balls and the total reduced to14 Association, what is the probability that the member is
2 neither a Doctor nor a Lawyer?
= (A) A 3 /5 B 9/10 C 3/20 D 1/4
21
151
Solution 2005/44
Pr(neither Doctor nor Lawyer) = 1 – Pr(Doctor or Lawyer) A fair die numbered 1 to 6 is rolled once. What is the
 15 9  probability of obtaining 3 or 5?
= 1−  + 
 60 60  A 1/ 6 B 1/ 4 C 1/ 3 D 1/ 2
24 Solution
= 1− A die sample space is {1, 2, 3, 4, 5, 6}
60
36 3 Pr(3 or 5) = Pr(3) + Pr(5)
= i.e (A) This is so since nothing is common to both
60 5
1 1
= +
2016/29 6 6
Find the probability of picking the letter T from the 2 1
= i.e (C)
word “STUDENT” 6 3
A 1/ 7 B 1/ 6 C 2/ 7 D 1/ 3
Solution 2005/50
Pr(T) =
Numbers of letter T A bag contains four red, three white and five green balls. If
Total numbers of letters in STUDENT one ball is picked at random, what is the probability that it is
2 not green?
= (C)
7 A 7/12 B 5/12 C 1/7 D 1/12
Solution
Total ball is 12
Pr(not green) = 1 – Pr(green)
2005/7a Neco good technical example 5
= 1–
The probability that Audu and Tunde will pass a 12
mathematics test is 0.6, 0.8 respectively and the =
7
(A)
probability that Nnah will not pass the same test is 0.25. 12
Find the probability that: 2005/26 (Nov)
(a) all of them passed; The probability that a member of a club owns a car
(b) none of them passed; is 1/3. If two members are selected at random, what is the
(c) one of them failed; probability that only one of the two owns a car?
Solution A 1/ 9 B 2/9 C 4/ 9 D 5/ 9
(a) Pr(all pass) = Pr(Audu pass) Pr(Tunde pass) Pr(Nnah pass) Solution
= 0.6  0.8  (1 – 0.25) Let the two members be X and Y
= 0.6  0.8  0.75 Pr(only one owns car) = Pr(X) Pr(Y ) or Pr(X ) Pr(Y)
= 0.36
1 1  1 1
(b) Pr(all fail) = Pr(Audu fail) Pr(Tunde fail) Pr(Nnah fail) = 1 −  + 1 − 
3 3  3 3
= (1 – 0.6) (1 – 0.8)  0.25
= 0.4  0.2  0.25 1 2 2 1
=  + 
= 0.02 3 3 3 3
(c) Pr(One of them failed) = Pr(A) Pr(T) Pr(N ) or 2 2
= +
9 9
Pr(A) Pr(T ) Pr(N) or Pr(A) Pr(T) Pr(N)
4
= 0.6  0.8  0.25 + 0.6  (1 – 0.8) 0.75 + (1 – 0.6)0.8  0.75 = (C)
9
= 0.12 + 0.09 + 0.24
= 0.45 2005/ 5a (Nov)
A bag contains 5 white and 3 red identical balls. Two balls
2005/42 Neco are drawn at random, one after the other without
A bag contains 5 yellow, 6 red and 4 blue balls. If a ball replacement. Find the probability of drawing balls of the
is picked from the bag at random, find the probability same colour.
that it is NEITHER red NOR yellow Solution
A11/15 B 3/5 C 2/5 D 1/ 3 E 4/15 We have total of 8 balls
Solution Pr(same colour) = Pr(WW) or Pr(RR)
We have 15 balls in all Note: without replacement implies 1st draw will affect
Pr(neither red nor yellow) = 1 – Pr(red or yellow) 2nd draw i.e dependent event
5 4 3 2
= 1 − 
6 5 =  + 
+  8 7 8 7
 15 15 
5 3
11 = +
= 1– 14 28
15
10 + 3 13
4 = =
= (E) 28 28
15
2006/52 Neco (Dec) Prob of NOT counter eg
152
The probability that Usman, Femi and Emeka Let the three exams be A, B and C
fail a test are 1/4, 1/5 and 1/6 respectively. Find the Pr(He will not pass any of them) = Pr(fail all)
probability that all the boys pass the test. = Pr( A) Pr( B) Pr(C )
A 37/60 B 1/ 2 C 1/ 3 D 7/30 E 1/6 = (1 – 2/3) (1 – 2/3) (1 – 2/3)
Solution 1 1 1
Pr(all pass) = Pr(U) Pr(F) Pr(E) =  
3 3 3
= (1 – 1/4) (1 – 1/5) (1 – 1/6)
3 4 5 1
=   = (E)
4 5 6 27
= 1
/2 ( B ) 2007/5
Out of the 24 apples in a box, 6 are bad. If three apples are
2006/8a NATEB (Nov) taken from the box at random, with replacement, find the
Two perfect dice are thrown together. Calculate the probability that:
probability that the sum is ( i ) 9 or 10 ( ii ) at most 5 (a) the first two are good and the third is bad
Solution (b) all the three are bad
Sum 1 2 3 4 5 6 (c) all the three are good.
1 2 3 4 5 6 7 Solution
2 3 4 5 6 7 8 Let the 3 apples be X, Y and Z
3 4 5 6 7 8 9 Bad apples are 6 implies good apples are 24 – 6 i.e 18
4 5 6 7 8 9 10 Replacement implies independent events
5 6 7 8 9 10 11 (a) Pr(First two good and third bad)
6 7 8 9 10 11 12 = Pr(X)Pr(Y) Pr(Z ) + Pr(X)Pr(Z) Pr(Y ) + Pr(Y)Pr(Z) Pr(X )
7 18 18 6 18 18 6 18 18 6
( i ) Pr(9 or 10) = =   +   +  
36 24 24 24 24 24 24 24 24 24
( ii ) At most 5 implies total 5 and other lowest numbers First take does not affect the second one, this is so because
10 5 they are independent events
Pr(at most 5) = i.e
36 18 3 3 1 3 3 1 3 3 1
=   +   +  
2006/27 (Nov) 4 4 4 4 4 4 4 4 4
A class has 22 boys and 18 girls while another has 20 = 3   
3 3 1
boys and 20 girls. A student is chosen at random from 4 4 4
each class. Find the probability that both are boys. 27
A 31/40 B 11/20 C 11/40 D 9/40 =
64
Solution
Pr(Both boys) = Pr(Boy1st class) Pr(Boy in 2nd class) (b) Pr(all three bad) = Pr Pr( X ) Pr(Y ) Pr(Z )
22 20
= 
40 40 6 6 6
=  
11 24 24 24
= (C) With replacement, no take affects the other
40
1 1 1
2006/26 (Nov) =  
A bag contains 3 red, 2 blue and 5 green marbles. One 4 4 4
marble is drawn from the bag at random. What is the 1
=
probability that it is neither red nor green? 64
A 2 /3 B 3/5 C 1/2 D 1 /5 (c) Pr(all three good) = Pr(X) Pr(Y) Pr(Z)
Solution 18 18 18
=  
Total marbles is ten 24 24 24
Pr(neither red nor green) = 1 – Pr(red or green) With replacement, no take affects the other
3 3 3
= 1 – 
3 5 =
+   
 10 10  4 4 4
8 27
= 1– =
10 64
2 1
= i.e (D) 2008/ 55 Neco
10 5
Two fair dice are tossed together once. What is the
2007/51 Neco probability of getting a total of at most 7 from the outcomes?
The probability of Abdul passing any examination A 7/12 B 5/12 C 1/4 D 7/36 E 1/ 6
is 2/3. If Abdul takes three examinations, what is the Solution
probability that he will not pass any of them? A total of at most 7 implies total of 7 or less
A 2 /3 B 4 /9 C 1/3 D 8/27 E 1/27
Solution
153
Solution × 2 4 6 8
(a) Total 1 2 3 4 5 6 2 4 8 12 16
1 2 3 4 5 6 7 B 3 6 12 18 24
2 3 4 5 6 7 8 5 10 20 30 40
3 4 5 6 7 8 9 9 18 36 54 72
4 5 6 7 8 9 10 Pr(product odd) = 0 ( D )
5 6 7 8 9 10 11 All their products are even, no odd number among them
6 7 8 9 10 11 12
49. If a number is picked at random from each of the
21 7 two sets, what is the probability that their difference
Pr(at most 7 ) = i.e (A)
36 12 is 6 or 7?
A 1/256 B 1/16 C 1/8 D 1/ 2
2008/56 Solution
The chances of Usman and Dele passing a Mathematics We take absolute values of difference, since the question is
test are 3/5 and 1/3 respectively. What is the probability not restricted to any order in which the number be picked
that neither of them passes the test? A
A 3/ 5 B 2/ 5 C 4 /15 D 1/5 E 2/15 – 2 4 6 8
Solution 2 0 2 4 6
Pr(Neither pass) = Pr(both failing) B 3 1 1 3 5
= Pr(U fail) Pr(D fail) 5 3 1 1 3
= (1 – 3/ 5) (1 – 1/ 3) 9 7 5 3 1
2 2
=  Pr(Difference 6 of 7) = Pr(6) + Pr(7)
5 3 Nothing is common to both
4 1 1
= (C) = +
15 16 16
= 2/16 = 1/8 ( C )
2008/40 NATEB (Nov) 2009/41
A die is thrown twice. What is the probability that each A box contains black, white and red identical balls. The
throw results in a 2? probability of picking a black ball at random from the box is
A 1/36 B 1/ 9 C 1/ 3 D 2/ 5 3
/10 and the probability of picking a white ball at random is
Solution 2
/5. If there are 30 balls in the box, how many of them are red?
a, a 1 2 3 4 5 6
A3 B7 C9 D 12
1 1,1 1,2 1,3 1,4 1,5 1,6
Solution
2 2,1 2,2 2,3 2,4 2,5 2,6
Numbers of black balls
3 3,1 3,2 3,3 3,4 3,5 3,6
Pr(black) = number of black ball (b)
4 4,1 4,2 4,3 4,4 4,5 4,6
Total balls
5 5,1 5,2 5,3 5,4 5,5 5,6
3 b
6 6,1 6,2 6,3 6,4 6,5 6,6 =
10 30
1
Pr(each throw results in a 2) = (A) 3  30 = b
36
10
b = 9 black balls
2008/ 47 – 49 Numbers of white balls
Given the sets A = {2, 4, 6, 8} and B = {2, 3, 5, 9} Pr(white) = number of white ball (w)
Use the information to answer the questions 47 – 49. Total balls
2 w
47. If a number is selected at random from set B, =
what is the probability that the number is prime? 5 30
A1 B 3/4 C 1/ 2 D 1/ 4 2  30 = w
Solution 5
B = {2, 3, 5, 9} four of them w = 12 white balls
Prime at B = {2, 3, 5} three of them Thus red balls = 30 – (9 +12)
3 = 9 C.
Thus Pr(prime at B) =
4 2009/ 31 (Nov)
The probability that Shehu passes his examination is 3/5 and
48. If a number is picked at random from each of the the probability that he gets admitted to a University is 2/3.
two sets, what is the probability that their product
What is the probability that he passes his examination but
is odd?
does not get admitted?
A1 B 3/4 C 1/4 D 0
A 1/5 B 2/15 C 4/15 D 2/5
Solution
Solution
A
154
Pr(passes exam but not admitted) A bag contains three red balls, four blue balls, five white
= Pr(passes exam) Pr(not admitted) balls and six black balls. A ball is picked at random. What is
3  2 the probability that it is either red or blue?
=  1 − 
5  3 A 5/18 B 7/15 C 4/ 5 D 7/18
3 1
Solution
=  Pr(R or B) = Pr(R) + Pr(B)
5 3
1
Mutually exclusive events i.e nothing is common
= /5 ( A ) 3 4
= +
18 18
2009/3 (Nov) 7
The probability that a malaria patient (M) survives = /18 ( D )
when administered with a newly discovered drug
is 0.27 and the probability that a typhoid patient (T) 2010/24 NABTEB (Nov)
survives when injected with another newly discovered A coin is tossed and a die is thrown. What is the probability
drug is 0.85. What is the probability that of getting a head and perfect square?
(a) either of the two patients survives? A 1/12 B 1/4 C 1/6 D 1/3
(b) neither of the two patients survives? Solution
(c) at least one of the two patients survives? The sample space is as shown below:
Give your answers correct to 2 significant figures. Die
Solution 1 2 3 4 5 6
(a) Pr(either M or T survives) H H1 H2 H3 H4 H5 H6
M survives T dies or M dies T survives T T1 T2 T3 T4 T5 T6
= Pr(M) Pr( T ) or Pr( M ) Pr(T) Total sample space : 12
= 0.27  (1 – 0.85) + (1 – 0.27)  0.85 Head and a perfect square is 1
= 0.27  0.15 + 0.73  0.85 1
Pr(head and a perfect square) =
= 0.0405 + 0.6205 12
= 0.661
 0.66 to 2 s.f 2010/ 5
Two fair dice are thrown.
M dies T dies M is the event described by “the sum of the scores is 10” and
(b) Pr(Neither survives) = Pr( M ) Pr( T ) N is the event described by “the difference between the
= (1 – 0.27) (1 – 0.85) scores is 3”.
(a) Write out the element of M and N
= 0.73  0.15
(b) Find the probability of M and N
= 0.1095
(c) Are M and N mutually exclusive? Give reasons
 0.11 to 2 s.f
Solution
We generate two different tables
(C) Pr(at least one survives)
M survives T dies or M dies T survives or M survives T survives
(a) Table for M
Sum 1 2 3 4 5 6
= Pr(M) Pr( T ) or Pr( M ) Pr(T) or Pr(M) Pr(T)
1 2 3 4 5 6 7
= 0.27  (1 – 0.85) + (1 – 0.27)(0.85) + (0.27)(0.85) 2 3 4 5 6 7 8
= 0.27  0.15 + 0.73  0.85 + 0.27  0.85 3 4 5 6 7 8 9
= 0.0405 + 0.6205 + 0.2295 4 5 6 7 8 9 10
= 0.8905 5 6 7 8 9 10 11
 0.89 to 2 s.f 6 7 8 9 10 11 12

2010/ 55 Neco Table for N


A number is selected randomly from the set of integers Difference 1 2 3 4 5 6
1 to 20 inclusive. Find the probability that the number is 1 0 1 2 3 4 5
less or equal to 9 2 1 0 1 2 3 4
A 4/ 9 B 4/10 C 9/20 D 11/20 E 6/10 3 2 1 0 1 2 3
Solution 4 3 2 1 0 1 2
Set of integers = {1, 2, 3, … 20} 5 4 3 2 1 0 1
20 of them since 1 and 20 are inclusive
6 5 4 3 2 1 0
Subset of numbers less or equal to 9 ={1, 2, 3,…9} 9 of them
We take absolute values of difference, two fair dice are identical and when
9 tossed, we subtract the smaller value from bigger value
Pr(number less or equal to 9 ) = (C)
20 Pr(M and N) = Pr(M)  Pr(N)
3 6 1
=  =
36 36 72
(c) M and N are mutually exclusive event .
2010/25 NABTEB (Nov)
155
Event M sum occurring does excludes event N of Pr(passing all) = Pr(E) Pr(M) Pr(G)
difference. In other words we can’t take sum and 1 2 3 1
=   = (D)
difference at the same time on the values generated by 3 5 4 10
the throw of the two dice. 2011/58 Neco
In a bag containing 12 oranges and 18 apples, what is the
2010/ 47 probability of not picking apple?
In an athletics competition, the probability that an A 2/3 B 3/5 C 2/ 5 D 1/ 3 E 1/ 6
athlete wins a 100m race is 1/8 and the probability that Solution
he wins in high jump is 1/4 . What is the probability that Pr(not picking apple) = 1 – Pr(picking apple)
he wins only one of the events? 18
A 2/32 B 3/16 C 7/32 D 5/16 = 1–
30
Solution = 12
/30 i.e 2
/5 ( C )
Pr(wins only one) = 2012/52 Neco
Pr(win race) Pr(fail jump) or Pr(fail race) Pr(win jump)
The probability that Ahmed and Ade will fail Physics are 1/5
1  1   1 1
=  1 −  + 1 −   and 2/3 respectively. What is the probability that at least one
8  4   8 4
of them will pass?
1 3 7 1 A 1/15 B 2/15 C 8/15 D 3/5 E 13/15
=  + 
8 4 8 4 Solution
3 7 Note that: at least is that number or more
= +
32 32 Pr(at least one pass) = Pr(Ade pass) Pr(Ahmed fail) or
10 5 Pr(Ahmed pass) Pr(Ade fail) or Pr(Ahmed pass)Pr(Ade pass)
= i.e (D) Pr(Ahmed pass) is 1 – 1/5 i.e 4/5
32 16
2011/ 3a Neco Pr(Ade pass) is 1 – 2/3 i.e 1/3
A fair die is rolled twice. Find the probability of getting: 1 1 4 2 4 1
( i ) the same number Pr(at least one pass) =  +  + 
3 5 5 3 5 3
( ii ) at least a total of 7
1 8 4
Solution = + +
The two problems will carry different tables for clarity 15 15 15
13
Table for the same number = /15 ( E )
( i ) a, a 1 2 3 4 5 6 2012/12 Neco
The probabilities that Ahmed, Abu and Ola score 70% in a
1 1,1 1,2 1,3 1,4 1,5 1,6
Mathematic examination are 2/7, 1/6 and 3/5 respectively. If
2 2,1 2,2 2,3 2,4 2,5 2,6 the three of them sit for the mathematics examination, what
3 3,1 3,2 3,3 3,4 3,5 3,6 is the probability that:
4 4,1 4,2 4,3 4,4 4,5 4,6 (a) all of them score below 70%;
(b) at most two of them score 70%;
5 5,1 5,2 5,3 5,4 5,5 5,6
(c) all of them score 70% ?
6 6,1 6,2 6,3 6,4 6,5 6,6 Solution
6 1 Let A – Ahmed, B – Abu, O – Ola
Pr(same number) = i.e
36 6 (a) Pr(all score below 70%) = Pr(all fail)
= Pr( A ) Pr( B ) Pr( O )
( ii ) Table for total 7 and above = (1 – 2/7) (1 – 1/6) (1 – 3/5)
Sum 1 2 3 4 5 6 = ( 5/7) ( 5/6) ( 2/5)
1 2 3 4 5 6 7 = 5/21
2 3 4 5 6 7 8 (b) Pr(at most two of them score 70%)
3 4 5 6 7 8 9 = Pr(two pass) + Pr(one pass) + Pr(none pass)
4 5 6 7 8 9 10 = Pr(A)Pr(B) Pr(O ) or Pr(A) Pr(B ) Pr(O) or Pr(A ) Pr(B)Pr(O)
5 6 7 8 9 10 11
6 7 8 9 10 11 12 + Pr(A) Pr(B ) Pr(O ) or Pr(A ) Pr(B) Pr(O ) or Pr(A ) Pr(B ) Pr(O)

21 7 + Pr(A ) Pr(B ) Pr(O )


Pr(at least total 7) = i.e
36 12 2 1 2 2 5 3 5 1 3
=   +   +  
7 6 5 7 6 5 7 6 5
2011/57 Neco 2 5 2 5 1 2 5 5 3
The probability that Musa passes English, Mathematics +   +   +  
7 6 5 7 6 5 7 6 5
and Geography are 1/3, 2/5 and 3/4 respectively. What 5 5 2
is the probability of passing all? +  
7 6 5
A 3 /4 B 8/15 C 2/15 D 1/10 E 1/12
Solution
156
2 1 1 2 1 5 5 2012/ 60 Neco
= + + + + + + Two dice were thrown once. What is the probability of
105 7 14 21 21 14 21
Applying the LCM of the dominators 105, 21, 14 and 7 having a difference of absolute value of 2?
= 4 + 30 + 15 + 20 + 10 + 75 + 50 A 1/ 9 B 2/ 9 C 3/ 9 D 4/ 9 E 5/ 9
210 Solution
= 204 i.e 102 Difference of absolute value of 2 implies subtraction but all
210 105 negative results must be written in positive form. The
resulting table is
2012/ 54 Neco /– / 1 2 3 4 5 6
Two dice are thrown once. What is the probability of 1 0 1 2 3 4 5
getting a sum which is divisible by both 2 and 3? 2 1 0 1 2 3 4
A 2/ 3 B 1/ 2 C 1/3 D 1/ 4 E 1/ 6
3 2 1 0 1 2 3
Solution
We mark multiple of 2 and 3 in the sample space 4 3 2 1 0 1 2
+ 1 2 3 4 5 6 5 4 3 2 1 0 1
1 2 3* 4 5 6* 7 6 5 4 3 2 1 0
2 3* 4 5 6* 7 8 8
3 4 5 6* 7 8 9* Pr(difference of absolute 2) =
36
4 5 6* 7 8 9* 10 = 2
/9 ( B )
5 6* 7 8 9* 10 11 2012/4a
6 7 8 9* 10 11 12* A box contains 40 identical discs which are either red or
white. If the probability of picking a red disc is 1/4,
represents divisible by 2, * represent divisible by 3 calculate the number of:
Pr(sum divisible by both 2 and 3) ( i ) white discs;
= Pr(sum divisible by 2)  Pr(sum divisible by 3) ( ii ) red discs that should be added such that the probability
18 12 of picking a red disc will be 1/3
=  Solution
36 36
First, we get number of red discs i.e x
= 1/6 ( E ) 1
Note that some elements are circled and starred. Pr(red disc) =
4
Let us treat a case where this apply 1 Number of red disc
=
4 Total discs
2012/ 54 Neco Adjusted
Two dice are thrown once. What is the probability of 1 x
=
getting a sum which is divisible by 2 or 3? 4 40
A 2/ 3 B 1/ 2 C 1/3 D 1/ 4 E 1/ 6 40
= x i.e x = 10
Solution 4
We mark multiple of 2 and 3 in the sample space ( i ) Number of white disc = 40 – 10
+ 1 2 3 4 5 6 (since we have either red or white) = 30

1 2 3* 4 5 6* 7 ( ii ) Presently number of red is 10


2 3* 4 5 6* 7 8 Let r be the red discs to be added
Then from the statement in (ii)
3 4 5 6* 7 8 9*
1 10 + r
4 5 6* 7 8 9* 10 =
3 40 + r
5 6* 7 8 9* 10 11 40 + r = 3(10 + r)
6 7 8 9* 10 11 12* 40 + r = 30 + 3r
40 – 30 = 3r – r
represents divisible by 2, * represent divisible by 3 10 = 2r
Pr(sum divisible by 2 or 3) r = 5
= Pr(sum divisible by 2) + Pr(sum divisible by 3) – 2013/ 53 Neco
Pr(sum divisible by both 2 or 3)
A basket contains 50 eggs, 15 of them are bad. If an egg is
18 12 6
= + − taken out, find the probability that it is good.
36 36 36 A 3/20 B 1/ 5 C 3/10 D 7/10 E 17/20
24 Solution
= i.e 2/3 ( A )
36 Pr(good eggs) = 50 – 15 since we have 15 bad eggs
50
35
= i.e 7/10 (D)
50
157
2013/ 59 Neco 2014/11b
What is the probability that an integer x : 1 x  30 is Score 1 2 3 4 5 6
divisible by both 3 and 5? frequency 2 5 13 11 9 10
A 1/15 B 2/15 C 8/15 D 17/30 E 19/30 The table shows the distribution of outcomes when a die is
Solution thrown 50 times. Calculate the probability that a score
x : 1 x  30 = {1, 2, 3, … 30} i.e μ selected at random is at least a 4.
n(μ) = 30 Solution
Subset divisible by 3 = {3, 6, 9, 12, 15, 18, 21, 24, 27, 30} Pr(score at least 4) = f(4) + f(5) + f(6)
Subset divisible by 5 = {5, 10, 15, 20, 25, 30} Σf
Intersection: divisible by both 3 and 5 = {15, 30} = 11 + 9 + 10
n(intersection) = 2 50
2 = 30
/50 i.e 3/5
Pr(divisible by both 3 and 5) =
30
= 1/15 (A) 2009/3 Neco
During a school’s visiting day, a mother visited her daughter.
2014/ 46 She carried a bag containing 12 oranges, 16 apples and
The probability of an event P happening is 1/5 and that 8 mangoes, all of the same size. She draws 3 fruits for her
of event Q is 1/4. If the events are independent, what is daughter’s friend without replacement. Find the probability
the probability that neither of them happens? that: ( i ) the three are oranges
A 4 /5 B 3/4 C 3/5 D 1/20 ( ii ) one is apple, the other two mangoes:
Solution (iii) one is orange, one apple and a mango in that order.
Pr(P not happening) Pr(Q not happening) Solution
= (1 – 1/5) (1 – 1/4) O – Oranges, A – apples and M – mangoes
Note: “without replacement” implies one draw will affect
= 4/5  3/4
the other i.e dependent event.
= 3/5 (C)
12 11 10
2014/5 ( i ) Pr(three oranges) =  
A building contractor tendered for two independent 36 35 34
contracts, X and Y. The probability that he will win This is so, as after the first draw, the number of oranges reduced by
contract X is 0.5 and not win contract Y is 0.3. What is 1 likewise total fruits. This is also the case after the 2 nd draw.
11
the probability that he will win: =
(a) both contracts; 357
(b) exactly one of the contracts; 16 12 11
( ii ) Pr(Apple, two mangoes) =  
(c) neither of the contracts? 36 35 34
Solution After the first draw, the total fruit is reduced by 1. Now the 3rd
(a) Pr(X and Y win) = Pr(win X) Pr( win Y) draw being mango is affected by the 2nd draw both in mango
= (0.5)(1 – 0.3) since not win y is 0.3, win is 1 – 0.3 number and total fruits.
88
= (0.5) (0.7) =
= 0.35 1785
(b) Pr(win X)Pr(not win Y) or Pr(not win X)Pr(winY) 12 16 8
(iii) Pr(Orange, apple and mangoes) =  
= (0.5)(0.3) + (1 – 0.5)(0.7) 36 35 34
since win x is 0.5 not win x is 1 – 0.5 Here only the total fruits reduces after each draw
= (0.5)(0.3) + (0.5)(0.7) 64
=
= 0.15 + 0.35 1785
= 0.5
(c) Pr(not win X) Pr(not win Y) = (0.5) (0.3) 1992/5
= 0.15 (a) In a game, a fair die is rolled once and two unbiased
coins are tossed once. What is the probability of obtaining 3
2014/ 43 and a tail?
If a number is selected at random from each of the sets Solution
P = {1, 2, 3} and Q = {2, 3, 5}, find the probability that Die Coins
the sum of the number is prime. S = {1, 2, 3, 4, 5, 6} S = {HH, HT, TH, TT}
A 5/ 9 B 4/ 9 C 1/ 3 D 2/9 E (3) = {3} E (T) = {HT, TH}
1
Solution Pr (3) = / 6 Pr (a tail) = 2/4
Sum 1 2 3
2 3 4 5 Thus Pr (3 and tail) = 1/6 x 2/4
3 4 5 6 = 2 / 24 = 1/12
5 6 7 8
4
Pr(sum prime) = /9 ( B )
158
Alternatively 2007/27 Exercise 10.1
Tabulate the values as shown below The probability of an event occurring is x, what is the
1 2 3 4 5 6 probability of the event not occurring?
HH HH1 HH2 HH3 HH4 HH5 HH6 A1–x B x–1 C 0 D 1/ x
HT HT1 HT2 HT3 HT4 HT5 HT6 2005/43 Neco Exercise 10.2
TH TH1 TH2 TH3 TH4 TH5 TH6 When two dice are thrown once, what is the probability that
TT TT1 TT2 TT3 TT4 TT5 TT6 sum of the out come is less than 4?
Pr(3 and a tail ) = 2/ 24 = 1/12 A 1/12 B 1/ 9 C 5/36 D 1/ 6 E 7/36
2005/4a Exercise 10.3
1999/14
Two fair dice are thrown once. Find the probability of
A group of eleven people can speak either English or
getting: ( i ) the same digit ( ii ) a total score greater than 5
French or both. Seven can speak English and six can
speak French. What is the probability that a person 2015/5 ( Nov ) Exercise 10.4
chosen at random can speak both English and French? A fair die is thrown two times.
A. 2 B. 4 C5 D. 11 ( a ) Construct a table of the outcomes.
11 11 11 13 ( b ) Calculate the probability that the:
Solution ( i ) sum of the outcomes is 8
The given problem can not be solved without applying set theory. ( ii ) product of the outcomes is less than 10
French English U11 ( iii ) outcomes contain at least a 3
6 7
6-x x 7-x 2015/29 ( Nov ) Exercise 10.5
Out of the 20 girls in a class, 12 like music and 15 like
movies. If a girl is selected at random from the class, what is
the probability that she likes both hobbies?
6 – x + x + 7 – x = 11
A 3/ 4 B 3/ 5 C 7/20 D 3/10
13 – x = 11
13 – 11 = x 2015/1b Exercise 10.6
x = 2 people speak both languages A number is selected at random from each of the sets
Thus, Pr (speak both) = 2 (A) { 2, 3, 4 } and { 1, 3, 5 }. Find the probability that the sum of
11 the two numbers is greater than 3 and less than 7
2001/4 (b) & (c) Nov 2015/1b Exercise 10.7
U A letter is selected from the letters of the English alphabet.
T R
2 5 What is the probability that the letter selected is from the
7
3 word MATHEMATICS ?
4 6
A 9/13 B 11/26 C 4/13 D 1/26
1 V
2015/4a Neco Exercise 10.8
The venn diagram shows the number of traders that A basket contains 80 mangoes and 60 oranges. If two fruits
sell radio (R) television (T) and video (V) in a market. are picked one after the other without replacement, what is
(b) A man goes to the market to buy a set of radio and television. the probability that
What is the probability that the first trader he meets: (i)one of each fruit is picked? (ii) one type of fruit is picked?
(i) sells one of the three items only?
2015/58 Neco Exercise 10.9
(ii) Sells two of the three items only?
Five cards are numbered one to five and then mixed up. Two
(c) What is the probability that the man in (b) above,
of the cards are then chosen at random without replacement.
buys the radio and television in a video shop.
Find the probability that at least one of them is a prime
Solution
A good knowledge of set as treated in this book will give
number
candidate an upper hand in answering this question. A 3/ 5 B 18/25 C 3/4 D 19/25 E 9/10
b.(i.) Pr (one of three) = f (E) 2015/59Neco Exercise 10.10
f A number is chosen at random from the set
=2+7+1 {1, 2, 3, …,19, 20}. What is the probability that the number
28 is prime and even ?
= 10 = 5 A 1/20 B 7/20 C 2/5 D 13/20 E 19/20
28 14 2014/5 Neco ( Nov ) Exercise 10.11
(ii) Pr (two of three items) = f (E) Two fair dice, each of which has its face numbered
f 1, 2, 3, 4, 5, 6 are tossed at the same time.
=5+4+6 ( a ) Construct a table for the sample space of the products
28 of the numbers shown
= 15 ( b ) Find the probability of the products which are
28 perfect cube
C. Pr (R & TV) = 3 ( c ) What is the probability of obtaining the products
28 which are divisible by or a multiple of 3 ?
159
2014/58 to 60 Neco ( Nov ) Exercise 10.12 CHAPTER ELEVEN
The table below shows the marks scored by a number of
students in geography test. Sequence (AP & GP ) and series.
When numbers appear or are presented one after the other in
Marks(%) 12 16 27 30 36 49 64
an orderly manner, we then refer to the set of numbers as
No. of Cand. 12 6 3 10 10 14 5
sequence.
Use the information above to answer questions 58 to 60
Examples;
58. What is the probability that a student picked at 1, 2, 3, 4,…
random did not score a mark that is a perfect cube? 5, 10, 20, 40,…
A 2/15 B 5/12 C 13/15 D 11/12 E 19/20 -3, -1, 1, 3,…
59. If 50 is the pass mark, what is the probability that a Series is the summation (addition) of the terms of
student failed the exam? a sequence. For example, the series equivalent of
A 1/12 B 7/20 C 29/60 D 31/60 E 11/12 the above sequence are;
1 + 2 + 3 + 4 +…
60. What is the probability that a student picked at 5 + 10 + 20 + 40 +…
random scored a mark that is a perfect cube? ( -3 ) + ( -1 ) + 1 + 3 +…
A 7/20 B 5/12 C 29/60 D 1/ 2 E 7/12
Arithmetic Progression ( AP )
2014/6 NABTEB ( Nov ) Exercise 10.13
Arithmetic progression is a form of sequence in
A die numbered 1 to 6 is rolled once. What is the
which each term is gotten by the addition of a
probability of obtaining 8 or 10 ?
common difference to the preceding one. Alternatively, we
A0 B 1/ 2 C 1/3 D 1/ 4
can say that APs or linear sequence are sequence that
2014/8a NABTEB ( Nov ) Exercise 10.14 follows simple addition rule.
Two dice are thrown together once. Find the probability Examples; let us examine the pattern of the APs below;
that the following are obtained: ( 1) 1, 2, 3, 4, 5,…
( i ) a 5 ( ii ) a multiple of 3 ( iii ) equal likely numbers We observe that;
Items pattern
2014/38 NABTEB Exercise 10.15 1
A bag contains 4 blue biros, 5 red biros and 6 black 2 =1+1
biros. If a student picks one biro at random. What is the 3 =2+1
probability that it is not black ? 4 =3+1
A 2/25 B 4/25 C 6/25 D 9/25 5 =4+1
2013/49 UTME Exercise 10.16 Conclusion; 1 is the common difference and first term is 1.
What is the probability that an integer x (1 x  25) ( 2 ) 12, 19, 26, 33,…
chosen at random is divisible by both 2 and 3? We observe that
A 4/25 B 3/4 C 1/25 D 3/ 5 Terms pattern
2014/49 UTME Exercise 10.17 12
19 = 12 + 7
Numbers 1 2 3 4 5 6 26 = 19 + 7
Frequency 18 32 20 16 10 14 33 = 26 + 7
The table above represents the outcome of throwing a Conclusion; 7 is the common difference and first term is 12
die 100 times. What is the probability of obtaining at
least a 4 ? nth term of an AP
A 2/ 5 B 3/ 4 C 1/ 5 D 1/ 2 Conventionally, we denote the first term by ‘a’ and the
common difference by ‘d’. Let n represent the nth term, we
2014/50 UTME Exercise 10.18 observe that the examples given above follow a simple pattern:
A number is chosen at random from 10 to 30 inclusive, a, a + d, a + 2d, a + 3d
what is the probability that the number is divisible by 3? 1st 2nd 3rd 4th
A 1/ 3 B 3/ 5 C 2/15 D 1/10 For ( 1 )
2006/37 PCE Exercise 10.19 T1 = 1
A number is chosen at random from 1 to 24 inclusive. T2 = 2 = 1 + ( 2 – 1 ) 1
What is the probability that the number is divisible by 5? =1+1
A 1/ 5 B 1/ 6 C 1/12 D 1/24 T3 = 3 = 1 + ( 3 – 1 ) 1
2008/50 PCE Exercise 10.20 =1+2
If a die is tossed twice, what is the probability that the T4 = 4 = 1 + ( 4 – 1 ) 1
sum of the outcome is 6? =1+3
A 1/18 B 1/12 C 1/9 D 5/36
Then Tn = 1 + ( n – 1 ) 1
=1+n–1
=n
160
As for ( 2 ) 2010/10 Neco
T1 = 12 Find the common difference in an Arithmetic Progression
T2 = 19 = 12 + ( 2 – 1 ) 7 given that their first and 28th terms are –14 and – 5
= 12 + 7 respectively.
A–3 B – 1/3 C 1/3 D 11/4 E 3
T3 = 26 = 12 + ( 3 – 1 ) 7
Solution
= 12 + 14
a = – 14, T28 = – 5, d = ?
T4 = 33 = 12 + ( 4 – 1 ) 7 Applying Tn = a + (n – 1)d
= 12 + 21 T28  – 5 = –14 + (28 – 1)d
Then Tn = 12 + ( n – 1 ) 7 – 5 = – 14 + 27d
= 12 + 7n – 7 14 – 5 = 27d
= 5 + 7n 9 = 27d
Generally if we are given any A.P: a, a + d, a + 2d,… Thus 9/27 = d i.e 1/3 ( C )
Then
Tn = a + ( n – 1 )d 2006/9 Neco (Nov)
This is the formula for finding the nth term of an AP The 12th term of an AP is 32. If the 1st term is 11/2, what is
its common difference? (Correct to 1 decimal place)
2006/35 (Nov) A 2.7 B 2.8 C 5.4 D 5.5 E 5.6
What is the 23rd term of the Arithmetic Progression Solution
–7, –3, 1, …? d = ?, n = 12, T12 = 32, a = 11/2 i.e 3/2
A 65 B 77 C81 D90 Tn = a + (n – 1)d
Solution Substituting
a = – 7, T23 = ? d = –3 – (– 7) = – 3 + 7 i.e 4 T12  32 =
3
+ (12 − 1)d
Tn = a + (n – 1)d 2
T23 = –7 + (23 – 1)4 3
32 = + 11 d
= –7 + (22)  4 2
= – 7 + 88 Multiply through by 2 to clear fraction
= 81 (C) 64 = 3 + 22d
64 – 3 = 22d
2014/48 61 = 22d
Find the number of terms in the Arithmetic Progression d = 61/22
(A.P) 2, –9, –20, … , –141 = 2.773
A 11 B12 C 13 D 14
 2.8 to 1 dp ( B )
Solution
a = 2, d = –9 – 2 i.e – 11, n = ? But Tn = –141
2014/10 NABTEB (Nov) counter eg
Applying Tn = a + (n – 1) d
The first term of an arithmetic progression is 8. If the tenth
Substituting
term is double the second term, the common difference is
– 141 = 2 + (n – 1)(–11)
A 4/ 3 B 8/ 7 C 7/8 D1
– 141 = 2 – 11n + 11
Solution
– 141 = 13 – 11n
a = 8, T10 = 2  T2, d = ?
11n = 13 + 141
Applying Tn = a + (n – 1)d
11n = 154
From the statement
n = 154/11 i.e 14 (D)
T10 = 2  T2
2006/2 Neco 8 + (10 – 1)d = 2  [8 + (2 – 1)d]
The 12th term of an AP is – 41. If the first term is 3, 8 + 9d = 2  [8 + d]
find the 20th term. 8 + 9d = 16 + 2d
A – 77 B – 73 C 77 D 79 E 83 9d – 2d = 16 – 8
Solution 7d = 8 thus d = 8/7 ( B )
n = 12, T12 = – 41, a is 3, d = ? 2006/11b
Tn = a + (n – 1)d The 2nd, 3rd, and 4th terms of an AP are x – 2, 5 and x + 2
Substituting respectively. Calculate the value of x.
T12  – 41 = 3 + (12 – 1)d Solution
– 41 = 3 + 11d In AP common difference rule is
– 41 – 3 = 11d 3rd term – 2nd term = 4th term – 3rd term
– 44 = 11d 5 – (x – 2) = (x + 2) – 5
– 44
/11 = d Thus d = – 4 5–x+2 = x+2–5
20th term: T20 = 3 + (20 – 1)( – 4 ) 7–x = x–3
= 3 + (19) (– 4 ) 7+3= x+x
= 3 – 76 10 = 2x
10
= – 73 ( B ) /2 = x thus x = 5
161
2007/ 2b 2006/12b (Nov) Exercise 11.1
1 1 1 The numbers 11, p, q, 211/2, … form an arithmetic
If , , are successive terms of an arithmetic progression (A.P). Find the value of p and q
2 x 3
2− x 2 2006/4b NABTEB (Nov) Exercise 11.2
progression (A.P), show that =
x−3 3 If – 8, x, y, 19 are a sequence in Arithmetic Progression
Solution (A.P), find the value of x and y
In AP, common difference rule is 2008/13a Exercise 11.3
2nd term – 1st term = 3rd term – 2nd term If 3, x, y, 18 are in Arithmetic progression (AP), find the
1 1 1 1 value of x and y.
− = −
x 2 3 x 2015/47 ( Nov ) Exercise 11.4
Solving both sides separately by LCM The first four terms of a linear sequence ( AP)
2− x x −3 are 5, 9, 13, 17. What is the 10th of the sequence ?
=
2x 3x A 29 B 30 C 41 D 45
Having our target in mind 2015/9a Exercise 11.5
3x(2 − x) The first term of an Arithmetic Progression ( AP) is – 8
= x −3
2x If the ratio of the 7th term to 9th term is 5 : 8, find the
Divide through by 2 – x common difference of the AP
3x x −3
=
2x 2− x Term’s formula for particular AP
3 x −3 It has been noted earlier that:
=
2 2− x Tn = a + (n – 1) d
Taking the reciprocal of both sides This is the general formula for finding the nth term of any
2 2− x AP. But, every A.P has its own particular term formula
= QED which is a short-cut to deriving the other members of the
3 x −3
progression
2012/6b Neco
If 8, x, y, z, 32 is an AP, find the value of x, y, and z E.g 1 Find the nth term of the AP 10, 13, 16, …
Solution Analysis and solution
T5 = 32, n = 5, a = 8, d = ? Applying the general formula
Applying Tn = a + (n – 1) d Tn = a + (n – 1) d
Substituting n = n, a = 10, d = 3
T5  32 = 8 + (5 – 1) d Substituting
32 = 8 + 4d Tn = 10 + (n – 1) 3
32 – 8 = 4d = 10 + 3n – 3
24 = 4d = 7 + 3n or = 3n + 7
24
/4 = d thus d = 6 2009/9 Neco
x y z Find the nth term of an AP whose first three elements
AP : 8, 14, 20, 26, 32, … are 3/4, 2, 31/4
2005/4 Neco A ½(4 – 3n) B 1/3(4n – 1) C 1/4(5n – 2)
If 2, a, b, c, d, 22 are terms of an Arithmetic Progression, D /5(3 – 4n)
1
E /6(4 – 5n)
1

find bd – ac Solution
A 4 B 16 C 38 D 96 E 108 a = 3/4, d = 2 – 3/4 i.e 5/4, n = ?
Solution Tn = a + (n – 1) d
If we use common difference principle Tn = 3/4 + (n – 1)5/4
a – 2 = 22 – d it leads no where 3 5 5
Applying nth term formula = + n −
4 4 4
Tn = a + (n – 1)d
3 5 5
With n = 6 = − + n
T6 = 22 and a = 2, d = ? 4 4 4
T6  22 = 2 + (6 – 1)d −2 5 5 2 1
= + n i.e n − = (5n – 2) (C)
22 = 2 + 5d 4 4 4 4 4
22 – 2 = 5d 2005/1b NABTEB (Nov)
20 = 5d Un = 3n + 2 is the nth term of an AP.
d = 20/5 i.e 4 List the first three terms of the sequence
a b c d Solution
AP : 2, 6, 10, 14, 18, 22 1st term i.e n = 1
Thus bd – ac = 10  18 – 6  14 U1 = 3  1 + 2
= 180 – 84 = 3+2
= 96 ( D ) = 5

162
2nd term i.e n = 2 a + 3d = 13
U2 = 3  2 + 2 –(a+d = 3 )
=6+2 2d = 10
=8 d = 10/2 i.e 5
3rd term i.e n = 3 Substituting d value into (2)
U3 = 3  3 + 2 a + d = 3 becomes
= 9+2 a+5=3
= 11 a = 3 – 5 i.e –2
1995/7 Exercise 11. 6 T5 = – 2 + (5 – 1)5
Find the nth term Un, of the A. P. 11, 4, -3… = – 2 + (4  5)
A. Un = 19 + 7n B. Un = 19 – 7n C. Un = 18 – 7n = – 2 + 20 i.e 18
D. Un = 18 + 7n E. Un = 17 – 7n 1994/ 11a
2002/17 Exercise 11. 7 The fourth term of an A. P is 37 and the 6th term is 12 more
Find the nth term of the sequence 4, 10, 16 … than the fourth term. Find the first and seventh terms
A. 2 (3n –1) B. 2 (2 + 3 n – 1) C. 2n + 2 Solution
D. 2 (3n + 2) Applying the formula Tn = a + (n – 1 ) d, we have,
Fourth term i.e. T4 = 37 = a + (4 - 1) d
1993/25 PCE Exercise 11.8
37 = a + 3d ----- (1)
Given the progression 5, 8, 11, 14,…
Sixth term T6 = a + (6 – 1 ) d
Find the expression for the nth term of the progression
T6 = a + 5d
A. n( n + 1 ) B. 5 + 3n C. 3n + 2 D. 5 + 2n
‘…the 6th term is 12 more than the fourth term.’
T6 = T4 + 12
Unknown ‘a’ and ‘d’
i.e T6 = 37 + 12
(resulting to simultaneous Linear equations) i.e. 37 + 12 = a + 5d
There are cases in A. P where two terms of the 49 = a + 5d ------- (2)
progression are given and we are asked to find the first Solving equations (1) and (2),
term “a” and common difference “d ” ; the problem a + 5d = 49
that follows leads us to simultaneous equations that can – (a + 3d = 37)
be solved either by substitution or elimination method. 2d = 12
Eg1 In an A.P the 10th term is 68 while its 4th term is 26 d = 12/2 = 6
Find (a) common difference (b) first term Substitute d value into (1)
Solution a + 3d = 37 will become
T10 = 68 = a + (10 – 1) d a + 3(6) = 37
68 = a + 9d ----(1) a = 37 – 18
Also T4  26 = a + (4 – 1) d a = 19, first term
26 = a + 3d ------ (2) (b) 7th term
Solving equations (1) and (2) T7 = 19 + (7- 1) 6
a + 9d = 68 = 19 + (6) 6
– (a + 3d = 26 ) = 19 + 36 = 55
6d = 42 2000/ 12a
d = 42 = 7 The first term of an A. P is – 8. The ratio of the 7th term to the 9th
term is 5 : 8 . Calculate the common difference of the progression.
6
Solution
( b ) Substituting d value into (2) Applying the formula Tn = a + (n – 1) d, we have
a + 3d = 26 will become
T7 = a + (7 – 1)d
a + 3 x 7 = 26 = a + 6d
a + 21 = 26 Also T9 = a + 8d
a = 26 – 21 = 5 But the question specified that
T7 = 5
2014/11b NABTEB (Nov) T9 8
The 4th term of an Arithmetic Sequence is 13 and the a + 6d = 5
second term is 3. Find the common difference and the a + 8d 8
fifth term Cross multiplying,
Solution 8 (a + 6d) = 5 (a + 8d)
T4 = 13, T2 = 3, d = ? 8a + 48d = 5a + 40d
Tn = a + (n – 1)d 8a – 5a = 40d – 48d
T4  13 = a + (4 – 1)d 3a = – 8d But a = – 8, thus,
13 = a + 3d ------ (1) 3 (– 8) = – 8d
Also T2  3 = a + (2 – 1)d – 24 = – 8d
3 = a + d ----- (2) – 24 = d , d = 3, common difference.
Solving the resulting simultaneous linear equations –8
163
2014/ 19 UTME Exercise 11.9 (ii) find d from Tn = a + (n – 1)d
The fourth term of an A.P is 13 while the 10th term Since n = 22, T22 = – 5 , a = 79, d = ?
is 31. Find the 24 term.
A 73 B 69 C 89 D 75 T22  – 5 = 79 + (22 – 1) d
2010/22 UTME Exercise 11.10 – 5 = 79 + 21d
The 3rd term of an A. P is –9 and the 7th term is –29. – 5 – 79 = 21d
Find the 10th term of the progression. – 84 = 21d
A 44 B –165 C –44 D 165 – 84 = d
21
2004/7b Neco (Nov) Exercise 11.11 d= –4
The third and seventh terms of an AP are 27 and 91
respectively Find: 2006/10 NABTEB (Nov)
( i ) the first term and common difference Find the sum of the first 10 terms of the AP:
( ii ) the 22nd term of the Arithmetic progression. 2, 21/2 , 3,…
1 1
A 6 /2 B 7 /2 C 321/2 D 421/2
Solution
n
S10 = ? Applying Sn = {2a + (n − 1)d }
2
Sum of an A. P n = 10, a = 2, d = 21/2 – 2 i.e 1/2
The sum of terms of an A. P refers to the addition of all Substituting
the terms in the given A. P. It is usually denoted by Sn; 10  1
S10 = 2  2 + (10 − 1) 
where n is the number of terms in the progression. 2  2
Given any A. P. where T1 = a 1st term 1
T2 = a + d (2nd term) = 5{ 4 + (9) }
2
T3 = a + 2d (3rd term) 9 17 85
Tn = a + (n – 1) d (nth term). = 5{4 + } = 5( ) = i.e 421/2
2 2 2
Then, the sum from the 1st term to the nth term
Sn = a + (a + d) + (a + 2d) +… + [a + (n – 1) d] - -1 2005/7 NABTEB (Nov)
Also the sum from the nth term to the 1st term Find the sum of the first 10terms of an AP whose first term
Sn=[a + (n – 1) d] + [a + (n – 2) d] +… + (a + d) + a - -2 and common difference are –1 and 2 respectively.
Adding (1) and (2) termly as they occur A 80 B 85 C 90 D 95
2Sn =[2a+(n – 1)d] + [2a+(n – 1)d] +… + [2a + (n – 1)d] Solution
We observe that [2a + (n –1)d] is reoccurring n times in n
the right hand side of the equation S10 = ? Applying Sn = {2a + (n − 1)d }
2
Thus, we can say 10
Sn = {2(−1) + (10 − 1) 2}
2Sn = n x {2a + (n – 1)d} 2
 Sn = n {2a + (n – 1)d} = 5{–2 + (9) 2}
2 = 5{–2 + 18}
A case where the first term (a) and last term (L) are given is: = 5(16) i.e 80 (A)
Sn = n (a + L)
2 2008/6 NABTEB (Nov)
Find the sum of the first 12 terms of the A.P 2, 5, 8, 11, …
2002/8c NABTEB A 26 B52 C 222 D 444
The first and last terms of an A.P are 79 and – 5. Solution
If the sum is 814, find: S12 = ? n = 12, a = 2, d = 5 – 2 i.e 3
(i) the number of terms in the AP and Applying Sn =
n
{2a + (n − 1)d }
(ii) the common difference 2
Solution 12
S12 = {2  2 + (12 − 1) 3}
The question wants us to find n from the formula for the 2
1st and last term sum i.e. = 6{4 + (11)3}
Sn = n (a + L) = 6(4 + 33)
2 = 6(37)
814 = n {79 + (– 5) } = 222 (C)
2
814 x 2 = n (79 – 5)
814 x 2 = 74n
2009/1a Neco (Nov)
814 x 2 = n
4 5
74 1, , , … is an AP. Find the:
22 = n 3 3
( i ) 10th term ( ii ) sum up to the 16th terms
164
Solution 10  25 
Thus, S10 = 2  7 + (10 − 1) 
( i ) 10th terms is T10, Applying Tn = a + (n – 1)d 2  3
n = 10, a = 1, d = 4/3 – 1 i.e 1/3 25
Substituting = 5{ 14 + (9) }
3
1 = 5{14 + 75}
T10 = 1 + (10 – 1) ×
3 = 5(89)
1 = 445
= 1+9×
3
= 1 + 3 i.e 4 2014/9 Neco (Nov)
n The sum of 6 terms of an A.P is 6 and the sum of
( ii ) S16 = ? Applying Sn = {2a + (n − 1)d} 18 terms is 72. Find the common difference
2
Here n = 16, a = 1 , d = 1/3 A – 1/ 2 B – 1/ 4 C 1/ 4 D 1/2 E 3/ 4
16  1 Solution
S16 = 2 1 + (16 − 1)  S6 = 6 and S18 = 72, d = ?
2  3 n
Applying Sn = {2a + (n − 1)d }
1 2
= 8(2 + 15 × )
3 6
S6  6 = {2a + (6 − 1)d}
= 8(2 + 5) 2
= 8(7) 6 = 3(2a + 5d)
= 56 Divide both sides by 3
2 = 2a + 5d ------- (1)
18
2007/13 Neco counter eg on sum Also S18  72 = {2a + (18 − 1)d }
In an Arithmetic Progression (A.P), the first term is 3, 2
and the sum of the first and the sixth terms is 20. What 72 = 9(2a + 17d)
is the 8th term? Divide both sides by 9
A 22.6 B 25.6 C 28.6 D31.6 E 34.6 8 = 2a + 17d ------ (2)
Solution Solving the resultant simultaneous linear equations
a = 3, T1 + T6 = 20, T8 = ? 2a + 17d = 8
Applying Tn = a + (n – 1) d – ( 2a + 5d = 2 )
T1 + T6 = 20 12d = 6
3 + [3 + (6 – 1)d] = 20 d = 6/12 i.e 1/2 ( D )
3 + 3 + 5d = 20
5d = 20 – 6
5d = 14 2013/6 Neco
d=
14 The sum of the first twelve terms of an A.P is – 96 and the
5 sum of the first eighteen terms is – 252. Find the:
14 (a) common difference (b) first terms
Thus, T8 = 3 + (8 – 1) 
5 (c) product of 15th and 4th terms
14
= 3+7 Solution
5 S12 = – 96, S18 = – 252, d = ?, a = ?
98
= 3+ Applying Sn =
n
{2a + (n − 1)d }
5 2
113 12
= i.e 22.6 ( A ) S16  – 96 = {2a + (12 − 1)d }
5 2
– 96 = 6(2a + 11d)
2005/7a NABTEB (Nov) Divide both sides by 6
The first term of an AP is 7 and the fourth term is 32, – 16 = 2a +11d ------ (1)
find the sum of the first 10 terms 18
Solution Also, S18  – 252 = {2a + (18 − 1)d }
2
S 10 = ? a = 7, T4 = 32 –252 = 9(2a + 17d)
To find S10, we need a and d Divide both sides by 9
a is given but we get d from T4 = 32 –28 = 2a + 17d ------- (2)
Applying Tn = a + (n – 1)d Solving the resultant simultaneous linear equations
T4  32 = 7 + (4 – 1)d 2a + 17d = –28
32 = 7 + 3d – (2a + 11d = –16 )
32 – 7 = 3d 6d = –12
25 = 3d
25 d = – 12 /6 i.e – 2
/3 = d Substituting d value into (1)
n
Applying Sn = {2a + (n − 1)d } 2a + 11d = –16 becomes
2

165
2a + 11(– 2) = –16 Thus cost of the 12 poles at N 45.00 per meter
2a – 22 = –16 = 111  45
2a = –16 + 22 = N 4,995.00
2a = 6
2003/5 NABTEB good case
a = 6/2 i.e 3
The first term of an AP is 3 and the fifth term is 9. Find the
(c) T15  T4 = ? number of terms in the progression if the sum is 81
T15 = 3 + (15 – 1)(–2) Solution
= 3 + 14(–2) Sn = 81, find n a = 3, T5 = 9
= 3 – 28 i.e –25 To find n from Sn = 81, we need a and d
T4 = 3 + (4 – 1)(–2) a is given as 3, next we get d from T5 = 9
= 3 + 3(–2) Applying Tn = a + (n – 1)d
= 3 – 6 i.e –3 T5  9 = 3 + (5 – 1)d
Thus T15  T4 = – 25  (– 3) 9 = 3 + 4d
= 75 9 – 3 = 4d
6
/4 = d i.e d = 3/2
2009/4 Neco n 3
Thus Sn  81 = 2  3 + (n − 1) 
Twelve iron poles are to be used for pillar and the 2 2
lengths of the poles form an Arithmetic Progression
n 3 3
(A.P). If the third pole is 4m and the seventh is 10m, 81 = 6 + n − 
find the: 2 2 2
(a) lengths of the poles in order of the A.P;  3 3 
(b) cost of all the poles if a meter costs N 45.00 81  2 = n 6 − + n
 2 2 
Solution
T3 = 4m, T7 = 10m  9 3 
162 = n  + n
Tn = a + (n – 1)d  2 2 
T3  4 = a + (3 – 1)d 162 =
9 3 2
n + n
4 = a + 2d ------- (1) 2 2
T7  10 = a + (7 – 1) d Multiply through by 2 to clear fractions
10 = a + 6d ------ (2) 324 = 9n + 3n2
Solving the resultant simultaneous linear equations 3n2 + 9n – 324 = 0
a + 6d = 10 Divide through by 3
– ( a + 2d = 4 ) n2 + 3n – 108 = 0
4d = 6 Factorizing
d = 6/4 i.e 3/2 n2 + 12n – 9n – 108 = 0
Substituting d value into (1) n(n + 12) – 9(n + 12) = 0
a + 2d = 4 will become (n + 12)(n – 9) = 0
a + 2(3/2) = 4 n = –12 or 9
a+3= 4 We accept n = 9
a= 4–3 2013/9 Neco Exercise 11.12
a = 1 The sum of the first three terms of an AP is 12.
( a ) Lengths of poles in AP (12 of them) If its 4th term is 8, find its common difference
1m, 1 + 3/2, (1 + 3/2) + 3/2 , … A 2 B4 C6 D8 E 12
1m, 21/2, 4m, 51/2m, 7m, 81/2m, 10m, 111/2m, 2011/5a Neco Exercise 11.13
13m, 141/2m, 16m, 171/2m Find the sum of the first 9 terms of the AP:
For any A.P the first three terms are Ok except – 24, –18, –12, …
otherwise directed by the question as in this case. 2006/8b Neco Exercise 11.14
The fourth and ninth terms of an arithmetic progression
(b) First we find sum of all the 12 poles length are – 3 and 12 respectively. Find the
n ( i ) Common difference ( ii ) fifth term,
Sn = {2a + (n − 1)d }
2 (iii) number of terms which will give a sum of 135
12  3
S12 = 2  1 + (12 − 1) 
2  2 2009/18 UME Exercise 11.15
3 The sum of the first n terms of the arithmetic progression
= 6(2 + 11 × ) 5, 11, 17, 23, 29, 35, … is
2
33 A n(3n – 0.5) B n(3n + 2) C n(3n + 2.5) D n(3n + 5)
= 6(2 + )
2
2006/36 UME Exercise 11.16
37
=6× i.e 111m The sum of the first n positive integers is
2
A n(n – 1) B n(n + 1) C 12 n(n + 1) D 1
2
n(n – 1)
166
Geometric Progression (GP) ( ii ) T42 = ? a = 4, r = 8/4 i.e 2
A geometric progression (GP) is a form of sequence, Applying Tn = ar n – 1
which has a common ratio between any of the terms T42 = 4(2)42 – 1
and its preceding term. = 4(241)
Examples = 22  241
1.) 2, 4, 8, … = 243 in index form
We observe that common ratio(r) i.e. 4/2 = 8/4 = 2 2006/11 NABTEB (Nov)
2.) 14, 98, 686, … The first and third terms of a GP are 1 and 9 respectively.
Common ratio (r) = 98 = 686 = 7 What is the second term?
14 98 A 4 B 3 C 2 D 1/ 3
nth term of a G. P. Solution
Conventionally in a GP, we denote the first term by ‘a’ a = 1, T3 = 9, T2 = ?
and the common ratio by ‘r’ Applying Tn = ar n – 1
Analyzing the given examples, we have T3  9 = 1 r3 – 1
T1 T2 T3 Tn 9 = r2
Eg 1 2 2  21 2  2 2 =8 2  2n – 1 Raise LHS in power 2
= ar n – 1 32 = r2 Thus 3 = r
Eg 2 14 14  71 = 98 14  72 = 686 14  7n– 1 T2 = 1  3 2–1

= arn – 1 = 1  31
= 3(B)
In general, for any given G. P a, ar, ar2 ... arn Alternatively
Tn = arn - 1 Applying common ratio principle:
2013/8 Neco For a GP: a, ar, ar2
Find the 7th term of the geometric progression 4, 12, 36, … a T2 T3
A 365 B 729 C 1458 D 2916 E 4374 T3 T
Solution Common ratio r : = 2
T2 a
a = 4, r = 12/4 i.e 3, T7 = ?
a(T3) = (T2)2
Applying Tn = ar n – 1
T7 = 4(3) 7 – 1 1  9 = (T2)2
= 4(36) 32 = (T2)2
= 4(729) = 2916 ( D ) 3 = T2 (B)
2014/9 Neco
2009/9 NABTEB (Nov)
The first, second and last term of a GP are 3, 6 and 1536
The first term of a geometric progression is 6 and its
respectively. Calculate the number of terms in the G.P
common ratio is 3. Find the sixth term
A8 B 9 C 10 D12 E 14
A 185 B 3  95 C 3  65 D 6  35 Solution
Solution a = 3, ar = 6, Tn = 1536, n = ?
a = 6, r = 3, T6 = ? We can get n from Tn = ar n – 1
Applying Tn = ar n – 1 First, we get r from : ar = 6 and a = 3
T6 = 6(3) 6 – 1 (3)r = 6
= 6(35) i.e 6  35 (D ) r = 6/3 i.e 2
2007/2 Neco Then from Tn = 1536
If y + 2, y + 6 and y + 14 are consecutive terms of a Tn  1536 = 3(2) n – 1
geometric progression (GP), find the: Divide both sides by 3
( i ) value of y and hence the GP 1536
( ii ) 42nd term of the GP leaving your answer in index form. = 2 n −1
3
Solution
( i ) For the G.P: y + 2, y + 6, y + 14, … 512 = 2n – 1
By common ratio principle Next, we express LHS in 2 and its power
y +6 y + 14 29 = 2n – 1
= Both sides are now in same base 2, we equate powers
y+2 y +6
9=n–1
Cross multiply 9+1= n
(y + 6)(y + 6) = (y + 14)(y + 2) 10 = n (C)
y(y + 6) + 6(y + 6) = y(y + 14) + 2(y + 14)
2015/4 Neco Exercise 11.17
y2 + 6y + 6y + 36 = y2 + 14y + 2y + 28
Find the 12th term of the G.P –3, 6, –12, …
y2 + 12y + 36 = y2 + 16y + 28
A –12288 B – 6144 C – 2048 D 2048 E 6144
36 – 28 = y2 – y2 + 16y – 12y
8 = 4y 2005/2a Exercise 11.18
8
/4 = y i.e y = 2 The 6th term of a GP is 1215. If the common ratio is 3,
GP: 4, 8, 16, … find its 3rd term
167
2012/4 f/maths Exercise 11.19 Divide both sides by ar
Given that 6 , 3 2 , 3 6 , 9 2 , … are the first ar 4
= 8
four terms of an exponential sequence (GP). Find in its ar
simplest form, the 8th term. r3 = 8
Next, we raise RHS to power 3
A 27 2 B 27 6 C 81 2 D 81 6 r3 = 23
Thus, r = 2 ( D )
2014/20 UTME Exercise 11.20 2005/5 Neco
What is the common ratio of the G.P The 4th term of a GP is 384 and the third term is 96.
( ) (
10 + 5 + 10 + 2 5 + …? ) Find the first term.
A2 B4 C6 D 24 E 288
A3 B5 C 2 D 5 Solution
T4 = 384, T3 = 96, a = ?, r is not given
Applying Tn = ar n – 1
T4  384 = ar 4 – 1
384 = ar3 ------- ( 1 )
Also T3  96 = ar 3 – 1
96 = ar2 ------- ( 2 )
Unknown ‘a’ and ‘ r ’ leading to equation Divide (1) by 2
There are cases where two different terms of a GP that ar 3 384
=
are not close to each other will be given and we are ar 2 96
asked to find the first term a and common ratio r. r = 4
Substitute r value into (2)
2005/8b NABTEB 96 = a(4)2 [42 = 16]
The 2nd and 4th terms of a G.P are 10 and 40 96
respectively. Find the = a Thus 6 = a (C)
16
( i ) Common ratio ( ii ) first term
( iii ) 8th term of the series 2010/13a
Solution The third term of a Geometric Progression (G.P) is 24 and its
20
( i ) T2 = 10, T4 = 40, r = ? a is not given seventh term is 4 27 . Find its first term.
Applying Tn = ar n – 1 Solution
T2  10 = ar 2 – 1 20
T3 = 24, T7 = 4 27 , a = ? r is not given
10 = ar ----- (1)
T4  40 = ar 4–1 Applying Tn = ar n – 1
40 = ar 3 ----- (2) T3  24 = ar 3 – 1
Divide (2) by (1) 24 = ar2 ------- ( 1 )
128
40 ar 3 T7  = ar 7 – 1
= 27
10 ar
128
4 = r2 = ar 6 ------- ( 2 )
27
Raise LHS to a number in power 2
(2)  (1)
22 = r 2
Thus 2 = r ar 6 128
2
=  24
( ii ) Substitute r value into (1) ar 27
ar = 10 becomes 128 1
r4 = 
a(2) = 10 27 24
a = 10/2 i.e 5 16
r4 =
(iii) T8 = 5  28 – 1 27  3
= 5  27 Raise the RHS to number in power four
4
= 640 24 2
r4 = i.e  
2011/6 Neco 34 3
The fifth term of a GP is 8 times the 2nd term. Thus, r = 2/3
Find its common ratio Next, we substitute r value into (1)
A–4 B – 2 C 1/2 D2 E4 ar2 = 24 becomes
Solution a( 2/3 )2 = 24
T5 = 8(T2), r = ?, a is not given 4
Applying Tn = ar n – 1 a = 24
9
T5 = 8(T2) becomes 24  9
ar5 – 1 = 8(ar2 – 1) a = = 54
4
ar4 = 8ar
168
2011/19 f/maths Exercise 11.21 2005/10 NABTEB
The fourth term of a geometric sequence is 2 and In a geometric series a = 2 and r = 1/2,
sixth term is 8.Find the common ratio. find the sum of the first 5 terms
A 1 B 2 C 3 D 4 A 1/ 8 B 3 3/4 C 3 7/8 D4
Solution
2013/12 f/maths Exercise 11.22 S5 = ? Since r = 1/2 < 1
The fourth term of an exponential sequence is 192
a(1 − r n )
and its ninth term is 6. Find the common ratio of the Hence Sn =
sequence 1− r
A 1/ 3 B 1/ 2 C 2 D 3 
2 1−

( 12 )5 
S5 = 
2007/19 PCE Exercise 11.23 1 − 1
2
The 5th term of a G.P is 9 times the 3rd term. What is 1 1
the positive value of the common ratio? = 2 [1 – 32 ] 2
A5 B4 C3 D2 31 2
= 2[ ] 1
32
31
= /8 i.e 3 7/8 (C)

2012/7b Neco
10
/3, 5/3, 5/6 , … is a G.P. Find the
( i ) 8th term ( ii ) sum of the first 10 terms?
Solution
( i ) T8 = ? Applying Tn = ar n – 1
Sum of n terms of a GP. 10 5 10 5 3 1
The sum, Sn of n terms of a GP is : a= , r =  i.e  =
3 3 3 3 10 2
Sn = a + ar +ar2 +. .. + ar n – 2 + ar n –1 -----(1) 8 −1
rSn = ar + ar2 + ar3 +… +ar n – 1 + arn -----(2) 10  1 
T8 =  
Subtract (2) from (1) 3 2
On the RHS, there will be elimination of all the terms 10  1 
7

except a & arn =  


3 2
Sn – rSn = a - arn
Sn (1 – r) = a (1 – rn) 10 1
= 
Sn = a (1 – rn) 3 128
1 – r - (formula one) =
5
i.e
5
This formula holds when r  1 3  64 192
Also, subtracting (1) from (2) ( ii ) S10 = ? First we check the r i.e 1/2 , it is less than 1
rSn – Sn =arn – a a(1 − r n )
Sn (r – 1) = a (rn – 1) Thus, Sn =
1− r
Sn = a (rn – 1)
r–1 - (formula two)
10 
3 

1 − 12 ( )10 
This formula is used when r 1. S10 =
1 − 12
Candidates should note that the usage of either formula
10  1  1
one or formula two depends only on the common ratio = 1 −  
(r) of the Geometric progression given. 3  1024  2
10 1023  2
2006/35 Neco =   
3 1024  1
Find the sum of the first 4 terms of a G.P, whose first
term is 5 and the common ratio is 1/2 10  1023  1705
=   i.e
A 75/32 B 75/16 C 75/8 D 150/8 E 35/2 3  512  256
Solution 2008/12b
First, we check the common ratio 1/2 is less than 1 ( i ) The sum of the second and third terms of a geometric
a(1 − r n ) progression is six times the fourth term. Find the two
Hence Sn = possible values of the common ratio
1− r
( ii ) If the second term is 8 and the common ratio is
51 −

(12 )4  positive, find the first six terms.
S4 =  Solution
1 − 1
2
(i) T2 + T3 = 6  T4 Applying Tn = ar n – 1
= 5 [1 – /16]  1/2
1
ar 2 – 1 + ar 3 – 1 = 6  ar 4 – 1
15 2
= 5 [ 16 ]  1 ar + ar2 = 6ar3
Rearranging
= 5 [ 15/8 ] i.e 75/8 (C) 6ar3 – ar2 – ar = 0
169
Factor out common term 243  19  3
ar(6r2 – r – 1) = 0 =   
4  27  1
ar = 0 or 6r2 – r – 1 = 0
27 19 513
Solving the resultant quadratic equation for r = = i.e 1281/4
6r2 – r – 1 = 0 becomes 4 4
6r – 3r + 2r – 1 = 0
2

3r(2r – 1) + 1(2r – 1) = 0 2014/3 Neco Exercise 11.24


(2r – 1)(3r + 1) = 0 In a G.P, the 5th term exceeds the 4th term by 24 and the 4th
2r – 1 = 0 or 3r + 1 = 0 term exceeds the 3rd by 8, find the:
2r = 1 or 3r = –1 ( i ) Common ratio ( ii ) first term,
–1 (iii) Sum of the first five terms of the G.P
r = 1/2 or /3
( ii ) T2 = 8, r is positive i.e /2 here instead of – 1 /3
1
2015/1b Neco Exercise 11.25
we are asked to list a, ar, ar2, ar3, ar4, ar5, … 2
T2 = 8 The 5th term of a geometric progression ( G.P) is 81 .
ar 2 – 1 = 8 If the first term is 2, Find the : ( i ) common ratio
ar = 8 ( ii ) sum of the first five terms of the G.P
But r is 1/2
2014/6b ( Nov ) Exercise 11.26
a ( 12 ) = 8
The sum of the first third terms of a Geometric Progression
a = 8  2 i.e a = 16 ( GP) is 40 while the fourth and the sixth terms are in the
GP are 16, 16  1/2, 16  1/4, 16 1/8, 16 1/16, … ratio 1 : 4.
16, 8, 4, 2, 1, 1/2 , … Find the : ( i ) common ratio ( ii ) fifth term
2007/7a NABTEB
Find the sum of the first three terms of the GP
whose third term is 27 and whose 6th term is 8
Solution
S3 = ? T3 = 27 and T6 = 8
We know that to find Sn, we need a and r.
These we can get from T3 and T6 that are given
Applying Tn = ar n – 1
T3  27 = ar 3 – 1
27 = ar2 ------ (1)
T6  8 = ar 6 – 1 Sum To Infinity (S)
8 = ar 5 ------- (2) A GP whose common ratio is between – 1 and + 1 say
To get r, we divide (2) by (1) – 1/2, 1/2, -1/4, 1/4 e.t.c. has a sum which approaches a finite
ar 5 8 value as n approaches infinity. It is given as
2
=
ar 27
r3 = 8/27 S = a
We raise the RHS to numbers in power 3 1–r
3 E.g. The sum to infinity of the GP 4, 2, 1, … is ?
23 2
r3 = i.e   Solution
33 3 S = a
Thus, r = 2/3 1–r
Substitute r value into (2) a = 4, r = 1/2
ar5 = 8 becomes S = 4
a( 2/3)5 = 8 1 – 1/2
32 = 4  1/ 2
a ( 243 ) = 8
= 8.
32a = 8  243
8  243 243 2008/9 Neco (Nov)
a = i.e
32 4 The sum to infinity of a G.P is 80. If the first term of the
S3 = ? Since r is 2/3 < 1 series is 20, find the second term.
a(1 − r n ) A 15 B 111/4 C5 D 1 1/ 4 E¾
S3 = Solution
1− r S = 80, a = 20, T2 = ? We can get T2 from Tn = ar n – 1,
S3 =
243 
4 
1− (23 )3   2
 1 − 
3
First, we get r from S = 80
 S =
a
243  8 1 1− r
= 1 −  20
4  27  3 S  80 =
1− r
170
80(1 – r) = 20 2005/25
80 – 80r = 20 A sequence is given by 21/2, 5, 71/2,…
80 – 20 = 80r If the nth term is 25, find n
60 = 80r A9 B 10 C 12 D 15
60 3 Solution
= r i.e r =
80 4 The sequence is 2.5, 5, 7.5, …
2 −1 Test for AP: has a 2.5 and d as 2.5 (i.e 5 – 2.5 or 7.5 – 5)
Thus, T2 = 20 ×  
3
Tn = a + (n – 1)d
4
25 = 2.5 + (n – 1)2.5
3 25 = 2.5 + 2.5n – 2.5
= 20 ×
4 25 = 2.5n
= 15 (A) 25 2.5n
=
2.5 2.5
2007/26 UME Exercise 11.27
Find the sum to infinity of the series 10 = n (B)
2010/30
2 + 32 + 89 + 32
27
+… The nth term of the sequence: –2, 4, –8, 16, …is given by
A1 B2 C8 D4 A Tn = 2 n B Tn = (– 2) n C Tn = (– 2n) D Tn = n2
2006/13 f/maths Exercise 11.28 Solution
Find the sum of the exponential series 96 + 24 + 6 +… Sequence –2, 4, –8, 16, …
A 144 B 128 C 72 D 64 Test for AP: a is – 2, and d = 4 – (– 2) or – 8 – 4
= 6 or – 12 impossible
2009/18 UME Exercise 11.29 4 −8 16
Find the sum to infinity, of the sequence Test for GP: a is – 2, r = or or
−2 4 −8
1, 9
10
, (109 )2 , (109 )3 , … = – 2 or – 2 or – 2 true
A.10, B. 9 C. 10/9 D. 9/10 Applying Tn = ar n – 1
2012/20 UTME Exercise 11.30 = –2(– 2) n – 1
1 − 2(−2) n
The sum to infinity of a geometric progression is – =
10 −2
1 = (– 2)n (B)
and the first term is – . find the common ratio of the 2012/5
8
The nth term of a sequence is Tn = 5 + (n – 1)2,
progression
Evaluate T4 – T6
A – 1/ 5 B – 1/4 C – 1/ 3 D – 1/2
A 30 B 16 C – 16 D – 30
Solution
Tn = 5 + (n – 1)2
Problems on Sequence T4 = 5 + (4 – 1)2
The Author deliberately left problems on sequence for = 5 + 32
the later part of this chapter since the term ‘sequence’ = 5 + 9 i.e 14
could mean AP, GP or any other particular pattern of Also T6 = 5 + (6 – 1)2
numbers. Once a sequence or a progression is mentioned = 5 + 52
without specifying whether it is an AP or a GP, or any = 5 + 25 i.e 30
other patterned numbers, it is the candidates Thus T4 – T6 = 14 – 30
responsibility to identify the type of sequence = –16 ( C)
( progression ) and deal with it accordingly. 2005/6 NABTEB (Nov)
What is the 13th term of the series – 4 + 1 + 6 + 11 + …
2013/5 A 51 B 56 C 61 D 66
If Un = n(n2 + 1), evaluate U5 – U4 Solution
A 18 B 56 C 62 D 80 Series: – 4 + 1 + 6 + 11 + …
Solution A.P Test : a is – 4 and d = 1 – (– 4) or 6 – 1 or 11 – 6
Un = n(n 2 + 1) = 5 or 5 or 5 true
U5 = 5(52 + 1) T13 = ? Applying Tn = a + (n – 1)d
= 5(25 + 1) T13 = – 4 + (13 – 1)5
= 5(26) i.e 130 = – 4 + (12)5
Also = – 4 + 60
U4 = 4(42 + 1) = 56 ( B )
= 4(16 + 1) 2008/11 NABTEB (Nov)
= 4(17) i.e 68 Given the sequence 1, 4, 9, 16, …
Thus U5 – U4 = 130 – 68 The (n + 1)th term of the sequence is
= 62 (C) A 25 B n2 C (n + 1) 2 D n2 + 1
171
Solution 12
= {2 18 + (12 − 1)( −7)}
A.P Test: a is 1 and d = 4 – 1 or 9 – 4 or 16 – 9 2
= 3 or 5 or 7 not true = 6{36 + (11)( –7 ) }
4 9
GP Test: a is 1 and r = or or
16 = 6(36 – 77)
1 4 4 = 6(– 41)
= 4 or 9/4 or 4 not true = – 246 (E)
But from the AP test a pattern is already generated, 2014/8 Neco (Nov)
Difference between the number increases by simple Find the sum of the first 28 terms of the sequence
addition of 2 8, x, 18, 23, …
1 4 9 16, … A 2002 B 2114 C 4004 D 4228 E 8008
3 5 7 Solution
This will not help us Assuming the sequence to be A.P
A second look shows 22 is 4 a = 8, d = 23 – 18 i.e 5
32 is 9 To confirm 23 is T4 and T4 = 8 + (4 – 1)5
42 is 16 = 8 + (3)5
Thus 12 is 1 = 23
The nth term is n2 18 is T3 and T3 = 8 + (3 – 1)5
and (n + 1)th term is (n + 1)2 C. = 8 + (2)5 = 18
We conveniently say the sequence is an A.P
2012/3 Neco Thus x = 8 + 5
The nth term of a sequence is 23n – 2, which term of the = 13
sequence is 213? n
and S 28 = ? Applying S n = {2a + (n − 1)d }
A 6th B 5th C 4th D 3rd E 2nd 2
Solution 28
S28 = {2  8 + (28 − 1)5}
Tn = 23n – 2 2
213 = 23n – 2 = 14{16 + (27)5 }
Since both sides are in the same base, we equate powers = 14(16 + 135)
13 = 3n – 2 = 14(151)
13 + 2 = 3n = 2114 (B)
15 = 3n 2008/8 Neco (Nov)
15 3n Find the 32nd term of the sequence 482, 480, 478, 476,…
=
3 3 A – 420 B 420 C 450 D 514 E 544
5 = n (B) Solution
Sequence is : 482, 480, 478, 476,…
2014/2 Neco AP Test: a is 482 and d = 480 – 482 or 478 – 480
The nth term of a sequence is given by (–1) n – 2 • 2 n – 1 = – 2 or – 2 true
Find the sum of the second and the third terms T32 = ? Applying Tn = a + (n – 1)d
A–2 B1 C2 D6 E 12 T32 = 482 + (32 – 1)( –2)
Solution = 482 + (31)(– 2)
T2 + T3 = ? From Tn = (–1)n – 1 • 2 n – 1 = 482 – 62
T2 = (–1)2 – 2 • 2 2 – 1 = 420 (B)
= (–1)0 • 2 1
=12 2009/15 Neco (Nov)
=2 The nth term of a sequence is given by (3)2n – 1. Write down
Also T3 = (–1) 3–2
•2 3–1 the first three terms of the sequence given that n  0
= (–1)1 • 2 2 A 3/2, 3, 6 B ¾, 3/2, 3 C 3/2, 3, 1/3
3 3 3
=–14 D / 2, / 4, 3 E /4, 0, 6
= –4 Solution
Thus T2 + T3 = 2 + (– 4) Tn = (3)2n – 1
= 2–4 For n  0, first three terms are n = 0, n = 1, n = 2
= –2 (A) n=0
2011/5 Neco ( 3 )20 – 1 = ( 3 )2 – 1
Find the sum of the first 12 terms of the sequence 3
=
18, 11, 4, –3, … 2
A 678 B – 38 C – 41 D – 241 E – 246 n=1
Solution ( 3 )21 – 1 = ( 3 )20 i.e 3  1
AP Test: a is 18 and d = 11 – 18 or 4 – 11 = 3
= – 7 or – 7 true n=2
S12 = ? Applying Sn =
n
{2a + (n − 1)d } ( 3 )22 – 1 = ( 3 )21
2 = 6 Thus, sequence is : 3/2, 3, 6 ( A )
172
2014/17 NABTEB (Nov) CHAPTER TWELVE
Find the 28th term of the sequence 3, 8, 13, 18, 23,… Variation
A 114 B 133 C 135 D138 Variation is the term that refers to the relationship between
Solution two or more quantities. A change in one of the quantities
A.P test: a is 3 and d = 8 – 3 or 13 – 8 or 18 – 13 results in a change in the other (s). The symbol  is used to
= 5 or 5 or 5 indicate variation.
T28 = ? Applying Tn = a + (n – 1)d We shall consider four types of variations namely:
T28 = 3 + (28 – 1)5 Direct variation
= 3 + (27)5 Inverse variation
= 3 + 135 Joint variation
= 138 (D) Partial variation
2014/9 NABTEB
Find the sum of the first 90 positive integers Direct variation
A90 B 2045 C 4095 D 5050
Solution y  x
Pattern of positive integers: 1, 2, 3, 4, 5, … reads: y varies directly as x or y is directly proportional to x.
A.P Test: a is 1 and d = 2 – 1 or 3 – 2 or 4 – 3 To introduce equality sign, we must introduce a constant of
= 1 or 1 or 1 true variation i.e
n y = kx
S90 = ? Applying Sn = {2a + (n − 1)d } Where k is the constant of variation or constant of
2
90 y
S90 = {2  1 + (90 − 1)1} proportionality and k =
2 x
= 45{2 + (89)1} 2006/ 15 Neco (Nov)
= 45(2 + 89) If x  y, when x = 5, y = 15, what is y when x = 1.5?
= 45(91) = 4095 (C) A 50 B 45 C 5 D 4.5 E 2.25
2007/8b NABTEB Solution
Simplify without using mathematical tables the sum of x  y
the first 20 terms of the series 3 + 6 + 9 + 12 +… Introducing a constant k to enable us replace  by =
Solution x = ky ----------- **
A.P Test: a is 3 and d = 6 – 3 or 9 – 6 or 12 – 9 x
= 3 or 3 or 3 true = k
y
n
S 20 = ? Applying Sn = {2a + (n − 1)d } When x = 5, y = 15 then
2
20 5
S20 = {2  3 + (20 − 1) 3} = k
2 15
1
= 10{6 + (19)3} = k
= 10(6 + 57) 3
= 10(63) = 630 Substitute k value into ** to get the relation
1
2006/1Neco (Nov) Exercise 11.31 x = y
The nth term of a series is given by the 3
formula 2a + 2 n –1. If a = 6 Next, when x = 1.5, y = ?
( i ) what is the value of the 9th term? 1.5 =
y
( ii ) find the number of terms if the last term is: 3
(a) 76, (b) 524 1.5  3 = y
4.5 = y (D)
2015/5 Exercise 11.32
Find the 7th term of the sequence : 2, 5, 10, 17, 26,…
2006/ 43 Neco
A 37 B 48 C 50 D 63
The time of oscillation of a pendulum varies as the square
2014/6 ( Nov ) Exercise 11.33 root of its length. If the length of the pendulum which
The nth term of the sequence 5, 8, 11, … is 383. Find n oscillates for 35 seconds is 49cm, find the time of oscillation
A 125 B 126 C 127 D 194 of a pendulum with length 121cm
A 77sec B 55sec C 18sec D 11sec E 7 sec
2015/3 Neco Exercise 11.34
Solution
The nth term of the sequence –1, 5, –7, 17,… is given
by T  L
A 2n B –1 + (–2) n C –1 – 2 n Introducing a constant K to enable in replace  by =
D 1 + (–2) n
E1–2 n
T = K L --------**
T
2005/28 Exercise 11.35 = K
L
What is the nth term of the sequence 3, 8, 13…?
When T = 35s, L = 49cm then
A 5n + 3 B 5n – 2 C 5n – 3 D 5n – 5

173
35 35 25
= K i.e = K R = L
49 7 12
5 = K Next when R = 40 ohms, L = ?
Substitute K value into ** to get the relation 25
40 = L
12
T = 5 L 40  12 = 25L
Next, when L = 121cm, T = ? 40 12
T = 5 121 =L i.e L = 19.2m
25
= 5  11 2006/ 18 – 19 Counter example
= 55 sec ( B )
The table below satisfies the relation y = k x
Where k is a positive constant. Use it to answer questions 18
2011/ 33
– 19
G varies directly as the square of H. If G is 4 when H is
3, find H when G = 100 x 1 4 p
A 15 B 25 C 75 D 225 y 0.5 1 2.5
Solution 18. Find the value of k
G  H2 A 0.5 B1 C 1.5 D2
Introducing a constant k to enable us replace  by = Solution
G = k H2 ---------- ** We can pick any suitable set of values of x and y from the
G
= k table, say x = 4 and y = 1
H2 y = k x becomes
When G = 4, H = 3
4 1= k 4
= k 1 = 2k
32 1
/2 = k i.e k = 0.5 ( A )
4
= k 19. Find the value of p
9
A 2 B 4 C 10 D 25
Substitute k value into ** to get the relation Solution
4H 2 The relation now is
G =
9
y = 1/ 2 x
When G = 100, H = ?
When y = 2.5, x i.e p = ?
4H 2
100 = 2.5 = 12 x
9
100  9 = 4H2 2.5  2 = x
100 9
= H2 5 = x
4 To remove square root from x, we take the square of both sides
25  9 = H2 52 = x i.e 25 = x ( D )
To get H under square we take the square root of both 2006/40(Nov)
sides Q varies directly as the cube of P. when P = 1, Q = 3. Find P
25 9 = H when Q = 24
53 = H A 1/ 3 B 1/ 2 C2 D3
15 = H ( A ) Solution
Q  P3
2014/11a NABTEB (Nov) Introducing a constant k to enable us replace  by =
The electrical resistance, R ohm, of a piece of wire is Q = k P3 --------- **
proportional to its length, Lm. A piece of wire of length Q
= k
12m has resistance 25 ohms P3
( i ) Write the equation connecting R and L When P = 1, Q = 3 then
( ii ) Find the length of wire for a resistance of 40 ohms 3
= k i.e k = 3
Solution 13
RL Substitute k value into ** to get the relation
Introducing a constant k to enable us replace  by = Q = 3P3
R = KL -------- ** Next, when Q = 24, P = ?
Thus
R
= K 24 = 3P3
L 24
When L = 12m, R = 25ohms then = P3
3
25
= K 8 = P3
12 To get P under cube, we take the cube root of both sides
Substitute K value into ** to get the relation 3
8 = P i.e P = 2 (C)

174
2014/7 Neco (Nov) 2013/24 Neco
If P varies directly as the cube root of Q and P = 3 If a is directly proportional to b and a = –1 while b = –4,
and Q = 125, find Q when P = 18/5 the formula connecting a and b is
A 169 B 216 C 343 D 450 E 512 A a = 14 b B a = 12 b C a=b
Solution D a = 2b E a = 4b
P  3Q Solution
Introducing a constant k to enable us replace  by = a  b
P = K 3 Q ---------- ** Introducing a constant k to enable us replace  by =
a = kb ---------- **
P
= K a
= k
3 Q b
When P = 3, Q = 125 then When a = – 1, b = – 4 then
3 −1 1
= K = k i.e = k
3
125 −4 4
3
Substitute k value into ** to get the relation
= K or connecting formula
5
Substitute K value into ** to get the relation a = 14 b ( A )
33Q
P = 1975/5
5
18 y varies directly as an unknown power of x. Express y as a
Next, when P = ,Q=? function of x, given that when x = 2, y = 0.4 and when x = 4,
5
y = 6.4 ( Take the unknown power of x to be n )
18 33Q
= Solution
5 5 y  xn
18
5 = 3 3Q Introducing the constant of variation
5
y = kxn ------------**
18 5
 = 3Q When x = 2, y = 0.4 into **
5 3
1 0.4 = k 2n
6 = 3 Q i.e 6 = Q3 0 .4
k= ………….( 1 )
To remove cube root from Q, we take cube of both sides 2n
63 = Q When x = 4, y = 6.4 into **
216 = Q (B ) 6.4 = k 4n
6 .4
k= ………….( 2 )
2007/3b NABTEB 4n
If S is directly proportional to T and T = 120, when S = From (1) and (2) we equate k values
30, find the value of T when S = 136 0 .4 6 .4
Solution k =
2n 4n
S  T
4n × 0.4 = 2n × 6.4
Introducing a constant k to enable us replace  by =
S = kT ----------- ** 4n 6 .4
=
S 2 n 0 .4
= k
T 2n
2
T = 120, when S = 30 then = 16 ( Note 22n – n = 2n )
30 2n
= k 2n = 16
120
1 2n = 24
= k
4 n=4
Substitute k value into ** to get the relation Next, we substitute n = 4 into (1)
1 0 .4 0 .4 1
S = T k = = i.e
4 2 4 16 40
Next, when S = 136, T = ? The relation from y = kxn is
T
136 = 1 4
4 y = x
40
136  4 = T
544 = T
2012/7a Exercise 12.1
The electrical resistance R ohms of a wire varies directly as
the length Lcm. If R = 0.6 ohms when
L = 7.5cm, find R when L = 35.5cm
2008/23 Neco (Nov) Exercise 12.2
175
P varies directly as the cube root of Q. When P = 20, Substituting for the first set of y and x values
Q = 125. Find the value of P when Q = 27 k = 3  (4)2
A 12 B 15 C 24 D 25 E 30 = 48
2005/24 Neco Exercise 12.3 Substituting k value into ** to get the relation
48
If A varies directly as the square root of B and A = 35, The relation is y =
when B = 25. Find B when A = 14 x2
A 14 B7 C6 D5 E4 When x = 2, find y
48
2014/7 Neco ( Nov ) Exercise 12.4 y =
If p varies directly as the cube root of q and p = 3 when 22
48
q = 125, find q when p = 18/5 = i.e 12 (E)
A 169 B 216 C 343 D 450 E 512 4
2008/10
2015/40 Exercise 12.5 T varies inversely as the square root of F, when
The rate of consumption of petrol by a vehicle varies T = 7, F = 21/4. Find T when F = 2 7/9
directly as the square of the distance covered. If 4 litres A 1.5 B 6.3 C 10.5 D 12.6 E 21
of petrol is consumed on a distance of 15 km, how far Solution
would the vehicle go on 9 litres of petrol? 1
A 221/2 km B 30km C 333/4 km D 45km T 
F
Introducing a constant of variation k to replace  by =
Inverse variation k
T = -----------**
If y varies inversely as x, then mathematically, we write F
1 When T = 7, F = 21/4. then
y  k
x 7 =
9
Introducing a constant of variation, 4
k
y = 9
x k = 7
4
2009/5 (Nov)
= 7  3/2
If y is inversely proportional to x and x = 3 when y = 4,
find y when x = 2 = 21/2
A1 B3 C6 D9 Substitute k value into ** to get the relation
21
1 T=
y  2 F
x
When F = 27/9, T is
Introducing a constant k to enable us replace  by =
21 21
k T= i.e
y = ------------** 7 25
x 2 2 2
9 9
k = xy
When x = 3, y = 4 then 21
=
5
k = 34 2
= 12 3
Substitute k value into ** to get the relation = 21  10/3
12 = 21  3/10
y = = 63/10
x
When x = 2, y = ? = 6.3 (B)
12
y =
2
y = 6 (C) 2006/10b Neco
2014/ 26 Neco The weight, w, of a body varies inversely as the square of its
y varies inversely as x2. If y = 3 when x = 4, distance, d, from the centre of the earth. A body weighs 80
find y when x = 2 Newtons on the earth surface, if the radius of the earth is
A8 B9 C 10 D 11 E 12 6400km,
Solution i. Obtain an expression connecting the weight of a body
1 and its distance from the centre of the earth;
y  ii. Calculate the distance of a body of 20 Newtons from
x2
the centre of the earth;
Introducing a constant of variation k
iii. Calculate the weight of a body which is 1,600 km
k
y= -----------** above the surface of the earth.
x2
Thus yx2 = k Solution
176
1 = 2(5)
(i) w  2 = 10
d
Substitute k value into ** to get the relation
Introducing a constant of variation k to replace  by =
10
k The relation is (y + 2) =
w = -----------** x
d2 When x = 5, then find y
When w = 80N, d = 6400km then 10
(y + 2) =
k 5
80 =
6400 2 y+2 = 2
k = 80  (6400) 2 y = 2–2
y = 0 (A)
= 3.2768  109
2007/24 Neco
Substitute k value into ** to get the relation
n varies inversely as the square root of m. If n = 5
3.2768  10 9 when m = 9. State the relationship between n and m
w =
d2 k
A n = 15m B m = 15n C m=
( ii ) w = 20N, find d 15n
Substituting into the relation 405 15
9 Dn= E n=
3.2768  10 m m
20 = 2
d Solution
3.2768  10 9 1
d2 = n 
20 m
d2 = 163840000 Introducing a constant k to enable us replace  by =
d = 163840000 = 12800km k
n = ---------- **
( iii ) From the centre of the earth taken as reference m
Here d = (1600 + 6400) = 8000km , k = n m
w=? When n = 5, m = 9 then
k k= 5 9
Substituting into the relation w =
d2 =53
3.2768  109 = 15
w = Substitute k value into ** to get the relation
(8000) 2
15
3276800000 n = (E)
= = 51.2N m
64000000 2006/13 Neco
2012/ 43 Y varies inversely as the cube root of x.
If x and y are variables and k is a constant, which of the If Y = 4 when x = 0.125, find y when x = 8
following describe an inverse relationship between x A5 B4 C3 D2 E1
and y? Solution
1
A y = kx B y=
k
Cy=k x Dy=x+k Y 
3
x x
Solution Introducing a constant k to enable us replace  by =
1 k
y  Y = ---------- **
3
x x
Introducing a constant k to enable us replace  by = k = y3x
y =
k
(B) When y = 4, x = 0.125
x k = 4 3 0.125
2009/26 Neco (Dec) =2
If (y + 2) varies inversely as x and y = 3 when x = 2. Substitute k value into ** to get the relation
Find y when x = 5. 2
A0 B2 C4 D7 E 11 Y =
3
x
Solution Next, when x = 8, y = ?
1
(y + 2)  2
x Y =
3
Introducing a constant of variation k 8
2
k = i.e 1 (E)
(y + 2) = -----------** 2
x
x(y + 2) = k 2005/48 Exercise 12.6
When y = 3, x = 2 then
k = 2(3 + 2)
177
1 y
If y  and y = 1¼ when x = 4, find the x 
x2 z
value of y when x = 1/2 Introducing a constant of variation
A 2 1/ 2 B5 C 10 D 80 x = ky ----------**
z
2014/43 Neco (Nov) Exercise 12.7 When x = 5, y = 2 and z = 1 then
The resistance of an electric wire varies inversely as the k2
square of the potential difference. When the resistance 5 =
1
is 0.7Ω , the potential difference is 0.4v. Find the 5 = 2k
potential difference when the resistance is 44.8 Ω 5
= k
A 0.05V B 0.13V C 0.50V D0.52V E 5.18V 2
Substitute k value into ** to get the relation
2005/36 Exercise 12.8
5y
If p varies inversely as the square of q and p = 8 x =
2z
when q = 2, find p when q = 4
When y = 5, z = 2 x = ?
A2 B4 C8 D 16
55
2013/28 Neco Exercise 12.9 x=
2 2
Given that y is inversely proportional to x and y = 20 25
= i.e 6.25 (C)
when x = 1/4 , find x when y = 30 4
A 1/ 6 B 1/ 5 C 1/4 D 1/ 3 E 1/2
2012/15 Neco
Joint variation The cost of material for making a drum varies as the cube of
the radius and inversely as the surface area. When the cost is
Joint variation involves more than two variables. In N150.00, the area is 2250cm2 and the radius is 15cm. Find
most cases it is a combination of direct and inverse
the cost of the materials when the radius is 18cm and the
variation
area is 2700cm2
Example A N270.00 B N216.00 C N196.00
If p varies directly as the square of q and inversely as r D N180.00 E N168.00
q2 Solution
Then p 
r r3
Introducing a constant of variation C 
A
kq 2 Introducing a constant of variation
p =
r kr 3
2007/23 Neco C = ---------**
A
r varies directly as the cube of p and inversely as the When C = N150.00, A = 2250cm2, r = 15cm then
square of q. If k is a constant which of the following is
k  15 3
correct? 150 =
2250
kr
A q2 = kp3r B q2 = C q3 = kp2r 150  2250
p3 = k
15  15  15
kp 3 kp 2 100 = k
D q2 = E q3 =
r r Substitute k value into ** to get the relation
Solution
100r 3
p 3 C =
r  A
q2 When r = 18cm, Area = 2700cm2 C=?
Introducing a constant of variation 100  18 3
3 C =
r =
kp 2700
q2 18  18  18
=
From the options, we are to make q2 the subject of formula 27
q2r = kp3 = N216.00 ( B )
kp 3
Thus, q2 = ( D)
r
2008/22 Neco (Nov)
2008/11 Neco The energy E possessed by a moving object varies directly
x varies directly as y and inversely as z. when x = 5, as its mass m and the square of its velocity v. when m = 8kg,
y = 2 and z = 1. What is the value of x when y = 5 v = 5ms – 1 and E = 100 joules. Find E when m = 6kg and
and z = 2 ? v = 2ms – 1
A 2.5 B 5.0 C 6.25 D 6.52 E 7.5 A 6J B 12J C 24J D 48J E 96J
Solution Solution

178
E  mv2 b 1
3=
Introducing a constant of variation 3
E = kmv2 ----------** 33 = b
When m = 8kg, v = 5ms – 1 and E = 100 joules then 9= b (D)
100 = k × 8 × 52 2008/6b NABTEB (Nov)
100 B
= k A and A = 20 when B = 5 and C = 1/3
8  25 C2
1 Find the ( i ) relation between A, B and C
= k
2 ( ii ) value of A when B = 4 and C = 1/6
Substitute k value into ** to get the relation Solution
1 B
E= mv2 A  2
2 C
When m = 6kg, v = 2ms– 1 , E = ? Introducing a constant of variation
6 22 kB
E = A = 2 ------------**
2 C
= 12j (B) If A = 20, B = 5, C = 1/3 then
k5
2010/28 Neco 20 =
Q ( 13 )
2

F , when Q = 32, T = 4 and F = 20. 20  1/9 = 5k


T2
Find F when Q = 49 and T = 7 20  1/9  1/5 = k
4
A7 B 10 C 14 D 49 E 160 /9 = k
Substitute k value into ** to get the relation
Solution
4B
Q A=
F  9C 2
T2
( ii ) When B = 4, C = 1/6, A = ?
Introducing a constant of variation
4 4
kQ A=
F =
T2
---------** 9 (16 )2
When Q = 32, T = 4 and F = 20 then = 4  4  1/4
k  32 k  32 = 444
20 = i.e
4 2 4 4 = 64
20 = k  2 2005/ 12b Neco
20 The energy (E) varies directly as the resistance (R) and
= k i.e k = 10 inversely as the square of the distance (D). If E = 32/25,
2
Substitute k value into ** to get the relation when R = 16 and D = 10, calculate the value of R when
E = 32 and D = 7
10Q
F = Solution
T2 R
When Q = 49, T = 7 F = ? E 
D2
10  49 10  49 Introducing a constant of variation
F= i.e
72 49 KR
E = ---------- **
F = 10 (B) D2
When E = 32/25, R = 16 and D = 10, then
2012/19 Neco 32 k 16
=
Given that x  by when x = 2, y = 3 and b = 2, 25 10 2
what is the value of b when x = 3 and y = 1? 32
A 1/ 3 B3 C6 D9 E 12  10 2 = 16k
25
Solution
32 1
x  by  100  = k
Introducing a constant of variation 25 16
x = kby ----------** 8=k
When x = 2, y = 3 and b = 2 then Substitute k value into ** to get the relation
8R
2 = k 2 3 E =
2 D2
= k When E = 32, D = 7, R =?
23
1 8 R
/3 = k 32 =
Substitute k value into ** to get the relation 72
x = 1/3 by
32  72 = 8 R
When x = 3, y = 1, b = ?
179
32 49 If x  y2z when y = 5 and z = 6, x = 45.
= R
8 Find x when y = 3 and z = 7
196 = R A 189 B 181 C 18.9 D 1.89
2014/7 (Nov) 2008/6b NABTEB (Nov) Exercise 12.14
A quantity z varies directly as the square root of x and B 1
inversely as the cube of s. If z = 8 when x = 4 and s = A 2
and A = 20 when B = 5 and C =
C 3
1
/2, express z in terms of x and s. Find the ( i ) relation between A, B and C
2 x x 2S 3 x 1
A Z= 3
B Z= 3
C Z= D Z= 3
( ii ) value of A when B = 4 and C =
S S x 2S 6
Solution 2005/36 Neco Exercise 12.15
The mass of a spherical ball varies as the cube of its radius
x
Z  and inversely as its surface area, when the radius is 4cm, its
3
s area is 16cm2 and the mass is 20kg /cm3. Find its mass when
Introducing a constant of variation the area is 54cm2 and the radius is 3cm
k x A 20kg/cm3 B 18kg/cm3 C 62/3 kg/cm3
Z = ------------** D 51/3kg/cm3 E 21/2 kg/cm3
s3
When z = 8, x = 4 and s = 1/2, then 1976/8b Exercise 12.16
k 4 The electrical conductivity in a wire of circular cross-section
8 = varies directly as the square of the radius and inversely as the
( 12 )3
length. One wire is twice as long as the other but the radius
8 = 4 k  (12 )3 of the cross-section of the shorter is 32 times that of the
8 = 2k  1/8 longer. Find the ratio of their conductivity
8 = 2k  8/1
8 = 16k 1976/6 ( Nov ) Exercise 12.17
8/ = k i.e
16
1/ = k
2
Given that the energy E varies directly as the resistance R
Substitute k value into ** to get the relation and inversely as the square of the distance d, get an equation
x connecting E, R and d. If E = 3225
when R = 8 and d = 5,
Z = 3 (D)
2S calculate:
2014/16 ( a ) the value of R when E = 16 and d = 3
If x varies inversely as y and y varies directly as z, what is ( b ) the value of d when R = 5 and E = 56
the relationship between x and z? ( c ) the percentage increase in R when E and d each
1 1 increases by 3%
A xz B x  z C x  z2 D x  2
z
Solution Partial variation
When a variation is expressed as a sum of terms, we refer to
1 that type of variation as partial. For instance,
x  y and y  z
y = k + ax
By simple reasoning, the second statement simply means Reads: “y is partly constant and partly varies as x”. The
take y as z letter ‘a’ is a constant of variation. If we introduce same to K
Thus the relationship between x and y is the same as say b, then we will have ‘ bk’ but constant multiply by a
between x and z constant is also a constant. Thus, we do not apply another
1 constant to a constant as we do to variables.
... x  z
The statement “t varies partly as r and partly as inverse
of s” is represented as
2011/13a Exercise 12.10
1
P varies directly as Q and inversely as the square of R. t = ar + b
If P = 1 when Q = 8 and R = 2, find the value of Q s
when P = 3 and R = 5. 2013/7b Neco
The cost of sewing a garment is partly constant and partly
2008/ 6b Exercise 12.11
varies with the amount of time it takes to sew it. If it takes 6
Q2 hours to sew the garment, the cost is N2800.00 and if it
P, Q and R are related in such a way that P  .
R takes 10 hours to sew it, the cost is N3600.00. Find the cost
When P = 36, Q = 3 and R = 4, Calculate Q when if it takes 4 hours to sew the garment.
P = 200 and R = 2 Solution
2005/6c Exercise 12.12 c = k + t
x, y and z are related such that x varies directly as the Introducing constant b to variable t
cube of y and inversely as the square of z. if x = 108 c = k + bt
when y = 3 and z = 4, find z when x = 4000 and y = 10 When t = 6hours, c = N2800.00
2008/7 NABTEB (Nov) Exercise 12.13 2800 = k + 6b ------ (1)
When t = 10hours, c = N3600.00
180
3600 = k + 10b ----- (2) 10 x 2 52
Solving equations (1) and (2) The relationship is y= + (E)
31 31 x
3600 = k + 10b
– ( 2800 = k + 6b )
2008/1a Neco
800 = 4b A varies partly as B and partly as C. When A = 4, B = 2 and
800
= b i.e b = 200 C = – 2 and when A = 3, B = 3 and C = 1.5.
4 What is the value of C when A = 0.5 and B = 2.7?
Substitute b value into (2) Solution
3600 = k + 10b will become A = B + C
3600 = k + 10(200) Introducing constant k and d
3600 = k + 2000 A = kB + dC
3600 – 2000 = k When A = 4, B = 2 and C = – 2
1600 = k 4 = k(2) + d(–2)
The relationship is c = 1600 + 200t 4 = 2k – 2d ------ (1)
Next, when t = 4hrs, c = ? Next, when A = 3, B = 3 and C = 1.5
c = 1600 + 200(4) 3 = 3k + 1.5d ------- (2)
= 1600 + 800
(1)  3 and (2)  2 then subtract
= N2400.00
12 = 6k – 6d
2014/ 56 Neco
– ( 6 = 6k + 3d )
y varies partly as the square of x and partly as the
inverse of the square root of x. Write down the 6= – 9d
6
expression for y if – /9 = d i.e d = – 2/3
y = 2 when x = 1and y = 6 when x = 4 Substitute d value into (1)
1 1 4 = 2k – 2d will become
A y = x2 + B y = x2 + 4 = 2k – 2(– 2/3)
x x
4 = 2k + 4/3
2
x 1 10  2 1  4 – /3 = 2k
4
C y= + D y= x + 
31 
31   8
/3 = 2k
31 x x
Multiply through by 1/2
10 x 2 52
E y= + 8
/ 3  1/ 2 = k
31 31 x 4
/3 = k
Solution 4 2
1 The relationship is A = B – C
y = x2 + 3 3
x When A = 0.5, B = 2.7, C = ?
Introducing constants a and b Substituting into the relation, we have
1 4 2
y = ax2 + b 0.5 = × 2.7 – C
x 3 3
If y = 2, when x = 1 10.8 2c
b 0.5 = −
2 = a(12) + 3 3
1 10.8 − 2c
2 = a + b ------ (1) 0.5 =
3
y = 6 when x = 4 0.5  3 = 10.8 – 2c
b
6 = a(42) + 1.5 = 10.8 – 2c
4 2c = 10.8 – 1.5
6 = 16a +
b 2c = 9.3
2 c = 9. 3/2 = 4.65
Multiply through by 2 to clear fraction 2007/25 Neco
12 = 32a + b ------- (2) z is partly constant and partly varies as y, when y = 1, z = 2
Solving equations (1) and (2) and when y = 2, z = 1. State the equation connecting y and z
12 = 32a + b Az=2–y B z = 3 – 2y C z=3–y
–( 2= a + b ) Dz=2+y E z = 2 + 3y
10 = 31a Solution
a = 10/31 z = k+y
Substitute a value into (1) Introducing constant b to the variable y
2 = a + b will become z = k + by
10 When y = 1, z = 2, then
2= +b 2 = k + b -------- (1)
31
10 52 Next, when y = 2, z = 1
2– = b thus b = 1 = k + 2b -------- (2)
31 31
181
Solving equations (1) and (2) 1
y= k +
2 = k+b x3
– ( 1 = k + 2b ) Introducing constant to the variable x
1 = –b b
Divide both sides by –1 y = k +
x3
1 −b ( i ) When x = 1, y = 5,
=
−1 −1 b
–1 = b 5= k +
13
Substitute b value into (1)
5 = k + b -------- (1)
2 = k + b will become
Next, when x = 2, y = – 1
2= k–1
b
2 + 1 = k i.e k = 3 –1 = k +
The relationship (connecting equation) is 23
z=3–y (C) b
–1 = k +
2011/11a Neco 8
The resistance R to the motion of a car is partly constant Multiply through by 8 to clear fraction
and partly varies as the square of the speed V. At – 8 = 8k + b ----------(2)
80kmhr – 1, the resistance is 1060N and at 120kmhr – 1, Solving equations (1) and (2)
its resistance is 1460N. What will be the resistance at 5= k+b
140km hr – 1 ? – ( – 8 = 8k + b )
Solution 13 = – 7k
R = k + V2 −13
= k
Introducing constant to the variable V 7
R = k + bV2 Substitute k value into (1)
When V = 80km hr – 1, R = 1060N, 5 = k + b will become
1060 = k + b(802) −13
5= +b
1060 = k + 6400b ------- (1) 7
Next, when V = 120km hr – 1, R = 1460N 5 +
13
=b
1460 = k + b(1202) 7
1460 = k + 14400b ------- (2) 48
= b
Solving equations (1) and (2) 7
1460 = k + 14400b −13 48
The relationship is y= +
– ( 1060 = k + 6400b ) 7 7x3
400 = 8000b When x = 3, y = ?
400 1 −13 48
= b i.e b = y = +
8000 20 7 7(3 3 )
Substitute b value into (1) −13 48
1060 = k + 6400b will become = +
7 189
1
1060 = k + 6400  −351 + 48 −303
20 = = i.e – 1.603
189 189
1060 = k + 320
2005/37 Neco Counter example
1060 – 320 = k
x varies partly as y and partly as the square of y. When y = 2,
740 = k
x = 5 and when y = 5, x = 57.5, find x when y = 4
V2 A 57.5 B 50.0 C 40.0 D 34.0 E 25.0
The relationship is R = 740 +
20 Solution
Next, when V = 140kmhr – 1, R = ? x = y + y2
(140) 2 Introducing constant to the variable y
R = 740 +
20 x = ay + by2
140  140 When y = 2, x = 5
= 740 +
20 5 = 2a + 4b ------- (1)
= 740 + 980 = 1720 N When y = 5, x = 57.5
2006/4 Neco (Nov) 57.5 = 5a + 25b
The quantity y is partly constant and partly varies Multiply through by 2 to clear decimal
inversely as the cube of x 115 = 10a + 50b -------- (2)
( i ) Given that x = 1, y = 5 and x = 2, y = – 1, Solving the equations: (1)  5 and subtract
find the relationship between x and y 115 = 10a + 50b
( ii ) Hence, find the value of y when x = 3 – ( 25 = 10a + 20b )
Solution 90 = 30b

182
90 30b
= 1975/7b ( Nov ) Exercise 12.24
30 30 The cost of feeding a number of students during a certain
3 = b period is partly constant and partly varies directly as the
Substitute b value into (1) number of students. The cost of feeding 75 students during a
5 = 2a + 4b will become certain period is Le875.00 and the cost of feeding 100
5 = 2a + 4(3) students during the same period of time is Le1000.00. Find
5 = 2a + 12 the cost of feeding 220 students over the same period of time
5 – 12 = 2a
1977/7b Exercise 12.25
– 7 = 2a
The resistance R to the motion of a car is partly constant and
– 7/2 = a
partly proportional to the square of the speed v. When the
−7
The relationship is x = y + 3y2 speed is 30km/h, the resistance is 190 newtons and When the
2 speed is 50km/h, the resistance is 350 newtons . Find for
When y = 4, x = ? what speed the resistance is 302.5 newtons
−7
x =  4 + 3 4 2
2 CHAPTER THIRTEEN
= – 14 + 48 Trigonometry
= 34 ( D ) Trigonometry is simply the study of triangles in relation to
their sides and angles.
2008/20 NABTEB (Nov) Exercise 12.18 For any right angled triangle of the form below.
If a and b are constant and P varies partly as Q and B
partly as the reciprocal of the square root of Q, which of
these statements is correct ?
b b a c
A P=a + B P=a +
Q Q2
O
C P = aQ +
b
D P = aQ +
b
C A
Q Q 2 Fig I b
Where  is facing, is opposite (BC)
2008/31 Exercise 12.19
If x  (45 + 12 y ) which of the following is true?
Where right angle is facing, is the Hypotenuse (AB)
While the remaining side is the Adjacent (AC)
A x varies directly as y Also, where the capital letter A is facing is the small letter a;
B x varies inversely as y same for B and C
C x is partly constant and partly varies as y Diagrammatically, we have
D x varies jointly as 45 and directly as y Fig II
2009/4 Neco (Dec) Exercise 12.20
Hy

y is partly constant and partly varies as the square of x.


Opposite

p
ot

when y = 16, x = 2 and when y = 24, x = 3. Find :


he
nu

( i ) The relationship between x and y


se

( ii ) y when x = 4 O
2006/44 Neco Exercise 12.21 Adjacent
y is partly constant and partly varies as x. The three basic trig. ratios are Sine, Cosine and Tangent
when x = 1, y = 7 and when x = 2, y = 11. written in short form Sin, Cos and Tan respectively.
Find the equation connecting x and y We will apply SOH CAH TOA in Fig I above to define
A y = 3x + 2 B y = 2x – 3 C y = 4x + 3 them as :
D y = 2x + 4 E y = 3x + 4 Applying SOH in defining Sin 
2015/9a Neco Exercise 12.22 Here S – sine, O – opposite, H – hypotenuse
W is partly constant and partly varies inversely as the Sin  = Opposite = a
square of t. When W = 24, t = 4 and When W = 18, t = 2 Hypotenuse c
( i ) Determine the law connecting W and t Applying CAH in defining Cos 
( ii ) Find t when W = – 46 Here C – cosine, A – adjacent, H – hypotenuse
2015/6b Exercise 12.23
The cost ( c ) of producing n bricks is the sum of a fixed Cos  = Adjacent = b
amount, h and variable amount y, where y varies Hypotenuse c
directly as n. If it costs GH₡ 950.00 to produce 600
Similarly we apply TOA to define Tan 
bricks and GH₡ 1,030.00 to produce 1000 bricks,
Here T – tangent , O – opposite, A – adjacent,
( i ) find the relationship between c, h and n;
( ii ) calculate the cost of producing 500 bricks Tan  = Opposite = a
Adjacent b
183
The three basic trig ratios have reciprocals namely
Cosec (cosecant) = 1 600 0
30 30
0
Sine 2 2 2 2
Sec (secant) = 1
Cosine 600 60
0

600 600
Cot (cotangent) = 1
Tangent Fig I 2 1 1 FigII
In practice, we adopt one part of Fig II i.e.
Deductively, we have that
Cosec  = 1 = Hypo = c 0
30
Sine  Opp a
2 3
Sec  = 1 = Hypo = c
Cos Adj b 600
1 Fig III
and Cot  = 1 = Adj = b Applying SOH CAH TOA to fig III
Tan Opp a Sin 300 = 1/2

Special angles trigonometric ratios Cos 300 = 3


2
Angle of 450
To achieve the desired goal here; candidates must allow Tan 300 = 1 = 3
for the mathematical assumption that follows. A right –
3 3 by rationalization.
angled triangle whose  is 450 with opposite and
Similarly,
adjacent sides of unit 1. Then by Pythagoras theorem
the hypotenuse is √2 Sin 600 = 3 Cos 600 = 1
2 2
2
1 Tan 600 = 3 = 3
1
450
1 Eg 1 Find the value of the unknown sides in the diagrams
below:
Applying SOH CAH TOA i ii
4 y
Sin450 = Opp = 1
x
Hyp 2 rationalizing, we have 0
45 45 0
= 2 8
2 iv
Cos 450 = Adj = 1 iii z 9

Hyp 2 rationalizing, we have 3


60
0 300
= 2 k
2 Solution
0
Tan 45 = Opp = 1 = 1 ( i ) x – Opp; 4 – Hypo (OH  SOH)
Adj 1
Sin 450 = Opp = x
0
Angles of 30 and 60 0 Hypo 4
Here the mathematical assumption is that of an Substituting for Sin 450 from special trig. ratio
equilateral triangle of sides 2 unit. Then on bisecting 1 = x
the triangle from the top angle to the base. We have 300 2 4
each at the top and 1 unit each at the base of the
resulting two right – angled triangles. 4 = x Rationalizing, 4 x √2 = x
Then by Pythagoras theorem the bisector (height) is √3 2 √2 √2
represented below by Fig. 1 and Fig. II 4√2 = x i.e x = 2√2
2
184
E.g. 3 Find the value of x in the diagram below:
(ii) 8 – Adj; y – Hypo (AH  CAH)
Cos 450 = Adj = 8
Hyp y
8
450 X
0
60
Substituting for Cos450 from special trig. ratio Solution
1 = 8 Candidates can only find x if side of the sub-triangle of 450 is
√2 y known. The other triangle of 600 can be used if and only if
we find the value of the side opposite 600 using the available data.
y = 8√2
In the right - angled triangle 600
Let the Opp be y ; Hyp – 8 ( OH  SOH ) Thus,
(iii) 3 – opp; z – Hypo (OH  SOH)
Sin 600 = y
Sin 600 = opp = 3 8
Hypo z Substituting for Sin 600 from special trig. ratio
Substituting for Sin 600 from special trig. ratio
√3 = y
√3 = 3
2 8
2 z y = 8√3 = 4√3
z √3 = 6
2
z=6 rationalising , we have = 6 x √3 Then in right – angled Δ 450
√3 √3 x √3 y = 4√3 – Adj; x – Hypo ( AH  CAH ) Thus,
= 6√3 = 2√3
Cos 450 = 4√3
3
x
Substituting for Cos450 from special trig. ratio
(iv) 9 – Adj; k – Hypo (AH  CAH)
1 = 4√3
Cos 300 = Ad j = 9 √2 x
Hypo k
Substituting for Cos300 from special trig. ratio x = 4√3 x √2 = 4√6
√3 = 9
2 k Special angles trig ratios substitution
k√3 = 18 Problems
k = 18 Rationalising, k = 18√3 = 6√3 2009/ 17 NABTEB (Nov)
√3 3 Determine, without the use of table, tan 600
1 2 2
A B C 3 D
E.g 2. Without using tables evaluate 3 3 3
Solution
1 + tan 600 tan 300 By special trig ratios
tan 600 + tan 300 tan 600 = 3 (C)
Solution 2012/46 Neco
Substituting for the various special trig. ratio
1 + cos 30 0
= 1 + √3 (1/√3) = 2 Express in surd form
1 − sin 30 0
√3 + 1/√3 √3 + 1/√3
A 3 −2 B 2+ 3 C 2 2+ 3
We simplify as in ordinary fractions:
= 2 = 2 = 2 = 2√3 = √3 D 2 3+ 2 E 2 3+2 2
√3 x √3 + 1 3+1 4 4 2 Solution
Substituting for the values of the special trig ratios
√3 √3 √3
 
1 + cos 30 0
= 1 + 3   1 − 1 
1 − sin 30 0  2   2 

 2+ 3  1
=    
 2
 2 
2+ 3 2
=  = 2+ 3 (B)
2 1

185
2008/49 Neco (Dec) 4 6 + 2 12
=
1 − sin 45 0 8
Evaluate without using table
1 + cos 30 0 4 6 + 2 2 3
=
A 4−2 3 −4 2 − 6 B 4+2 3 −4 2 + 6 8
4 6 +4 3 1
C 4+ 2 3 −3 2 + 6 D 4−2 3 −2 2 − 6 = = ( 6 + 3)
8 2
E 4+ 2 3 −5 2 + 6 2009 /25
Solution 3 1
Substituting for the value of the special trig ratio Given that sin 600 = and cos 600 =
2 2
1 − sin 45 0  1   3  1 − sin 60 0
= 1 −   1 + Evaluate:
1 + cos 30 0  2   2  1 + cos 60 0
 2 −1  2 + 3  2+ 3 1− 3 1+ 3 2− 3
=  
 

 A B C D
 2   2  3 3 3 3
Changing from division to multiplication Solution
 2 −1  2  1 − sin 60 0  3   1 
=    = 1 −  1+ 
 
 
 2   2 + 3  1 + cos 60 0  2   2

2 ( 2 − 1)  2 − 3   2 +1
= =  
 
2 (2 + 3)  2   2 
2 2 −2 2− 3  3
= =   

2 2+ 6  2  2
Rationalizing by conjugate of the denominator surd 2− 3 2
(2 2 − 2) (2 2 − 6) =  

=  2  3
2 2 + 6 (2 2 − 6)
2− 3
2 2 (2 2 − 2) − 6 (2 2 − 2) = (D)
= 3
2 2 (2 2 + 6 ) − 6 (2 2 + 6 )
2005/4b
4  2 − 4 2 − 2 12 + 2 6 Given that x = cos300 and y = sin300, evaluate without using
=
4  2 + 2 12 − 2 12 − 6 a mathematical table or calculator
x2 + y2
Note that 12 = 4  3 i.e 2 3
y2 − x2
8 − 4 2 − 2 2 3 + 2 6
= Solution
8−6
x2 + y2 (cos 30 0 ) 2 + (sin 30 0 ) 2
8−4 2 −4 3 + 2 6 =
= y2 − x2 (sin 30 0 ) 2 − (cos 30 0 ) 2
2
= 4−2 2 −2 3 + 6 Substituting for the values of the special trig ratios
2
2005/1b  3   1 2
  +
Without using mathematical tables or calculator, evaluate,  2   2 
leaving your answer in surds =   2
2
 1   3 
sin 45 0 + cos 60 0    −
 2   2 
tan 30 0
=  +    − 
3 1 1 3
Solution
Substituting for the values of the special trig ratios 4 4 4 4
sin 45 0 + cos 60 0  1 3 +1 1− 3
1 1 = 
0
=  +   4 4
tan 30  2 2 3
4 −2
2+ 2  =   
=   3 4  4 

 2 2  1 −1
= 1   
3 (2 + 2 )  2 
=
−2
2 2 = 1   
2 3+ 6  1 
= = –2
2 2
2013/45 Neco Exercise 13.1
(2 3 + 6) 2 2 What is the value of sin450 sin300 – cos450 cos300 leaving
Rationalizing : = 
2 2 2 2 the answer in surd form?
186
3− 2 2 −1 2+ 6 Y
A B C
4 2 4
2− 6 6− 2 +x
D E x
4 4 3600 -
Fig iv
2010/33 NABTEB (Nov) Exercise 13.2
The value of sin 450 + cos 450 in surd form without
-y

1u
rationalizing is

nit
1 2
A 2 B C2 D
2 2
2014/54 Neco ( Nov ) Exercise 13.3 The x and y axes divide the plane into four quadrants as
If x = 300 and y = 600 , evaluate without using tables shown in Fig. I to Fig.IV. Any angle measured positively in
sin x + cos x an anticlockwise direction will be located in any of these
or calculator quadrants.
sin y + cos y
A3 B 2 C1 D –1 E–2 Fig. I (1st quadrant between 00
2015/10a Exercise 13.4 and 900)
Without using Mathematical tables or calculators, The resulting right – angled triangle has both x and y
simplify: positive, the hypotenuse that is 1 unit plays little or no role.
2 tan 60 0 + cos 30 0 Here Sin , Cos  and Tan  are all positive.
sin 60 0 Fig. II (2nd Quadrant between 900 and 1800).
2007/49 UME Exercise 13.5 The right – angled triangle here has negative x, which is the
tan 60 0 − tan 30 0 adjacent side and a positive y - which is the opposite side.
Find the value of Hence, only
tan 60 0 + tan 30 0
Sin  = Sin (1800 - )
4 2 1 = +y = + y is positive
A B C1 D
3 3 2 1
Others are negative in 2nd Quadrant i.e Cos  and Tan 
Trigonometric ratios of angles Fig III (3rd Quadrant between 1800 and 2700).
between 00 and 3600 The right - angled triangle formed here has both x and y
These type of trig ratios does not carry the condition negative.
“acute angle” or 00    900. Apart from our Only Tan  = Tan ( - 1800)
knowledge of special angles of 450, 300 and 600 ; we are =-y
also required to know the required quadrant we are -x
dealing with as discussed below: = y is positive
Diagrams of trigonometric ratio of angles x
between 00 and 3600.
Others are negative 3rd quadrant i.e Cos  and Sin 
Y Fig. IV (4th Quadrant between 2700 & 3600)
Our right – angled triangle here has negative y which is the
opposite side but a positive x which is the adjacent side.
1u
nit

+y
nit

Hence only
1u

+y
Cos  = Cos (360 - )
180- = +x
x x 1
+x -x = x is positive
1 quadrant
st
2 quadrant
nd
Others are negative in 4th quadrant i.e Tan  and Sin 
Y Summarily
We use “Acts” to recall the mentioned properties of
-x x trig.ratio between  0 and 3600 where
- 180 A is for all positive
-y Fig III C – Cosine only positive
nit T – Tangent only positive
1u S – Sine only positive
3rd quadrant Written
S A
nd st
2 Quad 1 Quad

T C
3rd Quad 4th Quad
187
1993/39 UME 1994/3 (Nov) fm
If sin = cos , find  between 00 and 3600 1
A. 450, 2250 B. 1250 , 3150 If sin = – and 00    2700 find 
0
C. 45 , 225 0
D. 1350, 2250
2
Solution A 300 B 600 C 1200 D 1500 E 2100
Solution
Sin  = cos 
Sin = – 0 .5
Then, divide both sides by cos 
 = sin –1(–0.5)
Sin  = cos 
But sin–1 0.5 is 300 and sin is negative in
Cos  cos 
3rd and 4th quadrants but our restriction here is 3rd
Tan  = 1 = 1 and tan  is positive
with formula  – 1800
1
Thus,  – 1800 = 300
Since we have 1 , then it must be 450 special trig. ratios
1  = 30 + 180
= 2100 (E)
If tan  = 1/1 then  = 450 Thus 450 in the first
1995/7 (Nov) fm
quadrant
Find the values of x, for which 2cos x + 1 = 0, 0 0  x  3600
Also tan is positive in the 3rd now with formula  – 1800,
A 00, 1800 B 900, 2700 C – 600 , 600
then 0 0 0 0
D 60 , 300 E 120 , 240
 – 1800 = 45
Solution
 = 45 + 1800 2cos x + 1 = 0
= 2250 2cos x = – 1
 = 450, 2250 ( A ) 1
Cos x = –
2002/34 PCE 2
The tangent of 1350 has the value x = cos–1(– 0.5)
A. – 1 B. – 1/2 C. 1/2 D. 1 But cos–1(0.5) is 600 and that cosine is negative
Solution in the 2nd and 3rd quadrants with formula
Tan 1350; tan is negative in the second quadrant where 180 –  and  – 180
135 falls into. The formula here is 180 –  Thus, x = 180 – 60 or  – 180 = 60
Thus, Tan 1350 = tan (180 – 1350) = 1200 or 2400 (E)
= – tan 450 2005/18 fm
2
From special angles 450 1 If sinӨ = 1 , 00 Ө  4500 , find the possible values of Ө
2
tan 450 = 1 i.e 1
1
45 A 300 , 2100, 3900 B 600 , 3300, 3900
0 0 0
Hence tan 1350 = – tan 450
1 C 60 , 210 , 450 D 300, 1500, 3900
= –1 ( A ) Solution
Sin Ө = 1/2
1990/37 PCE Ө = sin–1( 0.5)
Evaluate cos 600 + sin 300 = 300 or ( 180 – 300 ) Sine is +ve in 2nd quad.
Sin 1500 = 300 or 1500
A. 2 1
B. /2 C. – 2 D. – 1/2 = 300 or 1500 or 30 + 3600 i.e Plus complete cycle
Solution = 300 or 1500 or 3900 (D)
Ordinarily, on seeing cos 600 and sin 300 the author 2012 Neco
would have treated this question under acute angles trig ratios
Without using tables, find the value of sin 1200
but for the sin 1500 which is in a quadrant beyond 900.
S A Sin 1500 is the 2nd quadrant where sine is positive 3 3 1
A B C D 3 E1
T C with the general formula 180 –  . Thus
0 4 2 2
Sin 1500 = + sin ( 1800 – 1500 ) Solution
= sin 300 Sin 1200 is in the 2nd quadrant with the formula 180 –  i.e
We can now conveniently say; 180 – 120 = 600 and sine is positive there.
cos 600 + sin 300 = cos 600 + sin 300 Thus sin1200 = sin 600
Sin 150 sin 300 By special trig ratio
=1 + 1 =
3
(B)
2 2 2
1 2014/34 NABTEB Exercise 13.6
2 Which of these is equivalent to tan 3000?
=1  1 =1×2 A tan 600 B – tan 600 C sin 600 D – cos 600
2 1
1993/43 PCE Exercise 13.7
=2(A)
Evaluate tan 450 + cos 600
Sin 1500
A. – 3 B. 3 C. – ( 2 + 1 ) D 2 + 1
188
2005/15 UME Exercise 13.8 With formula 3600 – 
1
x = 3600 – 600
If sin  = – for 00< < 3600 , the value of  is = 3000 ( A )
2
A 2100 and 3300 B 1500 and 3300 2005/45 Neco (Dec) Exercise 13.11
C 1500 and 2100 D 300 and 1500 If cos 300 = 0.866, find the value of the angles
whose cosine is – 0.866
2012/35 UTME Exercise 13.9
A 1200 and 3300 B 1200 and 2100 C 1500 and 2100
If angle  is 1350, evaluate cos  0
D 150 and 330 0
E 2100 and 3300
1 2 2 1
A B – C D – 1978/44 fm Exercise 13.12
2 2 2 2
Evaluate without using tables sin (– 12900 )
2004/34 PCE Exercise 13.10 A. – 3 B. 3 C. 2 D. 1 E. – ½
The value of tan 3150 is 2 2 2
A -1 B0 C1 D2
Trigonometry of Negative angles ( i.e –  ) Identities
They are treated by adding 3600 to it or its multiples as
Complementary angles
the case demands. We use the resulting angle to identify
If  is an acute angle, then the angles  and 900 –  are
the quadrant it belongs.
complementary which appear together in the right angled
1994/38 UME triangle below;
What is the value of sin (– 6900 ) B
A. 3 B. – 3 C. – 1 D. 1
2 2 2 2
90-
Solution c
– 6900 + 3600 = –3300 a
So sin (–6900 ) = sin (– 3300 )
and 3300 falls into the 4th quadrant where sin is negative C A
with the general formula 3600 –  . Thus b
sin (– 6900 ) = sin (– 3300 ) 1 sin  = a/c = cos (90 - )
= – sin ( 3600 – 3300 ) cos  = b/c = sin (90 - )
= – sin 300 From special trig ratios tan  = a/b = cot (90 - )
=–1 (C)
2 2. Tan  = a/b = sin 
2001/35 PCE cos 
What is the value of cos (– 8400 )
Reason sin  =a b
A. – 3 B. – 1 C. 1 D. 3
Cos  c c
2 2 2 2
Solution = a xc = a
– 8400 + 7200 = – 1200 ( 720 is 2 × 3600) c b b
0 0
So cos (– 840 ) = cos (– 120 ) 3. sin2 + cos2 = 1
and 120 falls into 2nd quadrant where cosine is negative Proof
with quadrant formula180 – . thus By Pythagoras rule
Cos (– 8400 ) = cos (– 1200 ) a 2 + b2 = c 2
= – cos ( 1800 – 1200 ) Divide through by c2
= – cos 600 a 2 + b2 = c 2
=–1 (B) c2 c2 c2
2
2012/5 f/maths But a/c = sin  hence a2/ c2 = sin2 
3 Similarly b2/c2 = cos2  but c2/c2 = 1
Given that sin x = – and cos x > 0, find the value Then substituting
2
of x sin2  + cos2  = 1 or
A 3000 B 2400 C 1200 D 600 cos2  + sin2  = 1
Solution Since addition is commutative i.e. 2 + 3 = 3 + 2
3
sin x = – From identity 3 we can derive others
2 i.e. cos2  + sin2  = 1
3 Divide through by cos2 
x = – sin–1
2
x = – 600 Cos2  + sin2  = 1
Sine is negative in the 3rd and 4th quadrants Cos2  Cos2  Cos2 
But the condition cos x > 0 restricts us to 4th quadrant 4. 1 + tan2 = Sec2 
189
Similarly dividing identity 3 by sin2  Problems on identities I
Cos2  + Sin2  = 1
1999/1b (Nov) fm
Sin2  Sin2  Sin2 
Simplify 1 – Cos2
5. cot  + 1 = cosec2 
2
Sec2 – 1
Solution
Complementary angles problems By trig identities
2002/19 (Nov) fm
Given that sin(900 – 5) = cos (1800 –  ) find the value of . 1 – Cos2 = Sin2
0 0 0 0
A 15 B 22.5 C 30 D 45 Sec2 – 1 tan2
Solution
Sin (90–5) = cos (180 – ) = Sin
Under complementary angle tan
Sin(90 – 5 ) = Sin[ 90 – ( 180 –  ) = Sin ÷ Sin
 90 –5 = 90 – 180 +  Cos
90 – 5 =  – 90 = Sin × Cos
90 + 90 =  + 5 Sin
180 = 6 Thus,  = 300 (C) = Cos 
2005/3a Neco fm 2003/20 fm
If sin(x – ) = cos(x + ), prove that tan x = 1 Given that sin  = a – b what is 1 – cos2
Solution a+b
Under complementary angle A a–b B a–b C a+b D a–b 2

sin(x – ) = cos(x + ), implies a+b a+b a–b a+b


sin (x – ) = sin [ 90 – ( x + ) ] E a+b 2
Thus x –  = 90 – x –  a–b
x + x –  +  = 90 Solution
2x = 90 By trig identities Sin2  + cos2  = 1
x = 450 Sin2  = 1 – cos2
Hence tan x  tan 450 = 1/1 = 1 QED Taking the square root of both sides
1981/6 UME Sin  = 1 – cos2
If sin x0 = a. , what is sin ( 90 – x )0 ? Thus , a–b = 1 – cos2 (A)
b a+b
A. a B. 1 – a C. b2 – a2 1982/20 UME
b b b
If sin  = m – n , find the value of 1 + tan2 .
D. a 2 – b2 E. b2 – a2 m+n
b A. 2 ( m2 + n2 ) B. 2 (m2 + n2)
Solution m+n m+n
Recall that, under complementary angles C. m2 + n2 + 2mn D. m 2 + n2 + 2mn E. ( m + n )2
Cos  = sin ( 90 –  ). So that question is indirectly 2mn 4mn 4mn
asking us to find cos x.
First, let us complete the trig ratio triangle. b Solution
Applying SOH a Recall that 1 + tan2  = sec2 
Sin x0 = a implies x 0 and sec  = 1
b Cos 
Then we are to find sec2  but first, lets complete our trig
By Pythagoras rule the adjacent is equal to b2 – a2
ratio triangle.
Hence sin ( 90 – x )0  cos x0 = b2 – a2 ( C )
Sin  = m – n implies
b m+ n
m+n m-n
2010/9 fm Exercise 13.13
If sin x = – sin 700 , 00< x < 3600, determine the two Applying SOH
possible values of x By Pythagoras rule, the adjacent side
A 1100, 2500 B 1100, 2900 C 2000, 2500 D 2500, 2900
= (m + n )2 – (m – n)2
2000/7a fm Exercise 13.14
Such terms are not in our options so we simplify further .
Solve the equation Cos7 = Sin 5 Where 0   900
= [m + n – ( m – n ) ] [ m + n + ( m – n)]
2007/46 Neco Exercise 13.15
If sin (x + 200) = cos (x – 100), find the value of x = [m + n – m + n ] [ m + n + m – n]
A 900 B 600 C 450 D 400 E 300
= ( 2n ) ( 2 m )
190
= 4mn = 4-3
i.e Adj = 2 mn
= 1 i.e 1
Hence sec  = ( sec )
2 2
So, tan = 1 and sin  = 1
2 2
= Hypo = m+n
3 2
Adj 2 mn
Hence tan  = 1  1 2

= ( m + n )2 sin2 3 2
2
(2 mn )
= 1  4
=(m+n)(m+n)
3 1
4 mn
= m2 + n2 + 2mn (D ) = 4 rationalizing 4 3 ( A )
4mn
3 3
1985/34 UME
Without using tables, calculate the value of 1 + sec2 300 2008/2 fm
A. 2 1/3 B. 2 C. 1 1/3 D. ¾ E. 3/2 3 15
Solution If sin A = and cos B = , where A is obtuse and
5 17
Recall that 1 + tan2  = sec2  B is acute, find the value of cos(A + B)
tan2 = sec2 – 1 Solution
– tan  = 1 – sec2
2
cos(A + B) = cosA CosB – sinA sinB
The only way out here is direct substitution To proceed we first find cosA and sinB
sec2 300 = 1 = hyp 2 By trig identities
cos2 300 adj 3
If sin A = it follows that Cos A = 1 − sin 2 A
0
From special angles 30 trig ratio 5
30
2 9 4 4
cos 300 = 3
3 Cos A = 1− =  i. e − ( since A is Obtuse)
25 5 5
2 60
From above deduction 15
1
If cos B = it follows that Sin B = 1− cos 2 B
17
2
sec2 300 = 2 225 8 8
Sin B = 1− =  i. e + ( since B is Acute )
3 289 17 17
=4 cos(A + B) = cosA CosB – sinA sinB
3
Hence 1 + sec2 300 = 1 + 4 = 7 i.e 2 1/3 ( A ) =–4 × 15 – 3 × 8
3 3 5 17 5 17
= – 12 – 24
17 85
1985/38 UME
= – 84
If cos  = 3 and  is less than 900, calculate
85
2
cot ( 90 –  ) 2014/2 fm
sin2  Simplify ( 1– sin)(1 + sin)
A sin2 B sec2 C tan2 D cos2
A. 4 3 B. 4 3 C. 3 D. 1 E. 2 Solution
3 2 3 3 Applying difference of two squares principles
Solution ( 1– sin)(1 + sin) = 12 – (sin)2
Recall that cot ( 90 –  ) = tan  and sin2 = 1 – cos2 = 1 – sin2
But the latter is longer to solve for By trig identities = cos2 (D)
Hence cot ( 90 –  ) = tan 
2005/1b (Nov)
Sin2  sin2 Given that sin(A + B) = sinA cosB + cosA sinB, without
Let’s now complete the trig ratio triangle 2
using mathematical table or calculator evaluate sin1050,
Cos  = 3  applying CAH leaving your answer in surd form [Hint 1050 = 600 + 450]
2 Solution
By Pythagoras rule, the 3 Using the given hint and the quoted formula
opposite = 2 –( 3 )
2 2 sin1050  sin(600 + 450) = sin600 cos450 + cos600 sin450
191
Substituting for the values of the special trig ratios Tan  = opp = BC = a
3 1 1 1 Adj AB (b2 – a2 ) 1/ 2
=  + 
2 2 2 2
3 1
= + 2012/48 Neco
2 2 2 2 1
If tan  = and  is an acute angle, what is cos ?
3 +1 2
=
2 2 2 2 5 5 5
A B C D 2
E 5
( 3 + 1) 2 2 5 5 2
Rationalizing: = Solution
2 2  2 2
We find the unknown side of the resulting
( 3 + 1) 2 2 right – angled triangle by applying Pythagoras rule
=
4 2
2 6 + 2 2 1
= = ( 6 + 2)
8 4 x
1
1990/39 UME Exercise 13.16
If cos  = 12 , find 1 + cot2 
13 2
A. 169 B. 25 C 169 D 144 x2 = 1 2 + 2 2
25 169 144 169 x2 = 1 + 4
2002/31 UME Exercise 13.17 x2 = 5
If tan  = 4 , calculate sin2  – cos2  x= 5
3 2
A. 16 B 24 C. 7 D. 9 Thus cos  =
5
25 25 25 25
2 5
2006/9 UME Exercise 13.18 Rationalizing: = 
5 5
If tan  = 5 , find sin2  – cos2 
4 2 5
= (B)
A. 41 B. 9 C. 1 D. 5 5
9 41 4
1986/32 UME Exercise 13.19
2010/43 NABTEB (Nov)
If cos  = a, find 1 + tan2 
3
b Find the value of sinA + cosA if cosA =
A. b2 B. a2 C. a2 + b2 D. 2a2 + b2 5
a2 b2 b2 – a 2 a 2 + b2 A 7.0 B 5.0 C 1.5 D 1.4
Solution
We find the unknown side of the resulting
DERIVED TRIG RATIOS right – angled triangle by applying Pythagoras rule
Given a trigonometric ratio such as sin  = a/b , where
 is acute; we can obtain the trig ratios for Cos  and
5
Tan  and their reciprocals as follows: a
Draw a right –angled triangle, which contains  as
A
shown below. The side opposite  is a unit and the
hypotenuse (i.e. the side facing the right angle) is b unit 3
C 5 = a + 32
2 2

5 – 32 = a 2
2

25 – 9 = a2
a b 16 = a2
16 = a i.e a = 4
A 4 3
B Thus, sin A + cos A = +
Applying Pythagoras rule, we have 5 5
7
AB2 = AC2 – BC2 = i.e 1.4 (D)
= b2 – a 2 5
2006/8 Neco
AB = (b2 – a2) 1/ 2
If tan  = 3, find the value of sin •Cos (900 – )
Cos  = Adj = AB = (b2 – a2) 1/ 2 9 7 3 3 1
Hypo AC b A B C D E
10 10 5 10 9

192
Solution
We complete the trig- ratio triangle, applying TOA
3 5
tan  = implies 3
1

x
52 = 32 + x2
3 k
5 – 32 = x 2
2

25 – 9 = x2
16 = x2
1
16 = x i.e x = 4
By Pythagoras rule: k2 = 32 + 12 3
Thus, 2tan = 2 
k = 10 4
From complementary angles 3
= i.e 11/2 ( D )
cos(900 – ) = sin 2
sin • cos(900 – ) = sin sin 2008/27
3 3
If tan x = 1, evaluate sin x + cos x, leaving your answer in
=  the surd form
10 10 1
9
A2 2 B 2 C 2 D2
= (A) 2
10 Solution
2011/ 8a We find the unknown side of the resulting
Given that sinx = 0.6 and 00  x  900, right – angled triangle by applying Pythagoras rule
m 1
evaluate 2cosx + 3sinx, leaving your answer in the form Note tan x = 1 implies tan x =
n 1
where m and n are integers
Solution
Though our problem is presented in decimals, we are 1 k
directed to work in fractions
Sin x = 0.6 x
6 3 1
Sin x = i.e sin x =
10 5
k2 = 12 + 12
Next, we find the unknown side of the resulting
k2 = 2
right – angled triangles by applying Pythagoras rule
k= 2
1 1
Thus, sin x + cos x = +
3 5 2 2
2
x =
k
2
2 2
52 = 3 2 + k2 Rationalizing = 
5 – 32 = k2
2 2 2
25 – 9 = k2 2 2
= = 2 (C)
16 = k2 2
16 = k i.e k = 4 2012/3
4 3 1
Thus, 2cosx + 3sinx = 2  + 3 Given that cos x0 = , express tan x0 in terms of r
5 5 r
1
8 9 17 A B r C r2 +1 D r 2 −1
= + = r
5 5 5
Solution
We find the unknown side of the resulting
2008/28 NABTEB (Nov)
3 right – angled triangle by applying Pythagoras rule
If sin  = and  is acute, find the value of 2tan
5
5 4 1 1
A B C1 D1 r
8 5 5 2 a
Solution
We find the unknown side of the resulting right – x0
angled triangle by applying Pythagoras rule 1
r = 1 + a2
2 2

193
r2 – 12 = a2
r 2 – 1 = a2 2014/ 3b (Nov)
5
2
r −1 = a If sin x = , 00  x  900, evaluate, without table or
13
r 2 −1 cos x − 2 sin x
Thus tan x0 = calculator
1 2 tan x
= r 2 −1 (D) Solution
We find the unknown side of the resulting
2012/1b right – angled triangle by applying Pythagoras rule
2
Given that sin x = , evaluate, leaving your answer in
3
surd form and without using table or calculator 5 13
tan x – cos x
Solution x
We find the unknown side of the resulting right –
k
angled triangle by applying Pythagoras rule
2 2 2
13 = 5 + k
132 – 52 = k2
2 3 169 – 25 = k2
144 = k2
x 144 = k i.e k = 12
cos x − 2 sin x
=  − 2     2  
k 12 5 5
Thus,
2 2 2 2 tan x  13 13   12 
3 =2 +k
=  −  
32 – 22 = k2 12 10 5
9 – 4 = k2  13 13  6
5 = k2  12 − 10  6
= 
5 = k  13  5
2 5 2 6 12
Thus, tan x – cos x = − =  =
5 3 13 5 65
6−5
= 2008/48 Neco
3 5
1 1
1 If tan  = and tan  = and both  and  are acute,
= 2 3
3 5
find tan( +)
A 0.015 B 1.00 C 10.00 D 39.81 E 45.00
2014/22 (Nov) Solution
1
If sin x = , 0< x <900, calculate the value of cos x By trigonometric identities:
3 tan  + tan 
1 2 2 2 2 tan ( +) =
A B C D 1 − tan  tan 
8 5 3 3
1 1  1 1
Solution =  +   1 −  
 2 3  2 3
3+ 2  1
=    1 − 
1 3  6   6
5 5
x = 
6 6
k 5 6
= 
3 = 1 + k2
2 2
6 5
32 – 12 = k2 = 1 (B)
9 – 1 = k2
8 = k2 2008/46 Neco (Dec)
8 =k i.e 2 2 =k 3 3
If sin x = and tan x = which of the following is
Adj 2 2 5 4
Thus, cos x = = (D) correct?
Hyp 3
A 00 < x < 900 B 900 < x < 1800
0 0
C 180 < x < 270 C 2700 < x < 3600
E The value of x cannot be determined from the given information
Solution
194
3 We find the unknown side of the resulting right – angled
Using sin x = , let us complete the Pythagorean triangle by applying Pythagoras rule
5
triple, by right – angled triangle
x 29
3 5

x 21
k 292 = 212 + x2
292 – 212 = x2
52 = 3 2 + k2
841 – 441 = x2
5 – 32 = k2
2
400 = x2
25 – 9 = k2
16 = k2 400 = x i.e 20 = x
16 = k i.e k = 4 1 hypothenus e 29
Thus, cosec  = = = (E)
sin  opposite 20
Opp 3
Thus, tan x = = which same as given above
Adj 4
2005/7
We can conclude that 00 < x < 900 If tan  = 1, evaluate cos. (0    900)
1 1 1
A–1 B C – D
2 2 2
2008/44 Neco (Dec)
Solution
a
If tan x = , 0 < x < 900, find 1 + cos 2x We find the unknown side of the resulting
b right – angled triangle by applying Pythagoras rule
a2 − b2 2a 2 + b 2 a 2 + 2b 2 Note that tan  = 1 implies tan  =
1
A B C
2
a +b 2
a +b 2 2 2
a +b 2 1
2 2
a b
D 2 2
E
a +b a + b2
2
1
y
Solution
We find the unknown side of the resulting right –
angled triangle by applying Pythagoras rule 1
y2 = 12 + 12
a y2 = 2
k
Adj 1
y= 2 Thus, cos = = (B)
x Hyp 2
b
2011/44 Neco
8
k2 = a2 + b2 Given that sin  = , 0<<900, find the value of cos 
17
k= a2 + b2 14 15 14 8 8
A B C D E
2 15 17 17 15 17
 
Thus, 1 + cos x = 1 +  
b
2 Solution
 2 2  We find the unknown side of the resulting
 a +b 
right – angled triangle by applying Pythagoras rule
b2
= 1+
a2 + b2
Applying LCM in fraction method 17
8
a2 + b2 + b2
=
a2 + b2
a 2 + 2b 2 y
= (C) 2
17 = 8 + y 2 2
a2 + b2
172 – 82 = y2
2009/50 Neco (Dec) 289 – 64 = y2
21 225 = y2
Given that cos  = , where  is acute. Find cosec 
29 225 = y
20 21 20 29 29 Adj 15
A B C D E 15 = y Thus, cos  = = (B)
29 29 21 21 20 Hyp 17
2013/46 Neco Exercise 13.20
Solution
195
3 Find x if cos x = sin 400
Given that sin  = and 00    900 A 400 B 450 C 500 D 900
5
find tan  + cos  Solution
1 20 3 4 31 cos x = sin 400
A B C D E cos x = 0.64 28
20 31 4 5 20
x = cos – 1 0.6428
2014/9c Neco ( Nov ) Exercise 13.21 = 500 ( C )
5
If tan  = and  is acute, determine cos
12 2006/10
2014/25 Neco ( Nov ) Exercise 13.22 If tan y = 0.404, where y is acute, find cos 2y
3 A 0.035 B 0.719 C 0.808 D 0.927
Given that Cos = and 0    900 , Solution
2
find the value of tan2 tan y = 0.404
A 1/ 3 B 3/4 C3 D4 E5 y = tan – 1 0.404
= 21.999
2014/53 Neco ( Nov ) Exercise 13.23 Since y is acute, we are restricted to the 1st quadrant only
4 cos 2y = cos 2(21.999)
If sin  = and  is an acute angle,
5 = cos 43.998
evaluate 15( tan – cos) = 0.719 ( B )
A 29 B 11 C 114/15 D 14/11 E 11/15
2008/28
2015/23 Exercise 13.24
If cos (x + 25)0 = sin 450, find the value of x
2
Given that tan x = , where 00 x  900 , A 20 B 30 C 45 D 60
3 Solution
find the value of 2sin x Cos (x + 25)0 = sin 450
2 13 3 13 4 13 6 13 1
A B C D Cos (x + 25)0 = from special trig ratios
13 13 13 13 2
1
2015/4a ( Nov ) Exercise 13.25 (x + 25)0 = cos – 1
2
3
If sin x = , where 00 x  900 , without using tables By special trig ratio triangle
1
appears in 450
5
2
cos x + tan x
or calculator, calculate Thus (x + 25)0 = 450
sin x
x0 = 450 – 250
2010/38 UTME Exercise 13.26 = 20 ( A )
8 2012/49 Neco
If cot  = , where  is acute, find sin,
15 What is the value of  if sin 2 = cos 520
A 13/15 B 15/17 C 8/17 D 16/17 A 180 B 190 C 380 D 640 E 720
Solution
sin 2 = cos 520
INVERSE or ARC TRIG. RATIOS sin 2 = 0.6157
These are the inverse of the various trig ratios we 2 = sin – 1 0.6157
already discussed 2 = 380
38
Trig. ratios Inverse =
–1 2
Cos  Cos  or arc cos = 190 ( B )
Sin  Sin–1  or arc sin
Tan  Tan–1  or arc tan
Inverse trig. ratios and equations
2008/46 Neco
2012/24
Find y if sin y = cos 480
If cos (x + 40)0 = 0.0872, what is the value of x
A 210 B 240 C 420 D 480 E 1020
A 850 B 750 C 650 D 450
Solution
Solution
sin y = cos 480
sin y = 0.6691 cos (x + 40)0 = 0.0872
x + 40 = cos – 1 0.0872
y = sin– 1 0.6691
x + 40 = 850
= 420 ( C )
x = 850 – 40
= 450 (D)
2006/31 (Nov)
2008/ 26 NABTEB (Nov)
196
If 5cos – 3 = 0, find the value of  to 1 decimal place, If 00   900 , find the value of  in the equation
00    3600 sin(2 – 4 ) = cos(5 + 3 )
A 126.9, 306.9 B 126.9, 233.1 C 53.1, 233.1 A 120 B 130 C 140 D 300 E 450
D 53.1, 306.9
Solution 2014/11a Neco ( Nov ) Exercise 13.28
5 cos – 3 = 0 2 sin  + x 2
5 cos = 3 Given that cos  = , find the possible values
4x
cos = 3/5 i.e 0.6 of x if  = 45 0

 = cos – 1 0.6
= 53.10 Exercise 13.29
Within the range of 00    3600 cosine is also positive Solve: 2cos x – 1 = 0 for 00 x  3600
in 4th quadrant with formula 360 – 
 = 360 – 53.1 2015/37 fm Exercise 13.30
= 306.90 If sin2 = 1 + cos  , 00   900, find the value of 
Thus  = 53.10, 306.90 ( D ) A 900 B 600 C 450 D 300

2006/46 Neco (Dec)


Solve the equation 3cos + 1 = 0 Mathematical tables trig ratio problems
A 3600 B 2700 C 109.470 D 70.530 E 00 2010/22 (Nov)
Solution Evaluate cos 1160 using tables
3cos + 1 = 0 A 0.5354 B 0.4384 C – 0.3384 D – 0.4384
3cos = –1 Solution
−1 cos 1160 is in the 2nd quadrant with formula 180 –  and
cos =
3 cosine is negative there.
cos = – 0.3333 Thus, cos 1160 = – cos (180 – 116)0
 = – cos – 1 0.3333 = – cos 64
= 70.53 Check up 640 under zero in cosine of natural angles
Cosine is negative in 2nd and 3rd quadrants with formula = – 0.4384 ( D )
180 –  and  – 180 1990/40
180 – 70.53 applies here Use mathematical tables to evaluate (Cos 400 – sin 300)
= 109.470 ( C ) A – 0. 2660 B. – 0. 0266 C. 0. 0266
2009/2a D. 0. 2660 E. 1. 5320
If 9cos x0 – 7 = 1 and 00    900, find x Solution
Solution Simply check both cos 40 and sin 30 in the table of cosines
9cos x0 – 7 = 1 of angles and sine of angles respectively.
9cos x = 1 + 7 (0. 7660 – 0. 5000) = 0. 2660 (D)
9cos x0 = 8 1987/4b GCE
8 Evaluate ,correct to three significant figures, the expression.
cos x0 =
9 Tan 53.20 sin 42.30
8 Cos 28.20
x0 = cos – 1
9 Solution
= cos – 1 0.8889 Tan 53.20 sin 42.30 = 1.337 × 0.6730
= 27.260 Cos 28.20 0.8813
= 1.02099
2009/52 Neco (Dec)  1.02 is 3sf
Solve the equation 3sin – 4cos = 0, if  is acute Using logarithm of angles
A 53.120 B 46.480 C 46.430 D 41.660 E 36.380
Solution No log
3sin – 4cos = 0 Tan 53. 20 0.1260
3sin  = 4cos  Sin 42. 30 1. 8280
4 1 . 9540 1. 9540
sin  = cos  Cos 28.20 1. 9451 1. 9451
3
Divide both side by cos  0.0089
sin 
Check up antilog values 0.00 under 8 difference 9
4
= 0.0089 antilog is 1019
cos  3
+ 2
tan  = 4/3 i.e 1021
 = tan – 1 1.333 1.021 = 1.02 to 3sf
= 53.120
2014/52 Neco ( Nov ) Exercise 13.27
1995/6a
197
Using mathematical tables, x =
7
Find (i) 2 sin 63. 350 (ii) log cos 44. 740 sin 36
Solution 7
=
i. check up 63 under 0. 3 difference of 0. 05 i.e. 0. 05 0 x 60 0.5878
= 31 = difference of 31 which is 4 = 11.91 cm
Hence sin 63. 35 = 0. 8938  11.9cm
Thus 2 sin 63. 35 = 2 x 0. 8938
2006/3 NABTEB (Nov)
= 1. 7876  ___

ii. Check up log cos 44. 74 in the log of cosines. We look up Given that D A B = 600, A C B = 900, AB = 8cm with
440 under 0. 70 difference of 0. 040 i.e. 0. 04 x 60 = 21 ___ ___
difference which is 2. but note “subtract difference” AD // BC , calculate x
Hence log cos 44. 74 = 1. 8517 D A
0
-2 60

1 . 8515
8cm
2000/2a Exercise 13.31 xcm
Given that cos x = 0. 7431, 00 x 900,
x
use tables to find the values of ( i ) 2sin x ( ii ) tan B C
2
16 3 8 3
A cm B cm C 4 3 cm D 4cm
3 3
Trig. ratios triangular problems Solution
D A
0
2005/10b NABTEB 60

The hypotenuse of a right angle triangle is 17cm and


one of the angles is 430, find the 8cm
xcm
( i ) third angle ( ii ) side opposite the smallest angle
Solution 0
A 60
B C

430 DAB = ABC = 60 ( alternate angles ) 0

17cm In ABC, the sides relevant to 600 are Opp and Hypo (SOH)
x
sin 600 =
8
B C
8 sin 600 = x
3
( i ) The third angle C is 8 = x
2
43 + 90 + C = 180 (sum of angle in )
C = 180 – 133 4 3 cm = x (C)
= 470 2010/36 NABTEB (Nov)
( ii ) Side opposite the smallest angle is BC In the diagram below, find the value of the length /HQ/ given
Using 430, the sides relevant to it are Opp and Hypo (SOH)  
BC that G H Q = 900, H G Q = 750 and GHQ is right – angled
sin 43 = G
17
17 sin 43 = BC
11.59cm = BC 750
9cm
2014/36 NABTEB

H Q
x
7cm
A 8.70cm B 3.00cm C 2.50cm D 2.33cm
Solution
36 0
The sides relevant to 750 are Opp and Hypo (SOH)
Calculate the value of x in the figure above sin 750 =
HQ
A 10.7cm B 11.1cm C 12.2 cm D 13.7cm 9
Solution 9 sin 750 = HQ
The sides relevant to 360 are Opp and Hypo (SOH) 8.693 cm = HQ
sin 360 =
7 8.7 cm  HQ (A)
x
x sin 360 = 7

198
2008/4b  = tan– 1 2
S
= 63.430
Also 90 +  +  = 1800 (sum of angles in )
0

90 + 63.43 +  = 180
2cm
 = 180 – 153.43
= 26.570
In PRS, the relevant sides to  are Opp and Adj ( TOA)
P Q R
3 cm 12
____
tan  = = 1.2
4+6
In the diagram, PQR is a straight line QR = 3 cm
 = tan– 1 1.2
and = 50.190
____
SQ = 2cm. Calculate, correct to one decimal place PQS Also  + 90 + (x0 + ) = 1800 (sum of angle in )
Solution 50.19 + 90 + x0 + 26.57 = 180
Let  SQR = , x0 = 180 – 166.76
Sides of relevance here are Adj and Hypo (CAH) = 13.24
 130 (D)
3
cos  =
2
3
 = cos – 1 2005/12a
2 In the diagram, ABD is right – angle at B.
By special trig ratio  
 = 300 /AB/ = 3cm, /AD/ = 5cm, A C B = 610 and D A C = x 0
D
 PQS = 1800 –  (sum of angles on a straight line)
= 180 – 30
= 1500 5cm
S C
0
61
x0
A B
3cm
180-
Calculate, correct to one decimal place, the value of x.
P Q R Solution
D
2007/32 good case
In the diagram /PQ/ = 4cm, /QR / = 6cm, /RS / = 12cm
and QRS = 900. Find the value of x 5cm C
S 0
61
x0
x0
A B
12cm 3cm
To find x0, first we find  and 
In ABC  + 610 + 900 = 1800 (sum of angles in )
P 4cm Q 6cm R  = 180 – 151
= 290
A 27 B 26 C 18 D 13 In ABD,
Solution the relevant sides to  are Opp and Hypo (SOH)
S
3
sin  =
5
x0  = sin– 1 0.6
12cm = 36.870
Also in  ABD
 + 90 + ( + x0) = 1800
P
36.87 + 90 + 29 + x0 = 180
4cm Q R
6cm x0 = 180 – 155.87
= 24.13
To find x0, first, we find ,  and .  24.10 to 1 d.p
In QRS, the relevant sides to  are Opp and Adj ( TOA)
12
tan  = = 2
6
199
2006/13 But AB = AP + BP i.e 2BP (AP = BP)

In the diagram, /QR/ = 5cm, P Q R = 60 and P S R = 0

= 2  3.7678
= 7.5356 cm
450. Find /PS/, leaving your answer in surd form.
P  7.54cm to 3 s.f
( ii ) By Pythagoras rule in ABC
AC2 = AB2 + BC2
AC2 = 7.542 + 52
= 56.8516 + 25
AC2 = 81.8516
60 0 450 AC = 81.8516
Q S
5cm R = 9.047cm  9.05cm to 3 s.f
A 4 5 cm B 3 7 cm C 4 6 cm D 5 6 cm (b)
A
Solution
To find PS, first we solve for PR in PQR
In PQR, the sides relevant to 600 are Opp and Adj (TOA)
PR
tan 600 = P
QR
PR 
tan 600 =
5 37 0
0
5 tan 60 = PR B 5cm C

Instruction is leaving your answer in surd form
To find P C A i.e , first we solve for  in ABC,
Thus by special trig ratio
the sides relevant to  are Opp and Adj (TOA)
5  3 cm = PR i.e PR = 5 3 cm 5
tan  = = 0.6631
Next, in PRS, 7.54
the side relevant to 450 are Opp and Hypo (SOH)  = tan– 1 0.6631
PR = 33.550
sin 450 = Also  + 90 + (37 + ) = 1800 (sum of angles in )
PS
PS sin 450 = PR 33.55 + 90 + 37 +  = 180
PR  = 180 – 160.55
PS = = 19.45  19 0 to nearest degree
sin 45 0
2010/ 31 NABTEB (Nov)
1 A ladder leans on a wall of a building at an angle of 450. If
PS = 5 3 
2 the length of the ladder is 5m, find the height of the wall at
=5 3  2 = 5 6 cm (D) which the ladder touches it (leave your answer in surd form)
5 1
2006/7 (Nov) A 2m B 2m C m D 5m
A 2 2
Solution

45 0
P
r
5m
de

y
lad

37 0
B 5cm C
In the diagram, ABC is right - angled at B, The statement “ladder leans on a wall of building at an angle

P is the mid – point of AB, P C B = 37 and /BC/ = 5cm 0 of 450 ” shows that 450 is fitted as shown above.
( a ) Calculate, correct to three significant figures: The relevant sides to 450 are Adj and Hypo (CAH)
( i ) /AB/; ( ii ) /AC/. y
cos 450 =
 5
( b ) Calculate P C A correct to the nearest degree 5 cos 450 = y
Solution 1
( i ) To find /AB/, first we solve for BP in BCP 5 = y
2
In BCP, the relevant sides to 37 are Opp and Adj (TOA)
0
5
tan 370 =
BP y =
5 2
0
5 tan 37 = BP 5 2 5
3.7678cm = BP Rationalizing y =  = 2 m (A)
2 2 2

200
2005/49 (Nov) Exercise 13.32 Right – angled triangular problems
A ladder leans against vertical wall at an angle 600 to (Pythagoras rule)
the wall. If the foot of the ladder is 7cm away from the 2009/36 NABTEB (Nov)
wall, calculate the length of the ladder A ladder 25m long leans against a wall. If the top of the
7 3 14 3 ladder is 7m from the base of the wall, how far is the foot of
A m B 7m C m D7 3m
3 3 the ladder from the base of the wall?
A 12m B 24m C 49m D 54m
2005/12 Neco Exercise 13.33
Solution
A ladder 25m long rest against a vertical wall. If ladder
makes an angle of 600 with a wall, find the distance between
the foot of the ladder and the wall, correct to two place of
decimal.

wall
25
7m

er
A 25.00m B 21.65m C 20.00m D 19.65m E 12.50m

dd
la
2014/36 NABTEB Exercise 13.34
x
Applying Pythagoras rule:
x 252 = 72 + x2
7cm
252 – 72 = x2
35 0
625 – 49 = x2
576 = x2
Calculate the value of x in the figure above 576 = x thus x = 24m ( B )
A 10.7cm B 11.1cm C 12.2 cm D 13.7cm
2003/7b
2005/2b Neco Exercise 13.35
Use the dimensions given in the diagram below to
In the diagram below, find the value of x
A calculate /AD/ to 1 decimal place
A
300 x

D 8c
10m D m

5cm
60 0
B C
B 4cm C
2015/24 Neco Exercise 13.36
6cm Solution
A B First, in BCD, by Pythagoras rule
DC2 = BD2 + BC2
60 0 52 = BD2 + 42
D 5 – 42 = BD2
2

25 – 16 = BD2
9 = BD2
9 = BD i.e BD = 3
In ABC, by Pythagoras rule
C
AC2 = AB2 + BC2
In the above figure, D is such that BD : CD = 1 : 2. 82 = AB2 + 42
Find /BC/. 8 – 42 = AB2
2

A 3 B2 3 C 4 3 D6 3 E8 3 64 – 16 = AB2
48 = AB2
48 = AB i.e AB = 6.928cm
Thus, /AD/ = AB – BD
= 6.928 – 3 = 3.9cm to 1d.p
2008/16
In the diagram,QPR = 900 . if q2 = 25 – r2,
find the value of P
P

r q

Q R
p
A3 B4 C5 D6
201
Solution 25 = AC i.e AC = 5cm
By Pythagoras rule
Next, in ABC, by Pythagoras rule
p2 = r2 + q2
AC2 = AB2 + BC2
Substituting of q2
52 = 32 + BC2
p2 = r2 + 25 – r2
52 – 32 = BC2
p2 = 25
25 – 9 = BC2
p = 25 16 = BC2
p=5 (C) 16 = BC thus BC = 4cm
2005/12 (Nov)
In the diagram, /PR/ = (y + 5)cm, /QR/ = (5 – y) cm 2014/ 17 Neco
A 6cm B
and PQR = 900, find /PQ/
P

5cm

(y +5)cm y cm
C

Q R 8 cm
(5 - y)cm
D
A 4 5 y cm B 2 5 y cm C y 2 − 50 cm
Find the value of y in the figure above
D 2
y + 50 cm A 6.00cm B 7.81cm C 8.00cm
D 9.00cm E 11.18cm
Solution
Solution
Since PQR is a right – angled; by Pythagoras rule
First, in ABC, by Pythagoras rule
(y + 5)2 = (5 – y)2 + PQ2
AC2 = 62 + 52
(y + 5)2 – (5 – y)2 = PQ2
AC2 = 36 + 25
By difference of two squares rule
AC2 = 61
[(y + 5) + (5 – y)] [(y + 5) – (5 – y)] = PQ2
(y + 5 + 5 – y) (y + 5 – 5 + y) = PQ2 AC = 61 cm
10  2y = PQ2 Thus in ACD, by Pythagoras rule
20y = PQ2 AD2  y2 = AC2 +CD2
20 y = PQ y2 = ( 61) 2 + 82
4  5 y = PQ y2 = 61 + 64
y2 = 125
2 5 y cm = PQ (B)
y = 125 = 11.18cm (E)

2005/12 NABTEB 2005/19 NABTEB (Nov) Exercise 13.37


A
A ladder is placed on top of a vertical wall with its bottom
5m away from the wall on the level ground. If the ladder is
m

13m in length, how high up is the wall?


3c

13
cm A 8m B 12m C 14m D 18m
B
2010/36 Neco Exercise 13.38
Calculate the value of y to 3 significant figures in the
D diagram below
C 12cm 7cm
ABCD is a quadrilateral such that AD = 13cm,
CD = 12cm, AB = 3cm, ABC = 900, ACD = 900
12. What is the length of the side BC? 12cm y
A 4cm B 5cm C 10cm D 12cm
Solution
First, in ACD, by Pythagoras rule 5cm
AD2 = AC2 + CD2 A 13.00 cm B 11.00cm C 10.95cm
132 = AC2 + 122 D 10.9cm E 9.35cm
13 – 122 = AC2
2

169 – 144 = AC2


25 = AC2

202
2014/22 Neco (Nov) Exercise 13.39 Single-triangle cases involving angle of elevation
A ladder 23m long rests against a vertical wall so that
the foot of the ladder is 11m from the wall. Find the 2008/47 Neco (Nov)
angle that the ladder makes with the wall to the nearest The angle of elevation of a point P on a tower from a point Q
degree. on the horizontal ground is 600. If /PQ/ = 74m, how high is P
A 260 B 290 C 610 D 640 E 660 above the ground?
37 3 74 3
2015/17(Nov) Exercise 13.40 A m B m C 37 3 m D 74 3 m E
2 3
P
37m
x Solution
P
S
10m

10m

Tower
74
x
Q 10m R
60 0
In the diagram, /PQ/ = /QR/ = /RS/ = 10cm, Q ground
PQR = PRS = 900 .
Calculate the perimeter of quadrilateral PQRS The relevant sides to 600 are Opp and Hypo (SOH)
x
A (10 + 3 )cm B 10 3 cm C 10(2 + 3 )cm sin 600 =
74
D 10(3 + 3 )cm 74 sin 60 = x
3
74  = x
2
37 3 m = x (C)

2003/4b NABTEB
A ladder 6m long leans against a wall so that it makes an
angle of 650 with the horizontal ground. Calculate how far
CHAPTER FOURTEEN up the wall the ladder reaches, to 3 significant figures
Solution
Angles Of Elevation & Depression
6m

er

x
dd

φ
La

φ 65 0

The relevant sides to 650 are Opp and Hypo (SOH)


x
sin 650 =
6
  6 sin 650 = x
5.438m = x
Fig.I Fig.II x  5.44m to 3 s.f

2005/33 NABTEB (Nov)


A boy flies a kite with a string of length 50m. If the string
Fig.I shows a boy standing on the ground viewing a makes an angle of 300 with the ground, what is the height of
plane with an angle of elevation  while the pilot views the kite above the ground?
the boy at an angle of depression . A 25m B 45m C 56m D 100m
Fig II shows a man viewing his daughter in her office at Solution
Kite
an angle of elevation  and the daughter views her
father at an angle of depression .
Deductively in both fig I and fig II
 =  (Alternate angles).
m

The angles of elevation and depression can be used in


50

y
g
rin

calculating among others


St

(a) the height of an object above the ground


30 0
(b) the distance of an object from a given point of
observation. The relevant sides to 300 are Opp and Hypo (SOH)
203
sin 300 =
y Solution
50 Top
50 sin 30 = y
1
50  = y
2
25m = y (A)

Flag pole
25m
2014/27 Neco (Dec)
The shadow of an electric pole 75 3 m high is 75m Shadow
long. Determine the angle of elevation of the sun
18m
A 300 B 600 C 900 D 1200 E 1500
Solution The relevant sides to 600 are Opp and Adj (TOA)
25
tan  = = 1.3889
18
 = tan– 1 1.3889
Electric pole

= 54.20 to 1 decimal place


75 3 m
2008/7a Neco counter example
A flagpole XY of length 12m tilts towards an observation
Shadow
point A at an angle of 20 to the vertical. If A is on the same
75m level as the foot X of the flagpole and
The relevant sides to  are Opp and Adj (TOA) /AX/ = 10m, find the angle of elevation of the top of the
75 3 flagpole from A to the nearest degree
tan  = Solution
75 Y Top of
tan  = 3 flag pole

 = tan– 1 3
By special angle trig ratios
 = 600 ( B ) 12m
20

2006/20 Neco good case (90-2)


88 0
A ladder leans against a vertical wall making an angle X A Observation
whose cosine is 0.6 with the ground. The distance Foot of 10m point
flag pole
between the foot of the ladder and the base of the wall
is 1.2m. Calculate the length of the ladder Here no special trig ratio angles of 300, 600 and 450.
A 2.50m B 2.00m C 1.50m D 0.96m E 0.72m The resulting triangle has two sides, the third side
Solution /YA/ = x can be gotten by cosine rule
x2 = 122 + 102 – 2(10)(12) cos 880
th = 144 + 100 – 240 cos 88
le ng = 244 – 240 cos 88
e r
dd x2 = 235.62
wall

La
x = 235.62 = 15.35m
Next, we find  using sine rule
Foot of Base of 12 15.35
ladder 1.2m wall =
sin  sin 88
We are informed that 12 sin 88
= sin 
1.2 15.35
cos  =
ladder length 11.99
= sin 
1.2 15.35
0.6 = sin  = 0.7811
ladder length
0.6  ladder length = 1.2  = sin– 1 0.7811
1 .2 = 51.360
Ladder length =  510 to nearest degree
0 .6
= 2.00m (B)
2006/19 Neco 2005/ 43 Exercise 14.1
The shadow of a flagpole 25m long is 18m. What is the The angle of elevation of the top of a cliff 15metres high
angle of elevation of the top of the flagpole from the from a landmark is 600. How far is the landmark from the
shadow, correct to 1 decimal place? foot of the cliff? Leave your answer in surd form.
A 35.80 B 43.90 C 46.10 D 53.40 E 54.20 A 15 3 m B 15 2 m C 10 3 m D5 3m
204
2006/50 Exercise 14.2 In triangle with 370, relevant sides are Opp and Adj (TOA)
A ladder 16m long leans against an electric pole. If the tan 370 =
5
ladder makes an angle of 650 with the ground, how far 2+ x
up the electric pole does its top reach? (2 + x) tan 370 = 5
A 6.8m B 14.5m C 17.7m D 34.3m 2+x =
5
2005/10 Neco Exercise 14.3 tan 37 0
A man stands on the ground 12m away from a building 2 + x = 6.6352
which is 16m high. Find the angle of elevation of the x = 6.6352 – 2
top of the building from the man’s feet. = 4.6352m
A 53.130 B 48.590 C 41.410 D 36.870 E 30.960 Next, in triangle with ,
2006/49 Neco (Nov) Exercise 14.4 the relevant sides are Adj and Hypo (CAH)
If point A is 20m away from the foot of an electric pole cos  =
x
of height 15m, calculate the angle of elevation of the ladder length
top of the pole from point A. 4.6352
cos  =
A 18.430 B 36.870 C 51.340 D 73.740 E 75.250 8.3082
cos  = 0.5579
Further cases on single triangle  = cos– 1 0.5579
2013/13 = 56.089  560 to nearest degree
When one end of a ladder, LM, is placed against a
vertical wall at a point 5 metres above the ground, the 2006/3b Neco counter example
ladder makes an angle of 370 with the horizontal The angle of elevation of the top of a flagpole is 640 from a
ground. (a) Represent this information in a diagram point 32m away from the foot of the flagpole. Find the angle
(b) Calculate, correct to 3 significant figures, the length of elevation of a flag half way up the flagpole from that
of the ladder point correct to the nearest degree
(c) If the foot of the ladder is pushed towards the wall Solution
by 2 metres, calculate correct to the nearest degree, B
(Top of flag)
the angle which the ladder now makes with the ground
Solution
(a)

er D (Half way)
dd
La 5m

37 0
Fig I 0 O
A 64 C
0
(b) The relevant sides to 37 are Opp and Hypo (SOH) 32m
sin 370 =
5 Let’s get AC from ABC
ladder length AC
0 tan 640 =
Ladder length × sin 37 = 5 32
5 AC = 32 tan 640
Ladder length =
sin 37 0
It was stated that Length CD = 1/2 AC = 16 tan 640
= 8.3082m  8.31m to 3 s.f From ADC tan =
CD
BC
16 tan 64 0
n

=
itio

y
2n dder

32
os
dp
La

tan = 1/2 (tan 640)


 = tan– 1 1.02515
= 45.710  460 to nearest degree
der ion 5m
Lad posit 2013/5
1st
A boy 1.2m tall stands 6m away from the foot of a vertical
37 0 lamp pole 4.2m long. If the lamp is at the tip of the pole.
2m x (a) represent this information in a diagram
(c)
(b) Calculate
Since ladder length remains unchanged in both
( i ) length of the shadow of the boy cast by lamp
triangles, we find x from triangle with 370, thereby
( ii ) angle of elevation of the lamp from the boy
getting two sides in triangle with . correct to the nearest degree
205
Solution 8.3 − 1.7
Lamp tan  =
13
6 .6
tan  = = 0.5077
13
 = tan – 1 0.5077
 = 26.920  270 to the nearest degree

4.2m pole
2013/44 Neco
A 1.8m tall man observes a bird on top of a tree. If the man
6m is 21m away from the tree and his angle of sighting the bird
is 300. Calculate the height of the tree
A 10.50m B 11.48m C 12.13m D 13.92m E 18.19m
1.2m
1.2m
Boy

Solution
Bird
Shadow 6m

First, we calculate , before shadow length.


K
In the  with angle ,
the relevant sides are Opp and Adj (TOA) Tree
4.2 − 1.2 30 0
tan  = 21m
6
3
tan  = 1.8m 1.8m

Man
6
 = tan – 1 0.5
21m
= 26.570  270
In the resulting right – angle triangle,
 +  + 900 = 1800 the sides relevant to 300 are Opp and Adj (TOA)
27 +  + 900 = 1800 k
tan 30 =
 = 180 – 117 = 630 21
In the whole triangle ( bigger ) triangle with  , 21 tan 30 = k
The relevant sides are Opp and Adj ( TOA ) 12.12m = k
shadow + 6 Thus height of tree = k + 1.8m
tan  = = 12.12m + 1.8m = 13.92m ( D )
4.2
shadow + 6 2005/14 Neco Exercise 14.5
tan 630 = A man, 1.5m tall, observes a bird at the top of a tree,
4.2
4.2 tan 630 = shadow + 6 4.5m high. If the bird is 8m away from the man, calculate
the distance between the feet of the man and the base of the tree
8.2430 = shadow + 6
8.2430 – 6 = shadow length A 55 m B 61 m C 73 m D 11m E 14m
2.2430m = shadow length 2014/26 (Nov) Exercise 14.6
shadow length = 2m to the nearest whole number The angle of elevation of a bird in the air from a hunter
2014/38 standing on the ground is 350. If the hunter is 1.4m tall and
A man’s eye level is 1.7m above the horizontal ground the bird is 81.7m away along the hunter’s line of sight, how
and 13m from a vertical pole. If the pole is 8.3m high, high, correct to1 decimal place, is the bird from the ground?
calculate, correct to the nearest degree, the angle of A 48.3m B 58.8m C 68.3m D 71.1m
elevation of the top of the pole from his eyes. 2005/20(Nov) Exercise 14.7
A 330 B 320 C 270 D 260 The angle of elevation of the top of a fence from a boy
Solution 1.2m tall is 570, if the boy is 24m away from the fence, find,
Top of pole
correct to two decimal place, the height of the fence.
A 13.07m B 20.13m C 36.96m D 38.16m
2015/25 Exercise 14.8
The angle of elevation of an aircraft from a point K on the
horizontal ground is 300. If the aircraft is 800m above the
8.3m
Eye
ground, how far is it from K?
13m A 400.00m B 692.82m C 923.76m D 1,600.00m
1.7m 1.7m 2015/57 Neco Exercise14.9
Man

The angle of elevation of the top of a communication mast


from a point A is 600. If the mast is 120m high, how far is
13m the point A from the foot of the mast, correct to one decimal
In the right – angled triangle formed place?
The relevant sides to  are Opp and Adj (TOA) A 60.0m B 69.3m C 103.9m D 120.0m E207.8m
206
Man(top of building)
Single-triangle cases involving angle of depression

2014/30 NABTEB (Nov)


From the top of a building 10m high, the angle of
depression of a stone lying on the horizontal ground is 25m
690. Calculate, correct to 1 decimal place, the distance
of the stone from the foot of the building
Object
A 3.6m B 3.8m C 26.1m D 9.3m 30m
Foot of building

Solution
Top of building Angle of depression is equal to angle of elevation.
0
69 The side relevant to  are opp and adj (TOA)
25
tan  = = 0.8333
30
 = tan – 1 0.8333
10m
 = 39.804
 39.80 ( B )
69 0
Stone Foot of building 2013/49 Neco
y
A bird on top of a building roof edge 35m high sighted a
The sides relevant to 690 are Opp and Adj (TOA) grain on a level ground at angle 600. How far is the grain to
10 the foot of the building?
tan 690 =
y A 17.50m B 20.21m C 40.42m D 70.00m E 159.79m
0
y tan 69 = 10 Solution
Bird(top of building)
10
y = 60 0
tan 69 0
= 3.8386m  3.8m to 1 d.p
2014/4a Neco ( Dec) 35m
A boat can be sighted at the sea 71.5m from the foot of
a cliff which is 26m high. Calculate the angle of 60 0
Grain
depression of the boat from the top of the cliff, correct x
Foot of building

to 2 sig. figures.
Solution The sides relevant to 600 are Opp and Adj (TOA)
Top of cliff 35
tan 600 =
x
x tan 600 = 35
35
x=
tan 60
26m = 20.21m ( B )
2010/50 Neco
An erected electric pole is 7m high from the ground. It casts
Boat Foot of cliff
71.5m a shadow on the horizontal ground when the altitude of the
sun is 600. Calculate the length of the shadow.
The sides relevant to  are Opp and Adj (TOA) 7 3
26 A m B 7m C 7 3m D 21m E 21 3 m
tan  = = 0.3636 3
71.5
Solution
 = tan – 1 0.3636
= 19.98 60 0
 200 to 2 s.f
Electric pole

2006/50 Neco (Nov)


A man at the top of a building 25m high looks down at 7m
an object 30m away from the foot of the building.
Calculate the angle of depression of the object from the 60 0 Shadow
man x
A 19.90 B 39.80 C 50.20 D 70.10 E 79.60
Solution The sides relevant to 600 are Opp and Adj (TOA)
7
tan 600 =
x
x tan 600 = 7
x = 7  tan 60

207
=7 3 2013/5b Neco Exercise 14.10
1 7
A boat is on the same horizontal level as the foot of a cliff
= 7 × i.e and the angle of depression of the boat from the top of the
3 3 cliff is 600. If the boat is 150m away from the foot of the
Rationalizing cliff, find the height of the cliff correct to three significant figures.
7 3 7 3
=  = m (A) 2006/32 (Nov) Exercise 14.11
3 3 3 A man on top of a cliff 100m high observes that the angle of
2010/2a depression of a boat at sea is 150. How far is the boat from
The angle of depression of a boat from the mid – point the foot of the cliff?
of a vertical cliff is 350. If the boat is 120m from the A 268m B 373m C 425m D 500m
foot of the cliff, calculate the height of the cliff 2013/25 Exercise 14.12
Solution An object is 6m away from the base of a mast. The angle of
depression of the object from the top of the mast is 500. Find,
x correct to 2 decimal places, the height of the mast
A 8.60m B 7.51m C 7.15m D 1.19m
Mid point Mid point 2011/ 49 Exercise 14.13
0 0
35 35
A boy looks through a window of a building and sees a
x mango fruit on the ground 50m away from the foot of the
building. If the window is 9m from the ground, calculate,
35 0 35 0
Boat Foot of Boat Foot of correct to the nearest degree, the angle of depression of the
120m cliff 120m cliff
mango from the window
Fig I Fig II A 90 B 100 C 110 D 120
From fig II, in  with 35 , 0
2005/13 Exercise 14.14
the relevant sides to 350 are Opp and Adj (TOA) The angle of depression of a boat at sea from the top of a
x
tan 350 = cliff is 720. What is the angle of elevation of the top of the
120 cliff from the boat?
120 tan 350 = x A 180 B 360 C 720 D 900
84.02m = x
Height of cliff = x + x 2015/19 Neco Exercise 14.15
= 84.02 + 84.02 The angle of depression of a fowl on the ground from the top
= 168.04m of a pole is 300. If the distance from the fowl to the foot of
the pole is 63m, calculate the height of the pole and leave
2008/49 Neco your answer in surd form.
From the top S of a radio mast QS, the angle of
depression of the top T of a 20 metres tower PT is 600. A7 3m B 21m C 21 3 m D 63m E 63 3
If P and Q are on level ground and / PQ/ = 40m, m
find the height of the mast to the nearest metres.
A 40m B 43m C 69m D 89m E 120m
Solution Double triangles cases
S S
600
2009/13b (Nov)
The pilot of an air craft 2,000 metres above the sea level
x
observes at an instance that the angles of depression of two
Radio mast

60 0
T T boats which are in direct straight line are 580 and 720. Find,
correct to the nearest metres, the distance between the two
20m
Tower

20m boats.
Solution
P Q P Q A Pilot
40m 40m
720
580
Height of mast is 20m + x
In triangle with 600,
The relevant sides are Opp and Adj (TOA)
x
tan 600 =
40 2000m
40 tan 600 = x
69.28m = x
Height of mast = 20 + x 720 580
= 20 + 69.28
= 89. 28m  89m (D) B C x D

The distance between the two boats is x = BD – BC


208
2000 2000 2005/11 Neco
In ABC, tan720 = In ABD, tan580 = R Q
BC BD S
2000 2000 45 0 60 0
BC = BD =
tan 72 0 tan 58 0
4cm
x = BD – BC
2000 2000
x = 0

tan 58 tan 72 0 P
= 1249.74 – 649.84
= 599.9m In the figure above, the angles of depression of P from Q and
R are 600 and 450 respectively. If /PS/ = 4m, find the length of RQ
 600m to nearest metres
A 4.0m B 3.2m C 2.3m D 1.7m E 0.8m
Solution
2014/10b (Nov)
RQ = RS – QS
From a point P on a level ground and directly west of a 4
pole, the angle of elevation of the top of the pole is 450 In PRS, tan 450 =
RS
and from point Q east of the pole, the angle of elevation
RS tan 450 = 4
of the top of the pole is 580. If /PQ/ = 10m, calculate, 4
correct to 2 significant figures, the: RS = = 4m
tan 45
(i ) distance from P to the pole; (ii ) height of the pole
Solution 4
In PQS, tan 600 =
QS
O QS tan 600 = 4
4
QS = = 2.309m
tan 60 0
Height(H)

Thus, RQ = RS – QS RQ
Pole

= 4 – 2.309
45 0 580 = 1.691m
P Q East
West x R 10 - x = 1.7m (D)
10m 2005/6a NABTEB (Nov)
H A flag – pole is placed on the top of a tower. The angles of
In OPR, tan 450 = elevation of the bottom and top of the flag – pole from a
x
point on the ground 12m away are 450 and 540 respectively.
H = x tan 450 -------- ( i )
H
Find the length of the flag – pole to 2 decimal places
Also in  OQR, tan 580 = Solution
10 − x P
H = tan58(10 – x) --------- (2)
Equating (1) to (2)
H  x tan 450 = tan 58(10 – x)
x × 1 = 1.6 (10 – x)
x = 16 – 1.6x R
x + 1.6x = 16
2.6x = 16
16 540
x= i.e 6.15m
2 .6 450
O Q
x = 6.15m  6.2m to 2 s.f 12m
(ii) Substitute x value into (1) To find PR = PQ – QR
H = x tan 450 becomes QR
= 6.15 × 1 In OQR, tan 450 =
12
 6.2m to 2 s.f 12 tan 450 = QR
Alternatively 12m = QR
Substitute x value into (2) PQ
H = tan 580 (10 – x) becomes In OPQ, tan 540 =
12
H = tan 580 (10 – 6.15) 12 tan 540 = PQ
Don’t use approximated values for substitutions 16.5166m = PQ
= tan 580 (3.85)
= 6.16m  6.2m to 2 s.f Thus PR = PQ – QR
= 16.5166 – 12
= 4.5166m
 4.52m to 2 d.p
209
2012/8 different case 2009/7b Neco
A point H is 20m away from the foot of a tower on the A vertical mast is erected 24m away from a building. From
same horizontal ground. From the point H, the angle of the top of the building, the angle of elevation and depression
elevation of the point (P) on the tower and the top (T) of of the top and foot of the mast are 600 and 450 respectively.
the tower are 300 and 500 respectively. Calculate correct Calculate the:
to 3 significant figures (a) /PT/ ( i ) height of the mast
(b) The distance between H and the top of the tower; ( ii ) distance between the top of the building and the
(c) The position of H if the angle of depression of H top of the mast (correct all answers to the nearest metres)
from the top of the tower is to be 400 Solution
Solution A
T
(Top of tower)

Ma st
600 D
E 450
500

Building
300
H O
20m
(a) To find PT, we solve for OP and OT and subtract
OP
In OPH, tan 300 = B C
20 24m
0
20 tan 30 = OP ( i ) Height of the mast AB = AE + BE
11.547m = OP AE
In ADE, tan 600 =
Also in OTH, tan 50 =
OT 0 24
20 AE = 24 tan 600
0
20 tan 50 = OT = 24 × 1.732 = 41.57m
23.835m = OT BE
Thus PT = OT – OP In ABE, tan 450 =
24
= 23.835 – 11.547 BE = 24 tan 450
= 12.288m  12.3m to 3sf = 24 × 1.0 = 24m
20
(b) In OTH, cos 500 = Thus AB = AE + BE
HT
= 41.57 + 24 = 65.57m  66m to nearest metres
HT cos 500 = 20
20
( ii ) AD = ?
HT = 24
cos 50 In ADE, Cos 600 =
AD
= 31.114m  31.1m to 3s.f
(c) 24 24
AD = 0
=
T cos 60 0 .5
40 0 = 48m
2005/13a
Two towers XA and YB on the same level are 200m apart.
If their heights are 150m and 85m respectively, calculate the
(i) angle of elevation of the top of XA from the top of YB;
(ii) distance between the tops of the two towers,
40 0 50 0 correct to the nearest metres
H2 H 20m O X
x X
OT
In OTH2, tan 400 =
x + 20
tan 400(x + 20) = 23.835
0.8391x + 16.782 = 23.835 C Y Y C
0.8391x = 23.835 – 16.782
0.8392x = 7.053 150m 85m 85m 150m
7.053
x = = 8.405 8.41m to 3s.f m A B B A
0.8391 200m 200m
The position of H would be (8.41 + 20)m i.e 28.41m Fig I fig II
away from the foot of the tower ( i ) Fig I will give us headache to find 
210
From Fig II in CXY, Obtuse angle (i.e.  between 900 and 1800)
the sides relevant to  are Opp and Adj (TOA)
XC
tan  = (XC = 150 – 85) and (YC = AB)
YC
65
tan  = i.e 0.325
200
Reflex angle (i.e.  between 1800 and 3600)
 = tan – 1 0.325
= 180
( ii ) XY can be gotten from CXY by Pythagoras rule
XY2 = XC2 + YC2
= 652 + 2002
2
XY = 4225 + 40000
XY = 44225 Others
= 210.297m Right angle (i. e. 900)
 210m to the nearest metres

2015/10b Exercise 14.16


From an aeroplane in the air and at a horizontal distance
of 1050m, the angles of depression of the top and base
of a control tower at an instance are 360 and 410 Straight line angles is 1800
respectively. Calculate, correct to the nearest metre, the
( i ) height of the control tower;
( ii ) shortest distance between the aeroplane and the
base of the control tower
2007/11b Exercise 14.17 SOME BASIC FACTS ON THE FORMATION OF
From two points on opposite sides of a pole 33m high, ANGLES BETWEEN LINES
the angles of elevation of the top of the poles are 530
and 670. If the two points and the base of the pole are on
the same horizontal level, calculate, correct to three STRAIGHT LINES
significant figures, the distance between the two points. ( i ) Sum of angles on a straight line is 1800
2008/47 Neco Exercise 14.18 Fig. I
E C
Two ladders of length 5m and 7m lean against a pole
and make angles 450 and 600 with the ground
b
respectively. What is their distance apart on the pole a c
correct to two decimal places? A B
f d
A 9.60m B 6.06m C 2.54m D 2.53m E 2.00m e

D F
From Fig. I, the following holds :
a + b + c = 1800 on line AB
CHAPTER FIFTEEN b + c + d = 1800 on line EF
f + e + d = 1800 on line AB
Bearing & Elements Of Plane Geometry a + f + e = 1800 on line EF

Elements of plane geometry (ii) Sum of angles at a point is 3600


Angles From Fig. I
The units of measuring angles are degrees and minutes. a + b + c + d + e + f = 3600
1 degree = 60 minutes
i.e. 10 = 601 (iii) Vertically opposite angles are equal
Sometimes, smaller units like seconds are used; m
1 minute = 60 seconds
i.e. 11 = 6011
o.
a
n
Acute angle (i.e.  between 0 and 90 )
0 0
n

m = n (vertically opposite angles)


o + n = a (vertically opposite angles)

211
Problems on angles formed between 2011/38 Neco
Y
straight lines
2014/51 Neco (Dec)
1 1
From the diagram below, b = 2
d, c= 3
a and d = 320. O 20
0

2x
Find the angle marked a0 3x 5x

X
0
b c0
If XOY is a straight line, find the value of x
a0 d0
A 100 B 160 C 180 D 190 E 250
A9 B 33 C 48 D 99 E 132 Solution
Solution 20 + 2x + 5x + 3x = 1800 (sum of angles on straight line )
If b = 12 d then 2b = d 20 + 10x = 180
1 10x = 180 – 20
If c= 3
a then 3c = a 10x = 160
and d = 320 x = 160/10 i.e 160 B.
2b = 320
b = 160 2014/20
Re- labeling our diagram

n
0 m r
16 c P Q R
3c 320
In the diagram, PQR is a straight line, (m + n) = 1200 and
Thus, the marked angle a0 = 3c
(n + r) = 1000. Find (m + r)
3c + 16 + c + 320 = 1800 (sum of angles on a straight line)
A 1100 B 1200 C 1400 D 1600
4c + 48 = 180
Solution
4c = 180 – 48
4c = 132 m + n + r = 1800 (sum of s on a straight line)
But m + n = 1200
c = 132/4 i.e 33
120 + r = 1800
and 3c = 33  3 r = 180 – 120
= 990 D. r = 600
Substituting for r value into
2014/50 Neco n + r = 100
In the figure below, ABC is a right angle. Find the n + 60 = 100
value of EBD correct to one decimal place n = 100 – 60
A
E n = 400
Substituting for n value into
m + n = 120
m + 40 = 120
D
m = 120 – 40
3x 0 m = 800
5x-2 0 0
2x-2
C
Thus m + r = 80 + 60
B
= 1400 C.
A 21.60 B 26.40 C 42.00 D 65.60 E 68.40
Solution 2005/16
We are to find 5x – 2 In the diagram, PQ and MN are straight lines.
3x + 5x – 2 + 2x – 2 = 900 (sum of angles of a right–angle) Find the value of x
10x – 4 = 90 M
10x = 90 + 4 2(x+30 0 )
Q
10x = 94
x = 9.40 86
0

Thus, 5x – 2 = 5(9.4) – 2
P
= 47 – 2 N
= 450 0 0 0
A 13 B 17 C 28 D 300
Solution
2(x + 30) + 86 = 1800 (sum of angles on a straight line)

212
2x + 60 + 86 = 180 Solution
2x = 180 – 146 3(x + y) + 45 = 1800 ----- (1) (sum of angles on line AB)
2x = 34 (5x + y) + y = 1800 ----- (2) (sum of angles on line AB)
x = 34/2 = 170 B. From (1) 3x + 3y + 45 = 180
3x + 3y = 180 – 45
2005/4 3x + 3y = 135
R
Q
Divide through by 3
x + y = 45 -------- (a)
Also from (2) 5x + y + y = 180
x0
5x + 2y = 180 –------- (b)
T P (a)  2 and subtract
5x + 2y = 180
– ( 2x + 2y = 90 )
S 3x = 90
x = 90/3
In the diagram, PTQ = x0, QTR is twice as big as x = 300
PTQ , RTS is three times as big as PTQ and Substituting x value into (a)
PTS is four times as big as PTQ. Find RTS x + y = 45 becomes
A 540 B 720 C 1080 D 1440 30 + y = 45
Solution y = 45 – 30
Let us label the diagram as instructed y = 150
R 2012/23
Q P R
2x0
x0
3x 0 y
4x 0 m x
T P M N
t z
n

S
S Q
We are to find 3x
x + 2x + 3x + 4x = 3600 (sum of angles at a point) In the diagram, MN, PQ and RS are three intersecting
10x = 360 straight lines.Which of the following statement(s) is/are true
x = 360/10 i.e 36 I t=y II x + y + z + m = 1800
0
Thus 3x = 3  36 i.e 1080 C. III x + m + n = 180 IV x + n = m + z
A I and IV only B II only C III only D IV only
2006/44 Solution
The value of three angles at a point are 3y – 450 , Option I : t can only be equal to x (Vertically opp. s)
y + 250 and y0. Find the value of y. and not t = y
A 400 B 580 C 680 D 760 Option II: x + y + z = 1800 (sum of s on line PQ)
Solution and not x + y + z + m = 1800
3y – 45 + y + 25 + y = 3600 (sum of angles at a point) we can accept x + y + m = 1800 and
5y – 20 = 360 others as z = m (vertically opp s)
5y = 360 + 20 Option III: x + m + n = 1800 (sum of angles on line MN)
5y = 380 True as x + m + y = 1800
y = 380/5 Here y = n (vertically opp angles)
y = 760 D. Option IV : x + n = m + z (not true)
2007/11a x + n = m + t or z + t (vertically opp angles)
C
2009/20 (Nov) Exercise 15.1
R

3(x+y) 0 110 0
45 0
A y0
B P Q
O O 50 0
x0
(5x+y) 0
D
T
S
In the diagram, AOB is a straight line,
AOC = 3(x + y)0, COB = 450, AOD = (5x + y)0 In the diagram, POQ, ROS and OT are straight lines.
and DOB = y0. Find the value of x and y POR = 1100 and QOT = 500. Calculate the value of x
A 700 B 600 C 550 D 440
213
2010/10 Exercise 15.2 ( iii ) Sum of interior opposite angles is 1800

a c
4m - 15 0 m + 75 0
b d

What is the value of m in the diagram? a + b = 1800 (sum of interior opposite angles)
A 200 B 300 C 400 D 500 c + d = 1800 (sum of interior opposite angles)

2013/39 Exercise 15.3


Problems on angles formed between
2y 0
parallel lines
2x 0
P Q 2010/ 3a
x + 10 0 P Q
80 0 0
50
0
125
r R

In the diagram, PQ is a straight line. Calculate the value S 800

of the angle labeled 2y0 44 0


U T
A 1300 1200 C 1100 D 1000
In the diagram, PQR = 125 , QRS = r, RST = 800 and 0

STU = 440. Calculate the value of r


PARALLEL LINES Solution
Two lines are parallel; if they do not and can never Produce parallel lines at R and S and label as:
meet (intersect) when their length is increased. Any line P Q
cutting across a pair of parallel lines is called 125 0

transversal. x
r R
y

T
r
ans
ver
sal P
ar
al
lel
a
S 800
b

440
( i )Corresponding angles are equal U T
x
b = 440 (alternate angles)
a a + b = 800
a + 44 = 80
y
a = 80 – 44
b a = 360
y = a i.e 360 (Alternate angles)
x = y (corresponding angles)
a = b (corresponding angles) 125 + x = 1800 (sum of interior opp s)
x = 180 – 125 = 550
In corresponding angles, one angle is inside while the other Thus r = x + y
angle is outside and both angles are on the same side.
= 550 + 360
= 910
( ii ) Alternate angles are equal
2014/1b
A

k 3x0
m F
E
n
I
120 0
B

k = I (alternate angles) G
H
m = n (alternate angles) 7x 0 C
Under alternate angles both angles are within the parallel
In the diagram, EF is parallel to GH .If AEF = 3x0,
lines but on different sides.
ABC = 1200 and CHG = 7x0, find the value of GHB
214
Solution Solution
A
To get a clear picture, produce the parallel lines a little
longer and label as shown below:
3x 0 P
E F

120 0 Q 480
R
B y
b
a y H
G
7x 0 C
x
S U
T
a = 3x0 (corresponding angles)
b + 120 = 1800 (sum of angles on a straight line) y = 480 (alternate angles)
b = 180 – 120 = 600 y + x = 1800 (sum of interior opp. s)
y + 7x = 1800 (sum of angles on a straight line) 48 + x = 180
y = 180 – 7x x = 180 – 48
= 1320 (B)
a + b + y = 1800 (sum of angles in a )
3x + 60 + (180 – 7x ) = 180 2014/37 (Nov) NABTEB
– 4x + 240 = 180 In the diagram below, ML//PQ and NP//QR.
4x = 240 – 180
If LMN = 400 and MNP = 550, find PQR
x = 60/4 = 150 R
Next, we substitute x = 15 into y = 180 – 7x
y = 180 – 7(15)
Q
= 180 – 105 = 750 P
2006/36 Neco
B E
N
550
60 0
0
y0 110 400 L
M

C D A 150 B 250 C 350 D 400


A
Solution
Find the value of y in the figure above in which AB//DE,
Produce another parallel line at N and label as shown below:
ABC = 600 and CDE = 1100
R
A 600 B 700 C 1200 D 1300 E 1700
Solution
Produce another parallel line at C and label as: b
0 Q
B E P
a0

60 0
y0 0
110 N
r0 s0 550 t0
r0
C D
A 400 L
M

y0 = r 0 + s0 We are to find PQR i.e b0


r0 = 600 (Alternate angles) r0 = 400 (Alternate angles)
s0 + 1100 = 1800 (sum of interior opp s) t0 + r0 = 550
s0 = 180 – 1100 = 700 t = 550 – 400
0
Thus y = r + s = 150
= 60 + 70 i.e 1300 (D) 0 0 0
a = t i.e 15 (Alternate angles)
2015/18 b0 = a0 i.e 150 (Alternate angles) A.
P
2013/43
S T
y0

480 198 0
Q R
R

72 0
x P Q
S U ____ ____
T
In the diagram, ST // PQ reflex angle SRQ = 1980
In the diagram, PQ//RT,QR//SU, PQR=480 and RTS = x.
find the value of x and RQP = 720. Find the value of y0
A 1340 B 1320 C 960 D 480 A 180 B 540 C 920 D 1080
215
Solution Solution
Produce another parallel line at R and label as shown below PQR = y0 (alternate angle)
S
y0
T x0 + PQR = 1800 (sum of angles on a straight line)
x0 + y0 = 1800 D.
y 198 0
R x0 2006/3a Neco
Find the value of y0 in the diagram below.
72 0 If QP is parallel to ST
P Q P T
72 + x = 180 (sum of interior opp s)
0

x = 180 – 72
= 1080
y0
x + y + 1980 = 3600 (sum of s at a point)
0

108 + y + 198 = 360 430 R


y = 360 – 306 i.e 540 Q
y = y i.e 540 (alternate angles)
0
B. S 307 0

Solution
2009/19 (Nov) Produce another parallel line at R and label as :
P P T

450 O
Q

y0

430 c0
a0 b
0
80 0
R Q R
M N S 307 0

In the diagram, MR//QO, /MN/ = /NO/, ONR = 800 c0 + 3070 = 3600 (sum of s at a point)
and QOP = 450. Find the size of the reflex PON c0 = 3600 – 3070
A 1350 B 2350 C 2800 D 3150 = 530
Solution b = c i.e 530 (alternate angles)
0 0

Let us label our target angles as: a0 = 430 (alternate angles)


P a0 + b0 + y0 = 3600 (sum of s at a point)
43 +53 + y0 = 360
450
y0
O Thus, y = 360 – 96 i.e 2640
Q
x0
2008/43 Neco
In the figure below, what is the value of the angle marked r?
A B
80 0
R
M N E K J
F
r
108 0
0 0
We are to find x + y
80 + x0 = 1800 (sum of interior opp. s)
H
x0 = 180 – 80 i.e 1000
G 72 0
450 + y0 = 1800 (sum of s on straight line)
D
y0 = 180 – 450 i.e 1350 C

Thus, x0 + y 0 = 100 + 135 A 360 B 720 C 1080 D 1440 E 1600


= 2350 (B) Solution
r + 1080 = 1800 (sum of interior opp. s)
2006/15 r = 180 – 108 = 720
In the diagram, PQ//RS, find x0 + y0 2010/37 Neco
x0 Find the value of t in the diagram below:
60 0
P Q

80 0

y0 t
R S

A 3600 B 3000 C 2700 D 1800


A 200 B 600 C 800 D 1000 E 1600
216
Solution
Produce another parallel line and label as shown below:
6y 0
60 0

x 800
y 0
128
6y 2y 0
t 2y

6y0 = 6y (corresponding angles)


x = 600 (alternate angles) 2y0 = 2y (corresponding angles)
x + y = 800 6y + 2y = 1280
y = 80 – 60 8y = 128
= 200 128
y + t = 180 (sum of interior opp. s)
0 y =
8
t = 180 – 20 y = 160 (A)
= 1600 E.
2005/52 Neco
2006/33 Neco M

Find the value of x in the diagram below with AB//DE,


CDE = 1150 and BCD = 1000
D E L
K
115 0
245 0

C 100 0
80 0
Q P

A
x
B In the figure above, LK // PQ . Reflex angle KLM = 2450
A 150 B 350 C 650 D 800 E 1000 and angle QPM = 800. What is the value of angle LMP?
Solution A 350 B 650 C 800 D 1000 E 1150
Produce another parallel line at C and extend DE and AB as: Solution
D
Produce KL to N and label as:
E M
r0
115 0
x0
C y 0
100
t
L y0 t0
K

A
x 245 0
B
r + 1150 = 1800 (sum of s on a straight line) 80 0
Q P
r = 180 – 1150
0 0
= 650 t = 80 (corresponding angles)
y = r0 (alternate angles) KLM + 2450 = 3600 (sum of angles at a point)
But y + t = 1000 KLM = 360 – 245
65 + t = 100 = 1150
t = 100 – 65 i.e 350 KLM + y = 1800 (sum of angles on a straight line)
0

x = t (alternate angles) 115 + y0 = 180


x = 350 B. y = 180 – 115 i.e 650
LMP(x0) + y0 + t0 = 1800 (sum of s in )
2008/42 Neco x0 + 65 + 800 = 1800
Find y in the figure below x0 = 180 – 145
x0 = 350 A.
6y 0
2007/19

128 0 0
x0
2y 0 110

A 16 B 21 C 32 D 96 E 120
From the diagram, find the value of x
Solution
A 800 B 700 C 550 D 350
Produce another parallel line at 1280 as:

217
Solution 2008/22 Exercise 15.8
Produce the parallel lines as: In the diagram, PQ//RS, QU//PT and PSR = 420.
Find angle x
z y x0
x
Q U
1100

z = 1100 (corresponding angles)


z + y = 1800 (sum of angles on a straight line) P T
420 S
110 + y = 180
y = 180 – 110 = 700
R
x0 = y i.e 700 (corresponding angle) B. 0 0
A 84 B 48 C 420 D 320
2005/53 Neco Exercise 15.4
B A
2009/44 Neco (Dec) Exercise 15.9
50 0 In the figure below, AE //BC. Reflex angle AED = 2500 and
angle EDC = 500. What is the value of angle BCD?
D
C x
50 0

3200 E E
D A
____ ____ 2500
In the above figure, BA // DE . Reflex CDE = 3200,
CBA = 500. Find the value of x B C
A 400 B 1300 C 1400 D 2700 E 3200
0 0 0 0
A 20 B 50 C 60 D 70 E 1100
2009/46 Exercise 15.5
Find the value of x in the diagram 2013/22 Exercise 15.10
x
7m
260 0 153 0

2m
30 0

A 500 B 600 C 700 D 800


2006/27 Exercise 15.6 Find the value of m in the diagram
In the diagram, P, Q and R as three points in a plane A 340 B 270 C 230 D 170
such that the bearing of R from Q is 1100 and the 2007/39 Neco Exercise 15.11
bearing of Q from P is 0500. Find angle PQR In the figure below, PQ//ST, QPR = 680 and
N N N PRS = 760, find the value of y
P Q
1100 680
Q
50 0 y

P
R T
S
0
76

R
A 600 B 700 C 1200 D 1600 A 2520 B 2240 C 2160 D 1920 E 1440
2008/11a Exercise 15.7
A y 2009/21 Exercise 15.12
E F
C
t0
0
0
26 120
750 J
E

x D
130 0

B F I H G

ˆ E = 750 and
In the diagram, AB//CD and BC//FE. CD In the diagram, IG is parallel to JE, JEˆ F = 1200 and FHˆ G =
1300. Find the angle marked t.
DEˆ F = 260. Find the angle market x and y A 400 B 700 C 800 D 1000
218
2006/43 Neco (Dec) Exercise 15.13 MQN + MQR = 1800 (sum of s on a straight line)
P
600 + MQR = 1800
R MQR = 180 – 60
Q
S = 1200 B.
239 0 2012/4a Neco
A B
88 0
130 0 125 0

T
x
In the diagram above, QP//TS, reflex angle PQR = 2390 C
y
D
and STR = 880. What is the value of QRT?
A 330 B 590 C 880 D 1210 E 1470
If AB//CD in the above figure, find x and y
Solution
Problems on angles formed between lines in x + 1300 = 1800 (sum of interior opp. s)
Triangles, Quadrilaterals x = 180 – 130
= 500
2010/11 y + 125 = 1800 (sum of interior opp. s)
P
y = 180 – 125 = 550
2012/22
R
V

M 30 0
y
Q L
R

75 0
S T
700 x
In the diagram, QR//ST, /PQ/ = /PR/ and PST = 750. U
T

Find the value of y


S
A 1050 B 1100 C 1300 D 1500 Q

Solution In the diagram, MQ//RS, TUV = 700 and


PQR = 750 (corresponding angles) RLV = 300. Find the value of x
Also PRQ = 750 (base angle of isosceles QPR) A 1500 B 1100 C 1000 D 950
PRQ + y = 1800 (sum of angles on a straight line) Solution
75 + y = 180 Let us label the diagram as:
R
V
y = 180 – 75 M 30 0
= 1050 A.
L

a
2011/23
M N
b
650
700 x
T
U
O P

1250 Q
S

Q a = 300 (corresponding angles)


b = a (vertically opp. angles)
R 70 + b = x (exterior angle equal to interior opp. s)
70 + 30 = x
In the diagram, MN//OP, NMQ = 650 and 1000 = x C.
QOP = 1250. What is the size of MQR?
2012/38 Neco
A 1100 B 1200 C 1300 D 1600
In the diagram below BE//CF, determine the value of  + 
Solution
E
NOP + 1250 = 1800 (sum of angles in straight line)
NOP = 180 – 125 O 200
= 550 F
MNQ = NOP (alternate angles)
650 +  MNQ +MQN = 1800 (sum of s in  QMN) 110 0
O 300
65 + 55 + MQN = 180 A B C D
MQN = 180 – 120
= 600 A 1300 B 1000 C 700 D 600 E 500
219
Solution Solution
1100 + EBD = 1800 (sum of angles on a straight line) (MNP + 1290) + 370 = 1800 (Sum of interior opp s)
EBD = 180 – 110 MNP + 129 + 37 = 180
= 700 MNP = 180 – 166
EBD + ( + 200) + 300 = 1800 (sum of s in  EBD) = 140
70 +  + 200 + 30 = 180 y + 112 + MNP = 1800 (Sum of s in PMN)
0

 = 180 – 120 y + 112 + 14 = 1800


 = 600 y = 180 – 126
EBD +  + ECB = 1800 (sum of s in  BEC) = 540 B.
70 + 60 + ECB = 1800 2014/11 Neco
ECB = 180 – 130 Find the value of XRY in the figure below
M
= 500
ECF =  i.e 600 (alternate angles)
ECB + ECF +  = 1800 (sum of s on a straight line)
50 + 60 +  = 180 X R
 = 180 – 110 i.e 700
Thus,  +  = 60 + 70
= 1300 A. 5 00
97 0
Y
2011/4a
P
P

42 0
T A 47 0
B 50 0
C 730 D 830 E 1470
y Solution
x
Q RYX + 970 = 1800 (Sum of angle on straight line)
RYX = 180 – 970
= 830
380
300 YXR = 500 (alternate angles)
R S
XRY + YXR + RYX = 1800 (Sum of s in YXR)
In the diagram, PQRST is a quadrilateral. PT//QS, XRY + 50 + 83 = 1800
PTQ = 420, TSQ = 380 and QSR = 300. If QTS = XRY = 180 – 133
x and PQT = y, find : ( i ) x ( ii ) y = 470 A.
Solution 2014 / 21 – 22
R
( i ) TQS = 420 (alternate angles)
TQS + x + 380 = 1800 (sum of s in  QTS)
42 + x + 38 = 180 S 45 0
x = 180 – 80 W
x = 1000
V
( ii ) RQS + 900 + 300 = 1800 (sum of angle in  RQS) x0

RQS = 180 – 120


= 600
y0 20 0
y + TQS + RQS = 1800 (sum of s on a straight U T

line) In the diagram, SR is parallel to UW , WVT = x0,


y + 42 + 60 = 180 VUT = y0, RSV = 450 and VTU = 200
y = 180 – 102 = 780 use this diagram to answer questions 21 and 22
2012/36 21. Find the value of x
M N A 20 B 45 C 65 D 135
112 0 Solution
129 0
x = 450 (corresponding angles)
22. Calculate the value of y
y A 20 B 25 C 45 D 65
37 0 Solution
P O
UVT + x0 = 1800 (sum of angles on a straight line)
In the diagram, MN//PO, PMN = 1120, UVT + 45 = 180
PNO = 1290, NOP = 370 and MPN = y. UVT = 180 – 45
Find the value of y. = 1350
A 510 B 540 C 560 D 680 Next, UVT + 20 + y0 = 1800 (sum of angles in )
220
135 + 20 + y0 = 180 24. Find the size of angle SRT
y0 = 180 – 155 A 850 B 950 C 1300 D 1450
y0 = 25 B. Solution
PRS = ONU (corresponding angles)
2008/45 PRS = 850
P
PRS + SRT = 180 (sum of angles on a straight line)
0
U 60 85 + SRT = 180
S SRT = 180 – 85
y0
= 950 B.
50 0 3x
Q 2007/20
2x
Find the value of x in the diagram
R
T
45 0 85 0
In the diagram, QPR = 600, PQR = 500,
QRS = 2x0 , SRP = 3x0, UQP = y0 and RS//TU.
Calculate y. x

A 1020 B 780 C 700 D 600


Solution 34 0

500 + 600 + (2x + 3x) = 1800(sum of angles in )


50 + 60 + 5x = 180
5x = 180 – 110 A 2810 B 2690 C 2010 D 1790
5x = 70 Solution
Let us label the diagram as:
x = 70/5 i.e 140
TQR = 2x (alternate angles) 85 0
45 0
TQR = 2  14 i.e 280 v 0 y0
z0
TQR + 50 + y0 = 1800 (sum of s on a straight line)
28 + 50 + y = 180
s0 x
y = 180 – 78
y = 1020 A.
t0
34 0
2014/23 – 24 ( Nov )
U

M N O
y = 450 (vertically opp. angles)
85 + y + z = 180 0 (sum of angles on a straight line)
0 85 + 45 + z = 180
35
z = 180 – 130
P = 500
v = 850 (vertically opp. angles)
t + 340 = 85 (v0) (alternate angles)
500
Q S t = 85 – 34
R
t = 510
T t + z + s = 1800 (sum of angles in )
In the diagram, TU and OQ are straight lines, MO//QS, 51 + 50 + s = 180
angle OPN = 350 and angle RQP = 500. s = 180 – 101
Use the diagram to answer questions 23 and 24 s = 790
s + x = 3600 (sum of s at a point)
23. Calculate the value of angle ONU 79 + x = 360
A 350 B 500 C 850 D 950 x = 360 – 79
Solution = 2810 A.
QOM = 500 (Alternate angles) 2005/30 (Nov)
ONP + 35 + 50 = 1800 (sum of angles in  PNO) From the diagram, find an expression for y in terms of x
ONP = 180 – 85 A y = 750 + 2x B y = 750 – x C y = 750 – 2x
0
= 950 D y = 75 + x
ONP + ONU = 1800 (sum of s on a straight line)
95 + ONU = 1800 y +20 0

ONU = 180 – 95
= 850 C.
y 2x + 10 0

221
Solution 2010/43 Neco Exercise 15.14
The vacant angle on the straight line where we have In the figure below, if x = 800 and z = 360, find y
y and 2x + 100 is y + 200. (Alternate angles) x0
y + (y +20) + 2x + 10 = 1800 (sum of s on a straight line)
y + y + 20 + 2x + 10 = 180
2y + 2x = 180 – 30
2y + 2x = 150
2y = 150 – 2x z0
Divide through by 2
y0
y = 75 – x B.
A 320 B 360 C 580 D 640 E 800
2006 /45 Neco (Dec)
In the diagram below, RSTV is a straight line, XS//YT, 2012/5a Neco Exercise 15.15
Q S
SYT = 440 , YST = 620. Find x0 + y0
X Y 70 0

44 0

m n 50 0
x0 P R
0 y0
62
In the above figure, PQR is a triangle and PQ//RS,
R S T V
find m and n
A 1500 B 1060 C 740 D 440 E 300
2006/19 (Nov) Exercise 15.16
Solution
Find the value of x in the diagram
y0 = 44 + 62 (exterior  = sum of two interior opp s)
y = 1060 x0
x = 440 (alternate angles)
Thus, x + y = 44 + 106
110 0
= 1500 A.
40 0
2015/33
Q
A 400 B 600 C 700 D 1500
2010/44 Neco Exercise 15.17
In the diagram below, CE = ED , CD // AB and
R
EAB = 580, find angle marked x
L O E
m 3n-20 0
2m n

P T S x
_____ _____ C
D
In the diagram, QT and PR are straight lines,
ROS = (3n – 200), POT = 2m, SOT = n, POL 58 0
= m and QOL is a right angle. Find the value of n. A B

A 350 B 400 C 550 D 600 A 320 B 580 C 610 D 640 E 1220


Solution
900 + m + 2m = 1800 (sum of angles on a straight line) 2014/49 Neco (Dec) Exercise 15.18
3m = 180 – 90 Consider the diagram below. ACE is a straight line, AB//CD,
3m = 90 BCA = 490 and ABC = 610. Find the angle marked z.
B D
m = 90/3 = 300
2m + n + (3n – 200) = 1800 (sum of angles on a straight line) 610
2m + n + 3n – 20 = 1800
2(30) + 4n – 20 = 180
60 – 20 + 4n = 180 z
49 0
4n = 180 – 40 A C E
4n = 140 A 490 B 610 C 700 D 1100 E 1310
n = 140/4 = 350 A.

222
BEARING Acute – angled Triangle
This topic deals with measurement of points on the
A
earth surface based on some principles of which part
has been discussed. The resulting shapes are mostly
triangles whose lengths and angles are used to solve for c b
the unknown ones. This task (i.e. of solving for
unknown angles and lengths) need some prerequisite C
knowledge. Among these are; a
B
- Element of plane geometry as treated in the
preceding unit. Properties
- Types of triangles and ways of finding their Three angles are less than 900
unknown angles and lengths.
i. e. A < 900, B < 900, C < 900
TYPES OF TRIANGLES (REVIEW)

Equilateral triangle Obtuse – angled Triangle


A A
c b

c
b
B a C
Properties
C
All sides are equal a = b = c B a
All angles are equal A = B = C
Properties
One of the angles ,  is greater than 900
Isosceles Triangle but less than 1800
A
c To find the unknown angles or lengths of the last two
b triangles (acute and obtuse-angled triangles) leads to sine
B C and cosine rule.
a The sine rule states that :
Properties _a__ = b__ = __c__
Two sides are equal b = c Sin A Sin B Sin C
Base angles are equal B = C The notations a, b & c and A, B & C are as applied to acute
and obtuse angled triangles above.
Scalene Triangle
A
Conditions for applying sine rule
(i) One length and all angles, or
(ii) Two side and an angle opposite one of the given
c b
sides.
(iii) Two angles and one side opposite one of the given
C angles
a
B Examples
( a ) QRS is a triangle with QS = 12m,
Properties  RQS = 300 and QRS = 450, calculate the length RS
None of the sides are equal a  b  c
Solution
None of the angles are equal A  B  C S

Right- angled Triangle 12m


P
300 450
Q R

30 + 45 + S = 180 (Sum of  s in a ∆)
0

r q S = 180 – 75
= 1050
Two angles and one side opposite one of the given angles
Q p R condition fulfilled hence sine rule will apply
Properties RS = 12
One of the angles is 900 i.e. Q Sin 30 Sin 45
223
Cosine Rule
RS = 12 Sin 30 Applying the usual notations in acute and obtuse angled
Sin 45 triangles
Cosine rule states that:
= 12 x 1/2 =6÷ 1 =6x 2 =6 2 a2 = b2 + c2 – 2bc cos A
1/ 2 2 1 b2 = a2 + c2 – 2ac cos B
c2 = a2 + b2 – 2ab cos C
0
(b) Given that the two angles of a triangle are 30 each
and the longest side is 10 cm. Calculate the length of Conditions for applying cosine Rule
each of the other sides. (i) Given two sides and the included angle.
Solution (ii) Given all the three sides of the triangle.
The 3rd angle should be1200 facing the longest side. Here the above stated rule will have to be adjusted
Since to make cos A or cos B or cos C the subject of the
30 + 30 +  = 180 (sum of  s in Δ) formula as the case may be i.e.
 = 1200 cos A = b2 + c2 – a2
A 2bc
c b cos B = a2 + c2 – b2
0
3
0 3
00
2ac
B C the others follow suit.
1
0cm
Examples
All angles and one side hence sine rule applies ( i ) Find /XZ/ in the triangle below
10 = b
sin 120 sin 30 X
b = 10 Sin 30 Since sine is positive in the 2nd quad.
Sin 60
2
m 0
1
20
1
= 10 x /2 = 5 x 2 = 10 3 cm Z
Y 1
m
3 /2 3 3
Solution
Two sides and the included angle; thus cosine rule applies here,
(c) In an acute-angled triangle PQR with PQ = 10m, /XZ/2 = 22 + 12 – 2 x 2 x 1 x cos 120
PR = 15m ,  PRQ = 400. Evaluate  PQR = 5 + 4 Cos 600 since Cosine is – ve in 2nd quad
Solution = 5 + 4 x 1/2
P
/XZ/2 = 7
/XZ/ = 7
10m 15m
(ii) In ∆ ABC below, find  BAC
0
A
40
Q R

Two sides and one opposite angle, thus we can apply 8


sine rule. 6
10 = 15
sin 40 sin Q
B C
sin Q = 15 sin 40 5
10 Solution
= 1.5 ( 0. 6428) Three sides of ∆ then ,the adjusted cosine rule will apply.
Sin Q = 0. 9642 cos A = c2 + b2 - a2
Q = sin – 1 0. 9642 2xcx b
= 74. 620 cos A = 82 + 62 - 52
2x 8x 6
= 64 + 36 – 25
96
cos A = 0. 7813
A = cos - 1 0. 7813
= 38. 620
 38 .60
224
Problems on sine and cosine triangles ( ii ) To find a
The sides relevant to 430 are opp and hypo (SOH)
2014/17 Neco ( Nov ) a
ABC is a triangle in which BAC = 750, /AB/ = 3cm sin 430 =
17
and /AC/ = 4cm. Calculate /BC/ correct to one decimal place
17 sin 430 = a
A 3.9cm B 4.0cm C 4.3cm D 5.0cm E 7.0cm
11.59cm = a
Solution
A 2009/5b (Nov)
P
0
75
4cm
3cm
6cm 8cm

B C
Applying cosine formula
since we have two sides and an included angle Q R
T
BC2 = 32 + 42 – 2  3  4 cos 750
= 9 + 16 – 6.212 In the diagram QPR = PTR = 900, /PR/ = 8cm
BC2 = 18.788 and /QP/ = 6cm. find /TR/
Solution
BC = 18.788
In PQR, by Pythagoras rule
= 4.3345 QR2 = 62 + 82
 4.3cm to 1 d.p ( C ) QR2 = 36 + 64
2006/ 45 NABTEB (Nov) QR2 = 100
Calculate the angle marked P in the diagram below, QR = 100 i.e 10 cm
given that AB = 4cm , AC = 6cm, ACB = 300 Next, Applying sine rule in PQR
A 10 6
=
sin 90 sin R
4cm 6cm 10 sin R = 6 sin 90
6 sin 90
sin R =
P 30 0 10
B C sin R = 0.6
A 48.6 0
B 40.0 0
C 30.0 0
D 24.00 R = sin– 1 0.6
Solution = 36.870
Applying sine formula In right – angled PTR,
4
=
6 the relevant sides to 36.870 are Adj and Hypo (CAH)
sin 30 sin P Cos 36.870 =
TR
4 sin P = 6 sin 30 8
6 sin 30 8  cos 36.870 = TR
sin p = 6.40cm = TR
4
sin p = 0.75 2006/47 Neco (Dec)
p = sin– 1 0.75
In ABC, a = 8cm, b = 12cm, C = 820. Find c
= 48.590
A 6.73cm B 13.45cm C 29.92cm
2005/ 10b NABTEB Counter example D 33.46cm E 50.92cm
The hypotenuse of a right – angled triangle is 17cm Solution
and one of the angles is 430, find the A
( i ) third angle (ii) side opposite the smallest angle
Solution
c 12cm

O
82 0
B C
a 17cm
8cm
To find c, we apply cosine rule
43 0 since we have two sides and an included angle
b c2 = 122 + 82 – 2  12  8 cos 820
( i ) Third angle i.e  c2 = 144 + 64 – 26.72
c2 = 181.28cm
 + 430 + 900 = 1800 (sum of s in )
 = 180 – 133 c = 181.28
= 470 = 13.46 cm B.
225
2009/2a Neco (Dec) 2012/31 Neco Exercise 15.21
Two sides of a triangle measuring 10cm and 15cm Find y in the diagram below
enclose an angle of 550. B
( i ) calculate the third side
( ii ) find the two other angles
y 5cm
Solution
A
49 0
C A
3cm
10cm b
A 4.01cm B 3.87cm C 3.78cm
55 0 D 2.87cm E 2.78cm
B C
15cm 2009/9b (Nov) Exercise 15.22
In triangle XYZ, /XY/ = 9cm, /XZ/ = 10cm
( i ) Applying cosine formula to find b
and YXZ = 750. Find /YZ/
since we have two sides and an included angle
b2 = 100 + 152 – 2  10  15 cos 550
= 100 + 225 – 172.07 TYPES OF BEARING
b2 = 152.93 Any given bearing angle is represented in two ways, namely
b = 152.93 - Compass bearing (cardinal points)
= 12.37cm - Three-digit bearing.
( ii ) Applying sine rule to find A
12.37 15 Compass bearing (cardinal points)
= This concept is based on cardinal points as can be obtained
sin 55 sin A
in any standard compass such as shown below.
12.37  sin A = 15 sin 55
sin A =
15 sin 55 N (North)
12.37
sin A = 0.9933
A = sin – 1 0.9933
= 83.360 W (west) E (East)
Applying sine rule to find C
12.37 10
=
sin 55 sin C
12.37 sin C = 10 sin 55 S (South)
10 sin 55
sin C = N
12.37
sin C = 0.6622 NW (North West ) NE ( North East)
C = sin – 1 0.6622
= 41.470 W E

2010/47 Neco Exercise 15.19


In the figure below, /BC/ = 2cm, /AB/ = 3cm and SW( South West) SE (South East)
ACD = 1500, what is the value of sin A? S
A
The starting point are based on North or South which are
measured to either East or West. You will agree with the
3cm author that from North to East or North to west, we can only
150
0 cover between 00 and 900. The same applies to South to East
or South to West. Below are examples of compass bearing
D C 2cm B

3
(i) N 300 E (ii) N 150 W (iii) S 500E (iv) S 550 W
2 1 1
A 3 B C D E
2 3 2 3 N ( ii ) N
(i) 0
2014/42 Neco Exercise 15.20 30
A vehicle makes a journey that forms a triangular shape 150
such that the distance CD = 40km, AD = 50km, E W
ACD = 1150 and D = 250. Find the third side of the
triangular shape.
A 43.32km B 32.14km C 32.31km
D 20.20km E 18.65km

226
( iii ) ( iv ) N
( ii ) N
E W E 0
130
130
0
=
500 550 E = SS50
5 EE
0 0

S S 50
0

Three digit bearing S


This type of bearing bases its starting point from the (iii)
North only; then moves in a clockwise direction. The N N
0
movement is between 00 and 3600. 60
Below are examples of 3 - digit bearing
=
(i) 0100 (ii) 0800 (iii) 1500 (iv) 2500 (v) 3200 W = N 600 W
3000
The concept of quadrant is important here though to be 3000
numbered in clockwise direction instead of the usual
anti – clockwise numbering thus we have the format as:
(b) Convert the following compass bearing
4th Quadrant 1st quadrant to a 3 digit bearing
2700 and 3600 00 and 900 (i) N 650 E (ii) S 400 W
3rd quadrant 2nd quadrant Solution
1800 and 2700 900 and 1800 N
N
Solution diagrams (i) 650
N =
N N E 65
0
( iii )
(i) ( ii )
0
0
150
0
= 065
0
010 080

N N
N (ii)
N
(v)
( iv )
W E = 2200
0
250 i.e 40 + 180
0
320 400
2200

S
Relationship between three digits
1997/40
and compass bearing Express the true bearing of 2500 as a compass bearing.
Any three – digit bearing can be converted to a compass A. N 200 E B. S 200 E C. N 200 W
bearing and vice versa. 0
D. S70 W 0
E. S 70 E
Examples Solution
( a ) Covert the following 3 digit bearing to compass
bearing.
(i) 0300 (ii) 1300 (iii) 3000
Solution
N
N
0
(i) 30
=
The required angle is 
30
0
E = N30 E
0
But  = 250 – 180
= 700
Also  is between South and West
Hence  = S 70 W (D)
227
2001/26 Exercise 15.23 2010/49 Neco
The bearing S 400 E is the same as The bearing of a point A from a point B
A. 0400 B. 0500 C. 1300 D. 1400 is 0420. Calculate the bearing of B from A
A 2280 B 2220 C 1380 D 480 E 420
2002/32 (Nov) Exercise 15.24
Solution
The bearing S 500 W is the same as N
A. 0500 B. 1300 C. 1400 D. 2300
A
2004/48 Neco Exercise 15.25
The bearing S 400 W is the same as
N b0
A 0400 B 0500 C 1400 D 2200 E 2300
042 0

B
REVERSED BEARING
If the bearing of a point A is described from another
point B. Then, the bearing of B can be described from A bo = 420 (Alternate angles)
based on the first description. Bearing of B from A = 1800 + b0
2012/47 Neco = 180 + 420
If P, Q, R are points such that the bearing of Q from P = 2220 B.
is 3000 and the bearing of R from P is 1200, find the 2008/33 Neco (Dec)
bearing of Q from R Find the bearing of a point X from Y. If the bearing
A 0600 B 1200 C 1500 D 1800 E 3000 of Y from X is 1100.
Solution A 0700 B 1100 C 2500 D 2900 E 3100
N Solution
N

110 0
X
Q
N
N
x0
P
120 0
N Y
a0
300 0 y0
a0
R a0 = 1100 (Alternate angles)
0
b Bearing of X from Y = 180 + a0
= 180 + 110
x0 + 3000 = 3600 (sum of angles at a point) = 2900 D.
x0 = 360 – 300 = 600 2011/12
y0 = x0 i.e 600 (vertically opposite angles) Esther was facing S200W. She turned 900 in the clockwise
Alternatively direction. What direction is she facing?
y0 + 1200 = 1800 (sum of angles on a straight line) A N 700 W B S 700 E C N 200 W D S 200 E
y0 = 180 – 120 = 600 Solution
a = y i.e 600 (Alternate angles)
0 0 N
The bearing of Q from R is b0
a0 + b0 = 3600 (sum of angles at a point) NW

b0 = 360 – a0 200
b = 360 – 60 = 3000 W 900 E

2013/48 Neco 70 0
0
20
The bearing S 400E is the same as
A 0400 B 0500 C 1300 D 1400 E 1800
Solution S200 W
N S

Her new position is NW and not WN and we can not


say N 200 W here; it would have been correct if we had
W200N.
W E
From north to west is 900 ,
40
0 200 is out of it is remaining 700
The new position is N 700 W (A)
S400E
S
The compass bearing is = 1800 – 40
= 1400 ( D )

228
2009/ 26 – 27 Bearing of X from Y is  +  + 1800
Use the diagram below to answer questions 26 and 27
N 700 + 300 +  = 1800 (sum of s in )
 = 180 – 100
= 800
1200 Y  + (50 + 90 ) = 1800 (sum of interior opp. s)
0

N  = 180 – 140
= 400
50
m Thus bearing of X from Y is 80 + 40 + 180 = 3000 A.
0
60
X 2011/48 Neco Exercise 15.26
26. Find the bearing of X from Y A from Q is 120km from a town B in the direction 0500.
A 3000 B 2400 C 1200 D 600 What is the bearing of B from Q?
Solution A 0400 B 0500 C 1300 D 2300 E 3100
Bearing of X from Y = 360 – 120
= 2400 B 2012/26 Exercise 15.27
N
27. If /XY/ = 50m, how far east of X is Y?
A 25.5m B 40.6m C 40.8m D 43.3m
Solution Q
N
72 0
N N
0
120 Y
500
N
m
50 P R
The position of three ships P, Q and R at sea are illustrated
60 0 in the diagram. The arrows indicate the North direction. The
30 0
X bearing of Q from P is 0500 and
d
Using 300 in the resulting right – angled triangle PQR = 720. Calculate the bearing of R from Q
the relevant sides are Adj and Hypo (CAH) A 1300 B 1580 C 2220 D 2520
d
Cos 300 =
50
50 cos 300 = d Problems on bearings I
43.30m = d (D) 2006/15 Neco
2010/46 A village Y is 15km from a point X on a bearing
X N
of 0250. Another village Z is 20km from X on a
300 bearing 1150. Calculate the distance YZ.
A 35km B 25km C 20km D 15km E 5km
Solution
Y N

60 0 50 0 Y
W E
O m
N k
15
25 0
S
X
In the diagram, WOX = 600, YOE = 500 115 0 N
and OXY = 300.What is the bearing of X from Y?
2 0km
A 3000 B 2400 C 1900 D 1500
Z
Solution
Resulting diagram: By Pythagoras rule: YZ2 = 152 + 202
X
N
YZ2 = 225 + 400
300
YZ2 = 625
N
YZ = 625 = 25km (B)
O Y
70 0 O 2005/15 Neco
60 0 50 0 A boat sails 24km from a port X on a bearing of 0650
W E
O and thereafter 10km on the bearing of 1550. What is the
distance of the boat from X?
S A 34km B 30km C 26km D 24km E 14km
229
Solution 2007/4
N
Y is 60km away from X on a bearing of 1350. Z is 80km
155 0 away from X on a bearing of 2250. Find the:
(a) distance of Z fromY; (b) bearing of Z from Y.

10
km Solution

km
N 24 N
N

65 0 Boat 135 0
X 225 0
X 60
km
Resulting diagram km N
N 80
N
Y
155 0
Z
65 0 250
10
km
km
N 24 N Resulting diagram
N

65 0 B 135 0
(Boat) X
X 60
90 0 km
km N
We are to find XB N 80
O
By Pythagoras rule: XB2 = 242 +102 O Y
XB2 = 576 + 100
XB2 = 676 Z
XB = 676
( i ) Applying cosine rule
= 26km ( C ) since we have two sides and an included angle
2014/25 (Nov) ZY2 = 802 + 602 – 2  80  60 cos 90
A ship sails 5km due west and then 7km due south. ZY2 = 802 + 602 (reduces to Pythagoras rule)
Find; correct to the nearest degree its bearing from the = 6400 + 3600
original position. ZY2 = 10,000
A 0550 B 0560 C 2150 D 2160 ZY = 10,000 = 100km
Solution
N N ( ii ) The bearing of Z from y = 360 – ( + )

W
5km 1350 +  = 1800 (sum of interior opp. s)
E W E
O  = 180 – 135 = 450
S S Applying sine rule to find 
100 80
7km y
=
sin 90 sin 
100 sin  = 80 sin 90
80 sin 90
The bearing from the original position = 360 – ( +900) sin  =
100
By Pythagoras rule: y2 = 72 + 52 sin  = 0.8
y2 = 49 + 25
 = sin– 1 0.8
y2 = 74
= 530
y = 74 = 8.6km Thus the bearing of Z from Y = 360 – (53 + 45)
 can be gotten by sine rule = 360 – 98 = 2620
8.6 7 2006/9a Neco counter example
=
sin 90 sin  A hunter walked 250m on a bearing 0420. Calculate, correct
8.6 sin  = 7 sin 900 to the nearest kilometer, the
7 sin 90 ( i ) vertical height through which he has moved;
sin  = ( ii ) horizontal distance covered
8.6
Solution
sin  = 0.8140 N
 = sin– 1 0.8140
= 54.490 B C
Thus bearing from original position
0m
= 360 – ( + 90) 25
= 360 – (54.49 + 90) N 0m
0 42 0 25
= 360 – 144.49 42 Resulting
= 215.510 to A
≈ 2160 to nearest degree D.
230
( i ) We are to find AB; 2014/26 Neco (Dec) Exercise 15.31
the sides relevant to 420 are Adj and Hypo (CAH) A boat sails 8km from a port Q on a bearing of 0550 and
cos 420 =
AB then 15km on a bearing of 1450. What is the distance of the
250 boat from Q?
250 cos 420 = AB A 7km B 11km C 15km D 17km E 23km
AB = 185.79km
2006/33 (Nov) Exercise 15.32
 186km to nearest km A man walk 11km due north from P to Q and then 6.5km
( ii ) To find BC; due east to R. Given that x is the bearing of R from P, which
The sides relevant to 420 are Opp and Hypo (SOH) of the diagrams represents the statements?
BC N N
sin 420 = 6.5km
250 6.5km
R Q Q R
250 sin 420 = BC
BC = 167.28km 11km
11km
 167km to nearest km A B
x x

2005/10 (Nov)
P P
Points P and Q are respectively 24m north and 7m east
of point R. What is the bearing of Q from P to the N N
11km
nearest degree? R
11km
Q Q R
A 0160 B 0500 C 0740 D 1640 6.5km
Solution x 6.5km x
D
N C
P P

O 2014/18 Neco Exercise 15.33


P
Three towns P, Q and R are situated such that
O PQ = 15km and PR = 20km. The bearing of Q from P
is 0300 while the bearing of R from P is 3000. Calculate the
24m distance RQ
A 5km B 10km C 20km D 25km E 35km
R 2009/25 (Nov) Exercise 15.34
E
7m Q In the diagram, the bearing of X from Y is 0600.
We are to find  with the aid of  The bearing of Z from X is 1500. Find /YZ/
X 150 0
The sides relevant to  are opp and adj (TOA)
7
tan  = m
24 0k
50

60
0k

 = tan– 1 0.2917
m

0
0

 = 16.260
06

But  +  = 1800 (sum of angles on a straight line) Y


Z

 = 180 – 16.26 A 7.81km B 78.10km C 237.00km D 781.02km


= 163.740
2007/48 Neco Exercise 15.35
 1640 D.
The bearing of two points Y and Z from a point X
2009/49 Neco(Dec) Exercise 15.28 are 0600 and 1500 respectively. If /XY/ = 12m and
The bearing of two points Q and R from a point P /XZ/ = 5m, find the distance /YZ/
are 0300 and 1200 respectively. If /PQ/ = 24m and A 13m B 11m C 9m D 7m E 5m
/PR/ = 7m. Find the distance QR
2005/18 Exercise 15.36
A 7m B 15m C 19m D 21m E 25m
From a point R, 300m north of P, a man walks
2014/6b NABTEB Exercise 15.29 eastwards to a place Q which is 600m from P. Find the bearing of P
A man starts at A and walks 2km on a bearing of 0170. from Q correct to the nearest degree
He then walks 3km on a bearing of 1070 to C. What is A 0260 B 0450 C 2100 D 2400
the bearing of C from A? 2004/22 NABTEB Exercise 15.37
2006/48 Neco (Dec) Exercise 15.30 Three observation points P, Q and R are such that Q is due East of
P and R is due north of Q. If /PQ/ = 5km and
A man who could not trace his route was directed on the
/PR/ = 10km, find /QR/
following instructions. Walk 6km due East and 8km due A 5.0km B 7.5km C 7.6km D 8.7km
North. Find his bearing from his initial position, correct
to 1 decimal place. 2007/31 Exercise 15.38
A 18.550 B 26.550 C 36.90 D 53.10 E 126.90 Q is 32km away from P on a bearing 042 0 and R is 25km from P
on a bearing of 1320. Calculate the bearing of R from Q
A 1220 B 1840 C 1900 D 2260
231
Problems on bearings II We are to find AH :
Note that ACH = 300 (alternate s).
2001/12a (Nov) The sides relevant to 300 in ACH are opp and hypo (SOH)
AH
094
0 Sin 300 =
A 15km
7.3
B 7.3 sin 30 = AH
245 0 3.65 km = AH

2010/9
N

C N
C
(a) In the diagram, A and B are two ports 15km apart. 50 0
B is on bearing 0940 from A and C is a ship on the sea. A
The ship is seen from A on bearing 1500 and from B on
bearing 2450. N
( i ) Calculate, correct to one decimal place, the 40 0
distance of the ship from A
B
( ii ) How far east of A is C?
Solution In the diagram, /AB/ = 8km, /BC/ = 13km, the bearing of A
Resulting diagram from B is 3100 and the bearing of B from C is 2300.
94 0
Calculate, correct to 3 significant figures
(a) the distance AC; (b) the bearing of C from A
A
0
15km d0 B
(c) how far east of B, C is
150 56 0
Solution
e0 245 0
The resulting diagram is:
N
f0
N
C
C
Since we are given only one distance 15km, it is O 50 0
obvious we shall be working with sine rule; A O N
so we target the various angles in the resulting triangle 50 0

m
0
100

k
( i ) CAB = 150 – 94 i.e 560

13
8k
m 50 0
d0 + 940 = 1800 (sum of interior opp. s) 500
40 0
d0 = 180 – 94 i.e 860 E
d + e + 2450 = 3600 (sum of s at a point)
0 0 B
86 + e0 + 245 = 360 Breakdown:
e0 = 360 – 331 = 290 ABN = 360 – 310 i.e 500 OR 90 – 40 = 500
56 + e0 + f 0 = 1800 (sum of s in )
0
CBN = 500 (Alternate s)
560 + 290 + f 0 = 1800 angle at A = 500 (Alternate s)
f 0 = 180 – 85 = 950 (a) We can find AC by cosine rule since there are two sides
15 AC and included angle
=
sin 95 0 sin 29 AC2 = 82 + 132 – 2  8  13 cos100
15 sin 29 = AC sin 95 = 64 + 169 – [208  (– 0.1736) ]
15 sin 29 = 233 + 36.11
AC = AC2 = 269.11
sin 95 0
7.272 AC = 269.11 = 16.40km  16.4km to 3 s.f
= = 7.300 km to 1 d.p
0.9962 (b) The bearing of C from A
(ii)  = 180 – (50 + )
we get  by sine rule
1500
16.40 13
A H =
B sin 100 sin 
30 0 16.40 sin  = 13 sin 100
13 sin 100
S 7.3 sin  =
km 16.40
30 0 sin  = 0.7806
 = sin – 1 0.7806 = 51.320
C
  = 180 – (50 + 51.320 )  78.70 to 3 s.f
232
(c) In BCE, the sides relevant to 500 are opp and hypo (SOH) sin 600 =
AE
sin500 =
BE 50
13 50 sin 600 = AE
13 sin 50 = BE 43.33km = AE
BE = 9.959 km 2005/6 Neco
 9.96 km to 3s.f Three points, P, Q and R are on a horizontal plane. Point Q is on a
bearing 1200, at a distance 80m from point P. Point R is on a
2005/7 NABTEB bearing of 0450 and a distance of 150m from point Q.
The bearings of point P and Q from A are 0450 and 1200 (a) Calculate, correct to 2 significant figures the,
respectively. If the distance AP is 80km and AQ is ( i ) distance between point P and point R;
50km, calculate ( ii ) bearing of point P from point R
( a ) the distance between P and Q to 3 significant (b) How far west of R is Q to 2 decimal places?
figures (c) Find the perpendicular distance from Q to PR to 2 d.p
( b ) the bearing of Q from P to the nearest degree Solution
( c ) how far east of A is Q
R
Solution N

0
P 120 0
45
P
m O
80k

m
a0

150
45 0 45
0
80m
A
75 0 Q
120 0
Resulting diagram
50 k N
m
Q
w R
N
( a ) PQ = 80 + 50 – 2  80  50 cos 75
2 2 2 O O
0
PQ2 = 6400 + 2500 – 2070.55 120 0 45
PQ2 = 6829.45 P
T

m
PQ = 6829.45 = 82.64km  82.6km to 3 s.f N

150
0
80m 45
( b ) the bearing of Q from P a0
82.64 50
= Q
sin 75 sin 

sin  =
50 sin 75 Breakdowns:
82.64 NQR = QRT i.e 450 (Alternate angles)
sin  = 0.5844 1200 + a0 = 180 0 (sum of interior opp. s)
 = sin – 1 0.5844 a0 = 180 – 120 i.e 600
= 35.760 Thus PQR = a0 + 450 i.e 1050
Note that : a = 45 – 35.760
0
a ( i ) PR can be gotten using cosine rule
= 9.240  90 since we have two sides and an include angle
Bearing of Q from P = 180 + 90 i.e 1890 PR2 = 802 + 1502 – 2  80  150 cos 1050
(c) how far east of A is Q i.e AE = 6400 + 22500 – 24000(– 0.2588)
= 28900 + 6211.2
P PR2 = 35111.2
N 360 a 0 PR = 35111.2 = 187.38m  190m to 2s.f
a (ii) The bearing of P from R is =  + 450 + 1800
A 750 E we find  by sine rule
187.38 80
=
a0 sin 105 sin 
50k b
0
m 187.38 sin  = 80 sin 105
Q 80 sin 105
sin  =
187.38
(75 + 36) + b0 + a0 = 180 ( Sum of angles in triangle) sin  = 0.4124
180 – (75 + 36) = b0 + a0
 = sin– 1 0.4124
69 = b0 + a0 but a0 = 90
 = 24.360
69 = b0 + 90
Thus bearing = 24.36 + 450 + 180
69 – 90 = b0 i.e b = 600
The side relevant to b0 in AEQ are opp and hyp (SOH) = 249.360  2500 to 2 s.f

233
(b) Our attention here will be on QOR = 324 + 100 – 360(– 0.6947)
and we are to find WR = 424 + 250.092
The sides relevant to 450 are opp and hypo (SOH) AB2 = 674.092
Sin 450 =
WR AB = 674.092
150 = 25.96 km
150 sin 45 = WR  26 km to the nearest km
106.066m = WR ( ii ) Bearing of B from A =  + 490 + 1800
WR  106.07m to 2d.p But  can be gotten by sine rule
(c) We are to find QO in QOR. 25.96 18
=
 QOR = 180– (45 + 24.36) sum of s in a  sin 134 sin 
= 110.640 25.96 sin  = 18 sin 134
QO 150 18 sin 134
By sine rule : = sin  =
sin 24.36 sin 110.64 25.96
150 sin 24.36 sin  = 0.4988
QO =  = sin– 1 0.4988
sin 110.64
= 29.920  300 to the nearest degree
QO  66.11m to 2 d.p
 Bearing of B from A = 300 + 490 + 1800 = 2590
2006/6 Neco (Dec)
Two cyclists A and B leave a town C at the same time. dis tan ce AB
(iii) Speed =
cyclist A speeds at the rate of 5 km/h on a bearing of time
0490 and B at the rate of 9km/h on a bearing of 2750. 25.96
= = 6.49 km/hr
After two hours, calculate: 4
( i ) how far apart they are, correct to the nearest km, 2013/47 Neco
( ii ) the bearing of B from A to the nearest degree and A cyclist on training cycles for 4 hours at an average speed
( iii ) the speed at which A will get to B in 4 hours. of 2 km/hr from a point P on a bearing of 1350 to another
Solution point Z, then to another point X, 12km on a bearing of 0600.
N Calculate the distance /PX/ correct to two decimal places
N A 12.14km B 15.45km C 15.46km
B
D 16.05km E 19.55km
Solution
A N
N N
9k

h
m

49 0 X
m/
/h

5k

C 275 0 P
N
dis tan ce
Speed = 105 0
mk

time 8k
12

m
Distance = speed  time 45 0 600
Distance BC = 9  2 i.e 18km
Z
Distance AC = 5  2 i.e 10km
Resulting diagram: Breakdown:
N Distance PZ = speed  time
N = 2  4 i.e 8km
B 1350 + PZN = 1800 (sum of interior opp.  s)
PZN = 180 – 135 i.e 450
O A PX can be gotten by cosine formula
N 490 since we two sides and an included angle
1340
18

PX2 = 82 + 122 – 2  8  12 cos 1050


km

km

850 490 S = 64 + 144 – 192 (– 0.2588)


10

= 208 + 49.6896
C PX2 = 257.6896
Breakdown: PX = 257.6896  16.05km to 2 dp
NCB = 3600 – 2750 (s at a point)
= 850 2014/11b Neco (Dec)
NCA = CAS (alternate s) Two boats take off from a harbour at the same time. The first
( i ) Distance AB can be gotten by cosine formula travels at 12 km/h on a bearing 0770 while the second travels
since we have two sides and an included angle at 18km/h on a bearing of 1430. Calculate their distance apart
AB2 = 182 + 102 – 2  18  10 cos 1340 after 3 hours

234
Solution Distance AC can be gotten by cosine formula
H for Harbour, F for first and S for second travel since we have two sides and included angle
AC2 = 7502 + 4002 – 2  750  400  cos 1270
F = 562500 + 160000 – 600000 (– 0.6018)
N r
m/ h
77
0
12 k AC2 = 722500 + 361080
AC2 = 1083580
143 0 H AC = 1083580
= 1040.95km
18 km Bearing from the airport i.e bearing of C from A
/hr which is . It can be gotten by sine rule
S
1040.95 400
=
dis tan ce sin 127 sin 
Speed =
time 1040.95 sin  = 400 sin 127
Distance = speed  time 400 sin 127
sin  =
Distance H F = 12  3 i.e 36km 1040.95
Distance HS = 18  3 i.e 54km sin  = 0.3069
F HS = 1430 – 770 i.e 660  = sin – 1 0.3069
Resulting diagram: = 17.870
2014/12b
N
F The bearing of Q from P is 1500 and the bearing of P from R
m
77
0 3 6k is 0150. If Q and R are 24 km and 32km respectively from P:
( i ) Represent this information in a diagram;
0
66 ( ii ) Calculate the distance between Q and R,
H
( iii ) Find the bearing of R from Q, correct to the
5 4kk
nearest degree
m Solution
S
If the bearing of P from R is 0150 then,
Thus distance apart FS can be gotten by cosine rule the angle at P + 150 = 1800.
since we have two sides and an included angle Reason ¨sum of interior opp angles
FS2 = 362 + 542 – 2  36  54 cos 660 Thus angle at P is 1650 as shown which enabled us to locate
= 1296 + 2916 – 1581.39 R position
2 N
FS = 2630.61
FS = 2630.61 P
165
0 150 0
 51.29km
2014/8 Neco (Dec) 450
24

m
An aeroplane leaves an international airport and flies 2k N
km

3
due north for 11/2 hours at 500km/hr. It then flies for N
a0
30 minutes at 800km/hr on a bearing of N530E. 0 O
15 Q
Calculate its distance and bearing from the airport.
Solution R
C
N m RPQ is 360 – (165 + 150) = 450
4 00k
53 0
QR2 = 322 + 242 – 2  32  24 cos 45
W
B 370
E QR2 = 1024 + 576 – 1086.12
127
0 QR2 = 513.88
QR = 513.88 = 22.669km  22.67km to 2d.p
750km

(iii) The bearing of R from Q = 360 – ( + a0)


N a0 + 1500 = 1800 (sum of interior opp. s)
O a0 = 180 – 150 i.e 300
 can be gotten by sine rule
A 22.67 32
=
Breakdown: sin 45 0 sin 
Distance = speed  time 22.67 sin  = 32 sin 450
Distance AB = 500  1.5 i.e 750km 32 sin 45 0
sin  =
Distance BC = 800  0.5 i.e 400km 22.67
Note: 30 min is 0.5 hour sin  = 0.9981
CBE = 90 – 53 i.e 370  = sin– 1 0.9981 = 86.470
235
Thus bearing of R from Q = 360 – (86.47 + 300) Solution
= 360 – 116.47 Resulting diagram
= 243.530 C
 2440 to the nearest degree
2011/10b
A cyclist starts from a point X and rides 3km due west
to a point Y. At Y, he changes direction and rides 5km N 58m
north – west to a point Z. 71m
N
( i ) How far is he from the starting point, correct to the
nearest km? B 28 0 O A
( ii ) Find the bearing of Z from X to the nearest degree
D 96m
Solution
Z S S
The bearing of C from A =  + 2700
But  can be gotten from ACD by cosine rule
since we have all sides without any angle
N
712 = 582 + 962 – 2  58  96 cos 
5k
m

450 N
2  58  96 cos  = 582 + 962 – 712
O X
W E 58 2 + 96 2 − 712
Y East 3km West Cos  =
2  58  96
S S 3364 + 9216 − 5041
Cos  =
Note: North – west implies 45 as shown 0 11136
 ZYX = 450 + 90 i.e 1350 Cos  = 0.6770
( i ) ZX can be gotten by cosine formula  = cos– 1 0.6770
since we have two sides and an included angle  = 47.390
ZX2 = 32 + 52 – 2  3  5 cos 135 Bearing of C from A = 47.390 + 2700
= 9 + 25 – 30(– 0.7071) = 317.390
= 34 + 21.213 = 55.213  3170 to the nearest degree
ZX = 55.213 = 7.43km  7km to the nearest km (b) Distance BD can be gotten from AB in ABC as:

( ii ) Bearing of Z from X = 2700 +  C + 280 + 47.39 = 1800 (sum of s in )
we solve for  using sine rule 

7.43 5
C = 180 – 75.39
= = 104.610
sin 135 sin 
Applying sine rule
7.43 sin  = 5 sin 135 AB 58
5 sin 135 =
sin  = sin 104.61 sin 28 0
7.43
AB sin 280 = 58 sin 104.610
sin  = 0.4758
58 sin 104.610
 = sin – 1 0.4758 = 28.410 AB =
Bearing of Z from X = 270 + 28.41 sin 28 0
= 298.410  2980 58  0.9677
AB =
2005/7 (Nov) 0.4695
C AB = 119.55m
Thus BD = 119.55 – 96
= 23.55m  24m to the nearest metre
2009/3a Neco (Dec)
N 58m Point Q is on a bearing 0600 from P, point R is due west of
71m P, If the distance PQ is 100m and PR is 120m, find /QR/
Solution
28 0
B D A Q
96m

In the diagram, A is a house which lies due east of a


mosque B. Feeder roads run from A to C and from C to 0
150
0m

B. A foot path leads from C to the point D to AB. The


10

0
60
distance between these places are measured as follows:
R
/AC/= 58m, /CD/ = 71m, /AD/ = 96m and ABˆ C = 280, 120m P
calculate: (a) The bearing of C from A, correct to the
nearest degree (b) The distance BD, correct to the QR2 = 1202 + 1002 – 2  120  100 cos 150
nearest metre. = 14400 + 10000 – [2  120  100  (– 0.8660)]
236
QR2 = 24400 + 20784 2009/5b Neco Exercise 15.39
QR2 = 45184 A man travels from station A on a bearing of 0500 to
QR = 45184 station B, a distance of 12km. He then leaves to station C,
= 212.57m 10km away from B on a bearing of 1100. How far is he from
his starting point? (Answer to 2d.p)
2006/13b (Nov) counter example 2010/8a Neco Exercise 15.40
A point P is 40km away from point Q and on a bearing R
of 0330. Another point R is 65km from Q and on a
bearing of 1250. Calculate, correct to the nearest whole
number, the
( i ) distance between P and R; P

km
( ii ) bearing of P from R

250
Solution 45 0
20 30 0
P 0k
m
N Q
km
40
33 0 The above diagram represents the flight of an aeroplane from
city P on a bearing of S450E to another city Q a distance of
0
125 Q 200km. It then flew a distance of 250km on a bearing of
N300E from city Q to city R. If it flew back directly to city P,
65km Calculate the ( i ) total distance covered, correct to there
R significant figures ( ii ) bearing of city R from city P, correct
to the nearest degree.
Resulting diagram
2006/13b Exercise 15.41
P Y
330 a0
N km
40

32
km
33 0 O
km
50

92 0 X
Q a0 312
0

O 0220
65km
R
0 Z
b
The diagram shows the positions of three points X, Y and Z
on a horizontal plane. The bearing of Y from X
( i ) PR can be gotten by cosine rule
is 3120 and that of Y from Z is 0220. If /XY/ = 32km and
since we have two sides and an included angle
/ZY/ = 50km. Calculate, correct to one decimal place:
PR2 = 402 + 652 – 2  40  65  cos 92
( i ) /XZ/ ; ( ii ) the bearing of Z from X
PR2 = 1600 + 4225 – 2  40  65  (– 0.0349)
PR2 = 5825 + 181.48 2014/8c NABTEB (Nov) Exercise 15.42
PR2 = 6006.48 A ship steams 4km due North from a point and then 3km on
PR = 6006.48 = 77.5km a bearing of 0400. How far is the ship from the point?
 78km to nearest whole number 1978/5 Exercise 15.43
(ii) The bearing of P from R is  + b0 +1800 A man sets out to travel from A to C via B. From A he
b0 = 1250 ( 330 + 920) Alternate angles travels a distance of 8km on a bearing N300E to B. From B
We find  by sine rule he travels a further 6 km due east.
77.5 40 ( a ) Calculate how far C is
= ( i ) north of A ( ii ) east of A
sin 92 sin 
( b ) Then, calculate the distance AC,
77.5 sin = 40 sin 92 correct to1 decimal place.
40 sin 92
sin = 2007/3 Neco Exercise 15.44
77.5
A hunter walks from one forest X on a bearing of 0800 to
 = sin– 1 0.5158
another forest Y which is 40km away. From there, he went
 = 31.050 another forest Z a distance of 50km on a bearing of 2200 in
The bearing of P from R is 31.050 + 1250 +1800 search of an antelope which he shot. Calculate:
= 336.050 ( i ) distance of Z from X to the nearest kilometer
 3360 to nearest whole number ( ii ) bearing of Z from X to the nearest degree

237
` CHAPTER SIXTEEN Example 1
Find the angular distance between
A (400 N, 650E) and B (350 S, 650E)
Latitude & Longitude
Solution
This is one of the theory based areas to be tested in N
certificate exams. The formula of the length of an arc of 650
a circle plays a crucial role here. 0
A 40 N

O
r r 00
D
A B 350S
B
circle with centre O
 S
Length of arc AB = × 2 r
360
The Earth is spherical in shape. Any cut or slice through Angular distance = 400 + 350
it horizontally (Latitude) or vertically (Longitude) is = 750
circular in shape. To find the distance between any two Note that we add when the points are in different
points on such slice we have to use the above- hemispheres i.e South and North.
mentioned formula
N Example 2
Greenwich
meridian
Find the angular distance between X (600N, 200W )
and Y ( 400 N, 200W)
Equator Solution
W E N
X
X 600N
Y 400N Y
R
0
S Z
0
The measurement of any point on the latitudes either to 600 0
40
the east or west must start from the Greenwich Z R
meridian; while that of longitude either to south or north
must start from the equator - please note the difference. 200W
Any point on the earth surface is the intersection of S
(latitude, longitude) like in co-ordinates (x,y) Angular distance = 600 – 400 = 200

Great circles Small circles


2010/ 32 Neco
X and Y are two places on the same circle of latitude 800N,
X is on longitude 790W and Y is on longitude 110E. What is
the angular difference between X and Y?
A 600 B 690 C 800 D 900 E 1700
Solution
N
79 0 W

80 0N
X Y
Units of Measurement 11 0 E
Degrees and seconds are the units of measurement.
Latitude is from 00 to 900 south or North ; while W E
790 110
Longitude is from 00 to 1800 East or West
0 300E y
300W N x
600W 600E 800N0
60 N
0
30 N S
0
W 0
E 300S Angular difference between X and Y = 79 + 11
600S = 900 (D)
0
80 S Reason X and Y are on different sides of the globe west & east

Longitudes S Latitudes

238
Distance along great circles 
=  2  R
360
Distance along any of the longitudes and distance along 18 22
the equator only among the latitudes are called distance =  2  6370
360 7
along great circles. The formula used is = 2002km (B)

Lengths of arc = x 2R
360 2008/24 NABTEB (Nov)
Where R is the radius of the earth. Find the distance between A(650N, 30 E) and B(250S, 30E)
along their common meridian on the earth’s surface
2005/36 – 37 NABTEB (Use  = 22/7, R = 6370km)
Y and Z are two places on the surface of the earth along A 10010Km B 12460Km C12590Km D 13085Km
the meridian such that Y (lat 70N, long 65W) and Solution
Z (lat 38S, long 65W) N

36. Calculate the sectorial angle of YZ A


A 65 0 N
A 32 B 38 C 70 D 108
Solution
N
65 0 W 650
W E 250
700 N Y W
Y 25 0 S
B B
70 0
W E 0
38
0
38 S S
Z Z 
Distance AB (Great circle) =  2  R
360
S Here  is 65 + 25 i.e 900 since they are N and S
Sectorial angle of YZ = 70+38 90 22
=  2  6370
= 108 (D) 360 7
= 10010km (A)
37. Calculate the distance YZ if the radius of the earth
is 6400km 2007/6c NABTEB
A 12800km B 8960km C 3840km D Two places on the equator are 7900km apart measured along
the equator. Find the difference in their longitudes.
1138km
(Take R = 6370km and  = 3.14)
Solution
 Solution
Distance YZ  Distance along a great circle = 2R N
360
108
=  2    6400
360
= 3840km ( C )
W E

2011/33 Neco
An air craft flew from city A (lat 00, long 00) to another
city B (lat 00, long 180) at an average speed of
1000km/hr. Calculate the distance travelled to the S

nearest km, (Take  = 22/7, R = 6370km) Distance along the equator is in a great circle =  2R
A 2000km B 2002km C 2100km D 2200km E 2220km 360

Solution 7900 =  2  3.14  6370
N 0
18 E 360
7900  360
= 
2  3.14  6370
71.090 = 
W 00 E
2015/39 Neco Exercise 16.1
A B Two places on the same meridian have latitudes 100S and
530N.What is their distance apart measured along the
meridian? (Take  = 22/7, R = 6370km)
S A 7000km B 7040km C 7100km
Distance travelled along lat 00 is the equator D 7300km E 7400km
and that is distance along great circle

239
Distance along small circles
(Distance along parallel of latitudes) 2010/23 NABTEB
This is any distance along the latitudes except the Find the distance measured along the parallel of latitude
equator. between two places with the same latitude, 180S and with
longitudes 960E and 570E
N A 4020km B 4120km C 4130km D 4220km
A Solution
N
B
C 57 0E
0
96 E
D
E
W E
S 18 0S
Here distance between any
two points on latitudes along A, B,
C, D, E is derived as shown next
N S

Distance along parallel of latitude =  2R cos 
P r X 360
Y
Here  is 96 – 57 i.e 39 and  is 18
R
39 22
=  2  6400  cos 18
Z 360 7
O
39 22
=  2  6400  0.9511
360 7
= 4144.98km
≈ 4145km
S 2005/ 34 – 35 NABTEB (Nov)
Assuming X to be a point on the earth surface with X and Y are two places on the earth surface such that
latitudes Ф (phi), of course XY is a small circle with X (lat 600N, long 300E) and Y (lat 600N, long 850E)
radius r. The resulting ∆ OPX has 34. Find the sectorial angle XY measured along the
r
Cos Ф = parallel of latitude
R A 50 B 550 C 600 D 1150
Where R is the radius of the earth Solution
r – small r N 30 0 E
R – Big R 0
85 E
Thus to calculate the distance of any two points on the
0 y
parallel of latitudes, we apply the formula 60 N
y
 x
Distance = x 2 r But r = R cos Ф 85
0
360 W E
0
30

= x 2 R cos Ф
360 x
Where θ is the angular distance of the points on the
longitudes to East or West.
S
Ф is the angle between the given parallel of
latitude and the equator i.e latitude measurements Sectorial angle XY = 85 – 30
is to North or South = 550 (B)
Note: θ and Ф are dummy variables; they can be
changed But always remember what is attached to them 35. If the radius of the earth is 6400km, find the radius
of the parallel of latitude
2014/37 NABTEB A 3.200km B 3670km C 3671km D 5243km
Calculate the radius of latitude 750N Solution
A 1656km B 3911km C 4229km D 6182km 
Distance along parallel of latitude =  2r
Solution 360
Radius r = R cos  Where r = R cos 
= 6400  cos 750 and  is the latitude measure to N or S
= 6400  0.2588 Here we are simply asked to find r in
= 1656.32km R = r cos 
 1656km (A) R = 6400  cos 60
= 3,200km (A)
240
N
10 0 W
2009/32 Neco (Nov) counter example 480 E
Two points X and Y are on latitude 500N and are
directly opposite each other. If the longitude of X is
500E, what is the longitude of Y?
A 1300W B 1300E C 500W D 400W E 400E W E
Solution
N 15 0S
500 E

500N X Y
Y X

S
W E
Dis tan ce XY
Speed =
time
Here speed is 650km/hr, time is?
Distance XY is
S 
Longitude of Y is 500W Distance along parallel of latitude =  2R cos 
360
(Points X and Yare directly opposite each other on the Here  is 10 + 48 i.e 58 and  is 15
same latitude). 58
=  2  3.142  6400  cos 15
360
2006/31 Neco (Nov) 58
A(latitude 600N, longitude 450E) and =  2  3.142  6400  0.9659
360
B(latitude 600N, longitude 150E) are two towns on the = 6258.55 km
earth surface. Calculate, correct to 3 significant figures, Dis tan ce XY
the distance between A and B along the parallel of Thus, speed =
time
latitude (Take R = 6400km) 6258.55
A 6700km B 3530km C 3350km 650 =
time
D 1860km E 1680km 6258.55
Solution Time =
N
650
15 0 E = 9.629hrs
0
45 E  10hrs to the nearest 1 hour
60 0 N
A 2004/35 NABTEB (Dec) Exercise 16.2
B What is the angle between two points P and Q whose
W E longitudes are 1020E and 380W lying on latitude 300S
A 1400 B 940 C 640 D 520

2008/22 NABTEB (Nov) Exercise 16.3


Calculate the radius of the parallel of latitude 600N.
S (use R = 6400km)
 A 110km B 3200km C 4600km D 6340km
Distance along parallel of latitude AB = 2R cos
360
Here  is 45 – 15 i.e 30 and  is 60 2015/53 Neco Exercise 16.4
30 22 Two villages are on (150S, 1070E) and (150S, 170E)
=  2  6400  cos 60 respectively. What is their distance apart, along the line of
360 7
30 22 latitude? (Leave your answer in terms of  and R, the radius
=  2  6400  0.5
360 7 of the earth)
= 1676.19 km R R
A 2R cos150 B sin150 C cos150
 1680km to 3 s.f
(E) 4 4
2006/8b NABTEB (Nov) R R
D sin150 E cos150
An aeroplane flies at 650km per hour along the parallel 2 2
of latitude from a point X(150S, 100W) to
Y(150S, 480E). Calculate the time spent by the
aeroplane to fly from X to Y to the nearest 1 hour
(Take R = 6400km and  = 3.142)
Solution

241
Joint cases on small and great circles 
(a)Distance along parallel of latitude PQ =  2R cos
360
2009/9a (Nov)
A(lat 430N, long 770E), B(lat 430N, long 1030W) and Here  is 45 – 15 i.e 30 and  is 40
30 22
C(lat 570S, long 770E) are three points on the surface of =  2  6400  cos 40
360 7
the earth. Find the distance from:
30 22
( i ) A to B along latitude 430N =  2  6400  0.7660
360 7
( ii ) A to C along the great circle joining the two points
= 2567.92km
[Take  = 3.142 and R = 6400km].
 2600 km to the nearest 100km
Solution
N
1030 W 77 0 E Dis tan ce PQR
(b) ( i ) speed =
430 N time
B A PQ QR
2567.92 + 6700
850 =
time
W E 850 × time = 9267.92
9267.92
Time =
850
57 0S = 10.9 hrs
C

S (ii) Latitude of R can be gotten from  in:


 
( i ) Distance AB (Small circle) =  2R cos  Distance along longitude QR =  2R
360 360
 22
Here  = 103 + 77 = 1800 and  = 430 6700 =  2  6400
(Addition: east to west) 360 7
6700  360  7
Distance AB =
180
 2  3.142  6400  cos 430 = = 
360 2  22  6400
= 3.142  6400  cos 430 59.960 = 
= 14706.64km Thus latitude of R = 59.96 – 40
 = 19.96 N
( ii ) Distance AC (Great circle) =  2R  200 N to the nearest degree
360
Here  = 43 + 57 (North to South) = 1000 As the plane flies from South to north, once  exceeds 400, it
100 has entered the Northern Hemisphere
Distance AC =  2  3.142  6400
360
= 11171.56km 2012/9a Neco
The longitude of town B in 480E and the longitude of
2010/10 Neco another town C is 320W. Both towns B and C lie on latitude
Two points P and Q on latitudes 400S are on longitude 250N. Calculate, correct to 3 significant figures, the:
150W and 450W respectively. (i) distance between town B and town C along the
(a) Find the distance between P and Q along their parallel of latitude;
parallel of latitude, correct to the nearest 100km. (ii) Chord BC
(b) A third point R is on the same longitude as Q but (b) Town A is 4800km north of town B. If the latitude
due north of it. If the distance between Q and R is 6700km, of town B is 250N, what is the latitude of town A
( i ) Find the total time taken by an aircraft which flies correct to the nearest degree?
at a constant speed of 850km/hr from P to Q and then to R. (Take  = 3.142 and radius of the earth = 6400km)
( ii ) What is the latitude of R to the nearest degree? 0
32 W
N
48 0E
(Take  = 22/7 and radius of the earth as 6400km)
Solution A
N 25 0 N
0
45 W C B
15 0 W
W E

W E
R
S
0S 
40 ( i ) Distance along parallel of latitude BC = 2R cos
Q P 360
Here  is 32 + 48 i.e 80 and  is 25
S

242
80 40
=  2  3.142  6400  cos 25 =  2  3.142  6400  0.9397
360 360
80 = 4199.16 km
=  2  3.142  6400  0.9063
360 4199.16
= 8099.82 km Thus speed =
6
 8100km to 3 significant figures = 699.86km/hr
  700km/hr to 3 s.f
(ii) Chord BC = 2 r sin
2 ( ii ) The latitude of C can be gotten from  in:

= 2 R cos  sin Distance along longitude BC =

 2R
2
360
Substituting 
= 2  6400  cos25 sin40 420 =  2  3.142  6400
360
= 2  6400  0.9063  0.6428 420  360
= 7456.89km = 
2  3.142  6400
 7460km to 3 s.f 3.760 = 
(b) Latitude of town A can be gotten from  in: Thus latitude of C = 20 + 3.760 N
0

 = 23.76 N
Distance along longitude AB =  2R
360  240 N to the nearest degree
 Reason we added because latitude C is upper part of lat B; higher
4800 =  2  3.142  6400
360 latitude that B: that is increasing in latitude so you cannot subtract
4800  360
= 
2  3.142  6400 2014/7 Neco
42.970 =  An aeroplane flies eastward from a town M(600S, 40E) for
Latitude of A = (25 + 42.97)0 N 4hours at an average speed of 300km/hr to another town R. it
= 67.970N then flies due north from R to another town T a distance of
 680N to the nearest degree 1800km. calculate the:
Reason: we added because latitude A is in upper part of lat B; higher (a) Longitude of R;
latitude than B i.e increasing latitude so we cannot subtract. (b) Latitude of T, correct to the nearest degree
(Take  = 22/7, R = 6400km)
2011/13b Solution
An aeroplane flies from town A(200N, 600E) to town N
B(200N, 200E). 40 0 E
( i ) if the journey takes 6 hours, calculate, correct to 3
significant figures, the average speed of the aeroplane.
(ii) if it then flies due north from town B to town C, T
420km away, calculate, correct to the nearest degree,
the latitude of town C E
W
[Take radius of the earth = 6400km and  = 3.142]
Solution
N R 60 0 S
20 E0 M
C 60 0 E
S
200 N
A (a) Longitude of R can be gotten from:
B 
Distance along parallel of latitude MR =  2 r
W E 360
where r is R cos 
But Distance MR = speed from M to R  time
= 300  4
= 1200km
S Substituting
 22
dis tan ce 1200 =  2  6400  cos 60 0
( i ) Speed = 360 7
time 1200  360  7
Here distance is =
2  22  6400  0.5
 = 21.480
Distance along parallel of latitude AB = 2R cos 
360 (which is the angular difference between M and R)
Here  is 60 – 20 i.e 400 and  is 200 But longitude of R is 400 + 
=
40
 2  3.142  6400  cos 20
360
243
= 40 + 21.48 = 61.480E 2000  360  7
=
 610E to the nearest degree 2  22  6400
(b) The latitude can be gotten from : 17.900 = 
 Since the plane flew south from B to C
Distance along longitude RT =  2R
360 Latitude of C = 530N – 17.900
 22 = 35.1N
1800 =  2
 6400
360 7  350N to the nearest whole number
1800  360  7 Point C did not enter the southern
=
2  22  6400 Hemisphere as shown on the sketch
= 16.110
Latitude of T is 60 – 16.11 = 43.890 2013/12 Exercise 16.5
 440 to the nearest degree An aeroplane flies due north from a town T on equator at a
We subtracted because the flight to T from R was due North speed of 950km per hour for 4 hours to another town P. It
then flies eastward to town Q on longitude 650E. If the
2005/8 longitude of T is 150E,
A plane flies due East from A(lat 530N, long 250E) to a (a) Represent this information in a diagram
point B(lat 530N, long 850E ) at an average speed of (b) Calculate the:
400km/h. The plane then flies south from B to a point C (i) Latitude of P, correct to the nearest degree;
2000km away. Calculate, correct to the nearest whole (ii) Distance between P and Q, correct to 4 significant figures
number: [Take  = 22/7, Radius of the earth = 6400km]
(a) the distance between A and B;
(b) the time the plane takes to reach point B; 2009/11b Neco Exercise 16.6
(c) the latitude of C P(550N, 160W), Q(550N, X0E) and S(Y0S, 160W) are three
[Take radius of the earth to be = 6400km and  = 22/7] towns on the earth’s surface. The distance between P and Q
Solution along the parallel of latitude 550N is 3800km and the
N
25 0 E distance between P and S along the meridian 160W is
85 0 E 5,000km. Calculate, correct to the nearest degree, the
53 0 N
B
(i) value of x0,
A (ii) angle subtended by arc PS at the centre of the earth
C (Take R = 6400km and  = 22/7)
W E

S Shortest distance between any two points on


(a) Distance between A and B is the parallel of latitude i . e
 (Distance along the great circle of two points on
Distance along parallel of latitude =  2R cos  the parallel of latitude).
360
Here  is 85 – 25 i.e 600 ( is in the same east, east),  is 530 Apart from the normal questions on distance along the great
Substituting circle or along the parallel of latitudes, there are other
60 22 questions on the shortest distance between any two points on
=  2   6400 cos 53
360 7 the parallel of latitudes or distance along the great circle of
60 22 any two points on the parallel of latitude. Below is a sketch
=  2   6400  0.6018 of how the problem looks like:
360 7
= 4034.93km
 4035km to the nearest km
dis tan ce C
(b) Speed =
time A B
4034.93
400 =
time D
4034.93
Time = = 10.09hrs
400
 10 hrs to the nearest whole number
(c) The latitude of C can be gotten from  in This form of question are interwoven in the sense that the data
 given concerns mainly arc AB through D but candidates are
Distance along longitude BC =  2R required by the question to find out some facts on arc AB
360
 22 through C. This can be done in two ways namely:
2000 =  2  6400 Split and direct formula methods
360 7

244
1 length of chord PQ 1  3447
2 2
Example 1 = =
R 6400
P and Q are two points on latitude 550N and their
longitudes are 330 W and 230 E respectively. Calculate Sin  = 0.2693
the distance between P and Q measured along:  = sin –1 (0.2691) = 15.60
(a) the parallel of latitude Thus, Arc PQ = 2 x 15.6 x 2 x 22 x 6400
(b) great circle 360 x 7
(Take  = 22/7, Radius of the earth = 6400km) = 3486.47  3486 km
Solution Method II (Applied formula)
0
N To avoid error due to students’ judgment of three-
33 W
23 E
0
dimensional shape in two dimensions, the above procedures
55 N0
can be summed up as follows;
P Q  
55
2 sin −1  cos  sin   2R
W E  2
Distance PQ along great circle =
360
Where θ = angular distance or longitude difference.
Ф = is the given latitude to North or South
S Substituting
(a) From the resulting diagram,  56 
 2 sin −1  cos 55 sin   2  22  6400
Distance PQ =  2R cos   2 
360 Distance PQ =
360  7
Here θ = 33 + 23 i.e 560 and Ф =550
2 sin −1 (0.2691)  281600
PQ = 56 x 2 x 22 x 6400 x cos 55 =
360 x 7 2520
= 44 x 1280 x 0. 5736 31.20  281600
= = 3486km
9 2520
= 3589 km Note
θ and Ф are dummy variables, they can be changed to r
(b)
and s, or a and b etc. But the values attached to them
The distance between P and Q measured along the great remain unchanged.
circle is also the shortest distance between P and Q. The Author applied, method I to solve the given problem for
Method I (SPLIT) students to appreciate and be able to recall easily formula II which
0 N can be used without drawing much diagrams.
56
0
33 W 0
23 E
0
Candidates should note that the answer to the great circle distance
r
55 N
P X is always smaller in value than that of the distance along parallel of
r Q
0
28 28
0
P Q latitude.
R
W E
P Q R R
X 1975/11
P is a town on longitude 74042'E and Q is another town on
Fig I Fig II longitude 20018'W. Both towns are on latitude 48050'N.
S Calculate: (a)the distance between P and Q measured along
From fig I their common latitude (b) the length of chord PQ
The length of chord PQ = 2 XQ (c) the angle subtended by PQ at the centre of the Earth
 [Take  to be 3.142, R = 6400km]
= 2r sin
2 Solution
 Note that: 60' = 10
= 2 R Cos Ф sin N
2 0
20 18 W 74042 E
0
= 2 x 6400 x cos 55 sin 28 O
= 2 x 6400 x 0.5736 x 0.4695 48050 N
Q P
= 3447 km
From fig II E
W
2 C
Arc PQ is the required distance =  2R
360
 is derived from fig. II as shown below:
XQ
Sin  = S
R

(a) Distance along parallel of latitude =  2R cos 
360
Here  is 20018'+74042' i.e 94060' = 950 ,  is 480 50' i.e 48.80

245
Substituting 
(a) Length of chord XY = 2r sin
95 2
=  2  3.142  6400  cos 48.8 0
360 Here  is 86 – 14 = 720 and r = R cos 
=
95
 2  3.142  6400  0.6587 Where  is the latitude angle 320
360 72
Thus length of chord XY = 2  6400  cos 32  sin
= 6990.77km 2
 = 1280  0.8480  0.5878
(b) Length of chord PQ = 2 r sin
2 = 6380.22km
Where r = R cos   6380km to 3 s.f
95 (b)i Distance between X and Y along parallel of latitude
= 2  6400  cos 48.80  sin
2 
= 2  6400  0.6587  0.7373 =  2R cos 
360
= 6216.44km 72
=  2  3.142  6400  cos 32
360
(c) The angle subtended by PQ at the centre of the 72
=  2  3.142  6400  0.8480
earth C is 2 in the diagram below derived from the 360
main diagram. = 6820.90km
 6820km to 3 s.f
X
Q P (b)ii Distance between X and Y along great circle
 
2 sin −1  cos  sin   2R
 2
R R =
360
 72 
C 2 sin −1  cos 32 sin   2  3.142  6400
 2 
=
XP 360
In CXP, sin  =
R 2 sin −1 (0.8480  0.5878)  40217.6
1 =
2
length of chord PQ 360
= −1
2 sin (0.4985)  40217.6
R =
1 360
 6216.44
2  29.9  40217.6
= 2 (6216.44 from b) =
6400 360
sin  = 0.4857 = 6680.59km  6680km to 3 s.f
 = sin – 1 0.4857
= 29.060 2006/9 Neco (Nov)
Thus 2 = 2  29.06 If two places P and Q are on the same parallel of latitude
= 58.120 300N and they differ in longitude by 450. Calculate the;
(i) distance between P and Q measured along the
2007/12 Neco parallel of latitude
Two points X(320S, 860W) and Y(320 S, 140W) are on (ii) angle subtended at the centre of the earth by PQ and
the earth’s surface. Calculate correct to three significant (iii) shortest distance between P and Q
figures the: (R = 6400km)
(a) Length of chord XY Solution
(b) Distance between X and Y along the: For emphasis sake, we sketch as:
(i) parallel of latitude (ii) great circle N
[Take R = 6400 and  = 3.142]
Solution 300 N
N Q
P
86 0 W P Q
0
14 W
W E
O

O
W E

S
32 0 S 
X Y
( i ) Distance PQ along parallel of latitude =  2R cos 
360

246
=
45
 2
22
 6400  cos 30
(ii) Distance PQ along great circle
360 7  
45 22 2 sin −1  cos  sin   2R
=  2  6400  0.8660  2
360 7 =
360
= 4354.74km
 180 
(ii) Angle subtended at the centre of the earth by PQ =POQ 2 sin −1  cos 50 sin   2  22  6400
 2 
= 2 =
360  7
−1 
But 2 = 2 sin  cos  sin  =
2 sin −1 (0.6428  1)  2  22  6400
 2 2520
Where  is common latitude 30 0
2  40  281600
=
 is longitude difference 450 2520
−1  45  = 8939.68km
 POQ = 2 = 2 sin  cos 30 sin 
 2 2015/7a Neco Exercise 16.7
= 2 sin −1 (0.8660  sin 22.5) A(Lat 40N, long150E) and B(Lat 40N, long 400W) are two
= 38.70 points on the earth’s surface. Calculate to the nearest km, the
distance along
(iii) Shortest distance between P and Q ( i ) the parallel of latitudes: ( ii ) a great circle.
 ( Take radius of the earth to be 6400km)

2 sin −1  cos  sin   2R
 2
=
360
 45 
2 sin −1  cos 30 sin   2  22  6400
 2 
=
360  7
2 sin −1 (0.8660  0.3827)  2  22  6400
=
360  7
2 sin −1 (0.3314)  281600
=
2520
38.70  281600
=
2520
= 4324.57km

2014/4 NABTEB (Nov)


Find the distance between P(500S, 90W) and
Q(500S, 900E), along a
( i ) circle of latitude ( ii ) great circle
(Take  = 22/7, R = 6400km)
Solution
N
900 W 90 0 E

W E

50 0 S
P Q

S

( i ) Distance PQ along circle of latitude =  2R cos 
360
Here  is 90 + 90 i.e 180,  is 50
Substituting
180 22
=  2  6400  cos 50
360 7
22
=  6400  0.6428
7
= 12929.46km
247
CHAPTER SEVENTEEN Class marks (Midpoints)
This is the mid point of the class interval and is obtained by
Statistics II (Grouped data) adding the lower and upper class limits and dividing by two
Frequency Distribution (2). Thus the class mark of the interval 40 – 42 is
When summarizing large masses of raw data, it is often 40 + 42 = 41
useful to distribute the data into classes or categories and 2
to determine the number of individuals belonging to each
class called the class frequency. The tabular arrangement Measure of location for grouped data
of data by classes together with their corresponding class (Mean, median & mode)
frequency is called frequency distribution or frequency
table. Mean
Example Mean for grouped data can be calculated in two ways:
Mass (kg) Number of Students (i) Mean for problems without assumed mean
40 – 42 5 x =  fx
43 – 45 18 f
46 – 48 42 Where x is the class mark or class midpoint
49 – 51 27 (ii) Mean for problems with assumed mean
52 – 54 8
Total = 100 x = A +  fd
f
The table above shows a frequency distribution of
where A = Assumed mean ;
masses (recorded to the nearest Kg) of 100 students
d = deviation from the mean (x – A)
attending SSCE lesson.
The first method though easy to manipulate involves bigger
values and if the question did not restrict you to any particular
Any data organized and summarized as in the above
method i.e if the word “otherwise” is used in questions
frequency distribution are often called Group data.
involving assumed mean; then students should stick to the
easiest one for safety.
Class interval & Class limits
A symbol defining a class such as 40 – 42, 43 – 45 in the Example 1
above table is called a class interval – while the end The weights to the nearest kilogram, of a group of 50
numbers 40 and 42, 43 and 45 are called class limits; the students in a college of technology are given below:
smaller numbers 40, 43, ... are the lower class limits and 65 70 60 46 51 55 59 63 68 53
the bigger numbers: 42, 45; ... are the upper class limits 47 53 72 58 67 62 64 70 57 56
73 56 48 51 58 63 65 62 49 64
Class boundaries 53 59 63 50 48 72 67 56 61 64
If masses are recorded to the nearest kg, the class interval 66 52 49 62 71 58 53 69 63 59
40 – 42 theoretically includes all measurements from
(a) Prepare a grouped frequency table with class
39.5 to 42.5 kgs which are called class boundaries or
intervals 45 – 49, 50 – 54, 55 – 59 etc.
True class limits. The smaller number 39.5 is the lower
Solution
class boundary while the bigger number 42. 5 is the upper
Class internal Tally Frequency
class boundary. Displaying the given table to reflect class
45 – 49 IIII I 6
boundaries, we have
50 – 54 IIII III 8
Class interval Class boundaries Frequency 55 – 59 IIII IIII I 11
40 – 42 39.5 – 42.5 5 60 – 64 IIII IIII II 12
43 – 45 42.5 – 45.5 18 65 – 69 IIII II 7
46 – 48 45.5 – 48.5 42 70 – 74 IIII I 6
49 – 51 48.5 – 51.5 27
52 – 54 51.5 – 54.5 8 Applicable to all raw or scattered data problems
Note
In practice, the class boundaries are obtained by The problem of grouping data when they are scattered or raw
adding the upper limit of one class interval to the lower can be solved, if students use four figure table plus
limit of the next higher class interval and dividing by mathematical set or any opaque object to cover the data
two leaving one column or row at a time. Allocate the numbers
i.e 43 + 42, 46 + 45, 49 + 48 there to their various class intervals with the aid of tally. Then
2 2 2 move to the next row or column as shown below.
Class Size Alternatively write out the data as they appear in the column
This is the difference between the lower and upper class or row; then allocate them to the various class intervals with
boundaries or the difference between two upper limits, or the aid of tally. Repeat the process for the next column or row
two lower limits. The class size is also called the class depending on your chosen pattern.
width or class strength. In this case 42.5 – 39.5 = 3 or 43
– 40 = 3 or 45 – 42 = 3
248
Class internal x tally f fx
Class interval Tally
21 – 25 23 IIII II 7 161
65 45 – 49 I
26 – 30 28 IIII IIII I 11 308
47 50 – 54 I
31 – 35 33 IIII IIII 9 297
73 55 – 59
36 – 40 38 IIII IIII 9 342
53 60 – 64
41 – 45 43 IIII I 6 258
66 65 – 69 II
46 – 50 48 IIII III 8 384
70 – 74 I
f fx
Next, we attend to 2nd column = 50 = 1750
70 Class interval tally
53 45 – 49 I  fx = 1750
x =
56 50 – 54 III f 50
59 55 – 59 II = 35
52 60 – 64 2009/8
65 – 69 II The marks scored by 50 students in a Geography
70 – 74 II examination are as follows:
Next, we attend to 3rd column 60 54 40 67 53 73 37 55 62 43
60 Class interval tally 44 69 39 32 45 58 48 67 39 51
46 59 40 52 61 48 23 60 59 47
72 45 – 49 Il l
65 58 74 47 40 59 68 51 50 50
48 50 – 54 III
71 51 26 36 38 70 46 40 51 42
63 55 – 59 II
49 60 – 64 ll (a) Using class interval 21 – 30, 31 – 40, …
65 – 69 II prepare a frequency distribution table
70 – 74 IIl (b) Calculate the mean mark of the distribution
(c) What percentage of the students scored more
The pattern continues, each addition to the tally was than 60%
differentiated for emphasis. Solution
The lesser the figures the better. A student who chooses  fx
the row aspect of this data will likely have problems or x =
f
slower compared to the person picking his data from
column. The pattern to be followed depends on the way We prepare a table to reflect x, f and fx formula items.
the data given are presented. Class interval x tally f fx
Column 21 – 30 25.5 II 2 51
65 31 – 40 35.5 IIII IIII 10 355
47 41 – 50 45.5 IIII IIII II 12 546
73 51 – 60 55.5 IIII IIII IIII 15 832.5
53 61 – 70 65.5 IIII III 8 524
66 71 – 80 75.5 III 3 226.5
Row: 66 52 49 62 71 58 53 69 63 59 f fx
= 50 = 2535

(b) Mean ( x ) = 
2008/8d fx
The age in years of 50 teachers in a school are given f
below: 2535
21 37 49 27 49 42 26 33 46 40 = = 50.7
50
50 29 23 24 29 31 36 22 27 38 (c) Percentage mark more than 60%
30 26 42 39 34 23 21 32 41 46 = freq of more than 60% × 100
46 31 33 29 28 43 47 40 34 44 Total freq
26 38 34 49 45 27 25 33 39 40 8+3 11
= × 100 = × 100 = 22%
(a) Form a frequency distribution of the data using the 50 50
internals 21 – 25, 26 – 30, 31 – 35, etc
(b) Calculate the mean age 2014/7 f/maths
Solution The table shows the distribution of the lengths of 20 iron
x =
 fx rods measured in metres
f Length (m) 1.0-1.1 1.2-1.3 1.4-1.5 1.6-1.7 1.8-1.9
We prepare a table to reflect x, f and fx items in the frequency 2 3 8 5 2
formula. Using an assumed mean of 1.45, calculate the mean of the
distribution

249
Solution 2014/12 Neco (Dec)
Assumed mean formula for mean is x =A+
 fd The scores of 50 students in a mid – term test were as shown
f below:
We prepare be table to reflect x, f, d = x – 1.45, fd 11 20 30 24 13 28 33 40 23 28
Length (M) x d f fd 40 8 30 13 15 34 8 34 32 22
1.0 – 1.1 1.05 – 0.4 2 – 0.8 26 21 25 18 26 10 19 3 27 18
1.2 – 1.3 1.25 – 0.2 3 – 0.6 18 24 26 25 27 29 28 13 35 24
1.4 – 1.5 1.45 0.0 8 0.0 9 24 14 28 27 38 40 32 33 34
1.6 – 1.7 1.65 0.2 5 1.0 (a) Construct a frequency table using the class intervals
1.8 – 1.9 1.85 0.4 2 0.8 1 – 5, 6 – 10, 11 – 15, …
f = fd = (b) Estimate the:
20 0.4 ( i ) mean score (ii) median score (iii) modal score
 fd Solution
Mean ( x ) = A +
f Class interval x tally f fx
0 .4 1–5 3 I 1 3
= 1.45 + 6 – 10 8 IIII 4 32
20
11 – 15 13 IIII I 6 78
= 1.45 + 0.02 = 1.47
16 – 20 18 IIII 5 90
2005/11a Neco 21 – 25 23 IIII IIII 9 207
The marks obtained by students in a Mathematics test 26 – 30 28 IIII IIII III 13 364
are recorded as follows: 31 – 35 33 IIII III 8 264
Marks 1-10 11-20 21-30 31- 40 36 – 40 38 IIII 4 152
Number of 10 20 50 90 f fx
students
= 50 = 1190
−  fx 1190
41-50 51-60 61-70 71-80 (i) x= = = 23.8
110 70 40 10 f 50
For median
What is the modal class? Median formula for grouped data is given as
Solution
 n − Cf b 
Mode formula for grouped data is given as Median = L1 +  2 C
 1   fm 
Mode = L1 +  C  
 1 + 2 
 Where
Where : L1 = lower class boundary of the modal class L1 = Lower class boundary of the median class
△1 = Difference between the modal frequency and the n = Total frequency
Cfb = Cumulative frequency before the median class
frequency of the next lower class i.e class before it .
Fm = Frequency of median class
△2 = Difference between the modal frequency and the C = Size of the median class
frequency of the next higher class i.e class after it.
C = Size of the modal class we prepare a table to reflect class boundaries and cumulative frequencies
Modal table Marks class boundaries f cf
Marks class boundaries f 1–5 0.5 – 5.5 1 1
1 – 10 0.5 – 10.5 10 6 – 10 5.5 – 10.5 4 5
11 – 20 10.5 – 20.5 20 11 – 15 10.5 – 15.5 6 11
21 – 30 20.5 – 30.5 50 16 – 20 15.5 – 20.5 5 16
31 – 40 30.5 – 40.5 90 21 – 25 20.5 – 25.5 9 25
41 – 50 40.5 – 50.5 110 26 – 30 25.5 – 30.5 13 38
51 – 60 50.5 – 60.5 70 31 – 35 30.5 – 35.5 8 46
61 – 70 60.5 – 70.5 40 36 – 40 35.5 – 40.5 4 50
71 – 80 70.5 – 80.5 10
(ii) Median, first, we find n i.e 50 = 25 median position
2 2
First identify the modal class: the class with the
Next, we find the class interval where the median lies with
highest frequency which is 41 – 50
the aid of cumulative frequency:
L1 = 40.5, 1 = 110 – 90 = 20, 2 = 110 – 70 = 40
At cumulative frequency 25 occurs in class interval 21 – 25
and C = 10
L1 = 20.5, Cfb = 16, C = 5 i.e 25 – 20 or 26 – 21
Mode = 40.5 + 
20 
  10 Fmedian = 9
 20 + 40   25 − 16 
= 40.5 + 20  10 = 43.83 Median = 20.5 +  5
60  9 
9
= 20.5 +  5
9
= 20.5 + 5 = 25.5
250
(iii) Mode: 2013/12 Neco
 1  The marks of 100 students who passed Economics in a
mode = L1 +  C promotion examination are given below
 1 +  2 
Mark % 51– 60 61- 70 71- 80 81– 90 91-100
First, identify the modal class Frequency 11 23 39 17 10
Class with highest frequency which is 26 – 30
Find the: ( i ) mean mark ( ii ) mean deviation and
L1 = 25.5, 1 = 13 – 9 i.e 4, 2 = 13 – 8 i.e 5 and C = 5
( iii ) Standard deviation correct to the nearest whole number
Mode = 25.5 + 
4 
5 Solution
4 + 5
Mean ( x ) =  ,
fx  f / x−x /
= 25.5 + 2.22 M.D = and
= 27.72 f f
2
Measures of spread (plus measure of location cases)  fx 2   fx 
S.D = −  
Standard deviation and mean deviation f f 
Standard deviation and mean deviation are not the only
Next we prepare a table to reflect the formula items
measures of spread. The subtopic measure of spread has
been treated under statistics I. x, f, fx, / x − x /, f / x − x /, x2 and fx2
Marks % x f fx /x−x/ f/ x − x / x2 fx2
2009/12 Neco 51 – 60 55.5 11 610.5 19.5 214.5 3080.25 33882.75
62 54 53 44 46 55 46 56 68 63 61 – 70 65.5 23 1506.5 9.5 218.5 4290.25 98675.75
59 61 66 54 39 48 47 53 59 57 70 – 80 75.5 39 2944.5 0.5 19.5 5700.25 222309.75
50 35 40 30 46 44 36 49 54 51 81 – 90 85.5 17 1453.5 10.5 178.5 7310.25 124274.25
57 56 45 33 38 41 40 45 53 58 91 – 100 95.5 10 955 20.5 205 9120.25 91202.5
51 45 48 34 36 46 43 49 63 52 f = fx = f/ x − x / fx2 =
(a) Using class interval 30 – 34, 35 – 39, 40 – 44, … 100 7470 = 836 570345
construct the frequency distribution table
( i ) Mean ( x ) = 
fx
(b) Calculate the: f
( i ) Mean; ( ii ) Standard deviation 7470
(All answer correct to 2 decimal places) == 74.7  75 to nearest whole number
100
Solution
 f / x−x /
 fx 2   fx 
2 (ii) Mean deviation =
Mean ( x ) =  f
fx
and S.D = −  
f f f  836
=
We prepare our table to reflect formula items x, x2, f, fx, fx2 100
Class interval tally f x fx x2 fx2 = 8.36  8 to nearest whole number
30 – 34 III 3 32 96 1024 3072 2
 fx 2   fx 
35 – 39 IIII 5 37 185 1369 6845 (iii) S.D = −  
40 – 44 IIII I 6 42 252 1764 10584 f f 
45 – 49 IIII IIII II 12 47 564 2209 26508 2
570345  7470 
50 – 54 IIII IIII 10 52 520 2704 27040 = −  
55 – 59 IIII III 8 57 456 3249 25992 100  100 
60 – 64 IIII 4 62 248 3844 15376
= 5703.45 − (74.7) 2
65 – 69 II 2 67 134 4489 8978
f = fx = fx2 = = 5703.45 − 5580.09
50 2455 124395
= 123.36
= 11.107  11 to the nearest whole number
(b)i Mean ( x ) = 
fx 2455
= = 49.1 2012/14 f/maths
f 50
The table gives the distribution of heights in metres of 100
2
 fx 2   fx  students
(ii) S.D = −  
f f  Height 1.40-1.42 1.43-1.45 1.46-1.48
Freq 2 4 19
2
124395  2455 
= − 
50  50  1.49-1.51 1.52-1.54 1.55-1.57 1.58-1.60 1.61-1.63
30 24 14 6 1
= 2487.9 − (49.1)
= 2487.9 − 2410.81 (a) Calculate the
(i) mean height (ii) mean deviation of the distribution.
= 77.09 (b) What is the probability that the height of a student
= 8.780 selected at random is greater than the mean height of the
 8.78 to 2 d.p distribution?
251
Solution  fd
(a) Mean ( x ) = A +
 fx  f / x−x / f
Mean ( x ) = , mean deviation =
f f − 54 
= 19 +   = 19 – 2.45
Next, we prepare a table for the formula items:  22 
x, f, fx , / x − x / and f / x − x / = 16.55yrs  16.6 to 3sf
Height x f fx / x−x/ f / x−x/ 2 2
 fd 2   fd  360  − 54 
1.40 – 1.42 1.41 2 2.82 0.1 0.2 (b) S.D = −   = −  
f f  22  22 
1.43 – 1.45 1.44 4 5.76 0.07 0.28
1.46 – 1.48 1.47 19 27.93 0.04 0.76 = 16.36 − 6.025 = 10.335 = 3.215  3.22yrs to 3sf
1.49 – 1.51 1.50 30 45.00 0.01 0.30
1.52 – 1.54 1.53 24 36.72 0.02 0.48 2007/14 f/maths
1.55 – 1.57 1.56 14 21.84 0.05 0.70 The table shows the distribution of ages of workers in
1.58 – 1.60 1.59 6 9.54 0.08 0.48 a company
1.61 – 1.63 1.62 1 1.62 0.11 0.11 Age (in yrs) 17 – 21 22 – 26 27 – 31 32 – 36
f fx = f / x − x / frequency 12 24 30 37
=100 151.23 = 3.31 37 – 41 42 – 46 47 – 51 52 – 56
 fx 45 25 10 7
( i ) Mean ( x ) =
f ( a ) Using an assumed mean of 39, calculate the
151.23 ( i ) Mean ( ii ) standard deviation of the distribution
= = 1.5123
100 (b) If a worker is selected at random from the company for
 1.51m to 2d.p (approximated base on the given data) an award, what is the probability that he is at most
 f / x−x /
36 years old?
( ii ) Mean deviation = Solution
f
Assumed mean formula for mean and S.D
3.31
= = 0.0331  fd 2   fd 
2
100  fd
x=A+ , S.D = −  
 0.03 to 2d.p f f f 
(b) Prob (height > mean height) We prepare a table to reflect the formula items
= sum freq greater than 1.51 x, f, d = x – 39, fd, d2, fd2
Total freq Age (yrs) x d f fd d2 fd2
24 + 14 + 6 + 1 17 – 21 19 –20 12 –240 400 4800
=
100 22 – 26 24 –15 24 –360 225 5400
45 9 27 – 31 29 –10 30 –300 100 3000
= = = 0.45
100 20 32 – 36 34 –5 37 –185 25 925
37 – 41 39 0 45 0 0 0
2013/6 f/maths
42 – 46 44 5 25 125 25 625
The table shows the distribution of ages of 22 students
47 – 51 49 10 10 100 100 1000
in a school
Age (years) 12 –14 15 –17 18 –20 21 –23 24 –26
52 – 56 54 15 7 105 225 1575
f = fd = fd2 =
Frequency 6 10 3 2 1
190 –755 17325
Using an assumed mean of 19, calculate, correct to  fd
(a) i Mean ( x ) = A +
three significant figures, the: f
(a) Mean age (b) standard deviation of the distribution − 755 
Solution = 39 +   = 39 – 3.97 = 35.03
 190 
Assumed mean formula for mean and S.D
2 2
2  fd 2   fd  17325  − 755 
 fd  fd 2   fd  (ii) S.D = −   = −  
x=A+ , S.D = −   f f  190  190 
f f f 
We prepare a table to reflect the formula items = 91.184 − (3.97) 2 = 91.184 − 15.7609 = 75.4231
x, f, d = x – 19, fd, d2, fd2 = 8.6846  8.68
Age(years) x d f fd d2 fd2 (b) Pr(at most 36) = Pr( 36)
12 – 14 13 –6 6 –36 36 216 f (36) + f (31) + f (26) + f (21)
15 – 17 16 –3 10 –30 9 90 =
Total freq
18 – 20 19 0 3 0 0 0 37 + 30 + 24 + 12
21 – 23 22 3 2 6 9 18 =
190
24 – 26 25 6 1 6 36 36
103
f fd fd2 = = 0.542
190
=22 = –54 =360
252
Discrete data & continuous data
Discrete data have distinct values without intermediate 699837.34
points. The number of student(s) in a class could be SD =
50
0, 1, 2, 3, 4, 5,… but not 1.5 or 25.6 or 50.1
impossible ! Frequency can also be attached such as: = 13996.7468 = 118.3078 ≈ 118.31 to 2d.p
the number of girls, number of boys and number of
absentees in a class. 2007/13 Adjusted Exercise 17.1
Where as if we are to consider the weight of students in In a college, the number of absentees recorded over a period
a class; values such as 16kg, 16.4kg, 17.1kg, 19.3kg of 30 days was as shown in the frequency distribution table
20.0kg etc. the result will be both whole and fractional Number of 0–4 5–9 10 – 14 15– 19 20 – 24
absentees
or intermediate values. This will require a range say Number of 1 5 10 9 5
16–20. Hence continuous data have values over a days
range of whole number or fraction.
Note that when a continuous data is treated as fixed, it Calculate the: (a) Mean (b) standard deviation,
becomes discrete (c) mean deviation correct to two decimal places

2016/9 2016/12 Neco Exercise 17.2


The weight(in kg) of 50 contestants at a competition is The scores of 40 students in a Geography test are given
as follows: below:
65 66 67 66 64 66 65 63 65 68 62 64 67 68 55 61 62 65 67 69
64 62 66 64 67 65 64 66 65 67 69 64 61 68 70 70 69 64 63 61
65 67 66 64 65 64 66 65 64 65 63 70 61 62 69 61 64 68 65 68
66 65 64 65 63 63 67 65 63 64 50 51 53 54 55 57 52 56 58 60
66 64 68 65 63 65 64 67 66 64 (a) Using a class interval 50 – 54, 55 – 59,…, construct
a frequency distribution table
(a) Construct a frequency table for the discrete data (b) Find the: i. mean; ii. variance iii. Standard deviation
(b) Calculate, correct to 2 decimal places, the:
i. mean ii. standard deviation; of the data 2012/10 Exercise 17.3
Solution Class interval Frequency
Discrete data implies the given data are not continuous as it 60 – 64 2
may appear to be 65 – 69 3
(b) i. Mean x =
 fx 70 – 74 6
f 75 – 79 11
Table for mean
80 – 84 8
Weight( kg) tally frequency fx
85 – 89 7
62 I 1 62 90 – 94 2
63 IIII 5 315 95 – 99 1
64 IIII IIII II 12 768 The table shows the distribution of marks scored by students
65 IIII IIII IIII 14 910 in an examination, calculate, correct to 2 decimal places, the
66 IIII IIII 10 6600 (a) Mean (b) standard deviation of the distribution
67 IIII I 6 402
68 II 2 136 2014/15a & c Neco f/maths Exercise 17.5
Σf = 50 Σfx = 9193 A cattle rearer recorded the weight in kilograms of
Mean x =
 fx
=
9193
= 183.36
50 cows in his farm as follows:
f 50 28 45 46 52 56 83 26 48 28 53
72 50 32 77 41 43 53 50 80 45

( ii ) S.D =
(
f x−x )2 69 64 72 66 43 65 62 51 52 67
54 31 32 52 65 36 42 60 53 75
f 60 53 46 88 64 83 56 88 26 64
Table for SD (a) Construct a cumulative frequency table using the
x x− x (x − x ) 2 f
(
f x− x )
2 class intervals: 26 – 35, 36 – 45, …
(b) From your table, calculate the mode of the
62 –121.36 14728.250 1 14728.250
63 –120.36 14486.530 5 72432.650
distribution.
64 –119.36 14246.810 12 170961.720
65 –118.36 14009.090 14 196127.260
66 –117.36 13773.370 10 137733.700
67 –116.36 13539.650 6 81237.900
68 –115.36 13307.930 2 26615.86
f =50 −
f ( x − x) =
2

699,837.34

253
CHAPTER EIGHTEEN Expenditure on education is N 180,000
Sector of Education is 900
Graphs (Charts) 90
Statistical graphs (Charts) × Total income = 180, 000
360
PIE CHART
90  Total income = 180,000  360
A pie chart is a circular diagram in which each sector of
180 000  360
the circle represents a given frequency expressed in Total income =
90
degrees.
= N 720,000 ( E )
2012/51 Neco 36. Calculate the amount spent on transport
The pie chart below shows the division of the workforce A N 20,010.00 B N 30,000.00
of a factory C N 400,000.00 D N 720,000.00 E N 800,000.00
Solution
15
Labourers Amount spent on transport =  720,000
210 0
360
= N 30,000 ( B )
70 0
80 0 Clerks 37. What percentage of the income was spent on
Skilled utilities?
men
A 33.3% B 32.1% C 30% D 25.5%
E 20.2%
51. What fraction of the workforce is skilled men?
Solution
2 3 4 5 5
A B C D E 120
9 8 5 3 3 % on utilities =  100
360
Solution
= 33.3% ( A )
80 8 2
Fraction of skilled men = = = (A)
360 36 9
2006/30 and 31 NABTEB (Nov)
The pie chart below illustrates the age of student in a
2011/53
Technical college. The age brackets are A, B, C and D
A father was to share to his children N12,000,000.00 Use the information to answer questions 30 and 31
and demonstrated the sharing on a pie chart. If the
sectorial angle of his only son is 600, how much did the
son take? A B
A N 10,000,000.00 B N 8,000,000.00
0
C N 6,000,000.00 D N 4,000,000.00 145
E N 2,000,000.00 80 0
450
Solution
C
60 D
Son’s share =  12,000,000
360
= N 2,000,000.00 (E)
30. If there are 45 students under B, how many students
2014/35 – 37 Neco are in the college?
The pie chart below shows the expenditure on a man’s A 90 B 180 C 360 D 720
income whose expenditure on education is Solution
N180,000.00. Use the information to answer questions Sector under B
Age of B student is 45
35 to 37
Sector of B is 900
45
 360 = 90
Utilities Education Total students
120 0
45  360 = 90  Total students
45  360
150 135
0 = total student
Food 90
Transport
180 = total student (B)

31. How many students are under C?


35. What is the total income of the man?
A 160 B 80 C 40 D 30
A N180,000.00 B N270,000.00
Solution
C N300,000.00 D N 400,000.00 E
N720,000.00 80
Students under C =  180
Solution 360
The sector with useful information here is education = 40 ( C )

254
2014/31 – 32 NABTEB (Nov) 2006/54 Neco
The annual salary of Mr. Segun for 2001 was The pie chart below shows the distribution of students offering
N12,000.00. He spent this on agricultural project, Further Mathematics, Geography, Economics, Biology, and
education of his children, food items, savings, Agricultural Science. What angle represents the proportion of
maintenance and miscellaneous item as show in the pie students offering Biology?
chart
Geography Biology
0
Agric x - 22 3x + 10 0
0
F/maths 2x
Education 120 0 x + 60 0
80 0 3x - 18 0
18 0 Miscellaneous
Econs Agric
300
43 0 Ma
Food items in te
n an
ce
A 330 B 430 C 900 D 1090 E 1200
Savings Solution
First, we find value of x
x – 22 + 3x + 10 + x + 60 + 3x – 18 + 2x = 360
31. How much did he spend on food item? 10x + 30 = 360
A N 9700.00 B N 6700.00 10x = 360 – 30
C N 3700.00 D N 2300.00 10x = 330
Solution
x = 330 = 33
food sec tor 10
Amount spent on food item =  12,000
360 Sector angle of Biology = 3x + 10
(Note the pattern here) = 3(33) + 10
360 − (80 + 120 + 18 + 30 + 43) = 99 + 10 = 1090 (D)
=  12,000
360 2014/29 Neco (Nov) Exercise 18.1
360 − 291 The following numbers are to be represented on a
=  12,000
360 pie chart: 20, 25, 30, 35, 40. What is the sectorial angle
69 representing 25?
=  12,000
360 A 250 B 480 C 600 D 720 E 840
= N 2,300 (D)
2008/37 NABTEB (Nov) Exercise 18.2
32. How much did he spend on agriculture? Using the pie chart below, calculate the number of
A N 1200.00 B N 1440.00 candidates that offer chemistry if the total number of
C N 4000.00 D N 2910.00 candidate is 240
Solution
120
Amount spent on Agriculture =  12,000 Maths Chem
360
0
= N 4000.00 (C) 120

Biology
2005/45 Neco
The distribution of students during lessons is as shown Physics
below:
A 60 B 70 C 80 D 90
Subject Eng Math’s Bio Agric Fine art
No of 9 11 6 5 5 2005/20 – 21 Exercise 18.3
sutudents
Calculate the sum of the sectoral angles of Maths and Computer

Biology Journals
1620
A 3600 B 1700 C 1000 D 700 E 500 27 0
54 0
Solution 90 Others
Books
11
Stationary

Maths sectorial angle =  360


9 + 11 + 6 + 5 + 5
11
=  360 The pie chart shows the items purchased for a local library.
36 Use the information to answer question 20 and 21
= 1100 20. If the cost of the computer is N 90,000.
6 Calculate the cost of the books
Biology sectorial angle =  360
36 A N15,000 B N30,000 C N50,000 D N60,000
= 600 21. What was the total amount spent for the library?
Sum of Maths and Biology sectorial angle = 110 + 60 A N185,000 B N195,000 C N200,000 D N220,000
= 1700 ( B )
255
Live diagrams in pie chart
Live diagrams in pie chart is a circular diagram in 2014/1a Neco
which each sector of the circle represents a given In a certain year, the government of a certain state bought
frequency expressed in degrees; this is done with the aid 480 cars. 180 were Peugeot, 108 were Datsun, 72 were Ford,
of a protractor to mark out the various angles. 56 were Toyota and the rest were
Volkswagen. Represent the above information on a pie chart.
2008/3 Solution
The table below shows how a man spends his income Total number of cars = 480 (given)
in a month Portion for Volkswagen = 480 – (180 + 108 + 72 + 56)
Items Amount spent = 64
Food N 4500 Cars (brand) No of cars sector angle (degrees)
House rent N 3000 180
Peugeot 180  360 = 1350
Provisions N 2500 480
Electricity N 2000 108
Datsun 108  360 = 810
Transportation N 5000 480
Others N 3000 72
Ford 72  360 = 540
480
(a) Represent the information on a pie chart 56
(b) What percentage of his income is spent on Toyota 56  360 = 420
480
transportation? 64
Solution Volkswagen 64  360 = 480
480
Total amount spent = N 20,000.00
The required diagram is as shown below: it is drawn with the
(4500 + 3000+ 25000 + 2000 + 5000 + 3000)
aid of a protractor to mark out the various angles.
Items amount spent sector angle (degrees)
4500
Food N 4500  360 = 810
20,000 Volkswagen
3000
House rent N 3000  360 = 540 Peugeot
20,000
0
2500 Toyota 48
Provisions N 2500  360 = 450 0
20,000 42 1350
2000 540
Electricity N 2000  360 = 360
20,000 Ford 810
5000
Transportation N 5000  360 = 900
20,000 Datsun
3000
Others N 3000  360 = 540
20,000

(a) The required diagram is as shown below: it is drawn with 2006/1 Neco Exercise 18.4
the aid of a protractor to mark out the various angles The data below shows the number of students that
represented a Government Secondary School in a
Mathematics Competition
Class No. of students
House JS 1 12
rent Food JS 2 14
JS 3 24
540 810 SS 1 16
Transportation SS 2 13
900 540 Others SS 3 11
36 0
450 (a) Represent this information on a pie chart
(b) Find the probability that a student picked
at random is in SS 1
Electricity Provisions
2005/8b NABTEB (Nov) Exercise 18.5
Represent the following information on a pie chart using a
radius of 6cm
5000
(b) Transportation % = 100 Letter A B C D E
20000
= 25% Frequency 10 8 5 7 6

256
2013/52 Neco
1979/3 Exercise 18.6 The bar chart below shows the number of items produced by
The result of an opinion poll among 900 students in a cat bol ventures over a period of 5 years
Grammar School regarding the country they would like
4000
to visit outside Africa is given below. Represent the
information on a pie chart 3000
Country they wish to visit No. of students
Japan 90 2000
U.S.S.R. 225 1000
Canada 155
U.K. 185 0 2004 2006 2007 2008
2005
USA 135
China 110 52. What is the ratio of the least frequent production to that
BAR CHART of the most frequent production?
A bar chart is a chart in which rectangular shapes are A 3/ 8 B 1/ 3 C 2/ 5 D 3/ 7 E 3/ 5
used to represent frequency. The rectangles have equal Solution
width and unequal length. The lengths depend on the Ratio
frequency of each class. The distinct bars are separated Least frequent : most frequent
by a uniform gap all through. Students choose the scale 1500 (2006) : 4000 (2007)
used but it must be reasonable. 15 : 40
3 : 8 or 3/8 ( A )
2014/26 - 27 NABTEB 2011/ 50 – 52 Neco
Use the chart to answer questions 26 and 27
9
Girls

8 8
Boys

7
6
Women

6
Infant s

4 5
Men

4
2
3
2
26. What is the mode? 1
A6 B8 C Boys D Girls 0
Solution 50 55 60 65 70 75 80
Weight of the boxers (kg)
Mode = girls (since girl has the highest frequency of 8)
The bar chart above shows the distribution of boxers weight
in a row for a boxing competition: use the chart to answer
27 How many people are there altogether?
questions 50 to 52
A 25 B 23 C 20 D 17
50. How many boxers have weight of 65kg and below?
Solution
A 35 B 21 C 15 D 14 E 2
Total people = 3 + 6 + 8 + 4 +2
Solution
= 23 ( B )
Freq freq freq freq
2005/46 Neco Weight of 65kg and below = 65 + 60 + 55 + 50
The bar chart below shows how a family spends its = 7 + 9 +3 + 2
income. Find the proportion of rent on the total income = 21 B
51. How many boxers registered for the competition?
10
A 14 B 15 C 35 D 49 E 56
8 Solution
6
Total boxers = 2 + 3 + 9 + 7 + 8 + 4 + 2
= 35 C
4 52. If a boxer is chosen at random. What is the probability
2 that his weight is less than or equal to 60kg?
A 1/12 B 3/16 C 2/5 D 3/4 E 11/12
0
Food Saving Rent Others Solution
freq freq freq
A 8/21 B 2/ 7 C 5/21 D 1/7 E 2/21
Solution 9 + 3+ 2
Pr ( 60kg) =
Proportion of rent on total income = Pr(rent) 35
freq of rent 6 6 2 14 2
= = = = (B) = = C
Total freq 8 + 4 + 6 + 3 21 7 35 5
257
Live diagrams in bar chart Solution
Here we are required to draw the graph in a graph sheet Number of students who passed at 50% pass mark
as shown below = freq of 50% + freq of 65% + freq of 80%
= 45 + 25 + 15
2010/5a Neco = 85 ( B )
The table below shows the distribution of scores in
percentage obtained by 20 students in a class test 5. What percentage of the students has mark
Scores 20 25 30 35 40 45 50 ranging from 35 to 50?
No. of students 5 3 1 6 2 1 2 A 551/3% B 60% C 65% D 652/3%
Solution
(a) Draw a bar chart to illustrate the information. Percentage of marks between 35 to 50
6 2010/5a Neco
freq 35 + freq 40 + freq 50
=  100
Total freq
5 35 + 40 + 4 5
=  100
20 + 35 + 40 + 45 + 25 + 15
120
=  100
4 180
N um b e r o f st u de n t s

= 662/3% (D)

3 2014/ 4 and 5 Exercise 18.7


The bar chart below shows the scores of some students in a
test. Use it to answer questions 4 and 5
2 6 2014/4 - 5

1 5

0
20 25 30 35 40 45 50 4
N um b e r o f s t u de n t s

Scores
2010/ 4 and 5
50 2010/4 - 5
3

40
2
F r e q ue n c y

30
1

0
20 0 1 2 3 4
Scores
4. How many students took the test?
A 18 B 19 C 20 D 22
10
5. If one student is selected at random find the probability
that he/she scored at most 2 marks
A 11/18 B 11/20 C 7/22 D 5/19
0
20 35 40 50 65 80 2005/4 (old) Exercise 18.8
Marks
The table shows the frequency distribution of the scores
The bar chart shows the marks distribution in an
obtained in a test by a group of 50 students
English test. Use it to answer questions 4 and 5
Score 1 2 3 4 5 6 7 8
4. If 50% is the pass mark, how many students passed Frequency 2 4 7 12 11 6 4 4
the test? (a) Draw a bar chart for the distribution
A 100 B 85 C 80 D 70 E 45

258
HISTOGRAM 2014/12a F/math Adjusted
Histogram is almost the equivalent of bar chart. It is used The histogram below represents the score of same candidate in an
for frequency distributions. Unlike the bar chart, examination
20
histogram bars have no space between them. We use 2014/12 F/maths
class boundaries to plot against the frequencies. At this 18
level Histogram is used to estimate mode.

ST U D E N T S
2009/ 7 (Nov) 16
The marks scored by 50 students in a Geography
14
examination are as follow:
60 54 40 67 53 73 37 55 62 43
12
44 69 39 32 45 58 48 67 39 51

OF
46 59 40 52 61 48 23 60 59 47 10
65 58 74 47 40 59 68 51 50 50

N UM B E R
71 51 26 36 38 70 46 40 51 42 8
(a) Using a class interval of 21, 31, 40, …, prepare a
frequency distribution table 6
(b) Draw a histogram to represent the distribution
(c) Use your histogram to estimate the modal mark 4
(d) If a student is selected at random, find the
2
probability that he/she obtains a mark greater than 60
Solution 0
Class interval Tally frequency class boundaries 18.5 28.5 38.5 48.5 58.5 68.5 78.5 88.5
8.5
21 – 30 II 2 20.5 – 30.5 MARKS
(a) Using the histogram, construct a frequency distribution
31 – 40 IIII IIII 10 30.5 – 40.5 table indicating clearly the class intervals
41 – 50 IIII IIII II 12 40.5 – 50.5 Solution
51 – 60 IIII IIII IIII 15 50.5 – 60.5 Class interval Class boundaries f cf
61 – 70 IIII III 8 60.5 – 70.5 9.0 – 18.0 8.5 – 18.5 4 4
71 – 80 III 3 70.5 – 80.5 19.0 – 28.0 18.5 – 28.5 6 10
29.0 – 38.0 28.5 – 38.5 8 18
(b) Histogram is shown on page 260 39.0 – 48.0 38.5 – 48.5 13 31
(c) 50.5 + 3 = 53.5 49.0 – 58.0 48.5 – 58.5 15 46
8 + 3 11 59.0 – 68.0 58.5 – 68.5 10 56
(d) Pr (> 60) = =
50 50 69.0 – 78.0 68.5 – 78.5 3 59
2001/5 (Nov) 79.0 – 88.0 78.5 – 88.5 1 60
An experimental census count gives the following data 2006/3 (Nov) Exercise 18.9
Age (yrs) 10 – 14 15 – 19 20 – 24 25 – 29 The table shows the frequency distribution of marks obtained by
Number 3 8 16 26 30 students in test
Marks 1–5 6 – 10 11 –15 16 –20 21 –25
30 – 34 35 – 39 40 – 44
Freq 4 6 11 8 1
18 12 6
(a) Draw a histogram of the distribution
(a) (i) Using a scale of 2cm to 5 units on the vertical (b) Use your histogram to estimate the mode
axis, draw the histogram (c) If a student is selected at random, what is the probability
(ii) Estimate the modal age that he scored at most 15 Mark?
(b) Identify the median class
2008/8 Exercise 18.10
(c) What is the maximum age limit of the group?
The ages in years, of 50 teachers in a school are given below:
Solution
21 37 49 27 49 42 26 33 46 40
Age(year) Class boundaries frequency
50 29 23 24 29 31 36 22 27 38
10 – 14 9.5 – 14.5 3
30 26 42 39 34 23 21 32 41 46
15 – 19 14.5 – 19.5 8
46 31 33 29 28 43 47 40 34 44
20 – 24 19.5 – 24.5 16
26 38 34 49 45 27 25 33 39 40
25 – 29 24.5 – 29 .5 26
(a) Form a frequency distribution of the data using the
30 – 34 29.5 – 34.5 18
intervals: 21 – 25, 26 – 30, 31 – 35 e.t.c
35 – 39 34.5 – 39.5 12
(b) Draw the histogram of the distribution
40 – 44 39.5 – 44.5 6
(c) Use your histogram to estimate the mode
(a) i histogram is shown on page 260 2005/11 (New) Exercise 18.11
ii 24.5 + 2.8 = 27.3 years The table shows the age distributions of the members of a club
(b) Our total frequency is 89 and half of (89 + 1) = 90 is 45 Age (year) 10– 14 15 –19 20 –24 25 –29 30 –34 35 –39
and by cumulative frequency 3+8+16+26: 45 falls here Frequency 7 18 25 17 9 4
Thus median class 25–29. b ( i ) Draw a histogram to illustrate the information
(c) 44.5 years ( ii ) Use the histogram to estimate the modal age
Recall that the other name for class boundaries is c If a member is selected at random, what is the probability
true class limits that he/she is in the modal class?
259
260
Cumulative Frequency Curve (O - give) (ii) Minimum pass mark if 5% of the candidates are to
Cumulative frequency curve or O – give is a statistical be considered for appointment
graph gotten by plotting the upper class boundaries Solution
against cumulative frequencies. It is used to determine Marks Class boundaries f cf
among others: 11 – 20 10.5 – 20.5 9 9
Median 21 – 30 20.5 – 30.5 25 34
Percentiles (100 divisions) 31 – 40 30.5 – 40.5 43 77
Deciles (10 divisions) 41 – 50 40.5 – 50.5 49 126
Quartiles (4 divisions) 51 – 60 50.5 – 60.5 29 155
61 – 70 60.5 – 70.5 22 177
71 – 80 70.5 – 80.5 10 187
Median in O – give 81 – 90 80.5 – 90.5 8 195
The median is simply half of the cumulative frequency 91 – 100 90.5 – 100.5 5 200
traced to the curve then to the class boundaries to get (a) ii Graph is shown on page 262
the required value. Also the 50th percentile is a measure of
median likewise the 5th Deciles (D5) and 2nd Quartile (Q2) b (i) inter-quartile range = Q3 – Q1 = 58.5 – 34.5 = 24
(ii) 5% of the candidates to be considered
Percentile implies 100 – 5 = 95% of the candidate
This is the division of the cumulative frequency into 95
100 points. For instance =  200 = 190
100
75% = 75 x cumulative frequency Next, we trace 190 at Cf – axis (y) to the o-give
100
then to the mark – axis (x): 84.5
20% = 20 x cumulative frequency
100 2007/11 Neco
Then we trace the required values in the graph as discussed in median The table below shows the marks scored by a group of
Deciles students in a test
This is the division of the cumulative frequency Marks 1– 10 11–20 21–30 31–40 41–50
into 10 portions. Freq 4 6 9 12 20
1 51– 60 61– 70 71– 80 81– 90 91– 100
For instance , D1 = × cumulative frequency
10
15 7 5 0 2
5
D5 = × cumulative frequency (a) Construct the cumulative frequency table (b) Draw the o-give
10
(c) From your o-give, find the: (i) Median (ii) Lower quartile
Quartiles (d) A student was picked at random from the group,
This is the division of the cumulative frequency what is the probability that the student (using o-give)
into 4 portions. Where: (i) Obtained a distinction grade of 75% and above
1 (ii) failed the test if the pass mark is 40%
(Lower quartiles ) Q1 = of cumulative frequency Solution
4
Marks Class boundaries f cf
2 1 – 10 0.5 – 10.5 4 4
(Middle quartile) Q2 = of cumulative frequency
4 11 – 20 10.5 – 20.5 6 10
3 21 – 30 20.5 – 30.5 9 19
(Upper quartile) Q3 = of cumulative frequency 31– 40 30.5 – 40.5 12 31
4
Inter quartile range = Q3 – Q1 41 – 50 40.5 – 50.5 20 51
51 – 60 50.5 – 60.5 15 66
Q3 − Q1
Semi – inter quartile range = 61 – 70 60.5 – 70.5 7 73
2 71 – 80 70.5 – 80.5 5 78
81 – 90 80.5 – 90.5 0 78
2006/10 (Nov) 91 – 100 90.5 – 100.5 2 80
The table shows the distribution of marks obtained by C (i) Median mark: Half of the Cf traced to the mark axis:
200 candidates in a selection test for appointment 1 of 80 = 40 is traced to the mark axis = 45.5
Marks 11–20 21–30 31– 40 41– 50 51– 60 2
(ii) Lower quartile Q1 = 1 of 80 = 20
No of 9 25 43 49 29 4
Cand. Trace Cf = 20 to the curve then to mark axis: 31.5
61– 70 71– 80 81– 90 91– 100 (d)(i) This is the number of students that scored 75% to
22 10 8 5 100% divided by 80
Trace 75 at x –axis to the curve then to the Cf – axis : 77
(a) (i) Construct a cumulative frequency table for the 77
Pr (75% and above) =
distribution 80
(ii) Draw a cumulative frequency curve to represent (ii) Trace 40 at the mark – axis to the curve then
the information to Cf – axis which will give 30
(b) Use your curve to find the: 30 3
Pr(failure at 40% mark) = =
(i) Inter-quartile range: 80 8
261
262
2015/13 Solution
The table shows the marks scored by some candidates Marks Class boundaries f cf
in an examination. 1 – 10 0.5 – 10.5 6 6
Marks (%) 0– 9 10–19 20–29 30–39 40–49 11 – 20 10.5 – 20.5 9 15
Frequency 7 11 17 20 29 21 – 30 20.5 – 30.5 12 27
31 – 40 30.5 – 40.5 19 46
50–59 60–69 70–79 80–89 90–99 41 – 50 40.5 – 50.5 28 74
34 30 25 21 6 51 – 60 50.5 – 60.5 37 111
61 – 70 60.5 – 70.5 41 152
(a) Construct a cumulative frequency table for the
71 – 80 70.5 – 80.5 28 180
distribution and draw a cumulative frequency curve 81 – 90 80.5 – 90.5 13 193
(b)Use the curve to estimate, correct to one decimal 91 – 100 90.5 – 100.5 7 200
place, the
(i) lowest mark for distribution if 5% of the candidates Graph is shown on page 264
passed with distinction (i) Median mark: Half of the Cf traced to the mark axis:
1 of 200 is 100 traced to the Mark axis = 58.0
(ii) probability of selecting a candidate who scored 2
at most 45% (ii) 80% pass implies 20% fail
Solution 20
 200 = 40;
Marks (%) Class boundaries f cf 100
0–9 0 – 9.5 7 7 Trace Cf = 40 to the o-give then to Mark axis = 37.5
10 – 19 9.5 – 19.5 11 18 (iii) It is the same as 100 – 45 = 55% of 200
20 – 29 19.5 – 29.5 17 35 55
 200 = 110
30 – 39 29.5 – 39.5 20 55 100
40 – 49 39.5 – 49.5 29 84 Trace Cf =110 to the o-give then to the Mark axis= 60.5
50 – 59 49.5 – 59.5 34 118
2006/12 Neco Exercise 18.12
60 – 69 59.5 – 69.5 30 148
The following are the ages of 50 workers who worked in a factory
70 – 79 69.5 – 79.5 25 173 years back:
80 – 89 79.5 – 89.5 21 194 71 79 46 35 25 28 56 82 52 68
90 – 99 89.5 – 99.5 6 200 68 64 93 95 78 43 58 72 57 60
50 98 62 63 70 73 53 44 86 79
Graph is shown on page 264 72 68 88 51 32 59 72 73 46 85
(b) i. Distribution if 5% of candidates passed implies 40 55 52 61 96 67 82 72 48 59
100 – 5 = 95% (a) Using class intervals 21–30, 31– 40, 41–50, …
95 draw a frequency table for the distribution
Next,  200 = 190 (b) Draw the cumulative frequency curve (o-give)
100
and determine
Trace 190 at cf to the o–give then to the Mark – axis = 85.5
(i) the median (ii) inter – quartile range
ii. At most 45% mark is 45% or less (c) If a worker is picked at random, what is the
Trace 45 on the Mark – axis to the curve probability that he is more than 60 years?
then to the cf – axis = 70
70 7 2009/11 Neco (Dec) Exercise 18.13
Pr (at most 45%) = = The table below shows the height of maize plants in a given farm
200 20
Height of plants No of plants
45 – 49 10
2014/10 Neco 50 – 54 36
The table below shows the marks scored by 200 55 – 59 64
candidates in a certain examination 60 – 64 52
Marks 1– 10 11–20 21–30 31–40 41–50 65 – 69 28
No. of 6 9 12 19 28 70 – 74 10
Cand. (i) Construct the cumulative frequency table
(ii) Draw a cumulative frequency curve (o-give) for the
51– 60 61– 70 71– 80 81– 90 91– 100 distribution using a scale of 2cm = 20 units on
the y – axis and 2cm = 5 units on the x – axis
37 41 28 13 7
(iii) Use your graph to determine the inter-quartile range
2005/11 (old) Exercise 18.14
(a) Construct a
The table shows the examination marks for 120 students
(i) Cumulative frequency table
Mark 30–39 40– 49 50–59 60– 69 70–79 80– 89
(ii) Cumulative frequency curve (o-give) of the Freq 11 25 45 15 10 14
distribution
(a) i. Draw the cumulative frequency table for the
(b) From your o-give, estimate the:
distribution
(i) Median mark; ii. Draw the cumulative frequency curve
(ii) Pass mark if 80% of the candidates passed the (b) Use the cumulative frequency curve to estimate:
examination i. the median ii. the upper quartile
(iii) Minimum mark scored by the top 45% of the (c) What is the probability that a student chosen at
candidates random scored between 50 and 69 marks
263
264
Linear equations ( x2 y2 ) ( x1 y1 )
The general form of a linear equation is y = mx + c,
where m is the gradient and x is the independent
variable; c is the intercept at the y – axis.
When there is no gradient we could have two major
types of line such as:
( x1 y1 ) ( x2 y2 )
y y
y=c

x=c
and
+C
Gradient = y 2 – y1
x x2 – x1
+C
From the above table note that : y2 , y1 and x1 , x2 positions
x
differ for positive and negative gradients

Simultaneous linear equations graphical solution


This is the case where two straight lines graphs are drawn on same
Alternatively, it may occur as: graph sheet and their point of intersection traced to the x and y
y and y axes to give a coordinate value (x, y) which is the solution
x=-c

2007/22 Neco
x The graphical solution of simultaneous equations
x is represented on the graph below. What is the solution of
y=-c -C
the simultaneous equation?
-C
10
y

Where the gradient exist ; then it may be any of the


forms given below:
6
y
y

OR c 4
c

x x 2

A straight line where we can form a triangle under it or on it.


-8 -6 -4 -2
0
0 2
x 4

There are basically two types of gradients namely: -2


Positive and Negative gradients
Gradient (slope) =
Change in y A (–2, 6) B (6, –2) C (–2, –6) D (0, 6) E (4, 0)
Change in x Solution
The solution is the point of intersection of the two lines
Positive gradient Negative gradient traced to the x – axis and y – axis
Increase in y Decrease in y x = – 2 and y = 6 (–2, 6) A
Increase in x Increase in x 2015/ 21 and 22
The table below shows some values of a linear graph.
In physics it is called - In physics it is called -
acceleration deceleration
x 0 1 14 2 4
since y increases as since y decreases as y 3 5 12
x increases x increases
Use it to answer questions 21 and 22
y-axis starts at the down part y-axis starts at the upper part 21 Find the gradient of the line
and ends at the top and ends at the down part A1 B2 C 3 D4
Solution
y 2 − y1
Gradient =
x 2 − x1

265
5.5 − 3 2.5 (ii) For us to find y when x = 4, we have to extend the
= = =2 (B)
1.25 − 0 1.25 straight line as shown by dotted line. A straight line can be
22. What is the value of y when x = 2 extended anytime without knowing the equation of the line
A5 B7 C9 D 11 unlike curves
Solution Thus when x = 4, y = 3.6
For us to know the value of y when x = 2 (iii) y = mx + c equation of straight line with Gradient (m)
First, we find the equation for the line from: y 2 − y1
y = mx + c =
x 2 − x1
Here m is the gradient which is gotten earlier as 2
Next we find C 2 − (−3) 2+3 5
= = = = 1.5
When x = 0, y = 3, from the table 3 − (−0.3) 3 + 0.3 3 .3
y = mx + c becomes (iv) Where the line cut across the y – axis C = – 2.5
3 = 0 + c thus c = 3 Substituting
Thus our line equation is y = 2x + 3 y = 1.5x – 2.5
When x = 2 2y = 3x – 5
y = 2(2) + 3
=4+3 2010/39 and 40
=7 (B) x 0 2 4 6
2005/26 Neco y 1 2 3 4
Find the solution of the simultaneous equations in the
graph below The table is for the relation y = mx + c where m and c
are constants. Use it to answer questions 39 and 40
4
y 39. What is the equation of the line described in the table?
3 A y = 2x B y = x + 1 C y = x D y = 12 x + 1
Solution
2 y = mx + c
y 2 − y1
m(gradient) = (take any two points)
1 x 2 − x1
4 −1 3 1
0 = = =
-2 -1 0 1 x 6−0 6 2
To find c, we take any coordinate
-1
When x = 2, y = 2
y = mx + c becomes
-2
2 = 12 (2) + c
-3
2=1+c
2–1=c thus c = 1
-4
The line is y = 12 x + 1 (D)
40. What is the value of x when y = 5?
A (–2, 3) B (–2, –5) C (–2, –3) D (–3, –2) E (2, –3)
A8 B9 C 10 D 11
Solution Solution
The solution is the point of intersection between the two When y = 5
lines: x = – 2 and y = – 3 y= 1
x + 1 becomes
(–2, –3) C 2
1
5= 2
x+1
2005/8b (old) adjusted 5–1= 1
2
x
The table below gives the relationship between x and y 1
4= x
x –1 0 1 2 3 2

y – 4.0 – 2.5 – 1.0 0.5 2.0 42=x Thus 8 = x ( A )

( i ) Using a scale of 2cm to 1unit on y-axis and 2cm to


0.5unit on x-axis , use the table to draw the graph
of the relationship between x and y
(ii) From your graph, find the value of y when x = 4
(iii) Use the graph to calculate the gradient of the line
(iv) What is the equation of the relation?
Solution
Graph is shown on page 267
(i) The straight line graph stop at x = 3 and y = 2.0

266
4
y 2005/8b (old)

0
x
-1.5 -1 -0.5 0 0.5 1 1.5 2 2.5 3 3.5 4

-1

-2

-3

-4

-5
Gradient of a Curve
The gradient of any point on a curve is the gradient of 2006/8 (Nov)
the tangent to the curve at that point. The tangent must (a) Copy and complete the table
be produced equidistant to the point x –3 –2 –1 0 1 2 3 4 5
y = 2x2 – 5x – 4 14 –4 –6 8
Quadratic Equations (b) Using the scale 2cm = 1 unit on the x –axis and
Graphical Solution 2cm to 5 units on the y – axis, draw the graph of
There are two major types of quadratic curves from the y = 2x2 – 5x – 4 for – 3  x  5
general form: ax2 + bx + c (c)Use your graph to find the:
When a is positive When a is negative
(i) roots of the equation 2x2 – 5x – 4 = 0;
i.e a < 0 (ii) Range of values of x for which y  3;
i.e a > 0
(iii) Minimum value of y
Solution
Table for y = 2x2 – 5x – 4
x –3 –2 –1 0 1 2 3 4 5
2x2 18 8 2 0 2 8 18 32 50
– 5x 15 10 5 0 –5 – 10 – 15 – 20 –25
The roots or solutions or zeros of any quadratic equation
–4 –4 –4 –4 –4 –4 –4 –4 –4 –4
may take any of the forms listed below:
Y Y
y 29 14 3 –4 –7 –6 –1 8 21
Imaginary roots
Equal roots From the table, we note that:
x – axis

Fig I
X . Fig II
X
Has almost equal positive and negative sides – 3 and + 5
Thus rule close to the centre on the y – axis
y – axis
Has very small negative side – 7
Y
Has very big positive side 29
The ruling gives more space to positive side in x – axis

. .
Two distinct roots
Fig III The graph can be ruled in the format below:

Sometimes, quadratic equation comes along with linear


equation. The following examples will illustrate the required
concepts

267
(b) Graph is on page 269 2009/12 Neco (Dec)
(c)(i) The roots of the equation 2x2 – 5x – 4 = 0 (a) Copy and complete the following table of values
is the point where the curve cut the x – axis for y = 2x2 – 9x – 1
– 0.65 and 3.15 x –1 0 1 2 3 4 5 6
(ii) Range of value of x for y  3 is gotten as: y –1 – 8 – 11 17
Draw a line at y = 3 to touch the curve, (b) Using a scale of 2cm to represent 1unit on the x– axis
then trace to x – axis – 1and 3.5, and 2cm to represent 5units on the y– axis ,
in inequality form – 1  x  3.5 draw the graph of y = 2x2 – 9x – 1 for – 1  x  6.
(iii) The lowest position of the curve traced to y – axis (c) Use your graph to find the
as shown by dotted line – 7. (i) roots of the equation 2x2 – 9x = 4 ,
correct to one decimal place
2006/10 NABTEB (ii) gradient of the curve y = 2x2 – 9x – 1 at x = 3
(b) solve graphically, the simultaneous equation: Solution
y = x2 – 7x + 10 and y = x + 3 using the interval Table for y = 2x2 – 9x – 1
0  x  8 and a scale of 2cm to 1unit on the x – axis x –1 0 1 2 3 4 5 6
and 1cm to 2units on the y – axis 2x 2
2 0 2 8 18 32 50 72
(c) Use your graph in (b) to find the roots of –9x 9 0 – 9 –18 –27 –36 –45 –54
(i) x2 – 7x + 10 = 0 –1 –1 –1 –1 –1 –1 –1 –1 –1
(ii) x2 – 7x + 5 = 0 y 10 – 1 – 8 –11 –10 – 5 4 17
Solution
Table for y = x2 – 7x + 10 From the table, we observed that
x – axis
x 0 1 2 3 4 5 6 7 8 Has very small negative value of –1and big positive + 6
x2 0 1 4 9 16 25 36 49 64
– 7x 0 –7 –14 –21 –28 –35 –42 – 49 –56
We rule y – axis with more space for positive side
+10 +10 +10 +10 +10 +10 +10 +10 +10 +10 y – axis
y 10 4 0 –2 –2 0 4 10 18 Has positive side + 17 and negative side – 11
We rule the x – axis with little more space for the positive side.
From the table, it will be observed that
x – axis Thus, we rule the graph as
It has only positive axis
We rule y – axis from the beginning of the graph
y– axis
Has very small negative side – 2
Has very big positive side 18
We rule the x – axis with much space for the positive side.
Thus the graph is to be ruled in the format below: (b) The graph is shown on page 270
(c)i The roots of the equation 2x2 – 9x = 4 is gotten as:
Rearrange the equation
2x2 – 9x = 4
2x – 9x – 4 = 0
2

Add 3 to both sides to get our original equation


2x2 – 9x – 4 + 3 = 3
Table for y = x + 3 2x2 – 9x – 1 = 3
x 0 2 5 On the graph, draw a line at point y = 3 to cut the curve.
x 0 2 5 These (two) points traced to x – axis gives the solution
+3 +3 +3 +3 – 0.4 and 4.9
(c)ii At point x = 3 traced to the curve, we draw an
y 3 5 8
equidistant line as shown, then take the slope
Graph is shown on page 269 Change in y y −y
C ( i ) 2 and 5 the two points of intersection traced to Slope = = 2 1
Change in x x 2 − x1
the x – axis
(ii) x – 7x + 5 = 0
2 −5 − (−15)
=
What must be added to both sides to make it the original 4.7 − 1.3
equation x2 – 7x + 10 10
= = 2.94
x2 – 7x + 5 + 5 = 5 3 .4
x2 – 7x + 10 = 5
Next, we rule a straight line at y = 5 where it touches
the curve are traced to the x – axis as shown by dotted
lines 0.8 and 6.2

268
y35 2006/8 ( Nov)
z

30

25

20

15

y = 2x2 – 5x – 4
10

0
-4 -3 -2 -1 0 1 2 3 4 5
x6
-5

-10

20
y 2006/10 NABTEB
y = x2 – 7x + 10
18

16

14

12

y= x+3
10

0
0 1 2 3 4 5 6 7 8 x9
-2

-4

269
20
y 2009/12 Neco ( Dec)

15

y = 2x2 – 9x – 1

10

0
-2 -1 0 1 2 3 4 5 6 x7

-5

-10

-15

2010/10 Graph 4 y

0
-4 -3 -2 -1 0 1 2 3
x 4

-2

-4
y = – x2 + x + 2

-6

-8

-10

-12

270
2010/10 Solution
(a) Copy and complete the table of values for the Table for y = x2 – 2
relation y = – x2 + x +2 for – 3  x  3 x –3 –2 –1 0 1 2 3 4
2
x –3 –2 –1 0 1 2 3 x 9 4 1 0 1 4 9 16
y –4 2 –4 –2 –2 –2 –2 –2 –2 –2 –2 –2
y 7 2 –1 –2 –1 2 7 14
(b) Using scales of 2cm to 1unit on the x – axis and
2cm to 2 units on the y – axis, From the table, we observed that:
draw a graph of the relation y = – x2 + x + 2. x – axis
(c) From the graph, find the; Has almost equal sides – 3 and + 4
(i) minimum value of y; We rule the y – axis at the centre
(ii) root of the equation x2 – x – 2 = 0 y – axis
(iii) Gradient of curve at x = – 0.5 Has small negative side – 2 and very big positive side 14
Solution We rule x – axis with much space for the positive side
Table for y = – x2 + x +2 Thus the graph is to be ruled in the format below:
x –3 –2 –1 0 1 2 3
– x2 –9 –4 –1 0 –1 –4 –9
+x –3 –2 –1 0 1 2 3
+2 +2 +2 +2 +2 +2 +2 +2
y – 10 –4 0 2 2 0 –4

From the table, we observe that (b) Graph to be drawn


x – axis (c) i. The point where the curve cuts the x – axis
Has equal sides – 3 and + 3 – 1.4 and 1.4
We rule the y – axis at the centre ii. x2 – 3 = 0
y – axis What can be added to both sides to get y = x2 – 2
Has very small positive side + 2 x2 – 3 + 1 = + 1
Has bigger negative side – 10 x2 – 2 = 1
We rule the x – axis with much space for the negative side Next, we rule a straight line at y = 1 on the graph,
Thus the graph should be ruled as: where it cuts the curve is traced to x – axis with dotted line
as shown – 1.7 and 1.7
iii. Trace the point x = – 1 to the curve and
draw an equidistant line there
change in y y −y
Gradient = = 2 1,
change in x x 2 − x1
(b) Graph is shown on page 270 −4 − 2 −6
(c) i. Typing error from the question : = = = – 1.94
0.5 − (−2.6) 3 .1
This type of graph does not have maximum value
it has maximum value 2.3 2015/7
ii. – 1 and 2 where the curve cut the x – axis The table is for the relation y = px2 – 5x + q
iii. At x = – 0.5, we trace it to the curve and
draw an equidistant line as show by dotted line x –3 –2 –1 0 1 2 3 4 5
y 21 6 –12 0 13
change in y
Gradient =
change in x (a) (i) use the table to find the values of p and q
4 − (−2) 4+2 6 (ii) Copy and complete the table
= = = = 2.07 (b) Using scales of 2cm to 1unit on the x – axis and
0.8 − (−2.1) 0.8 + 2.1 2 .9
2cm to 5 units on the y – axis,
draw the graph of the relation for – 3  x  5
2009/ 11 (Nov)
(c) Use the graph to find
(a) Copy and complete the table of values for
(i) y when x =1.8
y = x2 – 2 for – 3  x  4 (ii) x when y = – 8
x –3 –2 –1 0 1 2 3 4 Solution
y 2 –2 2 a (i) substituting any 2 sets of values from the table
(b) Using a scale of 2cm to 1unit on the x – axis and x –3 –2
a scale of 2cm to 2units on y–axis, y 21 6
draw the graph of y = x2 – 2
(c) Use your graph to find the: into y = px2 – 5x + q
(i) roots of the equation x2 – 2 = 0 21 = p(–3)2 – 5(–3) + q
(ii) values of x for which x2 – 3 = 0 21 = 9p +15 + q
(iii) gradient of the curve at the point where x = –1 6 = 9p + q ----- (1)
270
25 y
2015/7 graph

20

15 y = px2 – 5x + q

10

0
-4 -3 -2 -1 0 1 2 3 4 5 x6

-5

-10

-15

-20

20 2009/10 Neco
y

15 y = x2 – 5x + 1
2 2

10

y = x/2 + 3

0
-4 -3 -2 -1 0 1 2 3 4 5 x6

-5

272
Also
6 = p(–2)2 – 5(–2) + q Solution
6 = 4p + 10 + q 5x 1
Table for y = x 2 − +
– 4 = 4p + q ---- (2) 2 2
Solving the resulting simultaneous liner equations x –3 –2 –1 0 1 2 3 4 5
6 = 9p + q x2 9 4 1 0 1 4 9 16 25
– ( – 4 = 4p + q ) –5x/2 7.5 5 2.5 0 –2.5 –5 –7.5 –10 –12.5
+ 1/2 0.5 0.5 0.5 0.5 0.5 0.5 0.5 0.5 0.5
10 = 5p y 17 9.5 4 0.5 –1 –0.5 2 6.5 13
p=2
Substitute p value into (1) From the table, we observed that:
6 = 9p + q becomes x – axis
6 = 9 (2) + q Has a little more positive side of +5
6 = 18 + q Has a small negative side of – 3
6 – 18 = q,  q = – 12 We rule the y –axis with little more space to positive side
Thus the equation is y = 2x2 – 5x – 12 y – axis
Has a very small negative side – 1
a ( ii )Table for y = 2x2 – 5x – 12 Has a very big positive side 17
x –3 –2 –1 0 1 2 3 4 5 We rule the x – axis with much space for positive side
2x2 18 8 2 0 2 8 18 32 50 The graph should be ruled in the format below:
–5x 15 10 5 0 –5 – –15 –20 –25
–12 –12 –12 –12 –12 –12 10 – –12 –12
–12 12
y 21 6 –5 –12 –15 –14 –9 0 13

From the table, it will be noted that


x – axis
Has a little higher positive side +5 than negative side of –3
We rule the y – axis with little more space for positive side (b) Graph is shown on page 272
y – axis (c) Table for y =
x
+ 3
Has a little higher positive side +21than negative side of -15 2
We rule the x – axis with little more space for positive side x –2 0 2
Thus the graph is to be ruled in the format below
x
/2 –1 0 1
+3 +3 +3 +3
y 2 3 4
The straight line is shown on the graph
(d) i. lowest point of the curve: x = 1.25 and y = 1
ii. – 0.7 and 3.7 are the points of intersection of the
curve and straight line traced to x – axis
(b) Graph is shown on page 272
(c) i. At point x = 1.8, a dotted line is traced to the 2014/9 (Nov)
curve then traced to the y – axis – 14.5 An object is thrown vertically upwards from the top of a cliff
ii. A straight line is ruled at y = – 8 to touch the curve and its height y metres, above sea level after t second is
and then traced to x – axis. – 0.65 and 3.15 given by y = – 16t2 + 64t + 5
(a) Copy and complete the table of values for
2009/10 Neco
y = – 16t2 + 64t + 5 ; 0  t  4.0
(a) Copy and complete the following table for the
5x 1 t 0.0 0.5 1.0 1.5 2.0 2.5 3.0 3.5 4.0
relation y = x 2 − + y 5 65 53
2 2
x –3 –2 –1 0 1 2 3 4 5 (b) Using scales of 2cm to 0.5second on the t – axis and
y 4 0.5 –1 0.5 2cm to 10m on the y – axis draw the graph of
(b) Using a scale of 2cm to 1unit on the x – axis and y = – 16t2 + 64t + 5 for 0  t  4.0
2cm to 5units on the y – axis, draw the graph of (c) Use the graph to find the:
i. height reached when t = 1.75 seconds
5x 1
y = x2 − + ii. times the object was at a height of 50m
2 2 iii. maximum height reached.
(c) Using the same scale and axes, draw the graph of Solution
x (a) Table for y = – 16t2 + 64t + 5
y = + 3
2 t 0.0 0.5 1.0 1.5 2.0 2.5 3.0 3.5 4.0
(d) From your graph, determine the: –16t2 0 –4 –16 –36 –64 -100 – 144 –196 –256
i. least value of y and the corresponding values of x: 64t 0 32 64 96 128 160 192 224 256
+5 +5 +5 +5 +5 +5 +5 +5 +5 +5
5x 1 x
ii. Solution of the equations. x 2 − + = + 3 y 5 33 53 65 69 65 53 33 5
2 2 2
From the table, it will be noted that
273
x – axis (t) (c) Table for y = 4x – 2
Has only positive side from 0.0 to 4.0 x –2 0 3
we rule the y –axis at the beginning part 4x –8 0 12
y – axis –2 –2 –2 –2
Has only positive side from 5 to 69 y – 10 –2 10
we rule the x –axis at the beginning part
Thus the graph is to be ruled in the format below The straight line graph is as plotted on the graph
(d) it is the point of intersection traced to the x – axis as
shown by dotted lines – 1.24 and 1.6
(e) The highest point of the curve traced to y – axis 6

2005/13 (New)
(b) The graph is shown on page 275 (a) Copy and complete the table y = x2 –2x – 2 for – 4 x  4
(c) i. 68m is indicated by dotted line
x –4 –3 –2 –1 0 1 2 3 4
t = 1.75 traced to the curve then to y – axis
y 22 –2 1 6
ii. 0.9 and 3.08 seconds is indicated by dotted line
y = 50 traced to the curve then to t – axis (b) Using a scale of 2cm to 1unit on the x – axis and
iii. 69m is the highest point of the curve traced 2cm to 5units on y– axis, draw the graph of
to the y – axis y = x2 – 2x – 2
(c) Use your graph to find:
2014/12 Neco (i) the roots of the equation x2 – 2x – 2 = 0
(a) Draw the graph of y = 4 + 5x – 3x2 for which (ii) the values of x for which x2 – 2x – 41/2 = 0
– 2  x  3. Use a scale of 2cm to represent 1unit on (iii) the equation of the line of symmetry of the curve
the x – axis and 1cm to represent 2units on the y – axis Solution
(b) Find the range of values of x for which 4 + 5x – 3x2 Table for y = x2 – 2x – 2
is greater than zero x – 4 – 3 – 2 –1 0 1 2 3 4
2
(c) Using the same scale and axes, x 16 9 4 1 0 1 4 9 16
draw the graph of y = 4x – 2 –2x 8 6 4 2 0 –2 –4 –6 –8
(d) Write down the solution of the equations which are –2 –2 – 2 – 2 – 2 – 2 – 2 – 2 – 2 –2
given by the point of intersection of the two graphs y 22 13 6 1 –2 –3 –2 1 6
(e) Find the maximum value of y = 4 + 5x – 3x2 From the table we observe that
Solution x – axis
Table for y = 4 + 5x – 3x2 Has equal sides – 4 and +4
x –2 –1 0 1 2 3 We rule the y– axis at the centre
4 4 4 4 4 4 4 y – axis
+5x – 10 –5 0 5 10 15 Has small negative sides –3 and very big positive side 22
–3x2 – 12 –3 0 –3 –12 –27 We rule the x – axis with much space for the positive side.
y –18 –4 4 6 2 –8 Thus the graph can be ruled in the format below
From the table, it will be noted that
x – axis
Has almost equal sides of – 2 and + 3
We rule the y – axis almost at the centre
y – axis
Has higher negative axis –18 than positive side of + 6 (b) The graph is shown on page 276
We rule the x – axis with more space for the negative side (c) i. Roots of x2 – 2x – 2 = 0 is the point where the curve
Thus the graph can be ruled in the format below cuts the x – axis – 0.75 and 3.3
ii. value of x for which x 2 – 2x – 41/2 = 0
What must be added to the equation to get the original one
x2 – 2x – 4.5 + 2.5 = 2.5
we just added 2.5 to both sides
x2 – 2x – 2 = 2.5
(a) The graph is shown on page 275 Next, we rule a straight line at y = 2.5 in the graph
(b) This is simply the points where the curve cuts the where it touches the curve is traced to the x – axis as shown
x – axis i.e the upper portion of the curve above the by dotted line – 1.35 and 3.3
x –axis; it is also the roots presented in inequality form iii. Line of symmetry is the straight line that divides the
– 0.6 < x < 2.25 graph into two equal halves as shown by dotted line
x=1

274
y80
2014/9(Nov)

70

60

50

40

30

20
y = –16t2 + 16t + 5

10

0
0 0.5 1 1.5 2 2.5 3 3.5 4 t 4.5

8
2014/12 Neco y
6 y = 4x – 2

0
-3 -2 -1 0 1 2 3
x4
-2

-4

-6

-8 y = 4+ 5x –3x2
+q
-10

-12

-14

-16

-18

-20

275
25
y 2005/13(New)

y = x2 – 2x – 2
20

15

10

0
-5 -4 -3 -2 -1 0 1 2 3 4
x5

-5

8 y
2008/10 NABTEB (Nov)
6

4
9y = 20x – 40
2

0
-4 -3 -2 -1 0 1 2 3 4
x5
-2

-4

-6

-8

-10

-12

-14

-16

-18

-20
y = 6 + x – 2x2
-22

276
2008/10 NABTEB (Nov) (c) From your graph, find the least point of the
(a) Copy and complete the following table for values relation y = 2x2 – 5x – 7
of y = 6 + x – 2x2 (d) Using the same scale and axes, draw the graph of y = x–2
x –3 –2 –1 0 1 2 3 4 (e) Use your graph to solve the equation 2x2 – 6x – 5 = 0
y – 15 6 0 – 22 Solution
( a ) Table for y = 2x2 – 5x – 7
( i ) Draw the graph of y = 6 + x – 2x2 for values of x
from –3 to +4 taking 2cm to represent 1unit on x –2 –1 0 1 2 3 4
2
x–axis and 2cm to represent 2units on y – axis 2x 8 2 0 2 8 18 32
( ii ) On the same axes and with same scale draw the – 5x 10 5 0 – 5 – 10 – 15 – 20
graph of 9y = 20x – 40 –7 –7 –7 –7 –7 –7 –7 –7
( iii ) From your graphs, determine the value of x y 11 0 – 7 – 10 – 9 –4 5
for which 6 + x – 2x2 = 19 (20x – 40) From the table, we observed that
( iv ) Find the maximum value of y x – axis
Solution Has small negative side – 2 and times two positive side 4
( a ) Table for y = 6 + x – 2x2 We rule the y – axis line in that order
y – axis
x –3 –2 –1 0 1 2 3 4
Have almost equal sides of –10 and 11
6 6 6 6 6 6 6 6 6 We rule the x – axis line at the centre
+x –3 –2 –1 0 1 2 3 4 This graph is ruled in the format below
– 2x 2
– 18 – 8 – 2 0 –2 –8 – 18 – 32

y - 15 - 4 3 6 5 0 -9 -22
From the table, it should be noted that:
x – axis
Has almost equal sides – 3 and + 4
we rule y – axis line almost at the centre (b) Graph is shown on page 278
y – axis (c) The lowest point of curve traced to the y – axis
Has small positive side +6 and very big negative side – 22 as shown by dotted line y = – 10.2
We rule the x – axis line with much space for the negative side (d) Table for y = x – 2
Thus the graph is to be ruled in the format below: x –2 0 2
x –2 0 2
–2 –2 –2 –2
y –4 –2 0
The straight line is as plotted on the graph

( i ) Graph is shown on page 276 (e) 2x2 – 6x – 5 = 0 is a combination of


20 x 40 y = 2x2 – 5x – 7 and y = x – 2 as:
(ii) Table for y = − 2x2 – 5x – 7 = x – 2
9 9
Collect like terms together
x –3 0 3 2x2 – 5x – x – 7 + 2 = 0
20x – 6.67 0 6.67 2x2 – 6x – 5 = 0
9 Thus the solution to 2x2 – 6x – 5 = 0 is the point of
– 40/9 – 4.44 – 4.44 – 4.44 intersection of the curve and the straight line traced to
y – 11.11 – 4.44 2.23 the x – axis as shown by dotted lines – 0.7 and 3.65
The straight is as shown on the graph 2010/12 Neco
(iii) This is the points of intersection of the straight line (a) Copy and complete the table of values for the
and the curve as shown by the dotted lines relation y = 2x2 – 3 for – 3  x  3
– 2.58 and 2
(iv) It is the highest point on the curve 6.2 x –3 –2 –1 0 1 2 3
y 5 15
2013/10 Neco
(a) The table below shows the relation y = 2x2 – 5x – 7, (b) Draw the graph of y = 2x2 – 3 using 2cm to 1unit
copy and complete the table on x – axis and 2cm to 5units on y – axis
(c) From the graph, find the
x –2 –1 0 1 2 3 4
(i) roots of 2x2 – 3 = 0
y 11 –7 –4
(ii) least value of y and the corresponding value of x.
(b) Using a scale of 2cm to 1unit on x – axis and (d) Using the same axes, draw the graph of 2y – x = 2
2cm to 2 units on y –axis, draw the graph of the (e) Use your graphs to find the values of x for which
relation. 2x2 – 3 = 12 (x + 2)

277
14
y
2013/10 Neco
y = 2x – 5x – 7
2 12

10

0
-3 -2 -1 0 1 2 3 4 x5
-2

-4
y = x– 2
-6

-8

-10

-12

20
2010/12 Neco y

15
y = 2x2 – 3

10

2y – x = 2

0
-4 -3 -2 -1 0 1 2 3
x4

-5

278
Solution y – axis
Table for y = 2x2 – 3 Have almost equal sides – 10 and 11
x –3 –2 –1 0 1 2 3 We rule the x – axis line almost at the centre
2x2 18 8 2 0 2 8 18 Thus the graph can be ruled in the format below:
–3 –3 –3 –3 –3 –3 –3 –3
y 15 5 –1 –3 –1 5 15
From the table, it will be noted that:
x – axis
Has equal positive and negative values –3 and + 3
We rule the y – axis line at the centre (b) Graph is shown on page 280
y – axis (c) Table for y = 3x + 1
Has small negative value of –3 bigger positive value of 15
We rule the x – axis line with more space for positive side
x –1 0 3
Thus the graph is to be ruled in the format below 3x –3 0 9
+1 +1 +1 +1
y –2 1 10
The straight line is shown on the graph
(d) i. the lowest point of curve, y = – 10.1 and x = 1.75
ii. This is the points of intersection of the straight line
(b) Graph is shown on page 278 and the curve traced to the x – axis.
(c) i. Roots of 2x2 – 3 = 0 are – 1.20 and 1.20 Here the two only meet at one point – 0.45
where the curve cuts the x – axis Candidates don’t need to bother themselves when such cases
ii. The lowest point of the curve traced to y and x – axis arise in exam. Work strictly by the question – No addition
x = 0 and y = – 3 No subtraction; any part of the question not clear – leave it
x alone.
(d) Table for y = +1 (When we make y subject formula)
2
For Practice only
x –2 0 2
If we extend the range of values from –1 to 5 to –1 to 6
x
/2 –1 0 1
(–1  x  6).
+1 1 +1 +1
Table for y = 2x2 – 7x – 4
y 0 1 2
x –1 0 1 2 3 4 5 6
(e) – 1.3 and 1.55 2x2 2 0 2 8 18 32 50 72
– 7x 7 0 –7 – 14 – 21 – 28 – 35 – 42
It is the point of intersection of the curve and the line
traced to x – axis as shown by dotted lines –4 –4 –4 –4 –4 –4 –4 –4 –4
y 5 –4 –9 – 10 –7 0 11 26
2006/11 NABTEB (Nov) We don’t need to extend the table for the straight line, as straight
(a) Construct a table of values for –1 x  5 for the can be extended to any given length with only two points.
function y = 2x2 – 7x – 4 The resulting graph is shown on page 280 will give us a better
(b) Using your table of values, plot the graph of result for (d) ii – 0.45 and 5.4
y = 2x2 – 7x – 4 taking 2cm to represent 1unit and
4units on the x – axis and y – axis respectively. 2007/9 Neco
(c) On the same axes and with the same scale, Copy and complete the table for y = 2x2 – 4x – 5
draw the graph of y = 3x + 1 and – 2  x  4
(d) Use your graph to find the: x –2 –1 0 1 2 3 4
(i) least value of y = 2x2 – 7x – 4 and y = 2x2 – 4x – 5 –7 1
the corresponding value(s) of x
(ii) roots of the equation 2x2 – 10x – 5 = 0 (a) Use the table to draw the graph of y = 2x2 – 4x – 5
Solution (b) On the same axes draw the graph of y = 2x + 1
(a) Table for y = 2x2 – 7x – 4 (c) Use the graph to solve
(i) 2x2 – 4x – 5 = 0
x –1 0 1 2 3 4 5
(ii) 2x2 – 4x – 5 = 4
2x2 2 0 2 8 18 32 50
(d) Use the graph to solve x2 – 3x – 3 = 0
–7x 7 0 – 7 –14 –21 –28 –35
(e) What is the minimum value of y and the corresponding value of x?
–4 –4 –4 –4 –4 –4 –4 –4
Solution
y 5 – 4 – 9 –10 –7 0 11 Table for y = 2x2 – 4x – 5
From the table, it can be noted that : x –2 –1 0 1 2 3 4
x – axis 2x2 8 2 0 2 8 18 32
Has very small negative side –1 and very big positive side + 5 – 4x 8 4 0 –4 –8 – 12 – 16
We rule the y – axis line giving much space for positive side –5 –5 –5 –5 –5 –5 –5 –5
y 11 1 –5 –7 –5 1 11
279
16
y 2006/11 NABTEB ( Nov)

12

y = 2x2 - 7x - 4
8

0
-2 -1 0 1 2 3 4 5
x6

y = 3x +1 -4

-8

-12

28 2006/11 NABTEB ( Nov) Adjusted


y
y = 2x2 - 7x - 4
24

20

16

12

0
-2 -1 0 1 2 3 4 5 6 x 7

-4
y = 3x +1

-8

-12

280
From the table, we observed that : From (1) a = b – 2
x – axis Substituting into (2)
Has small negative side – 2 and bigger positive side + 4 4(b – 2) + 2b = – 2
Rule the y – axis line with twice space for the positive side 4b – 8 + 2b = – 2
y – axis 4b + 2b = 8 – 2
Has small negative side – 7 and big positive side 11 6b = 6
We rule the x – axis line with little more space for the b=1
positive side. Thus a = 1 – 2
The graph should be ruled in the format: a=–1
a = – 1, b = 1 and c = 2
Equation of the curve: y = – x2 + x + 2
Or y = 2 + x – x2
(iii) Line which divide the curve into equal halves
x = 0.4
The graph is shown on page 282 though no scale was given 2009/9b Exercise 18.16
(b) Table for y = 2x + 1 The graph of the equation y = Ax2 + Bx + C passes through
x –2 0 3 the points (0, 0), (1, 4) and (2, 10). Find the
i. value of C
2x –4 0 6
ii. values of A and B
+1 +1 +1 +1
iii. co-ordinates of the other point where the graph cuts
y –3 1 7
the x – axis
The straight line is drawn on the graph 2013/20 Neco Exercise 18.17
C (i) it is the points where the curve cuts the x – axis A student plots a graph and gets the shape U, the equation
– 0.88 and 2.88 may be of the form
(ii) a line is drawn at y = 4 to touch the curve then A y = ax2 B y = – bx3 C y = – ax2 E y = ax4
traced to x – axis – 1.3 and 3.3 2015/12 Exercise 18.18
(d) It is off target as a result of typing error or Ada draws the graph of y = x2 – x – 2 and y = 2x – 1 on the
examiner’s omission. same axes. Which of these equations is she solving?
(e) y = 7 and x = 1 (lowest point of the curve) A x2 – x – 3 = 0 B x2 – 3x – 1 = 0
C x – 3x – 3 = 0 D x2 + 3x – 1 = 0
2

2011/7b
1978/9 Exercise 18.19
The diagram on page 282 shows the graph of
Copy and complete the following table of values for the
y = ax2 + bx + c and y = mx + k where a, b, c, m and k
function y = 2x2 – 4x + 3, from x = –2 to x = 4
are constants.
x –2 –1 0 1 2 3 4
Use the graph(s) to:
(i) find the root of the equation ax2 + bx + c = mx + k; y 19 3 1
(ii) determine the values of a, b and c using the Draw the graph of y = 2x2 – 4x + 3, using a scale of 2cm
coordinate of points L, M and N and hence write the to 1unit on the x – axis and 1cm to 2units on the y –axis.
equation of the curve; From your graph, find:
(iii) determine the line of symmetry of the curve (a) the equation of the line of symmetry of the curve
y = ax2 + bx + c (b) the gradient of the curve at x = 3
Solution (c) the values of x for which 2x2 – 4x – 3 = 0
(i) This is the points of intersection of the curve and the (d) the range of values of x for which 2x2 – 4x + 1< 0
straight line traced to the x – axis i.e P and H
P = – 1.45 and H = 1.48 1975/10 Exercise 18.20
(ii) y = ax2 + bx + c -----** A ball is projected vertically upwards from a point O on the
y – Intercepts ground. The height of the ball after t seconds have passed is
At M x = 0, y = 2 substituting into ** y metres, where y = 16 + 60t – 16t2
2 = a(0) + b(0) + c Draw the graph of y for t = 0 to t = 4, using a scale of 2cm
Implies c = 2 to 1second along the x – axis and 1cm to 10meters along the
y – axis.
At L (– 1.0, 0) and C is 2 substituting into ** Make use of intermediate values of t, i.e t = 1/2, 3/2, 5/2 and 7/2
0 = a(– 1)2 + b(– 1) + 2 From your graph, find:
a – b = – 2 ----- (1) (a) The greatest height above the ground which the ball reaches.
(b) The time that passes before the ball hits the ground.
At N(2.0, 0) and C is 2 substituting into **
(c) The velocity of the ball at t = 3
0 = a(2)2 + b(2) + 2 (Note: this is the same as the gradient of the curve at the time t = 3)
4a + 2b = – 2 ------ (2)
Solving the resulting simultaneous linear equations

281
3
2011/7b y

M
2

H y = mx + k

L N
0
-3 -2 -1 0 1 2 3 4
x

-1

P
-2

-3

y = ax2 + bx + c

-4

-5

12
y 2007/9 Neco
y = 2x2 - 4x - 5
10

0
-3 -2 -1 0 1 2 3 4 5
x
-2

y = 2x +1
-4

-6

-8

282
1975/11 (Nov) Exercise 18.21 Trigonometry (2)
Draw the graph of y = 3x (4 – x) for valves of x ranging Trigonometric functions like others have their graphical
from – 2 to + 6. On the same graph draw the line representations, which is of great importance to scientist.
y = 5(x – 2). Use a scale of 1cm to represent 1unit along the
Thus, it is one of the basic knowledge required in
x – axis and 5units along the y – axis.
From your graph:
mathematics. The sine, cosine and tangent can be
(a) Find the roots of the equation 3x(4 – x) = 0 represented graphically in either degrees or radians as units
(b) Write down the maximum value of of measurements, though degree is often used.
y = 3x (4 – x) and its value of x
(c) Write down the equation of the axis of symmetry of Sine Graph
the curve y = 3x(4 – x)
(d) Deduce the roots of the equation 3x(4 – x) = 5(x –2) y Fig I
+1
2005/13 (Nov) Exercise 18.22
(a) Copy and complete the following table of values for
the relation y = 2x – x2 for – 2  x  4
x –2 –1.5 – 1 – 0.5 0 0.5 1 1.5 x
y –8 –3 0
00 900 1800 2700 3600
2 2.5 3 3.5 4
0 –1.25 –5.25 -1

(b) Using 2cm to 1unit on the x – axis and 2cm to 2units on y


the y – axis, draw the graph of the relation +1 Fig II
(c) On the same axis, draw the graph of y = x – 4
(d) Use your graph to find the:
(i) roots of the equation 4 + x – x2 = 0;
(ii) line of symmetry of the curve x
(iii) value of x for which y  0 -1800 -900 00 900 1800 2700 3600
1976/11 (Nov) Exercise 18.23
Copy and complete the table below and use it to draw -1
the graph of y = 3x2 – 5x + 4. Use a scale of 1cm to
1unit along the x – axis and 10units along the y – axis y
+1 Fig III
x –3 –2 –1 0 1 2 3 4
3x2 12 3 0 3 12 27
–5x + 4 19 9 –1 –11 –16
y 46 26 4 32
x
Use your graph:
(a) to solve the equation 3x2 – 5x + 4 = 0 0 /2  3/2 2
(b) to solve the equation 3x2 – 5x = 2
-1
(c) to determine the range of value of x for which
Characteristics of a Sine curve
3x2 – 5x < 20
When plotting or sketching a sine curve on the positive x –
2012/8 Neco Exercise 18.24 axis only (fig 1), it starts from the origin (00, 0) and reach its
(a) Copy and complete the table of the values for peak of +1 at 900 (/ 2) then falls to 0 at
y = 3x2 – 5x – 1 x = 1800 ( ) from this point, it descends downward to a
x –3 –2 –1 0 1 2 3 4 lowest point of – 1 at x = 2700 ( 3/2). It then attains 0 at x =
y 41 –1 11 3600. The cycle continues in that order; other forms of sine
(b) Using the scale 2cm = 1unit on the x –axis and curves are multiples of this basic one.
1cm = 5units on the y – axis, plot the graph of the
equation y = 3x2 – 5x – 1 Sine graph for AMPLITUDE multiples
(c) (i) On the graph, find x when y = 0 When y = sin 
(ii) Draw the line of symmetry of the equation Actually the amplitude here is the coefficient of sin which
y = 3x2 – 5x – 1 and determine its equation is 1. Thus, the highest and lowest points of the graph are + 1
2009/11 f/maths Exercise 18.25 and –1 respectively
(a) On the same axes, draw the graphs of y = 15 – x – 6x2 when y = 2 sin.
and y = 2x – 1 for – 3  x  3, The amplitude (coefficient of sin ) is 2
using a scale of 2cm to 1 unit on the x – axis Thus, the highest and lowest points are +2 and – 2
and 2cm to 5 units on the y – axis . respectively.
(b) Using your graph
i. find the roots of the equation 16 – 3x – 6x2 = 0 Similarly, when y = 3 sin.
ii. state the equation of the line of symmetry The amplitude (coefficient of sin ) is 3
iii. find the range of x for which 16 – 3x – 6x2  0 Thus, the highest and lowest points are +3 and – 3
(c) Find the gradient of y = 15 – x – 6x2 at the point x = – 2 respectively.
283
y
+1 y = sin y
+1

00 1800 3600 Fig II
-1
y x
+2 y = 2sin 0 /2  3/2

 -1
00 1800 3600
-2 y
y +1 Fig III
+3 y = 3sin

 x
00 1800 3600 -270 -90 90 270
-3
-1
Sine graph for PERIODICITY Multiples
The periodicity of trigonometry graph is a term for the Characteristic of a cosine curve
repetition of the graph pattern at intervals; or Sketching cosine curve on the positive x – axis as shown in
completing of the graph cycle. Usually within 3600 fig, I and II. It starts from its peak of +1 (00,1) and then falls
When y = sin to zero at x = 900 (/2). From this point it continues to descend
Its periodicity is 3600 . This is the basic sine graph till it get to its lowest point of –1 at x = 1800 () . It then starts
shown earlier. climbing to zero at x = 2700 (3/2).
When y = sin2 The cycle continues in this pattern.
Its periodicity is 1800 – since two multiplies 1800 to Other forms of cosine graph are either multiples of its
give 3600. The graph pattern is shown below: amplitude or periodicity.
.
y Cosine graph for AMPLITUDE multiples
+1 y = sin2 The principles applied in sine graph multiples are same here.
Thus, we have: y
 +2 y = 2cos 
00 900 1800
-1
00 900 2700 
When y = sin 3  -2
The periodicity is 1200 – since three multiplies 1200 to y
give 3600.. The graph pattern is shown below. +3 y = 3cos
.
y
+1 y = sin 3 
00 900 2700 
 -3
00 600 1200
-1 y = -cos
+1
Cosine Graph

y 900 2700 
+1
Fig I -1

Cosine Graph for PERIODICITY Multiples


x The same principle applies as in the case of sine function.
00 900 1800 2700
+1 y = cos2  +1 y = cos3 
-1
 
450 1350 300 900
-1 -1

284
Solution
Tangent graph ( a ) Table for y = 2sinx – cos2x
y x 00 300 600 900 1200 1500 1800
2sinx 0 1.0 1.7 2.0 1.7 1.0 0
- cos2x - 1.0 - 0.5 0.5 1.0 0.5 - 0.5 - 1.0
y - 1.0 0.5 2.2 3.0 2.2 0.5 - 1.0
x In completing table we stick to the decimal place (one d.p)
00 1800 3600
given from x = 300 and y = 0.5
Graph is shown on page 286
(b) A straight line at y = 1.25 is drawn to cut the curve as
shown on the graph.
The curve of tangent function is not a continuous one
(c) i. the points where the curve cuts the x – axis
like that of sine or cosine. It appears in three distinct
200 and 1600
curves as shown above
ii. The points of intersection between the curve and
the straight line traced to the x – axis as shown by
GRAPH PLOTTING IN TRIGONOMETRY dotted lines 430 and 1370
The methods applied in table of trigonometry curves are
similar to that of quadratic graph.
2008/12 2011/12 Neco
(a) Copy and complete the table of values for (a) Copy and complete the table of value for
y = 3sinx + 2cosx for 00 x  3600 y = 2sinx + cosx for 00 x  3600
x 00 600 1200 1800 2400 3000 3600 x 00 600 1200 1800 2400 3000 3600
y 2.00 2.00 y 1.23 1
(b) Using a scale of 2cm to 600 on x – axis and (b) Using a scale of 2cm to 600 on x – axis and
2cm to 1unit on y – axis, draw the graph of 2cm to y – axis, draw the graph of y = 2sinx + cosx
y = 3sinx + 2cosx for 00 x  3600 for 00  x  3600
(c) Use your graph to solve the equation: (c) Use your graph to solve:
3sinx + 2cosx = 1.5 (i) 2sin x = – cosx
(d) Find the range of values of x for which (ii) 2sin x + cosx = 0.5
3sinx + 2cosx < –1 Solution
Solution (a) Table for y = 2sin x + cosx
(a)Table for y = 3sinx + 2cosx x 00 600 1200 1800 2400 3000 3600
x 00 600 1200 1800 2400 3000 3600
2sinx 0.0 1.73 1.73 0.0 –1.73 –1.73 0.0
3sinx 0 2.60 2.60 0 –2.60 –2.60 0 +cosx 1.0 0.5 - 0.5 - 1.0 - 0.5 0.5 1.0
+ 2cosx 2.00 1.00 - 1.00 -2.00 - 1.00 1.00 2.00
y 2.00 3.60 1.60 -2.00 - 3.60 - 1.60 2.00 y 1.0 2.23 1.23 - 1.0 - 2.23 - 1.23 1.0
In completing the table we stick to the decimal place (b) Graph is shown on page 287
given by x = 00 and y = 2.00 (c) i. Using the graph to solve for 2sinx = – cosx is same as
2sinx + cosx = 0
(b) Graph is shown on page 286 These are the points where the curve cuts the x – axis:
(c) Rule a line at y = 1.5 to cut the curve, then trace the 1530 and 3350
points to the x – axis 1220 and 3520 ii. Rule a straight line at y = 0.5
(d) Rule a line at y = –1, the points where it cuts the The point where it touches the curve traced to x – axis as
curve, we require the lower part of the curve in shown by dotted line: 1410 and 3480
range form 1620 < x < 3120

2006/12 2009/12
(a) Copy and complete the table of the relation (a) Copy and complete the table of value for
y = 2sinx – cos2x y = sinx + 2cosx correct to one decimal place.
x 00 300 600 900 1200 1500 1800 x 00 300 600 900 1200 1500 1800 2100 2400
y 0.5 –1.0 y 2.2 - 1.2 - 2.0 - 1.9
Using a scale of 2 cm to 300 on the x – axis and (b) Using a scale of 2cm to 300 on the x – axis and
2cm to 0.5 unit on the y – axis, draw the graph of 2cm to 0.5unit on the y – axis, draw the graph of
y = 2sinx – cos2x for 00  x  1800 y = sinx + 2cosx , for 00  x  2400
(b) Using the same axes, draw the graph of y = 1.25 (c) Use your graph to solve the equation:
(c) Use your graph to find the: (i) sinx + 2cosx = 0
(i) value of x for which 2sinx – cos2x = 0 (ii) sinx = 2.1 – 2cosx
(ii) roots of the equations 2sinx – cos2x = 1.25 (d) From the graph, find y when x = 1710

285
4 y
2008/12
3.5

2.5

1.5

0.5

0
0 30 60 90 120 150 180 210 240 270 300 330 360 390
-0.5 x

-1

-1.5

-2

-2.5

-3

-3.5

-4

3.25
y 2006/12 y = 2sinx – cos2x
3

2.75

2.5

2.25

1.75

1.5

1.25

0.75

0.5

0.25

0
0 30 60 90 120 150 180 210
-0.25 x
-0.5

-0.75

-1

-1.25

286
2.5 y
2011/12 Neco y = 2sinx + cosx
2

1.5

0.5

0
0 30 60 90 120 150 180 210 240 270 300 330 360
x390
-0.5

-1

-1.5

-2

-2.5

2.5 y
2.25 2009/12 y = sinx – 2cosx
2

1.75

1.5

1.25

0.75

0.5

0.25

0
0 30 60 90 120 150 180 210 240
-0.25 x270
-0.5

-0.75

-1

-1.25

-1.5

-1.75

-2

-2.25

-2.5

287
Solution
y8
(a) Table for y = sin x + 2 cos x
x 00 300 600 900 1200
sin x 0 0.5 0.9 1.0 0.9
2cos x 2.0 1.7 1.0 0 –1.0 4

y 2.0 2.2 1.9 1.0 –0.1

1500 1800 2100 2400 0 x


0.5 0 –0.5 –0.9 0 90 180 270 360
–1.7 –2.0 –1.7 –1.0
–1.2 –2.0 –2.2 –1.9
-4

(b) Graph is show on page 287


(c) i. 1170 This is the point where the curve cuts the x – axis
ii. Rearranging to fit our original equation
sinx = 2.1 – 2cos x becomes -8

sinx + 2cos x = 2.1 2007/49 Neco Exercise 18.26


Next, we draw a straight line at y = 2.1 to cut the curve, Which of the following represent the graph above?
trace the points to the x – axis as shown by dotted lines A y = sin 14 x B y = cos 4x C y = 4 sin x
120 and 450
(d) At x – axis mark 1710 and trace to the curve then D y = 4 cos x E y = cos 14 x
to y – axis as shown by dotted lines 2008/44 and 45 Neco
y = – 1.8 The graph below is the cosine curve. Use it to answer
questions 44 and 45
45. What is cos–1 (– 0.8) ?
A 1440 or 2210 B 1470 or 2160 C 1380 or 2210 44. Find cos1380
D 1380 or 2160 E 1320 or 2100 A 0.75 B 0.70 C – 0.75 D – 0.70 E – 0.068
Solution Solution
At the negative y – axis, trace – 0.8 to cut the curve Mark out 1380 in the x – axis by 138 – 120 = 180
then to x – axis: 1440 or 2160 Thus three boxes after 1200 gives 1380 (since each box is 60)
trace it to the curve then to y – axis
cos 1380 = – 0.75
1.5

y 2008/44-45 Neco
1

0.5

0
0 30 60 90 120 150 180 210 240 270 300 330 360 390

x
-0.5

-1

-1.5

288
2014/7 Exercise 18.27 Linear inequality II
Copy and complete the table of value for the relation Linear inequalities in two variables.
y = 2sin x + 1 This is another phase of inequalities, which is related to linear
x 00 30 60 90 120 150 180 210 240 270 equation in two variables. We shall concern ourselves mainly
0 0 0 0 0 0 0 0 0

y 1. 2.7 0.0 -0.7


with the graphical solution to inequality in two variables.
0 Here broken lines represents strict inequalities ( or ) while
(b) Using a scale of 2cm to 30 on the x – axis and
0 thick straight line shows weak inequality ( or  ). We always
2cm to 1unit on the y – axis, draw the graph of use the linear equation equivalent of equality to get the two
y = 2sin x + 1 for 00  x  2700 points necessary to plot the straight line.
(c) Use the graph to find the value of x for which Examples of inequality graphs are shown below
sin x = 14 (1) Solve the inequality : y  4x + 2
Solution
1995/8 Exercise 18.28 We consider the line equivalent
Copy and complete the table of value for the relation y = 4x +2
y = 2cos2x – 1 When x = 0 , y = 2 ( 0, 2 )
x 00 300 600 900 1200 1500 1800 When y = 0 , x = - 0.5 ( -0.5, 0 )
y
y = 2cos2x 1.0 0 1.0
–1
(b) Using a scale of 2cm = 300 on the x – axis and 2
2cm = 1unit on the y – axis, draw the graph of
y = 2cos2x – 1 for 00  x  1800 -0.5 0
x
(c) On the same axes draw the graph of
1
y= ( x – 360 )
180
(d) Use your graph to find the :
i. values of x for which 2cos2x + 1
= 0 The shaded region satisfies the inequality y  4x + 2 but
2
points on the broken line are not included since we are
x
ii. roots of the equation 2cos2x – + 1 = 0 dealing with strict inequality.
180
2013/10 f/maths Exercise 18.29 2. Show graphically the region represented by x + y  5
(a) Using a scale of 2cm to 300 on the x – axis and
2cm to 0.2 unit on the y – axis, on the same graph Solution
sheet, draw the graphs of y = sin 2x and y = cos x First we consider the line equivalent
for 00  x  2100 at intervals of 300. x+y=5
(b) Using the graphs in 10(a), find the truth set of: When x = 0, y = 5 ( 0, 5 )
i. sin 2x = 0 ii. sin 2x – cos x = 0 When y = 0, x = 5 ( 5, 0 )

2004/26 f/maths Exercise 18.30 y


Y
2
5
1

X x
0 5
0
300 600 900 120 0
150 0 1800

-1
The shaded region is x + y 5
-2 3 . Show graphically the region represented by 2x + y  8
The sketch above represents the graph of Solution
A y = 2sin x B y = 2cos x C y = cos 2x D y Considering the line equivalent,
= sin 2 x 2x + y = 8
When x = 0, y = 8 (0, 8)
When y = 0, x = 4 (4, 0)
y

x
0 4

289
Some inequality regions are shown below (shaded) 2000/18 Neco
The shaded region on the Cartesian plane below represents
y y the inequality:
y
x 2

y<0 y>0
0 x
x

y -2
y
A –2 < x < 0 B –2  x < 0 C –2 < y < 0
x<0
0>x D –2  y < 0 E –2 < y  0
Solution
x x The straight line at –2
y = –2
x<0 The shaded part is the upper part i.e > or ≥
The line is broken i.e strict inequality of >
Thus y > –2
The line at x = 0
The shaded part is the lower part and it is a thick line
2015/30 and 31 Neco y0
Use the inequality graph below to answer questions 30 Combining the two inequalities
and 31 –2 < y  0 ( E )

y 2005/18 and 19 (Nov)


The diagram below shows the graph of lines y = 2x and
y + x = 6. P is the point of intersection of the two lines. Use
P
the information to answer questions 18 and 19
y

2x
y=
P
x
O Q
y
3x + 4y = 9 +
x
=
6
30. Find the coordinates of point P O x
A (0 , 21/4) B (0 , 3) C ( 21/4 , 0)
D (3 , 0) E (3 , 4) 18. Find the coordinates of point P
Solution A (2 , 3) B (3 , 4) C ( 3 , 6) D (2 , 4)
Solution
At point p
The coordinates of the point P is gotten by equating the two
x = 0, then 3x + 4y = 9 becomes
equations and solving
3(0) + 4y = 9
y = 2x and y + x = 6 i.e y = 6 – x
4y = 9
y = 9/ 4 = 2 1/ 4 2x = 6 – x
Coordinates of point P (0 , 21/4) A. 2x + x = 6
3x = 6
31. The shaded portion shows the outer boundary of the x = 6/ 3 = 2
half plane defined by the linear inequality Substitute x value into y = 2x
A 3x + 4y = 9 B 3x + 4y < 9 C 3x + 4y > 9 y = 2(2)
y=4
D 3x + 4y  9 E 3x + 4y ≥ 9
Thus P (2 , 4) D.
Solution
Since the outer boundary is the region referred to in this
19. Which set of inequalities satisfies the shaded region ?
case and we have a thick line (weak inequality)
3x + 4y ≥ 9 ( E ) A y ≥ 0, y  2x , y + x ≥6 B y ≥ 0, y  2x , y + x  6
C y ≥ 0, y ≥ 2x , y + x 6 D y  0, y  2x , y + x 6
Solution
All the lines are thick ones-no broken line. The result is
weak inequality in all the values here.
290
The shaded region is: Note it is negative slope where y2 is the lower part
the positive side of y i.e y ≥ 0 0 − 3 −3
= =
the lower part of line y = 2x i.e y  2x 2 −0 2
the lower part of line y + x = 6 i.e y + x  6 Substituting y = – 32 x + 3
Thus the set of inequalities is y ≥ 0, y  2x , y + x  6 (B)
Multiply through by 2 to clear fraction
2010/33 2y = –3x + 6
y 2y + 3x = 6
4 Broken line implies > or <
Lower part of the line implies <
3 Thus 2y + 3x < 6
2 The shaded region is:
The positive side of y - axis x ≥ 0
1 The positive side of x - axis y ≥ 0
x Option A
0 2 3
-3 -2 -1 1 4
-1

-2
System of inequalities (more than one equation)
-3 The solution of a system of linear inequalities in two
variables is the region of intersection of all the equations
The shaded portion in the diagram is the solution of involved.
A x+y3 B x+y<3 C x+y>3
Dx+y≥3 2007/8a Neco
Solution Show the region on the graph which satisfies the
Applying the formula for straight line y = mx + c, inequalities :
Here c = 3 i.e where the line cuts the y – axis 4x – y  12
y 2 − y1
m(Slope) = 2x + y  18
x 2 − x1 x=1
Note it is negative slope where y2 is the lower part y≥0
0 − 3 −3 Solution
= = = –1
3 −0 3 Line 4x – y = 12 (Thick line and shade the back part)
Substituting y = –1x + 3 When y = 0 , x = 12/4 = 3 (3, 0)
y +x = 3 When x = 0 , y = –12 (0, –12)
Broken line implies > or < Line 2x + y = 18 (Thick line and shade the back part)
Lower part of the line implies < When y = 0 , x = 18/2 = 9 (9, 0)
Thus y + x < 3 ( B ) When x = 0 , y = 18 (0, 18)
Line x = 1 ( ordinary line)
2014/58 Neco Line y = 0 (Thick line and shade the positive side)
The shaded region on the Cartesian plane below 30
represents the inequalities: y
y
25
4x – y =12
(0, 3) 20
2x+ y =18
15

10

x 5
0, 0 (2, 0)
x=1 x
A x≥ 0, y≥ 0, 2y +3x < 6 B x≥ 0, y< 0, 2y +3x < 6 0
C x> 0, y< 0, 2y +3x < 6 D x≥ 0, y> 0, 2y +3x  6 0 1 2 3 4 5 6 7 8 9
E x> 0, y≥ 0, 2y +3x ≥ 6 -5
Solution
Applying the formula for straight line y = mx + c, -10
Here c = 3 i.e where the line cuts the y – axis
y 2 − y1
m(Slope) = -15
x 2 − x1

291
2012/7 Solution
(i) Using a scale of 2cm to 1unit on both axes, on the 3x
3x Line y – =3
same graph sheet, draw the graphs of y – =3 4
4 When x = 0, y = 3 (0, 3)
and y + 2x = 6 y = 0, –3x = 12 i.e x = – 4 (– 4, 0)
(ii) From your graph, find the coordinates of the point Line y + 2x = 6
of intersection of the two graphs When x = 0, y = 6 (0, 6)
(iii) Show on the graph sheet, the region satisfied by y = 0, 2x = 6 i.e x = 3 (3, 0)
3x (ii) Coordinates of the point of intersection (1.1, 3.8)
the inequality y – ≥3
4 (iii) Region is shown by area bounded by the thick line
on the graph below (we avoided shading for clearer graph)
14
2012/7
y
13

12

11

10

0 x
-4 -3 -2 -1 0 1 2 3
-1

2005/12a Neco Exercise 18.31 Exercise 18.33


Show graphically the region R which satisfies the (i) Using a scale of 1cm to 1unit on both axes, on the
following inequalities same graph sheet, draw the graphs of
3x + 4y  6 2x + 4y  16
x + 4y  10 , y ≥ 0 , x ≥ 0 x – 2y ≥ 10 , y ≥ 0 , x ≥ 0
2009/41 (Nov) Exercise 18.32 Show on the graph sheet, the region satisfied by
y the inequalities

x
0 1 2 3 4
y + x= 3
Which of the following inequalities represent the
shaded portion?
A y + x  3, x ≥ 0, y≥ 0 B y + x ≥ 3, x ≥ 0, y≥ 0
C y + x  3, x ≥ 0, y 0 D y + x ≥ 3, x  0, y≥ 0
292
II. SAS : Side, angle, side two sides and the included angle
CHAPTER NINETEEN of one triangle are equal to the two sides and the included
angle of the other
Plane and Circle Geometry R
V

Plane Geometry
Angles between straight and parallel lines, types of
triangles have been discussed under ‘elements of plane
Geometry’ (Bearing). Readers and students to take note. T U W X
Here TR = WV, TU = WX, T = W
Triangles Thus RTU = VWX (SAS)
Theorem: Sum of angles in a triangle is 1800 (2 right angles)
III. ASA : angle, side, angle
A AAS : angle, angle, side
P Two angles and a side of one triangle are
a2
respectively equal to two angles and a side of the
other
a1
b2 b1 ASA
c G
B X D
C
Given: any ∆ ABC
To prove: Â + B + C = 1800
Construction: Produce BC to point X. Draw CP//BA
Proof: E F H J
a1 = a2 (alternative s)
b1 = b2 (corresponding s )
AAS
c + a1 + b1 = 180 (s on straight line) C
A
 c + a2 + b2 = 1800
 AĈB + Â + B = 1800
 Â + B + C = 1800
Note: The above theorem and its proof is to be
P K L N
mastered by students as the theorem is among the
required ones to be proven in exams:
IV. RHS: right – angled, hypotenuse, side
The exterior angle of a triangle is equal to the Both triangles are right – angled, the hypotenuse
sum of the two interior opposite angles. and one side are equal
M
C
a

b c
D F N O
C=a+b
2006/56 Neco
Congruent triangles Which of the triangle X, Y, Z are congruent?
Two triangles are congruent () if they pass one of the
four tests listed below:
I . SSS : Side, side, side are equal i.e three sides of one
triangle are equal to the three sides of the other
O X Y
A

B C P Q A X and Y only B X and Z only C Y and Z only


D All the three E None of them
Here AB = OP, BC = PQ and AC = OQ Solution
Thus ABC  OPQ (SSS) X = Y (RHS) A.
293
2008/35 Neco (Dec) Similar triangles
Which of the following triangles are congruent? For two triangles to be similar it must meet (fulfill) one of
B the three conditions below:
B
II Two triangle are similar if
I 4cm
I. “Two angles in one triangle are equal to two angles in
4cm 0
70 the other” of course the third angle must follow suit.
C
70
0
C A 6cm
It can be restated as “Two triangles are similar if
A 6cm three angles of one are respectively equal to the
B B three angles of the other”
IV
II I II. Two triangles are similar if two pairs of sides are in
4cm the same ratio and their included angles are the same

m
4cm

6c
70 0
70 0 A C III. Two triangles are similar if the ratios of the
C 6 cm A corresponding sides are equal

A I and III only B II and IV only C I and II only Condition I


D II and III only E I and IV only
Solution
II and III only (SAS) D.

2005/55 Neco Exercise19.1


Which of the following conditions is NOT always true
about congruent triangles P and Q? Condition II
A The area of P is equal to the area of Q O R
B Three angles of P are equal to three angles of Q
C Three sides of P are equal to three sides of Q
D Two sides and the included angle of P are equal to
two sides and the included angle of Q P S
E Two angles and one side of P are equal to two angles
and one side of Q Q T
OP PQ
2008/40 Neco Exercise 19.2 OPQ = RST, =
RS ST
In proving the congruence of two triangles, which of the
following is NOT important?
A Two sides and the included angles. Condition III
B Two angles and a side A D
C Three sides
D Three angles
E Right – angle, hypotenuse and another side

B C E
2003/48 NABTEB Exercise 19.3
In the diagram below, name the triangle which is
AB = BC = AC
congruent to XYZ DE EF DF F
S X T Z

Intercept Theorem
If a straight line is drawn parallel to one side of a triangle,
the straight line divides the other two sides in the same ratio
A

Y
D E
A TYS B TYZ C XYS D XYZ

B C

Here DE Divides side AB and AC in equal ratios


AD AE
=
DB EC

294
Midpoint theorem 2009/24
The straight line joining the midpoints of two sides of a In the diagram, /SQ/ = 4cm, /PT/ = 7cm, /TR/ = 5cm
triangle is parallel to the third side and equal to half of it and ST//QR. If /SP/ = x cm, find the value of x.
O P

x 7cm

S T
R S
4cm 5cm

P Q Q R
1
RS//PQ and RS = 2
PQ A 5.6 B 6.5 C 6.6 D 6.8
2006/47 NABTEB (Nov) Solution
We apply corresponding angles rule in parallel lines and
In PQR, PT = TQ , PS = SR and TS // QR .
transversal to indicate the angles.
If QR = 16cm and PR = 20cm, find TS Angle P is common.
P
P

x 7cm
20cm
S T
T S
4cm 5cm
Q R
16cm Q R
A 8cm B 10cm C 32cm D 40cm PT
=
PS
Solution PR PQ
TS = 12 QR 7 x
=
= 1
2
 16 = 8cm ( A ) 7+5 x+4
7 x
=
2007/49 12 x + 4
In the diagram, /XR/ = 4cm, /RZ/ = 12cm, /SR/ = n, 7(x + 4) = 12x
_____ _____ 7x + 28 = 12x
/YZ/ = m and SR // YZ . Find M in terms of n. 28 = 12x – 7x
X
28 = 5x
4cm x = 5.6 ( A )
S R
n 2005/48 NABTEB
12cm In the figure below, BC is parallel to DE.
If /AE/ = 15cm, find the length of CE.
Z A
Y m

A m = 2n B m = 3n C m = 4n D m = 5n
Solution B C
Applying corresponding angles rule in parallel lines and 6cm
transversals to indicate angles.
Angles X is common. D E
X 10cm

A 4cm B 6cm C 9cm D 10cm


4cm Solution
S R We are dealing with similar angles. Applying corresponding angles
n format in parallel lines and transversal to indicate the angles.
12cm Angle A is common to both triangles
A
Y m Z

XR SR
= B C 15cm
XZ YZ 6cm
4 n
=
4 +12 m
D E
4m = n(4 + 12) 10cm
4m = 16n
m = 4n (C) CE = AE – AC

295
But
BC AC
=
2004/45 NABTEB
DE AE Find /PQ/ in the diagram, given that /PM/ = 2cm,
6 AC /PN/ = 4cm and /NR/ = 13cm
= R
10 15
6  15 = 10  AC
6  15 N 13 c
m
= AC
10
AC = 9cm 4cm
Thus CE = 15 – 9 P
2cm
= 6cm ( B ) M Q
3 3 1
2005/47 NABTAB (Nov) A /4cm B 2 /4cm C 5 /3cm D 71/3cm
What is the ratio x : y in the figure below? Solution
A3:4 B4:3 C3:7 D4:7 Note that PQ = 2 + MQ
4 2
=
4 4 + 13 2 + MQ
x 4(2 + MQ) = 2  17
34 17
3 2 + MQ = = = 81/2
4 2
y
2010/40 Neco counter example
Solution In the diagram below, BC//DF, BD//CE, /AC/ = 9cm,
We are dealing with similar triangles. Applying
/BC/ = 3cm and /EF/ = 5cm. Find the value of CF
corresponding angles format in parallel lines and A
transversal to indicate the angles.
The top angle is common to both triangles 9cm

B C
4 3cm
x

3 D E 5cm F
y
A 3cm B 15cm C 24cm D 45cm E 72cm
4 x Solution
= ABC and ADF are similar s
4+3 y
BC AC
4 x =
= DF AF
7 y
But DF = DE + EF and DE = 3cm (opposite side of llgm)
In ratio form and AF = 9 + CF
x : y 3 9
4 : 7 (D) =
3+5 9 + CF

2013/38 Neco 3(9 + CF) = 9  8


In the diagram below, find the value of x 27 + 3CF = 72
P 3CF = 72 – 27
3CF = 45 thus CF = 15cm (B)
3m 8m

A B
2014/19 (Nov) counter example
32m Triangles PQR and XYZ are similar. If the sides of triangle PQR
x 2m are 6cm, 7cm and 8cm and the shortest side of the triangle XYZ is
2cm, find the length of the longest side of triangle XYZ.
Q R A 4cm B 31/2cm C 22/3cm D 22/7 cm
46m
Solution
A 0.44m B 0.75m C O.82m D 1.83m
Let us sketch the two triangles as
E 2.74m P X
Solution
8 3
= 8cm
8+ 2 3+ x 7cm
8(3 + x) = 3  10
24 + 8x = 30 Q R Y 2cm
Z
6cm
8x = 30 – 24
8x = 6 Since they are similar the ratio of their corresponding sides are equal
Shortest side here is QR and YZ.
x = 6/8 = 0.75m ( B )
Longest side here is PQ and XY.
296
6
=
8 48. Which of the following statement is not correct
2 XY about QTPRS?
6 XY = 2  8 A /TS/ is proportional to /RP/
28 B /QT/ is proportional to /QR/
XY = C /QS/ is proportional to /SR/
6
= 8/3 cm = 22/3cm ( C ) D /QS/ is proportional to /QP/
Solution option D
2010/21
H 49. Calculate the length of PR
A 14cm B 16cm C 20cm D 24cm
Solution
Ratio: side facing ordinary angle: side facing common angle at Q
8 10
K x0 =
L 4 + 12 PR
8PR = 10  16
10  16
x0
PR =
J I 8
In the diagram, triangles HKL and HIJ are similar. = 20cm C
LH Note the ratio of side facing right angled triangle here is
Which of the following ratios is equal to ? 12 12
JH and not
KL HK JI HK 8 + TP 12 + 4
A B C D Since the right – angle in the big triangle is not facing 12 + 4
JI JK KL LK
Solution
Here the task is to identify the corresponding sides, 2014/3a (Nov)
P
since it is their ratio that is of concern.
In s HJI and HKL 6cm
J = L = x0 (given) S
H is common to both triangles 4 cm
It follows that K = I (3rd  of )
H Q 5cm T R

In the diagram, PQ and QR are straight lines, /PS/ = 6cm,


/QS/ = 4cm, /QT/ = 5cm and QTS = RPQ.
K Calculate /TR/
x0
L
Solution
Taking ratio of side facing marked angles : no common angle at Q
J
x0
I 4 5
=
5 + TR 4 + 6
LH HK KL
= = 4  10 = 5(5 + TR)
JH HI JI 40 = 25 + 5TR
In this case
LH
,are originating side from a given 40 – 25 = 5TR
JH 15 = 5TR thus 3cm = TR
angle x, we can’t take angle H, it is common and not
distinct, so we are left with  K and  I and the 2005/47 (Nov)
originating line there are KL and IJ though written as JI, In the diagram, PQT = PSR. Find the value of x
the ratio is R
KL 2
Option A Q
JI
x
4
2004/48 and 49 NABTEB 3
Use the information in the figure below to answer
questions 48 and 49 P 2 T S
P T 8cm
Q A5 B6 C8 D9
10 cm Solution
m The ratio of sides facing the marked angles: common angle P
12c
2 3
S =
2+4 x
4c m 2x = 3  6
R
3 6
The marked angles are equal, QTS = QRP = 900, x= =9 (D)
2
/TQ/ = 8cm, /TS/ = 10cm, /RS/ = 4cm and /QS/ = 12cm
297
2008/41 Neco 2007/40 Neco Exercise 19.4
Calculate the value of x in the figure below Two lines FG and HJ intersect at P. If FH//JG,
12cm /PG/ = 6cm, /FP/ = 8cm and /GJ/ = 9cm find /FH/
F
J
8cm
8c m

11cm P
xcm
9c m

6cm
14cm
G
A 7 1/ 2 B 91/3 C 93/7 D 10 E 102/3 H
Solution A 16cm B 15cm C 13cm D 12cm E 10cm
First we identity the equal angles,
applying alternate angles and vertically opposite angles 2004/47 NABTEB (Dec) Exercise 19.5
12cm In the figure below MN is parallel to QR. /PM/ = 4cm,
/MN/ = 2cm and /QR/ = 6cm. find the length of MQ
P
8cm

4cm
11cm M N
xcm 2cm

Q R
14cm 6cm
12 8 A 4cm B 8cm C 12cm D 16cm
=
14 x
2003/4a NABTEB Exercise 19.6
12x = 8  14
8  14 Calculate the value of /BZ/ in XYZ, if /XA/ = 8cm,
x= _____ _____
12 /AY/ = 4cm, /YZ/ = 20cm and AB // XZ
28
= = 91/3cm (B) Y
3
m
4c
2015/30 20cm
V W A B
m

6cm
8c

X
X Z
12cm 16cm
2003/42 NABTEB Exercise 19.7
Z Y In the figure below, /MP/ = 2cm, /MQ/ = 3cm,
20cm
/PN/ = 3cm. Calculate the length /QL/
M
In the diagram, VW//YZ, /WX/ = 6cm, /XY/ = 16cm,
/YZ/ = 20cm and /ZX/ = 12cm. Calculate /VX/ 3cm
2cm
A 3cm B 4cm C 6cm D 8cm
P Q
Solution
First, we identify the equal angles 3cm
applying alternate and vertically opposite angles rule N L
V W
A 3.5cm B 4.0cm C 4.5cm D 5.0cm
6cm
X 2005/54 Neco Exercise 19.8
J
12cm 16cm

Z Y
20cm M N

VX 6
=
16 12 K L
6  16 In the figure above, MN is parallel to KL ,
VX =
12
JM = 6cm, MK = 8cm and JL = 21cm. Find JN
= 8cm (D)
A 6cm B 9cm C 10cm D 12cm E 15cm

298
2005/26 Exercise 19.9 Area of similar triangles
D
The ratio of the areas of similar triangles is equal to the ratio
3cm of the squares of corresponding sides
E
R U
m
2c

1.6 c
m
A B C

In the diagram, BE//CD, /AE/ = 2cm, /BE/ = 1.6cm and S O T V X W


/ED/ = 3cm. Calculate /CD/.
A 2.4cm B 2.67cm C 3.25cm D 4.0cm Area of  RST RS 2 RT 2 ST 2 RO 2
= = = =
Area of  UVW UV 2 UW 2 VW 2 UX 2
2006/46 Exercise 19.10
In the diagram, /XY/ = 12cm, /XZ/ = 9cm, /ZN/ = 3cm 2007/12b
and ZY// NM. Calculate /MY/ X
M

m
2c
Y
6cm
m P Q
c
12

3cm
X 9cm 3cm N
Z Y Z
_____
A 3cm B 4cm C 5cm D 6 cm _____
In the diagram, PQ // YZ , /XP/ = 2cm, /PY/ = 3cm,
2011/36 Neco Exercise 19.11 /PQ/ = 6cm and the area of XPQ = 24cm2. Calculate the
Which of the following is correct about the figure area of the trapezium PQZY
shown below? Solution
C
Area of trapezium PQZY = area of XYZ – XPQ
Area of  XPQ XP 2
B
But = (Similar triangles)
Area of  XYZ XY 2
24 22
=
Area of  XYZ (2 + 3) 2
A E
D
24  52 = 22  Area of  XYZ
A ADB is similar to ADC 24  5 2
B ABD is similar to ACE = Area of  XYZ
22
C AB : AD is similar to BC : DE
150cm2 = area of  XYZ
D BDEC is similar to ABD
Thus area of trapezium PQZY = 150 – 24
= 126cm2
2004/48 NABTEB (Nov) Exercise 19.12
Which of the following statements is always true of two
2009/46 (Nov)
similar triangles? Their P
A areas are equal B corresponding sides are equal
C corresponding angles are equal
D shapes are not the same. Q R

2014/5b Neco Exercise 19.13


 
In the diagram below, X Y Z = K A Z , /YZ/ = 24cm. S T
/KY/ = 13cm, /ZK/ = 11cm and /AZ/ = 8cm, 10cm
calculate /XZ/ In the diagram, QR is parallel to ST and /ST/ = 10cm. The
X Y
areas of PQR and quadrilateral QRTS are 9cm2 and 16cm2
13cm respectively. Calculate /QR/.
A 5.6cm B 6.0cm C 7.5cm D 8.0cm.
A K Solution
8cm 11cm
We are dealing with similar s. Applying corresponding
angles rule in parallel lines and transversals to indicate the
Z angles.
Here P is common angle.

299
P 2005/7a (old)
P

Q R

4cm 8cm
X Y
S T
10cm
Area of PQR QR 2 6cm
= Q R
Area of PST ST 2
In the diagram, XY//QR, /XY/ = 4cm, /QR/ = 6cm
But Area of PST = Area of PQR + Area of quad QRTS
= 9 + 16
and /PR/ = 8cm. Calculate the area of PXY
= 25cm2 Solution
Two triangles are similar (RHS)
9 QR 2
Thus = Area of  PXY XY 2
25 10 2 =
Area of  PQR QR 2
9  102 = 25  QR2
Note that, we maintain order; small  at numerator on both sides
9 100
= QR2 But Area of  PQR = 12  6  8 sin 900
25
QR = 36 = 24cm2
Substituting
= 6cm ( B )
Area of  PXY 4 2
= 2
2005/6c (Nov) 24 6
In the diagram, QP is parallel to SR and /PQ/ = 8cm, 16  24
Area of  PXY =
/QO/ = 6cm, /PO/ = 5cm and /SO/ = 10cm. 66
If the area of POQ = 40cm2, Calculate the area of = 10.67cm2
ROS 2010/11a
R 14.4cm
P P T S
5c
m
O 6cm
8cm
10cm
10 Q
m

c
6c

m
Q
S R
Solution In the diagram, PTQ = PSR = 900, /PQ/ = 10cm, /PS/ =
Ratio of the areas of similar triangles is equal to the ratio of
14.4cm and /TQ/ = 6cm. Calculate the area of quadrilateral
the squares of corresponding sides, To identify the
corresponding sides we mark out equal angles by applying
QRST
alternate ad vertically opposite angles Solution
R Area of quad QRST = Area of  PRS – area of  PQT
P The two triangles here are similar (RHS)
5c
m Area of  PQT PT 2
=
O Area of  PRS 14.4 2
8cm But by Pythagoras rule in PQT
10 102 = 62 + PT2
m

c
6c

m
Q PT = 64 = 8cm
S Thus area of PQT = ½  6  8  sin 900 (sin 90 = 1)
Area of  POQ 52 = 24cm2
= 2
Area of  ROS 10 Substituting
Note: We maintain the order of the s, Taking the 24 82
=
smaller one first in area and sides at numeration Area of  PRS 14.4 2
40 25 24  14.42 = 82  area of PRS
=
area of  ROS 100
24  14.4 2
40  100 = 25 area of ROS = Area of PRS
82
40  100
= Area of ROS 77.76cm2 = Area of PRS
25
Area of quad QRST = 77.76 – 24
160cm2 = Area of ROS
= 53.76cm2
300
2008/24 counter example 13 5
=
In the figure /PX/ = /XQ/, PQ//YZ and XY//QR. 6 PS
What is the ratio of the area of XYZQ to the area of 13 PS = 6  5
YZR? PS =
30
P 13
= 2.3077cm  2.3cm to 1d.p

Polygons
X Y
A closed figure bounded by three or more straight sides is
called a polygon. A polygon is regular; if it has all its sides
and angles equal, otherwise it is irregular.
Q Z R
Sum of interior and exterior angles
A1:2 B2:1 C1:3 D3:1 The sum of interior angles of an n – sided polygon is
Solution (2n – 4) x 900
XYZQ is a parallelogram and YZR is a triangle both in A
same base QR and between same parallel XY and QR.
F B
Thus
1
2
Area parallelogram = area of 
Multiply through by 2 E C
Area of parallelogram = 2 Area of 
D
1  YZQ : 2 YZR
1 : 2 (A)
A + B + C + … = ( 2n – 4) x 900 and
Each interior angles of n – sides = (2n – 4) x 900
n
Internal bisector of an angle of a triangle
A While the sum of the exterior angles of an n – sided
polygon is 4 × 900
a

B D C b
d
Angle A is bisected by line AD c
The internal bisector of an angle of a triangle divides
the opposite side in the ratio of the sides containing the
angle a + b + c + … = 3600 and
BD AB
= each exterior angle of n sides = 4  900
DC AC
n
2012/13(a) Quadrilaterals
P Quadrilaterals are four sided figures. Quadrilaterals whose
6cm
diagonals bisect each other at right angles are square, kite
Q and rhombus. Below is a list of some quadrilaterals and their
properties:
13cm
Parallelogram

R
S 5cm

In the diagram, /PQ/ = 6cm, /QR/ = 13cm, /RS/ = 5cm


and RSQ is a right angle. Calculate, correct to one
decimal place, /PS/
Solution
By the theorem of internal bisector of an angle of a
triangle Opposite sides are equal and parallel.
QR SR Opposite angles are equal and the diagonals bisect each
=
PQ PS other. Note: other parallelograms among the quadrilaterals
are rhombus, rectangle and square; NOT kite & trapezium

301
Rhombus Polygons table
All sides are equal, opposite angles are equal and the Name No of sides
diagonals bisect each other at right angles. Triangle 3
Quadrilateral 4
Pentagon 5
Hexagon 6
Heptagon 7
Octagon 8
Nonagon 9
Decagon 10

Rectangle 2009/23
Opposite sides are equal and the angles are 900 . 2x
The diagonal bisect each other and are of equal length. 3x-54

64
x+
5x

2x
4x

+1
0
3x

Find the value of x in the diagram


Square A 500 B 300 C 220 D 170
All sides are equal and the angles are 900. The Solution
diagonals bisect each other at right angles and are of Sum of exterior angles of a polygon = 3600
equal length 2x + x + 64 + 2x + 10 + 3x + 4x + 5x + 3x – 54 = 360
20x + 74 – 54 = 360
20x + 20 = 360
20x = 360 – 20
20x = 340
x = 340/20
= 170 ( D )
2007/27
5x 0 120 0
Kite
Adjacent sides are equal and the diagonals bisect each 130 0
other at right angles

3x 0

From the diagram, find the value of x


A 40 B 30 C 25 D 20
Solution
We re-label the diagram to reflect all the interior angles
5x 0 180-5x 1200

130 0
90 0
Trapezium
One pair of opposite sides is parallel 180-3x
3x 0

It is five sided polygon


Sum of interior angle of polygon = (n – 2)180
180 – 5x + 90 + 180 – 3x + 130 + 120 = (5 – 2)180
700 – 8x = 540
700 – 540 = 8x
160 = 8x
20 = x ( D )

302
2013/37 2012/40
Q The sum of the interior angle of a regular polygon is 18000.
How many sides has the polygon?
R A 16 B 12 C 10 D8
P Solution
Sum of the interior angles of a polygon = (n – 2)180
1800 = (n – 2)180
T S V 1800 (n − 2)180
=
180 180
In the diagram, PQRST is a regular polygon with side 10 = n – 2
QR and TS produced to meet at V. Find the size of 10 + 2 = n
RVS 12 = n ( B )
A 360 B 540 C 600 D 720
Solution 2005/5a Neco
The polygon PQRST is five sided. In this case all the The interior angle of a heptagon are x0, (2x – 15)0,
sides and interior angles are equal. (x + 10)0, (3x + 5)0, (2x – 34)0, (x + 5)0 and (2x + 5)0.
Sum of interior angle of polygon = (n – 2)180 Find the value of x
= (5 – 2)180 Solution
= 540 Heptagon is a 7 sided polygon
Each of the interior angle = 540/5 Sum of interior angles of a polygon = (n – 2)180
= 1080 x + (2x –15)+ (x + 10) + (3x + 5)+ (2x –34)+(x + 5)+(2x + 5) = (7– 2)180
Exterior angle at R is same as at S x + 2x – 15 + x + 10 + 3x + 5 + 2x – 34 + x + 5 + 2x + 5 = 5 180
But exterior angle at R = 180 – 108 12x + 25 – 49 = 900
= 720 12x – 24 = 900
R 12x = 900 + 24
72 0
12x = 924
x = 924/12
72 0 = 770
S V
2009/6b Neco (Dec)
Thus RVS + 72 + 72 = 180 (sum of s in ). 0
The interior angles of a pentagon x0, (x + 10)0, (2x – 25)0,
RVS = 180 – 144
(3x + 5)0 and (x – 10)0,
= 360 ( A )
(i) Find x
(ii) What is the value of the biggest interior angle?
2012/47
(iii) Determine the exterior angle at the vertex where
4x the interior angle is (2x – 25)0
Solution
x Pentagon is 5 sided polygon
Sum of interior angle of polygon = (n – 2)180
3x
x + (x + 10) + (2x – 25) + (3x + 5) + (x – 10) = (5 – 2)180
2x x + x + 10 +2x – 25 + 3x +5 + x – 10 = 3 180
2x 8x + 15 – 35 = 540
8x – 20 = 540
The diagram is a polygon. Find the largest of its interior 8x = 540 + 20
angles 560
A 300 B 1000 C 1200 D 1500 x= = 700
8
Solution (ii) Biggest angle is (3x + 5)0 = 3 × 70 + 5
The figure is a five sided polygon with given exterior angles
= 210 + 5 = 2150
The largest interior angle is gotten from the smallest
(iii) Exterior angle + interior angle = 1800
exterior angle x (sum of s on straight line)
Sum of exterior angles of polygon = 360 Exterior angle + (2x – 25) = 180
4x + 3x + 2x + 2x + x = 360 Exterior angle = 180 – (2x – 25)
12x = 360 = 180 – (2 × 70 – 25)
x = 360/12 = 180 – (140 – 25)
x = 30 = 180 – 115
x + interior angle = 180 (sum of angles in straight line) = 650
30 + interior angle = 180 2008/23
Interior angle = 180 – 30 The angles of a quadrilateral are (x + 10)0, 2y0, 900 and
= 1500 ( D ) (100 – y)0. Find y in terms of x.
A y = 160 + x B y = 100 + x C y = 160 – x
D y = x – 100
303
Solution (ii) Sum of interior angles = (2n – 4)90
A quadrilateral is a four sided polygon = (2  12 – 4)90
Sum of interior angle of polygon = (n – 2)180 = (24 – 4)90
x + 10 + 2y + 90 + 100 – y = (4 – 2)180 = 20  90 = 1800
x + y + 200 = 360
y = 360 – 200 – x Alternatively (by algebraic representation)
= 160 – x ( C ) Let each exterior angle be x
Each interior angle will be x + 120,
2007/36 Neco We sketch as :
If the exterior angles of a quadrilateral are 2y0,
(x+120)
(3y – 5)0, (3y – 15)0 and (5y – 10)0, find y
A 61.430 B 600 C 52.860 D 51.430 E 300 x
Solution
Quadrilateral is a 4 sided figure x + 120 + x = 180 (sum of angles in straight line)
Sum of exterior angles of polygon = 360 2x + 120 = 180
2y + (3y – 5) + (3y – 15) + (5y – 10) = 360 2x = 180 – 120
2y + 3y – 5 + 3y – 15 + 5y – 10 = 360 2x = 60
13y – 30 = 360 x = 60/2 = 300
13y = 360 + 30 Other part follows as solved above
13y = 390
y = 390/13 2013/42
= 300 ( E ) An interior angle of a regular polygon is 5 times each
2008/4a exterior angle. How many sides has the polygon?
A pentagon is such that one of its exterior angles is 600. A 15 B 12 C9 D6
Two other are (90 – m)0 each while the remaining Solution
angles are (30 + 2m)0 each. Find the value of m Let each exterior angle be x
Solution Then each interior angle will be 5x
Sum of exterior angles of polygon = 4  90 5x
Here pentagon has 5 sides (angles)
x
60 + 2  (90 – m) + 2(30 + 3m) = 4  90
So after one, two angles, the remaining must be two 5x + x = 180(Sum of s on straight line)
angles as well 6x = 180
60 + 180 – 2m + 60 + 4m = 360 6 x 180
=
4m – 2m + 300 = 360 6 6
2m = 60 x = 300
m = 60/2 = 300 360
But each exterior angle of polygon =
n
360
2005/7a counter example 30 =
A regular polygon of n sides is such that each interior n
angle is 1200 greater than the exterior angle. 30n = 360
Find : (i) the value of n 360
n=
(ii) the sum of all the interior angles 30
Solution = 12 ( B )
Interior angle = exterior + 1200 2011/7b Neco
Recall that: The interior angle of a regular polygon is four times the
interior angle + exterior angle = 180(sum of angles on exterior angle. Calculate the:
straight line) (i) exterior angle;
It will be easier to work with exterior angle here. Thus (ii) number of sides of the polygon
exterior angle + 120 + exterior angle = 180 Solution
2exterior angle + 120 = 180 Let each exterior angle be x
2exterior angle = 60 Then each interior angle will be 4x
Exterior angle = 60/2 = 300 4x
360
But each exterior angle of a polygon = x
n
360 4x + x = 180 (sum of s on straight line)
30 =
n 5x = 180
30n = 360 5 x 180
=
360 5 5
n=
30 x = 36
= 12

304
(ii) Each exterior angle of polygon =
360 154 = 14n
n 154 14n
=
360 14 14
36 =
n 11 = n (D)
36n = 360
360 2014/2a Neco (Dec) interior angles
n= = 10 sides
36 One interior angle of a hexagon is 1400
2013/4a Neco counter example Calculate, to the nearest degree, each of the remaining
If the interior angle of a regular polygon is x times the angles, given that they are all equal to each other.
exterior angle, express the number of sides of the Solution
polygon in terms of x We are dealing with 6 sided polygon
Solution Sum of interior angle of polygon = (n – 2)180
Let each exterior angle be y Out of the six angles, one is 140
Then each interior angle will be xy Let x be each of the five angles left which are equal.
xy That is 5x
140 + 5x = (6 – 2) 180
y
140 + 5x = 4  180
xy + y = 1800 (sum of s on straight line) 5x = 720 – 140
y (x + 1) = 180 5x = 580
180 580
y= x=
x +1 5
Each exterior angle of polygon =
360 = 1160
n
180 360 2013/40 Neco interior angles
=
x +1 n The sum of six angles of an eleven sided polygon is 10000.
Make n subject of formula The other five angles are equal. Find the size of each of the
180n = 360 (x + 1) equal angles
360( x + 1) A 470 B 770 C 920 D 1240 E 1540
n=
180 Solution
n = 2(x + 1) We are dealing with 11 sided polygon
Since the angle is not specified to be exterior,
2006/6b Neco counter example it must be interior angle
A polygon has three of its interior angles to be 1600 Sum of interior angles of polygon = (n – 2)180
each and others are 1200 each. Calculate the number of Out of the eleven angles; six are 1000
sides of the polygon Let x be each of the five angles left which are equal.
Solution That is 5x
Sum of interior angles in polygon = (n – 2)  180 1000 + 5x = (11 – 2)180
The sum of its three interior angles = 160  3 = 480 1000 + 5x = 9  180
sum of the remaining interior angles = (n – 3)  120 5x = 1620 – 1000
Recall that sum of interior angles in polygon = (n – 2)  180 5x = 620
480 + (n – 3)120 = (n – 2)180 x=
620
= 1240 ( D )
480 + 120n – 360 = 180n – 360 5
480 – 360 + 360 = 180n – 120n
480 = 60n 2005/7b Neco interior angles
480 60n Three of the angles of a hexagon are 1220, 710 and 950 and
=
60 60 the other three angles are in the ratio 2 : 3 : 3. Calculate the
8=n size of each of the unknown angles.
2012/40 Neco Solution
Two of interior angles of a polygon are 630 each and the Hexagon is a six sided polygon
remaining interior angles are 1660 each. How many Since the angle is not specified to be exterior
sides has the polygon? It must be interior angle
A7 B9 C 10 D 11 E 13 Sum of interior angles of polygon = (n – 2)180
Solution Three of the six angles are given
Sum of interior angles in polygon = (n – 2)180 Let x be the value of the angles which are in ratio 2:3:3
The sum of its two interior angles = 63  2 = 1260 2x : 3x : 3x
Sum of the remaining interior angles = (n – 2)  166 Resulting to 8x
Recall that: Sum of interior angles in polygon = (n – 2)180 Thus 122 + 71 + 95 + 8x = (6 – 2)180
126 + (n – 2) 166 = (n – 2)180 288 + 8x = 4  180
126 + 166n – 332 = 180n – 360 8x = 720 – 288
126 – 332 + 360 = 180n – 166n

305
x=
432 Sum of interior angles of polygon = (n – n)180
8 = (9 – 2)180
x = 540 = 7 180
. . 2x = 2  54 = 108
. 0
= 12600 (B)
3x = 3  54 = 1620
3x = 3  54 = 1620 2007/21 Exercise 19.14
Each interior angle of a regular polygon is 1620.
2010/27 How many sides has the polygon?
The sum of the exterior angle of an n – sided convex A 8 B 12 C 16 D 20
polygon is half the sum of its interior angles. Find n 2007/ 28 Neco Exercise 19.15
A6 B8 C9 D 12 The sum of the interior angles of a polygon is
Solution 26 right angles. How many sides does the polygon have?
Sum of exterior angle = 12 sum interior angle A 45 B 35 C 25 D15 E5
4  90 = 12 (2n – 4)  90 2011/39 Neco Exercise 19.16
90 will cancel out How many sides has a regular polygon if each of the interior
4= n–2 angles is 1200?
4+2= n A8 B6 C5 D4 E3
6= n (A)
2014/49 Neco Exercise 19.17
The sum of the interior angles of a regular polygon is 30600.
2011/11
How many sides has the polygon?
A regular polygon of n sides has each exterior angle
A9 B 15 C 17 D 19 E 21
equal to 450. Find the value of n
A6 B8 C 12 D 12 2006/37 Neco (Dec) Exercise 19.18
Solution What is the name of a regular polygon in which one of its
Each exterior angle of polygon =
360 interior angles is 1350?
n A Decagon B Heptagon C Hexagon
45 =
360 D Nonagon E Octagon
n
2014/2b Neco (Dec) Exercise 19.19
45n = 360
360
A polygon has the following interior angles. k0,
n= (k + 15)0, (k + 30)0, (k + 35)0, (k + 45)0 and (k + 55)0.
45
Find the value of k
=8 (B)

2009/38 (Nov)
A regular polygon has 15 sides. Find the size of each of
the exterior angle
A 420 B 300 C 240 D 180
Solution
360
Each exterior angle of polygon =
n
360
=
15
= 240 (C)

2009/39
Each of the interior angles of a regular polygon is 1400.
Calculate the sum of all the interior angles of the
polygon
A 1080 B 1260 C 1800 D 2160
Solution
(n − 2)180
Each interior angle of polygon =
n
(n − 2)180
140 =
n
140n = (n – 2) 180
140n = 180n – 360
360 = 180n – 140n
360 = 40n
360
= n i.e n = 9
40
306
Other cases Solution
2010/39 Neco Obtuse angles are between 900 and 1800
In the diagram below, /UV/ = /UX/ and /VY/ = /YX/, 58 + 22 + reflex angle QRS + 41 = 3600 (sum of s in Quad)
VUX = 500 and VYX = 1200. Find UVY 121 + reflex  QRS = 360
U Reflex QRS = 360 – 121
= 239
50 0
Reflex QRS + obtuse QRS = 3600 (sum of s at a point)
Obtuse QRS = 360 – 239
Y = 1210 ( C )

120 0 X 2012/33 Neco


V
A 170 0
B 700
C 65 0
D 35 0 0
E 30 The vertex angle of an isosceles triangle is 800. If the base is
Solution 8cm long, what is the length of each of the equal sides?
U A 6.2cm B 8.0cm C 8.8cm D 9.0cm E 12.4cm
0
Solution
50 The isosceles triangle’s other two equal angles are
180 − 80
= = 500 each
2
Y
0
80
d 120 0 c
a b
V X x
x
In VXY,
0
50 0
a = b (base s in Isosceles) 50
8m
120 + a + a = 1800 (sum of s in )
2a = 180 – 120 8
=
x
Applying sine rule:
a = 60/2 = 300 sin 80 sin 50
In UVX, 8 sin 50
=x
a + d = b + c (Base s of isosceles) sin 80
30 + d = 30 + c 8  0.7660
50 + 30 + d + 30 + d = 1800 (sum of s in ) =x
0.9848
2d = 180 – 110 6.2cm = x ( A )
d = 70/2 = 350 (D)
2009/22 (Nov) 2006/33 Neco (Dec)
Find the value of x in the diagram Below is PQR such that /PQ/ = /PR/, /SQ/ = /PS/
P
and PRS = 500. What is the size of PSQ?
P
36 0

x
180
Q R
50 0
A 1840 B 1960 C 2040 D 2160 Q S R
Solution
36 + 90 + 18 + x = 3600 (sum of s in Quad) A 250 B 500 C 800 D1000 E 1100
144 + x = 360 Solution
x = 360 – 144 PQS = 500 (base angles of Isosceles PQR)
= 2160 (D) QPS = 500 (Base angles of Isosceles  PSQ)
2010/3 In PSQ
P
PQS + QPS + PSQ = 1800 (sum of angles in )
58 0 50 + 50 + PSQ = 180
PSQ = 180 – 100
= 800 ( C )
0
22 R 41 0
S Q
In the diagram, PSR = 220, SPQ = 580 and
PQR = 410. Calculate the obtuse angle QRS
A 990 B 1000 C 1210 D 1650
307
2006/52 Neco 2012/43 Neco
S
Find the value of x in the triangle below

x+30 0
R

0
x+10 x+20 0
0
72
P Q
A 100 B 200 C 300 D 400 E 500
In the Figure above, PQ = PR , RQ = RS and PRS Solution
is a straight line. If QPR = 720, find QSR (x + 300) + (x + 100) + (x + 200) = 1800 (sum of s in )
A 720 B 630 C 540 D 360 E 270 x + 30 + x + 10 + x + 20 = 180
Solution 3x + 60 = 180
In QPR, 3x = 180 – 60
Let x = PRQ = PQR (Base angles of isosceles 3x = 120
QPR) x = 120/3 = 400 ( D )
72 + x + x = 1800 (sum of s in ) 2012/36
2x = 180 – 72
x
x = 108/2 = 540
Next, PRQ + QRS = 1800 (sum of s on straight line)
54 + QRS = 180
550 105 0
QRS = 180 – 54
= 1260 Find the value of x in the above diagram
In QRS A 500 B 550 C 1000 D 1050 E 1100
Let y = QSR = RQS (Base angles of isosceles QRS) Solution
126 + y + y = 1800 (sum of s in QRS) x + 55 = 105 (exteriors and interior opposite s in )
0
2y = 180 – 126 x = 105 – 55
y = 54/2 = 500 (A)
= 270 ( E )
2006/59 Neco 2013/41 Neco
E
A D C
0
D 38

B
69 0 42 0 In the diagram above /AB/ = /BD/ and DAB = 380.
A B C
Find BDC
In the diagram above, AEB and BCD are two triangles
A 380 B 710 C 1040 D 1090 E 1420
on the straight line AC. If BC = BD and EAB = 690 Solution
and BCD = 420 , find AEB ADB = 380 (Base angle of isosceles )
A 960 B 840 C 690 D 480 E 270 ADB + BDC = 1800 (sum of s on straight line)
Solution 38 + BDC = 180
E
BDC = 180 – 38
z D = 1420 ( E )
x
2009/40 Neco (Dec)
Given triangle BCD with CB and CD produced to A and E
  
69 0 y
42 0 respectively, if BCD = 680, A B D = 1430. Find B D E
A C A
B
B
x = 420 (Base angle of isosceles DBC) 143 0
x + y + 420 = 1800 (sum of s in )
42 + y + 42 = 180
y = 180 – 84 68 0
C
= 96
Next, 69 + z = 96 (exterior s and interior opposite s in )
D
z = 96 – 69 E
= 270 ( E ) A 37 0
B 68 0
C 75 0
D 1050 E 1120
308
Solution 2014/15 Exercise 19.20
CBD + 1430 = 1800 (sum of angles on straight line) P Q
110 0
CBD = 180 – 143
= 370 R
68 + CBD = BDE (exterior s and interior opp s in )
68 + 37 = BDE
1050 = BDE ( D ) 20 0
T S

2006/43 In the diagram, QPT = PTS = 900, PQR = 1100 and


In the diagram, what is x + y in terms of z? TSR = 200. Find the size of the obtuse angle QRS
z0 A 1400 B 1300 C 1200 D 1100

2011/37 Exercise 19.21


S T U V
40 0 0
110
x0 y0
Y
A 180 – z
0
B 180 + z 0
C z – 180 0
Dz W
Solution 40 0
Exterior z = interior z (vertically opp. s)
x + y + z = 1800 (sum of s in ) X
x + y = 180 – z (A ) In the diagram, STUV is a straight line. UXY = 400 and
VUW = 1100. Calculate  TYW
2005/17 (old) A 1500 B 1400 C 1300 D 1200
The angles of a triangle are in the ratio 5:3:2.
What is the size of the smallest angles? 2011/10 Exercise 19.22
A 900 B 720 C 540 D 360
Solution X
Sum of angles in  is 1800,
sum of ratio is 5 + 3 + 2 = 10
2
Smallest angle =  180
10
= 360 520

2005/57 Neco
A Find the size of the angle marked x in the diagram
A 1080 B 1120 C 1280 D 1420
250
2015/50 Exercise 19.23
Y

68 0
125 0
54 0
M B D C O

In the diagram above, DCA = 540, DAC = 250 and


ABM = 1250. Calculate BAD X Z
A 1340 B 1010 C 790 D 550 E 460 In the diagram, OX bisects YXZ and OZ bisect YZX. If
Solution XYZ = 680, Calculate the value of XOZ
1250 = (BAD + 25) + 540 (Exterior angle equal sum A 680 B 720 C 1120 D 1240
of opposite interior angles)
125 = BAD + 25 + 54 2005/9a (Nov) Exercise 19.24
125 – 79 = BAD In the diagram show that r = m – n – t
460 =  BAD ( E )
t0

n0 m0 r0

309
2011/40 Neco Exercise 19.25 CIRCLE GEOMETRY
In the diagram below, what is the value of ABC ?
A B
500
CHORD

Properties of Chords
C
(a)The line joining the center to the mid-point of a
70 0 chord is perpendicular to the chord as indicated
A 1300 B 1200 C 700 D 600 E 500 in the diagram below
2013/35 and 36 Neco Exercise 19.26
A E

58 0

72 0
R (b) Equal chords of a circle are equidistant from the
S
C
centre and conversely
B D

Use the figure above to answer questions 35 and 36 (1) THEOREM : T


The angle which an arc of a circle subtends at the centre is twice
35. What is the value of R? that which it subtends at any point on the remaining part of the
A 140 B 580 C 720 D 1220 E 1300 circumference.
P
P
36. The ACD equals
y2
A 140 B 580 C 720 D 1220 E 1300 x2
x2 y2
y1
2006/41 Neco (Dec) Exercise 19.27 y1 x1 B
x1
Find the value of x in the diagram below O B A
O
A Q
Q
(55+x)0 (2)
(1)

O P
(75-x) 0 (2x)0 Q x2
y2
x1
A 500 B 250 C 200 D 180 E 120 y1
A (3)
2007/25 Exercise 19.28 B
In a PQR, PQR = PRQ = 450. Which of the Given: a circle APB with centre O
following statements is/are correct? To prove: AÔB = 2 x APB
I. PQR is an equilateral triangle Construction: Join PO and produce it to any point Q
II. PQR is an isosceles triangle Proof:
III. PQR is a right – angled triangle /OA/ = /OB/ (radii)
A II only B I and II only C II and III only  x1 = x2 (Base s of iso. ∆ )
D I and II only  AÔQ = x1 + x2 (Ext.  of ∆ AOP)
AÔQ = 2x2 (x1 = x2)
2005/16 (Old) Exercise 19.29 Similarly , BÔQ =2y2
The angles of a triangle are (2x – 30)0, (2x – 60)0
(1) AÔB
and (x – 30)0. What is the value of x?
(2) reflex AÔB = AÔQ + BÔQ
A 105 B 60 C 40 D 30
= 2x2 + 2y2
= 2(x2 + y2)
= 2 x APB
From (3) AÔB = B ÔQ – AÔQ
= 2y2 – 2x2
= 2(y2 – x2)
= 2 x APB
In all cases AÔB = 2 x APB
310
(2) Theorem : (6) Theorem
Angles in the same segment of a circle are equal A tangent is perpendicular to the radius at the point of
P Q contact
x1 x2

O
O
2x
A B
A B
C
Given :Point P,Q on the circumference of a circle APQB.
To prove : APB = AQB (7)Theorem
Construction: Join A and B to O, the center of the circle. The angle between a tangent to a circle and a chord through
Proof: the point of contact is equal to the angle in the alternate
AÔB = 2x1 (s at center = 2 ×  at circum) segment.
B B
= 2x2
 x 1 = x2

(3) Theorem: OR
C a a
The angle in semi- circle is 900 C
P
a a
Q A T Q A T

A B ( 8)Theorem
O
Two chords PT and RS of a circle intersect outside or inside
a circle at a point Q
Q

0
P = 90 T
R
P
(4) Theorem:
P S
Angles in opposite segments are supplementary.
(opposite angles of a cyclic quadrilateral are Q
supplementary)
B S

A p R
T
x
PQ ∙ QT = QR ∙ QS
y
q ( 9)Theorem
C Two tangents SR and SK to a circle with cenre O are equal
D in length
R
x + y = 1800
p + q = 1800

(5) Theorem: O S
The exterior angle of a cyclic quadrilateral is equal to
the interior - opposite angle.
B
A
x K

SR = SK,
OSR = OSK
x SOR = SOK
C D
R = K = 90o

311
( 10)Theorem 2015/9a Alternative solution
Q
ABC = 52 + x (Exterior and interior s in CBP)
R
DCQ = x ( vertically opposite s )
Also, ADC = 76 + x (Exterior and interior s in DCQ)
ABC + ADC = 180 (s in opp. Segment in Quad ABCD)
K
52 + x + 76 + x = 180
2x + 128 = 180
2x = 180 – 128
S 2x = 52
x = 260
If QK is a secant and QS is a tangent to a circle, 2010/3b
C
then
SQ2 = QR . QK D
600

( 11)Theorem
Transverse common tangent to two circles 5x 0 E
S B

r
K x0
W
O S A T
r
In the diagram, TS is a tangent to the circle at A.
AB//CE, AEC = 5x0, ADB = 600 and TAE = x.
T
Find the value of x
SK = OK = r Solution
TK WK r BAS = 600 (Alternate segment s)
BAE + 5x = 1800 (Sum of interior opp. s)
BAE = 180 – 5x
2015/9a
Q But BAS + BAE + x = 1800 (Sum of s on line SAT)
60 + 180 – 5x + x = 180
760 240 – 4x = 180
240 – 180 = 4x
60 = 4x
D 60
= x thus x = 150
4
2014/50 (Nov)
C
N
x
A

B 42 0 x
52 0 M
O P Q
P
In the diagram, ABCD is a cyclic quadrilateral.
AB and DC are produced to meet at P, AD and BC are
produced to meet at Q. if DQC = 760, BPC = 520
and BCP = x, calculate the value of x. In the diagram, MP is a diameter of the circle, MQ is a
Solution straight line, NMP = 420, NQP = x and NQ is a tangent
DAB = x (Exterior and interior s of cyclic quad ABCD) to the circle at N. Calculate the value of x.
DCQ = x (Vertically opp. s ) A 480 B 480 C 420 D 60
DCB = 180 – x (angles on straight line ) Solution
Thus QCP = x + x + 180– x QNP = 420 ( Tangent and chord, alternate segment)
= x + 180 NMQ + MNQ + x = 180(sum of s in MNQ)
QAP + 76 + 52 + x + 180 = 360(sum of s quad AQCP) 42 + (90 + 42) + x = 180 (sum of s in MNQ)
x + 76 + 52 + x + 180 = 360 174 + x = 180
2x = 360 – 308 x = 180 – 174
x = 52/2 = 260 x = 60 (D)
312
2012/3a 2014/12a (Nov)
R R

360

1000 Q
S
100 0 O
V
S
210

U T P
In the diagram, TU is a tangent to the circle.
T
RVU = 1000 and URS = 360, calculate the value of
angle STU In the diagram, TS is a tangent to the circle at S. If O is the
Solution center of the circle, TSP = 210 and RQP = 1000, find,
100 + USR = 1800 (opp. segment s) with reasons.
USR = 180 – 100 ( i ) SPR; ( ii ) QSR
= 800 Solution
RVU = UST = 1000 (Exterior & interior opp. s) SRP = 210 (Alternate segment s)
SUT = 360 (Alternate segment s) RSP + 1000 = 1800 (Opposite segment s)
In  STU, RSP = 180 – 100
SUT + UST + STU = 180 (Sum of s in ) = 80
36 + 100 + STU = 180 SRP + RSP + SPR = 180 (Sum of s in )
STU = 180 – 136 21 + 80 + SPR = 180
= 440 SPR = 180 – 101
= 790
2015/3b ( ii ) SPQ = 90 (Semi – circle angle)
0

P Q U RSP + RQP = 1800 (Opp. segments)


T
74 0
68 0 RSP + 100 = 180
RSP = 180 – 100
O = 80
S 40 0 QST = 90 (Radius and tangent at contact)
0

QSP = 90 – 21
= 690
R Thus QSR = RSP – QSP
= 80 – 69
The diagram shows a circle PQRS with centre O,
= 110
UQR = 680, TPS = 740 and QSR = 400. 2008/11 b & c
Calculate the value of PRS B
Solution
PSR = 680 (Exterior and interior opp s)
SRQ = 740 (Exterior and interior opp s)
C
SPQ + SRQ = 1800 (Opp s in cyclic quad) A O 7 cm
SPQ + 74 = 180 350
190 0
SPQ = 180 – 74
= 1060
PSR + PQR = 1800 (Opp s in cyclic quad)
68 + PQR = 180
D
PQR = 180 – 68
= 1120 (b) The diagram shows a circle ABCD with center O and
PSQ = PSR – 400 radius 7cm. The reflex angle AOC = 1900 and
= 68 – 40 angle DAO = 350. Find ( i ) ABC ( ii ) ADC
= 280
(c) Using the diagram in 11(b) above, calculate,
In SPQ, S + P + Q = 1800 (Sum of s in )
correct to 3 significant figure, the length of:
28 + 106 + Q = 180
( i ) arc ABC ( ii ) the chord AD (Take  = 3.142)
Q = 180 – 134
Solution
= 460
b. AOC = 360 – 190 (Sum of s at a point)
Thus PRS = PQS = 460 ( same segment s )
= 170

313
ADC = 170 (center & circumference s) 104
OTS = = 52
2 2
= 850 TSR = 900 (Semi – circle angle)
ABC + ADC = 1800 (Opp. s in cyclic quad) In TSR:
ABC + 85 = 180 RTS + TSR + TRS = 180(Sum of s in )
ABC = 180 – 85 = 950 52 + 90 + TRS = 180
 TRS = 180 – 142
C ( i ) Length of arc ABC =  2r
360 = 380
170 PTS + PRS = 1800 (Opp. s in cyclic quad)
=  2  3.142  7
360 (PTR + 52) + (33 + 38) = 1800
= 20.77cm = 180 – 123 = 570 ( C )
 20.8cm to 3s.f
 20. Find the size of angle PRS
(ii) Chord AD = 2r sin A 760 B 710 C 380 D 330
2
B Solution
 PRS = 33 + 38
= 710 (B)
C
A O 7 cm 2012/20
350
O

O S
D x0 86 0
In AOD : ADO = 350 (Base s of isosceles )
P R
35 + 35 +  = 180 (Sum of s in )
 = 180 – 70
= 1100 Q
110 In the diagram, O is the center of the circle PQRS
Chord AD = 2  7  sin
2 and PSR = 860. If POR = x0, find x
= 14  sin 55 A 274 B 172 C 129 D 86
= 14  0.8192 Solution
= 11.47cm  11.5cm to 3s.f x = 2  86 (Centre and circumference s)
= 1720
2015/19 and 20
Q 2012/41
R
0
33 R

P O
O
8x S
76 0 P
x

T S T
The diagram is a circle with center O. PRST are points on
In the diagram, O is the centre of the circle, RT is a the circle. Find the value of PRS
diameter, PQT = 330 and TOS = 760. Use the A 1440 B 720 C 400 D 360
diagram to answer questions 19 and 20 Solution
PRS = 8x (Centre and circumference s)
19. Calculate the value of angle PTR 2
A 730 B 670 C 570 D 370 = 4x
Solution PRS + PTS = 1800 (Opp. s in cyclic quad)
PRT = 330 (Same segment s) 4x + x = 180
In TOS : OTS = OST (Base s of isosceles) 5x = 180
OTS + OST + TOS = 1800 (Sum of s in ) x = 180 = 36
2OTS + 76 = 180 5
2OTS = 180 – 76 Thus PRS = 4x
2OTS = 104 = 4  36 = 1440 ( A )

314
2014/9b 2015/16
X
500
Y
O
P Q
O
E 140 0
R S
M N
Z
In the diagram, O is the centre of the circle. If PQ//RS and
ONS = 1400, find the size of POM.
W A 400 B 500 C 600 D 800
In the diagram, O is the centre of the circle. Solution
If WX is parallel to YZ and WXY = 500, find the MON is isosceles (OM = ON Radii)
value of: (i) WYZ; (ii) YEZ ONM + 1400 = 1800 (Sum of s on line MNS)
Solution ONM = 180 – 140
In WXY = 400
XWY + 50 + 900 = 1800 (Sum of s in ) OMN = ONM = 400 (Base s of isosceles)
XWY = 180 – 140 POM = OMN = 400 (Alternate s between parallel lines)
= 400
WYZ = XWY = 400 (Alternate s between parallel) 2005/6a NABTEB
Find the value of a and b in the figure below
(ii) WOZ = 2WYZ ( Centre  = 2 circumference ) D
WOZ = 2 × 400 0
E 80
WOZ = 800 1200
WOZ = YZO ( Alternate s )
A a
YEZ + YZO + WYZ = 1800 (Sum of s in )
YEZ = 180 – (80 + 40) b
B
YEZ = 180 – 120
C
= 600
Solution
2014/21 (Nov) 120 + b = 1800 (Opp. segment s)
X b = 180 – 120
Y = 600
a + 80 + b = 1800 (sum of s in )
P m
a + 80 + 60 = 180
a = 180 – 140
1020 = 400
O
2005/22 (Nov)
Z
300

W T
0
O 20
In the diagram, O is the center of the circle ,
WY and XZ are straight lines, WPZ = 1020,
XWY = 300 and PYZ = m. Find the value of m.
A 780 B 720 C 510 D 360 P S
Solution
XPW + 102 = 1800 (Sum of s on line XPZ)
XPW = 180 – 102
= 780 Q R
In XPW : 30 + 780 + PXW = 1800 (Sum s in ) In the diagram, O is the centre of the circle, PTS = 200
PXW = 180 – 108 and /OQ/ = /OR/. Calculate OQR
= 720 A 1400 B 800 C 700 D 400
m = PXW = 72 (Same segment s)
0
Solution
POS = 2  200 (centre and circumference s)
= 400
 
In QOR : Q = R (Base s of isosceles)
315
   Solution
O + Q + R = 1800 (Sum of s in) In PRS : P = S = R = 600 (Equilateral )

  PSR + PQR = 1800 (Opp s in cyclic quad)
O + 2 Q = 180
60 + PQR = 180

40 + 2 Q = 180 PQR = 180 – 60
= 1200

2 Q = 180 – 40 = 140 In PQR: P + Q + R = 1800 (Sum of s in )

But P = R (Base s of isosceles )
Q = 140 = 700 ( C ) Thus, 2R + 120 = 180
2 2R = 180 – 120
2R = 60
2009/7b R = 300
P  QRS = 30 + 60 i.e QRP + PRS
0
= 900 ( C )
15
2007/24
S In the diagram, O is the centre of the circle and
T  PQR = 1060, find the value of y.
O

32 0
Q R
O

2y 0

In the diagram, O is the centre of the circle, P R


OQR = 320 and TPQ = 150. 106 0
Calculate : (i) QPR; (ii) TQO
Solution Q
In QOR: A 16 B 37 C 74 D 127
QRO = 320 (Base s of isosceles) Solution
32 + 32 + QOR = 1800 (Sum of s in ) Reflex POR = 106  2 (Centre and circumference s)
QOR = 180 – 64 POR = 212
= 1160 212 + 2y = 3600 (Sum of s at a point)
1 2y = 360 – 212
QPR = QOR (Center & circumference s) 2y = 148
2
116 y = 148 = 740 ( C )
= = 580 2
2
(ii) QTR = 580 (Same segment s on line QR) 2005/6b (old)
TRQ = 150 (Same segment s on line TQ) A
In TQR : 58 + 15 +TQR = 1800 (Sum of s in )
TQR = 180 – 73
= 107
 TQO = 107 – 32 = 750 O
0
2008/20 30 124 0
In the diagram, /PQ/ = /QR/ and /PR/ = /RS/ = /SP/. B E
Calculate the size of QRS
Q
C
D
In the diagram, O is the centre of the circle,
P R ABO = 300 and BOE = 1240. Calculate OED
Solution
124
BAE = (Centre and circumference s)
2
= 62
S Reflex BOE = 360 – 1240 (Sum of s at a point)
A 150 0
B 1200 C 900 D 600 = 2360

316
2360 + 300 + 620 + OEA = 3600 (Sum of s in Quad) Solution
OEA = 360 – 328 Reflex MON = 360 – 80 (Sum of s at a point)
= 320 = 280
OEA + OED = 1800 (Sum of s on a line) 100 + 280 + 150 + x = 3600 (Sum of s in Quad)
32 + OED = 1800 x = 360 – 305
OED = 180 – 320 = 550 ( C )
= 1480
2005/3b (old) 2005/10
P
T In the diagram, PR is a diameter, PRQ = (3x – 8)0
and RPQ = (2y – 7)0. Express x in terms of y
P
750

(2y-7) 0
Q S

O Q

(3x-8) 0
R
In the diagram, PQRS is a cyclic quadrilateral with R

/PR/ = /QR/ and P S T = 75 , calculate QRP 0 75 − 2 y 105 − 3 y
A x= B x=
Solution 3 2
PQR = 750 (Exterior angle = interior opp angle) 105 − 2 y 75 − 3 y
C x= D x=
QPR = PQR = 750 (Base s of isosceles) 3 2
In QPR : 75 + 75 + QRP = 1800 (Sum of s in ) Solution
QRP = 180 – 150  PQR = 900 (Semi – circle s)
= 300 (2y – 7)0 + (3x – 8)0 + 900 = 1800 (Sum of s in )
2005/42 2y – 7 + 3x – 8 = 180 – 90
In the diagram PQRS is a circle, /PT/ = /QT/ and 2y + 3x – 15 = 90
QPT = 700. What is the size of PRS 2y + 3x = 90 + 15
2y + 3x = 105
Q
P
3x = 105 – 2y
70 0 105 − 2 y
x= (C)
3
T
2011/4
R Q
P
S K
0 0 0 0
A 40 B 70 C 80 D 140
Solution
PQT = 700 (Base s of isosceles )
S R
PRS = SQP = 700 (Same segment s) B.
2005/23
In the diagram, O is the centre of the circle, MON = The diagram shows a cyclic quadrilateral PQRS with its
800, LMO = 100 and  LNO = 150. Calculate the diagonals intersecting at K. Which of the following triangles
value of x is similar to triangle QKR?
A PQK B PKS C SKR D PSR
M
Solution
100 SPR = SQR (Angles on same segment)
Line SR is same base to SPR and SQR
800 Thus SPR is similar to SQR
O
 QKR is similar to PKS (B)
150 x L
N

A 400 B 500 C 550 D 750


317
2011/4b Solution
S y = 600 (same segment s)
3x = 2 × 600 (centre & circumference s)
400 3x = 120
120
R x= = 400
O 3
150 0
Thus x + y = 40 + 60
P = 1000 ( B )
45 0
Q 2009/39 (Nov)
Q
In the diagram, PQRS is a circle centre O.
If POQ = 1500, QSR = 400 and SQP = 450,
calculate RQS
Solution 640
P R
PSQ = 150 (centre & circumference s) S
2
= 750
PQR + PSR = 1800 (Opp. s in cyclic Quad)
(450 + RQS) + (75 + 400) = 180
45 + RQS + 115 = 180 In the diagram, PR is a diameter, /QS/ = /SR/ and
RQS = 180 – 160 = 200 SRQ = 640. Find PQS
2010/44 A 260 B 320 C 450 D 640
In the diagram, O is the centre of the circle. Solution
Find the value of x SQR = 640 (Base s in isosceles )
P
PQR = 900 (Semi – circle angle)
PQS =  PQR – SQR
R = 90 – 64
56 0
= 260 ( A )
O
2014/46 Neco
A
2x 0 146 0

P
Q R
A 34 B 29 C 17 D 14
Solution O
56 + ROQ = 1800 (sum of s on line POQ)
ROQ = 180 – 56 = 1240
2  (2x) + 124 = 1800 (Sum of s in isosceles ) h
4x + 124 = 180
4x = 180 – 124 M
4x = 56 In the figure above, O is the centre of the circle
x = 56 = 140 ( D ) and APR = 1460. Find the value of h.
4 A 340 B 560 C 680 D 730 E 1460
2010/31 Solution
146 + RPM = 1800 (Sum of s on line APM)
P Q RPM = 180 – 146
y = 340
60 0
O ROM = 2  340 (centre and circumference s)
3x
= 680
2h + 68 = 1800 (Sum of s in isosceles )
S R 2h = 180 – 68
2h = 112
h = 112 = 560 ( B )
In the diagram, O is the centre of the circle SQR = 600, 2
SPR = y and SOR = 3x. Find the value of (x + y)
A 1100 B 1000 C 800 D 700

318
2005/59 Neco 2006/32 Neco
In the figure below, O is the centre of the circle. If X Y
52 0 34 0
RPQ = 450 and PQR = 300, find the value of POS
P Q
45 0

30 0 Q
O

O
R

P R
S
A 450 B 600 C 750 D 1350 E 1500 In the figure above, P, Q, R are points on the circumference
Solution of a circle with centre O; if OXY = 520 and OYX = 340,
PRS = 45 + 300 (Exterior & interior opp. s in ) find PQR
= 75 A 940 B 860 C 620 D 470 E 430
POS = 2  750 (Centre and circumference) Solution
= 1500 E. In OXY : 520 + 340 + XOY = 180 (Sum of s )
2005/58 Neco XOY = 180 – 86
A
= 940
B
x  POR = 94 (Vertically opp. s)
0

94
PQR = (Centre and circumference s)
2
= 470 ( D )
350
95 0
C 2008/43 Neco (Dec)
D
Given a cyclic quadrilateral ABCD centre O with AD
produced to E, if AOC is 2160. Find the value of CDE
In the above diagram, what is the size of x? B
A 1300 B 1200 C 850 D 600 E 500
Solution
ABC = 950 (Exterior angle = interior opp s)
216 0
In ABC : x + ABC + 350 = 180 (sum of s in )
x + 95 + 35 = 180 O
C
x = 180 – 130 = 500 ( E ) A
2006/60 Neco x
Figure ABCD is a cyclic quadrilateral with D
E
DC produced to E. If O is the centre of the circle and
0 0 0
BOD = 1720, Calculate BCE A 72 B 108 C 144 D 1800 E 2160
A
Solution
B Obtuse AOC = 360 – 216 (s at appoint 3600)
AOC = 1440
172 0 ABC = 144 (centre and circumference s)
2
O = 720
D ABC + ADC = 1800 (Opposite segment s)
720 + ADC = 1800
C ADC = 1800 – 720
E = 1080
A 860 B 940 C 1720 D 1800 E 1880 ADC + x = 1800 ( sum of angles on line ADE)
Solution x = 180 – 1080
0 = 720 ( A )
BCD = 172 (Centre and circumference s)
2
= 860
BCD + BCE = 180 (Sum of s on line DCE)
86 + BCE = 180
BCE = 180 – 86
= 940 ( B)

319
2010/7b Neco Solution
In the diagram below, ABCD is a cyclic quadrilateral ADC + ABC = 1800 (opp. s cyclic quad)
and lines AB and DC produced meet at E. If ADC + 88 = 180
BCE = 950 and ADC = 620, ADC = 180 – 88 = 920
calculate: (i) BAD and (ii) BEC In ADC
A
420 + ADC + ACD = 1800 (sum of s in )
a 420 + 920 + ACD = 1800
B ACD = 180 – 134 = 460 ( B)
2012/39 Neco
Find y in terms of x in the diagram below
95 0 e E A
62 0
D
D C x
2y
Solution
a = 95 (Exterior and interior opp. s in cyclic quad)
0

(ii) ABC + 620 = 1800 (opp. s in cyclic quad) B C


ABC = 180 – 62
= 1180
EBC = 180 – ABC (Sum of s on line ABC) Ay=x B y = 2x Cy= x Dy= x E y = x2
= 180 – 118 2
= 620 Solution
In BEC : 95 + 620 + BEC = 1800 (Sum of s ) 2y = x (same – segment s)
BEC = 180 – 157 = 230 y = x C.
2011/11b Neco 2
P 2013/37 Neco
a In the diagram below, O is the centre of the circle, BOD is a
c
diameter and BAC is 420. What is CBD?
D
38 0 480 b
T R
Q O
A
420 O

In the diagram above, O is the centre of the circle PQR, C


what are the values of a, b and c?
B
Solution
a = 900 (Semi – circle angle)
A 420 B 480 C 770 D 900 E 1380
TQP = 1800 – 480 (s on line TQR)
Solution
= 1320
BDC = 420 (Same segment s)
In TPQ : 380 + 1320 + c = 1800 (Sum of s in )
c = 180 – 170 = 100 BCD = 900 (Semi – circle)
a + b = TQP = 1320 (Exterior  & interior opp s in ) In CBD ,
90 + b = 132 BDC + BCD + CBD = 1800 (Sum of s in )
b = 132 – 90 = 420 420 + 900 + CBD = 180
CBD = 180 – 132 = 480 ( B )
2014/ 45 Neco 2006/39 Neco
In the diagram below, ABCD is a cyclic quadrilateral,
ABC = 880 and DAC = 420. Calculate ACD Q
330
A
B O
420
88 0

P O
R
D
C
In the above diagram, O is the center of the circle.
Calculate the value 
A 420 B 460 C 480 D 520 E 880 A 900 B 660 C 570 D 330 E 240
320
Solution Solution
 PRQ = 900 (semi – circle s) PQS = PRS = 300 (Same segment s)
 ORQ = 330 (Base s in isosceles ) RPS = PRS = 300 (Base s of isosceles )
 = PRQ – ORQ QPR = 110 – RPS
= 90 – 33 = 570 = 110 – 30
2008/42 Neco (Dec) = 800
In the diagram below, O is the centre of the circle. In QTP: PQT + QPT + QTP = 1800 (Sum of s in )
If ZOX is a diameter and WYZ = 470, find WZX 30 + 80 + QTP = 180
Y
QTP = 180 – 110
= 700
47 0 (ii) QPS + QRS = 1800 (Opp s in cyclic quad)
110 + QRS = 180
X
QRS = 180 – 110
Z O = 700
PRQ = QRS – PRS
= 70 – 30
= 400
W 2009/10a Neco (Dec) PROOF
0 0
A 137 B 117 C 550 D 470 E 430 In the diagram below, O is the centre of the circle.
Solution P, Q, R are points on the circumference, show that x = 2y
ZXW = 470 (Same segment s) Q
ZWX = 900 (Semi – circle)
In WZX, y0
ZXW + ZWX + WZX = 1800 (Sum of s in )
47 + 90 + WZX = 180
O
WZX = 180 – 137 = 430 ( E )
2006/39 Neco (Dec) x0
In the diagram below, O is the centre of the circle XYZ. R
If XOZ = 3010, find XYZ P
Y

Solution
Q
3010

O x2 y2
Z
X

O
A 150.50 B 1180 C 59.50 D 590 E 29.50
y1
Solution x1
Obtuse XOZ = 360 – 301 (Sum of s at a point) Z R
XOZ = 590 P
XYZ = 59 (Centre & circumference s)
2 Given: a circle PQR with centre O
= 29.50 E. To prove: PÔR = 2 x PQR
2009/8a Neco Construction: Join QO and produce it to any point Z
PQRS is a cyclic quadrilateral QPS = 1100, PQS = Proof:
300, PS = SR and PS intersects QS at T. /OP/ = /OR/ (radii)
Calculate : (i) QTP; (ii) PRQ  x1 = x2 (Base s of iso. ∆ )
P  PÔZ = x1 + x2 (Ext.  of ∆ POQ)
PÔZ = 2x2 (x1 = x2)
1100
Similarly , RÔZ = 2y2
300 (1) PÔR
Q T
S (2) Obtuse PÔR = PÔZ + RÔZ
= 2x2 + 2y2
= 2(x2 + y2)
= 2 x PQR
R
 x = 2y

321
2010/45 Neco 204/47 Neco Exercise 19.31
In the diagram below, O is the centre of circle PQR. In the figure below, O is the centre of the circle MQRS
If POQ = 800 and PRQ = 2x0, find the value of x and SMR = 520, find RQM
R S

2x

52 0 M
O
R O
80 0
Q
P

Q
A 10 B 20 C 40 D 80 E 90
Solution A 260 B 380 C 880 D 900 E 1040
80 = 2(2x0) [Centre and circumference s] 2012/21 Exercise 19.32
80 = 4x P
80 Q
= x
4 2m n
20 = x B.

2011/35 Neco
In the diagram below, O is the circle. O
Find the value of x + y
A B R
x S
y
The diagram is a circle centre O. If SPR = 2m
and SQR = n, express m, in terms of n
P Q Am= n B m = 2n Cm=n–2 Dm=n+2
O
2
2012/44 Exercise 19.33
P S
480
A 90 0
B 120 0
C 180 0
D 270 0 0
E 360 T
Solution
x = 900 (angle in Semi – circle) 650
y = 900 (angle in semi – circle)
x + y = 1800 C. Q R
2012/37 Neco Exercise 19.30
Which of the angle(s) is/are equal to WZX in the
diagram below? In the diagram, /SR/ = /QR/, SRP = 650 and RPQ = 480,
W find PRQ
V
A 650 B 450 C 250 D 190

2009/21 (Nov) Exercise 19.34


P S
700
T
Z X

250
Y
40
I WVX II WYX III WVZ Q R
A I only B II only C III only
D I & II only E II & III only
In the diagram, QPR = 700, SQR = 400
and SRT = 250. Find PRQ
A 450 B 500 C 550 D 600
322
2009/38 Exercise 19.35 2006/21 Exercise 19.39
In the diagram P, Q, R, S are points on the circle, RQS =
300, PRS = 500 and PSQ = 200.
What is the value of x0 + y-0?
2000
P Q
y0
O x0 300
300

200
x0 500
S R
In the diagram, angle 2000 is subtended at the centre of
the circle. Find the value x.
A 300 B 500 C 800 D 1000
A 2600 B 1300 C 1000 D 800
2008/46 Exercise 19.36
2006/37 Neco Exercise 19.40
In the diagram, find the size of the angle marked a0
In the figure below, PQRS is a circle with centre O.
If POS = 1000 and RSQ = 400, find the marked angle x
P
Q
280 0 x
O O
1000
R
600 a0
R T
0
100 0 40

S S
0 0 0 0
A 60 B 80 C 120 D 160 A 40 0
B 500 C 900 D 1000 E 1400

2008/25 Exercise 19.37 2009/45 Neco (Dec) Exercise 19.41



In the diagram, O is the centre of the circle and PQRS is ABCD is a cyclic quadrilateral. Given that A D B = 760
a cyclic quadrilateral. Find the value of x  
P S and A B D = 360, find B C D
x0 B

O 360

C
Q R
65 0
A
76 0

A 250 B 650 C 1150 D 1300 D


0 0
A 56 B 68 C 760 D 1120 E 2480
2007/23 Exercise 19.38
P 2012/14 Neco Exercise 19.42
In the diagram below, O is the centre of the circle.
Q BOD is diameter and angle BAC is 520. Find DBC
A D

520
50 0 O
R S

B
In the diagram, /SP/ = /SR/ and PRS = 500. C
Calculate PQR.
A 140 B 380 C 520 D 1280 E 1420
A 1200 B 1100 C 1000 D 800

323
Circle geometry II Solution
2006/3b OMA = 900 (Radius and tangent s)
C QMO = 900 – 640
400 = 260
B p = QMO = 260 (Base angles of isosceles )

2014/35 (Nov)
D N

52 0
A T O
In the diagram, TA is a tangent to the circle at A. 130 0

If BCA = 400 and DAT = 520, find BAD


Solution
DCA = 520 (Alternate segment s)
L M
BAD + BCD = 180 (Opp. segment s )
BAD + (40 + 52) = 180 In the diagram, O is the centre of the circle, LM is a tangent
 BAD = 180 – 92 = 880 and angle MON is 1300. Find the size of angle OLM
2010/38 Neco A 650 B 500 C 450 D 400
Find the value of x in the diagram below where O is the Solution
centre of the circle, given that BCD = 380 and OML = 900 (Radius and tangent at contact)
BDC = 200 MOL = 180 – 1300 (s on line LON)
= 500
In OLM : OLM + 50 + 90 = 1800 (sum of s in )
OLM = 180 – 140
O B
A D = 400 D.
x 20 0

38 0
2005/5a
In the diagram, A, B, C and D are points on the
C circumference of a circle. XY is a tangent at A.
A 1420 B 1280 C 1100 D 580 E 320 Find : (i) CAX (ii) ABY
C
Solution
CBD + 200 + 380 = 1800 (Sum of s in CBD)
CBD = 180 – 58
= 122 B
D
ABC + CBD = 1800 (Sum of s on a straight line) 200
ABC = 180 – 122
= 58 600
x + ACB + 580 = 1800 (sum of s in ABC) X A Y
x + 900 + 58 = 180 Solution
(ACB is 900, angle in semi – circle) ACB = 200 (angle in same segment)
x = 180 – 148  
= 320 E. In ACY : A + C + 600 = 1800 (sum of s in )
2011/37 Neco 
In the diagram below, O is the centre of the circle. A = 180 – (60 + 20)
If /AN/ is a tangent to M and AMQ = 640. Calculate = 1000
the value of angle P (i) CAX + CAY = 180 (sum of s on line XAY)
0
N CAX + 100 = 180
CAX = 180 – 100
= 800
(ii) BAY = 20 (Alternate segment)
0

O In ABY: ABY + 20 + 60 = 1800 (sum of s in )


M ABY = 180 – 80
64 0 = 1000
p

Q
A
0 0 0
A128 B 64 C 52 D 320 E 260
324
2008/44 Solution
In the diagram, PX is a tangent to the circle and RST is OYT = 900 (Radius and tangent at contact)
an equilateral triangle. Calculate PTS. OY = OZ = 5cm (Radius)
By Pythagoras rule in OYT
OT2 = 52 + 122
S = 25 + 144
R
OT2 = 169
OT = 169 = 13cm C.
50. Find the size of OZY
A 260 B 320 C 580 D 640
P T X Solution
A 60 0
B 90 0
C 120 D 1500 0 OYZ = 90 – 58
Solution = 320
In RTS : R = T = S = 600 (equilateral ) OZY = OYZ = 320 (Base s of isosceles )
PTR = RST = 600 (Alternate segment)
2015/17
PTS = PTR + RTS P T
= 600 + 600 = 1200 C. R
2007/8b
E

O 1080
D
G 66 0
O N
F

57 0 In the diagram, PTR is a tangent to the circle centre O.


If angle TON = 1080, calculate the size of angle PTN
A B C A 1320 B 1260 C 1080 D 1020
Solution
In the diagram, O is the centre of the circle and ABC is
ONT = OTN (Base angles of isosceles )
a tangent at B. If BDF = 660 and DBC = 570,
108 + ONT + OTN = 1800 (Sum of s in )
calculate (i) EBF and (ii) BGF
Solution 108 + 2OTN = 180
BFD = 570 (Alternate segment) 2OTN = 180 – 108
BED = BFD = 570 (Angles in same segment) OTN = 72  2
= 360
OBC = 900 (Radius and tangent at contact)
OTP = 90 (Radius and tangent)
0
Thus OBD = 90 – 57 = 330
But PTN = OTP +OTN
In BDF: BFD + 66 + FBD = 1800(Sum of s in )
= 90 + 36
57 + 66 + FBD = 180
= 1260 B.
FBD = 180 – 123 = 570 2014/44
 EBF = 57 – 33= 240
(ii) In BGF
57 + 24 + BGF = 180
BGF = 180 – 81 = 990 O
2005/49 and 50 NABTEB (Nov)
86 0 S
Use the diagram below to answer question 49 and 50
XYT is a tangent to the circle centre O and radius 5cm.
/YT/ is 12cm and ZYT is 580
P Q R

In the diagram, O is the centre of the circle PR is a tangent


O
5 cm
to the circle at Q and SOQ = 860.
Calculate the value of SQR
Z
A 430 B 470 C 540 D 860
58 0
Solution
X Y 12cm T OQS = OSQ (Base s of isosceles )
86 + OQS + OSQ = 1800 (Sum of s in )
49. What is the length of OT?
A 5cm B 7cm C 13cm D 17cm 2OQS = 180 – 86

325
2OQS = 94 Solution
OQS = 470 Reflex QOP = 360 – 238 (angle at a point)
OQR = 900 (tangent and radius at a point of contact) = 1220
Thus, SQR = OQR – OQS 2  QRP = 1220 (circumference and centre s)
= 90 – 47 = 430 A. QRP = 122  2
2014/32 = 61
x + 610 = 1800 (Sum of s on line TRP)
U T x = 180 – 61
= 1190 B.
2013/7a Neco
S R

Q
x0
S 12 0
P R Q
In the figure, PQ is a tangent to the circle at R and UT is
parallel to PQ. If TRQ = x0, find URT in terms of x
A 2x0 B (90 – x)0 C (90 + x)0 D (180 – 2x)0 P
Solution 75 0 40 0

RUT = x0 (Alternate segment Angles)


A T B
UTR = TRQ = x0 (Alternate s between parallel lines)
URT + RUT + UTR = 1800 (Sum of s in URT) In the above diagram, ATB is a tangent to the circle at T.
  
URT + x0 + x0 = 180
If A T S = 750, B T P = 400 and P S Q = 120, calculate the
URT = (180 – 2x)0 D.
size of : (i) SRP (ii) SQP (iii) SPQ
2009/48 (Nov)
Solution
75 + STP + 400 = 1800 (Sum of s on line ATB)
STP = 180 – 115 = 650
X
SRP + STP = 1800 (s in opp. segment)
Y SRP = 180 – 65
= 1150
SQP = SRP = 1150 (s in same segment)
42 0 x0
In SPQ : 12 + SPQ + 1150 = 1800 (Sum of s in )
Thus SPQ = 180 – 127 = 530
L M N 2012/11b Neco
In the diagram, XMY is a triangle inscribed in a circle. In the figure below, TD is a tangent to the circle ABCT,
LMN is a tangent to the circle at M, /XY/ = /XM/ and AB = AT, BT = BC and ABT = 360. Calculate CTD
XML = 420. Find the value of x B
A 42 B 84 C 96 D 138
360
Solution
XYM = 420 (Alternate segment)
A
XMY = 420 (Base s of isosceles)
LMX + XMY + x = 180 (Sum of s on line LMN)
0 0
C
42 + 42 + x = 180
x = 180 – 84 = 960 C. x
2009/37 (Nov) T D
P
Solution
In ABT : T = 360 (Base angles of isosceles )
A + B + T = 1800 (Sum of s in )
O 238 0 R
A + 36 + 36 = 1800
A = 180 – 72 = 1080
x0
x = CBT (Alternate segment s)
BAT + BCT = 1800 (s in opposite segment)
Q T BCT = 180 – 108 = 720
In the diagram, QT is a tangent to the circle PQR centre BCT = BTC 720 (Base s of BTC)
O. If PT is a straight line, reflex POQ = 2380 and CBT + 2BTC = 1800 (sum of s in )
QRT = x0, find the value of x CBT = 180 – 144 = 360
A 1000 B 1190 C 1220 D 1300 x = CBT = 360 (Alternate segment s )
326
2005/60 Neco QTR = 900 (angles in semi circle)
In QTR : 55 + T + R = 1800 (sum of s in )
D
55 + 90 + R = 180
R = 180 – 145 = 350
O PTQ = R = 35 (Angle between tangent and chord through
0

point of contact equal to angle in alternate segment)


E
65 0 In  PTR : x + T + R = 1800 (sum of s in )
x + (35 + 90) + 35 = 1800
Note that: T in  PTR is PTQ + QTP
A T B x = 180 – 160 = 200 E.
2006/38 Neco (Dec)
IN the diagram above, AB is a tangent to a circle with
In the diagram below, XBY is a tangent to the circle ABC of
centre O at T. Given that TED = 650, find EAT
centre O at B. If CBY = 540 and /AB/ = /AC/, find XBA
A 250 B 400 C 650 D 700 E 1150
Solution
 ETD = 900 (Angle in semi – circle) A
In ETD : 65 + 90 + D = 1800 (Sum of s in ) C

D = 180 – 155 O
= 250
ATE = 250 [D] (Alternate segment, chord and tangent)
In DAT : EAT + (25 + 90) + 25 = 1800 (Sum of s in )
EAT = 180 – 140 X B Y
= 400 B.
2005/56 Neco A1260 B1170 C 1080 D 630 E 540
Solution
In the diagram below, KR is a tangent to the circle BAC = CBY = 540 (Angle between tangent and chord
with centre O at R. If RKP = 500, find RQP through point of contact equal to angle in alternate segment)
Q
ABC is isosceles (AB = AC  B =C )
Thus A(54) + B + C = 1800 (Sum of s in )
54 + 2B = 1800
2B = 180 – 54
P O 2B = 126
B = 63
XBA + ABC + CBY = 1800 (Sum of s on line XBY)
50 0
XBA + 63 +54 = 180
K R XBA = 180 – 117 = 630
0 0 2009/43 Neco (Dec)
A 140 B 130 C 900 D 500 E 400
Solution In the diagram below, AB and CB are tangent to the
QRK = 900 (tangent is perpendicular to the radius at contact point)  

In KRQ : 50 + 90 + Q = 1800 (Sum of s in ) circle. Given that C B A = 540, calculate A D C


Q = 180 – 140
= 400 A

2006/36 Neco (Dec)


D x
R

54 0
O B

C
0
Q 55 0 A 54 B 580 C 630 D 1080 E 1260
Solution
T ABC is isosceles (Tangents BA and BC are equal in lengths)
x0 Thus in ABC, A = C
P A + C + 540 = 1800 (Sum of s in )
In the diagram above, O is the centre of the circle QRT 2A + 54 = 180
and PT is a tangent to the circle at T. Calculate angle x 2A = 180 – 54
A 1450 B 1250 C 550 D 350 E 200 2A = 126
Solution A = 630

327
A = x = 630 (Angle between tangent and chord through 2006/17 Exercise 19.47
point of contact equal to angle in alternate segment) In the diagram, PR is a diameter of a circle centre O.
2014/18 (Nov) Exercise 19.43 RS is a tangent at R and QPR = 580.Find QRS
R S
S

O
Q

580
Q
730 P
P
0 0
T A 112 B 116 C 1220 D 1480
In the diagram, TP is a tangent to the circle PQRS 2005 /48 (old) Exercise 19.48
and RPT = 730. Find PQR In the diagram PQX is a tangent to the circle at Q,
A 1460 B 1340 C 1130 D 1070 RSQ = 550 and RQS = 480, calculate SQX
S
2006/57 Neco (Dec) Exercise 19.44
X
55 0

S
Q
R 48 0
Q

x0 36 0

M P
P
T
0 0 0
MT is a tangent to a circle SQP at P. If QPT = 360, A 42 B 48 C 55 D 770
find the angle marked x0.
A 180 B 360 C 540 D 720 E 1440 2010/35 Exercise 19.49
2014/48 Neco Exercise 19.45
In the diagram below, PQ is the tangent to the circle
RST at T. /ST/ = /SR/ and RTQ = 680. Find PTS. E F

S
54 0
R
G H I
In the diagram, GI is a tangent to the circle at H.
68 0 If EF//GI, calculate the size of EHF
A 1260 B 720 C 540 D 280
P T Q

A 340 B 560 C 680 D 720 E 1120 1995/27 UME Exercise 19.50


Z
2013/39 Neco Exercise 19.46
In the diagram below, BDE is a circle with centre O.
ABC is a tangent at B and BOE = 1060. Find CBE
D Y
X

O
106 0 E
50 0

P T

C
PT is a tangent to the circle TYZX, YT = YX and PTX =
A B
50o. Calculate TZY.
0 0 0
A 37 B 45 C 53 D 740 E 900 A. 50o B. 65o C. 85o D.130o
328
Circle geometry III Solution
2012/42 Applying two tangents to a circle theorem
OPT =  OQT = 900
OP = OQ = 6cm
Thus by Pythagoras rule
O
OT2 = PT2 + OP2
= 122 + 62
4 cm 2
OT = 144 + 36
Q
OT = 180
120 0 = 13.416cm
T P R
50. Find  PTQ
The diagram is a circle of radius |OQ| = 4cm. TR is a A 500 B 650 C 1150 D 1300
tangent to the circle at R. If TPO = 1200, find |PQ| Solution
A 2.32cm B 1.84cm C 0.62cm D 0.26cm In OPT : 90 + 65 + PTO = 1800 (Sum of s in )
Solution  PTO = 180 – 155
= 250
Similarly QTO = 250
Thus PTQ = 25 + 25
O
= 500 A.
4 cm
Q
4cm 2006/50 NABTEB (Nov)
120 0
60 0 In the diagram below, O is the centre of the circle with AB
T P R and AC as tangents. If BAC = 300, find BOC
B
OPR = 600 (120 + OPR = 1800 straight line)
ORP = 900 (Radius and tangent to a circle)
The resulting triangle is right – angled
O
The sides relevant to 600 here are :
Hypotenuse PO and opposite OR i.e SOH 300 A
4
Sin 600 =
PO C
4 A 2700 B 1650 C 1500 D 750
0.8660 =
PO Solution
4 Applying two tangents to a circle theorem
PO = ABO = ACO = 900
0.8660 90 + 30 + 90 + BOC = 3600 (Sum of s in Quad ABOC)
= 4.6189cm
BOC = 360 – 210
But PQ = PO – QO
= 1500 C.
= 4.6189 – 4
2009/26 (Nov)
= 0.6189cm
≈ 0.62cm .C

O
2005/49 and 50 NABTEB
Use the information given in the figure below to answer
7.0cm

questions 49 and 50
P
380 N
M
65 0 In the diagram, O is the centre of the circle and MN is a
O
tangent to the circle at M. If the radius of the circle is 7.0cm
T and MNO = 380, calculate |MN|
6cm
A 10.70cm B 8.96cm C 7.86cm D 6.42cm
Solution
Q
OMN = 900 (Radius and tangent to circle)
PT and QT are tangents to the circle centre O and radius The resulting triangle is right – angled
6cm | PT | = 12cm and  POT = 650 The sides relevant to 380 here are:
49. Calculate the length of OT Adjacent MN and opposite MO i.e TOA
A 6cm B 8cm C 10cm D 14cm
329
7 1999/31 UME
Tan 380 = Q
MN
7
0.7813 = 9
MN P
7
MN = x
0.7813
= 8.9594cm  8.96cm B. 6
T
2008/39 Neco Z
A

B In the figure above, TZ is tangent to the circle QPZ.


4cm Find x if TZ = 6 units and PQ = 9units
A. 3 B. 4 C. 5 D. 6
C Solution
Applying tangent and secant theorem
6cm
ZT2 = PT · QT
D
ZT2 = x( PQ + x )
Find |AB| in the above figure if |BC| = 4cm and |DC| = 62 = x( 9 + x )
6cm 36 = 9x + x2
A4 B5 C6 D9 E 14 x + 9x – 36 = 0
Solution factorizing: ( x + 12 )( x – 3 ) = 0
Applying tangent and secant theorem x = 3 or – 12
DC2 = BC ∙ AC x = 3 (A)
62 = 4 ∙ AC 1991/33 PCE
36
= AC I
4 H
9cm = AC
But AB = AC – BC E
G
= 9cm – 4cm = 5cm B. .O F

2004/50 NABTEB
In the diagram PR is a tangent at a point R on circle
centre O and radius OR = 8cm
R

Two chords EF and IH intersect at G when produced,


If EG = 8cm, GF = 3cm, and IG = 6cm, find GH.
O P A. 8cm B. 6cm C. 4cm D. 2cm
Solution
Applying two chords of circle intersecting outside theorem
IG.GH = EG.GF
6  GH = 8  3
If angle OPR is 600, calculate the length of OP to the
GH = 8  3
nearest whole number
6
A 4cm B 7cm C 9cm D 16cm
GH = 4cm (C)
Solution
ORP = 900 (Radius and tangent to circle)
Applying special angle trig ratio
1993/34 UME
The side relevant to 600 here are: S
Hypotenuse OP and opposite OR i.e SOH
OR
5 cm

Sin 600 =
OP Q
8 O
Sin 60 =
2 cm

OP
3 8 T
=
2 OP
In the figure above, the line segment ST is a tangent to the
2  8 16  3 two circles at S and T. O and Q are the centers of the circles
OP = =
3 3 with OS = 5cm, QT = 2cm and OQ = 14cm. Find ST
= 9.238cm  9cm to whole number A. 7 3 cm B. 12cm C. 87 cm D. 7cm
330
Solution 1991/35 and 36 UME Exercise 19.52
Let the point of intersection of the two lines be Z. Then T
let us re-label applying transverse common tangent
theorem
S

O Q P
5cm

Z x Q R
O 14 - x

2cm
M

N
T
PMN and PQR are two secants of the circle MQTRN
OZ = 5
and PT is a tangent.
QZ 2
14 – x = 5 35. If PM = 5cm, PN = 12cm and PQ = 4.8cm ,calculate the
x 2 respective lengths of PR and PT in centimeters.
2(14 – x) = 5x A. 7.3, 5.9 B. 7.7, 12.5 C. 12.5, 7.7 D. 5.9, 7.3
28 – 2x = 5x
28 = 5x + 2x 36. If PNR= 1100 and PMQ= 550 , find MPQ
28 = 7x A 400 B 300 C 250 D 150
Thus, x = 4
We can only get ST by Pythagoras rule in each of the
right – angled triangles.
In  OSZ
(14 – x)2 = 52 + SZ2
(14 – 4)2 = 52 + SZ 2
102 = 52 + SZ2
10 – 52 = SZ2
2

(10 – 5)(10 + 5) = SZ2


5  15 = SZ2
553 = SZ
5 3 = SZ
Also in  QTZ
x2 = 22 + T Z2
42 = 22 + TZ2
4 – 22 = TZ2
2

(4 – 2)(4 + 2) = TZ2
2  6 = TZ2
223 = TZ
2 3 = TZ
Therefore; ST = SZ + TZ
= 5 3 + 2 3 = 7 3 cm (A)
1986/33 UME Exercise 19.51
In the diagram below, PQ and RS are chords of a circle
center O which meet at T outside -the circle. If TP =
24cm, TQ = 8cm and TS = 12cm, find TR
P
Q

O T

R
S

A.16cm B.14cm C.12cm D.8cm


331
CHAPTER TWENTY Rearranging
x2 + 5x – 36 = 0
Plane mensuration Factorizing 36; 2  18, 2  2  9 i.e 4 and 9
Area & perimeter of plane shapes x2 + 9x – 4x – 36 = 0
Area and perimeter are terms used in mensuration. x(x + 9) – 4(x + 9) = 0
Students tend to confuse the two terms thus, (x + 9)(x – 4) = 0
interchanging the formulae to be used. x + 9 = 0 or x – 4 = 0
To give a clear difference between them, let us consider x = – 9 or 4
an example of a football field that is rectangular in shape. We accept x = 4cm as width
The 11 players of both teams play within the pitch area (ii) Perimeter = 2(L + b)
and the referee officiates within the pitch (area) = 2 [(x + 5) + x]
= 2 [(4 + 5) + 4]
Players andreferee = 2 (13)
= 26cm
2014/46 (Nov)
Find the dimensions of a rectangle whose perimeter and area
are 46cm and 112cm2 respectively
A 16cm by 7cm B 17cm by 6cm C 14cm by 9cm
As for the assistant referee (lines’ men ) they work D 12cm by 6cm
within the lines outside the field (perimeter). They run Solution
round the length and breadth of the field but not within Perimeter of rectangle = 2(L + b)
the field as the players 46 = 2 (L+ b)
23 = L + b
23 – L = b
Also area of rectangle = L  b
112 = L  b
Substituting for b
112 = L (23 – L)
112 = 23L – L2
Assistant referee (line’s men)
L2 – 23L + 112 = 0
Factorising
Thus, area is the content of any plane shape while the L2 – 16L – 7L + 112 = 0
perimeter is the distance round the shape. L(L – 16) – 7 (L – 16) = 0
(L – 16) (L – 7) = 0
Rectangle L – 16 = 0 or L – 7 = 0
L = 16cm or 7cm
Thus, dimensions of rectangle = 16cm by 7cm
b 2010/24
A rectangle has length x cm and width (x – 1) cm. if the
perimeter is 16cm, find the value of x
L A 31/2 cm B 4cm C 41/2 cm D 5cm
Solution
Area of rectangle = L x b xcm
Perimeter of rectangle = 2L + 2b
= 2 ( L + b) ( x - 1 )cm
2006/6b (Nov)
The length of a rectangle is 5cm longer than the width
and the area is 36cm2. Find the: Perimeter of rectangle = 2(L + b)
(i) width (ii) perimeter of the rectangle 16 = 2 [x + (x – 1)]
Solution 8 = x+x–1
Let the width be x, then we sketch as: 8 = 2x – 1
( x + 5)cm
8 + 1 = 2x
9 = 2x
9
xcm /2 = x,
x = 41/2 cm ( C )

Area of rectangle = L  B 2010/6b


36 = (x + 5)  x A rectangular playing field is 18m wide. It is surrounded by
36 = x2 + 5x a path 6m wide such that its area is equal to the area of the
path. Calculate the length of the field
332
Solution Solution
Area of rectangular field = 7200
6m
L  b = 7200 ----- (1)
Perimeter of rectangular field = 360
2(L + b) = 360 - --- (2)
6m 6m
18m
L + b = 180
L = 180 – b
Substitute L = 180– b into (1)
(180 – b)b = 7200
6m 180b – b2 = 7200
Rearranging
Resulting diagram b – 180b + 7200 = 0
2

(x + 12 )m Factorizing (– 120 and –60)


b2 – 120b – 60b +7200 = 0
x b(b – 120) – 60 (b – 120) = 0
30m (b – 120)(b – 60) = 0
18m
(18 + 12 )m b – 120 = 0 or b – 60 = 0
b = 120m or 60m
Alternative step to factorisation
Area of rectangular field = area of path When to factorise look difficult like we have here, we may
Area of rect. field = area of big rect. – area of small rectangle apply the general formula as
x  18 = (x + 12)  30 – x  18 − (−180)  (−180) 2 − 4  1  7200
18x = 30x + 360 – 18x b=
2 1
18x = 12x + 360
180  32400 − 28800
18x – 12x = 360 =
6x = 360 2
x = 60m 180  3600
=
2009/15 (Nov) 2
The diagonal of a rectangle is 6cm long and makes an 180  60
= = 90  30
angle of 350 with one of the sides. Find, correct to 1 2
decimal place, the perimeter of the rectangle. = 120 or 60
A 6.9cm B 9.8cm C 16.7cm D 16.9cm Without solving further L is 120m and b is 60m
Solution since both satisfies (1) and (2)
(b)
120 m
m
6c b 2m

35 0
60m

L 56m

Perimeter of rectangle = 2 (L  b)
We solve for L and b in the resulting right–angled  by trig ratio 2m
b
Sin 350 = Here the length remains the same as 120m the breadth is
6 reduced by 2m + 2m i.e 4m to become 56m
6 sin 350 = b Thus cost of clearing the field = per square metre
3.44cm = b = 120  56  6.5
L = N 43,680.00
Also cos 350 =
6 C. percentage of part not cleared
6 cos 35 = L Area of part not cleared
4.91cm = L = 100
Total area of rec tan gle
Thus, perimeter of rectangle = 2(4.91 + 3.44)
Area not cleared
= 2(8.35)
= area of total rectangle – area of smaller rectangle
= 16.7cm to 1 dp
= 120  60 – 120  56
2009/8 (Nov) = 7200 – 6720
= 480m2
The area of a rectangular football field is 7200m2 while
its perimeter is 360m. Calculate the: 480
Thus, % of part not cleared =  100
(a) dimensions of the field 7200
(b) cost of clearing the field at N 6.50per square metre, = 6.67%
leaving a margin of 2m wide along the longer sides;
(c) Percentage of the part not cleared
333
2009/11b 7b = 280
A rope 60cm long is made to form a rectangle. If the b = 40m
length is 4 times its breadth, calculate, correct to one Next, we substitute b value into (a)
decimal place the (i) length (ii) diagonal of the rectangle L + 40 = 140
Solution L = 140 – 40 = 100m
The first sentence implies
Perimeter of rectangle = 60cm 2006/45
2(L + b) = 60 A rectangle carpet 2.5m long and 2.4m wide covers 5% of a
L+ b = 30 ----- (1) rectangular floor. Calculate the area of the floor
Next, we are given that A 30m2 B 57m2 C 120m2 D 225m2
L = 4b ------- (2) Solution
Substitute (2) into (1) Let the area of the rectangular floor be x
L+ b = 30 becomes 5% of x = area of rectangular carpet
4b + b = 30 5
5b = 30 x = 2.5  2.4
100
b = 6cm
Substitute b value into (2) to get L 5
x = 6
L = 4b becomes 100
L=46 5x = 6  100
= 24cm 6  100
x = = 120m2 ( C )
(ii)
5
2007/10b
P Q
D 6cm

15
cm
24cm
By Pythagoras rule
D2 = 242 + 62 37 0
D2 = 576 + 36 S R
K
D2 = 612
In the diagram, PQRS is a rectangle. /PK/ = 15cm,
D = 612 

= 24.7386cm  24.7cm to 1d.p /SK/ = /KR/ and P K S = 370. Calculate, correct to three
significant figures:
2008/5a (i) /PS/; (ii) /SK/ and (iii) the area of the shaded portion
A rectangle field is l metre long and b metre wide. Its Solution
perimeter is 280metres. If the length is two and a half (i) Applying trig ratio to PKS
times its breadth, find the value of L and b Sin 37 = PS
Solution 15
L 15 Sin 37 = PS
15  0.6018 = PS
9.027cm = PS
b
PS  9.03cm to sf
(ii) Also by trig ratio in PKS
Perimeter of rectangle = 2(L + b) Cos 37 = SK
2(L + b) = 280 ----- (1) 15
and L = 21/2  b ---- (2) 15 cos 37 = SK
From (1) 15  0.7986 = SK
2(L + b) = 280 11.979cm = SK
L + b = 140 ----- (a) SK  11.98cm to 3sf
From (2) (iii) Area of shaded portion = area of rectangle – area of 
L = 2 1/ 2  b 1
5b = /PS/  /SR/ – /SK/  /PK/ Sin 370
L= --- (c) 2
2 1
Substitute (c) into (a) = 9.03  (2  SK) –  11.98  15  0.6018
2
5b = 9.03  (2  11.98) – 54.07173
+ b = 140
2 = 216.3588 – 54.07173
Multiply through by 2 to clear fraction = 162.28707 cm2
5b + 2b = 280  162cm2 to 3 s.f
334
2005/25 (old) (ii) Area of rectangle lawn = length  breadth
The sides of a rectangle are (x – 1) and (x + 3). If the = (p + 4)  (p – 1)
area of the rectangle is 45, find the positive value of x. = (5.46 + 4)  (5.46 – 1)
A4 B5 C6 D8 = 9.46  4.46
Solution = 42.1916m2
x+3
 42.2m2 to 1 d.p

2006/5a NABTEB
x-1 A diagonal of a rectangle is 15cm. If the length is 3cm
greater than the breadth, find the perimeter of the rectangle.
Solution
Length is bigger, of course x + 3 is bigger
Let the breadth be x
Area of rectangle = length  breadth
then we can sketch the diagram as:
45 = (x + 3) (x – 1)
45 = x2 – x + 3x – 3
45 = x2 + 2x – 3 m
15c x
x + 2x – 48 = 0
2

Factorizing
48, 2 24, 2 2 12, 2  2 2  6 i.e 8  6
x2 + 8x – 6x – 48 = 0 x+3
x(x + 8) – 6(x + 8) = 0 By Pythagoras rule
(x + 8) (x – 6) = 0 152 = (x + 3)2 + x2
x + 8 = 0 or x – 6 = 0 225 = x2 + 6x + 9 + x2
x = – 8 or 6 225 = 2x2 + 6x + 9
We accept x = 6 C. Rearranging
2x2 + 6x – 216 = 0
2014/9a Neco (Dec) Reducing further
A rectangle lawn of length (p + 4) m is (p – 1)m wide. x2 + 3x – 108 = 0
If the diagonal is (p + 5) m, find the Factorizing [108: 2  54; 4  27; 4  3  9 i.e 12  9]
(i) value of P correct to 2 decimal places; x2 + 12x – 9x – 108 = 0
(ii) area of the lawn correct to 1 decimal place x(x + 12) – 9(x + 12) = 0
Solution (x + 12) (x – 9) = 0
x + 12 = 0 or x – 9 = 0
5)m x = – 12 or 9
+ (P - 1)m Accept x = 9
(P
Thus perimeter of the rectangle = 2 (L + b)
= 2 [x + (x + 3)]
(P + 4)m = 2(9 + 9 + 3)
(i) Applying Pythagoras rule to the resulting right – angled  = 2(21)
(p +5)2 = (p + 4)2 + (p – 1)2 = 42cm
p + 10p + 25 = p2 + 8p + 16 + p2 – 2p + 1
2

p2 + 10p + 25 = 2p2 + 6p + 17 2001/21 (Nov)


2p2 – p2 + 6p – 10p + 17 – 25 = 0 The perimeter of a rectangle is 35cm. the ratio of the length
p2 – 4p – 8 = 0 of the rectangle to its width is 3 : 2. Calculate the dimension
The resulting quadratic equation is not factorisable of the rectangle.
Apply the general formula Solution
P = − b  b − 4ac
2
Perimeter of rectangle = 35cm
2a 2 (L + b) = 35 ----- (1)
− (−4)  (−4) 2 − 4  1  (−8)
p = Next, length : breadth
2 1 L : b
4  16 + 32 3 : 2
=
2 L 3
4  6.928 4 + 6.928 4 − 6.928
i.e =
4  48 b 2
= = = or
2 2 2 2 3b
10.928 −2.928 L=
= or 2
2 2 Substitute L into (1)
We accept the positive value since we are dealing with length,
breadth and diagonal
2 (L + b ) = 35 is rearranged as
= 5.464m 35
L+b=
 5.46m to 2dp 2

335
3b 35 2009/21 NABTEB (Nov)
+ b = Find the area of a square field with each diagonal
2 2
Multiply through by 2 to clear fractions 150m long
3b + 2b = 35 A 32,500m B 32,500m2 C 22,500m2
5b = 35 D 22,500m
b = 7cm Solution
Substitute b value into (1) Square has all sides equal
2(L+ 7) = 35
L + 7 = 17.5
L = 17.5 – 7 = 10.5cm 0m x
15

2010/28 Exercise 20.1


What is the length of a rectangular garden whose x
perimeter is 32cm and area 39cm2? By Pythagoras rule : 1502 = x2 + x2
A 25cm B 18cm C 13cm D 9cm 22500 = 2x2
2011/41 Exercise 20.2 11250 = x2
A rectangular garden measures 18.6m by 12.5m Since area of square = x  x
Calculate, correct to three significant figures, = x2
the area of the garden = 11250m2
A 230m2 B 231m2 C 232m2 D 233m2
2007/14
2008/30 Neco (Dec) Exercise 20.3 The area of a square field is 110.25m2. Find the cost of
The area of a rectangle is 30cm2 and its perimeter fencing it round at N 75.00per metre.
is 22cm. state the dimensions of the rectangle A N 1,575.00 B N 3,150.00 C N 4,734.30
A 10cm  3cm B 6cm  5cm C 15cm  2cm D N 8,268.75
D 12cm  2.5cm E 25cm  1.2cm Solution
L
2005/9c (old) Exercise 20.4
EFGH is the rectangular floor of a room in which a
L L
carpet 5m by 3m is laid, leaving a uniform margin of
x metres round it. If the total area of the margin is 20m2,
find the value of x L
Perimeter of a shape = distance round it
1975/8a Exercise 20.4b =L+L+L+L
The area of a rectangular park is 8000m2 . If it had been = 4L
10m shorter and also 10m wider, the area would have First, we find L from the given area
increased by 100m2 . Calculate the original length and Area of square = L2
width of the park 110.25 = L2
110.25 = L
Perimeter = 4  10.5
Square = 42m
Cost of fencing = 42  75
= N 3,150.00 ( B )

Area of square = L x L = L2
Perimeter of square = 4L

2008/15 NABTEB (Nov)


Find the area of a square with one side 4.5cm
A 16.25cm2 B 18.75cm2 C 20.25cm2
2
D 21.50cm
Solution
Area of square = side squared
= 4.52
= 20.25cm2 (C)

336
Circle Solution
Diameter is twice of radius and radius can be gotten from
Area of circle = r2
22
r 77 =  r2
7
77  7 = 22r2
77  7
= r2
Area of circle = r2 22
Perimeter of circle (circumference) = 2r 49
= r2
2
2008/16 NABTEB (Nov)
49
The area of a circle is 81cm2. Find the diameter of the = r
circle. 2
A 7cm B 9cm C 13cm D 18cm 7
cm = r
Solution 2
Note that diameter is radius r times 2
Thus ,diameter = 2r
Area of circle = r2
7
81 = r2 =2
81 = r2 2
81 = r 27 2
Rationalizing the surd =
9cm = r 2
Thus, diameter = 2  r = 7 2 cm ( D )
= 2  9cm
= 18cm (D) 2005/39 Neco
2008/17 NABTEB (Nov) Find the circumference of a circle whose area is 1386cm2
Calculate the circumference of a circle, centre O and (Take  = 22/7)
radius 3.5cm? (Take  = 22/7) A 21cm B 132 cm C 264 cm
A 11cm B 15cm C 19cm D 22cm D 441 cm E 2772cm
Solution Solution
Circle circumference = 2 r Circumference of circle (perimeter) = 2r
22 But r can be gotten from the given area of circle
= 2  3.5
7 Area of circle = r2
= 22cm ( D) 22
1386 =  r2
7
2005/22 (old) 1386  7 = 22r2
If the circumference of a circle is 88cm, find in m2, the 1386  7
area of the circle (Take  = 3.143) = r2
A 0.00616 B 0.0616 C 0.616 D 61.6 22
Solution 441 = r2
Area of circle = r2 441 = r
We can get r from the given circumference r = 21 cm
Circumference of circle = 2r 22
Thus, circumference of circle = 2   21
88 = 2  3.143  r 7
88 = 132cm ( B )
=r 2012/ 42 Neco
2  3.143
13.999cm = r The minute – hand of a wall clock is 9.5m long. What
distance does its tip cover in 72 hours?
Thus, area of circle = r2
A 59.7m B 59.0m C 58.0m
First, we convert r in 13.999cm to metres  13.999/100
2 D 45.0m E 43.0m
 13.999  Solution
= 3.143   
 100  12
2
= 0.0616m B.
2005/20
What is the diameter of a circle of area 77cm2?
(Take  = 22/7)
9 .5
cm

A 2 cm B 31/2 cm C 7cm D 7 2 cm
7

337
The minute - hand makes a complete cycle of 3600 2007/30 Neco
in 60min to cover 1 hour A bicycle wheel of radius 21cm is rolled over a distance of
 72 hours would require 72 complete cycles of 33m. How many revolutions does it make?
the minute - hand (Take  = 22/7)
Distance covered in 72hrs = 72  perimeter of circle A 2.5 B5 C 25 D 50 E 55
= 72  2r Solution
22 One revolution = perimeter of bicycle wheel.
= 72  2   9.5cm = 2r
7
= 4,299.43cm 22
=2  21
Converting to metres = 4,299.43  10–2 m 7
= 42.99m = 132cm
 43.0m (E) A distance of 33m is 33  100cm  3300cm
3300
Thus number of revolutions is = 25 ( C )
2009/17 (Nov) 132
X and Yare circle such that the radius of X is twice the 2006/41 counter example
radius of Y. if the radius of Y is 5cm, find in terms of , The wheel of a tractor has diameter 1.4m. What distance
the area of x does it cover in 100 complete revolutions?
A 6.25 B 10 C 20 D 100 (Take  = 22/7)
Solution A 140m B 220m C 280m D 440m
Circle Circle Solution
X and Y One revolution is the perimeter of the tractor wheel.
2r (radius) r = 2r
(2  5cm) 5cm Here radius is diameter divided by two
22
Area of circle x = r2 = 2  1 .4
7 2
=  (25)2
= 4.4m
=  102
Distance covered in 100 revolutions = 100  4.4m
= 100 (D)
= 440m ( D )
2010/32
2006/33
How many times, correct to the nearest whole number,
In the diagram, the two circles have a common centre O. If
will a man run round a circular track of diameter 100m
the area of the lager circle is 100 and that of the smaller
to cover a distance of 1000m?
A3 B4 C5 D6 circle is 49, find x
Solution
Once round the track = circumference of circular track
= 2r x
22 100 O
= 2 
7 2
= 314.2857
Number of times it will cover 1000m
1000
=
314.2857 A2 B3 C4 D6
= 3.18 Solution
 3 times (A) We are asked to find R – r in
2014/16 (Nov) Larger circle area is  R2 = 100 
A bicycle wheel covers 100cm in one revolution, find in R 2 = 100
terms of , the radius of the wheel. R = 10
Similarly, smaller circle area is r2 = 49
A 50 cm B 100 cm C 50cm D 100cm
  r2 = 49
Solution r=7
One revolution = circumference of bicycle wheel. Thus R – r = 10 – 7
100 = 2r =3 (B)
In terms of , radius r is
2009/46 Neco (Dec)
100
= r What is the area between two concentric circles of
2 diameters 21cm and 18cm respectively
50 A 91.93 cm2 B 254.57 cm2 C 346.50cm2
cm = r (A)
 D 367.70cm 2
E 601.07cm 2

338
Solution D 43.96cm
Area of two concentric circle = R2 – r2 Solution
=  (R2 – r 2) Let R for big circle and r for small circle
Here big circle R is /2cm and small circle r is 18/2cm
21 Difference in circumference = 2R – 2r
22 = 2 (R – r)
= (10.5 2 − 9 2 ) 22
7 = 2 (23 − 16)
22 7
= (110.25 − 81) 22
7 = 2   7 = 44cm (C)
22 7
= (29.25) 2006/17 (Nov) Exercise 20.5
7
Find the area between two concentric circles of radii 9cm
= 91.93cm2 (A)
and 2cm. ( = 22/7)
2014/15 NABTEB A 254. 57cm2 B 242.00cm2 C 12.57cm2
2
Calculate the area of the path round a circular lake of D 9.00cm
diameter 8m, if the path is 1m wide 2009/16 Exercise 20.6
A cm2 B 7cm2 C 8cm2 D 9cm2 The diameter of a bicycle wheel is 42cm. If the wheel makes
Solution 16 complete revolutions, what will be the total distance
covered by the wheel? (Take  = 22/7)
A 672cm B 1056cm C 2112cm D 4224cm
1m
2008/18 NABTEB (Nov) Exercise 20.7
3m

8m
The inner and outer radii of the ring below are 5cm and 8cm
4m
respectively. Calculate the area of the shaded part

5cm
The shaded area is the path 8cm
Area of path round the lake =  R2 – r2
=  (42 – 32)
=  (4 – 3) (4 + 3)
=  (1) (7) A 282/7cm2 B 1224/7cm2 C 3456/7cm2 D 616cm2
= 7 cm2 B.
Triangle
2006/47 (Nov) B
The sum of the areas of two circles each of diameter
64cm is equal to the area of radius rcm. Find the value a
of r. h c
A 38.40cm B 45.25cm C 64.00cm
D 90.50cm
Solution C A
b
Since the two circles have same diameter
Area of 2 circles = 2R2 1
The other circle with radius r has area Area of triangle = bxh;
2
= r2 1
Next, we are given that : 2R2 = r2 Area of triangle = base x height
2
2R2 = r2
1
2 = ab sin C
 64  2
2  = r2
 2  or = ½ ac sin B
2  32 = r
2 2 or = ½ bc sin A
2048 = r2 In triangles; any one of the three sides may by taken as base,
r= 2048 = 45.25cm ( B ) while the height is the perpendicular length from the
opposite vertex to the base

2007/47
e
B as

se

Two circles have radii 16cm and 23cm. What is the h


Ba
h

difference between their circumferences (Take  = 22/7) h


A 244.92cm B 149.92cm C 44.00cm
Base
339
Area of a right – angled triangle We solve for angle C using cosine rule
B
52 = 62 + 72 – 2  6  7 cos C
25 = 36 + 49 – 84 cos C
25 = 85 – 84 cos C
84 cos C = 85 – 25
A C 60
Cos C = = 0.7143
The perpendicular length from A to BC is AB 84
1 C = cos – 1 0.7143
Thus area of triangle ABC = BC  AB C = 44.410
2
Substituting
If all the sides of a triangle are given: use Hero’s 1
formula Area of  =  6  7  sin 44.41
2
Area of triangle = s(s – a) ( s – b ) (s – c) = 14.696cm2
Where s = ½ ( a + b + c)  14.70cm2 to 2d.p

2009/9a Neco (Dec) 2005/2b NABTEB


_____ 
Calculate the smallest angle of triangle ABC in which ABC is a triangle with BC = 8.4cm, A D C = 900 and area
/AB/ = 7cm, /BC/ = 12cm and /AC/ = 9cm. Hence or _____
otherwise, find the area of the triangle (correct to 2 40.16cm2. Find AD
decimal places) A
Solution
A

7cm 9cm

B D C
B C
12cm 8.4cm
The smallest angle is the angle facing the smallest side
7cm i.e angleC Solution
Applying cosine rule 1
Area of triangle = base  height
72 = 122 + 92 – 2  12  9 cos C 2
49 = 144 + 81 – 216 cos C 40.16 =
1
 8.4  AD
49 = 225 – 216 cos C 2
216 cos C = 225 – 49 40.16  2 = 8.4  AD
176 40.16  2
Cos C = = 0.8148 = AD
216 8.4
C = cos – 1 0.8148 = 35.430 9.56cm = AD
1
(ii) Area of  ABC = ab sin c 2011/42 Neco
2
1 Calculate the area of an equilateral triangle of side t cm,
=  12  9  sin 35.43 leaving your answer in surd from
2
= 6  9  0.5797 = 31.30cm2 to 2d.p t2 3 2 t2 3 2
A B t C D t E t2
2011/30 Neco 4 4 2 2
Calculate, correct to 2 decimal places, the area of a Solution
triangle whose sides are 5cm, 6cm and 7cm Equilateral  implies all sides are equal and the three angles
A 12.89cm2 B 14.70cm2 C 22.06cm2 are equal i.e 600
2 2
D 38.24cm E 60.28cm A
Solution
A 60 0
t cm t cm
5cm 7cm
600 60 0
B B C
C t cm
6cm
1 1
Area of  = a b sin C Area of triangle = a  b  sin c
2 2

340
1 By cosine rule
=  t  t  sin 600
2 172 = 102 + 122 – 2  10  12 cos  (Here cos is C)
1 2 3 289 = 100 + 144 – 240 cos 
= t  240 cos  = 244 – 289
2 2
− 45
t2 3 2 2 Cos  =
= 3 cm2 or t cm ( B ) 240
4 4 Cos  = – 0.1875
2010/34  = cos – 1 (– 0.1875) = 100.810
E
1
(ii) Area of  ABC = a b sin C
2
1
=  12  10  sin 100.81
2
8cm = 60  0.9823 = 58.938m2

2007/17
In the diagram, /LN/ = 4cm,  LNM = 900 and
F xcm G (x+2)cm H 2
tan y = , what is the area of the LMN?
In the diagram, /EF / = 8cm, /FG/ = xcm 3
L
/GH/ = (x + 2) cm, EFG = 900. If the area of the
shaded portion is 40cm2, find the area of EFG.
A 128cm2 B 72cm2 C 64cm2 D 32cm2 4cm
Solution
Area of  EFG = area of  EFH – area of  EGH y0
1 M N
Area of triangle = a b sin C
2
A 24cm2 B 12cm2 C 10cm2 D 6cm2
In  EFH Solution
1
Area of triangle =  8  [x + (x + 2)]  sin 900 Area of  LMN =
1
/MN/  /LN/  sin N
2 2
= 4  (x + x + 2)  1 2
= 4 (2x + 2) /MN/ can be gotten from tan y =
3
= (8x + 8) cm2
Opp
Also in  EFG tan y =
Adj
1
Area of triangle =  8  x  sin 900 In the  opposite side is 4
2
= 4x cm 2 But the given value in tan y is 2 which implies 2 2 to give 4
But Area of EFH – EFG = EGH and what we do to the numerator at tan y
(8x + 8) – 4x = 40 We do same to denominator
8x + 8 – 4x = 40 Thus the actual value of 3 is 3  2 = 6 i.e Adj : MN
8x – 4x = 40 – 8 Substituting
4x = 32 cm2 (D) 1
Area of  LMN =  6  4  sin 90
2
2008/9a = 12cm2 ( B )
The triangle ABC has sides AB = 17m, BC = 12m 2012/29 Neco
and AC = 10m. Calculate the: The base of a triangle is 3cm longer than height. If the area
(i) largest angle of the triangle; (ii) area of the triangle is 44cm2, find the length of its base.
Solution A 8cm B 11cm C 12cm D 20cm
Sketch the diagram as: Solution
A Let Height be x then we sketch as:

17m
10m x

B C
12m x +3
1
(i) The largest angle of the triangle is the angle facing Area of triangle = base  height
the longest side i.e C 2

341
1 Solution
44 = (x + 3)  x
2 Area of parallelogram = Base  height
44  2 = x(x + 3) The base here is JK, NK is given, JN can be gotten by
88 = x2 + 3x Pythagoras rule in the right – angled 
x + 3x – 88 = 0
2 132 = 52 + JN2
Factorizing 169 = 25 + JN2
x2 + 11x – 8x – 88 = 0 169 – 25 = JN2
x(x + 11) – 8(x + 11) = 0 144 = JN2
(x – 8) (x + 11) = 0 144 = JN ; JN = 12cm
x – 8 = 0 or x + 11 = 0 Thus, area of parallelogram = (12 + 10)  5
x = 8 or – 11 = 22  5
We accept x = 8cm = 110cm2 D.
Thus base x + 3 = 8 + 3 2014/13 NABTEB
= 11cm ( B ) A 17cm B

2014/12 NABTEB (Nov) Exercise 20.8


Calculate the area of an equilateral triangle of side 8cm 6cm
A 8 3 cm 2
B 16cm 2
C4 3 cm
2
D 16 3 cm
2
30 0
D C

ABCD is a parallelogram of height 6cm, A D C = 300 and
/AB / = 17cm. Calculate the perimeter
A 29cm B 34cm C 46cm D 58cm
Solution
Perimeter = distance round a particular shape
Here AB = DC = 17cm (opposite sides of llgm)
BC = AD
We solve AD in the right – angled  by trig – ratio
6
Sin 30 =
AD
0.5 AD = 6
AD = 12cm
Thus perimeter of llgm = 12 + 17 + 12 + 17
= 58cm ( D )
Parallelogram
2011/47 Neco
A B

a h 6cm

45 0
b D 8cm C
In the diagram above, determine the area of the
Area of parallelogram = b x h parallelogram.
= base x height
A 12 2 cm2 B 24cm2 C 24 2 cm2
= ab sin 
2007/34 Neco D 48cm2 E 48 2 cm2
JKLM is a parallelogram and MN⊥ JK. If /JM/ = Solution
13cm, /MN/ = 5cm and /NK/ = 10cm, find the area of Area of parallelogram = a b sin 
the parallelogram = 6  8  sin 45
M L 1
=68
13cm 2
5cm Rationalizing the surd
2
10cm =68
K 2
J N
= 24 2 cm2
A 50cm2 B 65cm2 C 80cm2 D 110cm2 E 130cm2

342
2008/9 NABTEB (Nov) Properties of Triangles & Parallelograms on the
The perimeter of a parallelogram is 38cm. if one of the same base and between the same parallels.
sides is 12cm, find the other side
A 7cm B 9cm C 13cm D 26cm 1. Triangles on the same base and between the same
Solution parallel are equal in area.
Perimeter = distance round a shape
Note that: opposite sides of parallelogram are equal
12cm

12cm
38 − (12 + 12)
The other side =
2
14
= = 7cm ( A )
2 Area of ∆ACD = Area of ∆BCD
2006/34 Neco (Dec) counter example 2. If a triangle and a parallelogram are on the same
In the diagram below, EAD is a straight line. ABCD is a
base and between same parallels, the area of the
parallelogram with base BC and height 4cm. If /AD/ = 10cm
triangle is equal to half of that of the
and the area of BAE is 30cm2, find the area of trapezium
BCDE parallelogram
E A 10cm D

4cm

B C
2 2
A 100cm B 50cm C 20cm2 D 15cm2 E 10cm2
Solution
Here area of trapezium =
area of BAE + area of llgm ABCD + area of right– angled  Area of ∆ECD = ½ area of parallelogram ABCD
Area of llgm ABCD = base  height
3. Parallelogram on the same base and between
= 10  4
the same parallels are equal in area.
= 40cm2
Note that  between same base and same parallel have equal areas.
P Q
Area of BAE = 30cm2 same for right– angled 
Thus, area of trapezium = 30 + 40 + 30 M1
M2 U1 U2
= 100cm2 ( A )
2013/29 Neco Exercise 20.9
Calculate the area of the figure below
12cm

9cm
1500
Given: Parallelograms ABCD, PQCD on the same base DC
and between the same parallels AQ and DC.
A 1.5cm2 B 10.5cm2 C 27cm2
D 54cm2 E 104.3cm2 To prove: ABCD = PQCD
Proof:
2009/ 30 Neco(Dec) Exercise 20.10 In ∆ s APD and BQC
The area of parallelogram ABCD is 425cm2, M1 = M2 (Corr. s)
/AB / = 12cm and / AD / = 7cm. Find the perpendicular U1 = U2 (Corr.  s)
height of the parallelogram to 2 decimal place. AD = BC (Opp. Sides of llgm)
12cm
A B
Therefore ∆ APD = ∆ BQC (AAS)
7cm
Thus, Quad. AQCD - ∆APD is equal quad.
AQCD - ∆BQC is equal quad.
Hence PQCD = ABCD
D C

A 17.74cm B 31.07cm C 35.42cm


D 62.14cm E 84.00cm
343
2006/11 Trapezium
In the diagram, PQUV, PQTU, QRTU and QRST are
parallelograms. /UV/ = 4.8cm and the perpendicular a
distance between PR and VS is 5cm. Calculate the area
of quadrilateral PRSV.
P Q R h

5cm b
1
Area of trapezium = (a+b)xh
2
V 4.8cm U T S 1
2 2 2
= (sum of two parallel sides) x height
A 96cm B 72cm C 60cm D 24cm2 2
Solution
Area of quadrilateral PRSV 2013/30 Neco
= area of llgm PQUV + area of llgm QRUT + area of RTS Calculate the area of a trapezium ABCD with
Recall that parallelogram between same base and between /AB/ = 20cm, /DC/ = 45cm and height = 8cm
same parallel are equal in area and half area of llgm is equal A 245cm2 B 260cm2 C 287cm2
to area of triangle between same base and same parallel. D 344cm 2
E 480cm 2
Area of llgm = base  height Solution
= 4.8  5 = 24cm2 1
1 Area of trapezium = (20 + 45)  8
Area of  = area of llgm 2
2 1
1 =  65  8
=  24 = 12cm2 2
2 = 260cm2 (B)
Thus, area of quadrilateral PRSV = 24 + 24 + 12
= 60cm2 ( C ) 2005/3a NABTEB (Nov)
____
2006/35 Neco (Dec) ABCD is a trapezium in which AB//CD, AB = 20cm,
____ ____ 
T 13.5cm U Z V
DC = 10cm, AD = 16cm and B A D = 300, calculate the
area of the trapezium
A 20cm B
h

Y X 16cm
In the diagram above, TUXY and ZVXY are
parallelograms. If /TU/ = 13.5cm and the area of D 10cm C
parallelogram ZVXY is 67.5cm2, Calculate its height. Solution
A 54cm B 27cm C 8.5cm D 5cm E 2.7cm
Solution A E 20cm B
Parallelogram between same base and same parallel are equal in area 300
Thus ZVXY = TUXY
and area of llgm TUXY = base  height
16cm
Note YX is same as TU (opp side of llgm)
67.5 = 13.5  h
67.5 D 10cm C
= h
13.5
1
5cm = h ( D ) Area of trapezium = (10 + 20)  DE
2
2016/5a Exercise 20.10b We solve for DE in ADE using trig ratio
T 8cm S DE
Sin 300 =
16
16 sin 30 = DE
6cm 16  0.5 = DE
8cm = DE
P Q M R 1
Thus area of trapezium = (10 + 20)  8
In the diagram, PQST is a parallelogram, PR is a 2
straight line, TS = 8cm, SM = 6cm and area of = 30  4
triangle PSR = 36cm2. Find the value of QR = 120cm2
344
2009/49 NABTEB (Nov) Solution
The trapezium below has an area of 456cm2? A 10cm
B
Calculate the distance between its parallel sides
17cm
12cm

60 0
hcm E D
C

It is common knowledge that AB = CD = 10cm;


31cm length of rectangle ABCD
A 29cm B 27cm C 19cm D 17cm Applying Pythagoras rule to the resulting right – angled  ACE
AC
Solution Sin 600 =
12
1
Area of trapezium = (17 + 31)  h 12 sin 60 = AC
2
3
1 12 cm = AC Thus AC = 6 3 cm
456 = (48)  h 2
2 EC
456 = 24h Also cos 600 =
12
456
= h thus, 19cm = h (C) 12 cos600 = EC
24
1
12  = EC ; EC = 6cm
2005/38 Neco 2
A 38m B 1
Thus, area of trapezium = (AB + ED)  AC
2
25m 1
= (10 + 16)  6 3
2
1
D 58m C =  26  6 3 = 78 3 cm2 ( B )
_____ 2
In the trapezium above, if AB = 38m 2009/17 and 18 Exercise 20.11
_____ _____ Q 6cm R
DC = 58m, BC = 25m and  BAD = ADC = 900.
Find the length AD
A 38m B 33m C 25m D 20m E 15m 5cm
Solution
38m 42 0
A B P S
10cm

25m
PQRS is a trapezium. QR//PS, /PQ/ = 5cm,
/QR/ = 6cm, /PS/ = 10cm and angle QPS = 420
Use the information to answer questions 17 and 18
D 38m E 20m C 17. Calculate the perpendicular distance between
58m the parallel sides
AD = BE A 3.35cm B 3.72cm C 4.50cm D 4.62cm
Appling Pythagoras rule to the resulting right – angled 
252 = 202 + BE2 18. Calculate, correct to the nearest cm2,
625 = 400 + BE2 the area of the trapezium
625 – 400 = BE2 A 27cm2 B 30cm2 C 36cm2 D 37cm2
225 = BE2
1976/4 Exercise 20.11b
225 = BE thus,15m = BE (E) In the accompanying figure which is not drawn to scale,
2006/30 Neco ABCD is a piece of land in the form of trapezium with AB
parallel to DC, AB = 400cm, AD = 500cm, ADC = 750 ,
10cm
BCD = 900. Calculate the distance between the parallel
sides and hence calculate the area of the piece of land, giving
12cm your answer in standard notation
A 400cm
B
60 0

The figure above is a trapezium. Calculate its area 500cm


A 156 3 cm2 B 78 3 cm2 C 120cm2 75 0
D 39 3 cm 2
E 60cm 2
D E C
345
Rhombus (with given diagonal) 2008/14 NABTEB (Nov)
A The side of a rhombus is 10cm. if one of the diagonal is
16cm, find the area
A 48cm2 B 96cm2 C 160cm2 D 180cm2
x 1 Solution
1
Area of rhombus = x y(where x and y are diagonals)
B D 2
y1 y2 One diagonal is given, we sketch as:
A
x2
10cm
8cm

C D B
O
8cm
Diagonal BD = y Where y = y1 + y2
and diagonal AC = x Where x = x1 + x2
Area of rhombus = ½ x y or = ½ x1 y1 x 4 C
Where x and y are the two stated diagonals The second diagonal is DB but DB = DO + OB
and OB can be gotten by Pythagoras rule in AOB
2010/10 NABTEB (Nov) 102 = 82 + OB2
Calculate the area of a rhombus whose diagonals are 100 = 64 + OB2
100 – 64 = OB2
10cm and 6cm long
36 = OB2
A 32cm2 B 30cm2 C 28cm2 D 20cm2 OB = 6cm
Solution  DB = 2  6cm
1 = 12cm
Area of rhombus = xy (where x and y are diagonals)
2 1
Thus area of rhombus =  16  12 = 96cm2
1 2
=  10  6
2 2006/8a Neco
= 30cm2 (B) Calculate the angles of a rhombus whose diagonals are of lengths
10cm and 16cm
2005/5b Neco Solution
A Rhombus of side 13cm has a diagonal of 18cm. Find Pictorially, we have
the area of the rhombus correct to 2 decimal places O
Solution
8
1
Area of rhombus = x y (where x and y are diagonals)
2 5 5
one diagonal is given, we find the other one from the 8
given data :

9 13
One set of the required angles is 2
But by trig – ratio
x 5
tan  =
9 8
tan  = 0.625
 = tan – 1 0.625
= 320 Thus 2 = 2  320 = 640
By Pythagoras rule : 132 = 92 + x2
Also another set of the required angles is 2
169 = 81 + x2 In the right – angled triangle :
169 – 81 = x2  + 90 +  = 1800 (sum of s in )
88 = x2 32 + 90 +  = 1800
88 = x  = 180 – 122 = 580 Thus 2 = 2 580 = 1160
The second diagonal is 2  9.381 = 18.762cm Thus the angles of the rhombus are 640, 640, 1160 and 1160
1
Thus, area of rhombus =  18  18.762 2010/32 NABTEB (Nov)
2
2 The area of a rhombus is 24cm2 and one of its diagonals is
= 168.858cm
8cm. Find the length of a side
 168.86cm2 to 2dp A 3cm B 4cm C 5cm D 6cm.

346
Solution 2012/49 Exercise 20.12
Pictorially, we have A side and a diagonal of a rhombus are 10cm and 12cm
respectively. Find its area
A 20cm2 B 24cm2 C 48cm2 D 96cm2
4 = Side? 2006/5a (Nov) Exercise 20.13
_____ _____
4 In a quadrilateral ABCD, the diagonals AC and BD bisect
k k
each at right angles, /AC/ = 16cm and /BD/ = 30cm
4
k (i) What type of quadrilateral is ABCD?
(ii) Find /AB/
(iii) Calculate the area of the quadrilateral
For us to know the rhombus side;
We first find K i.e half the 2nd diagonal of rhombus 2015/36 Exercise 20.14
1 The area of a rhombus is 110cm2. If the diagonal are 20cm
Area of rhombus = xy and (2x + 1)cm long , find the value of x.
2
A 5.0 B 4.0 C 3.0 D 2.5
1
24 = 8y
2
2  24
=y Length of an arc
8 For any circle, with arc PRQ (minor) or arc PSQ (major)
6cm = y R
6
Thus, K =
2
= 3cm
P Q P Q
By Pythagoras rule
Rhombus side = 4 2 + 32
= 16 + 9 Majorarc
= 25 i.e 5cm C. S
2008/19
S
Subtending an angle  at the center of the circle,
P

length of the arc =  2r
360

2007/42 Neco
An arc of a circle of radius 8cm subtends an angle of 850 at
Q R the centre of the circle. Find the length of the major arc
(Take  = 22/7)
In the diagram, PQRS is a rhombus. /PR/ = 10cm and A 11.87cm B 38.41cm C 272.45cm
/QS/ = 24cm. calculate the perimeter of the rhombus D 289.62cm E 474.87cm
A 34cm B 52cm C 56cm D96cm Solution
Solution 
P S Length of arc =  2r
360
5c m Here the minor arc subtends 850
m 12c But the major arc subtend 360 – 85 = 2750
275 22
m 5c Thus length of major arc =  2 8
12c m 360 7
= 38.41cm B
2010/33 Neco
Q R What is the length of an arc which subtends an angle of 600
Perimeter of rhombus = PS + SR + RQ + PQ at the circumference of a circle of radius 21cm?
All sides of rhombus are equal (Take  = 22/7)
By Pythagoras rule: SR2 = 122 + 52 A 141cm B 132cm C 44cm D 22cm E 11cm
SR2 = 144 + 25 Solution
SR2 = 169 
Length of arc =  2r
SR = 169 360
= 13cm 60 22
=  2   21
Thus, perimeter of rhombus = 13 + 13 + 13 + 13 360 7
= 52cm ( B ) = 22cm ( D )
347
2005/23 (Nov) 2008/35
The pendulum of a clock is 5cm long and swings An arc of a circle, radius 14cm, is 18.33cm long. Calculate,
through an arc of 8cm. Through what angle, in degrees, correct to the nearest degree the angle which the arc subtends
does the pendulum swing? (Take  = 22/7 ) at the centre of the circle (Take  = 22/7)
A 360 B 460 C 720 D 920 A 110 B 200 C 220 D 750
Solution Solution
The simple sketch is 
Length of arc =  2r
360
O
 22
5cm 18.33 =  2  14
360 7
18.33  360  7
=
2  22  14
74.9860 = 
  750 to the nearest degree

Length of arc =  2r 2006/40
360
The diagram shows an arc MN of a circle centre O, with
 22 radius 10cm. If MON = 720, calculate the length of the arc,
8=  2 5
360 7 correct to three significant figures
220 (Take  = 22/7)
8=
2520
8  2520
= M N
220
91.620 = 
  920 (D)

cm
720

10
2011/24 Counter example
A circle is divided into two sectors in the ratio 3: 7.
O
If the radius of the circle is 7cm, calculate the length of A 6.28cm B 12.6cm C 22.6cm D 62.8cm
the minor arc of the circle Solution
A 18.85cm B 13.20cm C 12.30cm D 11.30cm

Solution Length of arc =  2r
 360
Length of arc =  2r 72 22
360 =  2   10
 360 7
Here is replaced by ratio 3:7 their sum is 10 = 12.57cm
360  12.6cm to 3s.f
3
and for the minor arc is
360 10 2008/5 NABTEB (Nov)
3 22 An arc of a circle of radius 13.5cm subtends an angle of 1050
Length of minor arc =  2 7 at the centre. Calculate, correct to three significant figures, the:
10 7
= 13.2cm B.

2009/16 (Nov)
O
In a circle, an arc of length 110cm subtends an angle of
2100 at the centre. Find the radius of the circle 105 0
13

(Take  = 22/7)
.5c
m

A 75cm B 60cm C 55cm D 30cm


A
Solution B

Length of arc =  2r (i) chord length AB
360 (ii) arc length AB (Take  = 3.142)
210 22
110 =  2  r Solution
360 7 (i) Length of chord = 2 r sin 
110  360  7 2
=r 105
210  2  22 = 2  13.5  sin
30cm = r (D) 2

348
= 27  sin 52.5 A 4.89cm B 9.76cm C 9.78cm
= 27  0.7934 D 19.54cm E 78.22cm
= 21.42cm Solution
 21.4cm to 3s.f 
Length of arc =  2r
 360
(ii) Length of arc =  2r
360 Major arc here  is 360 – 40 i.e 3200
105 22 320 22
=  2   13.5 =  2  14
360 7 360 7
= 24.75cm  24.8cm to 3s.f = 78.22cm
2011/31 Neco
C 41. What is the area of the minor sector XOY?
(Correct to 2d.p)
A 547.56cm2 B 273.77cm2 C 78.22cm2
2 2
D 68.44cm E 58.66cm
O Solution

500 Area of sector =  r 2
7c

360
m

A
Minor sector has  as 400
B 40 22
=   14 2
Calculate the length of the major arc ACB correct to 2 360 7
decimal places = 68.44cm2
A 132.61cm B 37.89cm C 21.39cm
D 6.11cm E 5.42cm 2012/16
Solution The lengths of the minor and major arcs of a circle are 54cm
 and 126cm respectively. Calculate the angle of the major
Length of arc =  2r
360 sector
Here the minor arc is 500 A 3060 B 2520 C 2460 D 2340
But the major arc  is 360 – 50 i.e 3100 
Length of minor arc =  2r
310 22 360
Thus, length of major arc =  2 7
360 7 360 − 
Length of major arc =  2r
= 37.888cm  37.89cm to 2d.p 360
2008/34 Neco (Dec)  360 − 
54 =  2r and 126 =  2r
Calculate, in terms of , the length of an arc of a circle 360 360
of radius 12cm which subtends angle 2400 at the centre 54  360 =  (2r) and 126 × 360 = (360 – ) 2r
of the circle. 19440 =  (2r) ---(1) and 45360 = (360 – ) 2r ---(2)
A 16cm B 24cm C 32cm D 48cm E 69cm Equation (2) ÷ (1)
Solution 45360 (360 −  )2r
 =
Length of arc =  2r 19440  (2r )
360
360 − 
240 2.333 =
=  2    12 = 16cm ( A ) 
360
2.333 = 360 – 
2008/40 and 41 Neco (Dec) 2.333 +  = 360 – 
In the diagram below, O is the centre of the circle of 3.333 = 360
radius 14cm, XOY = 400. Use this information to 360
answer questions 40 and 41 (Take  = 22/7)  =
3.333
Z
 = 1080
Angle of the major sec tor = 360 – 
= 360 – 108
O = 2520 ( B)
c m 400
14 2008/12 NABTEB (Nov) Exercise 20.15
Find the length of arc of a circle of radius 15.4cm which
X Y subtends an angle of 600 at the centre of the circle
(Take  = 22/7)
40. What is the length of the major arc XZY? A 16.13cm B 15.83cm C 8.26cm D 7.67cm
(Correct to 2 d.p)
349
2010/47 NABTEB (Nov) Exercise 20.16 2012/17
O A sector of a circle which subtends 1720 and the centre of the
1200
circle has a perimeter of 600cm. find, correct to the nearest
cm, the radius of the circle. (Take  = 22/7)
21cm 21cm
A 120cm B 116cm C 107cm D 100cm
Solution
Perimeter of a sector = 2r + Length of arc
P Q
172 22
600 = 2r +  2  r
360 7
7568r
600 = 2r +
2520
Find the length of arc /PQ/ Applying LCM principle to the RHS
A 44cm B 21cm C 22cm D 33cm
5040r + 7568r
600 =
2009/34 Exercise 20.17 2520
An arc of a circle subtends an angle of 600 at the centre. 12608r
If the radius of the circle is 3cm, find, in terms of , the 600 =
2520
length of the arc Cross multiply to clear fraction
A cm B 2cm C 3cm D 6cm 1512000 = 12608 r
2009/40 (Nov) Exercise 20.18 1512000 12608r
=
A chord EF subtends an angle 600 at the centre of a 12608 12608
circle. If the radius of the circle is 7cm, calculate the 119.92cm = r
length of the major arc formed by the chord r  120cm to nearest cm
A 7.3cm B 14.7cm C 29.3cm D 36.7cm
2009/45 (Nov) Exercise 20.19 2015/14
An arc of a circle of radius 3.5cm is 6.6cm long. What A sector of a circle with radius 6cm subtends an angle of 600
angle does the arc subtend at the centre of the circle? at the centre. Calculate its perimeter in terms of .
(Take  = 22/7) A 2( + 6) cm B 2( + 3) cm C 2( + 2) cm
A 540 B 720 C 900 D 1080 D ( + 12) cm
Solution
Perimeter of a sector = 2r + Length of arc
Perimeter of a Sector
60
= 26+  2  6
360
MajorSector = 12 + 2
O
= 2(6 + ) cm A.
r r
Minor 2005/42 (old)
A B Sector OX and OY are radii of a circle centre O. if /OX/ is 4cm
and XOY is 600, calculate correct to the nearest whole
For any circle with center O and radius r from which a number, the perimeter of sector XOY of the circle
sector AOB is formed. The perimeter of the sector is the (Take  = 22/7)
sum of the two radii and the length of the arc AB since A 2cm B 4cm C 8cm D 12cm
perimeter is the distance round a given shape. Solution
Perimeter of a sector = 2r + L of arc
Where L of arc =  x 2r
360
O
2007/35 Neco m 600
The angle of a sector of a circle of radius 5.5cm is 600. 4c
What is the perimeter of the sector? (Take  = 22/7)
A 5.76cm B 11.26cm C 16.76cm D 30.25cm X Y
E 36.01cm
Solution Perimeter of a sector = 2r + Length of arc
Perimeter of a sector = 2r + Length of arc 60 22
=24+  2  4
60 22 360 7
= 2  5.5 +  2   5.5
360 7 = 8 + 4.19cm
= 11cm + 5.76cm = 12.19cm
= 16.76cm ( C )  12cm to nearest whole number

350
2009/38 Neco (Dec) 2005/30 Exercise 20.23
A sector is bounded by radii 6cm long and an arc An arc of a circle of radius 14cm subtends angle 3000 at the
subtending an angle of 500. Find, correct to 2 decimal centre. Find the perimeter of the sector formed by the arc.
places, its perimeter. (Take  = 22/7)
A 5.24cm B 11.24cm C 15.24cm A 14.67cm B 42.67cm C 101.33cm D 513.33cm
D 17.24cm E 27.24cm
Solution 2006/3 Neco Exercise 20.24
Perimeter of a sector = 2r + Length of arc Find the perimeter of a sector of a circle of radius 14cm
50 22 which subtends an angle of 1350 at the centre of the circle.
=26 +  2 6 (Take  = 22/7)
360 7
= 12cm + 5.238cm A 33cm B 47cm C 61cm D 75cm E 91cm
= 17.238cm
 17.24cm to 2d.p ( D)

2014/44 Neco Area of a sector


Find the radius of a sector whose perimeter is 32cm and
its arc length is 25cm. MajorSector
A 14.0cm B 7.0cm C 5.2cm D 4.0cm E 3.5cm O
Solution r r
Perimeter of a sector = 2r + Length of arc Minor
Substituting for the given values
32 = 2r + 25 A B Sector

32 – 25 = 2r
7 = 2r The area of a sector AOB of a circle radius r of center O is
7 given as
= r thus, r = 3.5cm (E)
2 Area of sector =  x area of circle
2014/39 360
A chord subtends an angle of 1200 at the centre of a 
=  r 2
circle of radius 3.5cm. Find the perimeter of the minor 360
sector containing the chord (Take  = 22/7)
A 14 1/3cm B 12 5/6cm C 8 1/7cm D 7 1/3cm 2006/5a NABTEB
Solution Calculate the area of the major sector of a circle which
Perimeter of a sector = 2r + Length of arc subtends an angle of 1300 at the centre and having radius
120 22 14cm (Take  = 3.14)
= 2  3.5+  2   3.5
360 7 Solution
1 22 7 
= 7 +  2  Area of sector =  r 2
3 7 2 360
22 130
=7+ =  3.14  14 2
3 360
43 = 222.24cm2
= = 14 1/3cm ( A )
3 2008/34 Neco
2011/32 Neco Exercise 20.20 A sector of angle 1200 is cut out from circle of radius
Calculate the perimeter of a sector of a circle of radius 13.5cm. What area of the circle is remaining?
15cm, where the angle of the sector is 600 (Take  = 22/7)
(Take  = 22/7) A 14.1cm2 B 95.5cm2 C 190.9cm2
2 2
A 90.24cm B 45.71cm C 22.82cm D 381.9cm E 763.7cm
D 15.71cm E 14.29cm Solution

2013/5a Neco Exercise 20.21 Area of sector =  r 2
360
The angle of a sector of a circle of a radius 9cm is 1200.
Calculate the perimeter of the sector, correct to three Here  is 360 – 120 i.e 240
significant figures. (Take  = 22/7) 240 22
=   13.5 2
360 7
2012/32 Neco Exercise 20.22 = 381.857cm2
A sector of a circle of radius 14cm subtends an angle of  381.9cm2 (D)
1350 at the centre of the circle. What is the perimeter of
the sector? (Take  = 22/7)
A 47cm B 51cm C 61cm D 88cm E 231cm
351
2005/45 22  2520
XOY is a sector of a circle centre O of radius 3.5cm =
528
which subtends an angle of 1440 at centre. Calculate, in
1050 =  i.e x0
terms of , the area of the sector

A 1.4cm2 B 2.8cm2 C 4.9cm2 D 9.8cm2 (ii) Area of sector =  r 2
Solution 360
 105 22
Area of sector =  r 2 =   12 2 = 132cm2
360 360 7
144
=    3.5 2 2008/17 Exercise 20.25
360 POR is a sector of a circle centre O, radius 4cm.
= 4.9cm2 
If P O R = 300, find, correct to 3 significant figures, the area
2009/4c NABTEB (Nov) of sector POR (Take  = 22/7)
In the diagram below, O is the centre of concentric A 4.19cm2 B 8.38cm2 C 10.5cm2 D 20.9cm2
circle of radii 13cm and 10cm respectively. Find the
area of the shaded portion in the sector with an angle of 2014/26 NABTEB (Nov) Exercise 20.26
1200 at the centre The angle of a sector of a circle of diameter 8cm is 1350, find
the area of the sector (Take  = 22/7)
A 93/7cm2 B 12 4/7cm2 C 186/7cm2 D 251/7cm2

O
Length of a chord
1200

O
A 1/3 cm2 B cm2 C 3cm2 D 23cm2
Solution r r
Area of shaded portion
= area of big circle sector minus area of small circle
sector A B
120 120
=    132 −    10 2 Length of chord AB = 2r sin 
360 360 2
120 On the other hand if  is not given but a shape like the one
=  (132 − 10 2 )
360 below is given; we apply Pythagoras theorem to solve for the
1 length of chord.
=  (13 − 10) (13 + 10)
3
1 O
=  (3) (23)
3
= 23cm2 D.
A C B
2010/3 Neco
An arc of a circle of radius 12cm subtends an angle of 2006/29 Neco (Dec)
x0 at the centre. If the perimeter of the sector formed by A chord of length 30cm is 8cm away from the centre of the
the arc is 46cm, calculate the: circle. What is the radius of the circle?
(i) value of x A 17cm B 18cm C 19cm D 20cm E 21cm
(ii) area of the sector (Take  = 22/7) Solution
Solution The sketch is as:
Perimeter of a sector = 2r + Length of arc
 22
46 = 2  12 +  2  12
360 7
Here  is x 0 O
528
46 = 24 +
8cm

2520
528 A 15cm C 15cm B
46 – 24 =
2520 AB = 30 cm i.e AC + AB
352
By Pythagoras rule in OAC 2007/29 Neco
OA2 = AC2 + OC2 The length of a chord is 24cm. if the distance of the chord is
OA2 = 152 + 82 8cm from the centre of the circle, find the radius of the circle
OA2 = 225 + 64 (correct to 2 d.p)
OA2 = 289 A 12.65cm B 14.42cm C 20.50cm
0A = 289 = 17cm ( A ) E 22.63cm E 25.30cm
Solution
The sketch is as:
2006/40 Neco (Dec)
The distance of a chord of a circle of radius 20cm from
the centre of the circle is 16cm.Calculate the length of
the chord
A 36cm B 24cm C 12cm D 6cm E 4cm O
Solution
The sketch is as:

8cm
A 12cm C 12cm B

AB = 24cm i.e AC + CB
O By Pythagoras rule in OAC
OA2 = AC2 + OC2
cm

20
16cm

OA2 = 122 + 82
cm
20

OA2 =144 + 64
A C B OA2 = 208
0A = 208 = 14.422cm  14.42 cm to 2d.p
Required length is AB
But AB = AC + BC and AC = BC 2014/1 Neco (Dec)
In OAC: By Pythagoras rule An arc PQ of a circle of radius 14cm subtends an angle of
202 = 162 + AC2 740 at the centre of the circle. Calculate, correct to 2 decimal
AC2 = 202 – 162 places, the:
AC2 = 400 – 256 (a) length of the chord PQ;
AC2 = 144 (b) distance of the chord PQ from the centre of the circle;
AC = 144 = 12cm (c) Area of  POQ
Thus AB = AC + BC 12 + 12 = 24cm ( B ) Solution
Sketch of the circle with centre O
2011/43 Neco

cm 74 0
14

O 14
cm

P
cm

Q
10


A X 8cm B (a) Length of the chord = 2r sin
2
74
O is the centre of the above circle and AXB is a chord. = 2  14  sin
Find /OX / 2
A 9cm B 8cm C 7cm D 6cm E 4cm = 28  sin 37
Solution = 28  0.6018 = 16.8504cm  16.85cm to 2d.p
By Pythagoras rule : OA2 = AX2 + OX2 (b)
AX = BX (The line joining the centre of a circle to the mid –
point of a chord is perpendicular to the chord)
102 = 82 + OX2
10 – 82 = OX2
2 O
(10 – 8) (10 + 8) = OX2
2(18) = OX2 cm
14
36 = OX2
36 = OX P 16 .85cm
R
OX = 6 cm ( D )
2 Q
353
The required distance is OR 
=  2 r + 27.5cm
By Pythagoras rule in OPR 360
142 = OR2 + PR2
132.8 22
142 = OR2 + 8.4252 =  2   15 + 27.5
196 = OR2 + 70.981 360 7
196 – 70.981 = OR2 = 34.78 + 27.5
125.019 = OR2 = 62.28cm
125.019 = OR  62.3cm to 3s.f
11.181cm = OR 2008/30 Neco
OR  11.18cm to 2d.p A chord XY of a circle with centre O and radius 5.32cm has
XOY = 1400. What is the length of the chord to the nearest
(c) Area of POQ = 1/2 r2 sin centimeter?
= 1/2  142  sin 74 A 10cm B 6cm C 5cm D 3cm E 1cm
= 1/2  142  0.9613 Solution
= 94.2074cm2 Length of chord = 2r sin 
 94.21cm2 to 2d.p 2
140
= 2  5.32  sin
2014/6 Neco Counter example 2
In the diagram below, O is the centre of the circle, PQ is = 10.64  sin70
a chord where /OQ/ = 15cm and /OR/ = 6cm = 9.998cm
P R  10cm to nearest cm ( A )
Q
6cm 2013/3a Neco Counter example
15cm
A chord PQ of length 24cm is drawn in a circle of radius
O
37cm. If point R is the centre of the circle, find the area of
PRQ
Solution
Calculate the: We sketch as:
(i) length of the chord, correct to 1 decimal place
(ii) angle subtended by POQ, correct to the nearest
degree;
(iii) perimeter of the minor segment, correct to R
3 significant figures (Take  = 22/7)
cm 74 0
37

Solution 37
cm

Length of chord is 2  RQ
By Pythagoras rule in ROQ P
24cm Q
152 = 62 + RQ2
225 – 36 = RQ2
Since all the sides of PRQ is given, we apply
189 = RQ2
Area of  PRQ = S (s − a)(s − b)(s − c)
189 = RQ
1
13.75cm = RQ Where S = (37 + 37 + 24)
2
Thus length of chord = 2  13.75cm 1
= 27.5cm also approximated to 3s.f = (98) = 49
2
(ii) Angle POQ is 2  ROQ And a is 37, b is 37 and c is 24
In ROQ, by trig ration, let  ROQ be  Area  PRQ = 49(49 − 37)(49 − 37)(49 − 24)
6
Cos  = = 49(12)(12)(25)
15
Cos  = 0.4 = 7 2  12 2  5 2
 = Cos – 1 0.4 = 7  12  5
= 66.4220 = 420cm2
Thus, POQ = 2  66.4220
= 132.8440
 1330 to the nearest degree 2014/8b NABTEB (Nov)
(iii)Perimeter of minor segment There are two chords /MN/ and /PQ/ in a circle
= length of arc + length of chord /MN/ = 10cm and /PQ/ = 8cm and the radius of the circle is
12cm. What is the distance of each chord from the centre of
the circle?
354
Solution 2006/11a NABTEB Exercise 20.30
We sketch the diagram as: In a circle of radius 6cm, calculate the distance from the
MN is 10cm  MK is 5cm centre to a chord which is 8.5cm long

5cm K 2009/23 NABTEB (Nov) Exercise 20.31


M N
In the diagram below, calculate the value of /OM /
12
cm
O
cm
12 O
P 4cm L Q cm
10

PQ is 8cm  PL is 4cm 6cm 6cm


A M B
We are asked to find OL in OPL and OK in OMK,
readers to complete it. A 8cm B 15cm C 20cm D 25cm
2006/48 (Nov) Exercise 20.32
2014/23 NABTEB Exercise 20.27 A chord subtends an angle of 1200 at the centre of a circle of
radius 10cm. Find the length of the chord.
A 15.00cm B 17.32cm C 18.42cm D 19.32cm
2006/15 (Nov) Exercise 20.33
O
A Chord of length 8.50cm is drawn in a circle of radius
8.50cm. Calculate, correct to two decimal places, the
14cm

distance of the chord from the centre of the circle.


A 7.36cm B 7.63cm C 8.00cm D 9.00cm
A M B
42cm 2014/24 Exercise 20.34
A chord, 7cm long, is drawn in a circle with radius 3.7cm.
In the figure, O is the centre of the circle, /AB/ = 42cm Calculate the distance of the chord from the centre of the

and /OM/ = 14cm. if O M A = 900, Calculate /OA/ circle
A 24cm B 25cm C 63cm D 73cm A 0.7cm B 1.2cm C 2.0cm D 2.5cm
2007/15 Exercise 20.35
2014/46 NABTEB Exercise 20.28 A chord of length 6cm is drawn in a circle of radius 5cm.
Find the distance of the chord from the centre of the circle.
A 2.5cm B 3.0cm C 3.5cm D 4.0cm
O

1200
7c

Perimeter of Segment of a circle


m

A
B

Calculate the length of chord /AB/ O Majorsegm


ent
A 2 3 cm B 3 3 cm C 5 3 cm D 7 3 cm r r
Min
orsegm
ent
2005/5 NABTEB Exercise 20.29
A B
The centre of the circle ABC is O. If its radius is 8cm
C
and ACB = 400, calculate the length of the
(a) Chord AB The perimeter of a segment
(b) Perpendicular OM
C = length of arc ACB + length of chord AB
 
= 2r + 2r sin
400 360 2

O 2008 /37
XY is a chord of circle centre O and radius 7cm. The chord
8c

XY is 8cm long subtends an angle of 1200 at the centre of


m

the circle. Calculate the perimeter of the minor segment


A M B
(Take  = 22/7)
A 14.67cm B 22.67cm C 29.33cm D 37.33cm
355
Solution Area of segment of a circle

O O
1200 r r

7c
m
X 8cm Y A B
Area of segment = area of sector – area of triangle
Perimeter of a segment  1
= length of arc XY + length of chord XY =   r 2 − r 2 sin 
360 2
Of course minor segment  is 1200 2
r   
 =  − sin  
= 2r + 8 2 180 
360 2005/16 Neco
120 22
= 2 7 + 8
360 7

m
= 14.667 + 8

12
= 22.67cm (B)
135 0
O
2014/43 Neco
Calculate the perimeter of a minor segment of a circle The figure above shows a sector of a circle radius 12m
of radius 14cm. subtending an angle 1200 at the centre. which subtends angle 1350 at the centre O. Calculate the area
( = 22/7) of the shaded segment
A 23.33cm B 24.25cm C 38.92cm
A (84 – 48 2 ) m2 B (66 – 42 2 ) m2
D 53.58 E 126.92cm
Solution C (54 – 36 2 ) m2 D (42 – 32 2 ) m2 E (36 – 28 2 )
Perimeter of segment = length of arc + length of chord m2
  Solution
= 2r + 2r sin Area of segment = area of sector – area of triangle
360 2
 1
120 22 120 =   r 2 − r 2 sin 
=  2   14 + 2  14  sin 360 2
360 7 2 135 1
= 29.333 + 24.249 =    122 −  122  sin 135
= 53.582cm 360 2
 53.58cm ( D ) But sin 135 is same as sin (180 – 135) i.e sin 450
135 1
=    12 12 − 12 12  sin 45
360 2
1
= 54 – 6  12 
2
2
Rationalizing 1 = 54 – 6  12 
2 2
= (54 – 36 2 ) m2 C.
2005/3a Neco
A segment of a circle of diameter 24cm subtends angle of
900 at the centre. Calculate the area of the segment to the
nearest cm2 (Take  = 22/7)
Solution
Area of segment = area of sector – area of triangle
 1
=   r 2 − r 2 sin 
360 2
Here r is diameter ÷ 2 i.e 24/2 is 12cm
90 22 1
=   12 2 −  12 2  sin 90
360 7 2
= 113.14 – 72
= 41.14cm2
 41cm2 to nearest cm2
356
2009/39 Neco ( Dec ) (b) Area of shaded portion = area of segment
= area of sector – area of 
 1
=   r 2 − r 2 sin 
360 2
73.74 22 1
=   5 2 −  5 2  sin 73.74
O 20cm 360 7 2
= 16.094 – 12.000
= 4.094cm2
 4.09cm2 to 2 d.p

Find the area of the shaded portion in the diagram 2014/4a


above if O is the centre of the circle. (Correct to 1 K
decimal place) X Y
A 114.3cm2 B 125.7cm2 C 157.1cm2
2 2
D 314.3cm E 514.3cm
Solution r r
Area of segment = area of sector – area of triangle
O
 1
=   r 2 − r 2 sin 
360 2
90 22 1
=   202 −  202  sin 90
360 7 2 (NOT DRAWN TO SCALE)
= 314.29 – 200 In the diagram, O is the centre of the circle radius r cm and
= 114.29cm2 XOY = 900, if the area of the shaded part is 504cm2 ,
 114.3cm2 to 1 d.p calculate the value of r. (Take  = 22/7)
Solution
2012/5 Area of segment = area of sector – area of triangle
X  1
=   r 2 − r 2 sin 
360 2
2
r   
Area of segment = − sin  
6cm

O  2 180 
2
r  22  90 
504 =
2  180  7 − sin 90
 
Y
r2
In the diagram, O is the centre of the circle and XY is a 504 = 1.571 − 1
2
chord. If the radius is 5cm and /XY/ = 6cm, Calculate,
correct to 2 decimal places, the: 504  2 = 0.571r2
(a) angle which XY subtends at the centre O 504  2
= r2
(b) area of the shaded portion 0.571
Solution 1765.32 = r2
We sketch as
Thus, r = 1765.32
X
= 42.02cm
m
5c
2014/10b
6cm

O O
5c P
m
m 7cm
Y 7c
Applying cosine rule to find  M 60 0 600 N
62 = 52 + 52 – 2  5  5 cos 
36 = 25 + 25 – 50 cos 
50 cos  = 50 – 36 Q
14
Cos  =
50 In the diagram, M and N are the centres of two circle of
 = cos 0.28
–1 equal radii 7cm. The circle intercept at P and Q.
= 73.7390 if PMQ = PNQ = 600, calculate, correct to the nearest
 73.740 whole number, the area of the shaded portion (Take  = 22/7)
357
Solution Irregular plane - shapes
When you draw a straight line from P to Q it become a (Area & Perimeter)
case of area of minor segment times two Irregular plane - shapes are figures that are combinations of
P two or more plane shapes . Methods of solving these types of
problems do not have a particular formula. We apply our
m
7c experience in plane shapes mensuration to solve them in
60 0 parts or as a whole.
M
7c 2015/ 7a ( Nov )
m A x P B
x
Q Q

Area of minor segment = area of sector – area of 


 1
=   r 2 − r 2 sin 
360 2 S
60 22 1 x
=   7 2 −  7 2  sin 60 0
360 7 2 D R x C
= 25.667 – /2  49  0.8660
1
In the diagram, ABCD is a square of side 5m, P, Q, R and S
= 25.667 – 21.217 are points on the square such that AP = BQ = CR = DS = x.
= 4.45 cm2 ( i ) Show that the area of the square PQRS is (2x2 – 10x + 25)m2
Thus, area of shaded portion = 2  4.45cm2 ( ii ) Find, correct to two decimal places, the values of x for
= 8.9cm2 3
2 which the area of PQRS is of that of ABCD
= 9cm to the nearest whole number 5
Solution
2005/4 NABTEB (Nov) Exercise 20.36 In right-angled QRC, C = 900 , /QC/ = 5 – x and
In the figure below, O is the centre of the circle of /RC/ = x . QR can be gotten by Pythagoras rule as:

radius 7cm and A O B = 800 QR2 = x2 + (5 – x ) 2
= x2 + 5(5 – x ) – x(5 – x )
= x2 + 25x – 5x – 5x + x2
QR = 2x2 – 10x + 25
2

Coincidentally, area of square PQRS = L2


O
= QR2
= (2x2 – 10x + 25)m2
80 0
7c

( ii )
m

Area of square ABCD = 52 i.e 25 m2


A 3
B Next Area of square PQRS = of ABCD
5
Calculate the (a) length of the minor arc AB 3
2x2 – 10x + 25 = × 25
(b) Area of the shaded portion 5
(Take  = 22/7) 2x2 – 10x + 25 = 15
2x2 – 10x + 10 = 0
x2 – 5x + 5 = 0
Applying the general quadratic formula
− (−5)  (−5) 2 − 4 1 5
x =
2 1
5  25 − 20
=
2
5 5
=
2
5  2.236
=
2
5 + 2.236 5 − 2.236
= or
2 2
7.236
= or 2.764
2 2
= 3.618 m or 1.382 m2
2

≈ 3.62 m2 or 1.38 m2 to 2 d.p


358
2005/27 (old) 2006/8
B C

x cm
D
A O
7cm
In the diagram, the shaded portion is 1cm wide.
Calculate, in terms of x, the area of the shaded portion
A (2x + 5) cm2 B (2x + 10) cm2 C (5x – 10) cm2 E
D (7x + 2) cm2 In the diagram, ABCDEO is two – thirds of a circle centre
Solution O. The radius AO is 7cm and /AB/ = /BC/ = /CD/ = /DE/.
Area of shaded portion Calculate, correct to the nearest whole number, the area of
= area of big rectangle – area of small rectangle. the shaded portion (Take  = 22/7)
= 7  x – [(7 – 2)(x – 2)] Solution
Note that in the small rectangle, the 1cm width of it is Angle on AD is 1800
subtracted twice from the length and breadth AOB + BOC + COD = 1800
= 7x – 5(x – 2) And since they are equal
= 7x – 5x + 10 180
= (2x +10) cm2 B. AOB = = 600
3
2005/31 (old) Since all the triangle are bounded by radius twice
B
B C
In the diagram, OPQR is a square of sides 10cm. The
shaded portion is a quadrant of a circle centre O, radius 7cm
OP. Calculate, leaving your answer in terms of , the 60
7cm 60 7cm
A O
ratio of the area of the shaded potion to that of the 7cm O
square OPQR C
O
7cm
D
Q P 60
7cm 7cm
60
O D
7cm E
10cm
Area of shaded portion
2
= area of (circle) – 4  (area triangle)
R 10cm O 3
2 1
A:2 B:3 C:4 D:5 =  r2 – 4  r2  sin 
3 2
Solution
The shaded portion is a sector with centre at O (900) or 2 22 1
=   72 – 4   72  sin 60
1 3 7 2
quadrant of a circle implies of 3600 i.e 900 = 102.667 – 98  0.8660
4
Thus the ratio of shaded portion to area square OPQR = 102.667 – 84.868
Area of sector ORP : area of square OPQR = 17.799 cm2
 = 18cm2 to nearest whole number
r 2 : LL
360 2015/2b
90
   10 2 : 10  10 P Q
360
Ten squared will cancel out
1
20cm

 : 1
4
Multiply through by 4 x cm
x cm
 : 4 (C) S R
10cm 10cm
The diagram shows a rectangle PQRS from which a square
of side x cm has been cut. If the area of the shaded portion is
484cm2, find the value of x
359
Solution 56l + 44l
Area of rectangle = length  breadth = 100
14
Here the breadth is 20cm and length is (10 + x + 10) 56L + 44L = 1400
and not (10 + x + x + 10) 100L = 1400
Area of shaded portion = area of rectangle PQRS – area of square
L = 14cm
484 = 20  (10 + x + 10) – x  x
L
484 = 400 + 20x – x2 But r is ; thus r = 14 ÷ 2 is 7cm
x2 – 20x + 84 = 0 2
factorising 2010/6c
R
(x – 6) (x – 14) = 0
x = 6cm or 14cm

2015/45 O
3.5m
x

P Q
2.2m
The diagram shows a circle centre O. if POQ = x0, the
diameter of the circle is 7cm and the area of the shaded
In the diagram, the shaded part is a carpet laid in a room portion is 27.5cm2, find, correct to the nearest degree, the
with dimensions 3.5m by 2.2m leaving a margin of value of x. (Take  = 22/7)
0.5m round it. Find the area of the margin Solution
A 4.7m2 B 4.9m2 C 5.7m2 D 5.9m2 The shaded part is the major sector of the circle
Solution 
Area of the margin Area of sector = r 2
360
= area of big rectangle – area of small rectangle
Here  is the reflex angle 360 – x and not x and
= 3.5 2.2 – (3.5 – 1.0) (2.2 – 1.0)
radius is diameter  2 i.e 7/2
Note that the margin is subtracted from the length and
Substituting
breadth twice 360 − x 22
= 7.7 – (2.5) (1.2) 27.5 =   3.5 2
360 7
= 7.7 – 3.0
(360 − x)269.5
= 4.7m2 A. 27.5 =
2520
27.5  2520
2009/11a = 360 – x
A circle is inscribed in a square. If the sum of the 269.5
perimeter of the square and the circumference of the 257.14 = 360 – x
circle is 100cm, calculate the radius of the circle x = 360 – 257.14
(Take  = 22/7) = 102.860  1030 to nearest degree
Solution 2010/42
L 22cm

L 8cm

4cm 4cm
Perimeter of square is 4L The diagram shows a rectangular cardboard from which a
(Circumference) perimeter of circle is 2r semi – circle is cut off. Calculate the remaining part.
Here r is L÷ 2 A 44cm2 B 99cm2 C 154cm2 D 165cm2
From the question’s data, we have Solution
4L + 2r = 100 becomes Area of remaining part
L = area of rectangle – area of semi – circle
4L + 2( ) = 100 But diameter of semi – circle is 22 – (4 + 4) i.e 14cm
2
22 L
It implies that its radius is 7cm
4L + 2   = 100 1
7 2 = Lb – (r2)
2
44l
4L + = 100 1 22
14 = 22  8 –   72
2 7
= 176 – 77
= 99cm2
360
2010/11a OY can be gotten from OCB when it is bisected as shown
A (3n-1)m B below:
Note ACB = 1300, Implies OCB = 650
O Z B
(2n+1)m

30cm Y
D C
In the diagram, ABCD is a rectangular garden 650
(3n – 1)m long and (2n + 1)m wide. A wire – mesh
135m long is used to mark its boundary and to divide it C
into 8 equal plots. Find the value of n OY
Solution By trig ratios sin 650 =
OC
We will apply the principle of perimeter here in OY
relabeling the diagram Sin 65 =
(3n-1)m 30
K G 30 sin 65 = OY
A I B
30 0.963 = OY
OY = 27.189 cm
Radius OY  27.2cm to 1d.p
E F (ii) Area of shaded portion
(2n+1)m
= area of ABC – area of semi – circle
1
L H C Area of ABC = base  height
D J 2
Our concern now is the total length of the wire mesh 135m long Half base here is OA or OB
At breadth: BC + GH + IJ + KL + AD = 5  (2n + 1) m 1
To find base : In  BCO, by trig ratio (Not bisected)
Since they are equal 2
O Z B
At length: DC + EF + AB = 3  (3n – 1) m
Since they are equal
Perimeter of wire – mesh = 135
30cm Y
5(2n + 1) + 3(3n – 1) = 135
10n + 5 + 9n – 3 = 135
650
19n = 135 – 2
19n = 133
C
19n 133
=
19 19 OB
Tan 65 =
n=7 OC
OB
2011/8b Tan 65 =
W O Z B 30
A 30 tan 65 = OB
Substituting
30cm

Area of ABC = 30 tan 65  30


X Y = 3 0  2.145  30
= 1930.5cm2
r 2
Thus area of shaded portion = 1930.5 –
2
C
22 (27.2) 2
In the diagram, a semi – circle WXYZ with centre O is = 1930.5 – 
inscribed in an isosceles triangle ABC. If / AC/ = /BC/, 7 2
 = 1930.5 – 1162.6
/OC/ = 30cm and A C B = 1300, calculate, correct to = 767.9cm2 to 1dp
one decimal place, the
(i) radius of the circle 2005/8 (Nov)
(ii) area of the shaded portion (Take  = 22/7) The diagram shows a rectangular lawn of length 12m and
Solution width 8m. Along one of the side of the lawn is a concrete
Radius is OW or OX or OZ or OY. wall and in the middle is a pond of diameter 5m and
surrounded by grass.
361
WALL Solution
1
Area of PQP =  15  8 sin Q
2
Note that PQR is 900 (angle in semi – circle)
1
Pond 8m =  15  8  sin 90
5m 2
1
=  15  8  1
2
12m = 60 cm2 ( B )
( a ) Calculate the cost of fencing the remaining sides of 15. Calculate, correct to 1 decimal place, the perimeter of
the lawn at the rate of D45 per metre the semi – circle PQR. (Take  = 22/7)
( b ) Calculate, correct to three significant figures the : A 106.9cm B 47.1cm C 43.7cm D 35.4cm
(i) area covered by grass Solution
(ii) Area covered by grass as a percentage of area of the 1
whole lawn Perimeter of semi – circle PQR = (2r)
2
( c ) If the pond is filled with water to a depth of 1.2m,
But r is half of PR and PR can be gotten by Pythagoras rule
calculate the volume of water in litres.
in PQR, since  PQR is 900 (angle in semi – circle)
(Take  = 3.142)
PR2 = 152 +82
Solution
PR2 = 225 + 64
(a) The remaining part is 8cm + 12m +8m = 28m
PR2 = 289
cost of fencing at D45 perimeter = D45  28
= D1260 PR = 289
(b) (i) Area covered by grass = 17cm,
= Area of rectangle – area of circular pond Radius = ½ of PR
= l  b – r2 = 1/2 of 17 = 8.5cm
= 12  8 – 3.142  (5/2)2 1
Thus perimeter of semi – circle PQR = (2r)
= 96 – 3.142  2.52 2
= 96 – 19.6375 = r
= 76.3625m2 = 22/7  8.5
Area (i) 76.3625 = 26.71cm
(ii)  100 = 100  26.7cm to one d.p
Total rec tan gle area 12  8
2007/50
In the diagram, O is the centre of the circle and PQ is a
76.3625 diameter. Triangle RSO is equilateral triangle of side 4cm.
=  100
96 Find the area of the shaded region
= 79.54%
(c) Volume of water in circular pond = r2h S R
= 3.142  2.52  1.2
= 23.565m3
Volume in litres = 23.565  1000
= 23565 litres P Q
O
2014/14 and 15 (Nov)
In the diagram, PR is a diameter /PQ/ = 15cm and
/QR/ = 8cm. use the information to answer questions 14
and 15
A 43.36cm2 B 32.07cm2 C 18.21cm2 D 6.93cm2
Solution
Area of shaded region = area of semi-circle – area of RSO
Since all the sides of RSO are given
P R Area of RSO = s( s − a)(s − b)(s − c)
Where S = 1/2 (a + b + c)
15
cm S = 1/2 (4 + 4 + 4) = 6cm
m
8c

Area of  RSO = 6(6 − 4)(6 − 4)(6 − 4)


Q
= 6(2)(2)(2)
14. Calculate the area of triangle PQR = 48
A 23cm2 B 60cm2 C 68cm2 D 120cm2 = 6.928cm2
362
Area of semi – circle = 1/2  r2 1979/9b
1 22 In the figure below, PQR is an equilateral triangle of side
=   42 20cm. Each arc is part of a circle whose centre is at the
2 7
= 25.143cm2 opposite vertex of the triangle. Calculate, correct to the
Thus area of shaded region = 25.143 – 6.928 nearest whole number, the area of the figure
= 18.215cm2 (Take  = 22/7)
P
 18cm2 to nearest whole number

2007/3
C

Q R

Solution
Note that we have three different circles
Since PQR is equilateral, R = 600 same as Q and P and
A 100m B PQ = QR = PR = 20cm. thus we sketch as:
P
The diagram shows the cross – section of a railway
tunnel. If /AB/ = 100m and the radius of the arc is 56m,
calculate, correct to the nearest metres, the perimeter of
the cross – section
Solution
60 0
We sketch the diagram as: Q R
The perimeter of the cross – section is perimeter of 20cm
major segment 1
Area of  PQR = a b sin C
= length of arc ACB + length of chord AB 2
 1
= 2r + 100m =  20  20  sin 60
360 2
We derive  with the aid of the diagram below = 173.2cm2
The arc PQ is part of the circle centre R
The sector PQR is part of the circle centre R
60 22
Area of sector PQR =   20 2
O 360 7
O = 209.5cm2
m

Area of shaded portion = area of sector PQR – area of PQR


56
56

= 209.5 – 173.2
= 36.3cm2
A 50m D 50m B From our question’s diagram, the other two circle segments
are the same as the one we solved for.
50 Required area of figure
In AOD, sin  =
56 = area of PQR + 3  area of shaded portion
Sin  = 0.8929 = 173.2cm2 + 3  36.3cm2
 = sin – 1 0.8929 = 282.1cm2  282cm2 to nearest whole number
= 63.240 2007/32 Neco
But  = 2 i.e 2  63.24 = 126.480 The composite figure below shows a semi-circle with centre
This is  in the minor segment of chord AB O and triangle PQR, right-angled at Q. Calculate the area of
 in the major segment of chord AB = 360 – 126.480 the figure (Take  = 22/7)
= 233.520 P

Thus perimeter of cross section =  2r + 100m
360
3.5
m O
233.52 22
=  2   56 + 100
360 7 R
= 228.33 + 100 Q 16m
= 328.33m A 751/4m2 B 871/2 m2 C 124m2
 328m to nearest metres
363
D 150 ½ m2 E 175m2 Solution
Solution We could sketch the diagram as:
Area of figure = area of semi-circle + area of  PQR A A
1 1
= (r2) + (PQ  OR)sinQ
2 2 A
O C
Note that : PQ is twice radius 3.5m B
O
1 22 1
=   3.52 +  7  16 sin 90 B C C
2 7 2 B
= 19.25 + 56
It is only the center diagram that can help us.
= 75.25m2 i.e 75 1/4m2
Here radius of the circle is the height of the inscribed ABC
1
2005 /48 (Nov) Area of ABC = base  height
A square of area 64cm2 has the same perimeter as a 2
rectangle of length 10cm. What is the width of the 1
Also area of ABC = a b sin C
rectangle? 2
A 6cm B 12cm C 16cm D 22cm Equilateral implies a = b = c = 600
Solution 1
=  12  12  sin 60
Area of square = perimeter of rectangle 2
L2 = (L + b) 3
64 = 2(10 + b) = 6  12  = 36 3 cm2
Divide through by 2 2
32 = 10 + b Substituting into area with height
32 – 10 = b 1
Area of ABC = base  height
22cm = b (D) 2
1
36 3 =  12  AO
2010/11b 2
Two opposite sides of a square are each decreased by 36 3 = 6AO thus , AO = 6 3 cm = ( A )
10% while the other two are each increased by 15% to
form a rectangle. Find the ratio of the area of the 2015/ 8a ( Nov )
rectangle to that of the square. X
Solution
Sketching
L 8cm

10%

W Z Y
64cm
15% 15% L In the diagram, WXYZ is a segment of a circle such that XZ
is the perpendicular bisector of WY. If /XZ/ = 8cm and
10% /WY/ = 64cm, calculate the radius of the circle.
Solution
Ratio of area of square : area of rectangle X
L  L : L – 20%  L – 30%
8cm

Note that the decrease is subtracted twice from length 32cm 32cm
W Y
and breadth. Z
100 L  100 L : 80 L  70 L r cm
100  100 : 80  70 O
10  10 : 87
100 : 56
25 : 14
Reducing further; divide through by 5 Given that XZ⊥WY; /WY/ = 64cm, WZ = ZY = ½WY= 32cm
Square 5 : 3 rectangle We draw a full circle centre O and radius r cm
Presenting properly as required 3 : 5 Then OY = OX = r
and OZ = OX – ZX = r – 8
2006/16 (Nov) In right-angled OZY : OY2 = OZ2 + ZY2
An equilateral triangle of side 12cm is inscribed in a r2 = (r – 8) 2 + 322
circle. Calculate the radius of the circle. r2 = r2 – 16r + 64 + 1024
16r = 1088
A 6 3 cm B 4 3 cm C 3 3 cm D 2 3 cm r = 68cm

364
Find, correct to 3 significant figures, the
1975/10b Exercise 20.36b (i) area of the triangle ABC
In the accompanying figure which is not drawn to scale, EF is (ii) total area of the shaded portion, and
a chord of the circle passing through E, C and F. CD bisects (iii) perimeter of the shaded area ACD
EF perpendicularly at D. If EF = 12cm and CD =2cm,
calculate the length of the radius of the circle whose segment 2009/7 Neco (Dec) Exercise 20.42
is shown. A
C
C

2cm
O

E D F D
12cm B
2005/29 (old) Exercise 20.37 The diagram above shows a sector of a circle of radius 10cm which
14cm subtends an angle of 800 at the centre O, of the circle. If C and D
are the mid point of OA and OB respectively, calculate, correct to
the nearest whole number, the: (i) area and
(ii) perimeter of the shaded region (Take  = 22/7)
O
14cm 2005/8 (Nov) Exercise 20.43
In the diagram, the sides of rectangle EFGH and PQRS are in the
ratio 3:1. Find the ratio of the area of shaded portion to the area of
The diagram shows a circle centre O inscribed inside a square rectangle PQRS
of side 14cm. Calculate the area of the circle. E F
A 44cm2 B 88cm2 C 154cm2 D 616cm2 P Q
2014/16 NABTEB Exercise 20.38
5cm
R
2cm S
3cm H G
A 3:1 B 6:1 C 8:1 D 9:1
8cm 2cm
2006/11 (Nov) Exercise 20.44
P
2cm
A
Calculate the area of the figure
A 26cm2 B 30cm2 C 34cm2 D 40cm2
2009/42 Neco (Dec) Exercise 20.39
O 120 0
Find the area of a circular field enclosed inside a square field
of side 21m such that each of the side of the square is tangent
to the circle. 12cm B
A 110.3m2 B 173.3m2 C 210.0m2 D 346.5m2 E 441.0m2 Q
2005/9c Exercise 20.40 The diagram shows a piece of cardboard in the form of a sector of
a circle. The radii OP and OQ are each equal to 12cm. A and B are
cm

the mid – points of OP and OQ respectively and POQ = 1200.


35

Calculate, correct to one decimal place, the:


(a) area of ABQP; (b) perimeter of ABQP (Take  = 22/7)
50cm
2005/25 (Nov) Exercise 20.45
In the diagram, O is the centre of the circle if
/PQ/ = 24cm and /RQ/ = 18cm, Calculate, correct to the nearest
The diagram shows a window consisting of rectangular and
whole number, the area of the circle.
semi-circular parts. The radius of the semi-circular part is
(Take  = 22/7)
35cm and the height of the rectangular part is 50cm. Find the
area of the window. (Take  = 22/7).
2006/8a NABTEB Exercise 20.41
In the diagram, ABCD is a circle centre O with diameter O
13cm. ABC is a triangle inscribed in the circle P R
B
24
1 2c m cm cm
18
A
O
1 3c m
C Q
A 2829cm2 B 1414cm2 C 707cm2 D 452cm2
D

365
CHAPTER TWENTY ONE Curved surface area & volume for one – open
Solid mensuration and closed end(s) cylinders
A solid is a three dimensional shape. It is usually
Curved surface area = base circumference × height
represented in two dimensions pictorially in books.
Examples of solid are shown below = 2r × h
= 2rh square unit.
Volume = base area × height
= r2 × h
= r2 × h cubic units

Total Surface area for open top cylinder only


Total surface area = area of all the faces
= Area of base + curved surface area
= r2 + 2rh
When we walk round our homes, it is observed that it
has some shape which are the same on both sides – but = r ( r + 2h)
on snapping a picture of the house, we can only see one
plane view of it. That is another example of a three Total Surface area for closed tops cylinder only
dimensional objects represented in two dimensions Total surface area = area of the faces
= Area of circular base + area of circular top + curved surface area.
Surface areas & volumes = r2 + r2 + 2rh
A painter hired to paint our house can only paint the walls = 2r2 + 2rh
round the home (surface area). On the other hand, when = 2r ( r + h ) square units
we want to sit, eat, sleep or even bring in new piece of
furniture, we occupy certain space in the house. We 2011/22
cannot sit on the wall, or lie on the wall – thus we occupy The height of a cylinder is equal to its radius. If the volume
space (volume) in our homes. We can say that surface is 0.216m3, calculate the radius
area in solids is the equivalent of perimeter in plane A 0.46m B 0.60m C 0.87m D 1.80m
shapes whereas volume in solids is an equivalent of area Solution
in plane shapes. Volume of cylinder = r2h
But here r is equal to h
CYLINDER 0.216 = r2r
A cylinder is a solid with circular cross section. 0.216 = r3
Examples are bournvita tin, tomatoes tin, milk tin, and 3
0.216 = r
open drum etc. 0.6m = r ( B )
There are two types of cylinders namely:
Open top cylinder & Closed ends cylinder 2008/4b NABTEB (Nov)
Of the three properties: curved surface area, total surface Find the total surface area of a closed cylinder
area and volume of a cylinder to be measured, only the of diameter 10cm and height 21cm (Use  = 22/7)
total surface area that has different formulae for the two Solution
types of cylinders.
Total surface area of closed cylinder = 2r (r + h)
r Note: radius is diameter  2
Open circular
top 22
= 2   5(5 + 21)
7
22
Height

= 2   5  26 = 817.14cm2
7
2013/2b Neco
If the volume of a cylindrical container of base radius 4cm is
Closed circular found to be 352cm3, calculate the depth of the container.
base
(Take  = 22/7)
Solution
r Closed circular Here, we are to find height h in:
top
Volume of cylinder = r2h
22
352 =  42  h
Height

7
352  7
=h
22  4 2
Closed circular 7cm = h
base

366
2007/8a 4327.71
A cylinder with radius 3.5cm has its two ends closed. Converting to litres : 4327.71cm3 = litres
1000
If the total surface area is 209cm2, calculate the height = 4.3277litres
of the cylinder. (Take  = 22/7)  4.328 litres ( A )
Solution
Total surface area of closed ends cylinder = 2r (r + h) 2005/22 NABTEB (Nov)
22 A cylindrical tank with diameter 1 metre and height
209 = 2   3.5 (3.5 + h)
7 2 metre is half filled with water. What is the volume of the
209  7 water?
= (3.5 + h)
2  22  3.5  
A m3 B m3 C m3 D 2m3
1463 4 2
= 3.5 + h
154 Solution
9.5 = 3.5 + h First, we find volume in m3 then take half of it
9.5 – 3.5 = h Volume of cylinder = r2h
6cm = h 1
2

2005/29 =    2
2
What is the total surface area of a closed cylinder of

height 10cm and diameter 7cm? (Take  = 22/7) = m3
A 77cm2 B 227cm2 C 297cm2 D 374cm2 2
Solution Thus, volume of half filled cylindrical tank
Total surface area of closed cylinder = 1  3  3
Area of circular base + area of circular top + curved surface area
=  m = m (A)
2 2 4
= r2 + r2 + 2rh 2015/35 and 36 (Nov)
= 2r ( r + h) A cylindrical tin with base diameter 14cm and
22 7  7  height 20cm is open at the top. (Take  = 22/7)
= 2   + 10  Note : radius is diameter  2
7 22  Use this information to answer question 35 and 36
 27 
= 22   = 297cm2 ( B ) 35. Find the total surface area of the tin.
 2 
A 976cm2 B 1034cm2 C 1188cm2 D 2376cm2
Solution
2014/46 Neco (Dec)
Total of surface area = area of base + curved surface area
A tin of milk has a height of 10.3cm and base radius
= r2 + 2rh
6.8cm. Find the volume of liquid in litres that can fill
the tin to capacity = r (r + 2h)
A 1.153litres B 1.208litres C 1.411litres Note: radius is diameter  2
D 1.497litres E 2.000litres 22
=  7(7 + 2  20)
Solution 7
First, we find the volume in cm3 then convert to litres. = 22(47)
Volume of cylinder = r2h = 1034cm2 ( B )
22 36. Calculate the volume of water in the tin when it is full.
=  (6.8) 2  10.3 A 3080cm3 B 4620cm3 C 6160cm3 D 12320cm3
7
= 1496.85cm3 Solution
1496.85 Volume of cylinder = r2h
Converting to litres : 1496.85cm3 = litres 22
1000 =  7 2  20
= 1.4969litres 7
 1.497 litres ( D ) = 3080cm3 ( A )
2006/28 Neco (Dec)
2007/31 Neco A solid cylindrical object of radius 7cm is 10cm high. Find
Find the capacity of a cylinder in litres whose its total surface area.
diameter is 18cm and height 17cm (Take  = 22/7) A 220cm2 B 594cm2 C 629cm2
A 4.328litres B 121.18litres C 43.28 litres D 660cm 2
E 748cm 2
D 4327.71litres E 4328.8litres Solution
Solution Total surface area of a solid cylindrical object is
First, we find the volume in cm3 then convert to litres. total surface area of closed ends cylinder = 2 r(r + h)
Note: radius is diameter  2 22
Volume of cylinder = r2h = 2   7(7 + 10)
7
22 2 22
=  9  17
7 = 2   7  17
7
= 4327.71cm3 = 748cm2 ( E )
367
2013/31 Neco Solution
A cylindrical tank of length 24cm has a volume The only item to show their similarity here is volume
of 14784cm3. Calculate its radius Volume of larger cylinder : volume of small cylinder
A 9.38cm B 14cm C 14.8cm 216 : 125
D 15.0cm E 24.8cm 2 : 1
Solution Thus if the height of larger cylinder is 30cm
Volume of cylinder = r2h ... Height of small cylinder is 15cm (C)
22 2 2014/38 (Nov)
14784 =  r  24 The ratio of the area of the base of a cylinder to the curved
7
surface area of the cylinder is 1: 4. If the radius of the
14784  7 cylinder is 4cm, find the height of the cylinder
= r2
22  24 A 1cm B 2cm C 4cm D 8cm
196 = r2 Solution
r = 196 = 14cm ( B ) Base area of cylinder : curved surface area
r2 : 2rh
2008/33 Neco
Some terms will cancel out
The radii of two similar cylindrical jugs are in
r : 2h
the ratio 3 : 7. Calculate the ratio of their volumes
1 : 4
A 1:4 B 3:7 C 9:49 D 1:64 E 27:343
When r = 1, 2h = 4
Solution
Thus, if r = 4, 2h = 4  4
Applying volume of cylinder formula
2h = 16
r2h : r2h
h = 16/2 = 8cm (D)
In volume terms r2 : r2
2014/9a Neco
In radius terms r=3 : r=7
The total surface area of a solid cylinder is 88cm2, while the
In volume terms 32 : 72
height is 7cm. Find, correct to three significant figure, the:
9 : 49 ( C )
(i) diameter of its cross section
2007/48 (ii)volume of the solid (Take  = 22/7)
The volume of a cylinder is 1200cm3 and the area of its Solution
base is 150cm2. Find the height of the cylinder (i) Diameter is twice radius
A 80.00cm B 8.00cm C 0.80cm D 0.08cm Total surface area of a solid cylindrical object is
Solution total surface area of closed ends cylinder = 2r(r + h)
Volume of cylinder =  r2h 22
1200 =  r2h ----- (1) 88 = 2   r (r + 7)
7
Area of base of cylinder =  r2 88  7
150 =  r2 ----- (2) = r (r + 7)
2  22
Replace  r in (1) by 150 from (2)
2
14 = r (r + 7)
1200 = 150h
14 = r2 + 7r
1200 r + 7r – 14 = 0
2
= h
150
Applying quadratic formula r = − b  b − 4ac
2

8 cm = h ( B )
2a
2012/15 2
− 7  7 − 4 1 (−14)
The curved surface area of a cylindrical tin is 704cm2. If r =
2 1
the radius of its base is 8cm, find the height
(Take  = 22/7) − 7  105
=
A 14cm B 9cm C 8cm D 7cm 2
−7  10.25
Solution =
Curved surface area of cylindrical tin = 2rh 2
−7 − 10.25 −7 + 10.25
22 = or
704 = 2   8  h 2 2
7
= − 17.25 or 3.25
704  7 2 2
= h
2  22  8 r = 1.625cm the positive value
14cm = h ( A ) Thus, diameter of its cross section = 2  r
= 2  1.625cm
= 3.25cm
2014/15 NABTEB (Nov) (ii) Volume of solid =  r2h
The volume of a cylinder is 216cm3. A similar cylinder 22
has a volume of 125cm3. If the height of the larger =  (1.625) 2  7
7
cylinder is 30cm, find the height of the smaller cylinder.
= 58.094cm3
A 25cm B 20cm C 15cm D 12cm
 58.1cm3 to 3s.f
368
2011/34 Neco Exercise 21.1 Open end – side view
A right circular cylinder of base radius 8cm has height
of 14cm. Find the volume of the cylinder.
A 988cm3 B 1, 408cm3 C 2, 816cm3 R = External radius
D 3, 423cm 3
E 5, 632cm 2 R
r r = internal radius
2009/3b Neco (Dec) Exercise 21.2
Find the volume of a cylinder whose height is 8cm and radius
5cm, correct to 3 significant figures (Take  = 22/7)
2011/9 Exercise 21.3 Area = R2 - r2
A cylindrical container has a base radius of 14cm and =  (R2 – r2 )
height 18cm. How many litres, correct to the nearest Difference in areas between the big circle with radius R and
litre of liquid can it hold? (Take  = 22/7) small circle with radius r
A 11 B 14 C 16 D 18 Similarly,
2007/18 Exercise 21.4 Volume = R2L - r2L
Find the volume of a solid cylinder with base radius =  L(R2 – r2 )
10cm and height 14cm (Take  = 22/7)
A 220cm3 B 880cm3 C 1400cm3 D 4400cm3 2006/14 (Nov)
The internal and external radii of a water pipe are 9cm and
2005/9a NABTEB (Nov) Exercise 21.5 10cm respectively. If the pipe is 35cm long, find, cm3, the
Calculate the volume of a cylinder of base radius 5.4cm volume of the material used in making it (Take  = 22/7)
and height 16.2cm. A 1, 100 B 1, 900 C 2,090 D 2,200
2014/9a Neco Adjusted Exercise 21.6 Solution
The total surface area of a solid cylinder is 88cm2. Here; volume of pipe = L (R2 – r2)
While the height is 5cm. Find the: 22
=  35 (10 2 − 9 2 )
(i) diameter of its cross section 7
(ii) volume of the solid (Take  = 22/7) = 22  5 (100 – 81)
2014 /18 Neco (Dec) Exercise 21.7 = 22  5  19 = 2,090cm3 ( C )
Find the volume of a cylinder that is closed at both ends 2005/2 (Nov)
with a radius 5.7m and height 6.3m. Give your answer A cylindrical pipe is 28metres long. Its internal radius
to the nearest m3 is 3.5cm and its external radius is 5cm. Calculate:
A 18 B 113 C 205 D 600 E 643 (a) the volume of water, in litres, that the pipe can hold
when full;
2010/33 UTME Exercise 21.8 (b) the volume in cm3 of metal used in making the pipe.
A cylindrical pipe 50m long with radius 7m has
(Take  = 22/7)
one end open. What is the total surface area of the pipe?
Solution
A 700m2 B 98m2 C 350m2 D 749m2 (a) First, we find the volume of water, it can hold in cm3.
2016/3b Exercise 21.9 Volume of water is the same as:
A cylindrical tin, 7cm high, is closed at one end. If its Volume of cylinder using the internal radius given.
total surface area is 462 cm2, calculate its radius Volume of water in cylindrical pipe =  r2h
(Take  = 22/7) 22
=  (3.5) 2  28
2016/16 Exercise 21.10 7
The curved surface area of a cylinder 5cm high is = 1078 cm3
110 cm2. Find the radius of its base. (Take  = 22/7) 1078
A 2.6cm B 3.5cm C 3.6cm D7.0cm Converting to litres 1078cm3 = litres
1000
= 1.078 litres
(b) Volume of metal used = L (R2 – r2)
HOLLOWED CYLINDER, PIPES & RINGS 22
=  28 (5 2 − 3.5 2 )
This is a special type of cylinder, which is open at both 7
ends but has very thick layer. Examples are culverts, = 22  4 (25 – 12.25)
ring pipes etc. Diagrammatic presentation of such = 22  4  12.75
shapes is shown below = 1122cm3
L
2002/14 (Nov) Exercise 21.10b
The internal and external radii of a cylindrical bronze pipe
are 1.5cm and 2cm respectively. If the pipe is 10cm long,
calculate the volume of the bronze used
( Take  = 22/7 )
A. 51/2 cm3 B. 55cm3 C. 196 2/5cm3 D 550cm3
369
CONE Radius r of new cone formed
A cone is a solid with circular base and slant sides. The r = R
cone of concern to us is the right circular cone. There is 360
the ordinary right circular cone and cone formed from
a sector of a circle 2. R = L
Radius of the sector of a circle is equal to the slant height of
the cone formed.

3. Recall L2 = h2 + r2
Slant (L) h2 = L2 – r2
height
Height 2
(h) h = L2 - R
360

r Problems on cones formed from a sector


Circular base
2007/7b NABTEB
A cone is formed by folding a major sector of a circle having
angle 2200 at the centre. Calculate the circumference of the
Properties to be measured base of the cone if the diameter of the circle is 14cm, correct
Curved surface area = rL to 1 decimal place.
Total surface area Solution
= Area of circular base + curved surface area
220 0
= r2 + rL
= r ( r + L) 7cm 7cm
h
Volume = 1/3 r2h
r
Slant height (L) , radius (r) and height (h) are related by
Pythagoras’ theorem : L2 = h2 + r2 sector Resulting cone

Cone base is circular, thus


Cones formed from a sector Circumference of cone base = 2r
The diagram 1 shown below is a sector of a circle radius R
But r = (circle radius)
R which subtends an angle  in the center O. then FigII 360
is the cone formed from the sector. 220
= 7
O 360
= 4.278cm
Fig II 22
Fig I Thus circumference of cone base = 2   4.278
O 7
R R=L = 26.89cm
h  26.9cm to 1d.p

2007/16
r A sector of a circle of radius 14cm containing an angle 600 is
A B
folded to form a cone. Calculate the radius of the base of the
cone
A 51/2cm B 42/3cm C 31/2cm D 21/3cm
To measure the properties of a cone formed from a Solution
sector we need, L and h. This is gotten from the R
relationship between a sector and the cone it formed. Cone base radius = (circle radius)
360
Relationship between a sector of a circle and 60  14
=
the cone it forms 360
7
From fig. I and II = = 21/3cm ( D)
3
1. Length of arc = circumference of base of cone 2013/50
 × 2R = 2r An open cone with base radius 28cm and perpendicular
360 height 96cm was stretched to form a sector of a circle.
R = r Calculate the area of the sector (Take  = 22/7)
360 A 8800cm2 B 8448cm2 C 4400cm2 D 4224cm2
370
Solution 2014/23
 The area of a sector of a circle with diameter 12cm is 66cm2.
Area of sector =  r2 If the sector is folded to form a cone, calculate the radius of
360
the base of the cone. (Take  = 22/7)
R
Cone base radius = (radius of circle) A 3.0cm B 3.5cm C 7.0cm D 7.5cm
360 Solution
R R
28 = Cone base radius = (circle radius)
360 360
R = L and L2 = h2 + r2 Here radius is diameter divided by 2
L2 = 962 + 282 
= 9216 + 784 But we find  from: area of sector =  r 2
L2 = 10,000 360
 22 2
L= 10,000 = 100cm 66 =  6
360 7
R 66  360  7
Thus, Cone base radius = (radius of circle) = 
360 22  6 2
  100 2100 = 
28 =
360 R
28  360 Thus cone base radius r = (circle radius)
=  360
100 210  6
100.80 =  = = 3.5cm ( B )
360
100.8 22
Area of sector =   100 2 2015/52 Neco Exercise 21.11
360 7 A sector which subtends an angle of 900 was cut from a
= 8800cm2 ( A ) circle of radius 8cm and folded to give a cone. Find the
radius of the cone
2009/4 (Nov) A 1cm B 2cm C 3cm D 4cm E 5cm
A sector of angle 1350 is removed from a thin circular
metal sheet of radius 40cm. It is then folded with the 2005/7b (old) Exercise 21.12
straight edges coinciding, to form a right circular cone. A sector of a circle of radius 12cm subtends an angle
Calculate the: of 1200 at the centre. If it is folded to form a cone, calculate:
(a) base radius of the cone, correct to two decimal ( i ) the base radius of the cone
places: ( ii ) the height and volume of the cone, correct to
(b) greatest volume of liquid which the cone can hold, three significant figures (Take  = 3.142)
leaving your answer correct to the nearest cm3
(Take  = 22/7)
Solution Problems on cones
R 2014/33 NABTEB (Nov)
(a) Cone base radius = (circle radius) Calculate the volume of a cone with radius 3cm and
360
135  40 perpendicular height 3.5cm (Take  = 22/7)
=
360
= 15cm
1 2
(b) Volume of cone = r h
3 3.5cm
But L2 = h 2 + r 2
where L is radius of sector and r is cone base radius
402 = h2 + 152 3cm
40 – 152 = h2
2
A 3.3cm3 B 33.0cm3 C 65.0cm3 D 66.0cm3
1600 – 225 = h2
Solution
h2 = 1375
1 2
h = 1375 Volume of cone = r h
3
= 37.08cm 1 22
1 2 =   3 2  3.5
Volume of cone = r h 3 7
3 = 33cm3
1 22 2
=   15  37.08 2009/2b Neco (Dec)
3 7
= 8740.29cm3 Find the volume of a cone whose height and base radius are
= 8740cm3 to the nearest cm3 15cm and 6cm respectively

371
Solution 22 6  6 
=   + 5
1 7 22 
Volume of cone = r2 h
3 22
1 22 =  3 8
=   6 2  15 7
3 7 = 75.43cm2  75cm2 to the nearest whole number
= 565.71cm3
2014/47 NABTEB
2014/14 NABTEB Two similar cones have radii 3cm and 5cm. Find the ratio of
Find the total surface area of a cone radius 7cm their volumes.
and slant height 8cm. A 2:1 B 3:5 C 9:25 D 27:125
A 56cm2 B 77cm2 C 110cm2 D 330cm2 Solution
Solution Cone A : Cone B
Total surface area 1 2 1 2
= area of circular base + curved surface area r h : r h
3 3
= r2 + rL
r2 : r2
= r (r + L) 32 : 52
22 9 : 25 (C)
=  7 (7 + 8)
7
22 2008/7b NABTEB (Nov)
=  7  15 = 330cm2 ( D )
7 A vertical cone has base diameter 16cm and
slant height 17cm. Find the
2006/49 and 50 (Nov) (i) curved surface area
A hollow right circular cone has a base radius of (ii) volume of the cone
3.50cm and a slant height of 4.61cm. Solution
Use the information to answer question 49 and 50 (i) Curved surface cone = rL
(Take  = 22/7) 22 16
  17
=
49. What is the curved surface area of the cone? 7 2
A 66.76cm2 B 50.71cm2 C 38.48cm2 D 32.94cm2 = 427.43cm2
Solution 1 2
(ii) Volume of the cone = r h
Curved surface area = rL 3
22 But L2 = h2 + r2
=  3.5  4.61
7 h is not given: L2 = h2 + r2
= 50.71cm2 ( B ) 172 = h2 + 82
50. Find, correct to one decimal place, 17 – 82 = h2
2

the volume of the cone 225 = h2


A 77.9cm3 B 38.5cm3 C 32.9cm3 D 11.5cm3 15cm = h
Solution 1 22 2
Thus Volume of cone =   8  15 = 1005.71cm3
1 2 3 7
Volume of cone = r h
3
But L2 = h2 + r2 2008/32 Neco
(4.61)2 = h2 + (3.5)2 The ratio of the base area of a hollow cone to that of its
21.25 – 12.25 = h2 curved surface is 1: 4. If its base radius is 7cm, calculate the
9 = h2 slant height of the cone
h = 3cm A 7cm B 22cm C 28cm
1 22 D 49cm E 154cm
Volume =   3.5 2  3 Solution
3 7 Base area of a cone is circular, r2
= 38.5cm3 ( B ) Base area : curved surface area
r2 : rL
2009/48 NABTEB (Nov)
1 : 4
Find the total surface area of a solid cone of diameter
Canceling out common items
6cm and a slant height 5cm to the nearest whole number
r : L
(Take  = 22/7)
Radius( r ) 7cm = 4  7cm (L)
A 362cm2 B 350cm2 C 180cm2 D 75cm2
= 28cm (L) C .
Solution
Total surface area
2011/4b Neco
= area of circular base + curved surface area
The base diameter of a cone is 18cm and height is 24cm.
= r2 + rL Find the volume and the curved surface area of the
= r (r + L) cone.(Correct to 2 decimal places)
372
Solution 2013/33 Neco Exercise 21.13
1 A cone has a base radius 7cm and height 9cm.
Volume of cone = r2 h Calculate its volume.
3
A 462cm3 B 624cm3 C 928cm3 D 987cm3 E 1, 254cm3
Radius is diameter divided by 2
1 22 2 2005/9b NABTEB (Nov) Exercise 21.14
=   9  24 A cone of height 12cm has base radius 5cm. Calculate the
3 7
= 2036.57cm3 to 2d.p (i) slant height,
Curved surface area of cone = rL (ii) curved surface area and (iii) volume (Take  = 22/7)
But L2 = h2 + r2
= 242 + 92 2010/4 Exercise 21.14b
= 576 + 81 r
L2 = 657
L = 657 = 25.63cm
22 h

5c m
Curved surface area =  9  25.63

10.
7
= 724.96cm2 to 2dp

2012/34 Neco The diagram shows a cone with slant height 10.5cm. If the
Find the radius of a cone with height 7cm and volume curved surface area of the cone is 115.5cm2 , Calculate,
of 550cm3 correct to 3 significant figures, the:
(a) base radius r; (b) height, h; (c) volume
A3 5 B3 2 C5 2 D5 3 E5 5
of the cone (Take  = 22/7)
Solution
1 2 2014/25 NABTEB Exercise 21.15
Volume of cone = r h Calculate the volume of a cone base radius 3cm and slant
3
height 5cm.
1 22
550 =   r 2  7 A 3cm3 B 9cm3 C 12cm3 D 24cm3
3 7
550  3  7 2013/47 Exercise 21.16
= r2 The slant height of a cone is 5cm and the radius of its base is
22  7
75 = r2 3cm. Find, correct to the nearest whole number, the volume
of the cone (Take  = 22/7)
r = 75
A 48cm3 B 47cm3 C 38cm3 D 13cm3
= 25  3
= 5 3 cm (D)

2012/30 Neco SPHERE

.
The curved surface area of a cone of base radius 7cm
is 550cm2. Find the vertical height of the cone.
(Take  = 22/7) r
A 78.60cm B 25.96cm C 25.00cm
D 24.00cm E 10.70cm
Solution
L2 = h2 + r2
h = ?, r = 7cm, L can be gotten from curved surface area.
Curved surface area of cone = rL (I) Surface Area of Sphere = 4r2
22 (II) Volume of sphere = 4/3  r3
550 = 7 L
7
550  7 HEMISPHERE
= L
22  7
25cm = L r
Thus L2 = h2 + r2 becomes
252 = h2 + 72
252 – 49 = h2
625 – 49 = h2
576 = h2
576 = h
i) Curved surface area = ½ (4r2)
24cm = h (D)
(Half surface area of sphere) = 2r2

373
ii) Total surface area 2014/7a (Nov)
= Curved surface + circular surface area A spherical tank of diameter 3m is filled with water from a
= 2 r2 + r2 pipe of radius 30cm at 0.2m per second. Calculate, correct to
= 3r2 3 significant figures, the time, in minutes, it takes to fill the
iii) Volume of hemisphere = ½ ( 4/3 r3 ) tank (Take  = 22/7)
Solution
= 2/3  r3 (half the volume of sphere)

2014/24 NABTEB
Calculate the total surface area of a hemisphere with radius
3cm 3m
A 9cm2 B 18cm2 C 27cm2 D 45cm2
Solution
Total surface area of hemisphere
= curved surface area + circular surface area 4 3
Volume of spherical tank = r
= 2r2 + r2 3
= 3r2 3
4 22  3 
= 3  32  = 27cm2 ( C ) =     = 14.14m3
3 7 2
2006/24 Neco Volume of water per second = r2h
Find the radius of a sphere, if 3/4 of its volume 22
=  (0.3)2  0.2
is 134.75cm3 (Take  = 22/7) 7
A 176.67cm B 134.75cm C 101.06cm = 0.05657m3/s
D 42.87cm E 3.50cm V tan k
Solution Time taken in sec =
V / sec of pipe
4
Volume of sphere =  r3 = 14.14 = 249.96 sec
3
0.05657
3 4
of  r3 = 134.75 (find r) 249.96
4 3 Time taken in minutes = = 4.166 minutes
60
r3 = 134.75
r3 = 134.75  1  4.17min to 3s.f
 2015/8
7 = 42.875cm A tap is leaking at the rate of 2cm3 per second into an empty
= 134.75 
22 container with capacity 45litres. How long will it take to fill
3 the container?
r = 42.875 = 3.50cm (E) A 8 hrs B 6 hrs 15 mins C 4 hrs 25 mins D 3 hrs
2005/24 NABTEB Solution
What is the capacity of a spherical tank whose diameter Volume of container in cm3 = 45  1000
is 1.5m? = 45000cm3
3 9 1
A m3 B m3 C 3m3 D 4 m3 Vol of container
4 16 2 Time taken in sec =
Rate (Vol ) of leaking
Solution
4 45000
Volume of sphere =  r3 =
2
3
3
= 22500 sec.
4
=     1.5  =
225000
minutes = 375 minutes
3  2  60
4 3 375
=     3  1  =
60
hours 6 hrs 15 mins (B)
3 2 2 2014/60 Neco Exercise 21.17
3
=      = 9  m3
4 3 What will be the volume of an hemisphere of diameter 21cm?
(B) A 231.0 cm3 B 808.5 cm3 C 1617.0 cm3
3 4 16
D 2425.5 cm3 E 7276.5 cm3
2006/7 Neco
Find the surface area of a sphere with radius 5cm 2007/42 UME Exercise 21.18
(Take  = 22/7) The volume of a hemispherical bowl is 7182/3 cm3.
A 78.6cm2 B 104.8cm2 C 220.0cm2 Find its radius ( = 22/7)
2 2
D 314.3cm E 550.0cm A 4.0cm B 5.6cm C 7.0cm D 3.8cm
Solution
Surface area of sphere = 4r2 2009/29 UME Exercise 21.19
22 Find the radius of a sphere whose surface area is 154cm2
=4  52
7 (Take  = 22/7)
= 314.29cm2  314.3cm2 ( D )
374
A 7.00cm B 3.50cm C 3.00cm D 1.75cm 2014/40 (Nov)
The height of a triangular prism is 6cm. If the cross section
TRIANGLE PRISM of the prism is an equilateral triangle of side 8cm, find its
A prism is a solid with uniform cross sectional area. volume
A 96 3 cm3 B 64 3 cm3 C 32 3 cm3 D 16 3 cm3
Solution
Volume of prism = base area  height
ht = cross sectional area  height
heig
a c Cross sectional area = area of equilateral triangle 8cm
= s(s − a)( s − b)( s − c)
1
b Where s = /2(a + b + c)
S = 1/2(8 + 8 + 8)
Volume = base area x height = 1/2 ab ×h = 1/2(24) = 12
Relationship between a, b and c is that of Pythagoras’ rule
Cross sectional area = 12(12 − 8)(12 − 8)(12 − 8)
c2 = a 2 + b2
= 12(4)( 4)( 4)
1
Cross sectional area = /2 a b
= 3 4  4  4  4
= 16 3 cm3
c
a Thus volume of triangular prism = 16 3  6
= 96 3 cm3 (A)
b
2005/8a (old)
Note D

C is not used; since the area of a triangle A


= 1/2 base × height (perpendicular height)
m

= 1/2 a × b E
8c

F
2009/5 B
20cm
F C
A The diagram shows a solid prism whose cross – section is an
equilateral triangle of side 8cm. If the length of the prism is
E 20cm, Calculate its total surface area correct to three
24cm significant figures
D Solution
B 4 0c m Total surface area = area of ABC + area DEF
7cm C + Area of rect. ADCF + Area of rect.BECF + Area of rect.ABED
In the diagram, ABCDEF is a triangular prism. Since the two triangles are equal, and the rectangles are equal
ABC = DEF = 900, |AB| = 24cm, |BC| = 7cm and Total surface area = 2(Area of  ABC) + 3(Area of rect.ADCF)
|CD| = 40cm. Calculate : Area of triangle with all sides given = s(s − a)( s − b)( s − c)
(a) |AC| ; (b) The total surface area of the prism. Where s = 1/2(a + b + c)
Solution s = 1/2(8 + 8 + 8)
(a) By Pythagoras rule in ABC s = 1/2(24) = 12
AC2 = 242 + 72
Area of  ABC = 12(12 − 8)(12 − 8)(12 − 8)
AC2 = 576 + 49
AC = 625 = 12(4)( 4)( 4)
= 25cm = 768 = 27.71cm2
(b) Total surface area of prism Area of rectangle ADCF = 20  8 = 160cm2
= Area of ABC + Area of DEF + area of rectangle ACDF
... Total surface area = 2 (27.71) + 3(160)
+ Area of rect. BCDE + Area of rect. ABEF
= 55.42 + 480
Area of ABC = 1/2base  height (perpendicular height)
= 535.42cm2
Area of rectangle = length  breadth
1 1 2007/33 Neco
Total surface area of prism =  7  24 +  7  24 The cross – section of a prism is a right – angled triangle
2 2
+ 40  25 + 40  7 + 40  24 13cm by 12cm by 5cm, calculate its volume if its height
= 84 + 84 + 1000 + 280 + 960 is 0.025m
= 2408cm2
375
A 0.75cm3 B 1.95cm3 C 75cm3
3 3
D 195cm E 200cm Area of CEFD = Area of trapezium
Solution
Volume of prism = base area  height =
1
( 3 + 8 )  12
1 2
= base   height  height
2 = 66m2
By simple knowledge of Pythagoras triangle, we draw Volume of block = base area  height
the right – angled side of the prism as 1
= ( 3 + 8 )  12  6
2
= 396m3
Example P6
13

The dimensions of a prism with surface area ABCD is given


c

5cm
m

in diagram below. Find its volume


m G
12cm 0. 0 25

1
=  12  5  (0.025 100) A
2 H F
Note 100cm is one metre

10
= 75cm3 C. 13m

m
D E
2015/15 (Nov) B 7m
21m
The dimensions of a triangular prism are as shown in C
the diagram. Find its volume Solution
A

6m
10
13m Z m
14cm
7m
1 2. 5 D
cm 20cm B
7m
3 3 3 3
A 1750cm B 1770cm C 1870cm D 1970cm C
Solution Volume of prism = base area  height
Volume of prism = base area  height = Area of trapezium ABCD  height
1 1
= base   height  height = ( 13 + 7 )  ZD  21
2 2
1
=  12.5  14  20 By Pythagoras rule : ZD2 = 102 – 62
2 ZD = 64 = 8m
= 1750cm3 (A)
1
Volume of prism = ( 13 + 7 )  8  21
2
2007/9b NABTEB
G = 1680m3
2011/15 Exercise 21.20
F The cross section of a uniform prism is a right – angled
triangle with sides 3cm, 4cm and 5cm. If its length is 10cm,
A 8m calculate the total surface area
A 142cm2 B 132cm2 C 122cm2 D 112cm2
3m E
D
B 2008/31 NABTEB (Nov) Exercise 21.21
12m
6m Calculate the volume of a triangular based prism with base
C area 24cm2 and height 10cm
A 12cm3 B 120cm3 C 240cm3 D 360cm3
The figure given above is a solid with CEFD as the
cross section, calculate the: 1997/30 UME Exercise 21.22
(i) area of CEFD and (ii) volume of the solid.
Solution
F 8cm

D 5cm
8m
3m 11cm
10cm

C 12m E Find the volume of the prism above

376
A 990cm3 B 880cm3 C 550cm3 D 495cm3 2008/36
What is the length of a cube whose total surface area is Xcm2
Cube x
and whose volume is cm3
A cube is a solid with six faces of equal dimension 2
A B A3 B 6 C 9 D 12
Solution
C D Total surface area of cube = 6x2
X = 6x2
P x Let X be y thus y = 6x2
O
x Volume of cube = x3
x R x
Q = x3
2
Surface area (s) = 6  area of one face y
= 6x2 Let X be y thus = x3
2
x is the length of the side
Now we have y = 6x2 ----- (1)
Volume = base area  height
y
= x2  x = x3 ------ (2)
= x3 2
Length of diagonal QB = x2 + x2 + x2 From (2) y = 2x3 ------ (4)
Equating y in (1) and (4)
= 3x2 6x2 = 2x3
= x√3 Divide through by x2
6x 2 2x3
Length of side diagonal = 2
x +x 2 =
x2 x2
= 2x2 6 = 2x
= x√2 6
= x
2
2009/41 Neco (Dec) 3= x (A)
Find the volume of a cube of side 12cm
A 36cm3 B 144cm3 C 864cm3 2009/18 (Nov)
D 1082cm 3
E 1728cm 3 Find the diagonal of a cube whose edge is 5cm long
Solution A 8.66cm B 9.08cm C 10.00cm D 11.18cm
Volume of cube = x 3 Solution
= 123 Length of diagonal of a cube = x 3
= 1728cm3 ( E ) = 5 3 cm
2006/4 Neco
= 8.66cm ( A )
Calculate the cost of parting the surface area of a
solid cube whose edge is 8m at N 5.00 per m2
2008/18 Exercise 21.23
A N 960.00 B N 1,280.00 C N 1,920.00
If the volume of a cube is 343cm3, find the length
D N 2,560.00 E N 3,200.00
of its side
Solution
A 3cm B 6cm C 7cm D 8cm
Total surface area of cube = 6x2
= 6  82
2005/6 (Old) Exercise 21.24
= 3842
What is the total surface area of a cube of side 4 cm?
Cost of painting at N 5.00 per m2 = 384  5 A 36cm2 B 64cm2 C 96cm2 D 144cm2
= N 1, 920.00 ( C )

2010/48 NABTEB (Nov)


The volumes of two similar solid cubes are 729 cm3
and 512cm3. Find the ratio of their lengths
A4:3 B9:7 C8:7 D9:8
Solution
Volume of cube = x3
3 Cube volume = x (length)
Ratio of lengths
3 3
729 : 512
9 : 8 (D)

377
CUBOID 2012/48
A cuboid is a solid shape with a rectangular base and The volume of a cuboid is 54cm3. If the length, width and
sides. height of the cuboid is in the ratio 2 : 1: 1 respectively,
find its total surface area
A 108cm2 B 90cm2 C 80cm2 D 75cm2
Solution
Volume of cuboid = xyz
Applying the given ratio values
54 = (2x)  x  x
54 = 2x3
Divide through by 2
It consists of 6 rectangles;. of which 2 opposite ones 27 = x3
are the same in area. Thus , 3
27 = x
Surface area = 6 rectangles
3cm = x
= 2xy + 2xz + 2yz
Thus the cuboid has length, width and height
Volume = Base area  height
2  3cm, 3cm and 3cm
= xy  z Total surface area of cuboid = 2  6  3 + 2  6  3 + 2  3  3
= xyz = 36 + 36 + 18
Alternatively,
= 90cm2 ( B )
Volume = Length  breadth  Height
2009/33
2005/39 The capacity of a water tank is 1,800litres. If the tank is in
The total surface area of the walls of a room, 7m long, the form of a cuboid with base 600cm by 150cm, find the
5m wide and xm high is 96m2. Find the value of x height of the tank.
A 2/ 3 B2 C4 D8 A 2cm B 20cm C 200cm D 2000cm
Solution Solution
Ceiling First we convert capacity in litres to volume in cm3
1,800litres = 1800  1000cm3
= 1800, 000cm3
xcm Volume of cuboid = base  height
Floor 1800,000 = 600  150  height
5cm 1800000
7cm = height
600  150
Total surface area = 4 rectangle of the rooms walls 20cm = height ( B )
96 = 2(5  x) + 2(7  x)
96 = 10x + 14x 2005/35 (Nov)
96 24 x A closed box of matches is in the form of a cuboid with
=
24 24 dimension 5.5cm by 3.5cm by 1.5cm. Find, correct to the
4 = x (C) nearest whole number, the total surface area of the box
A 66cm2 B 33cm2 C 29cm2 D 21cm2
2010/31 Neco Solution
A square based box with a height of 8cm has a volume Total surface area of cuboid = 2xy + 2xz + 2yz
of 2048cm3. Find the length of the side of the base = 2  5.5  3.5 + 2  5.5  1.5 + 2  3.5  1.5
A 4cm B 8cm C 16cm = 38.5 + 16.5 + 10.5
D 64cm E 256cm = 65.5cm2
Solution  66cm2 (A)
Volume of cuboid box = base area  height
= square base area  height 2008/38 Neco
E F
2048 = x2  8
2048 G
= x2 H
8
256 = x2 B 8cm
A 6c m
256 = x C
D 12cm
16cm = x ( C )
In the above rectangular box, |DC| = 12cm, |BC| = 6cm
and |FB| = 8cm. Calculate the angle between the planes
AFGD and ABCD
A 56.30 B 41.80 C 36.90 D 33.70 E 26.60

378
D 572cm3 E 678cm3
Solution
Solution 1
G Volume of pyramid = base area  height
3
Base area here is square(length2)
8cm 1
=  62  42
O 3
D 12cm C = 504cm3
2014/59 Neco
Angle between two planes Find the perpendicular height of a right pyramid whose
8 volume is 21cm3 and its base is a square of side 3cm
tan  = ,
12 A 3cm B 4cm C 5cm D 6cm E 7cm
 = tan – 1 0.6667 = 33.70 ( D ) Solution
1
2005/21 NABTEB (Nov) Exercise 21.25 Volume of pyramid = base area  height
3
Find the total surface area of a cuboid
Base is square (length2)
6cm by 5cm by 2cm
1
A 52cm2 B 60cm2 C 104cm2 D 120cm2 21 =  32  height
3
Exercise 21.26
21 = 3  height
Find the capacity in litres of the water tank with
dimensions given below 21 3  height
=
3 3
7cm = height ( E )
12cm
7c m 2010/38 NABTEB (Nov)
Find the area of the base of a square based pyramid whose
20cm volume and height are 2700cm3 and 9cm
2016/4 Neco Exercise 21.26b A 900cm2 B 270cm2 C 210cm2 D 90cm2
30,000 litres of fuel is transferred to a rectangular tank Solution
with dimensions (7.5  4.2  1.2)m. 1
Volume of pyramid = base area  height
a. Calculate, correct to 2d.p, the depth of fuel in the tank 3
b. How many more litres of fuel would be needed to fill Base area here is square (L2)
the tank 1
Pyramid on a square base 2700 =  L2  9
3
Pyramid is a solid shape with triangular faces meeting 2700 = 3L2
at a point (vertex).The shape lies on a base, which may 2700
be square, triangle, rectangle e t c = L2
3
Square base pyramid 900cm2 = L2 (Base area) A.
Vertex
2013/46
A pyramid has a rectangular base with dimensions
Sl
an

12m by 8m. If its height is 14m, calculate the volume


th

A 344m3 B 448m3 C 632m3 D 840m3


eig
ht

Solution
1
Volume of pyramid = base area  height
3
Base area here is rectangle (length  breadth)
1
Base
=  12  8  14
3
1
= 448m3 ( B )
Volume = /3 area of base  height 2008/5b
= 1/3 L2  h The base of a pyramid is a 4.5m by 2.5m rectangle. The
Since, the area of a square = L  L height of the pyramid is 4m. Calculate its volume
= L2 Solution
1
2013/32 Neco Volume of pyramid = base area  height
3
Find the volume of a right pyramid of square base 6cm
Base area here is rectangle (length  breadth)
and 42cm height
A 324cm3 B 385cm3 C 504cm3
379
1 O
=  4.5  2.5  4
3
= 15m3
2005/12b
O
13cm

12cm
P
Q N
S
R
4cm
A 100cm3 B 200cm3 C 310cm3 D 600cm3
Solution
B O
A
N
2cm
C
D 2cm
In the diagram, OABCD is a pyramid with a square 13cm

12cm
bases of side 2cm a slant height of 4cm. Calculate,
correct to three significant figures; P
Q
(i) the vertical height of the pyramid N
S
(ii) the volume of the pyramid R
Solution 1
O Volume of pyramid = base area  height
3
But first, we find side QP
In ONR, 132 = 122 + NR2
169 = 144 + NR2
4cm
169 – 144 = NR2
25 = NR2
NR = 5cm
B
A In QPR, PR = 2  NR and QP = QR(Both lines are equal)
N
2cm PR2 = QP2 + QR2
C 102 = QP2 + QP2
D 2cm
100 = 2QP2
(i) To find ON, we need to know NC from AC 50 = QP2
In ADC, AC2 = 22 + 22 (Pythagoras rule) 50 cm = QP
AC = 8 Base area here is square
1 1
Thus, NC = 8 Volume of pyramid =  50  50  12
2 3
In ONC, OC2 = NC2 + ON2 = 1/3  50  12 = 200cm3
(B)
2
 8 2013/9 Neco
42 =   + ON2 The base ABCD of a right pyramid with vertex V, is a
 2 
 
square of side 32cm and the length of a slant edge is 40cm.
8
16 = + ON2 Calculate, correct to 1 decimal place the:
4
(a) height of the pyramid and
16 – 2 = ON2
(b) total area of its surfaces.
14 = ON2
Solution
ON = 14 cm vertical height of the pyramid O
 3.74cm to 3s.f
1
(ii) Volume of pyramid = base area  height
3
Base area here is square
40cm
1
=  2 2  14
3 C
A
= 4.989cm3 32c E
 4.99cm3 to 3 sf m
D 32cm
B

2005/64 (old) Height of pyramid = OE


In the diagram, OPQRS is a right pyramid on a To find OE, we need ED which is half of AD
square base OS = 13cm and ON = 12cm. Calculate the In right – angled ACD : AD2 = 322 + 322
volume of the pyramid AD2 = 2048
380
AD = 2048 = 45.25cm 1
First, we find ED = AD
2
1
Thus, ED = (45.25) = 22.63cm In ACD (right angled) : AD2 = AC2 + CD2
2
AD2 = 172 + 172
In right – angled OED : OD2 = ED2 + OE2
402 = (22.63)2 + OE2 AD = 578 cm
1600 – 512.12 = OE2 Thus, ED =
1
578
1087.88 = OE 2
In ODE (right angled)
OE = 32.98cm  33.0cm to 1d.p O
(b) Total surface area
= square base area + Area of 4 triangles
It is so since the base is square and all the sides are equal 27cm
1
Area of ODB = base  height
2 A
O
E O
578 cm
2 D
40cm 40cm
578
tan  = 27 
2
D B tan  = 27  12.02
16cm K 16cm
32cm tan  = 2.246
 = tan – 1 2.246 = 66.00
OD 2 = DK2 + OK2
402 = 162 + OK2 2005/8 Neco Adjusted
O2
1600 – 256 = OK2
1344 = OK thus, 36.66cm = OK
Total surface area = 322 + 4(1/2  32  36.66)
A 12cm B
= 1024 + 2346.24 = 3370.24cm2 1 0c
m

2015/8b (Nov) O1 5cm O3


A silo, in the form of a pyramid on a square base, has
height 27cm and volume 2601cm3. Calculate the: D C
(i) length of a side of the base of the silo, correct to
3 significant figures.
(ii) angle between a sloping face and the base of the O4
silo,
correct to the nearest degree The diagram above shows the net of a right rectangular
Solution pyramid in which ABCD is base and O1, O2, O3, O4 are
O brought together to form the vertex O. when the solid is
formed, ABCD is the base |AB| = 12cm, |AD| = 5cm and
|BO3| = 10cm
(a) Draw a sketch of the pyramid
(b) Calculate the:
(i) height of the pyramid
27cm

(ii) volume of the pyramid


B (iii) area of triangle AOB
A E (iv) inclination of triangle BOC to the base ABCD
D
C Solution
1 O
(i) Volume of pyramid = base area  height
3
Base area is square (L2)
1
2601 =  L2  27
3
10

2601 = 9L2
cm

2601
= L2 A
B
9
289 = L Thus 17cm = L 5cm E
C
(ii) ODA = ODE D 12cm

381
(b) i. Height OE can be gotten from EC and EC from AC 2014/5 (Nov) Exercise 21.29
In right – angled ADC: O
AC2 = 122 + 52
AC = 169 = 13cm
13
 EC = cm = 6.5cm

13
2

cm
In right – angled OEC: OC2 = OE2 + EC2
102 = OE2 + 6.52
Z Y
100 – 42.25 = OE2
M
57.75 = OE 6cm
X
7.6cm = OE W 8cm
1 The diagram shows a right pyramid with a rectangular base WXYZ
(ii) Volume of pyramid = base area  height and vertex O. If |WX| = 8cm, |ZW| = 6cm and |OX| = 13cm,
3
calculate the:
Base area here is rectangle (length  breadth) (a) height of the pyramid
1 (b) value of OXZ, correct to the nearest degree
=  12  5  7.6 = 152cm3
3 (c) volume of the pyramid
(iii)
O 2010/41 Exercise 21.30
16cm
T
10cm 10cm

P 8cm Q
A B
6cm W 6cm
12cm X 5cm U 13cm
1
Area of AOB = base  height S R
2
But height OW can be gotten from
right – angled OWB : OB2 = OW2 + WB2 W
102 = OW2 + 62
100 – 36 = OW2 The diagram is a net of a right – rectangular pyramid. Calculate the
total surface area
64 = OW
A 208cm2 B 112cm2 C 92cm2 D 76cm2
8cm = OW
1 2006/7 NABTEB (Nov) Exercise 21.31
 Area of AOB =  12  8 = 48cm2 VABCD is a solid pyramid on a square base ABCD has vertex V.
2 The height of the pyramid, OV, is 12cm and the length AB is 10cm
(iv) V
O

7.6cm
12cm

10cm

C
D O
E O
6. 5 B
cm
C A 10cm
7 .6 Calculate the: (a) total surface area and (b) volume of the pyramid
From the sketch; tan  =
6 .5
tan  = 1.169 2005/14 (Nov) Exercise 21.32
 = tan– 1 1.169 = 49.460 The base of a solid pyramid is a square of side 6cm.
If the height of the pyramid is 7cm, calculate the volume of
2014/9b Neco (Dec) Exercise 21.27 the pyramid
The base of a pyramid is 6.5m by 3.5m rectangle and its A 84cm3 B 126cm3 C 198cm3 D 252cm3
height is 7m. Calculate its volume
2006/10 Neco (Dec) Adjusted Exercise 21.33
2009/33 Neco (Dec) Exercise 21.28 If a rectangular based pyramid VABCD is such that
A pyramid 7cm high stands on a square base 12cm. |AB| = 10cm, |BC| = 6cm and the slant height is 15cm,
Calculate the volume of the pyramid calculate the: (i) vertical height of the pyramid,
A 246cm3 B 336cm3 C 433cm3 (ii) angle face VBC makes with the base and
3 3
D 504cm E 528cm (iii) volume of the pyramid (iv) total surface area
382
Frustum
This is a shape obtained by cutting off the upper portion
of a pyramid or a cone.
E.g A bucket is a frustum of a cone while some flower
vase are frustums of a pyramid.

Comparing frustum of cone solving to frustum of pyramid


2014/44 NABTEB 2000/24 UME
A frustum of a pyramid with square base has its upper and
lower sections as squares of sizes 2m and 5m respectively
and the distance between them 6m. Find the height of the
pyramid from which the frustum was obtained.
A 8.0m B 8.4m C 9.0m D 10.0m
Solution
2m
The figure is a part of a cone with top radius 4cm
and bottom radius 8cm. If the slant height is 5cm, 6m
calculate the vertical height
A 3cm B 4cm C 5 cm D 8cm 5m Problem diagram
Solution
A

X
y
2m
4cm
B C
6m

5cm 5m Solution diagram


Height of the pyramid is x + 6m
D E By similar triangles
8cm
2 x
Triangles ABC and ADE are similar =
5 x+6
BC AC
= 2(x + 6) = 5x
DE AE 2x + 12 = 5x
4 y 12 = 5x – 2x
=
8 y+5 12 = 3x
4(y + 5) = 8y 4m = x
4y + 20 = 8y Thus, height of the pyramid is 4m + 6m = 10m ( D )
20 = 8y – 4y
20 = 4y thus 5 = y 2005/48 NABTEB (Nov) Exercise 21.34
By Pythagoras rule Find the height of the cone cut off to leave
Vertical height : AD2 = AE2 – DE2 the frustum below
4cm
= 102 – 82
= 100 – 64
= 36
8cm
AD = 36 = 6cm
By similar triangles
AB AD
= 6cm
4 8
AB = 6  4 = 3 cm A 8cm B 14cm C 16cm D 18cm
8
Vertical height = BD Exercise 21.35
= AD – AB A frustum of a pyramid with square base has its upper and
= 6cm – 3cm = 3cm (A) lower sections as squares of sizes 7m and 13m respectively
and the distance between them 10m. Find the height of the
pyramid from which the frustum was obtained.
383
Comparing frustum of cone solving to frustum of pyramid

2005/9b (Nov) Example FSP2


6cm A frustum of a squared base pyramid is 4m at the top and
12m at the bottom. If its height is 10m, calculate the :
(i) height of the whole pyramid
(ii) volume of the frustum, correct to the nearest whole number
10cm Solution
Problem diagram
4m
8cm

The diagram shows a frustum which is part of a solid 10m


cone. The radii of the top and bottom are 6cm and 8cm
respectively. Calculate the:
12m
(i) height of the whole cone A
(ii) volume of the frustum,correct to the nearest whole number
Solution
We draw the complete cone as shown below X
A

B 4m C
x

10m
6cm
B C

10cm
D 12m E Solution diagram
( i ) Height of the whole pyramid = x + 10m
By similar triangles
D E
8cm 4 x
=
Triangles ABC and ADE are similar 12 x + 10
AB BC 4(x + 10) = 12x
= 4x + 40 = 12x
AD DE
x 6 40 = 12x – 4x
= 40 = 8x
10 + x 8
5m = x
8x = 6(10 + x) Thus, height of the whole pyramid is 5m + 10m = 15m
8x = 6(10 + x)
8x = 60 + 6x (ii) Volume of a frustum = vol of big – vol of small pyramid
8x – 6x = 60
= vol of ADE – vol of ABC
2x = 60
x = 30cm 1
Recall that: volume of pyramid = base area  height
Thus height of whole cone = 10 + x 3
= 10 + 30 = 40cm 1 1
= × 122 ×15 – × 42 × 5
3 3
1 = 720m3 – 26.67m3
(ii) Volume of a frustum = h ( R2 + Rr + r2 )
3 = 693.33 m3
Here h is10, r is 6 and R is 8. Substituting, we have  693 cm3 to the nearest whole number
1 22
= × ×10( 82 + 8×6 + 62 ) 2005/10a NABTEB Exercise 21.36
3 7
A bucket is 28cm in diameter at the top, 18cm in diameter at
1 22
= × ×10( 64 + 48 + 36 ) the bottom and 20cm deep. Find the capacity, in litres, of the
3 7 bucket. (Take  = 3.142)
1 22
= × × 10 × 148
3 7 2015/6b Neco Exercise 21.37
32560 A flower vase 8cm high is in the shape of the frustum of a
= square base pyramid of side 6cm at the bottom and 10cm at
21
the top. What is the volume of water that will fill the vase
= 1550.476cm3
when it is empty?
 1550 cm3 to the nearest whole number

384
IRREGULAR SOLIDS 2003/12 Neco (Dec)
Irregular solids do not have any particular formula used; A drum of radius 50cm and height of 1.2m is to be filled
we apply the relevant solid formula as it affects a given with water using a bucket of radius 15cm and 10cm for top
shape. The use of common sense in interpreting and bottom respectively and height of 50cm. How many
diagrams is crucial so as to know the applicable formula buckets filled with water will fill
the drum to the nearest whole bucket (Take  = 22/7)
2006/11a Solution
V
Drum height 1.2m = 120cm

15cm
5m
A

120cm
B F
2m
C E
G 50cm 10cm
4m 3m
D Volume of drum(cylinder) = r2h
The diagram shows a pyramid standing on a cuboid. 22
=  502  120
The dimensions of the cuboid are 4m by 3m by 2m and 7
the slant edge of the pyramid is 5m. Calculate the = 942857.14cm3
volume of the shape Volume of bucket (frustum)
Solution 1
= h (R2 + Rr + r2)
Volume of shape = volume of pyramid + volume of 3
cuboid 1 22
=   50 (152 + 15  10 + 102)
1 3 7
Volume of pyramid = base area  height
3 1100
= (225 + 150 + 100)
Here base area is rectangle (length  breadth) 21
Our height h can be gotten as: 1100
= (475) = 24880.95cm3
V 21
Volume of drum
Number of buckets =
Volume of bucket
h 5m 942857.14
A =
24880.95
B = 37.89 buckets  38 buckets
O F 2009/40
2m
A bucket holds 10 litres of water. How many buckets of
C E
G water will fill a reservoir of size 8m  7m  5m.
4m 3m (1 litre = 1000cm3)
D A 28 B 280 C 2800 D 28000
H VO can be gotten from OG and OG from AG Solution
Bucket
In right – angled ABG : AG2 = BG2 + AB2
AG2 = 42 + 32 Reservoir
7m
AG = 25 = 5m
1 5m
Thus, OG = AG = 2.5m
2 8m
In right – angled VOG : 2
VG = VO + OG 2 2
Volume of bucket (frustum) = 10  1000 cm3
52 = VO2 + 2.52 = 10,000cm3
25 – 6.25 = OV2
Volume of Reservoir (cuboid) = xyz
18.75 = OV = 800  700  500cm3
4.33m = OV (Height) (Note: 100cm is 1m)
Volume of shape = 280,000,000cm3
1 Number of buckets = Volume of reservoir
= base area  height + length  breadth  height
3 Volume of bucket
1 280,000,000 = 28, 000 buckets
=  4  3  4.33 + 4  3  2 =
3 10,000
= 17.32 + 24
= 41.32m3
385
2014/17 (Nov) 1976/8(Nov)
P
A rectangular tank measuring 11m by 2m by 7m is
equal in volume to a cylindrical tank of height 4m.
Calculate the radius of the cylindrical tank
(Take  = 22/7)
A 14.00m B 7.00m C 3.50m D 1.75m
Solution
D
Volume of rectangular tank (cuboid) = 11  2  7 A C
= 154m3
Volume of cylinder = r2h B

154 =
22
 r2  4 In the figure, a hoop of radius 80cm is suspended
7 horizontally by four strings each 160cm long and each
154  7 attached to a nail vertically above the hoop at P. The
= r2
22  4 strings are attached to points A, B, C, D which are
12.25 = r2 equally spaced on the hoop. Calculate:
12.25 = r thus, 3.50m = r ( C ) (a) the angle which PA makes with the horizontal.
1975/9 (Nov) (b) the angle BPD
From a cylindrical object of diameter 70cm and height Solution
84cm, a right solid cone having its base as one of the Let M be the centre of the hoop. Then we sketch as:
P
circular ends of the cylinder and height 84cm is
removed. Calculate :
(a) the volume of the remaining solid object, expressing
your answer in the form a × 10n where 1< a<10 and

m
0c
n is a positive integer
16
(b) the surface area of the remaining solid object
D
Solution C
A
Height h is 84cm, radius is diameter ÷ 2 i.e 70÷ 2 = 35cm 80cm M
We sketch as :
B
PA = PB = PC = PD and AB = BC = CD = DA(Given)
Thus, PM is perpendicular to the horizontal hoop
PMA = 900.
AM 80
L In PAM, cos A = =
84cm

PA 160
1
cos A =
2
35cm A = cos– 1 0.5
Volume of solid cylinder = r2h A = 600
Volume of solid cone = 1/3 r2h PA makes an angle of 600 with the horizontal.
1 2 (b) In BPD , BP = DP = 160cm
Thus volume of remaining solid = r2h – r h
3 DB = diameter of hoop = 2× 80 = 160cm
=
2 2
r h Hence BPD is equilateral.
3  angle BPD = 600
2 22
= × × 352 × 84 = 215600 cm3
3 7 2012/11b
= 2.156 × 10 cm 5 3 A cylinder with base radius 14cm has the same volume as a
Surface area of the remaining solid object = cube of side 22cm. Calculate the ratio of the total surface
Inner surface area of cone shape left behind + outer surface area of the cylinder to that of the cube.
area of a cylinder + area of circular outer top (Take  = 22/7)
= rL + 2rh + r2 Solution
= r( L + 2h + r ) First, we get the height h of cylinder from its value of
L can be gotten from the right- angled  in the diagram volume in cube
L2 = 842 + 352 Volume of cube = x3
L2 = 7056 + 1225 = 223
L = 8281 = 91cm = 10648cm3
Surface area of the remaining solid object = r( L + 2h + r ) This is the same volume for the cylinder
22 Next, volume of cylinder = r2h
= × 35 ( 91+ 2× 84 + 35) = 32340cm2
7

386
22 Solution
10648 =  142  h Surface area of solid
7
10648  7 = surface area of cone + surface area of hemisphere
= h = rL + 2r2
22  14 2
But slant height (L)2 = 72 + 242
17.29cm = h
Total surface area of Cylinder : Total surface area of cube = 49 + 576
2r(r + h) : 6 x2 L2 = 625
22 L = 625 = 25cm
2  14 (14 + 17.29) : 6  222
7 22 22
Surface area of solid = × 7 × 25 + 2 × × 72
2753.52 : 2904 7 7
2754 : 2904 = 550 + 308
1 : 1 = 858cm2
2011/10a 2015/12
The total surface areas of two spheres are in the ratio A water reservoir in the form of a cone mounted on a
9 : 49. If the radius of the smaller sphere is 12cm, find hemisphere is build such that the plane face of the
correct to the nearest cm3 the volume of the bigger sphere hemisphere fits exactly to the base of the cone and the height
Solution of the cone is 6 times the radius of its base.
Surface area of sphere = 4r2 (a) illustrate this information in a diagram
Small sphere : bigger sphere (b) if the volume of the reservoir is 3331/3cm3
4r2 = 9 : 4R2 = 49 calculate, correct to the nearest whole number, the:
surface area of small sphere 9 (i) volume of the hemisphere
=
surface area of bigger sphere 49 (ii) total surface area of the reservoir (Take  = 22/7)
4r 2 9
Solution
=
4R 2 49
r2 9
= L
R 2 49 6r
Substituting for r = 12cm,
12 2 9 r
=
R 2 49
12 2  49 = R2
9
Volume of reservoir = volume of cone + volume of hemisphere
R = 12  7
2 2
1 2
32 = r2h + r3
3 3
12 7
R= = 28 cm 1 2 3
3 3331/3 =  ( r2  6r + r)
3 3
4 3 Pi will cancel out
Thus volume of bigger sphere = R
3 1000 6 3 2 3
= r + r
4 22 3 3 3
=   28 3 = 91989.33cm3
3 7 1000 8 3
= r
 91989cm3 to nearest cm3 3 3
1
2012/13b /3 will cancel out
1000 = 8r3
1000
= r3
24cm
8
1000
3 = r
7cm 8
10
= r
2
r = 5cm
The diagram shows a wooden structure in the form of a 2 3
( i ) Volume of hemisphere = r
cone mounted on a hemispherical base. The vertical 3
height of the cone is 24cm and the base radius 7cm. 2 22
calculate , correct to 3 significant figures, the surface =   53
3 7
area of the structure = 261.90cm3
(Take  = 22/7)

387
(ii) Total surface area of reservoir 2004/12b (Nov) Exercise 21.45
= surface area of cone + surface area of hemisphere
= rL + 2r2
But L2 = (6r) 2 + r2
14cm
L2 = 302 + 52
L = 925 = 30.41cm
22 22 6cm
Total surface area of reservoir =  5  30.41 + 2   5 2
7 7
= 477.87 + 157.14
The diagram show a hollow cone dug out of a stone block.
= 320.73cm2
The diameter of the level of water inside the cone is 14cm
2005/3b Exercise 21.38
and the height is 6cm. If 250cm2 of water is added to the
A cone and a right pyramid have equal heights and
water inside the cone, calculate, correct to one decimal place,
volume. If the area of the base of the pyramid is
the rise in water level. (Take  = 22/7)
154cm2, find the base radius of the cone (Take  = 22/7)
2014/41 Neco Exercise 21.39 2004/8 Neco Exercise 21.46
The volume of a frustum of a cone of height, hcm is given
h
by the formula  ( R 3 − r 3 ) where R and r are radii of its
3
4cm circular ends. If an open- ended metal pipe of external and
internal diameters of 20.5cm and 10.5cm respectively and
length 21cm are to be cast from a molten contents of the
3cm frustum of a cone of depth 1.5m and radii 0.5m and 0.3m.
Calculate the number of pipes that will be cast to the nearest
whole number (Take  = 22/7)

The total surface area of the above shape in terms of  is 1986/20 Exercise 21.47
A 6cm2 B 18cm2 C 21cm2 D 27cm2 E 33cm2 The diagram represents a lampshade whose top PQRS and
bottom UVWX are squares of sides 12cm and 20cm
2015/15 Adjust ed Exercise 21.40
respectively.
The dimensions of a rectangular tank are 2m by 10m by 12cm
11m. If its volume is equal to that of a cylindrical tank P S
of height 7m. Calculate the base radius of the
12cm
cylindrical tank (Take  = 22/7 )
A 14m B 10m C 3 1 / 2m D 1 3/ 4 m R
Q

1976/9a (Nov) Exercise 21.41


A solid cube of side 8cm is dropped vertically into a
cylindrical tank of radius 7cm. Calculate the rise in the U X
water level if:
( i ) the original depth of the water was 9cm 20cm
( ii ) the original depth of the water was 5cm
2004/13b (Nov) Exercise 21.42 V 20cm W
Two tanks, A and B contain 300 litres and 800 litres of
water respectively. If 3x litres is drawn from B and If the vertical depth is 14cm, calculate, correct to the nearest
x litres added to A, then B contains twice as much as A, whole number:
Find: (i) the value of x (a)the area of the material used in making the lampshade
(ii) the quantity of water each tank now has (b) the volume of the pyramid of which the lampshade is a part

2000/10a (Nov) Exercise 21.43


A Cylindrical tank of radius 35cm contains water 20cm
high. The water is poured into a barrel of rectangular
base 42cm by 30cm. Find, correct to two significant
figures, the height of the water level in the barrel.
(Take  = 22/7)
2000/8 Neco Exercise 21.44
A solid cone of height 56cm and base diameter 51cm is
formed from a cylindrical drum equal in height and
diameter with the cone. Find the volume of the
remaining part of the drum
388
CHAPTER TWENTY TWO I: With center B and a suitable radius draw an arc to cut AB
at Q and BC at P
Construction A
Foundation facts on construction A
To construct any given shape say triangle, we need
A compass, protractor, sharp pencil, very neat ruler Q
and a good eraser. All construction lines must not be Q
thick. Do not erase any arc or guiding lines to your final
result. Always confirm your lines or arc measurements B
with ruler or compass as the case may be B P C P
C
Construction of angle 900 at a given point C Where this arc already exist maintain it to step II
Draw a line AB II: With centers at “P” and “Q” each at a time; using the
A B same radius, draw arcs to intersect at point y.
A
With center at the point C, on line AB open your
compass to a suitable radius, cut arc twice on line AB at A
points E & F y Q y

A B Q
E C
F B
P
Changing or adjusting our compass to a bigger radius B P C C
wider than CF or CE. Next, make E and F our centers,
one at a time draw an arc to cut each other at top of C as III: Join B to Y. This is the Bisector
shown below: A
G
A
y Q
y
Q
B
A B
E C
F y P
Join point of intersection G to line AB at C, we have B P C C
GCB = 900 and GCA = 900 as shown below.
Confirm it with the use of a protractor.
G
Construction of 600
Step I : Draw line AB
A B

Step II: With center at A and a suitable radius draw


A C B an arc XY to cut AB at Y
E F
X

A Y B
Step III : With center at Y and same radius as before, draw
To bisect any given angle ABC an arc to cut XY at Z
Z
The angle could be Fig I or Fig II
A X
A

Fig I Fig II
A Y B
B
B C
C

389
Step IV: Join AZ . Angle ZAB = 600 (d) Join YZ and ZX using a ruler
Lastly, line ZD as instructed : with centre X and radius
4.5cm draw an arc to cut XY at D.Then join D to Z
Z To construct a triangle, given two sides and one of
X its angles
2002/5 Delta
Using a ruler and a pair of compasses only,
A Y B (a) Construct ∆ PQR such that PQ = 6.5cm,
QR = 8 cm and  PQR = 450
Construction of 1200 (b) Measure  QPR
We use same steps as in 600, but for step IV Solution Diagram on page 391
Do not join A Z Rather, Steps taken are:
Step IV: With same radius as before and center Z draw (i) Draw QR = 8 cm
another arc to cut XY at W. (ii) At “Q” construct angle Q = 450 i.e bisecting of 900 and
W Z measure out line QP = 6.5cm through it to P
(iii) With center Q and radius 6.5cm draw an arc to cut QP.
(iv) Join P to R
X
A Y B 2000/5 Delta
Using a ruler and a pair of compasses, construct a ∆PQR
Step V: Then join AW. Angle WAB = 1200 such that PQ = 9 cm, PQR = 600 and QR = 10 cm
Measure (i) PR (ii) QPR
Z Solution Diagram on page 391
W
For ease in measuring  QPR and constructing
 PQR = 600 ; we let Q and P be on the floor.
X Steps taken:
A Y B (a) Draw side PQ = 9 cm
(b) At point Q, construct Q = 600
Construction of triangles with given length of the (c) Measure out QR = 10 cm
three sides (No included angles) (d) Join PR
The steps we follow here are the same except for
change in length based on the question. 2004/1 FGC ( JSS Neco ) Exercise 22.1
Sketching is an important part of construction; so Construct ∆ ABC in which AB = 9cm, BC = 12cm
always remember to do so before the construction and ABC = 600
proper. ( i ) Construct the bisector of angle A and let it
1997/5a Delta meet BC at D.
Construct a triangle AB with /AB/ = 6cm, ( ii ) Measure DC
/BC/ = 8cm and /AC/ = 10 cm
Measure angle ABC Construction of a triangle, given two angles and
Diagram on page 391
one side
Solution
(a) Draw line AB = 6 cm 1997/5b Delta
(b) With center B and radius 8 cm, draw an arc Construct a triangle PQR with /PQ/ = 10 cm
(c) With center A and radius 10 cm, draw an arc to  RPQ = 300,  PQR = 600. Measure PR.
intersect the previous one at C.
Solution Diagram on page 391
(d) Join AC and BC using a ruler
Our Sketch must be reasonable enough to allow us construct
Angle ABC = 900 using a protractor to measure it.
the two given angles P and Q on the floor.
1994/5 Delta Steps taken are:
Construct a triangle XYZ in which XY =10 cm,YZ =6 cm (a) Draw PQ = 10cm
and ZX = 11 cm. On XY mark a point D such that (b) At P construct angle P = 300 i.e bisecting of 600
XD = 4.5 cm. Draw ZD, measure the length of ZD. (c) At Q construct angle Q = 600
Solution Diagram on page 391 (d) The point of intersection of these lines is R.
We have something to do with XY, So we let it lie on 2004/5 Delta Exercise 22.2
the floor . Steps: Construct ∆XYZ such that X = Z = 450 and XZ = 8 cm
Draw a line XY = 10 cm (b) Measure: (i) side XY and (ii) angle Y
(b) With center X and radius 11 cm draw an arc
(c) With center Y and radius 6 cm draw an arc to 2004/3 Osun Exercise 22.3
intersect the previous one at Z Using a pair of compasses and a ruler only,

390
A

1997/5a Delta 1994/5 Delta


o /ZD/ = 7.6cm
ABC = 90

X D Y
R
B C

2000/6 Delta
2002/5 Delta
/PR/ = 9.5cm
QPR = 60
o QPR = 65o

Q R
P Q

2004/5 Delta

1997/5b Delta
/PR/ = 5cm

X Z
o
XYZ = 90 /XY/ = 5.6cm

P Q

391
construct a triangle PQR in which /QR/ = 10cm , 5 : Join line AW and we have that  BAW = 150
PQR = 600 and  PRQ = 300
( i ) measure /QP/ ( ii ) measure /PR/ Z
K
( iii ) measure QPR X P
( iv ) What type of triangle is PQR W

SPECIAL ANGLES, AND LOCUS A Y B


Under this sub topic we shall treat construction of
150, 750(60+15),1050(90 +15),or 1050(60+30+15), Note that KAW is also 15 0

1350(120+15), or 1350(90 + 45),1500(120 +30). Readers to bisect KAZ = 300 at step5 to give 150 each as
shown below
Construction of 150
When we bisect angle 600, we get 300.Further bisecting Z 15o
of 300 gives 150 K
15o
Steps taken :
1: Using the steps given earlier on the construction of
angle 600, we have:
Z A Y
X Most of the special angles is based on one or two manipulations of the above
bisectings

A Y B Construction of 750
Construction of 750 is basically construction of two 600
(1200) and bisecting of one angle 600 to give 300 then further
bisecting to give 150
2:Using same radius and centers Z and Y bisect Steps taken:
ZAY as shown below: 1:We construct two angles 600 (or 1200 showing two arcs)
Z D A
K
X

A Y B B C

3 : Join AK. Angle BAK is 300 .Also angle ZAK is300 ABC = 600 and ABD is the other 600 to be bisected

Z 2 : Using same radius we bisect angle ABD = 600 at E to


K give 300. Also join EB. Note that centers here are D and A
X
E

A Y B
D
A
We are to either bisect angle BAK or ZAK
To make the work neat is always good to use an already
made arc in the construction. The visible one here is arc B C
YZ.If you have adjusted your compass before; ensure
that you bring it back to the said arc YZ with centre at 3 : Using same radius, we bisect angle ABE = 300 at F to
A on line AB. give 150 . Centres here are A and the point of intersection
between BE and arc AD
For emphasis sake we will note a point P on line AK i.e F
where the line touches the arc YZ E
4 : Using same radius and with centers at P and Y
bisect
BAK as shown below
D
A
Z
K
X P
W
B C
FBC = 750
A Y B
392
ALTERNATIVE STEP 3
From step2, using same radius we can bisect angle X Z
EBD = 300 to get 150(With centers at D and point of C
intersection between line EB and arc AD) o
N 135
E
D A B
We can choose to bisect ZAB to give 300 further
D bisecting to get 150 . BAF = 1350
A
1350( 90 + 45 )
First we construct angle 900at A
M B C
MBN = 750

Construction of 1050
Construction of two 600 (1200) and bisecting of one
angle 600 to give 300 each .The last of the 300 is
bisected to give 150
so we have 600 +300 +150 = 1050
Steps taken are as shown above in construction of 750 D A B
F
E Secondly, we bisect one part to get 450
C

D
A

Y B C D A B
YBF = 1050 OR CBN = 1050 shown below: CAB = 1350
N
E
Construction of 1500
1500(120 + 30).
We construct angle 1500 by bisecting the third part in 1200
D
A
C

o
M B C 1
50

D A B
Construction of 1350
1350(120 +15), or 1350(90 + 45) LOCUS
Locus has plural form loci. It can be defined as a set of
1350(120 +15) case, points satisfying or obeying a given condition (rule)
First we construct angle 1200 then bisect the last part There are basically three loci
X Z 1.LOCUS OF A POINT (IN A PLANE) WHICH IS
C EQUIDISTANT FROM A GIVEN POINT.
It is a circle at that point with radius equal to the fixed
distance
D A B 2. LOCUS OF A POINT (IN A PLANE) WHICH IS
EQUIDISTANT FROM TWO GIVEN POINTS.
It is the perpendicular bisector of the line between those two
Secondly, we bisect XAC = 300 to give 150
points
3. LOCUS OF A POINT (IN A PLANE) WHICH IS
EQUIDISTANT FROM TWO INTERSECTING
STRAIGHT LINES.
It is the bisector of the angle between them

393
In subsequent examples we shall utilize the above locus
as directed by the given questions C
D
PARALLEL LINES
There are basically two ways of constructing parallel
lines namely:
Case of parallel line to given line at a fixed
distance apart.
Firstly we draw two arcs of almost the size of semi A B
circle on the line. It’s radius equal the given distance
say 3cm LINE RATIO CONSTRUCTION
Find by a suitable construction, a point B on AC such
that /AB/ : /BC/ = 3:5 .
Steps taken
1 : Draw a line AK within a reasonable angle AC
2 : Divide AK into 8 reasonable segments
3 : At D draw parallel line to KC using the steps for a
parallel line at point outside a given line
A B
Secondly, we draw a straight line CD on the arcs as
shown below:
C D
C
K

A B

Case of parallel line to given line at a fixed point


outside the line B
First, we are given a line and a point outside the line D
C

A B
Secondly, with radius AB but centre at C cut an arc as ; A

C CONSTRUCTION OF ORDINARY TRIANGLE I


2009/10 (Nov)
Using a ruler and a pair of compasses only. Construct a
triangle ABC, such that AB= 7.1cm, AC = 7cm
and BAC = 1050. Construct the bisector of BAC to meet
BC at X and the bisector of AC to meet AX produce at Y.
A B Measure: (a) XY ; (b) BC
Analysis and solution Diagram on page 396
Thirdly, with radius AC but centre at B draw an arc to Using free – hand, sketch the required diagram showing all
cut the first one. Join CD; CD//AB as required the given data. Don’t wipe out or erase your sketch.
Steps taken
1.Draw a line AC = 7cm with extensions as shown
2.At point A construct 1050 ( i.e constructing 900 between
600 and 1200 then bisecting it to give 150 + 900 )
3. Produce the straight line at 1050 to 7.1cm long to get B
then join B to C

394
4 . Bisect BAC to meet BC at X Construction of triangles with ratios
5. Bisect line AC to meet AX at Y 1995/9
(a)Using a ruler and a pair of compasses only, construct
6 . Measure (a) XY ; (b) BC with a ruler
triangle ABC with /AB/ = 7.5cm, /BC/ = 8.1cm and
2007/6 Neco ABC = 1050
(b) Locate a point D on BC such that /BD/ : /DC/ is 3:2
A triangular plot of land ABC is such thatBC is
(c) Through D, construct a line L perpendicular to BC .
105m, ABC = 450, ACB = 750
(d) If the line L meets AC at P , measure /BP/ .
( a ) With the aid of a ruler and a pair of compasses only
Analysis and solution Diagram on page 397
construct the:
( i ) triangular plot using a scale of 10m to 1cm Steps taken
( ii ) locus l1 of points equidistant from B and C 1: Draw a line AB = 7.5cm with extensions as shown in
the diagram
( iii) locus l2 of points equidistant from AB and BC 2 :At point B construct 1050 and produce a line 8.1cm
( iv ) locus l3 of points 5cm equidistant from A through it to C
( b ) ( i ) Locate T1 and T2 which are the points of 3: Join C to A to complete the triangle
intersection of l1 and l3 4 : Draw a line CK and divide it into 5 reasonable equal
( ii ) Measure T1 T2 segments; join K to B
Analysis and solution Diagram on page 396 5 : Mark the 3rd segment point d on CK
Using free – hand, sketch the required diagram showing 6 : At d construct a parallel line to BK using steps of a
all the given data. Don’t wipe out or erase your sketch. point outside a parallel line and join d to D
Steps taken 7 : At D construct a perpendicular to meet AC at P
1. Draw a line BC = 10.5cm with extensions as shown 8 : Join B to P using a dotted line and measure
2. At point B construct 45 (Bisecting of 90 )
0 0

3. At point C construct 750 ( i.e 600 , 1200 then 900 in 1987/9(a) GCE Exercise 22.7
between, next bisect between 600 and 900 ) (a) Using a ruler, a protractor and a pair of compasses,
4. Produce B and C to meet at A construct a triangle ABC , given that /BC/ = 8.5 cm ,
5. Locus l1 is the perpendicular bisector of line BC /AB/ = 5.1 cm and BAC = 650
6. Locus l2 is the bisector of ABC ( i ) Measure /AC/.
7. Locus l3 is a circle with radius 5cm and centre A (ii) Find by a suitable construction, a point D on AC
8 . Measure T1 T2 with a ruler such that /AD/ : /DC/ = 3 : 5 .
( iii ) Measure angles ABD and CBD .
2014/7b NABTEB(Nov) Exercise 22.4 Author’s hint: Use protractor to get 650\ and not by
Construct a triangle PQR for which PQ= 8cm, construction
PR= 9cm and QPR = 60 . Measure QR
0

( i ) construct the locus of points at a distance of 5cm Construction of triangles with inscribed circle
from P It is done by bisecting any two angles of the triangle. Their
( ii ) construct the locus of a point equidistant from point of intersection is the centre of the circle touching all
P and R the sides of the circle.
( iii ) The locus meet at X. Measure XQ 2006/11 Neco
(a) Using a ruler and a pair of compasses only , construct :
2007/9a NABTEB Exercise 22.5 ( i ) a triangle PQR such that PQ = 8.5cm , PR = 7cm
With a pair of compasses and a ruler only, construct a
and QPR = 750 .
triangle PQR in which RPQ = 1200 , PQR = 450 ( ii ) the locus, L, of points equidistant from PQ and PR
andPQ= 6cm ( iii ) an inscribed circle which lies inside the triangle and
( i ) Find a point M on RQ such that PM is touches PQ , PR and QR.
perpendicular to RQ ( b ) Measure :
( ii ) Measure PM ( i ) QR , ( ii ) the radius of the inscribed circle .
Analysis and solution Diagram on page 397
1979/7 Exercise 22.6
Steps taken
Using a ruler and a pair of compasses only:
1: Draw a line PR = 7cm with extensions as shown in
(a) Construct a triangle ABC such that AB = 6 cm,
the diagram
AC = 8.5 cm  BAC = 1200
(b) construct the locus l1 of points equidistant 2 :At point P construct 750 and produce a line 8.5cm
from A and B through it to Q
(c) construct the locus l2 of points equidistant 3: Join Q to R to complete the triangle
from AB and AC 4 : Construction of locus L at P is the bisecting of angle P
(d) find the point of intersection P of l1 and l2 5 : For ( iii ) to be fulfilled; we must construct bisector of
and measure PC angle R or Q. Here we bisect R and the intersection K is the
centre of the circle touching all the sides
395
395
B 2009/10 (Nov)

7.1cm Y

XY = 1.3cm
BC = 11cm

A 7cm C

T2

2007/6 Neco

L3
T1

L2

B C

T1 T 2 = 7.7cm
L1

396
C
1995/9

L
P
d
D

/BP/ = 5.4cm

K
A B

2006/11 Neco

/QR/ = 9.5cm
K Radius = 2.3cm

P R
397
2006/9(a) Exercise 22.8 2004/9 Exercise 22.9
( a ) Using a ruler and a pair of compasses only : Using a ruler and a pair of compasses only .
( i ) construct XYZ such that /XY/ = 8cm and (a) Construct :
YXZ = ZYX = 450 ; ( i ) PQR such that /PQ/ = 8cm , /PR/ = 7cm and
( ii ) locate a point P inside the triangle equidistant QPR = 1050 .
from YX and YZ ; ( ii ) locus l1 of points equidistant from P and Q .
( iii ) construct a circle touching the three sides of the ( iii ) locus l2 of points equidistant from Q and R
triangle . (b) ( i ) Label the point T where l1 and l2 intersect .
( iv ) measure the radius of the circle . ( ii ) With centre T and radius /TQ/ , construct a
circle l3
Construction of triangles with circumcircle (iii). Complete quadrilateral PQSR such that
It is done by perpendicular bisector of any two sides of /RS/ = /QS/ and /TQ/ = /TS/ .
the triangle. Their point of intersection is the centre of
the circle touching all the vertices of the triangle and it
is of equal distance from all the vertices SCALED, COPIED DIAGRAM CONSTRUCTION
2002/11 ( Nov )
2015/10 Neco The diagram below represents a lawn ABCD where
(a) Construct an equilateral triangle ABC with AB = 45m, BC = 30m, CD = 35m, ABC = 1000 and
sides 8cm BCD = 900.
(b) Construct the perpendicular bisectors of AB and AC D
to meet at point M
(c) Draw a circumcircle to pass through points A, B, C.
(d ) Measure the radius of the circle

35
m
Analysis and solution Diagram on page 399
Steps taken
1. Draw a line AB = 8cm with extensions as shown C
2. With radius 8cm and centre at A draw an arc, A
next with centre at B and same radius draw another
100 0 30m
arc to cut the first one. 45m

3. Join their point of intersection C to A and B


B
4. With centre at A, radius beyond centre AB draw (a) Using a scale of 1cm : 5m draw an accurate
an arc at top and bottom of AB diagram of the lawn .
5. With same radius and centre B, draw an arc to cut the (b) Construct the locus:
set of arc in 4.
( i ) l1 of points equidistant from AD and DC ,
6. Join their point of intersection
( ii ) l2 of points 25m from B
7. Repeat steps 4, 5 and 6 at AC
(c) Shade the region of all points in the lawn which are
8. The point of intersection forms the centre of the
not more than 25m from B and nearer AD than DC .
circumcircle with radius A or B or C
Analysis and solution Diagram on page 400
2007/9 Steps taken
(a)Using a ruler and a pair of compasses only , 1 : Scale conversions- AB = 9cm from 45m,
Construct : BC = 6cm from 30m, CD = 7cm from 35m
(i) A triangle PQR such thatPQ= 10cm, 2 : Draw a line AB = 9cm with extension as shown in
the diagram
QR = 7cm and PQR = 900.
(ii) the locus l1 of points equidistant from Q and R;
3 : At B measure out 1000 with your protractor.(Do not
(iii) the locus l2 of points equidistant from P and Q construct) and draw line BC = 6cm through it
(b) Locate the point O equidistant from P, Q and R 4 : At C measure out 900 with protractor and draw
(c) With O as centre, draw the circumcircle of the line CD = 7cm through it
triangle PQR 5 : Join D to A to complete diagram
(d) Measure the radius of the circumcircle
6 : Construction of locus l1 is the bisector of
Analysis and solution Diagram on page 399 angle D with the aid of compass and ruler
Steps taken 7 : Construction of locus l2 is a circle with
1. Draw line QR = 7cm with extensions as shown radius 5cm( from 25m) and centre B
2. At Q, construct 900 with extension to P 10cm The region is shaded in the diagram
3. join P to R
4. locus l1 is the bisector of line QR
5. locus l2 is the bisector of line PQ
6. Point O is the intersection of l1 and l2, also the centre
of the circumcircle
398
2015/10 Neco C

Radius 4.6cm

A B

P
2007/9

10cm

O L2

Q 7cm R

Radius = 6.1cm
399
L1
B
1986/9 GCE

P
K
Q

A C

/QP/ = 2.3cm i.e 23m

2002/11 ( Nov )
C
L1

L2

A B

400
2005/10 Exercise 22.10 Construction of quadrilaterals
The sketch shows a plot of land. The basic ideas about the properties of quadrilaterals are
C useful in their construction. We will break down this
subtopic as:
*Triangles to quadrilateral construction
*Direct quadrilateral: whose name say trapezium,
parallelogram, rhombus etc. is given in the beginning of
the question
40 0 95 0 *Indirect quadrilateral; whose name is not mentioned.
A B In this case just be guided in your sketch by the fact that
85m
it is a FOUR SIDED FIGURE . Don’t be bias to sketch
( a ) Using a scale of 1cm to 10m draw an accurate in favour of any quadrilateral you know; let only the
diagram of the plot .
question’s conditions be your guide
( b )Construct :
(ii) The locus l1 of points equidistant from AC and BC .
(ii) The locus l2 of points 60m from A.
( c ) A tree T inside the plot, is on both l1and l2. Triangles to quadrilaterals construction
Locate T and find /TC/ in metres .
2016/10
( d ) A flag pole , P is to be placed such that it is Using a ruler and a pair of compasses only ,
nearer (a) construct :
AC than BC and more than 60m from A . (i)XYZ such that XY = 10cm , XYZ = 300
and YXZ = 450 ;
Shade the region where P can be located .
(ii) locus l1 of points equidistant from Y and Z;
(iii) locus l2 of points parallel to XY through Z
1986/9 GCE (b) locate point M, the point of intersection of l1 and l2
A plot of land is in the form of a triangle ABC in which (c) Measure ZMY
/AB/ = 121m , /AC/ = 105m and ACB = 750. Analysis and Solution Diagram on page 402
Using a ruler and pair of compasses only, and a scale Steps taken
of 1cm to 10m 1. Draw line XY = 10cm with extensions as shown
( a ) construct : 2. At Y construct angle 30o (Bisecting of 600)
( i ) the triangle ABC , 3. At X construct angle 45o (Bisecting of 900)
( ii ) the position of a flagpole P within ABC 4. Join the lines to meet at Z
which is 85m from A and 40m from AC ; 5. locus l1 is the perpendicular bisector of line YZ
( iii ) the position of another flagpole Q which is 6. locus l2 is a parallel line to a given line at a fixed point
equidistant from A, B and C; outside the line
( b ) find the distance between the flagpoles P and Q 7. With radius XY and centre Z draw an arc
Analysis and solution Diagram on page 400 8. With radius YZ and centre X draw an arc to cut the 1st one
Steps taken 9. Join Z to the point of intersection and produce to meet l1 at M
1 : Scale conversions- AB = 12.1cm from 121m,
AC = 10.5cm from 105m
2 : Draw a line AC = 10.5cm with extension as shown 1998/9
3 : At point C construct angle 750 and produce a line Using a ruler and a pair of compasses only , construct
through it to a reasonable extent (a) a triangle QRT with /QR/ = 8cm , /RT/ = 6cm
4 : With centre at A and radius 12.1cm draw an arc to and /QT/ = 4.5cm
cut the line drawn through 750 at point B (b) a quadrilateral QRSP which has a common base QR
5 : Join B to A to complete the triangle with QRT such that QTP is a straight line . PQ//SR,
6 : Construction of condition ( ii ) /QP/ = 9cm and /RS/ = 4.5cm .
First; Measure CK 4cm(40m) through 900 line as ( i ) Measure /PS/ .
shown. ( ii ) Find the perpendicular distance between RS and PQ.
Secondly, at K construct a parallel line to AC using ( iii ) What is QRSP ?
steps of point outside a parallel line Analysis and Solution Diagram on page 402
Thirdly, with centre at A and radius 8.5cm(85m) draw Steps taken:
an arc to cut the parallel line at P 1. Draw a line QR = 8cm with extensions as shown
7 : Construction of condition ( iii ) means the point of in the diagram.
intersection of constructed perpendicular bisectors of 2. With center Q and radius 4.5cm draw an arc.
two sides of the triangle as shown in the diagram 3. With center R and radius 6cm draw another arc to cut
the former one at T.
4. Join T to Q and T to R to complete the diagram.
401
L1 2016/10

Z L2
M

Y 10cm X
ZMY = 120 0

P 1 9 9 8 /9

/PS/ = 6cm
( ii )Perpendicular bisector = 5.9cm

QRSP is a trapezium
T
S

Q R

W Z0

2015/8 Q

8cm
Z
P cm
8. 3

QX = 6.7cm
X Y XWZ = 780
5.5cm
402
5. Produce T to P 4.5cm long since QP = 9cm Analysis and Solution Diagram on page 404
and QT = 4.5cm Steps taken
6. At P construct a parallel line to TR using step of a 1. Draw line PQ = 9cm with extension as shown
point outside a parallel lines 2.At P construct angle 60o
7. Produce R 4.5cm long to meet the parallel line at S. 3.To draw PQ//SR that is 5cm apart: with radius 5cm
8. To fulfill condition (ii) we construct a perpendicular and two centres that are not too close on line PQ
bisector on line TP and produce it to touch RS 4. Carefully draw a straight line SR = 6cm touching the
tops of the two arcs and parallel to line PQ
1997/7 Exercise 22.11 5. Join R to Q.
(a) Using a ruler and a pair of compasses only , 6. locus l1 is the perpendicular bisector of line PQ
construct ABC in which /AB/ = 7cm , /BC/ = 5cm
7 locus l2 is a circle with radius 7cm and centre P
and ABC = 750 . Measure AC .
(b) In (a) above, locate by construction , a point D such 1977/5 Exercise 22.12
that CD is parallel to AB and D is equidistant from ( a ) Using a ruler and a pair of compasses only ,
points A and C . Measure BAD construct a trapezium ABCD, in which the parallel
sides AB and DC are 2cm apart. DAB = 600,
Direct quadrilateral construction AB = 4cm and BC = 2.5cm.
Trapezium Measure the possible lengths of DC to the nearest cm
2015 /8
( a ) Using a ruler and a pair of compasses only ,
construct a: Parallelogram
(i) trapezium WXYZ such that WX = 8cm,
XY = 5.5cm, XZ = 8.3cm, WXY = 600
and WX//ZY 2014/8 (Nov)
(a) Using a ruler and a pair of compasses only , construct
(ii) rectangle PQYZ where P and Q are on WX (i) parallelogram PQRS with RS as the base such that
( b ) Measure : ( i ) QX; ( ii ) XWZ PQ= 7.8cm, QR = 5.6cm and angle QRS = 1200;
Analysis and Solution Diagram on page 404 ( ii ) rectangle ABRS, equal in area to parallelogram PQRS
Steps taken ( b ) Measure : ( i ) AP; ( ii ) AS
1. Draw a line WX = 8cm with extensions as shown Analysis and Solution Diagram on page 405
2. At X construct angle 60o Steps taken
3. Produce X to Y 5.5cm long 1. Draw line RS with extensions as shown
4. Construct a line ZY parallel to WX 2. At R construct angle 1200
5. With radius WX and centre Y, draw an arc 3. Produce R to Q 5.6cm long
6. With radius XY and centre W, draw an arc to cut 4. Construct line QP parallel to RS
the 1st one 5. With radius QR centre So draw an arc
7. join Zo to Y 6. With radius RSo centre Q draw another arc to cut the 1st
8. Measure X to Zo 8.3cm to give Z 7. Join Q to P 7.8cm long through the intersection of the arcs
9. At Y and Z, construct perpendicular bisectors 8. Construct another line PS parallel to QR
to touch WX at P and Q 9. With radius QP and centre R draw an arc
10. With radius QR and centre P, draw an arc to cut
Alternative step 1 - 3 the 1st one
1. Draw a line XY = 5.5cm with extensions as shown 11. Produce P to S
2. At X construct angle 60o 12. Draw a perpendicular at R and S to touch QP at A and B
3. Produce X to W 8cm long as specified rectangle ABRS ( i.e RS as the base)
step 4 – 9 are the same here
Alternative step 1 – 3 Diagram on page 402 2013/11 Neco
(a) Using a ruler and a pair of compasses only: construct;
( i ) a Parallelogram EFGH with EF= 9cm,
2012/10 Neco EG = 11.5cm and  EFG = 1050
( a ) Using a ruler and a pair of compasses only , ( ii ) a circle to touch angles E, F and G
construct :
(b) ( i ) measure FH and
(i) a trapezium PQRS in which the parallel sides PQ
( ii ) radius of the circle
and SR are 5cm apart,
(c) determine the area of the parallelogram
SPQ = 600, PQ = 9cm and SR = 6cm Analysis and Solution Diagram on page 405
(ii) locus l1 of points equidistant from P and Q Steps taken
(iii) locus l2 of points 7cm from P 1. Draw line EF = 9cm with extensions as shown
(b) ( i ) Locate M1 and M2, points of intersection of l1 2. At F construct angle 105o ( i.e constructing 900 between
and l2 600 and 1200 then bisecting it to give 150 + 900 )
( ii ) MeasureM1M2 3. Join E to G at 11.5cm long
403
Z0 Z Y 2015/8

5. 5
8. 3

c
c m

m
QX = 6.7cm
XWZ = 780
W Q P X
8cm

L2 2012/10 Neco

S M2 6cm R

P 9cm Q

M1 M1M2 = 10.8cm

L1
404
405
4. Construct a parallel line to EF Indirect quadrilateral construction
5. With radius EF and centre G draw an arc
6. With radius GF and centre E, draw another arc to cut 2008/9 NABTEB (Nov)
the 1st one ( a ) Using a ruler, a pair of compasses and a set squares
7. Join G to H point of the arc intersection only construct a quadrilateral PQRS such that
8. Construct a circle to touch angles E, F and G is PQR = 750 QRS = 600 PQ = 6 cm,
QR = 8 cm and PS //QR.
a circumcircle to EFG
Measure RS
9. Bisect line EF and line GF, their point of ( b ) construct the
intersection is the centre of the circle.
( i ) locus l1 of points equidistant from Q and R
( ii ) locus l2 of points equidistant from QR and RS
( c ) Locate the point of intersection T of l1 and l2.
Rhombus Measure PT
2011\9 Analysis and Solution Diagram on page 407
Using a ruler and a pair of compasses only , Steps taken
( a ) Construct a rhombus PQRS of side 7cm 1. Draw a line QR = 8cm with extensions as shown
and PQR = 600 2. At point Q construct angle 750 ( 1200, then bisect between
600 and 600 to get 900, then bisect between 900 and 600 to get 750)
( b ) locate a point X , such that X lies on the locus of 3. Produce line Q to P 6cm long
points equidistant from PQ and QR and also 4. Construct line PS parallel to QR
equidistant from Q and R : 5. With radius QR and centre P, draw an arc
( c ) Measure XR 6. With radius QP and centre R, draw another arc to cut
Analysis and Solution Diagram on page 407 the 1st one
Steps taken 7. Join P to the point of intersection So
1. Draw a straight at Q to a reasonable length 8. At R construct 600
2. At Q construct 600 at the top and 600 at the bottom 9. Produce R to meet PSo at S
as shown 10. locus l1 is the perpendicular bisector of line QR
3. Extend Q to P 7cm long and Q to S 7cm long
11. locus l2 is the bisector of angle QRS
4. Join S to the line at Q meeting at R 7cm long
also join P to it 7cm long
2003/12 Exercise 22.14
5. locus l1 of PQ and QR is bisector of angle PQR Using a ruler and a pair of compasses only,
6. locus l2 of Q and R is the perpendicular bisector (a) Construct a quadrilateral PXYQ such that
of line QR /PX/ = 9.9cm , /QX/ = 10.2cm , QPX =750,
/QY/ = 10.4cm and PQ//XY.
2015\9b (Nov) Exercise 22.13 (b) Construct the :
( i ) Using a ruler and a pair of compasses only , ( i ) locus l1 of points equidistant from X and Y,
Construct a rhombus MNOQ of length 10cm ( ii ) locus l2 of points equidistant from QY and YX .
and MNO = 450 (c) Locate M, the point of intersection of l1 and l2 .
( ii ) Measure : ( a ) MO , ( b ) NQ (d) Measure /PM/ .

2001/9 Exercise 22.15


Using a ruler and a pair of compasses only ,
(a) Construct quadrilateral MNOP with /OP/ = 11cm,
/PM/ = 6cm , /NO/ = 5cm , /MN/ = 8cm
and /MO/ = 9cm .
(b) Measure MPO .
(c) Locate the point Y which is equidistant from the
straight lines ON and OP and also equidistant from
points M and N.
(d) Draw YP and find the perimeter of OYP.

406
P
2011/9

7cm
L1 7cm
X

Q R

L2 7cm XR = 4cm
7cm

P S S0
2008/9 NABTEB (Nov)

6cm
T

Q 8cm R
L2 RS = 6.7cm
PT = 4.2cm

L1

407
Chapter twenty three From the table, which operation in column 2 gave RHS 3
Modulo arithmetic i.e 2  4 = 3
This is the arithmetic of remainders when a number is is only possible if 2 is unchanged
divided by a lower number. From our counting Thus, 2 × 1  4 = 3
numbers, each of them has its set of remainders hence n = 1
(residues) :
4 is {0, 1, 2, 3}
(ii) 2  (3  n )  2
5 is {0, 1, 2, 3, 4} From the table, which operation in column 2 gave RHS 2
6 is {0, 1, 2, 3, 4, 5} 2  1=2
Generally, for any number n, the residues are Thus, 2  (3  n)  2 implies
{0, 1, 2, 3, 4, …, n – 1 } 3 n 1
Some of the commonly used symbols are From the table, which operation in column 3 gave RHS 1
 for congruent or equivalent 3 2 = 1
 for multiplication and  for addition Hence n = 2
2009/14a
(a) Copy and complete the multiplication table 2016 / 13b
modulo 5 on the set {1, 2, 3, 4, } The operation Δ is defined on the set
 1 2 3 4 T = {2, 3, 5, 7} by XΔY = (x + y + xy) mod 8
1 1 3 (i) Construct modulo 8 table for the operation Δ on the
2 4 set T.
(ii) Use the table to find
3 3 2
i. 2 Δ (5 Δ 7)
4 3 1
ii. 2Δn=5Δ7
(b) From your table x
i. Solve the expression 2n  4  3 Δ 2 3 5 7
ii. Find the value of n for which 2 0 3 1 7
2  (3  n )  2 y 3 3 7 7 7
5 1 7 3 7
Solution
1  2 = 2 to mod 5 7 7 7 7 7
= 2
1  4 = 4 to mod 5 2Δ2 = (2 + 2 + 2 × 2) to mod 8
= (4 + 4) = 8 i.e 8 ÷ 8 = 1 remainder 0
= 4
2  4 = 13 to mod 5 =0
2 Δ 3 = (2 + 3 + 2 × 2) to mod 8
= 3
= (5 + 6) = 11 i.e 11 ÷ 8 = 1 remainder 3
3  2 = 11 to mod 5
= 3
= 1
2 Δ 5 = (2 + 5 + 2 × 5) to mod 8
3  3 = 14 to mod 5
= (7 + 10) = 17 i.e 17 ÷ 8 = 2 remainder 1
= 4
=1
4  1 = 4 to mod 5
2 Δ 7 = (2 + 7 + 2 × 7) to mod 8
= 4
4  3 = 22 to mod 5 = (9 + 14) = 23 i.e 23 ÷ 8 = 2 remainder 7
= 2 = 7
Multiple table for {1, 2, 3, 4} mod 5 3 Δ 2 = (3 + 2 + 3 × 2) to mod 8
 1 2 3 4 = (5 + 6) = 11 i.e 11 ÷ 8 = 1 remainder 3
1 1 2 3 4 =3
3 Δ 3 = (3 + 3 + 3 × 3) to mod 8
2 2 4 1 3
= (6 + 9) = 15 and 15 ÷ 8 = 1 remainder 7
3 3 1 4 2
=7
4 4 3 2 1 3 Δ 5 = (3 + 5 + 3 × 5) to mod 8
= (8 + 15 ) = 23 i.e 23 ÷ 8 = 2 remainder 7
b. (i) 2n  4 3 =7
408
3 Δ 7 = (3 + 7 + 3 × 7) to mod 8 1  6 = 1 + 6 + 2 (to mod 7)
= (10 + 21) = 31 i.e 31 ÷ 8 = 3 remainder 7 = 9 to mod 7 i.e 9 ÷ 7 = 1 remainder 2
=7 = 2
5 Δ 2 = ( 5 + 2 + 5 × 2) to mod 8 3  3 = 3 + 3 + 2 (to mod 7)
= (7 + 10) = 17 i.e 17 ÷ 8 = 2 remainder 1 = 8 to mod 7 i.e 8 ÷ 7 = 1 remainder 1
=1 =1
5 Δ 3 = (5 + 3 + 5 × 3) to mod 8 3 5 = 3 + 5 + 2 (to mod 7)
= (8 + 15) = 23 i.e 23 ÷ 8 = 2 remainder 7 = 10 to mod 7 i.e 10 ÷ 7 = 1 remainder 3
=7 = 3
5 Δ 5 = ( 5 + 5 + 5 × 5) to mod 8 3 6 = 3 + 6 + 2 (to mod 7)
= (10 + 25) = 35 i.e 35 ÷ 8 = 4 remainder 3 = 11 to mod 7 i.e 11 ÷ 7 = 1 remainder 4
=3 =4
5 Δ 7 = (5 + 7 + 5 × 7) to mod 8
5 5 = 5 + 5 + 2 (to mod 7)
= (12 + 35) = 47 i.e 47 ÷ 8 = 5 remainder 7
= 12 to mod 7 i.e 12 ÷ 7 = 1 remainder 5
=7
=5
7 Δ 7 = (7 + 7 + 7 × 7) to mod 8
5 6 = 5 + 6 + 2 (to mod 7)
= (14 + 49) = 63 i.e 63 ÷ 8 = 7 remainder 7
= 13 to mod 7 i.e 13 ÷ 7 = 1 remainder 6
=7
=6
(ii) i. 2 Δ (5 Δ 7)
6  6 = 6 + 6 + 2 (to mod 7)
From table (5 Δ 7) = 7
= 14 to mod 7 i.e 14 ÷ 7 = 2 remainder 0
Thus, 2 Δ (5 Δ 7) = 2 Δ 7
=0
From table 2 Δ 7 = 7
ii. 2 Δ n = 5 Δ 7
(ii) i. 3  n = 3
2Δn=7
From the table, which number did 3 operate on to give 3
From the table
2Δ7 = 7 35=3
Thus n is 7 Thus n = 5
2015/ 11c ii. n  n = 3
An operation  is defined on the set From the table, which number operate itself to give 3
It does Not exist.
X = {1, 3, 5, 6} by m  n = m + n + 2 (mod 7),
where m, n, X.
i. Draw a table for the operation
2012 / 14b
ii. Using the table, find the truth set of:
Addition: Modulo 6 Multiplication: modulo 6
i. 3  n = 3;
 0 1 2 3 4 5  0 1 2 3 4 5
ii. n  n = 3 0 0 1 2 3 4 5 0 0 0 0 0 0 0
Solution 1 1 2 3 4 5 0 1 0 1 2 3 4 5
m  n = m + n + 2 (mod 7) for {1, 3, 5, 6} 2 2 3 4 5 0 1 2 0 2 4 0 b 4
 1 3 5 6 3 3 4 a 0 1 2 3 0 3 0 3 0 3
1 4 6 1 2 4 4 5 0 1 2 3 4 0 4 2 0 4 2
5 5 0 1 2 3 4 5 0 5 4 3 2 1
3 6 1 3 4
5 1 3 5 6 Using the addition  and multiplication tables for modulo 6,
6 2 4 6 0 i. find the value of a;
ii. determines the value of b;
1  1 = 1 + 1 + 2 (to mod 7) iii. evaluate 4  (3  2) ;
=4 iv. find m if 5 + (m  3) = 2
1  3 = 1 + 3 + 2 (to mod 7) Solution
=6 i. a is 3 2 (mod 6) = 5 to mod 6
1  5 = 1 + 5 + 2 (to mod7) =5
= 8 to mod 7 i.e 8 ÷ 7 = 1 remainder 1 ii. b is 2  4 (mod 6) = 22 to mod 6
= 1 = 2
409
iii. 4  (3  2) = 4  0 [3 2 = 0 from Thus (9  5)  (10 10) = 1  1
multip.table] From the table 1  1 = 1
= 4 [4  0 = 4 from addition table] (ii) i. 10  m = 2
iv. 5 + (m 3) = 2 What did 10 multiply to give 2?
From the addition table, 10  9 = 2
5 + what = 2 Thus, m = 9
5+3=2 ii. n  n = 4
Thus, m 3 = 3 Which number multiple itself to give 4?
From multiplication table 99 = 4
value(s) that multiply 3 to give 3 are m = 1, 3 and 5 Thus, n = 9
But m = 3 is the accepted value
2009 / 15 (Nov)
Based on our earlier assertion
(a) Draw the
5 + 3 = 2 from additional table.
i. Addition 
ii. Multiplication 
2014/8a
tables for the set x = {1, 2, 3, 4, 5, 6} modulo 6
Copy and complete the following table for
(b) From your tables, solve
multiplication modulo 11
i. (4  n)  n = 4
 1 5 9 10
ii. t  (t  3) = 2
1 1 5 9 10 Solution
5 5 Mod 6  {1, 2, 3, 4, 5, 6} Mod 6  {1, 2, 3, 4, 5, 6}
9 9  1 2 3 4 5 6  1 2 3 4 5 6
10 10 1 2 3 4 5 0 1 1 1 2 3 4 5 0
Use the table to: 2 3 4 5 0 1 2 2 2 4 0 2 4 0
i. Evaluate (9  5)  (10 10); 3 4 5 0 1 2 3 3 3 0 3 0 3 0
4 5 0 1 2 3 4 4 4 2 0 4 2 0
ii. Find the truth set of
5 0 1 2 3 4 5 5 5 4 3 2 1 0
i. 10  m = 2,
6 1 2 3 4 5 0 6 0 0 0 0 0 0
ii. n n = 4
Solution Addition table
mod 11 of { 1, 5, 9, 10} 1 + 1, 1 + 2, ... 1 + 4 result less than 6
So we write them down like that
 1 5 9 10
1 1 5 9 10 1+5 = 10 to mod 6 [1+5 = 6 and 6 ÷ 6 = 1 no remainder]
= 0
5 5 3 1 6
1 + 6 = 11 to mod 6
9 9 1 4 2
= 1
10 10 6 2 1
2 + 6 = 12 to mod 6 [2 + 6 = 8 and 8 ÷ 6 = 1 remainder 2]
5  5 = 5 × 5 = 25 to mod11 i.e 25 ÷ 11 = 2 remainder 3 = 2
=3 The pattern goes on
5  9 = 5 × 9 = 45 to mod11 i.e 45 ÷ 11 = 4 remainder 1 6 + 6 = 20 to mod 6 [6 + 6 = 12 and 12 ÷ 6 = 2 remainder 0]
=1 = 0
5  10 = 5 × 10 = 50 to mod11 i.e 50 ÷ 11 = 4 remainder 6
=6 Multiplication table
9  9 = 9 × 9 = 81 to mod11 i.e 81 ÷11 = 7 remainder 4
1 × 1, 1 × 2, ... 1 × 5 result less than 6
=4 So we write the result down unchanged
9  10 = 9 ×10 = 90 to mod11 i.e 90 ÷ 11 = 8 remainder 2
1 × 6 = 10 to mod 6 [1×6 = 6 and 6 ÷ 6 = 1 remainder 0]
=2
= 0
10  10 = 10 × 10 = 100 to mod11 i.e 100 ÷11 = 9 remainder 1
=1 2 × 4 = 1 2 to mod 6 [2×4 = 8 and 8 ÷ 6 =1 remainder 2]
= 2
(i) (9  5)  (10 10) 3 × 4 = 20 to mod 6 [3 × 4 =12 and 12 ÷ 6 = 2 remainder 0]
From the table (9  5) = 1 and (10 10) = 1 = 0
410
3 × 6 = 30 to mod 6 [3×6 =18 and 18 ÷ 6 = 3 remainder 0] 2003/14a
= 0 Find the least integral value of n such that
4n + 3  1 (mod 6)
6 × 6 = 60 to mod 6 [6 × 6 = 36 and 36 ÷ 6 = 6 remainder 0 ] Solution
= 0 4n + 3 = 1 (mod 6)
b (i) (4  n)  n = 4 Same as
4n + 3 = 1 + 6 (mod 6)
our working is in Row 4
Take away 3 from both sides
Which number did 4 add to that produce 4 4n = 4 (mod 6)
46 = 4 Divide through by 4
Thus n = 6 n= 1
Confirmation by working further 2004/14a
(4  n)  n = 4 becomes Solve 2x + 4 = 0 (mod 5)
4  6 = 4 same result Solution
2x + 4 = 0 (mod5)
(ii) t  ( t  3) = 2 Same as
Here our attention will be on result 2 in the Addition tables 2x + 4 = 0 + 5 (mod 5)
1+1=2 Take away 4 from both sides
2+ 6 =2 2x = 1 (mod 5)
3+5=2 We get the multiple of the coefficient of x i.e 2 at the RHS
4+4 =2 2x = 1 + 5 (mod 5)
5+3=2 2x = 6 (mod 5)
6+2=2
Divide through by2
x= 3
Our the results, the bracket item t  3 will force in to be
2016/39
concerned with multiples of 3. If 20(mod 9) is equivalent to y(mod 6), find y
Reason t  1 = 2 implies 1 + 1 = 2 A1 B2 C3 D4
Then t 3 = 1 Solution
20(mod 9) compared in terms of tens is 20(mod 10)
Is it true that 1  3 = 1 No We say two tens i.e 20
Hence we must restrict to multiples of 3 Same applies to 20(mod 9)
2  6 = 2 Addition table two nines i.e 18
Next, convert 18 to mod 6
Then 2  (2  3 ) = 2  0 dead end We divide 18 by 6 = 3 times
53 = 2 20(mod 9) = 30(mod 6)
Thus y = 3
Then 5  ( 5  3 ) = 5  3
2016/41 (Nov)
= 2 true Evaluate 6 – 36(mod)9
2010/15c A. 3 B. 4 C. 5 D. 6
If x is the positive integer, determine the least value Solution
of x for which 13 + 2x = 3 (mod 8) 36(mod)9 is 36 ÷ 9 = 4 remainder 0
Solution =0
13 + 2x = 3 (mod 8) Thus 6 – 36(mod)9 = 6 – 0
Same as = 6 (D)
13 + 2x = 3 + 8 (mod 8) it will not help us
Same as 2016/1a Neco Exercise 23.1
13 + 2x = 3 + 16 (mod 8) Copy and complete the multiplication table below in mod 6.
Take away 13 from both sides  1 2 3 4
2x = 6 (mod 8)
1 1 3
Divide through by 2
x = 3 (mod 8) 2 2 4
3 3 3
2015/3 (Nov) 4 4 4
If x is a whole number such that 2x + 1  4 mod 7,
find the least value of x 2002/14 Exercise 23.2
A. 2 B. 3 C. 4 D. 5 (a) Draw the :(i) addition  (ii) multiplication 
Solution Tables for the set { 2, 3, 5, 6 } modulo 7
2x + 1 = 4 (mod 7) (b) From your tables in (a)
Same as
(i) evaluate (6  5)  (3  2) (ii) find the truth set of n  6 = 2
2x + 1 = 4 + 7 (mod 7)
Take away 1 from both sides 2004/15 (Nov) Exercise 23.3
2x = 10 (mod 7)
(a) Construct a multiplication  table modulo 7
Since the RHS is already in the multiple of 2;
On the set H = { 1, 2, 3, 5, 6}
Divide through by 2 (b) Using your table: (i) evaluate 6  2  5 (ii) solve n  6 = 2
x = 5 D. (iii) find the truth set of n  n = 1
411
Chapter twenty four Connectives
logic The words and, or, not,… then, and if and only if are
The basic problem of logic is the analysis of the the basic words used to connect propositions. They are
methods of reasoning. Reasoning is that special kind of called logical connectives
thinking called inference, in which conclusions are
drawn from premises. As an example consider:
Use of Symbols
All dogs have four legs. The power of symbolic Logic lies in its use of
Bingo is a dog symbols to represent proposition and connectives. We shall
Therefore, Bingo has four legs. represent propositions by the lower case letters p, q, r,…
The premises of this argument are all dogs have four usually, each letter will stand for a simple proposition and
legs and bingo is a dog. The conclusion is Bingo has compound will be formed by linking two or more letters
four legs. together by means of one or more of logical connectives and
also give the name by which the compound is known
The logician seeks the answer to this question. ”If the
premises are true, is one justified in saying that the Connectives Symbol Names of compound
conclusion is true?”. If so, the reasoning that led to the Not ~ Negation
conclusion is valid (logical); if not, the reasoning is And  Conjunction
invalid (illogical). The systematic formalization and Or  Disjunction
cataloging of valid methods of reasoning is one of the If,…. then → Conditional
major task of a logician. If his work uses abstract If and only if  Biconditional
symbols, then it is called symbolic logic.
PROPOSITIONS 1. Example LOG3
To avoid ambiguities, certain Given that:
technical terms will now be p represents “The baby is crying”
defined before using them. q represents “ The boys are singing” and
Proposition r represents” “The dog is barking”
A proposition is a declarative sentence, which is true or Represent the following propositions symbolically.
false but not both (A) The dog is not barking
(B) The baby is crying and the birds are singing
Truth Value: (C) The dog is barking or the birds are singing
The truth value of a proposition is true if the proposition (D) The dog is barking and the birds are not singing
is true and false if the proposition is false. (E) If the dog is barking and the birds are not
Examples LOG 1 singing, then the baby is crying.
These sentences are propositions: (F) The baby cries if and only if the dog barks.
(A) Abuja is a town in Nigeria. Solution
(B) France is one of the united states (a) ~r
(C) 3 + 2 = 5 (b) pq
(D) George is tall. (c) rq
Assertion (a) is true since it corresponds to reality. (d) r~q
Assertion(b) is false since it does not correspond to reality
(e) (r  ~ q) → p
Assertion (c) is true.
(f) pr
Some logicians do not consider a sentence such as(d) to
be a proposition because there are many individuals
named George and because tall is relative term.
Here we shall consider that words such as George, I and Example LOG4
they refer to specific persons and that arrangement on Let s and t stand for:
standards for words such as tall, short, and old has been s : sally is smart.
reached. If these assumptions are made, (d) is a t : tom is tall
proposition whose truth value depends upon the Which sentence has the same meaning as:
particular George referred to and upon the standard of (a) ~ s
tallness agreed. (b) s  t
KINDS OF PROPOSITION (c) (t  s) → t
A Proposition is simple if it does not contain any other
proposition as a component part. It is a compound Solution
proposition if it is composed of two or more proposition. (a) Sally is not smart
Examples LOG2 (b) Sally is smart and Tom is tall
These are compound proposition: (c) If Tom is Tall or Sally is smart then Tom is Tall
(a) 1 + 1 = 2, and 4 is less than 9.
(b) Owls are nocturnal and cats are playful.
(c) If it rains, then I will not attend the game.
412
Examples LOG5 propositions are involved and hence the numbers of truth
If p, q, r, stand for values required will be 22 = 4. Our convention for arranging
p : Birds fly truth values in the first column of the truth tables is TTFF,
q : The sky is blue and for those in second column is TFTF. The complete truth
r : The grass is green table for conjunction is:
1.Write the sentence that has the same meaning as: p q pq
(a) ~p (f) p  ~ q ~r T T T
(b) pq (g) p  ( q) T F F
(c) pq (h) (p  q) → r F T F
(d) pq (i) p↔q F F F
Disjunction The proposition pq is defined to be false
(e) pqr
if p and q are both false and true in all other cases.
2.Using the proposition p, q and r of the above problem,
p q pq
write symbolic representations of:
T T T
(a) The sky is blue and the grass is green. T F T
(b) Birds fly or the sky is blue. F T T
(c) Birds do not fly and the sky is not blue F F F
(d) If the grass is green and the sky is not blue Conditional
then the birds do not fly. The proposition p → q is defined to be false
Solution if p is true and q is false, and true in all other cases
1. (a) Birds do not fly p q p→q
(b) Birds fly and the sky is blue T T T
(c) Birds fly or the sky blue T F F
(d) Birds do not fly or the sky is blue F T T
(e) Birds fly and the sky is blue and F F T
the grass is green Biconditional
(f)Birds fly and the sky is not blue and the The proposition p  q is defined to be true when
grass is not green p and q have the same truth values and false if they have
(g)Birds do not fly or the grass is not green different values.
(h) If birds fly and the sky is blue then p q pq
the grass is green. T T T
(i) Birds fly if and only if the sky is blue T F F
2. (a) q  r F T F
(b) p q F F T
Constructing truth tables:
(c)  p   q
The tables that have been shown so far can be used in
(d) ( r   q) →  p constructing truth tables for more complex propositions. The
Truth values and truth tables. following examples will illustrate how this is done
In the preceding section, the five basic logical Example LOG 6
connectives were introduced. In this section, we Construct the truth table for p~q
consider the forms of compounds and the rules for Solution
assigning truth values to each of them. We start the table with the standard four rows and two
columns for compound involving the two propositions p and q.
Negation the proposition in question p ~q is the conjunction of p and
The negation p is defined to be true if p is ~q.
false and false if p true. It is customary to use symbol T p q ~q p~q
to denote the truth-value of a true proposition and F to T T F F
donate the truth value of a proposition that is false. T F T T
Using these symbols, the relationship between the truth F T F F
values of p and p are shown in the following truth F F T F
table. After obtaining the truth values under ~q, we then use
p ~p connective (and) to find the truth values under p~q
T F ExampleLOG7
F T Construct the truth for ~ (p q)
p q p q ~ (p q)
Conjunction T T T F
The word “and” is defined to have the same T F T F
meaning in logic as in common usage. The proposition F T T F
pq is true if p and q are both true and false in F F F T
other cases. To know how many truth value are The values for the proposition p q are obtained and then
required. We use the formula 2n, where n denotes the negated to obtain the values for ~ (p q)
number of propositions in value. In the case of p, q two
413
ExampleLOG8 (C) Show that ~pq is equivalent to p →q
Construct the truth table for the following statement Solution
forms: p q ~p ~pq p→q (~pq) (p→q)
(a) p → ~q T T F T T T
(b) p (q→q) T F F F F T
(c) (p  q) → r. F T T T T T
Solutions F F T T T T
(a) p q ~q p→ ~q From the table, (~pq)  (p→q) is a tautology .
T T F F Thus ~pq and p →q are equivalent.
T F T T
F T F T 2010/8 NABTEB (Nov) Exercise 24.1
F F T T What is the negation of the sentence
“John is older than me”
(b) p q q→p p (q→p)
A john is not older than me
T T T T
B john is neither older than me
T F T T
C john is younger than me
F T F T
D john is my age mate
F F T F
2016/5 Exercise 24.2
(C) In this case, we have three proposition p, q and r
Which of the following is a valid conclusion from the
so we shall have 23 = 8 truth values and write them
premise: “Nigerian footballers are good footballers”?
out as follows:
A Joseph plays football in Nigeria therefore
p q r pq (pq)→r he is a good footballer
T T T T T B Joseph is a good footballer therefore
T T F T F he is a Nigerian footballer
T T F T T C Joseph is a Nigerian footballer therefore
T F T F T he is a good footballer
F T T F T D Joseph plays good football therefore
F T F F T he is a Nigerian footballer
F F T F T
F F F F T 2015/20 f/m Exercise 24.3
Logically True propositions Given the statements:
A proposition is logically true if it is true for p : the subject is difficult
every possible arrangement of truth values of its q : I will do my best
component propositions, and it is false for every Which of the following is equivalent to “Although the
possible arrangement of truth values of its parts. subject is difficult, I will do my best”
Logically true propositions are often called
A pq B  pq C p( q) D pq
tautologies.

Equivalence
Two proposition p and q are equivalent if the Logic and the use of Venn diagram
bi-conditional pq is logically true i.e pq is Here we apply basic Venn diagram in set such as listed
logically false, p and q are contradictions. below to interpret logic

Examples LOG9
(a) pv~ p is a tautology
p ~p p v ~ p
T F T
F T T
P and G have nothing in common P and G have something in common
(b) p~p is a contradiction U U
A A B
p ~p p~p
B
T F F
F T F C

A contains B but A and B A, B and C have something in common


have nothing in common with C

414
Example LOG10 2015/49
Illustrate the statement below in a Venn diagram: Consider the following statements:
‘All Northerners in Nigeria speak Hausa, X : locally manufactured types are attractive
Isa is a northerner; Isa speak Hausa’ Y : many locally manufactured types do not last long
Is the conclusion valid?
Denoting locally manufactured types by M,
Solution
attractive types by R and long lasting types by L,
Universal set U: Nigeria which of these Venn diagrams illustrates the statements?
N for Northerner L R
L R
H for Hausa M
M
A B
I for Isa
U
H
N L R M
L R
Isa D M
C

From the Venn diagram we conclude that: Solution


The conclusion is valid Local tyres M
Attractive tyres R
2014/33 (Nov) Long lasting tyres L
U
G R L R
F H M M

First impression Second impression


In the Venn diagram, U = {students in a school} M is subset of R part of L is in M
G = {students in class 3G}, Part of L is in R
F = {students in the school’s football team} 2010/15
and H = {students in the school’s hockey team} Consider the following statements:
Which of the following statements is false? X : all lazy students are careless
A No member of the hockey team is in the football team Y : most dull students are lazy
B Only members of the class 3G are in the football team (a) Illustrate the information above on a Venn diagram.
C All of class 3G are members of the football team (b) Using the Venn diagram or otherwise, determine
D Some members of the hockey team are in class 3G. whether or not each of the following statements is valid
Solution
Option C is false ( i ) Stella is careless  Stella is lazy
as some members of the hockey team are in class 3G ( ii ) Osei is lazy  Osei is careless
( iii ) Kojo is dull  Kojo is lazy
2015/6a ( iv ) Kwame is lazy  Kwame is dull
(i) illustrate the following statement in a Venn diagram; ( v ) Adwoa is lazy and dull  Adwoa is careless
All good Literature Students in a school are in the General ( vi ) Fiifi is careless and dull  Fiifi is lazy
Arts class. Solution
(ii) Use the diagram to determine whether or not the Let Lazy students be L
following are valid conclusions from the given statement: ,, Careless students be C
,, Dull student be D
i Vivian is in the general Arts Class therefore she is a good
Literature student; C D L D C
L
ii Audu is not a good Literature student therefore he is not in L
the General Arts Class;
iii Kweku is not in the General Arts Class therefore he is not
a good literature student.
Solution fig I fig II fig III
First we break down the statement as: Statement X Statement y Result of X and Y
Good literature student (L) (i) Not valid, from figI, L is subset of C,
General arts (G) other students are careless but not lazy
School (S) i.e universal set (ii) Valid, from fig I, L is inside C
From the statement L is subset of G once you are lazy, you are also careless
U
G (iii) Not valid, from fig II, dull and lazy only have
L intersection
(iv) Not valid from fig II same reason as in (iii)
(v) Valid, from fig III, portion for lazy and dull falls
inside careless
From the Venn diagram we conclude that: (vi) Not valid; from fig III, there are portion left in the
I not valid ; II not valid ; III valid intersection between careless and dull that are not lazy
415
CHAPTER TWENTY FIVE Solution
Coordinate geometry Distance between two points = ( x2 − x1 ) 2 + ( y2 − y1 ) 2
Distance between two points 2 5 = [1 − (−3)]2 + [ y − (−2)]2
y

. T (x2 , y2 )
Take the square of both sides
( 2 5 )2 = (1 + 3)2 + (y + 2)2
4 × 5 = 42 + (y + 2)(y + 2)
y2 - y1
20 = 16 + y2 + 4y + 4
y2 + 4y = 0
.
S (x1 , y1 )

x2 - x1
y(y + 4) = 0
y = 0 or y + 4 = 0
x
y = 0 or – 4
For any two points S and T on the Cartesian plane to be accept y = – 4 D.
located, it will be through their x and y co-ordinates.
i.e. (x, y). To find the distance between such points, 2010/35 UTME
we complete a hypothetical right angled triangle as 1 1 1 1
Find the distance between the point ( , ) and (− ,− )
shown above. By Pythagoras theorem 2 2 2 2
( Distance ST )2 = ( x2 – x1 )2 + ( y2 – y1 )2 A. 2 B. 0 C. 1 D. √3
It follows that
Solution
Distance ST = ( x2 − x1 ) 2 + ( y2 − y1 ) 2
Distance between two points = ( x2 − x1 ) 2 + ( y2 − y1 ) 2
2014/37 f/m 2 2
 −1 1   −1 1 
Find the distance between the points (2, 5) and (5, 9) =  −  + − 
 2 2  2 2
A. 4 units B. 5 units C. 12 units D. 14 units
Solution = (−1) 2 + (−1) 2
Distance between two points = ( x2 − x1 ) 2 + ( y2 − y1 ) 2 = 1 +1 = 2 A.
= (5 − 2) 2 + (9 − 5) 2
2009 / 33 UME
= 32 + 4 2 What is the value of r, if the distance between the
= 9 + 16 points (4, 2) and (1, r) is 3 units ?
A. 1 B, 2 C. 3 D. 4
= 25 = 5 units B. Solution
2006 / 12 PCE
Distance between two points = ( x2 − x1 ) 2 + ( y2 − y1 ) 2
Find the distance between points Y(7, 9) and Z(15, 11)
A 3 17 B 2 17 C 2 34 D 4 17 3 = (1 − 4) 2 + (r − 2) 2
Solution 32 = (–3)2 + (r – 2)(r – 2)
Distance between two points = ( x2 − x1 ) 2 + ( y2 − y1 ) 2 9 = 9 + r2 – 4r + 4
r2 – 4r + 4 = 0
= (15 − 7) 2 + (11 − 9) 2 Factoring
(r – 2) (r – 2) = 0
= 82 + 2 2
r – 2 = 0 twice
= 64 + 4 r = 2 B.
= √68 2012/33 UTME
= 4  17 The distance between the point (4, 3) and the intersection
of y = 2x + 4 and y = 7 – x is
= 2 17 B. Solution
2008/35 PCE
Distance b/w two points = ( x2 − x1 ) 2 + ( y2 − y1 ) 2
If the distance between the point (–3, –2) and (1, y)
(x1, y1 ) is (4, 3)
is 2 5 units, find the value of y
(x2, y2 ) can be gotten from the intersection of
A. 4 B. 2 C. –2 D. – 4
y = 2x + 4 and y = 7 – x
416
Solving the simultaneous linear equations Solution
y – 2x = 4 Mid Point Coordinates =
x2 + x1
, y 2 + y1
– ( y + x = 7) 2 2
4 + 6 2 + 10
– 3x = – 3 = ,
2 2
x = 1 10 12
= ,
Substitute x = 1 into y = 2x + 4 2 2

y = 2 + 4 = (5, 6) D.
= 6
2013/31 UTME
Thus (x2, y2 ) is (1, 6)
Distance = (1 − 4) 2 + (6 − 3) 2 If the mid point of the line PQ is (2, 3) and the
point P is (–2, 1), find the coordinate of the point Q
= (−3) 2 + (3) 2
A (8, 6) B (5, 6) C (0, 4) D (6, 5)
= 9+9 Solution
Let the coordinate of the point Q be (a, b)
= 18 = 9  2 = 3 2
x2 + x1 y 2 + y1
Mid Point Coordinates = ,
2 2
2004/2 UME 𝑎− 2 𝑏 + 1
Find the value of 2 + 2 if  +  = 2 and the distance 2, 3 = 2
, 2
between the points ( 1,  ) and ( , 1 ) is 3 units Equating the corresponding coordinates
A.14 B.3 C.5 D.11 2=
𝑎− 2
and 3 =
𝑏 + 1
Solution 2 2
Distance b/w two points = ( x2 – x1 )2 + ( y2 - y1) 4 = a – 2 and 6 = b + 1
32 = (  - 1 ) 2 + ( 1 -  ) 2 4+2 = a and 6 – 1 = b
9 = 2 - 2 + 1 + 1 - 2 + 2 6 = a and 5=b
9 =  + 2 - 2 - 2 + 2
2
Q( 6, 5) D.
9 – 2 = 2 + 2 – 2( + )
put  +  = 2
2008/33 PCE
7 = 2 + 2 – 2( 2 )
7 = 2 + 2 – 4 If the mid point of the line joining points P(m, n) and
7 + 4 = 2 + 2 Q(1, 3) is R(2, 4), find the values of m and n respectively
11 = 2 + 2 (D) A 4, 3 B 2, 7 C 4, 2 D 3, 5
Solution
Here (m, n) is (x, y)
x2 + x1 y 2 + y1
Mid Point Coordinates = ,
2 2
1+ 𝑚 3 + 𝑛
Coordinates of mid - point 2, 4 = 2
, 2

1 1
Equating the corresponding coordinates
is given as [ (x2 + x1) , ( y2 + y1) ] 1 + 𝑚 3 + 𝑛
2 2 2= 2
and 4 = 2
2014/32 UTME 4 = 1 + m and 8 =3 + n
Find the mid point of S(–5, 4) and T(–3, –2) 4–1 = m and 8 – 3 = n
A –4, 1 B 4, –1 C –4, 2 D 4, –2 3 = m and 5=n
Solution P( 3, 5) D.
1 1
Mid point coordinates = 2
(x2 + x1) , 2 (y2 + y1)
1 1
2007/32 PCE
= 2
(–3 –5), 2 (–2 + 4) M (3, –4) is the midpoint of PQ . If P is
8 2
= − , (6, 5), find the coordinates of Q.
2 2
= – 4, 1 (A) A (0, –13) B (4.0, 0.5) C (3, 2.5) D (12, –3)
2009/33 PCE Solution
Find the coordinates of the midpoint of the straight line The coordinates of Q are (x2 , y2)
Mid point coordinates = x 2 + x1 ,
joining M(6, 10) and N(4, 2) y2 + y1
A (2, 8) B (10, 12) C (–5, –6) D (5, 6) 2 2

417
x2 + 6 y +5 2016/39 Neco Exercise 25.1
3, – 4 = , 2
2 2 Find the coordinate of the mid-point of the line joining the
Equating the corresponding coordinates points (–1, –1) and (–3, 5)
x2 + 6 y2 + 5 A (2, 2) B (2, –2) C (–2, –2) D (–2, 2) E (–1, 2)
3 = and –4 =
2 2
6 = x2 + 6 and –8 = y2 + 5 Exercise 25.2
6 – 6 = x2 and –8 – 5 = y2 Find the coordinate of the mid-point of the line joining the
0 = x2 and –13 = y2 points (2, 3) and (–2, 1)
Q (0, –13) A.

2007/47 UME
Gradient / slope of a straight line
What is the value of k if the mid - point of the line y2 − y1
joining is given as
x2 − x1
(1 – k, –4) and (2, k + 1) is (–k, k)?
A –3 B. –1 C. –4 D. –2 2010/36 UTME
Find the gradient of the line passing through the points
Solution P(1, 1) and Q(2, 5)
x2 + x1 y + y1 A4 B2 C3 D5
Midpoint coordinates = , 2
2 2 y2 − y1
2 +1− k k + 1−4
Gradient =
–k , k = , x2 − x1
2 2 5 −1
3−k k −3 =
–k , k = , 2 −1
2 2
= 4 A.
Equating the corresponding coordinates
3−k k −3
–k = and k = 2009/34 UTME
2 2 The gradient of the straight line joining the points
2k = k – 3 P(5, –7) and Q(–2, –3) is
k = –3 A. A 1/ 2 B 2/ 5 C – 4/7 D – 2/ 3
Solution
y2 − y1
2014/35 UTME Gradient =
x2 − x1
Calculate the mid point of the line segment −3 − (−7)
y – 4x + 3 = 0 which lies between the =
−2−5
x – axis and y – axis −3 + 7 −4
= = C.
−2 3 3 3 −7 7
A  − 3 , 3  B ,  C  3 , − 3  D , 
 2 2  3 2 8 2  8 2
2014/33 UTME
Solution The gradient of the line joining (x, 4) and (1, 2) is 1/2.
x 2 + x1 y 2 + y1 Find the value of x
Midpoint = , A–3 B–5 C5 D3
2 2
Solution
To get the coordinates for x and y, we proceeds as follows
y2 − y1
y – 4x + 3 = 0 Gradient =
x2 − x1
y = 4x – 3 1 2−4
When x = 0, y = 4(0) – 3 =
2 1− x
i.e x = 0, y = –3 (0, –3) 1 – x = 2(2 – 4)
when y = 0  0 = 4x – 3 1–x=–4
1+4=x
3 = 4x
5=x C.
¾ = x i.e ( ¾ , 0)
3
Mid point = ( + 0) ÷ 2, [ 0 + (−3)] ÷ 2
4
2008/34 PCE
3 1  −3
= 𝑥 ,   Find the Gradient of the line joining points P(4, – 1)
4 2  2  and Q(–3, – 5)
3 −3 −7
=  ,  C. A
4
B
7
C
−4
D
8 2  7 4 7 4

418
Solution 2015/13 and 14 f/m Exercise 25.6
y −y
A line passes through the origin and the point 1 , 2  . Use
1 1
Gradient = 2 1
x2 − x1  4 2
−5 − (−1) this information to answer question 13 and 14
=
−3− 4 13. What is the gradient of the line?
= −5 + 1 = −4 = 4 A. A1 B2 C3 D4
−7 −7 7
14. Find the y – coordinate of the line when x = 4
2006/4 PCE A2 B4 C6 D8
Find the gradient of the line joining S(5, 6)
and R(– 7, – 8)
A 7/ 6 B–1 C1 D – 7/ 6
Solution Equation of a straight line
y −y is given as y = mx + c
Gradient = 2 1
x2 − x1 Where m is gradient and c is the point where the line cuts
−8 − 6 the y-axis
=
−7−5
−14 7 2001/24 f/m
= = A.
− 12 6 A straight line makes an angle of 300 with the positive
x – axis and cuts the y – axis at y = 5. find the equation of
2009/32 UME the straight line
What is the value of P if the gradient of the line joining A.y = 1 x + 5 B.y = x + 5 C. 3 y = -x + 5 3
(– 1, P) and (P, 4) is 2/3 ? 10
A–2 B–1 C1 D2 D 3y = x + 5 3
Solution Solution
y
y2 − y1
Gradient =
x2 − x1 5
2 4− p
=
3 p − (−1)
2 4− p
= 30 0
3 p +1
x
2(p + 1) = 3(4 – p)
2p + 2 = 12 – 3p From the sketch we have a negative gradient
2p + 3p = 12 – 2 y = – mx + c
5p = 10 Here tan  is tan 300 = 1 and c is 5,
p=2 D. 3
y=– 1x + 5
2016/37 Neco Exercise 25.3 3
Find the gradient of line joining the points (2, 5) and (– 6, 3) Multiplying through by √ 3
A – 1/ 5 B – 1/4 C – 1/ 3 D – 1/ 2 E 1/4
3 y = –x + 5 3 ( C)
2009/34 PCE Exercise 25.4
Find the gradient of the straight line passing though 1997/34 PCE
the points x(– 1, 3) and y(5, –1) What is the y – intercept and the gradient of the line whose
A6 B 2/ 3 C – 2 / 3 D–4 equation is x + y = 1 ?
5 3 3
2016/24 and 25 Exercise 25.5 A. 1/3 , 3/5 B.1, – 3/5 C. –1/3 , – 3/5 D. –1, 3/5
A straight line passes through the points P (1, 2) and Solution
Q(5, 8). First, we rearrange the given equation into y = mx + c format
Use this information to answer question 24 and 25 To clear fractions, multiply through by the Lcm of 5 and 3
24. Calculate the gradient of the line PQ i.e 15
A 3/ 5 B 2/3 C 3/ 2 D 5/ 3 15 × x + y × 15 = 1 × 15
5 3 3
25. Calculate the length PQ 3x + 5y = 5
5y = – 3x + 5
A 4 11 B 4 10 C 2 17 D 2 13
y=–3x+1
5
y – intercept is + 1
Gradient is – 3/5 ( B )
419
1998/35 PCE Equation of a straight line through
Find the gradient of the line whose equation is given x1, y1 with gradient m is
( a – b )x + ( c – d )y = e – f y – y1 = m ( x – x1 ) OR
A. b – a B. a – b C. b – a D. e – f
c–d c–d c-d c–d m = y – y1
Solution x – x1
First, we rearrange the given equation into y = mx + c
format 1992/35 PCE
( c – d )y = – ( a – b )x + ( e – f ) Find the equation of a straight line of gradient 2 through the
Divide through the coefficient of y point ( 1, 4 )
y=–(a–b)x+ e–f A.y – x + 2 = 0 B.y – 2x + 4 = 0 C.y + 2x + 4 = 0
c–d c–d D.y – 2x – 2 = 0
Therefore, Solution
gradient m= –(a–b) Applying the formula
c–d m = y – y1
opening up the bracket x – x1
m = – a + b or b – a
c–d c–d (A) 2=y– 4
1993/39 PCE x–1
The intercept which the line 2x – 3y – 5 = 0 makes cross multiplying
with the y-axis is of length 2( x – 1 ) = y – 4
A. 5/3 B. 5/2 C. – 5/3 D. – 5/2 2x – 2 = y – 4
Solution 0 = y – 4 + 2 – 2x
First we rearrange the given equation in y = mx + c i.e. y – 2x – 2 = 0 (D)
format
We are to find C
2x – 3y – 5 = 0
2x – 5 = 3y Equation of a line passing through two points
y = 2x – 5 m = y – y1
3 3 x – x1
The intercept at the y-axis is – 5/3 ( C ) But m = y2 – y1
x2 – x1
2005/10 PCE Thus
The equation of the straight line which makes an y2 – y1 = y – y1
intercept of –2 with the y-axis and an intercept of 1 with x2 – x1 x – x1
the x-axis is
A. 3x + 2y – 6 = 0 B. 3x – 2y + 6 = 0 2014/34 UTME
C. 2x + y – 2 = 0 D. 2x – y – 2 = 0 y
Solution 5

General form of a straight line is y = mx + c ; where m 4

is gradient and c is the intercept on the y-axis 3

The given question only has c but no m. Though we can 2

simulate the two points needed in the formula 1

x
m = y2 – y1 -5 -4 -3 -2 -1
-1
0 1 2 3 4 5

x2 – x1 -2
At intercept –2 with the y-axis -3

x = 0 and y = -2 i.e ( 0, –2 ) -4

At intercept 1 with the x-axis -5

x = 1 and y = 0 i.e ( 1, 0 )
In the figure above, what is the equation of the line that
Thus, m = – 2 – 0 passes the y – axis at (0, 5) and passes the x – axis at (5, 0)?
0–1 A y = x–5 B y = –x – 5 C y =x+5
= –2 = 2 D y = –x + 5
–1
The required equation is with c = – 2 given earlier and Solution
m=2 Formula for equation of a line given two points is
i.e y = 2x – 2 y − y1 y 2 − y1
Rearranging 2x – y – 2 = 0 ( D ) =
x − x1 x2 − x1
y − 5 0− 5
=
x−0 5−0
420
y − 5 −5 2012/34 UTME Exercise 25.8
=
x 5 Find the equation of the line through the points (–2, 1)
5(y – 5) = –5x and (– ½, 4)
5y – 25 = –5x A. y = 2x – 3 B. y = 2x + 5
5y = –5x + 25 C. y = 3x – 2 D. y = 2x + 1
y = –x + 5 (D)
2016/32 Neco Exercise 25.9
2007 / 34 P C E Find the equation of the straight line which passes through
The equation of a straight line which has intercepts the points X(3, – 4) and Y(– 5, 2)
A 4y = 3x – 7 B y = 3x + 2
of 3 and –5 respectively on the x and y – axes is
C 2y = 3x + 5 D 3y = – x – 4 E 4y = 2x + 6
A. 5y – 3x + 15 = 0 B. 3y + 5x – 15 = 0
C. 5y + 3x + 15 = 0 D. 3y – 5x + 15 = 0
Solution
Intercept at x – axis is 3, then y = 0 i.e (3, 0) Parallel lines
Intercept at y – axis is –5 then x = 0 i.e (0, –5) If two lines are parallel then m1 = m2
Formula for equation of a line given two points is
y − y1 y 2 − y1 2007/40 UME
=
x − x1 x2 − x1 If the lines 3y = 4x – 1 and qy = x + 3 are parallel
y−0 −5 − 0 to each other, the value of q is
=
x−3 0 − 3 −4
B. −3
4 3
A. C. D.
3 4 3 4
y −5 y 5
= i.e =
x − 3 −3 x − 3 3 Solution
3y = 5(x – 3) If two lines are parallel then equal gradient
3y = 5x – 15 Firstly,
3y – 5x + 15 = 0 D. 3y = 4x – 1, qy = x + 3
4 1 1 3
y = x − y = 𝑞x + 𝑞
3 3
2012/35 f/m
Equating their gradients
A straight line makes intercepts of –3 and 2 on the 4 1
x and y axes respectively. Find the equation of the 3
= 𝑞
line. 4q = 3
A. 2x + 3y + 6 = 0 B. 3x – 2y – 6 = 0 3
q = 4
D.
C. –3x + 2y – 6 = 0 D. –2x + 3y – 6 = 0
2006/8 UME
Solution
PQ and RS are two parallel lines. If the coordinates of P, Q,
Intercepts at x – axis is –3, then y = 0 i.e (–3, 0)
R, S are (1, q), (3, 2), (3, 4), (5, 2q) respectively,
Intercepts at y – axis is 2 then x = 0 i.e (0, 2)
find the value of q.
Formula for equation of a line given two points is
A. 2 B. 4 C. 1 D. 3
y − y1 y − y1
= 2 Solution
x − x1 x2 − x1
If two lines are parallel then equal gradient
y− 0 2 − 0
= Applying the formula for gradient of two points
x − (−3) 0 − (−3)
y2 − y1 y2 − y1
2 =
y
= x2 − x1 x2 − x1
x +3 3
2 − q 2q − 4
=
3y = 2(x + 3) 3 − 1 5 − 3
3y = 2x + 6 2 − q 2q − 4
=
3y – 2x – 6 = 0 D. 2 2
Multiply both side by 2
2009/35 PCE Exercise 25.7 2 – q = 2q – 4
Find the equation of the line joining P(5, –1) to Q(–1, 3) 2 + 4 = 2q + q
A. 2x – 3y – 7 = 0 B. 2x + 3y – 7 = 0 6 = 3q
C. 2x + 3y + 7 = 0 D. 2x + 3y – 11 = 0 2 = q A.

421
2007/33 PCE Substituting for x = 2 and y = –3
What is the equation of a line parallel to the line 2x + y = k becomes
3y – x + 4 = 0 which passes through the point (2, – 4)? 2(2) – 3 = k
A. 3y + x + 14 = 0 B. 3y – x + 14 = 0 4– 3 = k
C. y + 3x – 14 = 0 D. y – 3x – 14 = 0 1 = k
Solution The line equation is 2x + y = 1
Two lines are parallel then equal gradient 2x + y – 1 = 0
First, we get the gradient M1 from 2006/ 6 PCE
3y – x + 4 = 0 Find the equation of a line parallel to line y = 3x + 4 which
3y = x – 4 passes through the point (2, –5)
1 4 A. x – 3y – 11 = 0 B. 3x – y – 11 = 0
y= x –
3 3
C. 3x + y – 11 = 0 D. x – 3y + 11 = 0
Comparing with y = mx + c
1 Solution
M1 = 3 If two lines are parallel then equal gradient
Applying the formula for a line with a given gradient ⅓ First, we get the gradient M1 from
and two points (2, – 4) y = 3x + 4
y − y1 Comparing with y = mx + x
M=
x − x1
M1 = 3
1 y − (−4) Applying the formula for a line, given gradient 3 and two
=
3 x−2 points (2, –5)
1 y+4 y − y1
= M=
3 x−2 x − x1
x – 2 = 3(y + 4) 3 =
y − (−5)
x – 2 = 3y + 12 x−2
3y – x + 14 = 0 B. y +5
3 =
2016/13a x−2
Find the equation of a straight line which passed through the 3(x – 2) = y + 5
point (2, –3) and is parallel to the line 2x + y = 6 3x – 6 = y + 5 thus, y – 3x + 11 = 0 B.
Solution
If two lines are parallel then equal gradient 2006 / 6 PCE
First, we get the gradient M1 from Alternative method
2x + y = 6 The line equation parallel to
y = –2x + 6 y = 3x + 4 is
Comparing with y = mx + c y = 3x + k at (2, –5)
Thus, M1 = –2 Substituting for x = 2 and y = –5
Applying the formula for a given line with gradient –2 y = 3x + k becomes
and two points (2, –3) –5 = 3(2) + k
y − y1 –5 = 6 + k
m =
x − x1 –5 – 6 = k
y − (−3) –11 = k
–2 =
x − 2 The line equation is y = 3x – 11 B.
y + 3 2010/ 37 UTME Exercise 25.10
–2 =
x − 2 Find the equation of a line parallel to y = – 4x + 2
–2 (x – 2) = y + 3 passing through (2, 3)
–2x + 4 = y + 3 A. y – 4x + 11 = 0 B. y – 4x – 11 = 0
y + 2x – 1 = 0 C. y + 4x + 11 = 0 D. y + 4x – 11 = 0

2016/13a 2002/32 PCE Exercise 25.11


The equation of a straight line through the point
Alternative method (–3, –2 ) that is parallel to the line 4x + y – 6 = 0 is
The line equation parallel to 2x + y = 6 is A.4x + y + 14 = 0 B.4x + y + 12 = 0
2x + y = k at (2, –3) C.4x + y – 14 = 0 D.4x + y – 12 = 0
422
2003/18 PCE Exercise 25.12 2013/38 f/m
Find the equation of the line passing through ( 4, –2 ) Find the equation of the straight line that passes through
and parallel to the line x – 3y – 4 = 0 (2, –3) and perpendicular to the line 3x – 2y + 4 = 0
A.3y – x + 10 = 0 B. y – 3x – 10 = 0
A. 2y – 3x = 0 B. 3y – 2x + 5 = 0
C. 3y – x – 10 = 0 D. y – 3x + 10 = 0
C. 3y + 2x + 5 = 0 D. 2y – 3x – 5 = 0
Solution
If intersection of two straight lines is Equation of line through (x1, y1) with gradient m is
perpendicular then m1 m2 = –1 y − y1
m =
x − x1
2014/36 UTME But two lines are perpendicular, when their gradient
Find the equation of the straight line through (–2, 3) m1m2 = –1
and perpendicular to 4x + 3y – 5 = 0 From 3x – 2y + 4 = 0
A. 4x + 5y + 3 = 0 B. 5x – 2y – 11 = 0 3x + 4 = 2y
C. 3x – 4y + 18 = 0 D. 3x + 2y – 18 = 0 y =
3
x + 2
Solution 2
3
Equation of the line through (x1, y1) with gradient m is Thus, m1 = 2
y − y1 But m1m2 = –1
m=
x − x1 3
It follows that m2 = –1
But two lines are perpendicular when their gradient 2
−2
m1m2 = –1 m2 =
3
From 4x + 3y – 5 = 0
−2 y+3
3y = –4x + 5 Equation of line is =
3 x−2
−4 5
y= x + –2(x – 2) = 3(y + 3)
3 3
−4 –2x + 4 = 3y + 9
Thus, m1 =
3 3y + 2x + 5 = 0 C.
m1m2 = –1
−4 2013/38 f/m
It follows that m2 = − 1
3 Alternative method
–4m2 = –3 The line equation perpendicular to
3
m2 = 3x – 2y + 4 = 0 is
4
3 y −3 3y + 2x = k at (2, –3)
Equation of line is = Substituting for x = 2, y = –3
4 x+2
3y + 2x = k becomes
3(x + 2) = 4(y – 3)
3(–3) + 2(2) = k
3x + 6 = 4y – 12
–9 + 4 = k
4y – 3x – 18 = 0 C.
–5 = k
The equation is 3y + 2x = –5
2014/36 UTME
3y + 2x + 5 = 0 C.
Alternative method
The line equation perpendicular to
2013/10 f/m
4x + 3y –5 = 0 is
Find the equation of the line that is perpendicular to
4y – 3x = k at (–2, 3)
2y + 5x – 6 = 0 and bisects the line joining the
Substituting for x = –2 and y = 3
points P (4, 3) and Q (–6, 1)
4y – 3x = k becomes
A. y + 5x + 3 = 0 B. 2y – 5x – 9 = 0
4(3) – 3(–2) = k
C. 5y + 2x – 8 = 0 D. 5y – 2x – 12 = 0
12 + 6 = k
Solution
18 = k 1 1
The line equation is 4y – 3x = 18 Mid point of PQ coordinates = 2
(x1 + x2 ) , 2
(y2 + y1)
1 1
4y – 3x – 18 = 0 = (−6 + 4) , (1 + 3)
2 2
i.e 3x – 4y + 18 = 0 C. = (–1 , 2)

423
The line equation perpendicular to 1994/37 PCE Exercise 25.13
2y + 5x – 6 = 0 is Find the equation of the line through ( 1, 1 )
2x – 5y = k at (–1, 2) perpendicular to 2x – 3y = 4
A.3x – 2y = 5 B.3x + 2y = 5 C.2x – 3y = 5 D.2x + 3y = 5
Substituting for x = –1 and y = 2
2x – 5y = k becomes 2005/7 PCE Exercise 25.14
2(–1) – 5(2) = k For what value of t are the lines joining points (– 3, 2 )
–2 –10 = k and ( 2, 1 ) as well as ( 5, 4 ) and ( 6, t ) perpendicular ?
–12 = k A.9 B.6 C.2 D.1
Thus the new line is 2x – 5y = –12
2x – 5y + 12 = 0
i.e 5y – 2x – 12 = 0 D.
Ordinary Intersection
2013 / 32 UTME 2007/44 UME
Find the equation of the perpendicular bisector of the k
If the lines 2y – kx + 2 = 0 and y + x – = 0 intersect at
line joining P(2, –3) to Q(–5, 1) 2
A. 8y + 14x + 13 = 0 B. 8y – 14x + 13 = 0 (1, –1), find the value of k.
C. 8y – 14x – 13 = 0 D. 8y + 14x – 13 = 0 A–4 B–3 C–2 D–1
Solution
Solution At intersection, the two lines are equal
1 1
Mid point coordinates = 2
(x2 + x1 ) , 2
( y2 + y1 ) 2y – kx + 2 = y + x –
k
1 1 2
= (–5 + 2) , ( 1 – 3) k
2 2
2y – y – kx – x + 2 + =0
=  −3  2
 , − 1
 2  k
y – kx – x + 2 + =0
Equation of line through two points P(2, –3) and Q(–5, 1) 2
y − y1 y 2 − y1 Substituting for (1, – 2) i.e x = 1 and y = – 2
=
x2 − x1 k
x − x1 –2–k–1+2+ =0
y+3 1 + 3
2
= 1
x−2 −5 − 2 –1–k+ k=0
2
y +3 4 1
= − – k=1
x −2 7
2
7(y + 3) = – 4(x – 2) –k=2
7y + 21 = – 4x + 8 k=–2 C.
7y = – 4x + 8 – 21
7y = – 4x – 13
4 13 Equation of a straight line through a given angle
y = – 𝑥 −
7 7 is Gradient = tan 
The bisector of the line PQ is perpendicular to it Also the Acute angle between two lines is
m1m2 = –1 m1 − m2
4 13
tan  =
From y = – 𝑥 − 1 + m1m2
7 7

m1 = − 4 2009/24 UME
7
Find the acute angle between the straight lines
It follows − 4 m2 = − 1 y = x and y = 3 x
7
A 150 B 300 C 450 D 600
7
m2 = 4
Solution
Acute angle between two lines is
Equation of bisector  −3


, − 1
 2  m1 − m2
tan  =
7 1 + m1m2
4
= y +1
x +
3 y=x (m1 = 1)
2
y= 3 x (m2 = 3 )
3
7(x + ) = 4( y + 1)
21
2
tan  = 1 − 3
7x + = 4y + 4 1+ 3
2
14x + 21 = 8y + 8 Rationalizing the surd
8y – 14x – 13 = 0 (A)
424
(1 − 3 ) (1 − 3 )
= Collinear points
(1 + 3 ) (1 − 3 )
If the three points lie on same straight line then they are
1− 2 3 + 3 4 − 2 3 collinear, hence same gradient
= =
1− 3 2
tan  = 2 – 3 2014/35 f/m
= 2 – 1.732 If the point (– 1, t – 1), (t, t – 3) and (t – b, 3) lie on the same
tan  = 0.268 straight line, find the values of t
 = tan – 1 0.268 A t = – 2 and – 3 B t = 2 and 3 C t = – 2 and 3
= 150 A. D t = 2 and – 3
Solution
2012/37 f/m If the three points lie on same straight line then they are
Find the acute angle between the lines collinear, hence same gradient
2x + y = 4 and – 3x + y + 7 = 0 Let P(– 1, t – 1), Q(t, t – 3) and R(t – 6, 3)
A 400 B 440 C 450 D 540 y 2 − y1
Applying the gradient formula for two point =
Solution x 2 − x1
Acute angle between two lines is
Gradient PQ = Gradient QR
m1 − m2
tan  = t − 3 − (t − 1) 3 − (t − 3)
1 + m1m2 =
t − (−1) t −6−t
2x + y = 4  y = – 2x + 4 (m1 = – 2)
– 3x + y + 7 = 0  y = 3x – 7 (m2 = 3) t − 3 − t +1 3 − t + 3
=
−2−3 t +1 −6
tan  =
1 + (−2)(3) −2 6−t
=
=
−5
=
−5
=1
t +1 − 6
1− 6 − 5 – 2  (– 6) = (t + 1) (6 – t)
tan  = 1 12 = t(6 – t) + 1(6 – t)
 = tan – 1 1 12 = 6t – t2 + 6 – t
= 450 C. 12 = 5t – t2 + 6
t – 5t + 6 = 0
2

Factorizing
1997/33 UME t2 – 2t – 3t + 6 = 0
The angle between the positive horizontal axis and a t (t – 2) – 3(t – 2) = 0
given line is 1350. Find the equation of the line if it (t – 3) (t – 2) = 0
passes through the point ( 2, 3 ) t – 3 = 0 or t – 2 = 0
A. x – y = 1 B. x + y = 1 C. x + y = 5 D. x – y = 5 t = 3 or 2 ( B )
Solution
Equation of a line through a point is given as
m= y – y1
x – x2
But gradient ( m ) = tan 
m = tan 1350
Tan 1350 falls into the 2nd quadrant with formula
( 180 – 0 ) and tangent is negative there
= – tan ( 180 – 135 )
= – tan 45
= –1
Substituting
–1= y –3
x–2
– 1( x – 2 ) = y – 3
–x+2=y–3
y+x=3+2
y+x= 5 (C)

425
Application of coordinates geometry and matrix to 4 (3, 4)
area of triangle and quadrilaterals
(1, 1)
Area of triangle whose vertices are (x1, y1), (x2, y2), (x3, y3) 1
1 1 1
= 1 1 3 4 5
x1 x2 x3
2
y1 y2 y3
-2 (5, -2)
1998/11(iii) (Nov)
Given the points P(11, 13), Q(-1, 12) and R(1, -2), find
the area of the triangle PQR (4, -7)
Solution -7

1 1 1
Area of triangle PQR = 1 1 1 1 1 1 1
11 − 1 1 1
2 Area of a quad = 1 4 3 + 1 4 5 3
13 12 − 2 2 2
1 −7 4 −7 −2 4
= 12 {1[(–1)× (–2) – 12×1] –1[11×(–2) –(13 ×1)]
+1[11×12 – 13 × (–1) } ={ 12 [(16+21) –1(4 – 3) + (–7– 4)]} + 12 {(20+ 6) –
= 12 { (2–12) – (–22– 13) + (132 +13)} 1(16 + 21) + ( –8+ 35)}
1 1
= 2 (37-1-11) + 2 (26-37+27)
= 12 { –10 + 35 + 145}
= 12 (41)
= 85 sq units
= 20.5 sq units

2009/29 Neco
Find the area of triangle ABC whose vertices are A(0,0)
B(3, 4) and C(2, 6)
A 20 sq units B 15 sq units C 10 sq units
D 7 sq units E 5 sq units
Solution
1 1 1
Area of triangle = 1 0 3 2
2
0 4 6
= 1
2 [ 1(3×6 – 4×2) –1×0 + 1×0 ]
= 12 (10)
= 5 sq units

Area of quadrilateral whose vertices are R(x1, y1),


S(x2, y2), T(x3, y3), U(x4, y4)
Quad RSTU = RST + ST U
1 1 1 1 1 1
1 1
= x1 x2 x3 + x1 x3 x4
2 2
y1 y2 y3 y1 y3 y4
A good sketch to locate the various coordinates will be
useful then we pick our values from the beginning
lower point going along the boundaries from right to
left

Example
Find the area of a quadrilateral with vertices
(1, 1), (3, 4), (5, -2), and (4, -7)
Solution
We sketch as:

426
Chapter twenty six Eg3. Our previous example ( i.e Eg1 ) is a 3 × 2 matrix
Matrix 1 2 3
 
Matrix (matrices for plural) like every other topic in Eg4. B =  5 6 7  is a 3×3 matrix
Mathematics is to be covered from the very basics then  9 10 0 
we advance to the highest point that the syllabus  
permits. The test on this topic shall be simple but  1 y 4
technical as it is with all Mathematics questions- they  
Eg5. C =  x 6 7  is also 3×3 matrix
are not meant to consume your time.  0 0 2
A matrix is a simple mathematical structure that holds  
numerical information in a rectangular form usually 11 25 
Eg6. D =   is a 2×2 matrix
enclosed by bracket 1 0
Eg1. A college has its number of students in each of 1
the SS classes classified as follows:  
Eg7. M =  2  is a 3×1 matrix ( column matrix )
Male female
 3
SS1 14 8  
SS11 6 12 Eg8. N = (4 5 6 10) is a 1×4 matrix( row matrix)
SS111 21 18
The above information can be represented in matrix form say
Matrices are usually denoted by capital letters while their
14 8 elements are in lower case.
 
Matrix A =  6 12  b b  b b 
 21 18  B =  11 12  and not b =  11 12 
  b21 b22   b21 b22 
Where the first row represents SS1, the second SS11
and so on. Similarly, the first column is the number of
male students and the second column for female Types of matrix
students. Thus it is important to retain each element in ( i ) Zero matrix
its original position when dealing with matrix. It is This is a type of matrix whose entries are all zeroes
customary to write a matrix in the form  0 0 0
 a11 a12 a13   0 0  
  A2  2 =   or B3 3 =  0 0 0 
A =  a 21 a 22 a 23  three by three matrix  0 0  0 0 0
a   
 31 a32 a32  ( ii ) Square matrix
b b12  This is a matrix whose number of rows equal to number of
B =  11  two by two matrix columns. i.e m×n matrix but m = n
 b21 b22 
1 4 5
The horizontal components  2 4  
A2  2 =   or B3 3 =  0 7 11 
( a11 a12 a13 ), ( a21 a22 a23 ) and 6 8 13 17 19 
 
( a31 a32 a33 ) are the rows while the vertical components
( iii ) When the elements’ row  column we say that the matrix
 a11   a12   a13 
      is non-square matrix
 a 21  or  a 22  or  a 23  10 15 
a  a  a  1 2 5  
 31   32   33  C =   D =  21 3 
are the column of the matrix . 3 4 6 0 1
 

Eg2. Identify the position of the elements listed below ( iv ) identity matrix – it is denoted by I or l
in 3 by 3 matrix: a22 = 12 and a31 = 24 1 0 0
1 0  
Solution I 2  2 =   I 3 3 =  0 1 0 
Element a22 being 12 i.e the element on the second row 0 1 0 0 1
and 2nd column.  
a31 = 24 i.e element on the 3rd row and first column Students should master the above pattern of generalizing the
Class discussion/self test identity matrix for 2×2 and 3 by 3.
Try to identify elements in a23, a32 and a12 of the matrix
in example 1 above.

Definitions
A matrix with m rows and n columns is called an
m by n matrix. Where m × n or m by n is called the
size or shape of the matrix.
427
Addition & subtraction of matrices Multiplication of matrix by a number
( scalar multiplication)
Eg. 1 If A = 
6 4  2 3
12 6  ; B =  4 5  1 2
Eg.1 If A =  find ( i ) 5A (ii) – A (iii) –2A
     3 4 
find ( i ) A + B ( ii ) A – B  
Solution Solution
 5  1 5  2   5 10 
6+2 4 + 3 8 7 ( i ) 5A =   =  
i. A + B =   =   5  3 5  4   15 20 
12 + 4 6 + 5  16 11
 −1 − 2 
(ii) − A =  
 6 − 2 4 − 3   4 1  −3 −4 
ii. A − B =   =  
12 − 4 6 − 5   8 1 Students/readers to find – 2A

Eg. 2 Perform C + D and C – D on the matrices below:  0 − 1


Eg2. If B =   find ( i ) –B ( ii ) – 3B (iii) – ½ B
 3 2 4  − 1 2 3  − 15 10 
   
C =  − 1 0 2  and D =  0 1 1  Solution
 − 3 1 6  0 2 1 ( i ) − B =  0 1  observe that 0 × (-1) = 0 no negative in zero
    15 − 10 
 
Solution
 2 4 7 Students/readers to find – 3B
 3 + (−1) 2 + 2 4 + 3   
   0 12 
C + D =  − 1+ 0 0 +1 2 +1  =  − 1 1 3  ( iii ) − 1 2 B =  
 − 3 + 0 1+ 2 6 +1 − 3 3 7  15 2 − 5 
   
 3 − (−1) 2 − 2 4 − 3  Eg3. The following matrix X gives the prices of comparable
   4 0 1
=   articles by size and type ( in N )
C − D =  − 1 − 0 0 −1 2 −1   − 1 − 1 1
 − 3 − 0 1 − 2 6 −1   − 3 − 1 5
A  0.40 0.10 
     
X = B  0.30 0.50 
Eg.3 Part-time teachers/school matrix has the structure: C  0.20 0.60 
Schools If all prices were increased by 50%, the new prices matrix Y will be?
A B Solution
Teachers 1 x x Normal prices 100%
Number 2 x x Increment 50%
3 x x New price = 100 + 50%
and it represents the number of periods taken by a = 150%
particular teacher in a particular school in a certain = 150 = 1.5
week. If Wi is the teacher/school matrix for week i and 100
4 0 3 1 2 0  0.40 0.10 
       
W1 =  0 0  ; W2 =  2 1  ; W3 =  2 4 Y = ( 1.5 )  0.30 0.50 
2 3  0 2 2 0  0.20 0.60 
       
find the value of  1.5  0.40 1.5  0.10   0.60 0.15 
   
( a ) W1 + 2 ( where W1 + 2 = W1 + W2 ) =  1.5  0.30 1.5  0.50  =  0.45 0.75 
( b ) W 1 + 2 + 3 ( c ) W 1 + 2 + 3 - W2  1.5  0.20 1.5  0.60   0.30 0.90 
Solution  
Students/readers poser
 4 + 3 0 + 1  7 1 If all the prices were decreased by 50%, the new price matrix Z
   
( a ) W1 + 2 = W1 + W2 =  0 + 2 0 + 1  =  2 1 will be?
 2 + 0 3 + 2  2 5 
  

7 + 2 1+ 0 
  9 1
( b ) W1 + 2 +3 = W1 + 2 + W3 =  2 + 2 1 + 4  =  
4 5
 2 + 2 5 + 0 4 5 
  

9 −3 1−1  6 0
   
( c ) W1 + 2 +3 − W2 =  4 − 2 5 −1 = 2 4
4 −0 5 − 2  4 3 
 

428
2014/15 Neco Eg.2 If the matrix X is used to hold the records of sets A, B
 −1 3 − 4 and C of books in a library and matrix Y to hold the unit cost
  ( N 000,000 ) each set as follows
Given that A =  2 − 6 3  , find 3A
 5 2 − 4  A B C  2.50 
 X = ( 10 25 5 ) Y =  0.50  , find XY
 − 1 3 − 4  3 9 12 
     0.20 
A  2 −6 3  B  6 18 9   
5 2 − 4  15 6 12   2.50 
    
 − 3 9 − 12   − 3 9 − 12  X Y = (10 25 5) •  0.50 
     0.20 
C  6 − 18 6  D  6 − 18 9  E  
 15 6 12   15 6 − 12  = 10 ( 2.50 ) + 25( 0.50 ) + 5( 0.20 )
 
= 25 + 12.5 + 1.0
 2 6 − 1
  = N ( 38.5 ) 000 000
5 − 3 6  = N 38 500 000
 8 5 − 1
  This is the total cost of all the books in the library.
Solution The principle in the example above is to be applied always in
 − 1 3 − 4 − 3 9 − 12  multiplying matrices i.e. row by column operation.
   
3A = 3  2 − 6 3  =  6 − 18 9  D.
 15 Pattern of movement
5
 2 − 4   6 − 12 

 2 3 5 7 
Eg. 3 If A =   and B =   find AB
2001/20 UME Exercise 26.1 6 4  9 11 
3 − 2 4 
If   then -2P is Solution
P = 5 0 6
 7 5 − 1  2  5 + 3  9 2  7 + 3  11
  A • B =  
 − 6 4 − 8  −6 4 − 8  6  5 + 4  9 6  7 + 4  11 
A.  5 0 6 
 B.  − 10 0 6 

 10 + 27 14 + 33   37 47 
 =   =  
 7 5 − 1
 
 − 14 5 − 1 
   30 + 36 42 + 44   66 86 
 −6 −4 2   −6 4 −8 
C.   D.  
 − 10 − 2 − 12   − 10 0 − 12  1 8 3 4
 − 14 − 10 Eg.4 If X =   and Y =   find XY
2   − 14 − 10 2 
  0 2 5 6
 1 3 + 8  5 1 4 + 8  6 
X Y =  
 0  3+ 2  5 0  4 + 2  6
 3 + 40 4 + 48   43 52 
=   =  
 0 + 10 0 + 12   10 12 
Multiplication of matrix by a matrix Students/readers’ poser
3 4 1 6
Multiplication of a row matrix by a column matrix can If A =   and B =  
be done by combining the matrix as follows: 1 2 0 5
 b1  find ( i ) A B ( ii ) A2 = ( i.e A2 = A × A )
A = (a1 a2 ) and B =   (iii ) B2
( iv ) show that AB  BA i.e multiplication
 b2  of matrix is not commutative, but
A × B = A•B = a1b1 + a2 b2 AA = AA if they are the same
3
Eg.1 If A = (1 2) and B =   find AB
4  General discussions/observation
Solution Condition under which multiplication is possible with matrix
AB = 1 × 3 + 2 × 4 is once number of rows = no of columns. From the list
=3+8 below, identify as many as possible two matrices that fulfill
= 11 such condition
d  2 10 
Also if C = (c1 c2 c3 ) and D =  d     
1
0
2 A =  3  B =  7  , C = (1 0 3) , D =   , E = (8 1)
,
d  4 8 5 
 3    
Multiplication of CD is possible

429
1 0 1 2 3  1 0 2
Solution
F =   , G =  0 4 10  , H =   , 5
5 6 0 1 5  
1 0 5  (2 3 1) •  6  = 2  5 + 3  6 + 1  7
 
7
1 2  
  = 10 + 18 + 7
J = 3 4
5 6 = 35 A.
  2002/19 Neco f/m
15 
K =   , L = (0 17) , M = 
7 0 2 1 2 
 ,  
3 9 1 If A =  1 2 − 2  , What is A3 ?
 4 9 0  2 − 2 −1 
N =    
0 2 1
some are CA, ED, GA, FD, LK and so on. A 3A B 6A C 8A D 9A E 10A
2014/12 Neco (Dec) Solution
 1 − 3  5 1 A3 is same as A2 × A and A2 is A × A
Given that A =   and B =   , evaluate
4 2   −1 0 2 1

2 

2 1

2 

AB A × A =  1 2 − 2  1 2 − 2
2 − 2 −1 2 − 2 −1
 8 1  2 1  8 1    
A   B   C  
 22 4   18 4   18 4   4 + 1 + 4 2 + 2 − 4 4 − 2 − 2
 
 6 − 2   8 1  =  2 + 2 + −4 1 + 4 + 4 2 − 4 + 2 
D   E    4 − 2 − 2 2 − 4 + 2 4 + 4 + 1
 3 2   18 − 4   
Solution
9 0 0
 1 − 3  5 1   
A  B =     = 0 9 0
 4 2   −1 0 0 0 9
 
1 5 + (−3)  (−1) 11 + (−3)  0 
=   Then A3 = A2 × A
 4  5 + 2  (−1) 4 1 + 2  0  9 0 0  2 1 2 
   
5 + 3 1 =  0 9 0  1 2 − 2
=  
 20 − 2 4  0 0 9  2 − 2 −1
   
 8 1 18 + 0 + 0 9 + 0 + 0 18 + 0 + 0 
=   C.  
18 4  =  0 + 9 + 0 0 + 18 + 0 0 − 18 + 0 
2009/32 f/m  0 + 0 + 18 0 + 0 − 18 0 + 0 − 9 
 
 2 3  2  18 18 
Evaluate     
9

 4 1  3  = 9 18− 18  i.e. 9A (D)
18 − 18 − 9 
13   11  
A (13, 11) B (11, 13) C   D  
 11 13 
Solution
2010/9b
 2 3   2   2  2 + 3  3
    =    2 0 1  1   k 
    
 4 1   3   4  2 + 1 3  Given that  5 − 3 1   m  =  2 
 4 + 9  0 4 6   r   26 
    
=  
 8 + 3  find the values of constants k, m and r
Solution
13 
=   C.  2 1 + 0  m +

1 r   k 
  
11
   5 1 + (−3)  m + 1 r  =  2 
 0 1 + 4  m + 6  r   26 

2015/56 Neco 2 + 0 + r k
Calculate the product of the matrix    
 5 − 3m + r = 2 
5  0 + 4m +
   6r   26 
(2 3 1) and  6 
7 Equating rows
  2 + 0 + r = k ------ (1)
A 35 B 23 C 18 D5 E2 5 – 3m + r = 2 ----- (2)
430
0 + 4m + 6r = 26 ---- (3) 2 1 1 0
Solving (2) and (3) f(x) = 3x – 2I and P =   , I2 =  
r – 3m = – 3 ---- (a)  −1 0 0 1
6r + 4m = 26 ----- (b)  2 1 1 0
f(p) = 3   – 2  
(a)  4 + (b)  3
 −1 0 0 1 
4r – 12m = – 12
+ 18r + 12m = 78  6 3  2 0 
=   −  
22r = 66  − 3 0  0 2
r=3  4 3 
Substitute r = 3 into (a) =   D.
3 – 3m = – 3  − 3 − 2
3 + 3 = 3m 1994/11
6 = 3m and m = 2  2 1 1 −1 
Substitute r = 3 into (1) If P =   and Q =   , find PQ + 3Q
2+3=k  0 1 3 − 2
5=k A  0 3 B  0 − 3 C  2 − 1 D  8 − 7  E  2 − 7 
    
 − 6 − 9  − 6 4 
 
 − 6 5    12 − 8  − 6 − 8
2005/44 PCE Exercise 26.2 Solution
 1 3 3 2
If P =   and Q =  ,  2 1  1 − 1  1 −1 
− 4 − 2 2 1  PQ + 3Q =     + 3  
what is the matrix PQ?  0 1  3 − 2  3 − 2
− 3 5   9 5   2  1 + 1  3 2  (− 1) + 1  (− 2)  3 − 3
A   B   =   +  
 8 10   − 16 − 10   0  1 + 1  3 0  (− 1) + 1  (− 2)  9 − 6
 − 3 8  3 6
C   D    2 + 3 − 2 − 2  3 − 3
 − 6 2  − 8 − 2  =   +  
2007/23 PCE Exercise 26.3 0 + 3 0 − 2  9 − 6
 3 1  5 2 5 − 4  3 − 3  8 − 7
If R =   and 5 =   , find RS =   +   =   (D)
 2 5 − 2 4 
3 − 2 9 − 6 12 − 8 
13 10   17 10 
A   B   2015/8 f/m
 0 24   20 24  − 2 1  5 − 3
Given that P =   and Q =   , find PQ – QP
 13 10  17 10   3 4  2 − 1
C   D  
 20 24   0 24  A 
0 0
 B 
 27 12 
 C 
 − 20 − 6 

 11 12 
D  
0 0  16 −15   12 − 8   30 −11
2009/24 PCE Exercise 26.4 Solution
 2 3 3 2  − 2 1   5 − 3
If Q =   and R =   , find QR PQ =    
1 2 1 3  3 4  2 − 1
 9 13   5 6  8 9  9 5  (−2)  5 + 1 2 (−2)  (−3) + 1 (−1) 
A   B   C   D   =  
5 8  13 9  13 5  13 8   3 5 + 4  2 3  (−3) + 4  (−1) 
 − 10 + 2 6 − 1
=  
 15 + 8 − 9 − 4 
Joint cases of Addition, subtraction and −8 5 
multiplication PQ =  
 23 − 13 
2007/25 UME
 5 − 3  − 2 1 
 2 1 QP =    
If f(x) = 3x – 2I, P =   and I is 2  2 identity  2 − 1  3 4
 −1 0
matrix, evaluate f(p)  5  (−2) + (−3)  3 5  1 + (−3)  4 
=  
 6 3  2 0  8 3  2  (−2) + (−1)  3 2  1 + (−1)  4 
A   B   C   D
 − 3 0  0 2  − 3 2  − 10 − 9 5 − 12 
=  
 4 3  − 4 − 3 2 − 4 
   − 19 − 7 
 − 3 − 2 =  
Solution  − 7 − 2

431
−8 5   − 19 − 7   4 2   − 4 8
Thus, PQ – QP =   −   =   +  
 23 − 13   − 7 − 2  10 − 6   − 1 2 
 11 12   0 10 
=   D. =   D.
 30 −11 9 − 4
2009/29 f/m 2007/24 PCE
 1 1 2 0  4 2
If P =   , find (P2 + P) If X =   and Y =   , find 2X – Y
 2 1  3 −1   −1 3 
 4 3   4 3  3 2  3 2 8 2   0 − 2
A   B   C   D   A   B  
 6 1  6 4 6 1  6 4  5 − 5 7 − 5
Solution  0 2  − 6 − 4
 1 1  1 1 C   D  
P2 =     5 1  5 − 7
 2 1  2 1 Solution
1  1 + 1  2 1  1 + 1  1 
=   2 0 4 2
 2  1 + 1  2 2  1 + 1  1 2X – Y = 2  –  
3 −1 −1 3 
1 + 2 1 + 1   3 2
=   =   4 0  − 4 − 2
 2 + 2 2 + 1  4 3 =  + 
 3 2  1 1 6 − 2   1 − 3 
P2 + P =   +  
 4 3   2 1 0 − 2
=   B.
 4 3
=   B. 7 − 5 
 6 4

2003/21 f/m 2013/20 Neco f/m


 2 − 1 Given that A = 
−3 1
 and B =
2

− 1

Given that P =   and I is a 2 by 2 identity 2  4 
− 3 3   4 3
3
matrix, find P + 2I evaluate A + B
2
 5 − 2  4 1
A   B   5 1
− 6 7   −1 5 A  2 − 2  B 
 − 9 3

 2 1  4 − 1 6 3   12 6 
C   D  
 − 1 3 −3 5  −5 1 5 1  −5 1
Solution C  2 2
 D  2 − 2  E  2 2

 7     7 
1 0  9 9 7  −9
I a 2  2 identity matrix =   Solution
0 1 −3 1 2 − 1
3 3
 2 − 1 1 0 A + B =   +  
P + 2I =   + 2   2 2  4 2 3 4 
− 3 3  0 1  −9 3  2 − 1
 2 − 1  2 0  =  2 
2  +  
=   +    6 3  3 4 

 − 3 3   0 2
 −5 1
 4 − 1 =  2 
2  C.
=   D.  9 7 
− 3 5  

2012/16 f/m
2 1  4 − 8
Given that P =   and Q =   , find (2P – Q) 2007/27 UME
 5 − 3  1 − 2  3 − 2 − 2 4
 − 6 17   − 2 9 0 − 6  0 10  Evaluate   + 2 
A   B   C   D   − 7 5  3 − 1
 3 1  4 1 9 − 8 9 − 4  3 4  − 1 6
Solution A   B  
 − 2 6  − 1 3
2 1  4 − 8
2P – Q = 2   –    −1 6  3 4
C   D  
 5 − 3  1 − 2   1 3   2 6

432
Solution 2012/10 f/m
 3 − 2  − 2 4   3 − 2 − 4 8  (a) Write down the matrix A of the linear transformation A
  + 2  =   +  
 − 7 5   3 − 1 − 7 5   6 − 2
(x, y) (2x – y, – 5x + 3y)
 −1 6 3 1
(b) if B =  
=   B.
 −1 3 5 2
find (i) A2 – B2 (ii) Matrix C = B2A
2015/11 Neco
(c) What is the relationship between matrix B and matrix C
3 0 1 2 3 1
    Solution
If P =  2 4 3  and Q =  5 1 2  find 2P – Q A (x, y) (2x – y, – 5x + 3y)
5 0 0   3 4 2
   Coefficients only
6 0 2  4 −3 1   2 − 1  x1 y1 
    Matrix A =   i.e  
A 4 8 6 B −1 7 4  −5 3   x2 y 2 
10 00   7 − 4 − 2
    2 − 1  2 − 1
A2 =    
4 3 1
 


8 3 3 

 1 −3 0 
  −5 3  −5 3 
C 1 7 4 D  9 9 8 E − 3 3 1 
7 4 2 13 4 2   2 − 4 − 2  2  2 + (−1)  (−5) 2  (−1) + (−1)  3 
=  
(−5)  (−1) + 3  3 
     
Solution  (−5)  2 + 3  (−5)
 3 0 1  6 0 2  4 + 5 −2 − 3 
 − 2 − 3 −1  =  
       − 10 − 15 5 + 9 
2P = 2  2 4 3  =  4 8 6  and –Q =  − 5 −1 − 2
 5 0 0  10 − 5
   0 0   − 3 − 4 − 2
 
 9
=  
 − 25 14 
 6−2 0−3 2 −1  4 −3 1 
    3 1 3 1
2P – Q =  4 − 5 8 − 1 6 − 2  =  − 1 7 4  B. B2 =    
10 − 3 0 − 4 0 − 2   7 − 4 − 2 5 2 5 2
  
2001/26 Neco f/m  3  3 + 1 5 3 1 + 1 2 
=  
 b

4 1   4 p 2
  
7 4 k 
  5  3 + 2  5 5  1 + 2  2 
If  − 7 m 2  +  − 3 6 l  =  c − 2 4   9 + 5 3+ 2   14 5
 1 − 3 n  a 1 5 4 j 6 =  =  
      15 + 10 5 + 4   25 9 
m n p
   9 − 5   14 5 
then  k l j  is A2 – B2 =   −  
a b c
   − 25 14   25 9 
 3 3 − 10  − 8 1 0   − 5 − 10 
    =  
A − 8 1 0  B  3 − 10 − 2   − 50 5 
 3 2 −2   3 3 2 
   (ii) Matrix C = B2A
 3 3 3  − 8 1 0  − 8 3 3   14 5   2 − 1
      =    
C  − 8 − 2 − 10  D  3 2 − 2  E  1 2 3  25 9   − 5 3 
 1 2   3 3 − 10   0 − 2 10 
 0     14  2 + 5  (−5) 14  (−1) + 5  3 
=  
Solution  25  2 + 9  (−5) 25  (−1) + 9  3 
 b 4 1   4 p 2  7 4 k 
       28 − 25 − 14 + 15   3 1 
 − 7 m 2 +  − 3 6 l  =  c − 2 4 =   = 
 1 − 3 n  a 1 5  4 j 6  50 − 45 − 25 + 27   5 2 
     
Solving column by column (c) The relationship is matrix B is the same as matrix C
b+4=7 thus b = 3
–7–3=c thus c = – 10 2009/12b f/m Adjusted Exercise 26.5
1+a=4 thus a = 3 2 1
4+p=4 thus p = 0 If A =   , find T = A2 + A + 2I,
 3 2 
m+6=–2 thus m = – 8
–3+1=j thus j = – 2 where I is the 2  2 units matrix
1+2=k thus k = 3 2013/22 UTME Exercise 26.6
2+L=4 thus L = 2  5 3  4 2
n+5=6 thus n = 1 If P   and Q =   , find 2P + Q
m n p − 8 1 0   2 1  3 5
    7 7   8 14   7 7 14 8 
Thus  k l j  =  3 2 − 2  D A   B   C   D  
a b c  3 3 − 10   8 14  7 7  14 8   7 7
   

433
2004/22 PCE Exercise 26.7 − 4 5 
=  
1 4 0 4   5 − 12 
If R =   and S =   , find 3R + S
3 2  1 − 5 Thus (A + AT )T = 
− 4
 C.
5 
 3 16   3 16   16 3  16 3   5 − 12 
A   B   C   D  
10 −1 10 1   − 10 1   1 −10  Exercise 26.10
Find the transpose of the matrices below
 4 0 13 1 0 
2006/40 PCE Exercise 26.8 P =   , Q = (10 0 1) , K  
= 15 4 2 
 2 3  3 − 2  1 3  0 1 7
If P =   and R =   , find the value of 3P – R  
 − 3 1 2 −1
2006/38 PCE Exercise 26.11
 3 − 11  4 9  3 11 5 4 
A   B   C   D    3 6
11 4 − 8 6 − 11 4  4 − 8
Find the transpose of the matrix  
     
−5 − 4 
 6 3  3 6   − 5 − 4 3 − 5
2006/18 UME Exercise 26.9 A   B   C   D  
 − 5 4  − 4 − 5   6 3  6 − 4
 1 0 1  −1 1 2
   
If x =  2 − 1 0  and y =  0 −1 −1 
 −1 0 1   2 −1 1 
 
Solving simultaneous linear equations with
find 2x – y
two unknowns using 2×2 matrix
 3 −1 0  3 −1 0
    2002/1 f/m
A  4 − 3 −1 B 4 −1 1
− 4 Express
 1 1  − 4
 1 1 
2x + 5 y = 3
 3 −1 0  3 −1 0
8 x + 7y = 5
   
C  4 − 3 1 D  4 1 1 in matrix form
 − 4 1 1 − 4 − 1 1 
    x  2 5  3  2 5
A     =   B (x, y )   = (3, 5)
 y 8 7  5 8 7
Transpose of a matrix  2 5   x   3 2 8  x   3
t C     =   D     =  
Defn: The transpose of a matrix A, written A is the  8 7   y   5 5 7  y   5
matrix obtained by writing rows of A in orders as Solution
columns
Since matrix multiplication is not commutative
1 2  1 3
If A =   then At =   2x + 5 y = 3
3 4  2 4 8 x + 7y = 5
6 Can only be represented as
If B = (6 11) then Bt =    2 5   x   3
11     =   (C)
0 1 4  0 7 3   8 7   y   5
   
If C =  7 10 2  then C =  1 10 5 
t

 3 5 16   4 2 16  2014/22 UTME
   
 5 − 6  x   7 
Find y, if     =  
2003/29 Neco  2 − 7   y   − 11
− 2 4  A3 B2 C8 D5
Given that A =   obtain (A + AT )T
 1 − 6  Solution
− 4 8  − 4 2   5  x + (−6)  y   7 
A   B     =  
 2 − 12   8 − 12   2  x + (−7)  y   − 11
− 4 5   − 4 5 − 2 1   5x − 6 y   7 
C   D   E     =  
 5 − 12   − 12 5   4 − 6  2 x − 7 y   − 11
Solution Equating terms
− 2 4  − 2 1  5x – 6y = 7 ---- (1)
A =   and AT =   2x – 7y = – 11 ---- (2)
 1 − 6   4 − 6 Solving the resulting simultaneous equations
− 2 4  − 2 1  (1)  2 – (2)  5 to eliminate x
A + AT =   +   10x – 12y = 14
 1 − 6  4 − 6
−(10 x − 35 y = −55)
23 y = 69
y=3 (A)
434
2004/21 PCE Solution
 1 2   − 1 1   3 11  3  2 + 2  3   12 
Given that     =   , find k   =  
 k 1   2 5  5 2   7  2 + x  3   29 
A–5 B–3 C6 D7  6 + 6   12 
  =  
Solution 14 + 3x   29 
1  (−1) + 2  2 1  1 + 2  5   3 11 Equating terms in x
 = 
 k  (−1) + 1  2 k  1 + 1  5   5 2  14 + 3x = 29
 −1+ 4 1 + 10   3 11 3x = 29 – 14
 = 
− k + 2 k + 5   5 2  3x = 15
3x 15


3 11   3 11
= 
= thus, x = 5 (A)
− + 2 k + 5   5 2  3 3
 k
Equating terms in k
2005/12c adjusted
–k+2=5
1 2  − 3 2
Or k + 5 = 2 If A =   and B =  
k=2–5 =–3 (B) 3 4  1 4
Find C such that : AC = B
2014/18 Neco f/m
Solution
 3 x + 2 y   3 10 
If  =  find x and y We list the C a 2×2 matrix such that AC = B
 3x − y xy   2 8  1 2  a b   − 3 2
A (2, 4) B (2, 5) C (3, 4) D (3, 5) E (5, 2)     =  
3 4  c d   1 4
Solution
Equating terms  a + 2c b + 2d   − 3 2 
  =  
x + 2y = 10 ----- (1)  3a + 4c 3b + 4d   1 4 
3x – y = 2 ------ (2) Equating entries
xy = 8 ------ (3)
a + 2c = –3 b + 2d = 2
Solving (1) and (2); (2)  2 and add
3a + 4c = 1 3b + 4d = 4
x + 2y = 10
Solving simultaneously
(6 x − 2 y = 4) 3a + 6c = –9 3b + 6d = 6
7x = 14 – (3a + 4c = 1 ) – (3b + 4d = 4)
x=2 2c = – 10 2d = 2
Substitute x = 2 into (1) c = –5 d=1
2 + 2y = 10 becomes Substituting for c value Substituting for d
2y = 10 – 2 value
2y = 8 a = –3–2(–5) i.e 7 b = 2–2(1) i.e 0
y=4 thus, x, y = 2, 4 (A)
 7 0
2006/11 UME Thus C =  
2p 8   1   24   − 5 1
Find P, Q, for which     =  
 3 − 5q   2   − 17 
A – 4, – 2 B – 4, 2 C 4, – 2 D 4, 2 2009/28 (Nov) f/m Exercise 26.12
If 
Solution 5 x  2   4  , find (x, y)
 4 − 1   =  
 2 p  1 + 8  2   24     y   5
  =  
 3  1 + −5q  2   − 17  A (3, – 2) B (2, 3) C (–2, 3) D (–3, 2)
 2 p + 16   24 
  =   2006/29 Exercise 26.13
 3 − 10q   − 17   2 −3   −6   3 
Equating terms Given that     =   ,
3 – 10q = – 17 2p + 16 = 24  1 4   p   − 26 
3 + 17 = 10q 2p = 24 – 16 find the value of P.
20 = 10q 2p = 8 A–8 B–5 C–4 D–3
2=q p=4
Thus P, Q = 4, 2 ( D )
2011/18 f/m
 3 2   2   12 
If     =  
 7 x   3   29 
find the value of x
A5 B6 C7 D8

435
Determinant of matrix 2009/9 (Nov) f/m
The determinant of any matrix say B is denoted by B  x − 1
If the determinant of the matrix   is – 6,
or det B is a number computed by the format shown 2 4 
below; depending on the order of the matrix i.e 2×2 or find the value of x
3×3.If the value of matrix determinant is zero then the A–2 B–1 C1 D2
matrix is called singular matrix, otherwise it is non- Solution
singular and it is only non-singular matrix that has x  4 – 2  (–1) = – 6
inverse. 4x + 2 = – 6
4x = – 6 – 2
2013/10 Neco f/m Exercise 26.14 4x = – 8
A matrix is said to be singular if the determinant is 4x − 8
A negative B one C positive =
4 4
D undefined E zero x = –2 (A)
2×2 matrix determinant 2012/23 UMTE
a b  5 3 3 5
A =   then, If = , find the value of x
c d  x 2 4 5
det A = ad – bc A3 B4 C5 D7
Eg. ( i ) If A = 
3 4 Solution
 1 2 
find detA
  5 3 3 5
=
Solution x 2 4 5
det A = 3×2 – 1× 4 52 – 3x = 35 – 54
=6–4 =2 10 – 3x = 15 – 20
 2 6 10 – 3x = – 5
Eg.( ii ) If D =   find D 10 + 5 = 3x
 − 1 4  15 3x
Solution =
3 3
D = 2 × 4 – 6×(- 1) 5= x (C)
= 8 + 6 = 14 2003/7 f/m
2006/19 f/m 3− x 9
Which of the following matrices is a singular matrix? If = 0, find the two possible values of x
− 1 1 + 2x
1 0 2 1 3 8  0 1
A   B   C   D  
A 2 or – 3 B 4 or
−2
C 4 or
−3
D 3 or – 2
0 1  3 2  6 16  1 0 3 2
Solution Solution
A matrix is singular if det = 0 (3 – x) (1 + 2x) – 9(–1) = 0
3 8  3(1 + 2x) – x(1 + 2x) + 9 = 0
It is det of   = 3  16 – 6  8
 6 16  3 + 6x – x – 2x2 + 9 = 0
= 48 – 48 5x – 2x2 + 12 = 0
=0 (C) 2x2 – 5x – 12 = 0
2014/13 Neco Factorizing
 5 3 2x2 – 8x + 3x – 12 = 0
If x =   , find the determinant of x 2x(x – 4) + 3(x – 4) = 0
− 2 − 2 
(2x + 3)(x – 4) = 0
A – 16 B–4 C4 D8 E 16 2x + 3 = 0 or x – 4 = 0
Solution −3
Determinant of x = [5  (– 2) – (– 2)  3] x= or 4 ( C )
2
= – 10 + 6 2014/23 UTME
= –4 B −x 12
2007/25 PCE If = – 12, find x
−1 4
2 1
Evaluate A3 B6 C–6 D–2
4 6
Solution
A2 B4 C6 D8 [– x  4 – (–1)  12] = – 12
Solution – 4x + 12 = – 12
2 1 – 4x = – 24
= 26 – 41
4 6 −4 x −24
=
= 12 – 4 −4 −4
= 8 (D) x = 6 ( B)
436
2009/23 PCE 2013/15 f/m Exercise 26.16
 2 1
P =   , find the value of – 5P + 6I  y−2 y − 1
Given that P =   and  P  = – 23,
 0 3 y−4 y + 2 
A 41 B 36 C 31 D – 36 find the value of y
Solution A–4 B–3 C–1 D2
1 0
Here I implies 2  2 identity matrix  
0 1 2010/25 UMTE Exercise 26.17
 2 1 1 0 x 3
– 5P = – 5   and 6I = 6   If = 15, find the value of x
 0 3  0 1 2 7
 − 10 − 5  6 0 A3 B5 C4 D2
=   =  
 0 − 15  0 6
2009/22 UME Exercise 26.18
 − 10 + 6 −5+0 
– 5P + 6I =    9 −2 
 0+0 − 15 + 6  If Q =   , then  Q  is
− 7 4 
− 4 − 5
=   A – 50 B – 22 C 22 D 50
 0 − 9 

Thus, | – 5P + 6I | = [(– 4)  (– 9)] – 0  (–5) 2016/11 Neco Exercise 26.19


= 36 – 0 −3 3
If = 33, find the value of x + y
= 36 (B) x y
A –27 B –24 C –11 D–9 E–6
2009/23 UME
x +3 x + 2 Inverse of matrix (2×2 matrix)
If P =   , evaluate x
 x +1 x − 1 
if  P  = – 10 The inverse of a square matrix A denoted by A–1
A –5 B–2 C2 D5 is given by
d −b
Solution If A =  a b  then A−1 = 1  

 c d  
det A  − c a
 P  = – 10   
(x + 3) (x – 1) – (x + 1) (x + 2) = – 10
exchange of ‘a’ and ‘d’ and negation of ‘b’ and ‘c’
x(x – 1) + 3 (x – 1) – [x (x + 2) + 1 (x + 2)] = – 10
x2 – x + 3x – 3 – [x2 + 2x + x + 2] = – 10  4 5
Eg1. Find the inverse of matrix A =  
x2 + 2x – 3 – (x2 + 3x + 2) = – 10  2 3
x2 + 2x – 3 – x2 – 3x – 2 = – 10
Solution
– x – 5 = – 10
1  3 − 5  But det A = 3×4 – (5× 2 )
– x = – 10 + 5 −1
A =   =12 – 10 i .e 2
det A  − 2 4 
 
–x = –5
x=5 (D) 1 3 −5 
A−1 =  
2008/25 PCE 2 −2 4 
7 3  32 − 52 
If = 24, find x =  
x x  −1 2 
A3 B4 C6 D8 Readers to confirm that AA–1 = I
Solution It is true for all matrices ie. any matrix multiplied by its
7 3 inverse equals identity
= 24
x x
7x – 3x = 24
4x = 24 2010/27 UTME
24 2 − 3
x = =6 (C) If P =   , what is P – 1 ?
4 1 1 
1 3  1 3
2015/28 f/m Exercise 26.15    
1 + 2x −1 A 5 5 B 5 5
If = − 3 , find the value of x 1 2  −1 2
6 3− x    
5 5  5 5
−2  −1 3  −1 −3
A x = 3, – 2 B x = 4,    
3
C 5 2 D 5 5 
C x = – 4,
3
D x = 4,
−3  −1 5  −1 −2
   
2 2 −5 2  5 5 

437
Solution Solution
1  1 3 Taking the inverse of P −1 will give P i.e from indices law
P–1 =  
det P  − 1 2  ( P −1 ) −1 = P1 i.e P
det P = [2  1 – 1  (– 3)] 1 1 3
=2+3 inverse of P −1 =  
=5 det P − 1  0 1 
 1 3 But det P–1 = 1× 1 – ( 0× -3 )
 
= 1 1 3
P–1   =  5 5 B. = 1 – 0 i .e 1
5  − 1 2  −1 2
 
 5 5 1  1 3
=  
2008/24 PCE 1  0 1 
 8 − 4
Find inverse of    1 3
− 7 6  =   D
 0 1 
 3 −1   3 1  2 −1   2 1
       
A  10 5 B  10 5 C 5 5  D  2 5
2014/14 f/m Exercise 26.20
 −7 2   7 2  7 3  7 3
       
 10 5   20 5   20 10   10 10   − 2 − 5
If T =   , find T – 1, the inverse of T
Solution  3 8 
 8 − 4 1  6 4  − 8 − 5 − 8 5   − 8 − 5  − 8 − 5
Inverse of   =   A   B   C   D  
− 7 6  det  7 8   3 2   3 − 2 − 3 2   − 3 − 2
 8 − 4
But det of   = [8  6 – (– 7)  (– 4)]
− 7 6  2013/23 UTME Exercise 26.21
= (48 – 28) i.e 20  5 3
Find the inverse of  
 8 − 4  6 4 
1  6 4
Inverse of   =  
20  7 8  
 3  3
− 7 6   2   2 
2
A  2  B
 6 4   3 1 − 3 −5 − 3 5
       
=  20 20  =  10 5 . B  2   2
 7 8   7 2  −3  −3
     2   2 
 20 20   20 5 C 2  D 2 
− 3 −5  − 3 5 
   
2014/20 Neco (Dec)  2   2 
 2 4
Find the inverse of the matrix Q =  
−1 − 3  Solving simultaneous linear equations with two
− 1 3 − 4 −1  − 3 − 4 unknowns using 2×2 matrix (inverse method)
A   B  
2  1 2  2  1 2  If the equation is of the form
ax + by = c
− 1  3 − 4 −1  − 3 − 4 − 13 4
C   D   E   dx + ey = f
2  − 1 2  2  1 0  2  1 2 
The matrix form is then
Solution  a b  x   c 
2 4  1  − 3 − 4      =  
Inverse of   =    d e  y   f 
 − 1 − 3 det Q  1 2 
Readers to verify that the LHS is gotten by matrix
 2 4 multiplication to get to step 1 above
But det   = [2  (–3) – 4  (– 1)]
−1 − 3  −1
x  a b  c
= (–6 + 4) i.e – 2 Then solution   =    
−1  − 3 − 4  y  d e  f
=   B.
2  1 2  Eg. 1 Solve the simultaneous equations:
5x + 9y = - 3
6x + 2y = 2 using matrix format
Solution
2003/21 UME
The matrix expression is
 1 −3  9   x   −3
A matrix P has an inverse P −1 =   . Find P  5
0 1     = 
 
 6 2   y   2 
 −1 3   1 3   1 −3   1 3 −1
A   B   C   D   x   5 9  −3
 0 −1   0 −1   0 − 1   0 1 Then solution   =    
y   6 2   2 
438
2002/2 Exercise 26.23
−1
 5 9 1  2 − 9 det = 2×5 – ( 9×6 ) 3 0  x   − 6 
But   =       =  
 6 2 det  − 6 5  = 10-54 = -44
 3 1   y   − 10 
A. x = 2, y = – 4 B. x = –2, y = 4
1  2 − 9
= −   C. x = –2, y = –4 D. x = 2, y = 4
44  − 6 5 
2009/10a Neco f/m Exercise 26.24
If A = 
3 7 

x  1  2 − 9  − 3  − 4 2 
  = −   
y  44  − 6 5   2 
(i) find A– 1 (ii) Compute A.A– 1
1  − 6 − 18  (iii) Hence, using inverse method, (x = A– 1b)
=−  
44  18 + 10  solve the equation 
3 7   x   − 1
   =   .
 − 4 2  y   1 
1  − 24 
=−  
44  28  3×3 matrix determinant
 x  24
  b11 b12 b13 
 
  =  28 44
 B =  b21 b22 b23  then
 y  − 44   b 
7  31 b32 b33 
i.e x = 6 and y = − b22 b23 b21 b23 b21 b22
11 11 B = + b11 − b12 + b13
b32 b33 b31 b33 b31 b32
1997/( 23 ) UME = + b11 (b22 b33 − b23 b32 )− b12 (b21 b33 − b23 b31)+ b13 (b21 b32 − b22 b31)
Determine x + y if
 2 − 3   x   −1  The sign attached to b11, b12 b13 on and on is given by the
    =  chess board format or
 −1 4   y   8 
A. 3 B. 4 C. 7 D. 12
+ − +
  (−1) i + j where i is row and j is column
− + −
Solution + − + b11 = ( - 1 )1 +1 = ( - 1 )2 = + 1
 
−1 b12 = ( - 1 ) 1 + 2 = ( - 1 )3 = - 1
x   2 − 3   − 1 and so on
  =    
 y   −1 4   8 
2014/24 UTME
−1
 2 − 3 1  4 3 0 3 2
But   =  
 −1 4  det  1 2  Find the value of 1 7 8
3  det = 4×2 – (-3 × -1) 0 5 4
1 4
=   = 8-3 = 5 A–1 B–2
5  1 2  C 12 D 10
Solution
 x  1  4 3   −1  0 3 2
  =    
5  1 2   8 
7 8 1 8 1 7
 y  1 7 8 = 0 −3 +2
5 4 0 4 0 5
1  4  −1 + 3  8 0 5 4
=  
5  1 − 1 + 2  8  = 0 – 3[1 4 – 0  8] + 2[1  5 – 0  7]
= 0 – 3(4 – 0) + 2(5 – 0)
1  − 4 + 24  1  20  = 0 – 3(4) + 2(5)
=   =  
5  − 1 + 16  5  15  = 0 – 12 + 10 =–2 B.
x   4 2011/10 Neco f/m
  =   3 2 1
y   3  
Given that A =  6 2 1 
Thus x + y = 4 + 3 7 5 2
= 7(C)  
(a) Find the: (i) determinant of A
2004/2 Exercise 26.22 Solution
−3 2 1 6 1 6 2
Solve the equation    x  1
2
 1    =   Det A = 3 −2 +1
 3   y  2 5 2 7 2 7 5
1 1
A. x = –1, y = – /3 B. x = 1, y = – /2 = 3[2  2 – 5  1] – 2[6  2 – 7  1] + 1[6  5 – 7  2]
C. x = 1, y = /3 1
D. x = –1, y = /2 1 = 3(4 – 5) – 2(12 – 7) + 1(30 – 14)
= 3(– 1) – 2(5) + 1(16)
= – 3 – 10 + 16 = 3

439
2014/3 Neco f/m = 2[10  10 – 2  (–10)] + 4[0  10 – 6  (– 10)] +
1 + cos  1 + sin  1 8[0  2 – 6  10]
Evaluate 1 − sin  1 + cos  1 = 2(100 + 20) + 4(0 + 60) + 8(0 – 60)
1 1 1 = 2(120) + 4(60) + 8(– 60)
= 240 + 240 – 480
A 1 + cos  B 1 + sin  C sin  D cos  E1 = 0 QED
Solution
1 + cos  1 1 − sin  1 2015/11b Neco
= 1 + cos  – (1 + sin  ) +
1 1 1 1 2 3 −1
1 − sin  1 + cos  Evaluate the determinant 1 0 2
1
1 1
3 −2 3
= 1 + cos[(1+cos) 1 –1 1] – (1+ sin)[(1–sin) 1 – 11]
+ 1[(1 – sin)  1 – 1 (1 + cos)
Solution
= 1+cos(1+cos –1)–(1+sin)(1– sin–1) + (1- sin–1-cos) 2 3 −1
0 2 1 2 1 0
= 1 + cos (cos) – (1 + sin) (– sin) + (– sin – cos) 1 0 2 =2 −3 −1
= cos + cos2 + sin + sin2 – sin – cos −2 3 3 3 3 −2
3 −2 3
= cos2 + sin2
= 2[0  3 – (–2)  2] – 3[1  3 – 3 2] – 1[1  (–2) – 3  0]
= 1 (by trig identity)
= 2(0 + 4) – 3(3 – 6) – 1(– 2 – 0)
= 2(4) – 3(– 3) – 1(– 2)
2013/9 Neco f/m
=8+9+2
1 1 0
= 19
Given that 0 x −1 = 2. Find x
1 0 1 2010/26 UTME Exercise 26.25
A3 B2 C1 D0 E–3 2 0 5
Solution
Evaluate 4 6 3
x −1 0 −1 0 x
1 −1 +0 = 2 8 9 1
0 1 1 1 1 0 A – 42 B 102 C 18 D – 102
1[x  1 – 0(– 1)] – 1[0  1 – 1  (– 1)] + 0 = 2
1(x – 0) – 1(0 + 1) + 0 = 2 2009/11b f/m Exercise 26.26
1(x) – 1(1) = 2 2 −3 1
x–1 =2
x = 3 A. (i) Evaluate 0 1 −2
1 2 −3
2012/24 UTME
Given that I3 is unit matrix of order 3, find | I3 |
2015/10 bi f/m Exercise 26.27
A–1 B0 C1 D2
2 −1 2
Solution
1 0 0 Evaluate 1 3 4
  1 2 1
I3 =  0 1 0 
0 0 1
 
1 0 0 0 0 1
| I3 | = 1 −0 +0
0 1 0 1 0 0
= 1(1  1 – 0  0) – 0 + 0
= 1(1)
=1 C.

2013/9b Neco f/m


Show that the matrix A has no multiplicative inverse,
2 − 4 8 
 
where A =  0 10 − 10 
6 2 10 

Solution
We are to show that | A | = 0
10 − 10 0 − 10 0 10
|A|= 2 − (−4) +8
2 10 6 10 6 2
440
Chapter twenty seven Then A. B = AB
Vectors
A vector is a quantity that has both magnitude and A B
direction. It is represented by a small arrowhead on top
of a letter showing the direction PQ ; underlining of A. B = AB cos 0
letter r , or boldly written letters t are other forms of A. B = AB
representing vectors.
We denote the vector in the direction P to Q by PQ ( 5 ) If two vectors are opposite to each other(  = 180 )
and the vector in the direction Q to P by QP Then A. B = − AB

r 0
1
80
Q

B A
PQ P
P
A. B = AB cos 180
QP
Q A. B = − AB
The following terms are relevant to the concept of
Representation of vectors in three dimensions
vector
A vector A has 3 components in x, y, z directions. These
Scalar product directions are perpendicular to each other where i, j, and k
are the unit vectors
Suppose we have two vectors forming an angle as y
shown below
j
A
s
co

i
B

x
B z k
A
s
co

B
A

i . j = ijcos 900 i.e 0


B
A Hence we do not multiply unlike terms (vectors) and their
coefficients as we have seen that cos 900 = 0
( 1 ) The scalar product or dot product
Vector product
A. B = AB cos 
The vector product or cross product of two vectors

( 2 ) Cosine of angles between two vectors


A and B which is denoted as A × B and is defined as the
product of the magnitude of the vectors A and B and the
A and B
sign of their included angle 
A. B
Cos = A × B =AB Sin
AB
A B
( 3 ) If two vectors A and B are perpendicular ( i ) Sin  =
A B
to each other (  = 900 ) Then A • B = 0

( ii ) If two vectors A and B are perpendicular(orthogonal)


A to each other ( = 900 )
900 A × B =AB Sin 900 since Sin 900 = 1
B A × B =AB
( iii ) If two vectors A and B are collinear or parallel to
A. B = AB cos 90 0 each other ( = 00 )

A• B =0 A × B =AB Sin 00 since Sin 00 = 0


A×B = 0
( 4 ) If two vectors have the same direction ( i.e  = 0 ) A = a1 i + a2 j + a3 k
441
B = b 1 + b2 j + b 3 k Unit vector
Then 1
u = a
A + B = (a1 + b1 ) i + (a2 + b2)j + (a3 + b3)k a
A – B = (a1 – b1 ) i + (a2 – b2)j + (a3 – b3)k

Also by determinant method


Resultant of vectors
Resultant of vectors is the addition of the vector involved
i j k
A  B = a1 a2 a3 same applies to 2×2 Projection of vector
b1 b2 b3 Projection of vector a on b
b a
P= a , Projection of vector b on a P= b
Basic theorem to work with b a
Two non-zero vectors a and b are parallel if and if  a
scalar t such that a = t b Magnitude of a vector
i.e one is a multiple of the other The magnitude of the vector a = 3i + 2j –7k
Addition of Vectors a  = 32 + 2 2 + (−7 ) 2
If two vectors a and b represent the two adjacent
sides OA and OB respectively of a parallelogram Sum, subtraction and scalar multiplication
OACB, then the diagonal OC which passes through in vectors
their point of intersection will be equal to their vector sum 2013/29 f/m
OC = a + b If s = 3i – j and t = 2i + 3j, find (t + 3s) (t – 3s)
And the other diagonal BA is equal to A – 77 B – 71 C – 53 D – 41
BA = a – b Solution
A (t + 3s)(t – 3s) = [2i + 3j + 3(3i – j)] [2i + 3j – 3(3i – j)]
b = [2i + 3j + 9i – 3j] [2i + 3j – 9i + 3j]
= (11i + 0j) (– 7i + 6j)
a
C = 11×(– 7) + 0×6
a+b
= – 77 + 0
= – 77 A.
O
a-b

1994/32 (Nov )
In the diagonal BA, the movement to A
b is -ve to B direction and so we have -b Find the resultant of the vectors
a = 3i
B
b = –2i – j
c = i + 4j
Triangle rule A 4i + 4j B. 2i +3j C. 6i + 5j
First, we need to follow the vector arrows as given in D. – 4i – 4j E – 2i – 3j
the diagram or a triangle with arrows showing the Solution
Resultant of the vectors = a + b + c
vectors AB , BC and AC
= 3i – 2i – j + i + 4j
B = 2i + 3j (B)
2000/36
 3  2 8
Given that p   + q   =   , where p and q are
1  4 6
A C
constants, find the value of ( p + q )
Here AB follows BC but AC contradicts them A –4 B 1 C 3 D 4
AB + BC = AC deductively Solution
One column matrix ( same as vector ) problem
BC = AC – AB and AB = AC – BC 3p + 2q = 8 ------ ( 1)
Other definition of terms p + 4q = 6 ------ ( 2 )
(1)  2 and subtract
Origin vectors are: 6p + 4q = 16
2×2 row vector( 0, 0 ) or 3×3 row vector( 0, 0, 0 ) –( p + 4q =6 )
0 0
or 2×2 column vector   or 3×3 column vector  0  5p = 10
0  
  0 p = 10 i.e 2
  5
442
Substitute p = 2 into (2) Solution
p + 4q = 6 will become
4q = 6 – 2 Two vectors are parallel then, A × B = 0
4q = 4 i j
q = 4 i.e 1 and A × B = 3 −2
4
Thus p + q = 2 + 1 4 p
= 3 (C)  3p + 8 = 0
1999/33 (Nov)  3p = -8 Thus p = – 8/3 ( B)
Given that a and b are scalars, solve the equation 2003 / 7 (b)
2 4  −2   4   − 12 
a  + b  =   Given that p =   and q =   , find a vector U such
1  −2  3 − 3   5 
A. a = - 1 ; b = - 1 B. a = - 1; b = 1 that U is parallel to p + q and is half the size of p + q
C. a = 1 ; b = - 1 7
D a = 54 ; b = 10 Solution
 4   −12  − 8 
Solution p + q =   +   =  
2a + 4b = – 2 --------(1) −3   5   2
a – 2b = 3---------(2)
(2)  2 By the theorem:
2a – 4b = 6 Two non-zero vectors a and b are parallel if and only if  a
− (2a + 4b = − 2) scalar t, such that a = t b
− 8b = 8 Thus, a vector parallel to p + q and half the size of p + q is simply

b=-1 1 1 − 8  − 4 
Substitute b value into (1) (p+q)=   =  
2 2  2  1
2a + 4(-1) = -2
2a – 4 = - 2 2005 / 16(a)
2a = - 2 + 4 The position vectors of points R, S and T in the X – Y plane
2a = 2 are r = 5i + 3j, s = 8i – j and t = 11i – 5j
a = 1 Thus a = 1 and b = -1( C) respectively.
(i) Show that, R, S and T are collinear.
2014/5 NABTEB(Dec) Exercise 27.1 (ii) Find scalars k1 and k2 such that
Let P = 2i + 3j, q = i – 2j and r = 4i + 3j, 15i + 9j = k1 r + k2 t
Find (i) 2p + q (ii) p + q + r
(iii) 3q – 2r (iv) 4p – 3q – 2 Solution
(i) R S =s–r
2014/50 NABTEB Exercise 27.2
= ( 8i – j ) – ( 5i + 3j )
 2  − 1
If a =   and b =   , evaluate 2a – 3b = 3i – 4j
0  3
8
A  
 4
B   C 
 7 
 D 
 2 
 Similarly S T = t – s
6 3    − 9  − 3 = (11i – 5j ) – ( 8i – j )
= 3i – 4j
2014/49 NABTEB Exercise 27.3  R S and S T are parallel which is possible only when R,
 5  − 1
Find the sum of the vectors   and   , S and T lie on one line; hence result
 3  3
 4  4  2  3 ( ii ) 15i + 9j = k1( 5i + 3j ) + k2( 11i – 5j )
A   B   C   D   = 5k1 i + 3k1 j + 11k2 i – 5k2 j
5 6   5  5 15i + 9j = 5k1 i + 11k2 i + 3k1 j – 5k2 j
Equating terms
15 = 5k 1 + 11k 2 ------( a ) and
9 = 3k 1 – 5k 2 -----------( b )
a × 3 and b × 5 and subtract
Problems on parallel vectors 45 = 15k1 + 33k 2
2000/17 ( Nov) – ( 45 = 15k1 – 25k 2 )
Given that the vectors 3i – 2j and 4i + Pj are parallel, 0 = 58 k 2  k2 =0
find the constant P
A –6 B – 8/3 C – 3/ 8 D 1/ 6 Substituting k 2 value into ( a ) 15 = 5k1 + 0
15 = k  k = 3
1 1
5

443
2002/49 Neco Problems on angle between two vectors
What is the unit vector which is parallel to the 2015/16 f/m
vector 7i – 4j ? If x = i – 3j and y = 6i + j.
Solution Calculate the angle between x and y
Two vector a and b are parallel A 600 B 750 C 810 D 850
If a = sb where S is a scalar Solution
Thus unit vector here (i − 3 j ) . (6i + j )
Cos  =
7i − 4 j 7i − 4 j ( 1 + (−3) 2 ) ( 62 + 12 )
2
= =
7 2 + (−4) 2 65 1  6 + (−3)  1
Cos  =
( 1 + 9 ) ( 36 + 1)
Problems on projection of vectors
6 − 3
2004/ 45 Cos  =
( 10 ) ( 37 )
The projection of the vector a = 5 i – 2 j + k on the 3
vector b = 2 i + 3 j . Cos  =
370
A 4 13 B 5 30 C 5 13 D 4 13 E 30 Cos  = 0.15597
15 13 13 15
 = cos– 1 0.15597
Solution
= 81.0260  810 C.
P = ˆ a
b.
b
2014/24 f/m
where b is 2 + 3
2 2
But b b = Find the angle between (5i + 3j) and (3i – 5j)
A 1800 B 900 C 450 D 00
1 Solution
= (2i + 3 j ) (5i + 3 j ) . (3i − 5 j )
13 Cos  =
( 52 + 32 ) ( 32 + (−5) 2
Thus P  ˆ a=
b. 5  3 + 3  (−5)
Cos  =
1
(2i + 3 j ). (5i − 2 j + k ) ( 34 ) ( 34 )
13 Cos  =
15 − 15
=
0
1 34  34 1156
= (10 − 6) Cos  = 0
13
 = cos– 1 0
1 4 13  = 900
=  4 i.e (D) B.
13 13
2014/14a f/m
2000/24 Exercise 27.4 Given that x = 3i – j, y = 2i + kj and the cosine of the angle
Find the projection of the vector a on the vector b, if 5
between x and y is , find the values of the constant k.
a = 2 i + 3 j and b = – i + 4 j 5
Solution
10
A 17 B. 10 17 C 10 17 D. E. 5 17 → →
10 17 17 x. y
Cos  =
| xy |
Problems on equal vectors 5 (3i − j ) . (2i + kj )
=
5 ( 3 + (−1) 2 ) ( 22 + k 2 )
2
2002/50 Neco
What is the value for which vectors x i + 8j and 5i + 40 j 5
=
6 − k
x 5 ( 10 ) ( 4 + k 2 )
are equal ?
A +5 B +4 C –5 D –9 E –10 5 ( 10 ) ( 4 + k 2 ) = 5 (6 − k )
Solution Square both sides
Two vectors are said to be equal if their 5(10)(4 + k 2) = 25(6 – k) 2
i – components are equal 2(4 + k 2) = (6 – k) 2
8 + 2k2 = 36 – 12k + k 2
In this case
k + 12k – 28 = 0
2
i : i
Factorizing k 2 – 2k + 14k – 28 = 0
x : 5 thus x = 5 ( A)
k(k – 2) + 14(k – 2) = 0
Confirm it at the j-component
(k – 2) (k + 14) = 0
40
= 8 thus x = 5 k – 2 = 0 or k + 14 = 0
x k = 2 or – 14
444
2000/16 (Nov ) 1996/39
Given that p = 3, q = 4 and p·q = – 6, If PQ = 4 i + 3 j and QR = 5 i + 12 j,
find the angle between p and q. Find the cosine of the angle between the two vectors.
A. 300 B 450 C 1200 D 1350 A. 16 B. 13 7 4
E. 56
65 13 C. 65 D. 13 65
Solution
Solution
p. q
Cos = PQ . QR
p q Cosine of angle between the two vectors =
Substituting for the given values PQ QR

Cos =
−6 (4i + 3 j ). (5i + 12 j )
3 4
Cos = – 0 .5
=
(4 2
+ 32 )( 5 + 12 )
2 2

 = – Cos– 1 0.5 20+ 36


Though it is 600 but the minus sign shows that Cosine
here is in 2nd quadrant with formula 180 – 
=
( )(
25 169 )
 = 180 – 60
56
= i.e 56
65 (E)
= 1200 (C) 5 13
2004/ 7 Neco
2004/9 Given that u = - i + 2 j – 2 k and v = 3i + 4 j –12 k
Find the angle between i + 5j and 5i – j
A 300 B 450 C 600 D 900 Find (i) the dot product of u and v
Solution
(ii) the angle between u and v
Cos  =
(i + 5 j ). (5i − j )
( 12 + 5 2 ) 5 2 + (− 1) 
Solution
2
(i). u . v = − 1 3 + 2  4 + (−2  −12)
 
= − 3 + 8 + 24 
5−5 0
Cos  =
( 26 )( 26 ) i.e.
26
=0
(ii)
= 29
u • v = u v cos
Cos  = 0
 = Cos –1 0 u .v
= cos
= 900 (D) u v
  ( )
1 1

2004/40 Neco u = (−12 ) + 2 2 + (−2) 2 2


; v = 32 + 4 2 + (−12) 2 2

If m = 4 i + 3 j and n = i + 2 j what is the


= (169 ) 2
1 1
= (9 )2
cosine of angle between m and n ?
=3 = 13
A 2 5 B 5 2 C 2 D 12 5 E 2 5 29 29
2 25 5 Thus, cos = i.e
Solution 3 13 39
m . n cos = 0. 7436
Cosine of angle between m and n =  = cos −1 0 .7436
m n
= 41.960
m . n = 4 1+ 3 2
1996/38 (Nov) Exercise 27.5
= (4 + 6)
If p = 2i + j and q = 5i +3j find the cosine
= 10
of the acute angle between p and q
( ) ( )
1 1

m = 4 2 + 32 2
and n = 12 + 2 2 2
A. 13 170 B. 7 170 C. 7 D. 13 E. 7
170 170 13 170 170
= 25 = 5
2003/16 Neco Exercise 27.6
=5
If a = 2 i + 3 j and b = 2i + 4 j, find the cosine of angle
Thus, Cosine between m and n = 10
5 5 between a and b
= 10 5 2 4 8 8 4
25 A B C D E
33 65 65 20 13
= 2 5 (E)
5 2002/27 (Nov) Exercise 27.7
Evaluate the angle between a = – 4i + 2j and b = i – 3j
A 450 B 600 C 1350 D 3150

445
Problems on vector magnitude 2002/ 31 Exercise 27.8
If p = ( 2i + 3j ) and q = ( i – 7j ) then,
2004/21 Neco (Dec) f/m
A q + p = 63 B p+q = 5
Calculate the magnitude of the vector a = 4i – 3j +
12k C p−q = 101 D p − 2q = 17
A 169 B 26 C 19 D 13 E 8 Problems on dot product of vectors
Solution 1995/31 (Nov )
Magnitude of vector Evaluate; (8i – 15j ) • (8i – 15j)
a = 4i – 3j + 12k A. 49 B. 161 C. 189 D. 225 E. 289
Solution
a = 4 2 + (−3) 2 + 12 2 Dot product = (8i – 15j ) • (8i – 15j)
= 16 + 9 + 144 = 169 i.e 13(D) = 8  8 + ( −15)  ( −15)
= 64 + 225
2013/37 f/m = 289 (E)
 − 2  − 1 2003/4 Neco
If V =   and U =   , find |u + v| If r = i + 2j and q = 2i – 3j, find r • q
 4  5
A -4 B -3 C 21/2 D 2 E 4
A 3 10 B 82 C 46 D 5
Solution
Solution
r • q = (i + 2j) • (2i – 3j)
 − 1  − 2
U + V =   +   =2–6
 5  4 = –4 A
 − 3 2002/48 Neco
=   i.e – 3i + 9j
 9 The scalar product of vectors –2i – 3j and 4i + 5j is
A 23 B 7 C –7 D – 10 E – 23
|u + v| = (−3) 2 + 92
Solution
= 9 + 81 Scalar product of two vectors is same as dot product
= 90 = 9 10 (–2i – 3j ) • ( 4i + 5j ) = (–2 × 4) + (–3 × 5 )
= –8 – 15
= 3 10 A.
= – 23 (E)
2015/23 f/m 2005/15
Given that r = 2i – j, s = 3i + 5j and t = 6i – 2j, find the Find the magnitude and direction of the resultant of
magnitude of (2r + s – t) a = 2i + 3j and b = 2i + j
A 15 B4 C 24 D 26 A ( 4 2 , 0300 ) B (4 2 , 0450 )
Solution
2r + s – t = 2(2i – j) + 3i +5j – (6i – 2j) C ( 2 2 , 0450) D ( 2 2 , 0300 )
= 4i – 2j + 3i + 5j – 6i + 2j Solution
= i + 5j Resultant of ab =a+b
= ( 2i + 3j ) + ( 2i + j )
|2r + 5 – t | = 12 + 52 = 4i + 4j
= 1+ 25 = 26 D. magnitude of ab = 42 + 42
1995/35 (Nov ) = 16 + 16 = 32 = 4 2
 6
Given that a =   and b =  
 1
Direction of resultant of ab
3  −2  a.b
Cos  =
Evaluate a + b − a − b a b
B 14 C. 2 5 D. 5 2 E. 10 2 (2i + 3 j ). (2i + j )
(2 )( 2 )
A.0 =
Solution + 32
2 2
+ 12
a+b and a–b 4+3
6+  1  = 7
   
 3   −2 
 
1
6–  1 = 5
     
 3   −2   5 
=
( 13 )( 5 )
7
Cos  =
Next, │a + b│= 7 2 + 12 and │a – b│ = 5 +5
2 2
65
= 50 and = 50  = Cos -1 7
65
Thus, a + b − a − b = 50 – 50 = 29.740  300
= 0 (A) Thus magnitude and direction is ( 4 2 , 0300 ) (A)
446
1996/38 Exercise 27.9 8 – 3k = 0
8 = 3k
If PQ = 2 i − 3 j and QR = 3 i − 5 j , find PQ • QR 8
3 = k (B)
A 21 B. 15 C. 9 D. 1 E. –9
1994/7 June
OP, OQ and OR are the vectors 3i + 4j, 3ki + 40j and 8i – 2j
Perpendicular condition of vectors respectively, where k is a scalar.If the resultant of OP and
2003 / 15 OR is perpendicular to OQ, find K .
Given that p = ( 6i – 9j ) and q = ai + 2j are Solution
perpendicular, calculate the value of the constant a. First, we find resultant of OP and OR
A –3 B −4 C 3 D 3 = OP + OR
3 4 = (3i + 4j ) + (8i – 2j)
Solution = (11i + 2j)
Two vectors are perpendicular to each other if their Next, for two vector to be perpendicular;
dot product equals zero. their dot product = 0
p•q=0 (OP +OR) • OQ = 0
(6i – 9j ) • ( ai + 2j ) = 0 (11i + 2j ) • (3ki + 40j) = 0
6 × a + (– 9) × 2 = 0 33k + 80 = 0
6a – 18 = 0 33k = –80
6a = 18 k = – 80
a = 3 (D) 33
1996/37 (Nov)
2004/42 Neco For what value of p are the vectors pi – 3j and 3i – 2j
If a = 3i + 4 j and b =  j + 2 k are perpendicular, perpendicular?
A. -3 B. -2 C. 23 D. 2 E. 3
what is the value of  ?
A 4 B 0 C – 3/2 D – 8/3 E–4 Solution
Since the two vectors are perpendicular; then
Solution (Pi – 3j) • (3i – 2j) = 0
3p + 6 = 0
a . b = 0 Condition for perpendicularity 3p = – 6
p = – 63 i.e –2 (B)
3 0 + 4   + 0  2 = 0
4 = 0 Problems on Unit vectors
 = 0 (B ) 2012/24 f/m
Find the unit vector in the direction of the vector – 12i + 5j
2004/32 Neco (Dec) −12 5 −1 5 −12 5
Find K such that a = 4i – 10j and b = Ki + 2j are A i − j B i − j C i + j
13 13 13 13 13 13
perpendicular vectors
−5 12
Solution D i − j
13 13
a . b = 0 Condition for perpendicularity
Solution
(4i – 10j ) • ( k i + 2j ) = 0
−12i + 5 j
4k + (–20 ) = 0 Unit vector =
4k – 20 = 0 (−12) 2 + 52
4k = 20 −12i + 5 j
=
4k 20 144 + 25
=
4 4 −12i + 5 j
=
k = 5 169
−12i + 5 j −12 5
1994/34 = = i + j C.
13 13 13
If p = 4i + kj and q = 2i – 3j are perpendicular,
find the value of k, where k is a scalar quantity.
2013/34 f/m
A. 3 B. 83 C. 23 D. −23 E. – 4
P and Q are point (3, 1) and (7, 4) respectively
Solution Find the unit vector along PQ.
Two vectors p and q are perpendicular, if  4  0 .6   0. 8   − 0.8 
p •q = 0 A   B   C   D  
3
   0. 8   0 .6   − 0.6 
(4i + kj ) • (2i – 3j) = 0
Solution
Vector PQ = q – p
4 2 + (k  − 3) = 0
447
= (7, 4) – (3, 1) (4 i + j ) (4i− j )
= (4, 3) i.e (4i + 3j) C.  D. 
4i + 3 j 17 17
Unit vector PQ = 4i + 3 j =
4 2 + 32 25 Rule is AB = b −a
1
= (4i + 3j) PQ = (2 i – 5j – (i – j )
5
1  4 = i – 4j
In column form =  
5  3
 0. 8  Next, Unit vector perpendicular to PQ = i – 4j
=   C.
 0 .6  12 + ( – 4 )2
1993/31 ( Nov)
=
(i − 4 j)
Find the unit vector in the direction of the resultant of 17
p = 5i + 3j and q = 7i – 8j
2000/47 Neco Exercise 27.10
1 1 12 5 2 5
A. i– j B. i− j C. i + j The unit vector in the direction of (3i + 4j) is
12 5 13 13 13 13
A 1 (3i + 4j) B 1 (3i + 4j) C 1 (3i + 4j)
5 12 1 1 6 5 4
D. i+ j E. i+ j
13 13 12 5 1
D (3i + 4j) 1
E (3i + 4j)
Solution 3 2
Resultant of p and q = p + q 1994/34 (Nov ) Exercise 27.11
= (5i + 3j) + (7i – 8j) Find the unit vector acting in the direction of 2i – j
= (12i – 5j) 2i − j 2i − j 2i − j
p+q A i+j B i–j C D E
Next, unit vector is = 5 5 3
p+q 1999/26 (Nov) Exercise 27.12
12i − 5 j Find unit vector in the direction of the
= Resultant of P = ( 4i – j ) and q = (2i – 7j )
12 2 + (−5) 2 3 4 1 1 1 7
A. i − j B. i − j C. i − j
12i − 5 j 5 5 6 8 5 10
=
169 2 1
D. i − j
1
= (12i − 5 j ) ie 12 i − 5 j ( B) 5 10
13 13 13
1996/32
Problems on Joint cases of vectors properties
Given OQ = 4 i − 3 j , the unit vector in the direction

of OQ is 1996/14
The position vectors of the points P and Q are
1
A 20 i −15 j B 4i − 3 j C (4i − 3 j ) p = – i + 3j and q = 2i + 2j respectively. Find
5
(a) 3p – 4q (b) p + q (c) the angle between p and q
D. –
1
5
(
4i + 3 j ) E.
1
7
(
4i − 3 j ) (d) the scalars h and k such that hp + kq = 5i + 7j
Solution
Solution
(a) 3p – 4q = 3(– i + 3j) – 4(2i + 2j)

Unit vector in OQ the direction of OQ =
OQ = – 3i + 9j – 8i – 8j
OQ = – 11i + j
(b) p + q
4i −3 j
= but p + q = – i + 3j + 2i + 2j
4 2 + (−3) 2 = i + 5j
= (4 i − 3 j ) (C)
1 Thus, P + q = 1 + 5
2 2

5
2000/37 = 26
The position vectors of points P and Q are i – j and (c) Angle between p and q
2i – 5j respectively. Find a unit vector perpendicular to p.q
cosine of angle b/w p & q =
PQ p q

(− i + 4 j)
B 
(i + 4 j) (− i + 3 j ). (2i + 2 j )
( (−1) )( 2 )
A. =
17 17 2
+ 32 2
+ 22

448
−2 + 6 A B
=
( 10 ) ( 8 ) (C) Cosine of angle b/w A and B =
A B
4 1
= i.e A • B = 4 2 + (−2 3)
4 5 5
Cosine of angle b/w p and q = 0.4472 = (8 – 6)
=2
Angle b/w p and q = cos −1 0.4472
= 63. 43 0 A = 4 2 + (−2) 2 and B = 2 2 + 32
(d) hp + kq = 5i + 7j 16+ 4 4+ 9
= =
h(–i + 3j) + k(2i + 2j) = 5i + 7j
–hi + 3hj + 2ki + 2kj = 5i + 7j = 20 13
=
Collect like terms together at LHS 2 2
–hi + 2ki + 3hj + 2kj = 5i + 7j Cosine of angle = =
(–h + 2k)i + (3h + 2k )j = 5i + 7j 20 13 2 5  13
Equating terms 1
–h + 2k = 5 ------(1) =
– (3h + 2k = 7) -----(2) 65
– 4h = –2
h = 1/2
=
65
65
i.e
1
65
65 ( )
Also from (2) 2k = 7 – 3( 12 ) 1994/37 (Nov )
If p and q are two parallel vectors in opposite directions,
2k = 7 – ( 32 ) Then which of the following statements is/are true?
11
2k = 2 I p•q= p q
11
k= 4 II p • q = – p q
1992/14 (Nov)
III p • q = 0
The position vector of two points A and B are
a = 4i – 2j and b = 2i + 3j respectively. Find: A II and III only B. I only C. III only
(a) 3 A − 2 B D. I and III only E. II only
(b) Scalars  and  such that
Solution
8i + 8j = A + B; By condition of parallel and opposite vector.
(c) the cosine of acute angle between A and B, leaving Let the vectors be
your answer in surd form p = 3i + 2i and q = – 3i – 2j
Solution Testing
(a) 3 A − 2 B I p•q= p q
First we find 3A – 2B = 3(4i – 2j ) – 2(2i + 3j) LHS: (3i +2j) • (–3i – 2j) = – 9 – 4
= 12i – 6j – 4i – 6j = – 13
= 12i – 4i – 6j – 6j
= 8i – 12j
RHS ( 3 + 2 )(. 3
2 2 2
) ( )( )
+ 2 2 = 13 13

Next, 3 A − 2 B = 8 + − 12
2
( 2
) = 13 LHS ≠ RHS
II p•q=– p q
= 64 + 144 From the test at I we deduce that II is true
= 208 or 14.42
III p • q = 0
= 4 13 From the test at I, we deduce that III is not true
The answer is II only (E)
(b) 8i + 8j = A + B;
1994/7 (Nov)
= (4i – 2j ) + (2i + 3j)
Given a = 2i + 5j and b = 4i – 3j, find
= 4 i – 2j + 2i + 3j
(a) The unit vector in the direction of b.
= 4 i + 2i – 2j + 3j (b) The projection of a on b
8i + 8j = (4 +2)i + (–2 + 3)j (c) │3a – 2b│
Equating terms Solution
4 + 2 = 8 -------- (1) b
–2 + 3 = 8 ---------(2) (a) Unit vector in the direction of b =
Multiply (2) by 2 and add b
– 4 + 6 = 16 4i − 3 j
=
4 + 2 = 8
4 2 + 32
8 = 24 Thus  = 3 and  = 1/2
449
4i − 3 j 1 5
i.e (4 j − 3 j )
(2 )( 1 )
= =
25 5 2
+3 2 2
+ 12

(b) Projection a on b = b • a 5 5 5
= = =
b ( 13 )( 2 ) 26 5.099
= • a
b Cos  = 0.9806
 = Cos– 1 0.9806 = 11.300
= 1 ( 4i – 3j ) (2i + 5j) ( iii ) If x is perpendicular to z then x . z = 0
5
1
= (8 –15) i.e – 75 x . z  ( 4i – 3j ) · ( i + j ) = 4  1 + (–3)  1
5
= 4–3
(c) First is 3a – 2b = 3(2i + 5j ) –2(4i –3j ) = 1
= (6i + 15j) + (–8i + 6j ) One is not zero so x is NOT perpendicular to z.
= –2i + 21j
2000/15
Thus │3a – 2b│ = (−2) 2 + 212 Given that p = 2i + 3j; q = i + j ; r = i – 2j ;
Find: (i) 2 p − 3q + r ;
= 445
= 21. 095 (ii) The Unit vector in the opposite direction of 2p – 3q + r;
2000/22 (Nov) (iii) The angle between p and r, correct to the nearest degree.
Given that p = 9i + 12j and q = – 6 i – 8j Solution
( i ) First we do the scalar multiplications
Evaluate p − q – { p - q } 2p – 3q + r = 2(2i + 3j) – 3(i + j) + (i – 2j)
A 10 B 15 C 20 D 30 = 4i + 6j – 3i – 3j + i – 2j
Solution Collect like terms together
First vector Subtraction for p and q = 4i – 3i + i + 6j – 3j – 2j
p – q = [ 9 – (–6 ) ]i + [ 12 – (–8 ) ] j = 2i + j
= (9 + 6)i + (12 + 8 )j
Thus, 2 p − 3q + r = 2 +1
2 2
p – q = 15i + 20j
Thus p−q = 15 2 + 20 2 = 5
( ii ) 1st we find unit vector of 2p – 3q + r
= 225 + 400 2 p − 3q + r
=
= 625 i.e 25 2 p − 3q + r
2i + j
=
p = 9 + 12
2 2
q = (−6) + (−8)
2 2
5
= 81+144 = 36 + 64 Thus, Unit vector in opposite direction of 2p – 3q + r
− (2i + j )
= 225 = 100 =
5
= 15 = 10
− 2i − j
Therefore p−q – { p – q } =
5
= 25 – { 15 – 10 } (iii) angle between p and r is given by
= 25 – 5
= 20 (C)
p. r
Cos  =
2000/15 a (Nov) p r
If x = 4i – 3j ; (2i + 3 j ). (i − 2 j )
y = 2i + 3j
z =i +j
=
(2 2
+ 32 ) ( 1 + (−2) )
2 2

( i ) find z · y ; ( ii ) find the angle between y and z 2  1 + 3  (−2)


=
( iii ) show that x is not perpendicular to z
Solution ( 13 ) ( 5 )
(i) z· y = ( i + j ) · ( 2i + 3j ) −4
=1×2 + 1×3 Cos  =
=2+3 65
=5 Cos  = – 0.4961
( ii ) Angle between y and z  = 60. 260  600
y.z Cosine is negative in 2nd & 3rd quadrant
Cos = i.e.  is either 180 – 60 or 180 + 60
y z = 1200 or 2400

450
2002/7 Unit vector parallel to PQ
The position vectors of points P and Q are; 9i + j 9i + j
p = ( i + 3j ) = =
q = (9i + 8j ) 9 2 + 12 82
respectively. Find:
(iii) Length of the projection of p on q = p Cos 
(a) PQ
P. Q
(b) correct to the nearest degree, the angle First, we find Cos  =
between p and q. p q
(c) The length of the projection of q on P (3i + 4 j ). (12i + 5 j )
( 3 + 4 )( 12 )
Solution =
2 2 2
+ 52
(a) PQ =q – p 36 + 20
= (9i + 8j ) – (i + 3j ) =
(5) (13)
= 8i + 5j
P. Q 56
Cos  =
(b) Angle between p and q  Cos  = 65
p q
Thus length of projection of p on q = p Cos 
1 9 + 3 8
(1 )( 9 ) ( 3 + 4 )  5665 
Cos  =
2
+ 32 2
+ 82 = 2 2

9 + 24
25 × 56
( 10 ) ( 145 )
= =
65
=5× 56
33
= 65
1450 = 4.31 units
Cos  = 0.8666
 = cos–1 0.8666 1999/32 (Nov) Exercise 27.13
= 29.930 Given that a, b and d are vector quantities and  the
 300 to the nearest degree angle between b and d, which of the following statements is
(c)Length of projection of q on p = q Cos  /are true?
I. a . b = b . a
= (9 2
+ 82 ) × 0.8666 II. (a . b) + d = a . b + a .d
= 145 × 0.8666 III. b . d = b d cos
= 10.43 units A. I and II B. II and III C. I and III D. I only
1997/7
The position vectors of points p and q relative to a fixed 1997/38 Exercise 27.14
point O are Which of the following is /are always true of vector
p = 3i + 4j and q = 12i + 5j respectively. P, Q and R ?
Find: ( i ) 2 p − 3q I P•Q = Q•P
II P• (Q + R ) = P•Q + P•R
( ii ) the unit vector parallel to P Q ; III P•Q = O if p is perpendicular to Q
(iii) the length of the projection of p on q. A I only B II only C I and II only
Solution
D II and III only E. 1, II and III
(i) 2 p − 3q
First find 2p – 3q 2004/10 Exercise 27.15
= 2( 3i + 4j ) – 3( 12i + 5j ) If p and q are two parallel vectors in the same direction,
= 6i + 8j – 36i – 15j which of the following statement is/are true ?
= –30i – 7j I p.q = p q
Thus 2 p −3p = (−30) 2
+ (−7) 2
II p.q = – p q III p.q =0
= 949
A I only B I and II only C II and III only D III only

( ii ) P Q = q – p
= (12i + 5j) – (3i + 4j)
= 12i + 5j – 3i – 4j
= 9i + j
451
Problems on position vectors OP = P – O
2012/9 f/m 8 − 0
 8
=  i.e  
QRS is a triangle such that QR = (3i + 2j) and 4 − 0 4
SR = (– 5i + j). Find SQ . Similarly 3OQ = 3Q
A 8i + j B 2i – j C – 2i – 3j D – 8i – j  −6 
= 
Solution 9
With the aid of the vectors arrow, we sketch as: and 2OR = 2R
R
=
 −6 

 −10 
Hence OP – 3OQ + 2OQ =   -   + 
 8   −6   −6 

4  9   −10 
Q S
We absorb the minus sign as
By triangle rule
8  6 
=  +  + 
 −6 
SR = SQ + QR 
 4   −9   −10 
5i + j = SQ + 3i + 2j  8 
=  A
– 5i + j – (3i + 2j) = SQ  −15 
2003 / 27
–5i + j – 3i – 2j = SQ  3 1
Given that O P =   and P Q =   ,
–8i – j = SQ D.  5  4
2014/23 f/m where O is the origin, find O Q
4  2   − 9  − 2 1  4
Given that AB =   and AC =   , find BC A   B   C   D  
 3  − 3 9
 − 4  −1  2  
A. 4 2 B. 6 2 C. 2 10 D. 4 10 Solution
Solution We have a simple rule for vectors AB = b – a
We draw the diagram based on the arrow flow of the vectors
given 0
A and origin vector 0 in this case is  
0  
 i =−3 
Thus O P is  0 − i  =  3     i.e P =  − 3
 
 0 − j   5
B C
     j = − 5  − 5

AB + BC = AC  o −i  1 i=−4  − 4
Also P Q .is   =      i.e Q =  
4  2 
  + BC =   o − j  4  j = − 9  − 9
 3  − 3
 2  4 0 + 4 i  4
BC =   –   Hence, O Q is   =   i.e.   (D)
 − 3   3  0 + 9   j 9
 −2  1997/39
BC =   i.e –2i – 9j
 − 6 The position vector of P and Q are p = i – 3j
and q = 4i + 5j respectively. Find the unit vector
Thus BC = (−2) 2 + (−6) 2
parallel to PQ
= 4 + 36 1
A. (3i + 8 j ) B. − 1 (3i − 8 j ) C. 1 (i − 3 j )
= 40 = 410 = 2 10 73 73 73

D. 1 1
1994/36 (4i + 5 j ) E. (5i + 8 j )
The position vectors of points P, Q and R relative to an 73 89
origin O are Solution
 8  ,  −2  and  −3  respectively. Find OP-3OQ + 2OR First we find PQ = q – p
     
4  3   −5  = (4i + 5j ) – (i – 3j )
 8   20   20  8  −6  = 4i – i + 5j + 3j
A.   B.   C.   D.   E.   = 3i + 8j
 −15   −16  3  −3   −15 
Solution Next, we find unit vector parallel to PQ
452
Recall that two non – zero vectors b and c are 2005 / 7 June
parallel if and only if  a scalars.
Such that b = Sc Given that O C = a – b and O D = 2a + 3b,

Thus what we need to do is to get the unit Vector of PQ where a = 2i + 3j and b = 3i – 2j , find C D
PQ Solution
= Let us sketch the diagram following the directions of vectors
PQ arrow.
C
3i + 8 j
=
32 + 8 2

b
a-
1
= ( 3i + 8j ) (A)
73 O D
1997/32 2a + 3b
Given that AB = 5i + 3j and AC = 2i + 5j, and find CB By triangle law of vectors
A. 7i + 8j B. – 3i + 2j C. 3i – 2j OC + CD = OD
D. 3i + 8j E – 7i – 8j CD = OD – OC
= ( 2a + 3b ) – ( a – b )
Solution = 2a + 3b – a + b
Sketching the vector triangle diagram noting the = a + 4b
direction of the arrows = ( 2i + 3j ) + 4( 3i – 2j )
B
= 2i + 3j + 12i – 8j
= 14i – 5j
2005 / 35
A C In a triangle ABC, A B = 3i – 4j and A C = –2i – 7j.
From the flow of the arrows
Find B C .
AC + CB = AB
A - 5i + 11j B - 5i + 3j C 5i – 3j D 5i + 11j
CB = AB - AC Solution
= ( 5i + 3j ) – ( 2i + 5j ) Let us sketch the diagram following the directions of vectors
= 5i – 2i + 3j – 5j arrow.
= 3i – 2j (C) A
2004/7
The position vectors of points P and Q relative to the
origin are: p = – i – 4j , q = 2i – 4j
( a )Find a unit vector parallel to the resultant of p and q
( b ) Calculate 2 p − 3q , leaving your answer in surd form. B C

Solution By triangle law for vectors


( a ) Resultant of p and q = p + q AC = A B + B C
= (–i – 4j ) + 2i – 4j
= – i + 2i – 4j – 4j – 2i – 7j = 3i – 4j + B C
= i – 8j (– 2i – 7j ) – ( 3i – 4j ) = B C
i −8 j
Unit vector parallel to p q = – 5i – 3j = B C
65
= 1 ( i – 8j) 2005 / 16 ( b )
65 The position vectors of points P, Q and R in the x - y plane
(b) 2 p − 3q are O P = 3i + 4j , O Q = 5i + 6kj and O R = 7i + 2j
First 2p – 3q = 2(–i – 4j ) – 3( 2i – 4j ) respectively, where k is a scalar. If the resultant of O P and
= - 2i – 8j – 6i + 12j
= - 8i + 4j O R is perpendicular to O Q , find the value of k .
Thus, 2 p − 3q = (−8) + 4
2 2 Solution

= 64 +16 First, resultant of O P and O R = O P + O R


= (3i + 4j ) + ( 7i + 2j )
= 80 i.e 4 5 in surd form = 10i + 6j
Next, resultant of OP and OR perpendicular to O Q

453
Implies ( OP + OR) •( OQ) = 0 In ABC; by law of triangle
( 10i + 6j ) • ( 5i + 6kj ) = 0
10 × 5 + 6 × 6k = 0 AC = AB + BC
50 + 36k = 0  − 6  − 5
  =   + BC
36k = –50  − 9  −1
k = − 50 = − 25  − 6  − 5
36 18 BC =   –  

  9  −1
2003/16b  − 6 + 5  −1
The position vectors of points P, Q and R are =   =   Thus ABCD is a parallelogram
p = 7i + 2j, q = 5i + 5j and r = 3i + 8j respectively  − 9 + 1  − 8
( i ) show that P, Q and R lie on a straight line
( ii ) find scalars n and m such that 7i – 8j = np + mq
2006/15b
Solution     2 −1
In the pentagon ABCDE, AE =   , DE =   , DC
( i ) If P,Q and R lie on a straight line then 0
   2
the vector PQ is same as the vector QR  − 2  − 3
=   and CB =  
 − 5  2 
PQ = q – p Show that ABCD is a parallelogram
= (5i + 5j ) – ( 7i + 2j ) Solution
= –2i + 3j Let us sketch the diagram progressively as:
and QR = r – q Bearing in mind the parallelogram ABCD
Parallelogram pentagon guided
= ( 3i + 8j ) – (5i + 5j )
by the vectors arrows
= –2i + 3j QED A

( ii ) p = 7i + 2j and q = 5i + 5j
then np = 7ni + 2nj mq = 5mi + 5mj
E E

But np + mq = 7i – 8j
D

 7ni + 2nj + 5mi + 5mj = 7i – 8j Since ABCD is a parallelogram


Equate terms in i to i and also do same for j CB//AD and equal
7n + 5m = 7 B A B A
– (2n + 5m = – 8 )
5n = 15 E E
n=3
Substitute for n = 3 into 7n + 5m = 7 C D C D

7(3) + 5m = 7 By triangle rule of vectors in ADE


Contradiction arrow = sum two flowing/following arrows
m = –14/5
DE = DA + AE (triangle law)
Problems on vectors and plane shapes  − 1  2
2014/14b f/m   = DA +  
 2 0
In the quadrilateral ABCD
 − 1  2
 − 5  − 6  4  DA =   –  
AB =   , AC =   and BD =    2 0
 −1  − 9 − 7
 − 3
Show whether or not ABCD is a parallelogram =   same as CB
Solution  2
We interpret the diagram progressively as: Hence ABCD is a parallelogram
D C D C

1993/37 (Nov) f/m


ABCD is a rectangle with AB = 2a and AD = 2b
Find the magnitude and direction of the
A B A B vector AB – AD
In ADB the arrow at AD will point AD , BC follow suit A. 2a – 2b in the direction AC
We are to show that AD = BC B. 2 (a 2
)
+ b 2 in the direction DB
AD = AB + BD (triangle law) C. 2 (a 2
− b ) in the direction BD
2

 − 5
= 
 −1
 4 
 +  
− 7
D. 2 (a 2
+ b ) in the direction AC
2

 −1
=  
E. 2 (a − b) in the direction DB
 − 8 Solution
Alternatively
454
B C
Thus cos BCx is the angle b/w BC and  c
2a BC .  c = Bc  c cos
A D (3i + 3 j ). (6i + j )
( 3 + 3 ) ( 6 + 1 ) = cos
2b
2 2 2 2
AB – AD = 2a – 2b
Magnitude = 2a − 2b 18 + 3
= cos
= (2a )2 + (2b)2 25.81
21
= 4a 2 + 4b 2 = ( )
4 a2 + b2 = 2 a2 + b2 ( ) 25.81
= cos

cos = 0.8137
(b) Direction of AB – AD  0.814 to 3 s.f
B
B
AB normal flow of
arrow from A to B The resulting diagram 2005/17 (Nov)
By vector’s Law of triangle The position vectors of points P, Q , R and S relative to the
-AD negative flow of
arrow from D to A origin are
A D  − 2  ,  0  ,  3  and  1  respectively.
A D      
 1   −1  
 2  4
       
Thus 2 (a 2 + b 2 ) in the direction of DB (B)
(a) Show that PQRS is a rectangle
(b) Find the perimeter of the figure
2002/5 Neco f/m Solution
Three vertices A, B and C of a triangle have position (a) Let P, Q, R and S be sketched as:
vectors a, b and c respectively relative to the origin O, Q R
where a = 3i – 2j , b = 6i + j and c = 9i + 4j
( i ) Show that ABC is isosceles
(ii) determine the value of cos BĈX if X is the
P S
mid point of AC ( give your answer to 3 s.f)
If it is a rectangle then
Solution
Isosceles triangle has two equal sides . Q P = R S and Q R = P S
Lets sketch as :
A
QP = p – q and RS = s – r
 − 2 0 1  3
=   –   =   –  
 1   − 1 4
    2
 − 2  − 2
B C =   =  
 2   2 
The three lengths are :
QP = RS QED
AB = b – a
= (6i + j) – ( 3i – 2j ) To confirm, let us try Q R = PS
= 3i + 3j
QR =r–q and P S = s – p
AC = c – a
 3 0 1  − 2
= (9i + 4j ) – (3i– 2j) =   –   =   –  
= 6i + 6j  2  −1  4  1 
BC = c – b =  3  =  3 
   
= ( 9i + 4j) – (6i + j)  3  3
= 3i + 3j Verified as Q R = P S
Since AB = BC ABC is isosceles Thus PQRS is a rectangle

(ii ) x = mid point of AC ( b ) Perimeter of rectangle = 2( L + b )


= 1 (6i + 6j ) = 2 ( QR + RS )
2  3   − 2 
= 2   +  
= 3i + 3j
 3   2 
Next we find x = C – X 1
=2
= 9i + 4j – ( 3i + 3j)  
 5
= 9i + 4j – 3i – 3j
= 6i + j

455
Problems on vectors and coordinate geometry The position vectors and points P, Q and R are
2014/5 f/m 3i – 5j , –2i + 3j and 4i – 7j respectively.If T is the
The position vectors of point P, Q and R are 11i + j, mid – point of QR, Find P T
5i +
13
j and 2i + 6j respectively A - 2i + 3j B 3i – 5j C -2i + 5j D -4i + 6j
3 Solution
(a) Show that P, Q and R lie on a straight line x +x y +y
Co-ordinates of mid point  1 2 , 1 2 
 2 2 
(b) Find the ratio  PQ  :  QR 
Solution Mid point of QR co-ordinates =  − 2 + 4 , 3 − 7 
(a) We are to show that they  2 2 
are collinear i.e they are the =(1,–2)
same or multiple of each other i.e T = i – 2j in vectors representation
PQ = q – p PT =T–P
= 5i + 13 j – (11i + j) = ( i – 2j ) – ( 3i – 5j )
3 = i – 3i – 2j + 5j
13 = – 2i + 3j ( A )
= 5i + j – 11i – j
3
2005/8 ( Nov)
= – 6i + 10 j (b)  PQ  :  QR  The position vectors of the points A, B, C, and H
3
are 3i + 2j, 2i + 3j, 3i – 2j, and 8i + 3j respectively.
QR = r – q – 6i + 10 j :–3i + 5 j If D is the mid point of BC,
3 3
( a )find the position vector of D.
= 2i + 6j –  5i + 13 j 
2 2
 10  :  5
3  ( −6)2 +   ( −3) 2 +  
  3  3 ( b )show that A H = 2 O D where O is the origin
= 2i + 6j – 5i – 13 j 36 +
100 : 9+
25
Solution
3 9 9
D is mid point of BC i.e b = (2i + 3j) and c = (3i – 2j)
= – 3i + 5 j 424 : 106
x 2 + x1 y 2 + y1
3 9 9 x for D = y for D =
PR = r – p 4 : 1 2 2
(106) (106)
9 9 3+ 2 −2+3
2 1 106
= =
= 2i + 6j – (11i + j) 106 : 2 2
3 3
= 5 = 1
= 2i + 6j – 11i – j 2 : 1
2 2
3 3
5 1
= –9i + 5j 2 : 1 Thus, position vector D is i + j
rearranged with fraction 2 2

= 3  − 3i + 5 j  ; ( b ) Show that A H = 2 OD
 3 
→ First we find A H and O D
PQ is 2  − 3i +
5  result
j
 3  AH = h–a
1998/14a (Nov) = ( 8i + 3j ) – ( 3i + 2j )
The position vectors of points P, Q and R are: = 5i + j
 2  4  5 and O D = d – o
p =   q =   r =   , respectively
 3 −1  − 3  =
5 1
i + j – ( 0i + 0j )
Show that P, Q and R are concurrent 2 2
Solution 5 1
= i + j
We are to prove collinearity or parallel properties 2 2

i.e PQ = QR or PQ = t QR where t is a scalar Next, A H = 5 2 + 12 = 25 + 1 = 26

PQ = q – p  2 2 
 5 1 
Also 2 O D  = 2    +   
 4  2  2 2 
  = 
2
=   –   
−1  3
  − 4 
 25 1 
QR = r – q = 2  + 

 4 4
 5  4  1 
=   –   =  
= 2
26
=
2
26 i.e 26
− 3  −1  − 2  4 2

Thus, AH = 2 O D = 26 QED
Thus PQ = 2 QR QED
2003/27 (Nov) midpoint, co-ordinate vector
456
Chapter twenty eight Now let us debate it
Mapping Will any new baby have weight? Answer capital yes
Mapping is simply an association or a relation between Can any new baby have two or three different weights
two sets. at the same time? Answer capital NO
Let S be the set of cars { Benz 190, 505, Camry} Thus, for a relation to be a mapping; it must be that:
and T the set of car producer - Every element of the domain has an image in the co domain
{ Peugeot, Mercedes Benz, Toyota}, then there is a - The image of every element of the domain is unique
mapping which associate each car with its producer and You will agree that two or three or more new babies can have the
we write as f same weights
f : S → T or S → T
Also by mapping diagram or arrow graph, we can
Types of mapping
show it as
S T Injective (one-one)
A mapping f : S → T is said to be injective or one to one
Benz190
505
M ercedes when elements of S are uniquely paired to elements of T
Peugeot
Cam ry Toyota
or if no element of T corresponds to more than one element of S
S T P K
a 1
Benz 190 Mercedes b 2
505 Peugeot c 3
S = { Benz 190, 505, Camry} is called Domain Camry Toyota d 4
T ={ Peugeot, Mercedes Benz, Toyota} is called co domain 5

f : S → T is Injective f : P → K is Injective
Example MP2 f : P → K as shown below
P K Surjective (onto)
a 1 A mapping f : S → T is said to be surjective or onto if every
b 2
c 3 element of T is an image
d 4 S T P K
5 a 1
Domain Co domain Benz 190
505
Mercedes b 2
Peugeot c 3
Note that not all the elements of K are paired or related to P Camry Toyota d 4
Domain of the mapping f is {a, b, c, d} 5

Co domain of the mapping f is{1, 2, 3, 4, 5}


f : S → T is surjective f : P → K is NOT surjective
Image or range of the mapping is {1, 2, 3, 4}

Example MP2 is a mapping and is also a function: Bijective


A mapping that is both injective and surjective is bijective
reason the co domain is a set of real numbers. So you
S T
can say that:
Benz190
A function is a mapping whose co domain is numbers 505
M ercedes
Peugeot
The mapping f : H → L Cam ry Toyota
H L
a 1
b 2 f : S → T is bijective
c 3
d
composition of mapping
is acceptable since every element of the domain H is Let f : S → T and g : T → U be any function then we can
uniquely paired to an element of the co domain L compose then to get a mapping h : S → U, given by
Here the co domain is same as Image or range h(x) = g( f(x) ) or gof or gf for xS
we can represent it as:
The relations represented below are not mapping T
Fig I Fig II g
L f
T U V
a 1
b 2 a 1 S U
c 3 b 2 h
d 3
c 4 Identity mapping
A mapping S : T→ T, where the domain is the same as the
Reasons Reasons co domain i.e S(x) = x for all xT
All the elements in the domain one element a in the T T T R
are not paired to the co domain domain is not uniquely paired a a a 1
i.e paired to more than one element b b b 2
Example MP5 c c c 3
With each newborn baby associate its weight in grams
to the nearest gram. This is a function from the set of identity mapping not identity mapping
newborn babies to the set of natural numbers domain = co domain domain ≠ co domain

457
Constant mapping To determine position of a mapping
A mapping T : R→ S, which assigns all the elements of (Whether it is injective,surjective or both
the domain R to a single element in the co domain S and to locate its domain and co domain)
R S
x
Here we work based on the given problem; if it has any
y t formula, its application helps to determine whether it is
z r injective, surjective or both and to locate its domain and co
w domain depending on the question.

Inverse function 2009/26 Neco


For any function f to have an inverse f- –1 it must be Consider the mapping below
bijective i.e the function must be injective and A g B
surjective p 1
q
r 2
2002/12 Neco Exercise 28.0 s 3
t
Which of the following illustrates a function?
a 1 What are the elements in the co-domain of g?
Ab
c
2
3 A.{1, 2, 3} B.{g, 1, 2, 3} C.{p, q, g, l}
d 4 D.{p, q, r, s, t} E.{p, q, r, s}
Solution
a
b
2
co-domain of g = { 1, 2, 3} i.e set B
C E c
d
1
4
e 3
5
Counter illustration
A g B
2005/28 Neco Exercise 28.1 p 1
q
Suppose f : A → B be defined in the diagram below. r 2
A B s 3
1 s t
4
t
2 Here co-domain of g = { 1, 2, 3, 4} i.e set B
w
3 Here image of g = { 1, 2, 3} i.e subset of B
Then f is said to be 2003/22 Neco
A1–1 B. both 1 – 1 and onto C. inverse Find the domain of f : x → x2 + 6 ,
D. onto E. orthogonal if the range of f is {6, 7, 10, 15}
A. {0, 1, 2, 3} B. {-1, 0, 1, 2} C. {5, 6, 8, 12}
2010/3 Neco Exercise 28.2 D. {-1, 0, 1, 2} E. {-2, -1, 0, 1}
A mapping f : x → y described in the diagram Solution
below refers to what type of mapping? Here we are to reverse the process as
x y
range 6
x2 + 6 = 6, find x
x2 = 0
x=0
range 7
A Constant B Equality C Identity D One-one x2 + 6 = 7, find x
E Onto x2 = 1
Ordered pair mapping x =  1 i.e –1, 1
A typical ordered pair mapping is the Euclidean plane
which is made up of (x, y); e.g any point on the graph range 10
is ordered pair: (3,7) you know that x is 3 and y is 7 x2 + 6 = 10, find x
Mathematically X ×Y = {(x,y) : xX, yY} x2 = 4
All elements of x make up the domain x =  4 i.e –2, 2
All elements of y make up the co-domain
Example range 15
Identify with reason a mapping from the pairs below: x2 + 6 = 15
i{(a,4), (b,5), (c,6), (d,7)} ii.{(a,2), (b,2), (c,1), (d 0)} x2 = 9
Solution. Judge by yourself diagrammatically it is: x =  9 i.e –3, 3
Alphabet Natural no. Alphabet Natural no. The domain set = -3, -2, -1, 0, 1, 2, 3
a 4 a 2
b
Thus the domain here = {0, 1, 2, 3}(A) or {-3, -2, -1, 0}
b 5
c 6 c 1
d 7 ii d 0
i
458
1997/12 2010/1
Let the function f: w → R be defined by f : x2 – x – 2 , Find the domain of f(x) = x , where x  R,
where W = {- 1, 0, 2, 5, 11} and R the set of real 3− x
numbers. Find the range of f. the set of real numbers.
A. {- 2, 0, 18, 108} B. {- 2, 18, 108} A {x : x R, x ≠ 3} B {x : x R, x ≠ 1}
C. {-1, 0, 2, 5, 11} D. {-2, 0, 18} E. {0} C {x : x R, x ≠ 0} D {x : x R, x ≠ - 3}
Solution Solution
What we are to find here is the image of the function The domain here will be R – f(x) undefined
(though identified here as “range of f”) i.e R minus 3 – x = 0
f(x) = x2 – x – 2 i.e R minus x = 3
Taking the value domain w = {-1, 0, 2, 5, 11} one i.e R , x ≠ 3 ( A)
by one Because if x is equal to 3 then f(x) becomes undefined
f(-1) = (-1)2 – (- 1) – 2 f(5) = 52 – 5 – 2
=1+1–2 = 25 – 7 2010/16 Exercise 28.3
=0 = 18 Given that P = {x : x is a factor of 6} is the domain of
g(x) = x2 + 3x - 5, find the range of g(x).
f(0) = 02 – 0 – 2 f (11) = 112 – 11 – 2
A { -1, 5, 13 } B {5, 13, 49 } C {1, 2, 3, 6 }
=–2 = 121 – 13
D { -1, 5, 13, 49 }
= 108
f(2) = 22 – 2 – 2 2002/2 (Nov) Exercise 28.4
=4–4 Given that the domain of f (x) = x2 + 1 is
=0 {-2, -1, 0, 1, 2}, find the range of f.
Thus, the required set = {-2, 0, 18, 108} (A) A.{3, 0, -1} B.{5, 2, 1} C.{3, 1, 5} D.{2, 0, -1}
2004/32 2009/11 Neco Exercise 28.5
If the domain of f : x → x2 – 2 is {-2, -1, 0, 1}, Find the domain of f: x→ x2 +1 if the range is {1, 2, 5, 10}
find its range. A. {0, 1, 2, 3} B. {-3, -2, -1, 0} C. {-2, -1, 0, 1, 2}
A{2, 1, 0, -1} B{2, -1, -2} C{0, -1, -2} D{2,1, -2, -1} D. {-3, -2, -1, 0, 1, 2, 3} E. {0, 5, 26, 101}
Solution
Its “range” here is the result of the values gotten by To determine injectivity and surjectivity
substituting {-2, -1, 0, 1} into f (x) = x2 – 2 Examples
f (- 2) = (-2)2 – 2 f (0 ) = 02 – 2 Which of the following function f defined below is injective,
= 4 – 2 i.e 2 = 0 – 2 i.e –2 surjective and bijective
(1) f : R → R, f(x) = 2x+1
f (- 1) = (-1)2 – 2 f (1) = 12 – 2
= 1 – 2 i.e – 1 = 1 – 2 i.e –1 (2) f : R → R, f(x) = ex
The set is = {- 2, - 1, 2} (B) (3) f : R → +R, f(x) = ex
(4) f : R → R, f(x) = x2 + 2
2007/17 Neco x2 + 1
If f : x → sec x , find the range ; if the domain is Solution
the set {x : 00  x  600} (1) Let a : bR and suppose that
A. {y : 1  y  2} B. {y : 12  y  1} f(a) = f(b)
Then 2a + 1 = 2b + 1
C. {y : 0  y  1} D. {y : – 12  y  1
}
2 So 2a = 2b and hence a = b
E. {y : –1  y  1} Thus, f is injective
Solution
Here we substitute x value between 00 and 600 To show surjectivity
When x = 00 Let yR be such that
Sec x = 1 f(x) = y
Cos x so 2x + 1 = y
= 1 = 1 i.e 1 x = 1 (y – 1)R
0
cos 0 1 2
When x = 600 Even if we take y = 1 then x = 0R
Sec x = 1 Accordingly f is surjective and hence bijective
cos 60
= 1 i.e 2 (2) Let a, bR such that
½ f(a) = f(b)
Introducing y to represent the range and minding our Then, ea = eb
inequalities. Log e e a = Log e e b a Log e e = b Log e e
Domain {x : 00  x  600} becomes range  a = b Hence f is injective
{y : 1  y  2 } (A)
459
To show surjectivity Value of a function
Let yR be such that A function takes another value when the function is
f(x) = y redefined; it’s redefinition can be real numbers or the
Then ex = y multiple of the former variable.
Thus x = Log e y
2003/14b
Log of negative numbers do not exist (Range = R) The functions f and g are defined as
Hence f is not surjective hence not bijective f : x → 3x – 2
1
(3) However, since we restrict our range to R+ , we g:x→ ( x  0)
x
conclude here that f is surjective and so it is bijective
Evaluate : (i) f(– 2) (ii) g ( 12 )
(4) Let a, bR such that Solve (iii) f(x) = g( 12 )
f(a) = f(b) (iv) f(x) = g(x)
Solution
Then a2 + 2 = b2 + 2 (i) f(x) = 3x – 2
f(– 2) = 3(–2) – 2
a2 + 1 b2 + 1 =–6–2
You can observe that a2 = b2 or a =  b. Accordingly =–8
it is possible to have a ≠ b when f(a) = f(b). 1
( ii ) g(x) =
Thus f is not injective ( and so not bijective) x
1
g ( 12 ) = 1 ÷
To show surjectivity 2
Let yR be such that 2
=1 ×
f(x) = y 1
Then , x2 + 2 = y = 2
x2 + 1
Solving x =  y −2 ( iii ) f(x) = g( 12 )
1− y 3x – 2 = 2
y − 2 is not defined for all values of 3x = 4
Observe that x = 4/3
1− y
y( y = 0 makes the result becomes −2 which is ( iv ) f(x) = g(x)
not real.Thus, f is not surjective 3x – 2 =
1
x
2003/8 Exercise 28.6 x(3x – 2) = 1
Which of the following functions is/are one – to –one? 3x2 – 2x = 1
f : x →x2 , g : x →2x + 1 , h : x → x 3x – 2x – 1 = 0
2

Factorizing
A f only B g only C h only D f and h only
3x2 – 3x + x – 1 = 0
3x(x – 1 ) + 1(x – 1) = 0
2000/10 theory Exercise 28.7
(3x + 1 ) + (x – 1) = 0
( a ) If g : x → 2x – 1 , for all x R, the set of 3x + 1 = 0 or x – 1 = 0
5x + 3 3x = – 1 or x = 1
real number, find the : ( i ) domain of g; x = – 1/3 or 1
(b) Determine whether or not g in 10(a) is onto
(c) find the range of value of x for which 2x2 + x - 6< 0 2012/12 f/m
A polynomial is defined by f(x + 1) = x3 + 3x2 – 4x + 2,
find f(2)
A–8 B–2 C2 D8
Solution
First, we solve for x in f(x + 1) and f(2)
x+1=2
x=2–1
x=1
Thus f(2) is obtained by substituting 1 for x
f(2) = 13 + 3(1)2 – 4(1) + 2
=1+3–4+2
=2 C

460
1989/16 UME 2006/15a
If f (x) = 2x2 – 5x + 3, find f (x + 1) Under the mapping h(x) = px2 – qx + 2 ,
A.2x2– x B.2x2 – x + 10 C.4x2 + 3x + 2 D.4x2 the image of 3 is 14 and the image of –2 is 24. Find:
+ 3x + 12 (i) the values of p and q
Solution (ii) the elements whose image is 4
The question says substitute x + 1 for x Solution
i.e. f (x +1) = 2 (x +1)2 – 5 (x + 1) + 3 (i) h(x) = px2 – qx + 2
= 2(x2 + 2x + 1) – 5x – 5 + 3 When x = 3, h(x) = 14
= 2x2 + 4x + 2 – 5x – 5 + 3 14 = 32p – 3q + 2
Collect like terms 14 = 9p – 3q + 2
= 2x2 + 4x – 5x – 5 + 3 + 2 12 = 9p – 3q --------(1)
= 2x2 – x (A) When x = –2, h(x) = 24
24 = (–2)2p – (–2)q + 2
1988/16 UME 24 = 4p + 2q + 2
If g (y) = y – 3 + 11 what is g (y + 3) ? 22 = 4p + 2q --------(2)
11 y2 – 9 From (1) divide through by 3
A. y + 11 B. y + 11 From (2) divide through by2
11 y (y + 6) 11 y (y + 3) 4 = 3p – q
C. y + 30 + 11 D. y + 3 + 11 + 11 = 2p + q
11 y (y +3) 11 y (y – 6) 15 = 5p
Solution 3= p
The question means substituting y + 3 for y Substitute p value into (1)
g (y + 3) = ( y + 3) – 3 + 11 4 = 3p – q becomes
11 (y + 3)2 – 9 4 = 3(3) – q
4=9–q
= y+3–3 + 11
q=9–4 =5
11 (y + 3)2 – 32
To enable us solve by difference of two squares ( ii ) h(x) = 4, find x
= y + 11 First we substitute for p and q
11 (y + 3 – 3) (y + 3 + 3) h(x) = px2 – qx + 2 becomes h(x) = 3x2 – 5x + 2
= y + 11 Thus 4 = 3x2 – 5x + 2
11 y (y + 6) (A) 3x – 5x – 2 = 0
2

Factorizing (3x + 1) ( x – 2) = 0
2014/5 f/m x = – 1/3 or 2
If f(x) = 3x3 + 8x2 + 6x + k, and f(2) = 1, 2007/15a Exercise 28.8
find the value of k Given that f(x) = 2x – 1 and g(x) = x2 + 1
A – 67 B – 61 C 61 D 67 (i) find f(1 + x);
Solution (ii) find the range of value of x for which f (x) < – 3
f(x) = 3x3 + 8x2 + 6x + k (iii) simplify f(x) – g(x)
f (2) = 1
1 = 3(2)3 + 8(2)2 + 6(2) + k 2008/14a Exercise 28.9
1 = 24 + 32 + 12 + k The function f is defined as f : x → 3x2 – 5x
1 = 68 + k (i) Evaluate f (–2)
– 67 = k (A) −7
(ii) Find the values of x for which f(x) =
4
2013/20 2009/14c Exercise 28.10
The function f : R → R is defined by The function f and g are defined as
3x + 2 : x > 4
f : x → x – 2 and g : x → 2x2 – 1
f( x ) = 3x – 2 : x = 4
Solve ( i ) f(x) = g  − 
1
5x – 3 : x < 4 ( ii ) f(x) + g(x) = 0
Find f(4) – f(–3 )  2
A 28 B –26 C –26 D –28 2004/15c ( Nov ) Exercise 28.11
Solution The functions f and g are given as
f(x) is defined for three conditions here 3
x > 4 i.e 5, 6, 7… +∞ f(x) = x + and g(x) = 2x + 1
x
x=4 Evaluate : (i) g(–2) (ii) f (– 12 )
x < 4 i.e 3, 2, 1… – ∞
For f(4), only 3x – 2 fits into it
2004/14b Exercise 28.12
For f(–3), only 5x – 3 fits into it
Thus, f(4) – f(–3 ) = [ 3(4) – 2 ] – [ 5(–3) – 3 ]
Given f : x → 3x – 1 and g : x → x2 + 1
= (12 – 2 ) – (–15 – 3) (i) Evaluate g(–2) – f(–1)
= 10 + 18 (ii) Simplify f(x) + g(x)
= 28 ( A ) (iii) Solve f(x) = g(x)
461
Composition of function In the diagram, g : x → y and h : y → z
Use the diagram to answer questions 25 and 26
If f(x) = x and g(x) = 2x – 1, find ( i ) gf(x) ( ii ) fg(x)
2
25. Find h[g(3)]
Solution
gf(x) = g( f(x) ) AS B C {s, } D {s, w, }
= g(x2 ) Solution
= 2x2 – 1 g(3) = s
But fg(x ) = f( g(x) ) Thus, h[g(3)] = h(s)
= f ( 2x – 1 ) = B.
= ( 2x – 1 )2
= 4x2 – 4x + 1 26. goh is
A one to one B one to one onto
2015/6 f/m C a relation D a series
Given that f: x → x2 and g : x → x + 3 Solution
where xR find fog(2)
A 25 B9 C7 D5
Solution
fog (x) = f (x + 3)
= (x + 3)2
fog(x) = x2 + 6x + 9 2012/1 f/m
fog (2) = 22 + 6(2) + 9 Two functions g and h are defined on the set R of real
= 4 + 12 + 9 = 25 1
Alternatively numbers by g : x → x2 – 2 and h : x → , x  – 2, find
x+2
fog (2) 1
g(2) = 2 + 3 : (a) h– 1 the inverse of h : (b) goh when x = –
2
=5 Solution
fog(2) = f(5) 1
= 52 = 25 (a) h : x →
x+2
2014/11a f/m 1
x −1 Let y=
x−3 1 x+2
If f(x) = ,x and g(x) = , x  – 1,
2x − 1 2 x +1 y(x + 2) = 1
find gof yx + 2y = 1
Solution yx = 1 – 2y
x−3  1− 2y
gof = g   x =
 2x − 1  y
x−3 Change y to x for proper identification
−1
1 − 2x
= 2x − 1 h– 1 (x) = , x0
x−3 x
+1
2x − 1  
  3 2
x − 3 − 1(2 x − 1) x − 3 + 1(2 x − 1)  1 1
=  (b) goh  −  = g   i.e 1 =1
2x −1 2x −1  2   1  2 3
− +2
x − 3 − 2x + 1 x − 3 + 2x −1  2 
=  2
2x −1 2x −1 = g 
−x − 2 3x − 4 3
=  2
4 14
2x −1 2x −1 =   − 2 =
2
–2= −
−x − 2 2x −1 −x − 2 3 9 9
=  =
2x −1 3x − 4 3x − 4 2013/5 f/m Exercise 28.13
If f(x) = x2 and g(x) = sin x, find gof
2012/25 and 26 f/m A sin2x B sin x2 C (sin x) x2 D x sin x
2006/6 f/m Exercise 28.14
X g Y h Z Two functions f and g are defined by f : x→3x – 1 and
0 r g : x →2x3, evaluate fg (-2)
1 s A - 49 B - 47 C -10 D -9
2 t 2009/29 (Nov) f/m Exercise 28.15
3 u The functions f and g are defined on the set R, real
O numbers by f : x →2x2 – 3 and g : x → 4 – x.
4 v Find fog.
5 w A x2 – 16x + 29 B x2 + 16x – 29
C 2x – 16x – 29
2
D 2x2 + 16x – 29
462
Inverse of a function 1 − 2 (2)
g – 1 (2) =
If y = f (x) is a function, we obtain the inverse function f –1 2 − 1
by solving the equation y = f (x) for x in terms of y then 1− 4
later change y to x = = –3 D.
1
2015/9a f/m
The function f : x → x2 +1 and g : x → 5 – 3x are 2014/15 and 16 f/m
defined on the set of real numbers, R 1 3
(i) state the domain of f – 1, the inverse of f A function is defined by h : x → 2 – ,x .
2x − 3 2
(ii) find g – 1 (2) Use the information to answer question 15 and 16
Solution
f : x → x2 +1 15. Find h – 1 (x), the inverse of h.
First, we find f – 1 3x − 4 7 3x − 7
A h– 1 (x) = ,x B h– 1 (x) = ,x2
Let y = x2 + 1 2x − 7 2 2x − 4
Next, we make x the subject formula 2x − 7 3 4x − 7
C h– 1 (x) = ,x D h– 1 (x) = ,x2
y – 1 = x2 4x − 3 4 2x − 4
y −1 = x Solution
1
Change y to x for proper identification h: x→2–
2x − 3
f – 1 (x) =  x − 1 1
The domain here will be R – f – 1(x) not real number Let y = 2 –
2x − 3
f– 1 (0) =  − 1 is not real 1
= 2–y
f – 1 (– 1) =  − 1− 1 =  − 2 is not real 2x − 3
In fact all x = R– is not real 1 = (2 – y)(2x – 3)
Thus domain is {x : x R+, x  0, x  R –} 1 = 2(2x – 3) – y (2x – 3)
i.e {x : x R+} 1 = 4x – 6 – 2xy + 3y
1 + 6 – 3y = 4x – 2xy
(ii) g– 1(2) 7 – 3y = x (4 – 2y)
First, we find g – 1 7 − 3y
= x
g : x → 5 – 3x 4 − 2y
Let y = 5 – 3x Change y to x for proper identification
3x = 5 – y 7 − 3x 3x − 7
5− y h– 1 (x) = or ,x2 B.
x= 4 − 2x 2x − 4
3
Change y to x for proper identification 1
–1 5− x 16. Find h– 1  2 
g (x) =
3
A6 B 11/6 C 11/4 D 5/ 3
Thus, g – 1 (2) = 5 − 2 Solution
3
3 3x − 7
= =1 h – 1 (x) =
3 2x − 4
2013/35 f/m 1
1 3  − 7
x +1   = 2
If g(x) = , (x  – 2), find g – 1 (2) h –1 2
x+2 1
2  − 4
A3 B2 C–2 D–3 2
Solution 3 2
x +1 = −7  −4
g(x) = 2 2
x+2 11
x +1 = –  −3
Let y = 2
x+2 11 1
y(x + 2) = x + 1 =– −
2 3
yx + 2y = x + 1 11
yx – x = 1 – 2y =
6
x(y – 1) = 1 – 2y 2015/10 f/m
1− 2y 2x − 1
x= Given that f: x → , x  2,
y −1 x+2
Change y to x for proper identification find f – 1, the inverse of f
1 − 2x 1 + 2x 1 − 2x
g – 1 (x) = ,x1 A f –1: x → , x  2 B f –1: x → , x  –2
x −1 2− x x+2

463
1 − 2x 1 + 2x 2004/16 Neco (Dec) f/m Exercise 28.23
C f –1: x → , x2 D f –1: x → , x  –2
x−2 x+2 If f : x → 3 – 2x, find f – 1
Solution A. x – 3 B. x – 1 C. 3 – 3x
2x − 1 2
f:x→ , x2
x+2 E. 3x – 3 D. 2x – 3
2x − 1
Let y = 2000/10 theory f/m Exercise 28.24
x+2 ( a ) If g : x → 2x – 1 , for all x R, the set of
y(x + 2) = 2x – 1 5x + 3
xy + 2y = 2x – 1 real number, find the :
2y + 1 = 2x – xy ( ii ) g – 1 (iii) value of x for which g – 1 does not exist
2y + 1 = x(2 – y)
2y +1 2005/33 Neco f/m Exercise 28.25
= x
2− y Let g : x → x3 + 5 be a function on the set of real number,
Change y to x for proper identification find the inverse of f(x)
f – 1 : x → 2x + 1 , x  2 A. A. x − 5 B. 3 x + 5 C. 4 x 2 − 5
2− x
D. 3 x −5 E. x2 + 5
2006/9 f/m Exercise 28.16
1 1
If f (x) = , x ≠ 2, find f – 1 (- ). Joint cases
2− x 2
A 4 B 0 C -2 D -4 1992/10 (Nov) f/m
Two functions f(x) and g(x) are defined on the set of real
2009/8 (Nov) f/m Exercise 28.17 numbers by f(x) = 3x2 – 2 and g(x) = x + 3.
A function f is defined by f: x →3x - 2 , Find: (a) f(–2) ; (b) g – 1  − 3 
5
 4
find the inverse of f. (c) the value of x for which f (g (x)) = g (f(x))
A 1 (3x + 2) B 1 (3x + 6)
15 15 (d) f – 1 (g (4))
C 1 (5x + 2) D 1 (5x + 6) Solution
15 15 (a) To find f(–2)
f(x) = 3x2 – 2 becomes
2003/35 f/m Exercise 28.18
f(–2) = 3 (–2)2 – 2
4 x −1 = 12 – 2 i.e. 10
If f : x → , where x R, find f – 1 (- 2).
3
A–3 B −5 C −4 D −1 (b) g – 1  − 3  First, we find g – 1
4 5 3
 4
2010/11b Neco f/m Exercise 28.19 Let y = x + 3
Find the inverse of the function f(x) = 5x + 11 Next, we make x subject of formula
2x – 4 y–3=x
2000/9 f/m Exercise 28.20 Changing y to x for proper identification
Given that f : x → 7 – x , xR (the set of numbers),  g – 1 (x) = x – 3
find f – 1
Thus g – 1  − 3  = − 3 – 3
A. x + 7 B. x C.–7 D.7 – x 4
 4
7 x
15
2002/25 f/m Exercise 28.21 = −
Given that f; x → 1 , x  – 1 , find f – 1(x) 4
2x + 1 2 (c) f (g (x)) = g (f (x))
A. 2x + 1 B. 1 – x , x  0 f (x + 3) = g (3x2 – 2)
2x 3(x + 3)2 – 2 = 3x2 – 2 + 3
C. 2x , x  1 D. 2x – 1 3(x + 6x + 9) – 2 = 3x2 + 1
2

1–x 3x2 + 18x + 27 – 2 = 3x2 + 1


24 = –18x
2002/20 f/m Exercise 28.22 4
x = − 24 = −
If f (x) = x – 4 , (x  4,  32 ), for all 18 3
2x – 3 xR, find the value of x for (d) f – 1 (g (4))
which f – 1 does not exist. First, we find g(4) from g(x) = x + 3.
A. 1 B. 3 C. 2 D. 4 g (4) = 4 + 3 i.e 7
2 2
Next, f – 1 (g (4)) = f – 1 (7) , But we find f – 1 of f(x)
Let y = 3x2 – 2
464
Next, we make x the subject of formula = 2x + 2 − 2
y + 2 = 3x2
y + 2 = x2 = 2x
3
y+2 =x
Thus f – 1 g (2) = 2 (2) = 4 i.e 2 (B)
3
Changing y to x for proper identification 2002/15 Neco f/m
If f : x – 3x, and g : x → 2sin x, the formula for gf – 1 is
x+2
 f – 1 (x) = A. 2 sin x B. 2 sin x C.2sin 3x D. 3 sin 2 x
3 3 3 3
7+2 Solution
Thus ,f – 1 (7) = =  3 To find gf – 1 , first, we find f – 1
3
f (x) = 3x
1998/4 (Nov) f/m Let y = 3x
(a) If f(x) = x – 1 , x  1 and x 2, find f – 1 (x) y =x
x– 2 3
Solution f –1(x) = x
Let y = x – 1 3
x– 2 Thus, gf – 1 = g ( 3x )
Next, we make x the subject formula = 2 sin x (B)
y(x – 2) = x – 1 3
xy – 2y = x – 1 2007/ 26 Neco f/m
Collect terms in x together If f : x → 2 , g : x → x + 1 and h : x → x ,
xy – x = 2y – 1 x 2
x(y– 1) = 2y – 1 find f –1ogoh –1 (1)
x = 2y – 1 A. 1 B. 2 C. 4 D. 3 E. 2
y–1 3 3 3 2
Thus f – 1 (x) = 2x – 1 Solution
x–1 First , we find f –1 and h –1
( b) If f : x → 2x – 5 and g : x → sin x, f(x) = 2 h(x) = x
where x is a real number , find : x 2
( i ) (fog)(x) (ii) (gof)    let y = 2 let y = x
2 x 2
Solution x= 2 x = 2y
( i ) (fog) = f(sin x) y h –1(x) = 2x
= 2(sin x) – 5 –1
f (x) = 2
= 2 sin x – 5 x
(ii) (gof)    First, we find f –1ogoh–1(x) = f –1og(2x)
2 = f –1(2x + 1)
First (gof) (x) = g(2x – 5) = 2
= sin(2x – 5) 2x + 1
Thus, (gof)  = sin 2 ×  – 5 Thus, f –1 ogoh–1 (1) = 2 = 2 (B)
2 2 2 (1) + 1 3
= sin 175
= 0.08716 2004/9 Neco f/m Exercise 28.26
Given f(x) = 1 , and g(x) = x where xR,
2002/14 Neco f/m x2 – 1 x+2
If f : x → x2 + 2 and g : x → 2x + 2, find f – 1g(2) find the : (i) inverse of g(x) ;
A. 1/2 B. 2 C. 4 D. 36 E. 38 (ii) expression g–1 f(x)
Solution (iii) largest domain of f(x) and g(x) respectively
To find f – 1 g(2) first, we resolve f – 1
f(x) = x2 + 2 2001/1 Neco f/m Exercise 28.27
Let y = x2 + 2 (a) State the condition necessary for a function to have
Next, we make x subject formula inverse.
y – 2 = x2 4x − 3 4
(b) If f : x → , and g : x → 1+ , find
x= y−2 2x +1 2x +1

f – 1 (x) = x − 2 i. f –1 (x) ii. g –1 (x) iii. f –1 o g –1 (0) iv. g –1 of –1 (0)


Next, we find f – 1 g(x)
= f – 1 (2x + 2)
465
Chapter twenty nine If x + 2 is a factor of the polynomial
Remainder, factor theorems and partial fractions x +2 = 0
x=–2
Remainder and factor theorems Put x = – 2 into the given equation
These theorems take their roots from long division with 6(– 2)4 + 2(– 2)3 + 15(– 2) + 5 =
numbers. 96 – 16 – 30 + 5 = 55
Now, let us study some long division cases Since the result is not zero, x + 2 is not a factor
Evaluate ( i ) 13  2 ( ii ) 12  2 If 3x + 1 is a factor of the polynomial
Solution 3x + 1 = 0
(i) 6 3x = – 1
2 13 x = – 1/ 3
–1
12 Put x = /3 into the given equation
1 2 2
6(– 1/3)4 + 2(– 1/3)3 + 15(– 1/3) + 5 = 27 – 27 – 5 + 5
When 2 divide 13, we get 6 remainder 1
=0 (A)
13 = 2 × 6 + 1
divided = divisor × quotient + remainder
2014/9 f/m
If (x + 2) and (3x – 1) are factor of 6x3 + x2 – 19x + 6,
( ii ) 6
find the third factor
2
12
A 2x – 3 B 3x + 1 Cx–2 D 3x + 2
12
Solution
00
If (x + 2) is a factor: then
When 2 divides 12, we get 6 there is no remainder
Performing the long division
Thus 2 is a factor of 12.
6x2 – 11x + 3
In both cases, 2 is their divisor and 6 is their quotient.
(x + 2) 6x + x2 – 19x + 6
3
while 13 and 12 are divided for i and ii respectively.
– ( 6x3 + 12x2)
– 11x2 – 19x
Remainder theorem becomes factor theorem when the
– ( – 11x2 – 22x )
divisor divides the divided without a remainder.
3x + 6
– ( 3x + 6)
Remainder theorem states that when we divide a
0 0
polynomial f(x) by another polynomial ax – b and
To get third factor;
there is a remainder r, then the value of the polynomial
We factorize 6x2 – 11x + 3 = 6x2 – 9x – 2x + 3
f(x) for the value x,
= 3x(2x – 3) – 1(2x – 3)
Where ax – b = 0
= (3x – 1)(2x – 3)
ax = b
(2x – 3) is the third factor A.
x = b
a
2013/9 f/m
i.e. f ( ba ) = r,
Given that f(x) = 2x3 – 3x2 – 11x + 6 and
Also if f( c ) = r f (3) = 0, factorize f(x)
Then r is the remainder when f(x) is divided by x – c A (x – 3)(x – 2)(2x + 2) B(x + 3)(x – 2)(x – 1)
Meanwhile C (x – 3)(x + 2)(2x – 1) D (x + 3)(x – 2)(2x – 1)
if f ( ba ) = 0 Solution
If f(3) = 0
Then ax – b is a factor of the polynomial f( x ) Then x = 3 and
Also if f(c ) = 0 x–3=0
Then x– c is a factor of f(x), this is the factor theorem. (x – 3) is a factor of f(x) here
To get the other factors, we perform the long division
2015/9 f/m 2x2 + 3x – 2
Which of the following is a factor of the polynomial (x – 3) 2x3 – 3x2 – 11x + 6
6x4 + 2x3 + 15x + 5? – ( 2x3 – 6x2)
A 3x + 1 Bx+1 C 2x + 1 D x + 2 3x2 – 11x
Solution – ( 3x2 – 9x )
For ease of working, let us try x + 1 and x + 2 –2x + 6
If (x + 1) is a factor of the polynomial – ( –2x + 6 )
x+1=0 0 0
x=–1 To get the other factors,
Put x = – 1 into the given equation we factorize 2x2 + 3x – 2 = 2x2 + 4x – x – 2
6 (–1)4 + 2(– 1)3 + 15(–1) + 5 = 6 – 2 – 15 + 5 = 2x(x + 2) – 1(x + 2)
=–6 = (2x – 1)(x + 2)
Since the result is not zero, x + 1 is not a factor Thus f(x) factorize to (x – 3)(x + 2)(2x – 1)
466
2012/10 f/m then 2x = –1
If (x + 1) is a factor of the polynomial x=–½
x3 + px 2 + x + 6 = 0, find the value of P Then f(– ½ ) = Remainder
A–8 B–4 C4 D8 f(– ½ ) = 2
Solution i.e. k(– ½ )3 + (– ½ )2 – 5(– ½ ) – 2 = 2
If (x + 1) is a factor, then –k + 1 + 5 – 2 = 2
x+1=0 8 4 2
x=–1 Multiply through the LCM 8
Put x = – 1 into the given equation – k + 2 + 20 – 16 = 16
(– 1)3 + p(– 1)2 + (–1) + 6 = 0 – k + 22 = 32
–1 + p – 1 + 6 = 0 – k = 10 i.e k = – 10 ( B )
p+4=0 1994/14 PCE
p=–4 (B) When a polynomial is divided by c + 2, the quotient is 3c – 3
and the remainder is 5. find the polynomial.
A.3c2 +3c –6 B.3c2 +3c –1 C.3c2 +3c +5 D.3c2 –3c +1
Remainder cases Solution
2004/35 Recall that
Find the remainder when 3x3+5x2–11x + 4 is divided by x +3. Divided = divisor × quotient + remainder
A. – 4 B.4 C.1 D. – 1 Polynomial a(x) = divided by b(x) × quotient q(x) + remainder
Solution Polynomial = ( c + 2 ) × ( 3c – 3 ) + 5
Since x + 3 divides the expression Opening the brackets
Then f(–3 ) = remainder = 3c2 – 3c + 6c – 6 + 5
Thus; 3(–3 ) + 5 (–3 )2 – 11 (–3 ) + 4
3 = 3c2 + 3c – 1 ( B )
= 3(–27) + 5(9) + 33 + 4 2007/14 PCE Exercise 29.1
= – 81 + 45 + 33 + 4 Find the quotient when x3 + x2 – x – 1 is divided by x – 1.
= – 81 + 82 A x2–2x + 1 B. x2 – 2x –1 C.x2 + 2x +1 D.x2 + 2x – 1
=1 (C)
1993/18 PCE Exercise 29.2
2003/ 26 PCE What is the remainder when x3 + 5x2 – 3x – 6 is
Find the remainder when the polynomial divided by x + 1 ?
f(x) = 3x3 + x2 – 8x + 4 is divided by x + 2 A.6 B.1 C. – 1 D. – 11
A.0 B. – 2 C.2 D.1
Solution 2005/ 48 PCE Exercise 29.3
Since x + 2 divides the given expression What is the remainder when 5x3 – 13x + 4 is divided
Then f(–2 ) = remainder by x + 2 ?
= 3(– 2 )3 + (– 2 )2 – 8(– 2 ) + 4 A. – 4 B.14 C. – 10 D. 4
= 3(– 8 ) + 4 + 16 + 4
= – 24 + 24
= 0 Thus the remainder is 0 ( A )

1995/14 PCE special case


Find the remainder when 6x3 – 5x + 4 is divided by 2x – 1
A.2 ¼ B.2 ½ C.2 ¾ D.3
Solution
Since 2x – 1 divides the expression
Then f( ½ ) = Remainder from 2x – 1 = 0
1 5
6 ( ½ ) – 5 (½) + 4 = 6 ( /8) – /2 + 4
3
2x = 1
= ¾ – /2 + 4
5
i.e x = ½
= 3 – 10 + 16
4
= 19 – 10
4
= 9/4 i.e. 2 ¼ ( A )
2006/29
Find the value of k if the expression kx3 + x2 – 5x – 2
leaves a remainder 2 when it is divided by 2x + 1.
A.8 B. – 10 C.10 D. – 8
Solution
If 2x + 1 divides the function leaving a remainder 2
467
Partial fractions Substituting for A and B values into our initial proposition
Partial fraction is a form of algebraic fractions that 2 x + 11 3 1
= − (D)
follows certain rules in resolving it instead of the usual
LCM method of solving fractional problems. The given
(x − 2) (x + 3) x − 2 x + 3
fraction, which is single; is resolved into (parts) partial
fractions depending on the following: 2005/5 Neco (Dec)
1 x+3
Express in partial fractions.
I. To every linear factor of the form
x−a
in the (x − 2)(x + 4)
denominator; there corresponds a partial fraction of Solution
A x+3 A B
Let  +
the form
x−a (x − 2)(x + 4) x − 2 x + 4
II. To every repeated linear factor of the form (x − a )
r
Multiplying by (x − 2)(x + 4) on both sides
in the denominator, there corresponds partial of the equation
fractions of the form x + 3 = A (x + 4) + B (x − 2) ----- *
A1 A2 Ar To eliminate B, we put x = 2 into *
+ + …+
x − a ( x − a) 2
( x − a) r 2 + 3 = A(2 + 4) + B(2 – 2)
5 = A(6) + B(0)
III. To every quadratic factor of the form ax2 + bx +c
5 = 6A
in the denominator, there corresponds a partial
Ax + B
5
fraction of the form 2 if it cannot be A=
ax + bx + c 6
factorized. But if it can be factorized we treat as in Next, to eliminate A, we put x = – 4 into *
I above; as for repeated quadratic factors, treat – 4 + 3 = A (0) + B (– 6)
as in II above –1 = – 6B
1
IV. If the degree of the numerator is greater than or B =
6
equal to that of the dominator, we carry out a
Substituting for A and B values into our initial proposition
division until the degree of the numerator is strictly
less than that of the denominator. x+3 5 1
= +
(x − 2)(x + 4) 6(x − 2) 6(x + 4)
In order to determine the constants we introduced in the
above rules, we may substitute values for x (which will Test your answer
eliminate some of our propose A, B,C…). However, for Put x = 1 on both sides.
greater speed and simplicity this may be combined with
4 5 1
equating coefficients of x, x2 or x3 as the case may be required to prove +
(− 1)(5 ) 6(− 1) 6(5)
2004/ 26 Neco (Dec)
2 x + 11 4 5 1
Resolve into partial fractions RTP − +
(x − 2)(x + 3) −5 6 30
2x 11 11 1 − 25 + 1 24 4
A + B + RTP =− i.e − QED
x−2 x+3 x−2 x+3 30 30 5
3 1 3 1 E 1 − 3
C + D −
x−2 x+3 x−2 x+3 x−2 x+3 1994/1
10 x + 1 K 3
Solution If  + ,find the value of K
2 x + 11 A B (x − 2)(x + 1) x − 2 x + 1
Let  +
(x − 2)(x + 3) x − 2 x + 3 A–3 B1 C3 D7 E 13
Multiplying by (x – 2) (x + 3) on both sides Solution
of the equation Multiplying both sides by (x – 2) (x + 1)
2x + 11 = A(x +3) + B(x – 2) ------*** 10x + 1 = K(x + 1) + 3(x – 2)
Put x = 2 into *** to eliminate B Since we are to find K, eliminate 3(x – 2) by
2(2) + 11 = A(2 + 3) + B(0) Putting x = 2
15 = 5A 10(2) +1 = K(3) + 3(0)
A=3 21 = 3K
Next, to eliminate A, put x = – 3 into *** K = 7 (D)
2(–3) + 11 = A(0) + B (–3 – 2)
– 6 + 11 = –5B
5 = –5B
it follows that B = –1
468
1994/4 a Equating coefficient of x from *
1 –12 = 2A + 3B ------------- (2)
Resolve into partial fractions From (1) A = 5 – B
r (r + 2)
Substitute into (2)
Solution
–12 = 2(5–B) + 3B
1 A B – 12 = 10 – 2B + 3B
Let  +
r (r + 2) r r + 2 – 12 = 10 + B
Multiplying both sides by r(r + 2) – 22 = B
1 = A (r +2) + B r ------** Thus A = 5– (–22)
To eliminate B, we put r = 0 into ** = 27
1 = A (0 +2) + B (0) Substituting for A and B value into our initial proposition
1 = 2A 5 − 12 x 27 22
= −
A=
1 ( )
6 x + 5 x + 1 3x + 1 2 x + 1
2

2 You can confirm by putting x = 1 on both sides; you


To eliminate A, we put r = – 2 into **
will get
1 = A (– 2 + 2) + B (– 2 ) 7 7
1 = – 2B − =−
12 12
1
B= –
2 2000/10b (Nov)
Substituting for A and B value into our initial 2 − 4x
Resolve into partial fractions
proposition (
x x2 − x − 2 )
1 1 1 Solution
= −
r (r + 2) 2r 2(r + 2) Factorizing x 2 − x − 2 = ( x − 2) ( x + 1)
2 − 4x 2 − 4x
Thus, =
2015/7 f/m Exercise 29.4 ( 2
)
x x − 2 x − 2 x(x − 2)(x + 1)
2x P Q 2 − 4x
Given that  + ,
Let
A
 +
B
+
C
( x + 6)( x + 3) x + 6 x + 3 x(x − 2)(x + 1) x x − 2 x + 1
find P and Q
Multiplying through by x(x – 2)(x + 1)
A P = 4 and Q = 2 B P = 2 and Q = 4
2 – 4x = A(x – 2)(x + 1) + Bx(x + 1) + Cx(x – 2)__**
C P = 4 and Q = – 2 D P = – 2 and Q = 4
To eliminate B & C ; Put x = 0 into **
2 = – 2A
1997/ 11 Exercise 29.5
Thus, A = –1
Given that
4 + 3x + 2 x 2 p Q R To eliminate C & A ; Put x = 2 into **
 + + 2 – 8 = 6B
(1 − 2 x )(1 + x )(1 − x ) 1 − 2 x 1 − x 1 + x Thus, B = – 1
Where P, Q and R are constants, find the value of R.
1 1 To ensure we get C; Put x = –2 into **
A −1 B− C D1 E2 2 + 8 = A(–4)(–1) + B(–2)(–1) + C(–2) (–4)
2 2 10 = 4A + 2B + 8C
But A is –1 and B is –1 Thus,
Cases with quadratic denominator that 10 = –4 – 2 + 8C
can be factorized 16 = 8C
2005/ 11 a (Nov) C=2
Substituting for A, B& C value into our initial proposition
5 − 12 x
Express in partial fractions 2 − 4x 1 1 2
6 x 2 + 5x + 1 =− − + Rearranging
x(x − 2)(x + 1) x x − 2 x +1
Solution
Factorizing the denominator 2 1 1
= − −
5 − 12 x 5 − 12 x x +1 x − 2 x
=
6 x + 5 x + 1 (3x + 1)(2 x + 1)
2 2000/1 Neco
5 − 12 x A B 4 + 3x + 2 x 2
 +
Let
(3x + 1)(2 x + 1) 3x + 1 2 x + 1
Resolve
( )
(1 − 2 x ) 1 − x 2
into partial fraction

Multiplying through by (3x + 1)(2 x + 1) Solution


The (1– x2 ) in denominator can factorize into (1– x)(1 + x)
5 −12 x = A(2 x + 1) + B(3x + 1) _____*
Equating constants
4 + 3x + 2 x 2 4 + 3x + 2 x 2
5 = A + B ------------ (1)
Thus
( =
)
(1 − 2 x ) 1 − x 2 (1 − 2 x )(1 − x )(1 + x)
469
4 + 3x + 2 x 2 A B C 2008/11a
Let  + +
( )
(1 − 2 x ) 1 − x 1 − 2 x 1 − x 1 + x
2
Express
2 x 2 − 5x +1
in partial fractions.
Multiplying through by (1–2x)(1– x)(1+ x) x 3 − 4 x 2 + 3x
4+3x+2x2 =A(1–x)(1+x)+B(1–2x)(1+ x)+C(1–2x)(1–x)--** Solution
Applying factorizing of polynomial to the denominator
To eliminate A and C ; Put x = 1 into **
4+3 +2 = A(0) + B(–1)(2) + C(0)
We try to make f(x) = 0 by trial and error
9 = –2B f(1) = (1)3 – 4(1)2 + 3(1) i.e 0
Thus, B = − 9 Thus x – 1 is a factor of x3 – 4x2 + 3x
2 Next, we apply long division to get the other factors
To eliminate A and B ; Put x = –1 into ** x2 – 3x
4 – 3 + 2 = A(0) + B(0) + C(3)(2)
3 = 6C; x–1 x3 – 4x2 + 3x
1 x3 – 1x2
Thus, C = 2
–3x2 + 3x
To get A ; Put x = 0 into ** –3x2 + 3x
4=A+B+C 0 0
But B is − 9 and C is 12 Thus, x3 – 4x2 + 3x factorizes into (x – 1)(x2 – 3x) and x2 – 3x
2 factorizes into x(x–3).
9 1 Thus x3 – 4x2 + 3x factorizes into x(x – 1)(x–3).
4=A– +
2 2 2 x 2 − 5x +1 2 x 2 − 5x +1
9 1 =
4+ − = A it follows that A = 8 x 3 − 4 x 2 + 3x x( x − 1)( x − 3)
2 2
Substituting for A,B & C values into our initial 2 x 2 − 5x +1 A B C
Let  + +
proposition x − 4 x + 3x
3 2
x x −1 x − 3
4 + 3x + 2 x 2 8 9 1 Multiplying through by x(x – 1)(x–3)
= − +
(1 − 2 x )(1 − x )(1 + x ) 1 − 2 x 2(1 − x ) 2(1 + x ) 2x2 – 5x + 1 = A(x – 1)(x–3) +Bx(x–3) + Cx(x – 1)----***
1994/15 Counter example To eliminate A and C ; Put x = 1 into ***
6x + P 4 2 2 – 5 + 1 = A(0) +B(–2) + C(0)
Given that = − –2 = –2B
2 x + 7 x − 15 x + 5 2 x − 3
2
B=1
Find the value of the constant P To eliminate A and B ; Put x = 3 into ***
A 20 B 12 C 6 D –10 E –22 2(3)2 – 5(3) + 1 = A(0) +B(0) + C(6)
Solution 4 = 6C
We apply the same partial fraction format: i.e C= 2
3
6x + P 4 2
= − To eliminate B and C ; Put x = 0 into ***
2 x + 7 x − 15 x + 5 2 x − 3
2
1 = A(–1)( –3) + B(0) +C(0)
Then multiplying through by (x + 5)(2x – 3) 1 = 3A
6x + P = 4(2x – 3) – 2(x + 5) A= 1
We equate constants to get P 3
P = –12 –10 Substituting for A,B & C values into our initial
P = –22 ( E ) proposition
1998/18 PCE Counter example 2 x 2 − 5x +1 1 1 2
If 3+x express in partial fractions is 5 - 4 find b = + +
(x–1)(x–b) x–b x–1 x − 4 x + 3x
3 2
3x x − 1 3( x − 3)
A. – 2 B. – ½ C. 2 D. 3 PPF/1 Exercise 29.6
Solution 3x 2 + 11x + 4
We apply the same partial fractions format: i.e Express in partial fractions.
x3 + 4x 2 + x − 6
3+x = 5 – 4
(x – 1 )(x – b) x–b x–1 2002/38 Neco Exercise 29.7
Then, multiplying through by (x – 1)(x – b) Resolve the rational function
3 + x = 5(x – 1) – 4 (x – b) 10 x + 11
Since our target is to find b; into partial fractions.
we put x = 1 to eliminate 5(x – 1)
x2 + x − 2
3 + 1 = 5(1 – 1) – 4(1 – b) 3 7 3 7
A + B −
4 = 5 × 0 – 4 + 4b x −1 x + 2 x −1 x + 2
4 = – 4 + 4b
7 7 7 3 7 7
4 + 4 = 4b C − D − E +
8 = 4b Thus b = 2 (C ) x + 2 x −1 x + 2 x −1 x + 2 x −1
470
2004/5 Neco (Dec) Exercise 29.8 Next, we substitute A value into (2)
3x + 4 x + 2
2 5A + 10C = –1 becomes
Resolve into partial fractions
(
x x 2 + 3x + 2 )  21  + 10C = –1
5 − 
 25 
1995/20 (Nov) Exercise 29.9 10C = –1 + 21
x+3 p Q 5
Given that 2 = + C= 8
x − 9 x + 18 x − 6 x − 3 25
Find the values of the constants P and Q Substituting for A,B & C values into our initial
A P = 4, Q = 3 B P = 3, Q = –2
proposition
C P = –3, Q = –1 D P = – 2, Q = 3
E P = – 4, Q = 2 2x 2 − x + 2 21 12 8
=− + +
( x + 2) (1 − 2 x)
2
25( x + 2) 5( x + 2) 2
25(1 − 2 x)
1996/11 Exercise 29.10
3x − 7 2
7 P Q Example PPF/2
If = + + , find P + Q
x + 2 x − 8x 8x x + 4 x − 2
3 2 3x 3 + 3x 2 + 3x + 2
Resolve into partial fractions
x 3 ( x + 1)
31 41 17 51 5
A B C D E
24 24 8 12 12 Solution
3x 3 + 3x 2 + 3x + 2 A B C
Let  + 2 + 3 + D
3
x ( x + 1) x x x ( x + 1)
Cases with quadratic denominator that has
repeated roots Multiplying through by x3(x +1)
3x3 +3x2 +3x +2 = Ax2(x +1) +Bx(x +1) + C(x+1) +Dx3 ---**
2000/11b To get C we put x = 0 into **
2x − x + 2
2 2=C
Resolve into partial fractions To get D we put x = –1 into **
( x + 2) 2 (1 − 2 x)
–3 + 3 –3 + 2 = –D
Solution D=1
We proceed as follows To enable us equate coefficients, let us expand **
2x 2 − x + 2 A B C 3x3+3x2+3x+2 = (Ax3 + Ax2) + (Bx2 +Bx) + (Cx+C) + Dx3
 + +
( x + 2) (1 − 2 x) x + 2 ( x + 2)
2 2
(1 − 2 x) Equating coefficients of x3 from the expansion
2 2
Repeated root (x + 2) was broken down as (x + 2) & (x + 2) 3=A+D
Multiplying through by (x + 2)2(1 – 2x ) But D is 1 3= A+1
2x2–x +2 = A(x +2)(1– 2x) + B(1–2x) + C(x +2)2---** A=2
By observation we eliminate C and A putting x = – 2 into** Equating coefficients of x2 from the expansion
2(– 2)2 – (–2) + 2 = A(0)(5) + B(5) + C(0)2 3=A+B
12 = 5B But A is 2 3=2+B
12 = B B=1
5 Substituting for A,B,C & D values into our initial proposition
To get A and C let us open up RHS of **
3x 3 + 3x 2 + 3x + 2 1 2
2x2– x +2 = A(2 –3x –2x2) + B(1– 2x) + C(x2 + 4x + 4) = 2+ 2 + 3 + 1
3
x ( x + 1) x x x ( x + 1)
Equating terms in x2 from the expansion
2 = –2A + C ---------( 1 ) 2004/11 Neco Exercise 29.11
Equating constants from the expansion 2x + 1
2 = 2A + B + 4C (a) Express in the form
But B is 12
(x + 1)(x + 2)2
5 A B C
+ + Where A, B and C are constants
2 = 2A + 12 + 4C x +1 x + 2 (x + 2)2
5
Cases with quadratic denominator that cannot
2 – 12 = 2 A + 4C
5 be factorized
2
− = 2 A + 4C
5 2005/6 Neco
–2 = 10A + 20C Resolve
3
into partial fractions
–1 = 5A + 10C -------(2) (x − 1)(x 2
− 5x + 2 )
Multiply ( 1 ) by 10 and subtract (2) Solution
–20A + 10C = 20 You will observe that (x2 – 5x + 2) cannot be factorized
–(5A + 10C = –1) 3 A Bx + C
Let  + 2
–25A = 21 (x − 1)(x 2
− 5x + 2 ) x − 1 x − 5x + 2
A = − 21
25 Multiplying through by (x – 1)(x2 – 5x + 2)
471
3 = A(x2 – 5x + 2) + (Bx + C)( x – 1) PPF/3 Counter example
3 = A(x2 – 5x + 2) + Bx(x – 1) + C( x – 1)------** Resolve x 2 + 4x + 1 into partial fractions
To get A, we put x = 1 into** x 3 + 3x 2 + 4 x + 2
3 = A (–2) + B(0) + C(0) Solution
3 = –2A i.e A critical look at the denominator shows that
3 f(-1) = -1 + 3 - 4 + 2 i.e 0
A= – x +1 is a factor
2
To eliminate B, we put x = 0 into** x2 + 2x +2
3 = 2A – C x+1 x3 + 3x2 + 4x + 2
But A is – 3 3 =2(–3 ) –C x3 + x 2
2 2 2x2 + 4x
C =–6 2x2 + 2x
2x + 2
Equating coefficient of x2 2x + 2
Note that Bx(x – 1) = Bx2 – Bx at ** 0 0
0 = A+B x3 +3x2 +4x +2 factorizes into (x +1)(x2 + 2x +2)
0 =–3+B You will observe that (x2 + 2x +2) cannot be factorized
2
x 2 + 4x + 1 A Bx + C
B= 3 Let  + 2
2
3 2
x + 3x + 4 x + 2 x + 1 x + 2x + 2
3 Multiplying through by (x +1)(x2 + 2x +2)
x−6
3 −3 2 x2 + 4x + 1 = A(x2 + 2x +2) + (Bx + C)(x +1)
Thus = +
( )
(x − 1) x 2 − 5x + 2 2(x − 1) x 2 − 5x + 2 x2 + 4x + 1 = A(x2 + 2x +2) + (Bx2 + Bx) + (Cx +C)---**
3x − 12 3 To eliminate B and C we put x = –1 into **
= −
2
2 x − 5x + 2 (
2(x − 1) ) 1– 4 +1 = A(1– 2 + 2) + B(0) + C(0)
–2=A
2007/36 Neco To eliminate B, we put x = 0 into **
x+3 1 = A(2) + C(1)
Express into partial fractions
(
x x2 + 2 ) 1 = 2A + C
1 = 2(–2) + C Thus C = 5
3 2 − 3x 3 2 − 3x
A + −
2x 2 x 2 + 2 ( ) B
(
2x 2 x 2 + 2 ) Equating coefficient of x2 at **
1=A+B
3 2 + 3x 3 2 + 3x −3 2 − 3x 1 = –2 + B thus, B = 3
C + − E +
2x 2 x 2 + 2( ) D
(
2x 2 x 2 + 2 ) (
2x 2 x 2 + 2 )
Thus
x 2 + 4x + 1
= –
2
+ 2
3x + 5
Solution 3 2
x + 3x + 4 x + 2 x + 1 x + 2x + 2
You will observe that (x2 + 2) cannot be factorized 2013/25 f/m
x+3 A Bx + C x2 + x + 4
Let  + 2
x x +2
2
(
x x +2 ) Express
(1 − x )( x 2 + 1)
in partial fractions
Multiplying through by x(x2 + 2) Solution
x + 3 = A(x2 + 2) + x(Bx + C) --------*** x2 + x + 4 A Bx + c
Equating constant Let  +
(1 − x)( x 2 + 1) 1− x x2 +1
3
3 = 2A; A = x2 + x + 4 = A(x 2 + 1) + (Bx + C)(1 – x)
2
C is not among the constants because x(Bx + C) = Bx2 + Cx x2 + x + 4 = A(x2 + 1) + Bx – Bx2 + C – Cx
x2 + x + 4 = Ax2 + A + Bx + C – Bx2 – Cx
Equating coefficient of x2 By comparing coefficients
0=A+B x2 : 1 = A – B ---- (1)
0= 3 +B x: 1 = B – C ---- (2)
2
By comparing constants
Thus B = – 3
2 Constant : 4 = A + C ---- (3)
Equating coefficient of x (2) + (3) gives A + B = 5 --- (4)
1= C (1) + (4) gives 2A = 6
care must be taken to fix B value into our given A=3
proposition Substituting A value into (1)
x+3 3 ( − 23 x + 1) 1 = 3 – B thus, B = 2
= + 2
(
x x2 + 2 2x )x +2
Substituting B value into (2)
1 = 2 – C thus, C = 1
3 2 − 3x x2 + x + 4 3 2x + 1
= + A. Hence = + 2
2x 2( x 2 + 2) 2
(1 − x)( x + 1) 1− x x +1

472
2014/12 f/m To eliminate B ; put x = – 2
x+P Q 2 6 (–2) = –3A + B(0)
If  + , –12 = –3A
( x − 1)( x − 3) x − 1 x − 3
A= 4
find the value of (P + Q) 6x 4 2
A–2 B–1 C0 D1 Hence = +
Solution (x + 2)(x − 1) x + 2 x − 1
x + P = Q(x – 3) + 2(x – 1) Confirm by putting x = 2 and you get 3 = 1+2
Put x = 3 to eliminate Q x3 + x 2 + 4x 4 2
Therefore = x+ +
3 + P = Q(0) + 2(3 – 1) 2
x + x−2 x + 2 x −1
3+p=4
P=4–3=1 2007/37 Neco
Equating constants x2 +1 B C
P = – 3Q – 2 If = A+
+ Where A, B and C are
4− x 2 −
2 x 2 +x
1 = – 3Q – 2
A+ B
3 = – 3Q constants, calculate
–1=Q C
Thus P + Q = 1 – 1 9 4 5 1
A. B. C. D. E. − 1
=0 C. 5 5 16 5
Alternatively : put x = 1 instead of equating constants Solution
1 + P = Q(1 – 3) + 0 The degree of x in numerator is equal to that of the
1 + P = – 2Q denominator; which is not allowed.
1 + 1 = –2Q Applying long division
Q = –1 –1
– x2 + 4 x2 + 1
– ( x2 – 4 )
1992 /11 (Nov) Exercise 29.12 5
3x + 4 x
2
P Qx + R x2 +1 5
= + 2 Thus = −1 +
If
( )
( x − 2) x + 1 x − 2 x + 1
, find P + Q + R 2
2 4− x 4 − x2
5
A–1 B2 C4 D5 E7 Next, we resolve into partial fractions
4 − x2
5 5 5
 i.e
4− x 2
2 −x (2 − x )(2 + x )
2 2
Cases with degree of numerator greater 5 A B
than or equal to the denominator Let  +
(2 − x )(2 + x ) 2 − x 2 + x
2005/10a Neco (Dec) Multiplying through by (2 – x)(2 + x)
3 2
x + x + 4x 5 = A(2 + x) + B(2 – x) -----**
Resolve into partial fraction
x2 + x − 2 To eliminate B; put x = 2 into**
Solution 5 = 4A + B(0)
The highest degree of x in the numerator is higher than 5 = 4A;
that of the denominator; which is not allowed; A= 5
4
Applying long division
x To eliminate A; put x = –2 into**
x2 + x–2 x3 + x2 + 4x 5 = A(0) + 4B
– ( x3 + x2 –2x ) 5 = 4B;
6x B= 5
4
x3 + x 2 + 4x 6x 5 5 5
Thus = x+ Thus, = +
x2 + x − 2 x2 + x − 2 (2 − x )(2 + x ) 4(2 − x ) 4(2 + x )
6x
Next, we resolve into partial fractions x2 +1 5 5
x2 + x − 2 Therefore = − 1+ +
4− x 2 4(2 − x ) 4(2 + x )
6x 6x
  A + B 5
x2 + x − 2 (x + 2)(x − 1) x + 2 x − 1 A+ B
−1+
1 5 1 4 1
and = 4 =  =  ie (D )
Multiply through by (x + 2)(x – 1) C 5 4 4 4 5 5
6x = A(x – 1) + B(x + 2) ------*** 4
To eliminate A; put x = 1 into ***
6 = A(0) + B(3)
6 = 3B
B=2
473
Chapter thirty (Differentiation) To differentiate by first principle means to work strictly by
Differentiation by first principle the ordinary definition or the derived definition and not
employing any theorem or result in calculus that will be
y Q ( x + x, y + y )
given later. Our working here shall be based on the ordinary
definition.
y IPLF 1 Examples
y)
P (x, Find the derivative of the following with respect to x using
x L the first principle, 1
y+ y
(i)x ( ii ) x2 ( iii ) 2x2 + x + 2 ( iv ) 2x + 1
(v) x–1 ( vi ) xn (n is a positive integer)
Solutions
(i) let y = x
Then y + y = x + x
O x subtracting
A B y + y – y = x +  x – x
x+ x
y = x
Consider the graph of a function y = f(x) with points
y = x = 1
P and Q very close to each other on the curve, where
x x
P(x, y) and Q (x + x, y + y). y being a small dy
increase in y due to a small increase x in x . = Lim y = Lim 1 = 1
Here, y and x are enlarged in the diagram, otherwise,
dx x 0 x x 0
they could not be seen from the diagram,
QL = y and PL = x (ii) Let y = x2
QL y Then y + y = (x + x)2
The slope of the chord PQ is = Subtracting
PL x
y + y – y = (x + x)2 – x2
As x 0, Q moves into coincidence with P and the
chord PQ produced becomes a tangent at P. y = x2 + 2xx + (x)2 – x2
y = 2xx + (x)2
Hence the slope of the tangent at P is the value that y
x Then y = 2xx + (x)2
approaches as x approaches zero. x x x
Thus, the slope of the tangent at P is y = 2x + x
Lim y x
dy
x 0 x = Lim y = Lim ( 2x + x )
This value is denoted by the symbol dx
x 0 x x 0
dy
(dee y , dee x) = 2x
dx
2
It is known as the first differential coefficient of y with (iii) Let y = 2x + x + 2 , then
respect to x.. It represents the rate of change of y with y + x = 2(x + x)2 + (x + x) + 2
respect to x. If y increase as x also increases then dy /dx y + x = 2[x2 + 2xx + (x)2] + (x + x) + 2
is also positive whereas if y decreases as x increases Subtracting
then dy /dx is negative. y + x – y = 2x2 + 4xx + 2(x)2 + x + x +2 – 2x2 – x – 2
y = 4xx + 2(x)2 + x
Derived definition form of first principle Then y = 4xx + 2(x)2 + x
Consider the function x x x x
y = f(x) ………..(1) y = 4x + 2x + 1
Deduction from the diagram above ,
x
Let y be a small increase in y due to a small increase x in x.
y + y = f(x + x)……..(2) dy = Lim y = 4x + 1
Subtracting (2) – (1) dx x 0 x
y + y – y = f(x + x) – f(x)
y = f(x + x) – f(x) (iv) Let y = 1 then
2x + 1
Then, y = f(x + x) – f(x)
y + y = 1
x x
2(x + x) + 1
Take limits as x 0
Subtracting
dy y + y – y = 1 – 1
= Lim y = Lim f(x + x) – f(x)
dx x 2(x + x) + 1 2x + 1
0 x x 0 x
dy y = 2x + 1 – 2x – 2x – 1
is known as the derived definition of /dx
[2(x + x) + 1] [2x + 1]
474
y = –2x 2003/10 (Nov)
[2(x + x) + 1] [2x + 1] Find from the first principles, the derivatives, with respect to
x, of y = 3x2
Next we multiply both sides by 1
x Solution
y = –2 y = 3x2
x [2(x + x) + 1] [2x + 1] y + y = 3(x + x)2
y + y = 3[ x2 + 2xx + (x)2 ]
dy = Lim y = –2 y + y = 3x2 + 6xx + 3(x)2
dx x 0 x [2(x + 0 ) + 1] [2x + 1] Subtracting
–2 y + y – y = 3x2 + 6xx + 3(x)2 – 3x2
= (2x + 1)(2x + 1) y = 6xx + 3(x)2
–2 Then y = 6xx + 3(x)2
= (2x + 1)2 x x x
( v ) Let y = x – 1 then y = 6x + 3x
y + y = (x + x) – 1 x
Subtracting dy = Lim y = 6x
y + y – y = (x +x) – 1 – x–1 dx x 0 x
y = [ (x + x) -1 – x - 1] [ ( x + x) -1 + x–1 ]
2004/11a Neco (Nov)
( x + x) – 1 + x–1 Differentiate 1 from the first principle
Nothing happened, we just multiplied by 1/1 x3
The top is a surd × its conjugate; Solution
y = ( x + x – 1) – (x – 1) Let y = 1
x3
x + x – 1 + x–1
1
y = x y + y = (x + x)3
x + x – 1 + x–1 = 1
y = x x3 + 3x2x + 3x(x)2 + (x)3
Subtracting
x ( x + x – 1 + x – 1)x 1 1
y = 1 y + y – y = x3 + 3x2 x + 3x(x)2 + (x)3 – x3
x + x – 1 + x–1 y = x3 – x3 – 3x2x – 3x(x)2 – (x)3
[x3 + 3x2x + 3x(x)2 + (x)3 ] [x3]
dy = Lim y = 1
dx x 0 x x –1 + x–1 y = – 3x2x – 3x(x)2 – (x)3
[x3 + 3x2x + 3x(x)2 + (x)3 ] [x3]
= 1
y = – 3x2 – 3x(x)– (x)2
2( x –1 ) x [x + 3x2x + 3x(x)2 + (x)3 ] [x3]
3

( vi ) Let y = xn Then dy = Lim y = – 3x2


y + y = (x + x)n dx x 0 x (x3)(x3)
= x + nxn–1 (x)1 + n(n–1)xn–2 (x)2 + … + (x)n
n
= –3
2! x4
Subtracting 1992 /2 (Nov)
y + y – y = xn + nxn – 1 (x)1 + n(n–1)xn–2 (x)2 + ….+ (x)n – xn
2!
Differentiate 5x – 1 from the first principles.
x2
y = nx (x) + n(n –1)x (x) + …
n–1 1 n–2 2
Solution
2!
Let y = 5x – 1
y = nx + n(n – 1)x x +...+ terms in higher power of x.
n–1 n–2
x2
2!
y + y = 5(x + x) – 1
dy = Lim y = nxn – 1 (x + x)2
dx x 0 x y + y = 5x + 5x – 1
Indeed, for any constant n(positive; negative or x + 2xx +(x)2
2
fractional), if Subtracting
n
dy y = x y + y – y = 5x + 5x –
2
1
2
– 5x + 1
n–1 x +2xx+(x) x2
Then dx = nx
General formula: x here is the independent variable but any
other variable could be used.
y = 5x – 1 + 1
x2 + 2xx + (x)2 x2
475
RHS is purely fractional in nature and must be 3. If u, v, w,… are functions of x the sum
regularized through our basic knowledge in d du dv dw
fractional LCM. ( u + v + w +,… ) = + + +…
dx dx dx dx
= 5x[x2 + 2xx + (x)2 ](x 2) – x2 + x2 + 2xx + (x)2
[x2 + 2xx + (x)2 ][ x 2 ]
4. Product Rule: If u and v are some functions of x, then
= [5x2x + 10x(x)2 +5(x)3](x 2) + 2xx + (x)2 d du dv
[x2 + 2xx + (x)2 ][x 2] ( u v) = V +U
dx dx dx
y = 5x4x + 10x3(x)2 +5x2(x)3 + 2xx + (x)2
[x2 + 2xx + (x)2 ][ x 2 ] 5.Quotient Rule:If u and v are some functions of x, then
du dv
y = 5x4 + 10x3x + 5x2(x)2 + 2x + x V −U
x [x2 + 2xx + (x)2] [x2] d u dx dx
  = 2
dx  v  V
dy = Lim y = 5x4 + 2x
dx x 0 x [x2] [x2] 6. Chain Rule: If y is a function of z and z is a function
= 5x4 + 2x of x, then
x4 dy dy dz
= ×
= 5 + 2 dx dz dx
x3 dx 1
7. If y is a function of x then, =
2015/33 f/m dy dy
Given that y = 4 – 9x and x = 0.1, Calculate y dx
A 9.0 B 0.9 C – 0.3 D – 0.9 General formula method
Solution For any function y = axn ( where a and n are constants )
y = 4 – 9x dy = n×a x n–1
y dx
= –9
x i.e differentiating y with respect to x pronounced dee y-dee x
y = – 9  x
Examples: Differentiate the following with respect to (wrt)
= – 9  0.1
their variable
= – 0.9 (D) 2
(i )y= x ( ii ) y = 3x5 ( iii ) y = 14x2
6 4
( iv ) y = - 12 x ( v ) s = 3t ( vi ) y = 3x
Alternatively
( vii ) y = x ( viii ) y = 4x3
y = 4 – 9x
Solution
By first principle of differentiation
( i ) y = x2
y + y = 4 – 9 (x + x) dy = 2x2 – 1
y + y = 4 – 9x – 9x dx
Subtracting y from both sides = 2x1 i.e 2x
y + y – y = 4 – 9x – 9x – (4 – 9x)
y = – 9x ( ii ) y = 3x5
Substituting for x value dy = 5×3×x5 – 1
y = – 9(0.1) dx
= – 0.9 D. = 15 x4
( iii ) y = 14 x2
2011/2b Neco f/m Adjusted Exercise 30.0 dy = 2×14× x2 – 1
From first principles, differentiate y = x2
dx
= 28x1 i.e 28x

Basic results (rules) in differentiation ( iv ) y = –12 x6


achieved by first principle dy = 6×( – 12 )× x 6 – 1
dx
1. If C is a constant then = – 72 x5
d
(c) = 0 ( v ) s = 3t4
dx ds = 4 × 3 × t4 – 1 and not dy
/dx or dy
/dt or dt/ds
dt
2. If a is a constant then = 12t3
d d
a f(x) = a f(x)
dx dx

476
( vi ) y = 3x 2007/38 PCE
Here the power of x is one i.e x1 If y = x , find dy
 y = 3x1 dx
dy = 1×3 ×x1 – 1
1 2 x 1
dx A B C D
= 3x0 since x0 = 1 2 x x 2 x
= 3×1 i.e 3 Solution
y = x
1
y= x  2

( vii ) y = x
Here the power of x is one i.e x1 dy 1
= 1 x 2 −1 = 1 x − 2 i.e A.
1 1

 y = x1 dx 2 2 2 x
dy = 1x1-1
dx 2001/38 PCE Exercise 30.1
= 1x0 since x0 = 1 − 32
= 1×1 i.e 1 If y = x then dy/dx is
A. – /3x
2 ½
B. – 3/2x – ½ C.- 2/3x – 5/2 D. – 3/2x –5/2
3
( viii ) y = 4x
dy = 12x2
dx
The Derivative of the sum of two ( or more ) simple
Examples i to vii involves explanation with solution. But
functions is the sum of the separate derivatives of the
viii have solution only which is the acceptable standard
functions: What it implies is shown below
we shall be following except when the author wants to
explain further. Eg.1 If y = 4x3 – 12x2 + 3x + 12, find dy/dx.
Solution
dy = 12x2 – 24x + 3
Further examples dx
Differentiate the following with respect to their
variables. 2012/37 UTME
1
( 1 ) y = 1 ( 2 ) t = 3 ( 3 ) if y = 10 find dy/dx If y = x2 – x , find
dy
x2 s6 dx
( 4 ) if y = 13x, find dy/dt 1 1
A 2x – x 2 B 2x + x2 C 2x – x2 D 2x + x 2
Solution
(1) y= 1 Solution
x2 1
 y = x–2 y = x2 – x
dy = - 2x – 2 – 1 y = x2 – x – 1
dx dy
= - 2x – 3 =-2 = 2x – (–1)x – 1 – 1
dx
x3 = 2x + x – 2
1
(2) t= 3  t = 3s – 6 = 2x +
x2 D.
s6
dt = - 6×3×s – 6 – 1 2014/32 Neco
ds dy
If y = 3x3 + x2 + 6, find at x = – 2
= - 18 s – 7 dx
= - 18 A 40 B 32 C 0 D 32 E 40
s7 Solution
( 3 ) y = 10  y = 10x0 y = 3x3 + x2 + 6
( the variable x was not shown because it has a power of zero ) dy
dy = 0 ×10 x 0 – 1 = 9x2 + 2x
dx
dx At x = – 2
= 0 ( since zero times any number is zero ) dy
= 9(– 2)2 + 2(– 2)
dx
Generally any constant differential is zero
=94 – 4
( 4 ) y = 13x, but we are asked to find /dt dy = 36 – 4 = 32 D.
dy = 0 since 13x is a constant with respect to t 2005/6 PCE
Find the derivative of f(x) = 2x(x2 + 3)
dt
A. 4x2 + 3 B. 2(x2 + 3) C. 6(x2 + 1) D. 6(x2 + 2)
Solution
We can easily open up the bracket as :
477
f(x) = 2x(x2 + 3) becomes Special derivatives’ result
f(x) = 2x3 + 6x
/
Functions Derivatives’result
f ( x ) = 6x2 + 6 i.e 6(x2 + 1) C lnx or logex 1
/x
x
2014/38 UTME Exercise 30.2 e ex
dy Sin x cos x
If y = 4x3 – 2x2 + x, find Cos x - sin x
dx
A 12x2 – 2x + 1 B 12x2 – 4x + 1 Tan x sec 2 x
C 8x2 – 2x + 1 D 8x2 – 4x + 1 Cot x - cosec2 x
2007/15 UME Exercise 30.3 Sec x sec x tan x
dy Cosec x - cosec x cot x
If y = ( 1 + x )2 , find dx
A. 2x – 1 B. x – 1 C. 2 + 2x D. 1 + 2x 1995/38 UME
1994/44 PCE Exercise 30.4 The derivative of cosec x is
Differentiate y = 3x2 + 2x + 3 with respect to x. A tan x cosec x B. – cot x cosec x C. tan x secx
A.6x + 2 B.6x2 – 2 C.6x + 3 D.6x2 - 3 D. – cot x sec x
Solution
1995/39 PCE Exercise 30.5
Derivative of cosec x = – cot x cosec x ( B )
Differentiate 3x2 + 2/x with respect to x
A. 6x + 2 B. 6x – 2 C. 6x + 22 D. 6x – 22 So – the above derivatives should be part and parcel of us
x x x
2014/14 Neco Exercise 30.6
Find the derivative of tan x
Second derivative A cos2x B cosec x C sec2x
Differentiation can be repeated as many times as D secant x E sin x
required on a given function. Under this subheading,
we shall only discuss 2nd differentiation. 2012/38 UTME Exercise 30.7
Eg. 1 If y = 3x3 – 2x2 + 4 find d2y/dx2 Find
dy
if y = cos x
Solution dx
y = 3x3 – 2x2 + 4 is differentiated wrt x, i.e A sin x B – sin x C tan x D – tan x
dy = 9x2 – 4x
dx
d2y/dx2  differentiating the result, i.e Differentiation rules
2
d y = 18x – 4 We have been discussing the general formula so far, there
are certain rules to follow when we have
dx 2
‘complex ‘ or complicated functions to differentiate.
d2y/dx2 is read as ( dee squared y dee x squared)
Note: d2y/dx2 is different from ( dy/dx)2 Function of a function ( chain rule )
as the first is 2nd derivative while the latter is the square of
the result of dy/dx
If we have a case like:
Given that y = ( x + 1 )2 find dy/dx
Sometimes a function can be written as For us to apply the formula
f( x ) = 2x2 + 3x + 1 We must first expand i.e
instead of y = 2x2 + 3x + 1 y = ( x + 1 )2 will become y = ( x + 1)( x + 1)
then we have that its own form of dy/dx will be f / ( x ) . = x2 + 2x + 1
Hence f / ( x ) = 4x + 3 Then we differentiate y = x2 + 2x + 1
dy = 2x + 2
Also d2y/dx2 will be f // ( x ) = 4
dx
we can afford to expand it. But if we have
y = ( x + 1 )10 find dy/dx
I bet you – you will exhaust the whole of your note to expand it.
The chain rule comes into play here.
Starting
( 1 ) Given that y = ( x + 1 )2, find dy/dx
Solution
Let z = x + 1 – the core or bracket content
then our original function will be
y = z2
Then, dy = dy  dz
dx dz dx
Please note that z is a dummy variable
It can be changed to u or v etc depending on the author or teacher
478
Let’s go gradually, = 2(cos – sin)(–sin – cos)
z = x + 1 then dz = 1 = 2[cos(–sin – cos) – sin(–sin – cos)]
dx = 2[– cossin – cos2 + sin2 + cossin]
Also, y = z2, then dy = 2z = 2[sin2 – cos2]
dz = 2[(1 – cos2) – cos2]
Substituting, = 2[1 – 2cos2] from trig identity we have
dy dy dz will become = – 2(2cos2)
= 
dx dz dx = – 2 cos2 C.
= 2z × 1
= 2z 2014/14 Neco f/m
Substituting for z value d y 2

= 2(x + 1 ) Find , if y = e 5x + 3
dx 2
if we expand. = 2x + 2 as gotten earlier. A 5e5x + 3 B 25e5x + 3 C 5ln(5x + 3)
D 25ln(5x + 3) E 25[ ln(5x + 3) + e5x + 3]
2010/40 UTME Solution
dy
If y = (2x + 1)3, find y = e5x + 3
dx dz
A 3(2x + 1)2 B 3(2x + 1) C 6(2x + 1) D 6(2x + 1)2 Let z = 5x + 3; = 5
dx
Solution
dy
y = (2x + 1)3 y = ez ,
= ez
dy dz
Let z = 2x + 1 then =2 dy dy dz
dz = 
dy dx dz dx
y = z3 then = 3z2
dz = ez  5
dy dy dz = 5e5x + 3
Thus = 
dx dz dx d 2 y dy dz
= 
= 3z2  2 dx 2
dz dx
= 6(2x + 1)2 D. dz
Let z = 5x + 3, =5
2009/36 UME dx
dy dy
If y = 3cos4x, equals y = 5ez, = 5ez
dx dz
A 6sin8x B – 24sin4x C 12sin4x D – 12sin4x d2y
Solution = 5ez  5 = 25e5x + 3 B.
y = 3cos4x dx 2
dz 2014/13 f/m
Let z = 4x, =4
dx Find the derivative of 3
(3x 3 + 1) with respect to x
dy
y = 3cos z and = – 3sinz 3x 3x 2
dz A B
dy dy dz
3
(3x + 1) 2 3
(3x 3 + 1) 2
Thus = 
dx dz dx 3x 3x 2
C D
= – 3sin z  4 3
(3x 2 + 1) 3
(3x 2 + 1) 2
= – 12 sin z Solution
= – 12 sin 4x (D) 1
3 3
y = (3x + 1)
2006/45 UME dz
Differentiate (cos – sin)2 with respect to  Let Z = 3x3 + 1 then = 9x2
dx
A 1 – 2cos B – 2sin C – 2cos2 1 1 2
dy 1 −1 1 −
D 1 – 2sin 2 y = Z then 3
= Z3 = Z 3
Solution dz 3 3
y = (cos – sin)2 dy dy dz
dz = 
Let Z = cos – sin, = – sin – cos dx dz dx
d 2
1 −3
y = z2,
dy
= 2z = Z  9x 2
dz 3
dz dy dz 9x2 3x 2
=  = = B.
d dz d
2
3Z 3
3
(3x 3 + 1) 2
= 2z (–sin – cos)
479
2006/47 UME C – 3cos   − 3x  D – sin   − 3x  E – 3sin   − 3x 
2 2  2  2 
 1
Differentiate  x 2 −  with respect to x
 x
2005/6 UME Exercise 30.9
2 2
A 4x – 2 + 3
3
B 4x3 – 2 – 3 Find the derivative of y = sin(2x3 + 3x – 4)
x x
A – cos (2x3 + 3x – 4) B – (6x2 + 3) cos(2x2 + 3x – 4)
2 2
C 4x3 – 3x + D 4x3 – 3x – C cos (2x3 + 3x – 4) D (6x2 + 3) cos(2x3 + 3x – 4)
x x
Solution 2013/35 UTME Exercise 30.10
2
 1 dy
y =  x2 −  If y = (2x + 2)3, find
 x dx
1 dz 1 A 3(2x + 2) B 6(2x + 2)2 C 3(2x + 2)2 D 6(2x + 2)
Let z = x2 – , = 2x + 2014/39 UTME Exercise 30.11
x dx x2 dy
dy If y = cos3x, find
y = z2, = 2z dx
dz
A 3sin3x B – 3sin3x C 1/3sin3x D – 1/3sin3x
dy dy dz
Thus =  2007/15 UME Exercise 30.12
dx dz dx dy
If y = (1 + x)2, find
= 2z   2 x + 12  dx
 x  A 2x – 1 Bx–1 C 2 + 2x D 1 + 2x
 1  1 
= 2  x 2 −  2 x + 2  2013/22 Neco f/m Exercise 30.13
 x  x  1

2 d2y
  2 1  1  2 1  Given that y = (2x + 3) , find
= 2 2 x x −  + 2  x −  dx 2
  x x  x 
1 1
−1 −2
= 2[ 2x – 2 + 1 – 3 ] 1
3
A – (2x + 3) 2
B 3(2x + 3) 2
x 1 1 1
1 −1 3 −2
 3 1  C − (2x + 3) 2
D (2x + 3) 2 E 2(2x + 3) 2
= 2  2x −1 − 3  2 4
 x 
2 2016/10a Neco Exercise 30.14
= 4x – 2 – 3
3
B.
x Differentiate y = (2x2 + 3)5 with respect to x.
2009/39 PCE 2016/60 Neco Exercise 30.15
dy dy
If y = sin2x – cos3x, find If y = (2x2 + 7)4, find
dx dx
A cos2x + sin3x B cos2x – sin3x A. 4x B. 4(2x2 + 7)3 C. 4x(2x2 + 7)3
C 2cos2x – 3sin3x D 2cos2x + 3sin3x D. 16(2x2 + 7)3 E. 16x(2x2 + 7)3
Solution
y = sin 2x – cos 3x
dy d (sin 2 x) d (cos 3x)
= – Product Rule
dx dx dx
If y = UV where u and v are some functions of x. then
d (sin 2 x) d (cos 3x) dy dv du
as y = sin z as y = cos z =U +V
dx dx dx dx dx
dz dz dy
z = 2x and =2 z = 3x and =3 Simply as = Udv + Vdu
dx dx
dx
dy dy dz dy dy dz
=  =  Example 1
dx dz dx dx dz dx If y = (3x6 – 10)(x4 + 30) , find dy/dx
= cos z  2 = – sin z  3 Solution
= 2cos 2x = – 3sin3x Let y = uv
Substituting i.e y = (3x6 – 10)(x4 + 30)
dy dy
Thus = 2cos2x – (–3sin3x) Thus, = Udv + Vdu
dx dx
= 2cos2x + 3sin3x D. = ( 3x6 – 10 ) × 4x3 + ( x4 + 30 ) × 18x5
2010/17 Neco f/m Exercise 30.8 = 12x9 – 40x3 + 18x9 + 540x5
  = 30x9 + 540x5 – 40x3
Differentiate cos  − 3x  with respect to x
2 
2000/6 Neco f/m
A3 cos   − 3x  B 3sin   − 3x  Find the derivative of y = (x + 1)(x + 2)
2  2  A. 2x – 3 B. 2x + 3 C. 2x + 4 D. 3x – 2 E. 3x + 2
480
Solution d2y d d
y = (x + 1) (x + 2) Thus, 2
= ( x cos x) + (sin x)
dx dx dx
Then y = u v
dy
/dx = udv + vdu Let y = x cos x
= (x + 1) × 1 + (x + 2) × 1 u v
=x+1+x+2 d/dx ( x cos x ) dy/dx = udv + vdu
= 2x + 3 (B) = x ×( - sin x ) + cos x × 1
= - x sin x + cos x
2005/37 Neco fm
d2y
If y = x sin 2x, find dy/dx Thus, = - x sin x + cos x + cos x
A. –2cos2x + sin2x B. –2x sin cosx + 2 sin2x dx 2
C. – 2x cos2x + sin2x D. sin2x – xcos2x = - x sin x + 2 cos x
E. sin2x + 2x cos2x = 2 cos x – x sin x ( A )

Solution 2004/37 PCE


If y = (x2 + 2x + 4)(x – 2), find dy/dx
y = x sin 2x
dy A. – 3x2 B.3x2 C.3x4 D.2x4
/dx = Udv + Vdu Solution
= x d(sin 2x) + sin2x × d(x) Applying product rule
Recall that applying chain rule d (sin 2x) = 2cos2x y = ( x2 + 2x + 4 ) ( x – 2 )
dx u v
dy
Thus, /dx = x × 2cos2x + sin 2x × 1 dy
= 2x cos2x + sin2x (E) = Udv + Vdu
Addition is commutative i.e 2 + 3 = 3 + 2 so,
dx
2xcos2x + sin2x is same as sin2x + 2xcos2x = (x2 + 2x + 4 ) × 1 + (x – 2 )( 2x + 2 )
= ( x2 + 2x + 4 ) + ( 2x2 + 2x – 4x – 4 )
2001/38 UME = x2 + 2x + 4 + 2x2 – 2x – 4
If y = x sin x, find dy
/dx when x =  Collect like terms together
2 = x2 + 2x2 + 2x – 2x + 4 – 4
A. -  B. – 1 C. 1 D.  = 3x2 ( B )
2 2 2004/15
Solution Find the derivative of the function y = 2x2( 2x – 1 )
Let y = uv at the point x = - 1
i.e y = x sin x A.18 B.16 C.- 4 D.- 6
dy Solution
Thus, = Udv + Vdu
dx Applying product rule
= x cos x + sin x × 1 y = 2x2( 2x – 1 )
= x cos x + sin x u v
When x =  means 900 dy
= Udv + Vdu
2 dx
=  cos 900 + sin 900 = 2x2 × 2 + ( 2x – 1 ) × 4x
2 = 4x2 + 8x2 – 4x
=  ×0 +1 = 12x2 - 4x
2 At point x = - 1
= 1 ( C) 12x2 – 4x = 12 (-1)2 – 4 (-1)
= 12 + 4 i.e 16 ( B )
1992/37 UME
If y = x sin x, find d2y/dx2 2007/12UME Exercise 30.16
A. 2cos x – x sin x B. sin x + x cos x If y = x cos x, find dy/dx
A.sinx - xcosx B. sinx + xcosx C. cosx + xsinx D. cosx – xsinx
C. sin x – x cos x D. x sin x – 2 cos x
Solution
2004/13 UME Exercise 30.17
The function is in form uv,
Find the derivative of (2 + 3x )(1 – x) with respect to x.
we find dy/dx first before d2y/dx2
A.6 B.- 3 C.1 – 6x D.6x – 1
y = x sin x
u v
2005/6 PCE Exercise 30.18
dy
= Udv + Vdu Find the derivative of f( x ) = 2x( x2 + 3 )
dx A.4x2 + 3 B.2(x2 + 3) C.6(x2 + 1) D. 6x2 + 2
= x cos x + sin x ×1
= x cos x + sin x
Then, d2y/dx2 will be differentiating the result.
Where x cos x is another product rule case.
481
Cases involving product and chain rules Solution
2015/11a Neco y = x sin x
Differentiate y = x2(2x + 1)2 with respect to x u v
dy
Solution = udv + vdu
y = x2 (2x + 1)2 dx
u v = x cos x + sin x  1
dy = x cos x + sin x B.
= udv + vdu
dx
= x2dv + (2x + 1)2  2x 2001/35 UME Exercise 30.19
Here dv is a function of function problem Differentiate ( 2x + 5 )2 ( x – 4 ) with respect to x
Let y = (2x +1)2 A. 4( 2x + 5 ) ( x – 4 ) B. 4( 2x + 5 ) ( 4x – 3 )
dz C. ( 2x + 5 ) ( 2x – 13 ) D. ( 2x + 5 ) ( 6x – 11 )
z = (2x + 1) then =2
dx
dy
y = z2 then = 2z
dz
dy dy dz
Thus dv  =  Quotient Rule
dx dz dx u
= 2z  2 If y = , where u and v are some functions of x, then
= 4z i.e 4(2x + 1) v
Substituting for dv dy vdu − udv
=
dy
= x2  4(2x + 1) + (2x + 1)2  2x dx v2
dx It has to do with functions in fraction quoted as:
= 4x2(2x + 1) + 2x(2x + 1)2 Bottom dee top minus top dee bottom , all over bottom square
= 2x(2x + 1)(2x + 2x + 1)
= 2x(2x + 1)(4x + 1) 2005/11a Neco f/m
2009/37 UME Differentiate with respect to x, the function
ds y = 4x3 + 8
If S = (2 + 3t)(5t – 4), find when t = 4/5 sec. 2x2 + 3
dt
A 0 units per sec B 15 units per sec
Solution
C 22 units per sec D 26 units per sec
Solution y = u(x) i.e 4x3 + 8
S = (2 + 3t) (5t – 4) v(x) 2x2 + 3
u v dy = vdu – udv
ds dx v2
= udv + vdu
dt = (2x + 3) × 12x2 – (4x3 + 8) 4x
2

= (2 + 3t)  5 + (5t – 4)  3 (2x2 + 3)2


= 10 + 15t + 15t – 12 = 24x4 + 36x2 – 16x4 – 32x
= 30t – 2
(2x2 + 3)2
When t = 4/5 secs
= 8x4 + 36x2 – 32x
ds
dt
= 30 ( 54 ) –2 (2x2 + 3)2
= 24 – 2
= 22 units per sec ( C ) 2004/18 Neco f/m
Differentiate the function 4x4 + x3 – 5 with respect to x.
2014/90 f/m
4x2
Differentiate (x – 3)(x2 + 5) with respect to x
A. 2x + 5 + 1 B. x + x – 5
2
C. 2x2 + 5 + 1
Solution
y = (x – 3) (x2 + 5) 2x3 4 4 4x 2x 4x
u v D. 2x – 5 + 1 E. x2 – x – 5
dy 2x3 2 4 4x
= udv + vdu Solution
dx
= (x – 3)  2x + (x2 + 5)  1 y = u(x) i.e 4x4 + x3 – 5
= 2x(x – 3) + (x2 + 5) v(x) 4x2
= 2x2 – 6x + x2 + 5 dy = vdu – udv
= 3x2 – 6x + 5 dx v2
2013/36 UTME = 4x (16x3 + 3x2) – (4x4 + x3 – 5) × 8x
2

dy (4x2) 2
If y = x sinx, find
dx = 64x5 + 12x4 – 32x5 – 8x4 + 40x
A cos x + x sin x B sin x + x cos x 16x4
C sin x – cos x D cos x – x sin x
482
= 32x5 + 4x4 + 40x Quotient and chain rule
16x4 16x4 16x4
2003/5 Neco
= 2x + 1 + 5 (A) Find the derivative of y with respect to x if y = 1+x
4 2x3 1–x
2008/39 PCE Solution
cos x Applying chain rule
If y = + 1 , find dy
sin x dx y = 1+x is same as y = (1 + x)
1
2

A tan x B cot x C – tan2x D – cosec x 2


1

Solution 1–x (1 – x) 2

1
cos x Let U = 1 + x then y = U 2

y = + 1
sin x 1–x
By simple fractional operation dy = dy × du
cos x + sin x u dx du dx
y= 1
sin x v 1 −2 du
= u 
dy vdu − udv 2 dx
=
dx v2 Next we find
du
by applying Quotient rule
sin x (− sin x + cos x) − (cos x + sin x) cos x dx
= du = (1– x) × 1 – (1 + x) × ( –1)
(sin x) 2
dx (1– x)2
− sin 2 x + cos x sin x − cos 2 x − cos x sin x) = 1– x+1+x
=
sin 2 x (1– x)2
− sin 2 x − cos 2 x = 2
= (1– x)2
sin 2 x
1
− (sin 2 x + cos 2 x) 1 1+ x 
−2
= 
= dy 2
2 Thus,  ×
sin x dx 2 1− x  (1 − x) 2
By trig identities 1
1 1− x  2
= 
1 2
= − = – cosec2x D.  ×
sin 2 x 2 1+ x  (1 − x) 2
1
2005/11(ii) Exercise 30.20
Differentiate with respect to x: (1 − x) 2 1
= 
(1 − x) 2
1
y = 2x – 3 (1 + x) 2

x2 + 5 1
= 1 3
2003/11b (Nov) Exercise 30.21 (1 + x) (1 − x) 2 2

Find the derivative with respect to x of y = 2x


x2 + 3 2000/28 Neco
1996/12 (Nov) Exercise 30.22 Differentiate with respect to x, cos x
If y = 1 + x, find dy sin 2x
1–x dx A.– sin 3x B. – sin2 2x
A. 1 B. 2 C. 1 + x D. x sin2 2x sin 3x sin2 2x
(1 – x)2 (1 – x)2 (1 – x)2 (1 – x)2 C. – sin 2x sin x + 2 cos x cos 2x
E. –2 sin2 2x
(1 – x)2 D. sin 2x cos x + cos 2x sinx
2010/19 Neco Exercise 30.23 sin2 2x sin3x sin2 2x
Differentiate 4x3 + 3x2 – 5 with respect to x E. – [sin 2x sin x + 2 cos x cos2x ]
5x2 sin22x
A 2 + 4 B 2 + 4x C 12x3 + 6x
3 3
x 5 x 5 10x Solution
D 12x3 + 6x E 4x – 1 + 3 Let y = cos x
25 x4 5 x2 5 sin 2x
2012/22 f/m Exercise 30.24 Applying quotient rule
 x  dy = sin 2x × (- sin x) – cos x × (2cos2x)
Differentiate   with respect to x
 x +1  dx (sin2x)2
1 1 1− x 1 = - sin 2x sin x – 2cosx cos2x (E)
A B − C D
x +1 ( x + 1) 2
( x + 1) ( x + 1) 2 sin22x

483
1994/10 b (Nov) Exercise 30.25 Solution
Differentiate, with respect to x y = x2 – x + 4
y = 3 – 2x , simplifying your answer
dy
(1 + x)2 = 2x – 1 same as gradient
dx
1993/15 (Nov) Exercise 30.26 But we are told that gradient is 3
x dy Thus, dy/dx = 3
If y = find
( x −1) 2 2x – 1 = 3
2 1
dx
2x = 3 + 1
A. 2 B. – 1 C. 1 D. –2x2 + 1 2x = 4
(x2 – 1)3 (x2 – 1)3/2 (x2 – 1)3 (x2 – 1)3/2 x=2
E. 2x – 1
2
To get the y co – ordinates, substitute x = 2 into the
(x2 – 1)3/2 original equation.
y = (2)2 – 2 + 4
2001/10 Neco Exercise 30.27 =4–2+4
sin 3x =6
The derivative of with respect to x is
e −2 x Co – ordinate of the point (2, 6)
3 cos 3x + 2 sin 3x 3 sin 3x + 2 cos 3x
A. B.
e −2 x e −2 x 1995/2 (Nov)
3 cos 3x + 2 sin 3x 3 cos 3x − 2 sin 3x Find the differential coefficient of the function
C. D. y = 1 at the point (4, 1/25)
e −4 x e −2 x
3 cos 3x − 2 sin 3x x2 + 9
E. A. -8/625 B. -4/625 C. 2/625 D.4/625 E. 8/625
e −4 x
2010/3 Neco Exercise 30.28 Solution
dy x3 Differential coefficient is same as dy/dx. Thus
Find if y = y = 1 : i.e y = (x2 + 9)–1
dx 2x 2 + 3 2
x +9
Let z = x2 + 9 then y = z – 1
Application of Differentiation
Gradient ( slope at a given point). dy = dy × dz
The concept of differentiation is the same as that of dx dz dx
gradient or slope. So when we say the gradient or slope = – 1z–2 × 2x i.e –2x
of a function at a point we are only referring to the z2
differentiation of that function and substituting for the x = –2x
value given or among the points given. Other concepts (x2 + 9)2
required in this subtopic will be shown as we go At point (4, 1/25), we put x = 4 into our dy/dx result
2004/15 = –8 i.e – 8 (A)
The gradient of the curve y = 3x2 + 11x + 7 at the point (25)2 625
P(x, y) is – 1. find the co – ordinates of P.
A. ( -2, +3) B. (-2, -3) C. (-1, -5/2) D. (-3, -2) 1995/40 UME
Find the gradient of the curve y = 2 x – 1 at the point x = 1
Solution x
y = 3x2 + 11x + 7, A.0 B.1 C.2 D.3
dy
= 6x + 11 i.e gradient Solution
dx
But we are told that gradient is –1 y = 2 x – 1  y = 2x ½ – x – 1
Thus, dy/dx = –1 x
6x + 11 = –1 dy
= x– ½ + x–2
6x = – 12 dx
x = –2 = 1 + 1 which is the gradient
To get y co-ordinates x x2
Substitute x = - 2 into the original equation At point x = 1
y = 3(-2)2 + 11(-2) + 7 = 1 + 1 = 1+ 1 =2(C)
= 12 – 22 + 7 1 12 1 1
= –3
Co-ordinates of P is (-2, -3) (B) 2002/17 UME
The slope of the tangent to the curve y = 3x2 – 2x + 5 at the
2002/15 point ( 1, 6 ) is
Find co-ordinates of the point at which the gradient A.6 B.5 C .4 D.1
of the curve y = x2 – x + 4 is 3 Solution
A. (1, 4) B. (2, 6) C. (1, 6) D. (3, 5) y = 3x2 – 2x + 5
484
dy 2009/11 Exercise 30.29
= 6x - 2 i.e slope The gradient of point P on the curve y = 3x2 – x + 3 is 5.
dx Find the coordinates of P.
At point ( 1, 6 ) ; x = 1
A. (1, 5) B (1, 7) C (1, 13) D (1, 17)
put x = 1 into 6x – 2
6( 1 ) – 2 = 4 ( C ) 2006/26 Exercise 30.30
Find the coordinates of the point on the curve
2003/39 UME y = x2 + 4x – 2, where the gradient is zero.
Find the slope of the curve y = 2x2 + 5x – 3 at ( 1, 4 ) . A (-2, 10) B (-2, 2) C (-2, -2) D (-2, -6)
A. 4 B.6 C.7 D.9 1994/15 Exercise 30.31
Solution Find the differential coefficient of y = – 5 at the point (1, -1).
y = 2x2 + 5x – 3 x2 + 4
dy A. -2 B. -1 C. -2/5 D. 2/5 E. 2
= 4x + 5 i.e slope
dx 2009/30 (Nov) Exercise 30.32
At point ( 1, 4 ); x = 1 The equation of a curve is given by y = 2
Put x = 1 into 4x + 5 x+3
4( 1 ) + 5 = 9 ( D ) Find its gradient at the point  1 .
 1, 
 2
1995/42 PCE
A- 1 B-1 C 1 D 1
Find the points on the curve y = x3 + 6x2 – 15x + 1 at 4 8 8 4
which the gradient is 0. 2004/38 PCE Exercise 30.33
A.( - 5, 101 ), ( 1, - 7 ) B.( - 5, -101 ), ( 1, - 7 ) The gradient of the curve y = 3x2 – 5x + 6 when x = 1 is
C. ( 5, - 101 ) , ( - 1, - 7 ) D.( 5, 101 ) , ( - 1, 7 ) A.1 B.4 C.6 D.7
Solution
y = x3 + 6x2 – 15x + 1 2001/37 PCE Exercise 30.34
dy The gradient of the curve y = x2 + 4x at the point ( 2, 10 )
= 3x2 + 12x – 15 A.0 B.8 C.12 D.24
dx
Gradient is 0  dy/dx = 0 i.e
3x2 + 12x – 15 = 0 divide through by 3 Turning Points
x2 + 4x – 5 = 0 factorising ( maximum, & minimum )
x + 5x – x – 5 = 0
2
At turning points P and Q shown below;
x(x+5)–1(x+5)=0
y

.
[

(x+5)(x–1 )=0 dy
P
x + 5 = 0 or x – 1 = 0 dx =0
x = – 5 or 1

.
Use x values to find y
when x = 1
y = (1)3 + 6(1)2 – 15(1) + 1 Q
= –7 x
( 1, - 7 )
when x = - 5 y = ( - 5 )3 + 6 ( - 5 )2 - 15( - 5 )+ 1 But it is the second differential ( d2y/dx2 ) that reveal
= - 125 + 150 + 75 + 1 whether the turning points leads to a maximum or minimum.
y

.
2
dy
= 101 dx2 0
( - 5 , 101 ) ( A ) Maximum

P
2005/3 PCE
At what point is the gradient of the curve x2 – 6x + 3
equal to zero ?
.
Q
Minimum
2
dy
A. – 2 B. 3 C. – 3 D. 2
dx2 0
Solution x
f( x ) = x2 – 6x + 3
Gradient of curve implies The value of x for maximum or minimum point
f / ( x ) = 2x - 6 In this case the question already specifies the turning point to
Gradient = 0 implies 2x – 6 = 0 solving be a minimum or maximum point. Note the use of the
2x = 6
phrase ‘value of x’ in the subsequent questions
x = 6/2 Thus, x = 3 ( B )
2009/38 UME
What value of x will make the function x(4 – x) a minimum?
A4 B3 C2 D1
485
Solution 1992/19 (Nov) f/m
dy The maximum point of the curve y = x – 2sin x,
At minimum =0
dx 0  x < 2 occurs when x equals
y = x(4 – x) A. /6 B. /3 C. 5/3 D. 2/3 E. 4/3
y = 4x – x2 Solution
dy
= 4 – 2x At maximum or minimum dy/dx = 0
dx
Thus, y = x – 2sin x will become
 4 – 2x = 0 dy
4 = 2x /dx = 1 – 2 cos x
4
/2 = 2x/2 Hence 1 – 2 cos x = 0
2=x (C) 2 cos x = 1
We stopped here as specified by the question “value of x” cos x = 1/2
cos x = 0.5
2010/42 UMTE x = cos–1 0.5
At what value of x does the function y = – 3 – 2x + x2 = 600
attain a minimum value The rage is from 00 to 2 i.e 360
A1 B–4 C–1 D4 Next we find other multiples of 600 that will give 0.5
Solution 1200, cos 120 = - 0.5 not accepted
dy 1800 cos 1800 = -1 not accepted
At minimum =0
dx 2400 cos 2400 = - 0.5, not accepted
y = – 3 – 2x + x2 300 cos3000 = 0.5 accepted
dy Thus, x = 600 or 3000
= – 2 + 2x
2
dx But for maximum d y < 0
 – 2 + 2x = 0 dx2
2x = 2 2
which is d y = 2 sin x < 0
x=1(A) dx2
We stopped here as specified by the question “value of x” For 2sin x < 0, x = 3000 and not 600
Next convert 3000 to radians
1998/36 UME i.e 300 ×  = 5/3  (C)
For what value of x does 6sin ( 2x – 25 )0 attain its 180
maximum value in the range 00  x  1800 ? 2010/38 Exercise 30.35
A. 12 ½ B. 32 ½ C. 57 ½ D. 14 ½ Find the maximum value of 2 + sin ( + 250)
Solution A1 B2 C3 D4
We let y = 6 sin ( 2x – 25 )0
At min or max; dy/dx = 0 2003/46 PCE Exercise 30.36
Applying function of function rule If y = 3x2 – 2x + 1 , find the value of x when dy/dx = 0
dy/dx = 12 cos ( 2x – 25 )0 A. 1/3 B.1/2 C.1 D.1/4
It follows that
12 cos ( 2x – 25 ) = 0 Cubic function ( x3 ) case
which means cos ( 2x – 25 ) = 0
it implies 2x– 25 = cos –1 0 2012/15 f/m
2x– 25 = 90 If y = x3 – x 2 – x + 6, find the value of x
2x = 25 + 90 at the turning points
x = 115 i.e 57 ½ ( C ) 1 1 1 1 1
A ,–3 B , − C 1, − D 1,
2 2 3 2 3 3
1996/38 PCE Solution
A swimming pool is treated periodically to control At turning point
dy
=0
harmful bacterial growth. The concentration of bacteria dx
per cm3 after t days is given by y = x3 – x2 – x + 6
C( t ) = 30t2 – 240t + 500, In how many days after a dy
= 3x2 – 2x – 1
treatment will the concentration be minimal ? dx
A.10 B.8 C.6 D.4 At turning point: 3x2 – 2x – 1 = 0
Solution Factorizing
At minimal, dc/dt = 0 3x2 – 3x + x – 1 = 0
In this case, dc/dt = 60t – 240 3x(x – 1) + 1(x – 1) = 0
It follows that 60t – 240 = 0 (3x + 1)(x – 1) = 0
60t = 240 3x + 1 = 0 or x – 1 = 0
60t = 240 3x = – 1 or x – 1 = 0
60 60 1
x= − or 1 C.
t = 4 days ( D ) 3

486
2009/40 PCE 2006/50 UME
Find the value of x at which the function Find the value of x for which the function 3x3 – 9x2 is
y = x3 – 2x2 + x + 4 is minimum minimum
A 1/ 3 B1 C3 D4 A2 B0 C5 D3
Solution Solution
dy dy
At minimum =0 At minimum =0
dx dx
y = x3 – 2x2 + x + 4 y = 3x3 – 9x2
dy dy
= 3x2 – 4x + 1 = 9x2 – 18x
dx dx
3x2 – 4x + 1 = 0  9x2 – 18x = 0
Factorizing Factorizing
3x2 – 3x – x + 1 = 0 9x(x – 2) = 0
3x(x – 1) – 1(x – 1) = 0 9x = 0 or x – 2 = 0
(3x – 1)(x – 1) = 0 x = 0 or x = 2
x = 1/3 or 1 d2y
To know the minimum point But at minimum 0
2
dx 2
d y
= 6x – 4 d2y
dx 2 = 18x – 18
dx 2
The minimum point is at
For 18x – 18 > 0, we check
d2y When x = 0
>0
dx 2 18(0) – 18 < 0
6(1) – 4 > 0 ( B) When x = 2
18(2) – 18 > 0
Thus min value is 2 A
2014/8 Neco Adjusted
At what stationary point of x is the function
y = x3 – 2x2 + x maximum
The value of y for maximum or minimum point
Solution Here we are required to go a step further from finding the
dy value of x for max or min points; by substituting the value of
At stationary points = 0 x into the original equation. Though we have two types of
dx
y = x3 – 2x2 + x cases here:
dy Quadratic functions ( x2 )
= 3x2 – 4x + 1 Cubic functions ( x3 )
dx
 3x2 – 4x + 1 = 0
Factorizing Quadratic function case
3x2 – 3x – x + 1 = 0 2005/5 UME
3x(x – 1) – 1(x – 1) = 0 The maximum value of the function f(x) = 2 + x – x2 is
(3x – 1) (x – 1) = 0 3 1 9 7
3x – 1= 0 or x – 1 = 0 A B C D
2 2 4 4
3x = 1 or x – 1 = 0 Solution
x = 1/3 or 1 At maximum f /(x) = 0
d2y f(x) = 2 + x – x 2
At maximum <0
dx 2 f /(x) = 1 – 2x
d2y  1 – 2x = 0
= 6x – 4 1 = 2x
dx 2 1
When x = 1/3 /2 = x
6 (1/3) – 4 < 0 To get maximum value, we substitute
1
Thus /3 is maximum x = 1/2 into f(x) = 2 + x – x2
2
1
= 2 + 1/ 2 –  
2
= 2 + 1/ 2 – 1/ 4
8 + 2 −1
=
4
9
= C.
4

487
2015/12 f/m d
Find the minimum value of y = x2 + 6x – 12 ( 30 x − x 2 ) = 0
dx
A – 21 B – 12 C–6 D–3
30 – 2x = 0
Solution
30 = 2x
dy
At minimum value =0 30 = 2x
dx 2 2
y = x2 – 6x – 12 15 = x
dy
= 2x + 6 To get maximum area value
dx Substitute x = 15 into 30x – x2
 2x + 6 = 0 Maximum area = 30( 15 ) – ( 15 )2
2x = – 6 = 450 – 225
x = – 6/2 = – 3 = 225m2 (C)
To get minimum value: we substitute x = – 3 into
y = x2 – 6x – 12 2009/6 (Nov) Exercise 30.37
y = (– 3)2 + 6 (– 3) – 12 Find the minimum value of the function
= 9 – 18 – 12 = – 21 A. y = 2x2 – 6x + 3.
2003/4 f/m A –1.5 B –1.0 C 2.5 D 3.0
A blacksmith produced x articles at a total cost of 2006/27 f/m Exercise 30.38
$ (200 – 48x + 3x2). If each article is sold at $ 3/5x, Find the least value of the function f(x) = 3x2 +18x +32
find: A5 B4 C3 D2
( a ) The value of x for which the blacksmith makes a
maximum profit. 2001/23 Neco f/m Exercise 30.39
( b )The overall maximum profit The maximum value of the function y = 2 + 3x – 4x2 is
Solution 23 5 13 41 59
A.- B. C. D. E.
( a ) CP Cost price of x articles = $(200 – 48x + 3x2) 16 16 16 16 16
SP Selling price of x articles
2014/40 UTME Exercise 30.40
= selling price of one article × x articles
Find the minimum value of y = x2 – 2x – 3
= $ 3/5x × x
A–1 B–4 C4 D1
= 3/5x2
Profit(P) = Sp – Cp
= 3/5x2 – (200 – 48x +3x2)
P = 3/5x2 – 200 + 48x – 3x2 Cubic function’s case
At maximum profit dp/dx = 0 2014/15 Neco f/m
Calculate the minimum value of the function
dp = 6 x + 48 – 6x y = 2x3 – 6x + 3
dx 5 A – 1.5 B – 1.0 C 1.5 D 2.5 E 3.0
At maximum profit 6 x + 48 – 6x = 0 Solution
5 dy
6x – 30x = –240 At minimum =0
dx
x = 10 articles y = 2x3 – 6x + 3
(b) For overall maximum profit, dy
= 6x2 – 6
we put x = 10 into our P equation dx
P overall max, = 3 (10)2 – 200 + 48(10) – 3(10)2  6x2 – 6 = 0
5 6x2 = 6
= 300 – 200 + 480 – 300 x2 = 6/6 = 1
5 x=1
= $40 To get minimum value: we substitute x = 1 into
y = 2x3 – 6x + 3
2000/39 PCE = 2(1)3 – 6(1) + 3
30metres of fencing wire is available to make a =2– 6+3 =–1 B.
rectangular enclosure. Find the maximum area possible.
A.629m2 B525m2 C.225m2 D.125m2 2008/40 PCE
Solution Find a maximum value of the function f(x) = x3 – 12x + 5
Let the length of the rectangle = x A–9 B–2 C9 D 21
It follows that breath = 30 – x Solution
Area of rectangle = length × breath At maximum f /(x) = 0
= x( 30 – x ) f(x) = x3 – 12x + 5
Area ( A ) = 30x – x2 f /(x) = 3x2 – 12
For area to be maximum Factorizing
dA/dx = 0  3x2 – 12 = 0
488
3(x2 – 4) = 0 Turning points (x,y)
x2 – 4 = 0 [the value of (x,y) for maximum or minimum point]
x2 = 4 2000/6 f/m
x = 2 If the point p(x, y) is the maximum point on the curve
At maximum f (x) < 0
//
y = x3 + 3x2 – 7 , find the co-ordinates of p.
f // (x) = 6x A. (0, -2) B. (- 2, -3) C. (0, 7) D. (-1, -5)
6x < 0, When x = – 2 Solution
6 (– 2) < 0
To get maximum value, we put x = – 2 into At maximum or minimum; dy/dx = 0
f(x) = x3 – 12x + 5 y = x3 + 3x2 – 7
f (– 2) = (– 2)3 – 12 (–2) + 5 dy = 3x2 + 6x
= – 8 + 24 + 5 dx
= 21 D. At maximum or minimum; dy/dx = 0
Thus, 3x2 + 6x = 0
1992/39 UME 3x(x + 2) = 0
Obtain a maximum value of the function 3x = 0 or x + 2 = 0
f( x ) = x3 – 12x + 11 x = 0 or – 2
A. – 5 B. – 2 C.2 D.27 To know which of the values is maximum
Solution d2y = 6x + 6
Recall that f / ( x ) is equivalent of dy/dx dx2
While f / / (x ) is equivalent of d2y/dx2 When x = 0
Already, 6x + 6 = 6  0 minimum point not our target
f( x ) = x3 – 12x + 11
At max or min f / ( x ) = 0 Next, When x = – 2
In this case, f / ( x ) = 3x2 – 12 6x + 6 = – 12 + 6
It follows that = – 6 < 0 maximum point target gotten
3x2 – 12 = 0 To get the y co-ordinate; we substitute x = – 2 into
3x2 = 12 y = x3 + 3x2 - 7
x2 = 12 i.e 4 i.e y = (-2)3 + 3(-2)2 – 7
3 = – 8 + 12 – 7 i.e – 3
x =  4 P(x, y) is (-2, -3) B
= 2 2000/38 PCE
To know which of the values of + 2 and – 2 is Find the turning points of the function y = 2x3-6x2-18x + 3
maximum: A.( - 1, 13 ), ( 3, - 51 ) B.( 0, 3 ), ( 2, - 23 )
f / / (x ) = 6x C.( - 1, - 19 ), ( 2, 23 ) D.( - 1, 19 ), ( 3, 51 )
When x = + 2 Solution
6x = 6( 2 ) Already, y = 2x3 – 6x2– 18x + 3
= 12 > 0 minimum point At turning point dy/dx = 0
When x = - 2 In this case dy/dx = 6x2 – 12x – 18
6x = 6( - 2 ) It follows that
= - 12 < 0 maximum point 6x2 – 12x – 18 = 0
To get maximum value of the function, Divide through by 6
We substitute x = - 2 into f( x ) = x3 – 12x + 11 x2 – 2x – 3 = 0
= ( - 2 )3 – 12( -2 ) + 1 factorizing
x2 – 3x + x – 3 = 0
= - 8 + 24 + 11 x( x – 3 ) + 1( x – 3 ) = 0
= 27 (D ) (x+1)(x–3)=0
x + 1 = 0 or x – 3 = 0
2003/4 Exercise 30.41 x = – 1 or + 3
Given that y = x (x + 1)2 , At x = – 1
calculate, the maximum value of y. y = 2( - 1 )3 – 6( - 1 )2 – 18 ( - 1 ) + 3
A–2 B0 C1 D2 = – 2 – 6 + 18 + 3
= – 8 + 21
2014/9a Neco f/m Exercise 30.42 = 13
A curve is defined by f(x) = x3 – 6x2 – 15x – 1 i.e ( x, y ) = ( - 1, 13 )
find the: At x = + 3
(i) gradient of the curve at the point where x = 1 y = 2( 3 )3 – 6 ( 3 )2 – 18 ( 3 ) + 3
(ii) maximum and minimum points = 2 ( 27 ) – 6 ( 9 ) – 54 + 3
= 54 – 54 – 54 + 3
= – 51
i.e ( x, y ) = ( 3, - 51 ) Option A fits in
489
2002/38 PCE Implicit functions
Find the coordinates of the minimum point for the
equation y = 4t2 – 40t + 300 2003/6(b) Neco f/m (Dec)
A.( 5, 200 ) B.( 5, 100 ) C.( 4,300) D.( 4, 100 ) dy
Solution Find of the function 2x3y2 – 3xy2 = 4
dx
Given y = 4t2 – 40t + 300
At minimum point dy/dt = 0 Solution
Note that we did not use dy/dx; since the function is in y and 2x3y2 is differentiated as product:
t. d (2x3 y2) = 2x3 d ( y2) + 2y2 d (x3)
In this case, dy/dt = 8t – 40 dx dx dx
It follows that
Note that: d ( y2) is d ( y2) × dy i.e 2y dy
8t – 40 = 0
dx dy dx dx
8t = 40
d (2x3 y2) = 2x3 × 2y dy + 2y2 × 3x2
8t = 40
dx dx
8 8 t = 5
= 4x3y dy + 6x2y2
The y value is gotten by putting t = 5 into
dx
y = 4t2 – 40t + 300
3xy2 is differentiated as product:
y = 4 ( 5 )2 – 40 ( 5 ) + 300
d (3xy2) = 3x d (y2) + 3y2 dx
= 4 ×25 – 200 + 300
dx dx dx
= 100 + 300 – 200 = 200. 2
Note the result of d (y ) as gotten earlier
i.e ( t, y ) is ( 5, 200 ) ( A ).
dx
= 3x × 2y dy + 3y2
2004/39 PCE dx
The turning point of the curve y = 5 – 2x – x2 occurs at = 6xy dy + 3y2
A.( - 2, 5 ) B.( - 1 – 6 ) C.( - 1, 6 ) D.( 1, 6 ) dx
Solution 4 is a constant, hence its differential is 0
At turning point dy/dx = 0 2x3 y2 – 3xy2 = 4 differentiated becomes
y = 5 – 2x – x2 will become 4x y dy + 6x2 y2 – 6xy dy + 3y2 = 0
3
dy/dx = –2 – 2x dx dx
At turning point : dy/dx = 0 Opening up bracket we have
i.e –2 – 2x = 0 4x3y dy + 6x2y2 – 6xy dy – 3y2 = 0
–2x = 2 dx dx
– 2x = 2 Collect terms in dy/dx together
–2 – 2 4x3y dy – 6xy dy = 3y2 – 6x2y2
x =–1 dx dx
The y value is gotten by putting x = - 1 into Factor out dy/dx
y = 5 – 2x – x2 dy (4x3y – 6xy) = 3y2 – 6x2y2
y = 5 – 2( - 1 ) – ( - 1 )2 dx
=5+2–1= 6 dy = 3y2 – 6x2y2
i.e ( x, y ) = ( –1, 6 ) (C) dx 4x3y – 6xy

1998/41 PCE Exercise 30.43 2004/2(a) Neco (Dec) f/m


The turning point of the curve y = 6 – 4x – x2 occurs at dy
Find if 4x4 + y3 = 12x2y
A.( - 2, 10 ) B.( 0, 6 ) C.( 2, - 6 ) D.( 4, -26 ) dx
Solution
1992/41 PCE Exercise 30.44 4x4 is differentiated directly as
The turning point of the curve y = 5 – 8x – 2x2 occurs at d ( 4x4) = 16x3
A.( - 2, 13 ) B.( 2, 13 ) C.( 2, - 13 ) D.( - 2, - 13 ) dx
y3 is differentiated as chain :
2005/36 f/m Exercise 30.45 d ( y3 ) = d (y3) × dy = 3y2 dy
Find the stationary point of the function f(x) = –3x2 + 3x + 2 dx dy dx dx
A.(–11/4 , 9/4 ) B.( – 3/2 , 1/2) C. ( 1/2, 11
/4 ) D.(–1/2 , 9/4 ) 12x y differentiated as product:
2

d (12x2 y) = 12x2 dy + 12y d (x2)


1994/5(Nov) f/m Exercise 30.46 dx dx dx
= 12x2 dy + 24xy
Determine the turning point of the curve y = x2 + 250
x dx
A. (-5, -25) B.(5, 75) C. (5, 150) 4x + y = 12x2 y differentiated becomes
4 3

D. (10, 75) E. (5, 0) 16x + 3y2 dy = 12x2 dy + 24xy


3

dx dx
Collect like terms together
490
3y2 dy – 12x2 dy = 24xy – 16x3 2013/11 f/m Exercise 30.47
dx dx Differentiate x2 + xy – 5 = 0 with respect to x
Factor out dy/dx in the LHS −( 2 x + y ) 2x − y −x 2x + y
dy (3y2 – 12x2) = 24xy – 16x3 A B C D
x x 2x + y x
dx
dy = 24xy – 16 x3 2013/2 f/m Exercise 30.48
Calculate the gradient of the curve
dx 3y2 – 12 x2 x3 + y3 – 2xy = 11
at the point (2, – 1)
2013/26 Neco f/m
dy
If x3 + y3 = 27xy, find Implicit functions in gradient at a point
dx
2
x2 + 9 y x − 9y 2014/1 Neco f/m
A B
9x − y 2 9x + y 2 Find y / at point (1, 1) if x2y + y3 = 2
9y − x2 3(9 y − x 2 ) 9 y − x2 A–1 B – 1/ 2 C0 D 1/2 E1
C D E Solution
y 2 − 9x y 2 − 9x 3( y 2 − 9 x)
x2y + y3 = 2
Solution First, we differentiate implicitly
x3 + y3 = 27xy dy d (x2 ) dy
d d ( y3 ) dx dy x2 + y + 3y2 = 0
(x3) + = 27y + 27x dx dx dx
dx dx dx dx dy dy
x2 + 2xy + 3y2 = 0
dy dy dx dx
3x2 + 3y2 = 27y + 27x
dx dx dy
Collect terms in together
dy dx
Collect terms in together dy dy
dx x2 + 3y2 = – 2xy
dy dy dx dx
3y2 – 27x = 27y – 3x2 dy 2
dx dx (x + 3y2) = – 2xy
dy dx
(3y2 – 27x) = 27y – 3x2
= 2− 2 xy 2
dy
dx
dx x + 3y
dy 27 y − 3x 2
= At point (1, 1), x is 1 and y is 1
dx 3 y 2 − 27 x
dy
= −2 2(1)(1)2
= 3(9 y2 − x )
2
dx 1 + 3(1)
3( y − 9 x ) −2 −2 1
= = = − B.
9y − x2 1+ 3 4 2
= (C)
y 2 − 9x 2000/10 (C) (Nov) f/m
Find the gradient of the curve y2x + 3x2 y = 1,
at the point (1, 1)
2013/ 3b Neco f/m Solution
Given that x2y2 – 3xy + 4xy3 = 2, find
dy First, we differentiate implicitly.
dx y2 × dx + x × d (y2) + 3 x2 dy + 3y × d (x2) = 0
Solution dx dx dx dx
x2y2 – 3xy + 4xy3 = 2 2 2
y + x × 2y dy + 3 x dy + 3y × 2x dx = 0
d(y2 ) d (x 2 ) dy dx d(y3 ) dx dx dx dx
x2 + y2 – 3x – 3y +4x +4y3 =0
dx 2 2
dx dx dx dx dx y + x × 2y dy + 3 x dy + 3y × 2x = 0
dy dy dy dx dx
x2 2y + y2 2x – 3x – 3y + 4x 3y2 + 4y3 = 0
dx dx dx y2 + 2xy dy + 3 x2 dy + 6xy = 0
Collect terms in
dy
together dx dx
dx Collect like terms together
2x2y
dy
– 3x
dy
+ 12xy2
dy
= – 2xy2 + 3y – 4y3 2xy dy + 3x2 dy = – (y2 + 6xy)
dx dx dx dx dx
dy
(2x2y – 3x + 12xy2 ) = – 2xy2 + 3y – 4y3 dy (2xy + 3x ) = – (y2 + 6xy)
2

dx dx
dy = – (y2 + 6xy)
dy − 2 xy 2 + 3 y − 4 y 3 dx 2xy + 3x2
= At point (1, 1) , x is 1 and y is 1
dx 2 x 2 y − 3x + 12 xy 2
dy/dx = – (1 + 6 ) = -7/5 i.e -12/5
2+3
491
1996/16 (Nov) f/m Comparing rates of change
The gradient of the curve x2– 2xy – 2y2 – 2x = 0
If we are given a ‘main’ function say y = f( x ). Then
at the point (1, – 4) is
differentiating the main function y w.r.t time t
A. -4/3 B. -4/7 C. -1/2 D. -4/9 E. -1/8 ( instead of wrt to x ) will have a connecting formula
Solution through chain rule as:
First, we differentiate implicitly. dy dy dx Where our dy/dx here can be gotten
2x – 2x dy – 2y – 4y dy – 2 = 0 =  from the main function which is
dx dx
dt dx dt either given directly or mentioned
Collect terms in dy/dx together in the question as shown in the
2x – 2y – 2 = 2x dy + 4y dy examples below:
2000/38 UME
dx dx If the volume of a hemisphere is increasing at a steady rate
2x – 2y – 2 = dy (2x + 4y) of 18m3 s –1, at what rate is its radius changing when
dx it is 6m ?
2x – 2y – 2 = dy A. 2.50ms –1 B. 2.00ms –1 C.0.25ms –1 D.0.20ms –1
2x + 4y dx Solution
2(x – y – 1) = dy The main function here is the vol. of hemisphere V = 2/3r3
2(x + 2y) dx dv dv dr
Thus, dy = x – y – 1 But =  ---------***
dt dr dt
dx x + 2y
Here dv/dt is given as 18m3s – 1 and dv/dr is gotten from
At point (1, – 4) ; x is 1and y is – 4
dy/dx = 1 + 4 – 1 the main function
1–8 V = 2/3r3
= – 4 (B) dv/dr = 2r2
7 At r = 6m
1997/11a f/m 2r2 = 2 ( 6 )2
Given the curve 50x2 + 36xy + 5y2 = – 2 = 72
Evaluate dy/dx at the point (1, – 2) Substituting for dv/dt and dv/dr into *** ,
dr
Solution 18 = 72 ×
Differentiating implicitly, we have dt
50 d(x2) + 36x dy + 36y dx + 5 d (y2 ) = 0 18 = dr/dt
dx dx dx dx 72
dr/dt = ¼ = 0.25ms – 1 ( C )
100x + 36x dy + 36y + 10y dy = 0 The unit is so because length r in metres and time t in
dx dx seconds. dr/dt means unit metres per second or
Collect terms in dy/dx together metres/seconds or metres second – 1 ( ms- 1 )
36x dy + 10y dy = – (36y + 100x)
dx dx 2002/21 UME
dy (36x + 10y) = – (36y + 100x) A circle with a radius 5cm has its radius increasing at the
dx rate of 0.2cm s- 1 .What will be the corresponding increase
dy = – (36y + 100x) in the area ?
dx (36x + 10y) A.  B.2 C.4 D.5
At point (1, –2) ; x is 1 and y is – 2 Solution
dy/dx = – (– 72 + 100)
The main function here is Area of a circle A = r2
36 – 20
dA dA dr
= -7/4 i.e -13/4 But =  ---------***
dt dr dt
1993/12 (Nov) f/m Exercise 30.49 Here dA/dt is not given.
If x2 + 3xy + 2y2 = 3, find dy/dx at the point (– 1, 2 ). dA/dr is gotten from the main function
A. ¾ B. 8/11 C. 7/11 D. 2/3 E. – 4/5 A = r2
dA/dr = 2r
IPLF 3 Exercise 30.50 When r = 5cm
If ax2 + 2hxy + by2 = 0, find dy/dx 2r = 2 ( 5 )
= 10
IPLF 4 Exercise 30.51
We were given that: dr/dt = 0.2cms- 1
y4 + x2 = x + y2 , find dy/dx at the point (– 1, 1). Substituting for dA/dr and dr/dt into ***,
dA
2010/4 f/m Exercise 30.52 = 10 × 0.2
If 3x2 + 2y2 + xy + x – 7 = 0, dt
dy = 2 ( B )
find at the point (-2, 1).
dx
492
1998/42 PCE Curve sketching
The radius of a circle is increasing at the rate of The basic guides in curve sketching are:
0.4cm s- 1. Find the rate of increase in cm2 s- 1 of the Turning points one
area when the radius is 5cm. At turning points dy = 0
A.40 B.10 C.5 D.4 dx
Solution Provided NO TWO ROOTS ARE EQUAL in the given function
when factorised after differentiation then it could be
The main function here is area of circle i.e A = r2
maximum or minimum
dA dA dr
But = 
dt dr dt
We are to find dA/dt , dr/dt is given as 0.4cms- 1.
d 2y d 2y
But dA/dr can be gotten from the main function 2 0
2 0
dx dx
A = r2
dA/dr = 2r
When r = 5cm Next is the intercept at the y-axis and x-axis
2r = 2 ( 5 )
= 10 Turning points two
Therefore If TWO ROOTS ARE EQUAL in the given function when
factorised after differentiation then it is an inflexion
dA/dt = 10 × 0.4
= 4 ( D )
d2y
2 =0
2013/37 UTME Exercise 30.53 dx
The radius of a circle is increasing at the rate of
0.02cm5 – 1 . Find the rate at which the area is increasing
when the radius of the circle is 7cm d2y
Note that at the point of inflexion =0
A 0.35cm2 s – 1 B 0.88cm2 s – 1 dx 2
2 –1
C 0.75cm s D 0.53cm2 s – 1 To know the function’s pattern before and after the point of
inflexion
2005/3 UME Exercise 30.54 we get x and y values to the left and right of the point of
The radius r of a circular disc is increasing at the rate of inflexion say x = -2 and +2 into the main equation to get
0.5cm/secs. At what rate is the area of the disc the corresponding y values. You could try more –3and+3
increasing when its radius is 6cm ?
A.3cm2/sec B.36cm2/sec C.18cm2/sec Example CSK 1
D.6cm2/sec Sketch the curve y = x2 – 1
Solution
2006/25 PCE Exercise 30.55 y = x2 – 1
The radius of an ink drop on a blotting paper expands at At turning points dy = 0
the rate of 0.02 cm/sec. Find the rate of change of the dx
area when the radius is 5 cm dy  2x = 0
A.cm2/sec B.2cm2/sec C.0.2cm2/sec dx
D.0.5cm2/sec x = 0 (No equal roots here hence no inflexion)
To determine whether max or min
2013/24 f/m Exercise 30.56 d2y
= 2  0 min point
A circular ink dot on a piece of paper increases its area dx 2
at the rate of 4mm2/s. Find the rate of increase of the
radius of the dot when the radius is 8mm (Take  = 22/7) Intercept at the y-axis i.e x = 0 in the original equation
A 0.25mm/s B 0.20mm/s C 0.08mm/s D 0.05mm/s y = 02 – 1
= – 1 the curve cut the y-axis at – 1
Intercept at the x-axis i.e y = 0 in the original equation
0 = x2 – 1
x=1
The curve cut the x-axis at x = -1 and at x = +1 .Sketching
y

x
-1 1

-1

493
Example CSK 2 Intercept at the x-axis i.e y = 0 in the original equation
Sketch the curve y = x3 0 = x3 – x2 – 5x
Solution i.e x(x2 – x – 5) = 0
y = x3 x = 0 or (x2 – x – 5) = 0
At turning points dy = 0 cannot be factorized (x – x – 5) = 0
2

dx + 1  1 − 4  1  (−5)
dy  3x2 = 0 x =
dx 2 1
x = 0 twice (equal roots present hence inflexion) + 1  21
d2y =
At the point of inflexion =0 2
dx 2 1 − 4.6 1 + 4.6
i.e 6x = 0 = or
2 2
x = 0 thus inflexion occurred at x = 0 = - 1.8 or 2.8
To know the function’s pattern before and after the
Thus the curve cut the x-axis at x = –1.8, 0, and 2.8
point of inflexion take - 2 and +2
Intercept at the y-axis i.e x = 0 in the original equation
Before inflexion x = –2 after inflexion x = +2
y = 03 – 02 – 5(0) i.e 0
y = (-2)3 y = (+2)3 Thus the curve cut the y-axis at y= 0 Sketching
y
= –8 = +8

curve going down curve going up x


-1.8 -1 1.7 2.8

Sketching
y
8

0
2 x
-2

-8

Example CSK 3
Sketch the curve y = x3 – x2 – 5x
Solution
y = x3 – x2 – 5x
At turning points dy = 0
dx
 3x2 – 2x – 5 = 0
3x2 + 3x – 5x – 5 = 0
3x( x + 1) – 5( x + 1) = 0
(3x – 5)( x + 1) = 0
x = – 1 or 5/3 (1.7)
(No equal roots here hence no inflexion)
To determine which of x = –1 or 5/3 (1.7)is max or min
d2y
= 6x – 2
dx 2
when x = – 1
d2y
value becomes – 8 < 0 max point
dx 2
when x = 5/3
d2y
value becomes +8 > 0 min point
dx 2
494
Chapter Thirty One We can also relate i to iv with differentiation for better
Integration understanding; since they are opposite.
Integration is the opposite of differentiation. It is Differentiation integration
represented by elongated s i.e ∫ d
(sin x ) = cos x  cos xdx = sin x + k
x n +1 dx
In general :  x dx = n

n +1 d
(cos x ) = − sin x  sin xdx = − cos x + k
dx
i.e increase the power by one and divide by new power.
This is the fundamental formula of integration.
d
(ln x ) = 1 1
 x dx = ln x + k
dx x
Indefinite Integrals I
Lets us check this out
d x
dx
( )
e = ex  e dx = e
x x
+k
y = 10x3 + 3x and y = 10x3 + 3
Now differentiating 1994/41 UME
dy = 30x2 + 3 while dy = 30x2 Integrate 1 – x with respect to x
dx dx x3
To get back our original functions, we integrate i.e A. x – x2 + k B. 4 – 3 + k C. 1 – 1 + k D.1 – 1 + k
reverse the differentiation. x4 x4 x3 x 2x2 3x2 2x
Solution
 (30 x )
dy
 dx = +3
2
 1− x   1 x 
  x 3  dx =   x 3
−  dx
x3 
cross multiply the dx
 dy =  30 x + 3 dx
2
( ) =
1
 x3 dx −
1
 x 2 dx
Since there seems to be nothing on the LHS, it means 1
and 1 implies y0, i.e the variable there. Thus, =  x −3 dx −  x −2 dx
y0 + 1
=  30 x dx +  3dx
2

0 +1 = x – 3+1 – x –2+1 + k
Applying the formula to the RHS –3+1 – 2+1
y = 30x3 + 3x0+1 = x – 2 – x –1 + k
3 0+1 –2 –1
3
y = 10x + 3x
=–1 + 1 + k
We are back.
2x2 x
Similarly,
Rearranging
dy = 1– 1 + k
 dx =  30x
2
x 2x2 (C)
cross multiply the dx Note: All integrals must be simplified as far as possible

 dy =  30 x dx
2 before integrating as shown above.
1997/41 UME
y = 10x3 Integrate 1 + cosx with respect to x
We are back but we lost something which is the x
constant + 3. Thus we always add constant “C” or “K” A. –1 + sinx + k B. Inx + sin x + k C. Inx - sinx + k
to any indefinite integral like the one shown above. x2
Hence our actual answers are: D. –1 - sinx + k
y = 10x3 + 3x + k and y = 10x3 + k x2
Solution
There are some basic integrals we need to know.
1  1
i.  cos xdx = sin x + k   x + cos x  dx =  x dx +  cos x dx
ii.  sin xdx = − cos x + k Recall their results.
1 = Inx + sin x + k (B)
iii .  dx = ln x + k 1996/40 PCE
x
If dy = x3 – 2x2 – 3x + 1 , find y
iv. e dx = e + k
x x
dx
A. x4 – 2x3 – 3x2 + x + c B. x4 – 2x3 – 2x2 + x + c
Also integral of sum of functions is the sum of the
separable integrals i .e 4 3 2 4 3 3 2
C. 4x – 3x – 3x – x + c D. 4x – 2x – 3x - x + c
4 3 2 4 3 2
( 2
)
 x + 3x − 5 dx =  x dx +  3xdx −  5dx
2
2 2 3 2
495
Solution Solution
dy = x3 – 2x2 – 3x +1 Applying trig identities
dx
 tan xdx =  (sec x − 1)dx
2 2

Cross multiply dx
 sec xdx −  dx
2
dy = (x3 –2x2 – 3x + 1)dx =
Integrating both sides = tanx – x + c (A)
3
(
 dy =  x − 2x − 3x + 1 dx
2
) 2003/23 Neco
Applying integration formula Evaluate  (e x − sec x tan x)dx
y = x4 – 2x3 – 3x2 + x + c (A) Solution
4 3 2
 (e − sec x tan x)dx =  e dx −  sec x tan xdx
x x

1996/39 PCE Applying result of the respective integrals


x + 3x + 2
2
= ex – secx + c
Simplify  x +1
dx
2006/5 UME Exercise 31.0
3 2 3
A. x + 3x +2x + c B. x + 2x + c C. 2x + 3 + c If dy = x + cosx, find y. A. x2 – sinx + c
3 2 3 2 dx 2
D. x2 + 2x + c B. x2 - sinx + c C. x2 + sinx + c D. x2 + sinx + c
2 2
Solution 1992/42 PCE Exercise 31.1
First we simplify the numerator by factorization. Integrate x2(x +2) with respect to x A.x4+ x3 + 4 + c
x2 + 3x + 2 = (x + 2)( x + 1) will give x + 2 B.x3 + x4 + 7 + c C.x3 + x2 + c D. x4+ 2x3 + c
x+1 x+1 4 3 4 3
x 2 + 3x + 2
 x + 1 dx =  (x + 2)dx
Thus,
Indefinite Integrals II (change of variable)
In some problems, the integral does not fit in directly, in
2 such cases, a substitution involving change of the variable
= x + 2x + c (D)
will help.
2
1994/45 PCE Example INTER 1 Solve  cos 2 d
Integrate 2x (2x2 – 3x + 4) with respect to x. Solution
A.12x2– 12x + 8 B.x4 – x3 + 8x2 + c Our target will be to change 2 and d
C.x – x +x + c
4 3 2
D. x4 – 2x3 + 4x2 + c We let u = 2 
Solution Differentiating u with respect to 
We simplify the function by opening up the bracket, du = 2
( )
 2x 2x − 3x + 4 dx =  4x − 6x + 8x dx
2 3 2
( ) d
Applying integration formula Cross multiply
= 4x4 – 6x3 + 8x2 + c du = 2d
4 3 2 Making d subject formula
= x4 – 2x3 + 4x2 + c ( D ) du = d
2005/4 PCE 2
A function whose derivative is 1 has its integral as Substituting for 2 and d. Our new integral is
x2 du
A. –1 + k B.1 + k C –1 + k D. 2 + k  cos u 2
2x x x x
Solution Factor out the constant. ½
We are simply asked to integrate 1/x2. 1
2
= cos u du
1
x x
−2
Thus dx = dx
2 = ½ sinu + k; Replacing u with 2 = 1sin2 + k
Applying the formula: 2
= x –2 + 1 + k 2002/18 UME
–2+1 Evaluate  sin 3xdx
k or c is just a constant usually added to indefinite
A.–1 cos3x +c B.1cos3x +c C. 2cos3x +c D.– 2cos3x + c
integrals
3 3 3 3
= x –1 + k = –1 + k (C)
Solution
–1 x
Our target here will be to change 3x and dx
2005/39 Neco Let u = 3x
Integrate tan2x Differentiating u with respect to x
A tanx – x + c B tan2x – 2x +c C x – tan2x +c du = 3
D x – tanx +c E – tanx + 2x + c dx
496
cross multiply 2001/40 PCE
du = 3dx Evaluate  (cos 3x + sin 4 x ) dx
making dx the subject formula
du = dx A. 1 sin3x + 1 cos4x + k B. 1 sin3x – 1 cos4x + k
3 3 4 3 4
Substituting for 3x and dx ; our new integral is. C. 3sinx + 4cosx + k D. .sin3x + 4cos 4x + k
Solution
du
 sin u 3
Let’s break them into bits
=  cos 3x dx +  sin 4 x dx
1
3
Factoring out the constant 1/3; = sin u du Changing variables
For  cos 3x dx
= - 1 cos u + c Let u = 3x
3 Differentiating u with respect to x
replacing u with 3x = - 1 cos 3x + c (A) du = 3
3 dx
Note: The constant k is a dummy variable, hence the Cross multiply
examiner here used C; any other alphabet can be used in its du = 3dx
place. Though C and K are commonly used Making dx subject formula.
2001/34 UME du = dx
 2(2 x − 3) dx
2

Evaluate
3
3
Substituting for 3x and dx ; our new integral is
A. 3 (2x – 3 )5/3 + k B. 6 (2x –3 )5/3 + k du
5
C .2x – 3 + k
5
D.2(2x – 3 ) + k
 cos u 3
Solution Factor out the constant 1/3 ,
We can not expand the bracket as in the case of say 1
3
= cos u du
 (2x − 3) dx =  (2 x − 3)(2 x − 3) dx
2

= 1 sin U + k
=  (4 x − 12 x + 9)dx2
3
Next we replace u with 3x
=  4 x dx −  12 xdx +  9dx
2
i. e = 1 sin 3x + k
you know what follows 3
In this case we have in hand, Similarly
= 2 (2 x − 3) dx 1
2


3

We have factored out the constant 2; next let’s target 2x - 3 and dx


 sin 4 x dx = − cos 4 x + k
4
let u = 2x – 3 ;Differentiating u with respect to x
du = 2
Thus,  (cox3x + sin 4 x) dx =  cos 3x dx +  sin 4 x dx
= 1 sin3x – 1cos4x + k (B)
dx
3 4
Cross multiply
No need of adding two ks
du = 2dx I HOPE YOU OBSERVE A FORMAT HERE.
Making dx the subject formula 1
du = dx  cos 3x dx = 3 sin 3x + k
2
Substituting for 2x –3 and dx, our new integral is 1
du
 sin 4 x dx = − 4 cos 4 x + k
2 u 3
2

1
2
Factor out the constant 1/2  cos 5x dx = 5 sin 5x + k the pattern goes on
2 2 While that of Sine carries a minus sign along as shown below:
=  u 3 du 1
2
= U2/3 + 1 + k
 sin 4 x dx = − cos 4 x + k
4
2
/3 +1 1
 sin 5x dx = − 5 cos 5x + k
= U5/3 + k 1
5
/3  sin 6 x dx = − 6 cos 6 x + k the pattern goes on
= 3U5/3 + k ; Making a replacement of U = 2x – 3 1994/20 Exercise 31.2
5 Using the substitution u = cosx or otherwise, find
= 3 (2x -3)5/3 + k (A)
 cos x sin xdx
5

5
1996/41 PCE Exercise 31.3
497
Find the integral of 3cos2x – 2sinx 1 x2 1 1 3x 2
A.6sin2x – 2 cosx + c B. – 6sinx – 2cosx + c Thus  0 x 3 +1
dx =
3  0 x 3 +1
dx
C. 3/2 sin2x – 2cosx + c D. 3/2 sinx + 2cosx + c
1 1
1999/46 PCE = ln(x3 + 1)
3 0
If dy = 1 (sin4x + sin2x + 1), find y.
dx 2 Substituting for the upper& lower limits
A. cos4x + cos2x + x + k B. –2cos4x – cos2x + x + k
8 4 2 2
1
=
3
( 1
)
log e 13 + 1 − log e (0 3 + 1)
3
C. – cos4x – cos2x + x + k D. 2cos4x + cos2x + x + k 1 1
8 4 2 2 = log e 2 − log e 1
Solution 3 3
dy = 1 (sin4x + sin2x + 1) 1 2 1
dx 2 = log i.e log e 2
3 1 3
Cross multiply the dx
dy = 1/2( sin4x + sin2x +1)dx 1996/16 Exercise 31.4
Integrating both sides 2 1

1 Evaluate 
0 1 + 2x
dx
 dy =  2 (sin 4 x + sin 2 x +1) dx 1 1
1 A log e 2 B − log e 3 C log e 3 D log e 3 E 0
=  (sin 4 x + sin 2 x + 1) dx 2 2
2
Applying change of variable format to RHS INTER 3 Exercise 31.5
7x
y = 1 – 1cos4x – 1 cos2x + x + k
Evaluate  3( x 2
− 3)
dx
2 4 2 INTER 4 Exercise 31.6
Opening the bracket
x3
= – cos4x – cos2x + x + k ( C ) Evaluate  4 dx
8 4 2 2 x +1
Definite Integrals
If you observe that: In definite integrals, constant “k” or “c” are not added to our
the numerator is a derivative of the result since the value of the constant is gotten from the data in
denominator the question.
g 1 (x ) Definite integrals are of two kinds:
i.e.  dx = ln g(x) I. The problems where values of x and y are given to
g ( x) enable us find the value of the constant “k” or “c”
where g1(x) is the derivative of g(x) II. The problems, where lower and upper limits are
given, thus taking care of the “k” or “c”
Example INTER 2
Definite Integral I
4x − 4
 x 2 − 2 x + 5 dx 1998/41 UME
Find the equation of the curve which passes through the point
Solution
(2,5) and whose gradient at any point is given by 6x – 5
4x − 4 2x − 2 A.6x2 – 5x + 5 B.6x2 + 5x +5 C.3x2 – 5x – 5 D.3x2 – 5x + 3
Observe that x 2
− 2x + 5
dx = 2 2
x − 2x + 5
dx
Solution
And it can be seen that the numerator is the derivative Gradient  dy = 6x – 5
of the denominator dx
2x − 2 dy = (6x - 5)dx
2 dx = 2ln(x2 – 2x + 5) integrating both sides
x − 2x + 5
2

 dy =  (6 x − 5) dx
2003/7i y =  6 x dx −  5dx
1 x2
Evaluate  0 x 3 +1
dx
y = 6x 1+1 – 5x0+1
Solution 1+1 0+1
If we differentiate the denominator x3 + 1 it results to 3x2 y = 6x 2 – 5x1
but the numerator has x2 only 2 1
so we introduce 1/3 to take of the coefficient 3 imported y = 3x2 – 5x + k
Next we find k value

498
Point (2,5) means x = 2, y = 5 A = 3r3 – r + c
Substituting x and y values into y = 3x2 – 5x + k 3
5 = 3(2)2 – 5(2) + k = r3 – r + c
5 = 12 – 10 + k To get C value, we substitute for the first set of
5–2 = k A and r values
k=3 2 = (1)3 – 1 + c
Hence y = 3x – 5x + 3 (D)
2
2=c
Thus A = r3 – r + 2
2000/39 UME
Next we are to find A,when r = 2
A function f(x) passes through the origin and its first
A = (2)3 – 2 + 2
derivation is 3x + 2 . What is f(x)?
A = 8 (C)
A.y = 3x2+2x B.y = 3x2+ x C.y = 3x2+x D.y =3x2+2x
2 2 2
Solution
1994/4 Exercise 31.7
First derivation  dy/dx = 3x + 2
dy = (3x + 2)dx dA
If = 3x3–1 and A=1 when x = 0, find A when x = 1
Integrating both sides dx
 dy =  (3x + 2) dx A–10 B –8 C8 D10
y = 3x2 + 2x + k 2005/37 Exercise 31.8
2
Next, we find k value
dy
Given that = 6x2 – x + 2 and y = 5 when x = 1,
The origin means (0,0) i.e x = 0, y = 0 dx
Substituting x and y values into y = 3x2 + 2x + k find an expression for y in terms of x
2 A. 2y = 4x3 – x2 – 4x + 3 B. 2y = 4x3 – x2 + 4x + 3
0 = 3(0)2 + 2(0) + k C. 2y = 4x3 + x2 – 4x + 3 D. 2y = 4x3 – x2 – 4x – 3
2
i.e k = 0 2003/47 Neco Exercise 31.9
Hence y = 3x2 + 2x (A) dy
2 If = 4x–3 and y = 5 when x = 2, find y in terms of x
dx
2002/20 UME A 2x2 – 3x + 5 B 2x2 – 3x + 3 C 2x2 – 3x
If dy/dx = 2x – 3 and y = 3 when x = 0, find y in terms of x. D 2x2 – 3x – 4 E 4x
A. x2 – 3x – 3 B.2x2 – 3x C. x2 – 3x + 3 D. x2 – 3x.
Solution 2006/1 UME Exercise 31.10
dy = 2x - 3 The gradient of a curve is 2x +7 and the curve passes
dx through point (2,0). Find the equation of the curve.
dy = (2x - 3)dx A. y = x2 + 14x + 11 B. y = x2 + 7x + 9
Integrating, we have 2
C. y = x + 7x - 18 D. y = x2 + 7x + 18
 dy =  (2 x − 3) dx Definite integral II
y = 2x2 – 3x + k
2 2004/16 UME
y = x2 – 3x + k
 (x )
3
Next we find the value of k. Evaluate
2
−1 dx .
1
Given : x = 0, y = 3
Substituting x and y values into y = x2 - 3x + k A. 2 B. –2 C. –62/3 D. 62/3
3 = (0)2 – 3(0) + k 3 3
3 = k Solution

 (x )
3 3
Hence y = x2 – 3x + 3 (C) 2
−1 dx = x3 - x
1
2002/5 (Nov) 3 1
dA We substitute for upper and lower limits
If = 3r2–1 and A = 2 when r = 1, find A when r = 2 = (3)3 – (3) – ( 1 ) 3 – (1)
dr 3 3
A–10 B –8 C8 D10 = 27 – 3 – [ 1/3 – 1 ]
Solution 3
dA = 27 – 9 – 1 – 3
= 3r2–1
dr 3 3
dA = (3r2–1)dr = 18 – – 2
Integrating both sides, we have 3 3
(
 dA =  3r − 1 dr
2
) = 18 + 2 = 20 = 62/3 (D)
3 3 3
499
The general pattern is = 1 – [ –8 – 12 – 6 ]
3 3
 f (x ) dx = F (x)
b
= F(b) – F(a)
b
a
a = 1 – [ –26 ]
3 3
Where capital F is the result of the integral and b is the = 1 + 26
upper limit while a is the lower limit. 3 3
Though we will treat the formula pictorially under = 27 = 9 (C)
application in later part of this topic, 3
1998/40 UME
2003/37 UME
 (sec )

 (x )
Evaluate
3
2
− 2 x dx Evaluate
2
x − tan 2 x dx
2
2
A.  B.  - 2 C.  D.  + 2
A.4 B.2 C.4 D.1
2 3
3 3 Solution
Solution Recall the trig identity : 1 + tan2 = sec2
 (x )
3 3
 (sec )  (1+ tan )
2
− 2 x dx = x – 2x 3 2
Thus
 2
x − tan 2 x dx =
 2
x − tan 2 x dx
2
3 2 2 2 2

= ( 3 )3 – ( 3 ) 2 – ( 2 )3 – ( 2 )2
3 3
=  2
1dx

= 27 – 9 – 8 – 4 
3 3 = x
2
= 0 – 8 – 12
3 = –2(B)
= – [ – 4/3 ] i.e + 4/3 (C) 1997/42 UME
1
2000/35 UME
cos  − 1 
2
If y = x(x4 + x2 + 1), evaluate  −1
y dx
Find the value of  d A. 11 B.11 C. 5 D.0
0 sin 2  12 16 6
A.  B.  C. - D. - Solution
2 2 First, we expand the bracket
Solution
 ( )  (x )
1 1
 cos  −1
2
 − sin  2 x x 4 + x 2 +1 dx = 5
+ x 3 + x dx
 0 sin 2 
d =  0 sin 2 
d −1
1
−1

Recall from sin  + cos  = 1 2 2 = x6 + x4 + x2


sin2 = 1 – cos2 6 4 2 -1
But – sin2 = cos2 – 1 = ( 1 )6 + ( 1 )4 + ( 1 )2 – ( -1 )6 + ( -1 )4 + ( -1 )2
6 4 2 6 4 2
 You will observe that all the powers of –1 in the bracket are
= −  0
d even; hence all their values will be positive
 =1 + 1 + 1– 1 + 1 + 1 =0(D)
= – 6 4 2 6 4 2
0 1992/41 UME
= –  – [–0 ] i.e –  (D) 

 2 cos 2 x dx
4
1999/37 UME Evaluate the integral 
2

 (x −1) dx
1 12
Evaluate A. –1/2 B. –1 C. 1/2 D. 1
−2

A. –3 /3 B.7 C.9 D.11


1 Solution
 

 2 cos 2 x dx = 2 4 cos 2 x dx
Solution 4

We expand the bracket 


12 12

 (x )
2

−2 (x −1)
1 1
dx = 2
− 2 x + 1 dx We change variables 2x and dx
−2
Let u = 2x
Differentiating u with respect to x
= x3 – 2x2 + x 1
du = 2
3 2 -2
dx
We substitute for upper and lower limits
du = 2dx
= (1)3 – (1)2 + 1 – [ (-2 )3 – ( -2 )2 + (-2) ]
3 3 du = dx
2
= 1 – 1 + 1 – [ –8 – 4 –2 ]
3 3 Substituting

500
 (1− x )dx

du 2  1  2
 
−2
= 2 cos u 1 − 2  dx =
4

12 2 −1
 x  −1

Factor out constant 1/2 ;


2 4 = 1x1 – x -1 2

2 12
= cos u du 1 –1 -1


/4 2
= x + 1
= Sin u
/12  x -1
= 2 + 1 – –1 + 1
substituting for u

2 –1
/4 = 21/2 – ( –1 –1 )
= Sin 2x

/12 = 21/2 – (– 2 )
= 2 1/ 2 + 2 = 41/2 (A)
= sin2(/4) – sin2(/12)
1998/43 PCE
= sin /2 – Sin /6 


2
= Sin 900 – Sin 300 Evaluate cos 2 x dx
0
= 1– 1/2 i.e 1/2 (C)
A. –1 B.0 C.1 D.2
2003/48 PCE Solution

 (3x )
4 Applying Change of variable format;
Evaluate
2
− 2 x + 1 dx
2 We let u = 2x
A.42 B.64 C.52 D.46 Differentiating u with respect to x
Solution du = 2
Applying integration formula dx
4 du = 2dx
= 3x – 2x + x
3 2 Making dx the subject formula
3 2 2 du = dx
= (4)3 – (4)2 + 4 – [ (2)3 – (2)2 + (2) ] 2
= 64 – 16 + 4 – [ 8 – 4 + 2 ] Substituting for dx and 2x
= 64 + 4 – 16 – [ 8 + 2 – 4 ]  
du
 cos 2 x dx = 
2 2

= 68 – 16 – [ 10 – 4 ] cos u
0 0 2
= 52 – 6 i.e 46 (D).
1 2
2 0
2002/40 PCE = cos u du
4 1 = 1sin u /2
Evaluate  0
x
dx
2 0

A. – 4 B. –2 C.2 D.4 = 1sin 2x /2


0
Displaying the power of x properly, we have; 2
4 1 4 1 = /2 sin2 ( /2 ) – 1/2 sin 2(0)
1 

 0
x
dx =  0
1
x2
dx = 1/2 sin  – 1/2 sin 0
4
= 0 (B)
 x − 2 dx
1
= 2005/4 UME Exercise 31.11
0 


2
integrating, we have Evaluate sin 2 x dx
0
4 A. – 1/2 B. 1 C. –1 D. 0
= x1/2
1
/2 0
2005/1 PCE Exercise 31.12

 (3x )
3
4 Evaluate
2
− 2 x dx
1
= 2x1/2 A.18 B.17 C.16 D.15
0
2009/20 Exercise 31.13
Substituting for upper and lower limits
= 2(4)1/2 – 2(0)1/2 = 2( 2 ) – 0 = 4 (D)  x3 −1 2
Evaluate   2
 dx
1999/39 UME 1
 x 
2  1  A 0.5 B 1.0 C 1.5 D 2.0
Evaluate  −1
 1− 2
 x
 dx
 1994/42 UME Exercise 31.14
A.41/2 B.17/8 C.11/2 D –11/2 2

−1 (2 x +1)
1
Solution Evaluate dx
Displaying the power of x properly; A. 32/3 B.4 C.41/3 D.42/3
501
Area Under a Curve and Volume Sketching
y
y = 4 (line of symmetry)
4
Area under a Curve
y y

)
Case II

f(x
y=
x
0 2 4 6

A
x
B
x
a b Area = A - B -12

The diagram above shows a curve of a function f(x). (c) Area of bdd region =
The area under the curve along the x-axis is denoted by: 2 6
−  (8 x − x 2 − 12)dx +  (8 x − x 2 − 12)dx
 f (x ) dx = F (x)
b
0 2
= F(b) – F(a)
b
a
a
2 8x 2 x3 2
Also F is the integral result of f(x). The unit of First −  0
(8 x − x 2 − 12)dx = –
2

3
− 12 x
0
measurement is square unit. You will observe that this
is an application of definite integrals. Case II is special Substituting for upper & lower limits

( ) (23 ) − 12(2) – 4(0 ) − (03 ) −12(0)
3 3
Under this subtopic curve sketching
= – 4 2 −
2 2
is necessary
   
Two types of problems exist in areas under a curve:  32  32
= − −  = Square unit
1. Case where the value of ordinates are not given  3 3
(No restriction), here the sketched diagram is our guide
6 8x 2 x3 6

2. Case where the value of ordinates are given


Secondly  2
(8 x − x 2 − 12)dx =
2

3
− 12 x
2

(there is restriction), here the ordinate given is our guide


along with the sketched diagram Substituting for upper & lower limits

= 4(62) –
(6 ) – 12 (6) – 4(2 ) − (2 ) −12(2)
3
2
3
1996/11  
(a) Sketch the curve y = 8x – x2 –12 3  3 
(b) Draw the line of symmetry of the curve 216  8 
(c) Find the area of the finite region bounded by the = 144 – − 72 − 16 − 3 − 24
curve and the x - axis
3
Solution  8
= 0 – − 8 − 
(a) y = 8x – x2 –12 3
 
dy 32
= 8 – 2x = Square unit
dx 3
dy 32 32 64
At turning point =0 Thus, Area of bdd region = + = Square unit
dx 3 3 3
Thus, 8 – 2x = 0 1999/10 (Nov)
2x = 8 ; x = 4 Calculate the area of the finite region bounded by the
Value of y at the turning point curve y = x2 – 7x + 10 and the x – axis
y = 8(4) – 42 – 12 Solution
= 32 – 16 –12 i.e 4 y = x2 – 7x + 10
Next we determine whether it is a max or min
dy
point = 2x – 7
d2y dx
= – 2  0 Hence a maximum point dy
dx 2 At turning point =0
Intercepts ( x – axis ) here y is 0 dx
8x – x2 –12 = 0 is same as Thus, 2x – 7 = 0
x2 – 8x +12 = 0 2x = 7 ; x = 3.5
x2 – 6x – 2x +12 = 0 Value of y at the turning point
x(x– 6) – 2(x– 6) = 0 y = (3.5)2 – 7(3.5) +10
(x– 6)(x– 2) = 0 = 12.25 – 24.5 +10 i.e – 2.25
x– 6 = 0 or x – 2 = 0 Thus, x = 6 or 2 Next we determine whether it is a max or min point
Intercept (y – axis) here x is 0 d2y
y = 8(0) – (0)2 – 12 i.e – 12 = 2  0 Hence a minimum point
dx 2
502
The intercept (x-axis) and here y is 0 2000/7 (Nov)
x2 – 7x + 10 = 0 Calculate, in square units, the area of the finite region
x – 5x – 2x + 10 = 0
2
bounded by the cure y = x2 + 2x –3 and the x-axis
x(x – 5) – 2(x – 5) = 0 14 20 32 35
(x – 5)(x – 2) = 0 A B. C. D.
x = 5 or 2
3 3 3 3
Solution
Intercept (y – axis) here x is 0
y = x2 + 2x – 3
y = (0)2 – 7(0) +10
= +10 dy
= 2x +2
Sketching dx
y
10 dy
At turning point =0
dx
Thus, 2x +2 = 0
2x = – 2 ; x = –1
x
Value of y at the turning point
1 2 3 4 5
0
-1
y = (–1)2 +2(–1) – 3
-2.25
-2 = 1– 2 – 3 i.e – 4
-3 Next we determine whether it is a max or min point
Area of bdd region = d2y
2 5 = 2  0 Hence a minimum point
 0
( x 2 − 7 x + 10)dx –  2
( x 2 − 7 x + 10)dx dx 2
the intercept (x-axis) and here y is 0
2 x3 7x2 2 x2 + 2x – 3 = 0
First  − + = − + 10 x
2
( x 7 x 10) dx x + 3x – x – 3 = 0
2
0 3 2 0
x(x + 3) – 1(x + 3) = 0
Substituting for upper & lower limits
(x + 3)(x – 1) = 0
=
23 7 2 2

( )  03 7 0 2
+ 10(2) −  −

+ 10(0) 
( ) x = –3 or 1
3 2 Intercept (y – axis) here x is 0
3 2 
y = (0)2 +2(0) – 3
8 28 =–3
= − + 20
3 2 Sketching
y
26
= Square unit
3
5 x3 7x2 5 x
Secondly  ( x − 7 x + 10)dx = − + 10 x
2 -3 -2 -1 0 1
2 3 2 2
Substituting for upper & lower limits -3

=
5 75

3
( )2
2
+ 10(5) −  −
72 
+ 10(2) 
3
( )
2 -4

3 2 3 2  1

=
125 175

 8 28
+ 50 −  − + 20
 Area of bdd region = –  −3
( x 2 + 2 x − 3)dx
3 2 3 2  1 x3 2x2 1
125 8 175
But  −3
( x 2 + 2 x − 3)dx =
3
+
2
− 3x
−3
= + − + 50 − 20 + 14
3 3 2 Substituting for upper & lower limits
250 + 16 − 525 + 264 1  (− 3)3 
= = +1− 3 −  + (− 3) − 3 (− 3)
2
6 3  3 
5
= square unit
6 1
Thus, Area of bdd region = − 2 − − 9 + 9 + 9
2 5 3
=  0
( x 2 − 7 x + 10)dx –  2
( x 2 − 7 x + 10)dx
=−
32
26 5 3
= – 1
3 6 Thus Area of bdd region = –  −3
( x 2 + 2 x − 3)dx
47
= square unit 32
6 = square units (C)
3
503
2001 /11 Neco (c) Area enclosed by the curve and x-axis
y
Sketch the curve y = (x +2)( x – 2)( x – 4) 16
(b) Find the maximum and the minimum value of the
function to the nearest whole number
(c) Calculate area enclosed by the curve and the x-axis. -2 0 2 4 x

Solution
First, we expand to know the function proper.
(x +2)( x – 2)( x – 4) = (x - 4) [(x +2)(x - 2)] ( x 3 − 4 x 2 − 4 x + 16)dx + −  ( x 3 − 4 x 2 − 4 x + 16)dx
2 4

= (x - 4)(x2 - 2x + 2x - 4)
=  −2  2 
= (x - 4) (x2 - 4) 4
= x(x2 - 4) - 4(x2 - 4 ) = x - 4 x - 4 x + 16x
4
3
x 4 4x 3 4x 2
2 2
− − + 16 x −
= x3 – 4x2 – 4x + 16 4 3 2 −2 4 3 2 2
i.e y = x3 – 4x2 – 4x + 16 Readers to substitute and complete
dy
Next, we sketch the curve at max or min =0 2005/11b
dx
dy ( i ) Sketch the curve y = x2 – 4
 3x 2 − 8 x − 4 = 0 ( ii ) find the area of finite region enclosed by the curve
dx
y = x2 – 4, the x-axis and the ordinates x = –2, x =3
By quadratic general formula
Solution
x = − (−8)  (−8) − 4  3  (−4)
2
First, we sketch the curve
23 y = x2 – 4
8  112 dy
= = 3.1 or – 0.4 At max or min =0
6 dx
Next, we determine which of the turning points is max
dy
or min.  2x = 0
d 2y dx
= 6x – 8 x=0
dx 2 Value of y at turning point
When x = 3.1 y = (0)2 – 4
6x – 8 = 10.6 > 0 minimum point = –4
When x = – 0.4 To determine max or min point
6x – 8 = –10.4 < 0 maximum point
Intercept at x – axis here y = 0
d 2y
= 2  0 min point
0 = x3 – 4x2 – 4x + 16 dx 2
i.e (x +2)( x – 2)( x – 4) = 0 Intercept at x – axis , here y is 0
x = –2 or 2 or 4 0 = x2 – 4
Thus ,the curve cuts the x–axis at -2, 2 and 4  x2 –22 = 0 thus (x–2)(x + 2) = 0 ; x = 2 or –2
Intercept at y – axis, here x = 0
y = 03 – 4(0)2 – 4(0) +16 Intercept at y – axis, here x is 0
y =16 y = (0)2 – 4
The curve cut the y – axis at y =16. =–4
Sketching y Sketching
y
16

-2 0 2 x

-2 0 2 4 x

-4

(b) Maximum value of the function The area of bounded region here is between – 2 and 2 but the
value of x is -0.4 required area of bounded region is between – 2 and 3. Hence
Max value y = (-0.4 )3 – 4(-0.4)2 – 4(-0.4) + 16 we extend our graph to the point where x = 3 by simple
= –0.064 – 0.64 + 1.6 + 16 substitution of values.
= 17.6 – 0.704 When x = 3, y = 32- 4 i.e. 5
= 16.896 Our new sketchy is
≈ 17 5
Minimum value of the function
value of x is 3.1
Min value of y = (3.1)3 – 4(3.1)2 – 4(3.1) + 16 -2 0 2 3 x

= 29.791– 38.44 – 12.4+16


= 45.791 – 50.84 -4

= –5.049 Area of bounded region = – 


2
(x 2
)
− 4 dx +  (x
3
2
− 4 dx)
≈ –5 −2 2

504
 (x )  (x )
3 2 Example AUC 1
=
2
− 4 dx – 2
− 4 dx
2 −2 Find the area of finite region enclosed by the curve y = 2 x
 x3 2  and the lines x = 3 and x = 0
= x − 4x 3 –
3

 − 4x  A 4 3 B 2 3 C 16 3 D 18 3
3 2 3 −2
Solution
= 3 − 4(3) −  2 − 4(2) −  2 − 4(2) −  (−2) − 4(−2) 
3 3 3 3

    y= 2 x
3 3 3   3 
y = 2x ½
= 27 −12 −  8 − 8 −  8 − 8 −  − 8 + 8 
3 3  3  3  dy
     = x –½
dx
= –3 + 16 −  − 16 − 16  = 13 square unit
3  3 3  At max or min point dy/dx is zero
1 ½=0
2003/25
x
Calculate, in square units, the area of the finite region x=0
bounded by the curve y = 1 + x – 2x2, the x-axis and the Value of y turning point
coordinates x = 0 and x = 1 y = 2 (0) i.e 0
A 5/ 6 B1 C 13/6 D4 we are heading to a dead end, hence we try sketching the
Solution curve using values within the given range x = 0 to x = 3
First, we sketch y = 1 + x – 2x2 y= 2 x
dy When x = 0, y = 0
At max or min =0,
dx When x = 1, y = 2
When x =2, y = 2.8
dy
 1 − 4x = 0 When x = 3, y = 3.5
dx y
1
x = /4 i.e 0.25
Value of y at turning point
y = 1 + (0.25) – 2(0.25)2
= 1 + 0.25 – 0.125 = 1.125 ≈ 1.1 0 x
3
To determine max or min point
From our sketching the curve lies on the upper part of the
d 2y x–axis within our given range of x = 0 to x = 3
= - 4 < 0 max point
dx 2 3
1

Intercept at x–axis, here y is 0 Thus, area of bdd region =  0


2 x 2 dx
0 = 1 + x – 2x2
3
 1 + 2x –x –2x2 = 0 2x 2 3

1(1 + 2x ) –x(1 + 2x) = 0 =


3 0
(1 – x )(1 + 2x ) = 0 2
x = 1 or – ½ 4 32 3
Intercept y – axis , here x is 0 = x
y = 1 + 0 – 2(0)2 3 0

=1 3
4(3) 4
2 3
Sketching = (0) 2 −
y
3 3
1 4 1 43 3
= (27) 2 = = 4 3 (A).
3 3
-1 -0.5 0 0.25 0.5 1
x Example AUC 2
Find the area bounded by the curve y = x2, the
The area of bounded region here is between – 0.5 and 1 x-axis and the ordinates x = – 2 and x = 2
but the required area of bounded region is between 0 Solution
and 1. First, we sketch the curve y = x2
Hence, we don’t need to extend graph.
1 At max or min dy = 0
Area of bounded region =  0
(1 + x − 2 x 2 )dx
dy  2x = 0
dx

x 2 2x3 1 dx
=x+ − x=0
2 3 0
Value of y at turning point
= 1 + 1/ 2 – 2/ 3 y = (0)2 i.e 0
= 5/6 square unit We are heading to a dead end, hence we try sketching the
curve using values within the given range x = -2 to x = 2
505
y = x2 Area bounded between two Curves
When x = – 2 , y = (– 2 )2 i.e. 4
A
When x = –1 , y = (– 1 )2 i.e.1 y
f2(x) Cubic function
When x = 0 y = (0)2 i.e. 0
When x = 1 y = 12 i.e. 1
When x = 2 y = 22 i.e. 4 h
x
Sketching a b
y

dx Cubic function
f1(x)

y
x B
-2 -1 0 1 2
Qu
0 2 ad
 x dx +  ra
2 2
Area of bounded region = x dx tic
fun
−2 0
cti
on
x3 0 x3 2
= + f2(x)
3 −2 3 0
a b
0  ( −2) 
3 x
23 0
= −  + −
3  3  3 3 Linear function
f1(x)
= 8 + 8
3 3 Area between two curves can be treated under:
= 16 ≈ 5.3 square units ( i ) a case where the sketching can be done by 1st and 2nd
3
differential and solving for maximum or minimum points
2010/15 Neco Exercise 31.15
( ii ) a case where the sketching cannot be done by 1st and
Find the area bounded by the curve y = 2x2 + 5,
2nd differential since all the results will play around zero
the x-axis and the ordinates x = 1 , x = 3
hence we sketch by substitution of values (simple range )
2 1 into the two equations.
A 5 square unit B 27 square unit
3 3
1 2
 (f ( x) − f1 ( x) ) dx
b
C 28 square unit D 33 square unit E 38 square Area between two curves A = 2
3 3 a
unit The order of substituting equation f1(x) and f2(x) is based on
the uppermost function from the sketch or the presented
Exercise 31.16 (diagram)
Find the area bounded by the curve y = 3x2 and the
x-axis and the ordinates x = 0 and x = 3
2007/2 Neco Exercise 31.17
Find the area bounded by the curve y = x2 – 5x , the
x-axis and the co-ordinates x = 0 and x = 8 to the
2000/34 UME
nearest square unit y
A 4 B 6 C 8 D 11 E 13
y = 16

2002/35 (Nov) Exercise 31.18


Calculate in square units, the area of the finite region
bounded by the x - axis, y – axis and the
line 2y = x + 6
A3 B6 C 9 D 12 x
(0,0) (4,0)

1995/3 (Nov) Exercise 31.19


Calculate in square units, the area of the finite region If the diagram above is the graph of y = x2, the shaded area is
x y A. 64square units B.128square units C.64square units
bounded by the lines + =1 , y = 0 and x = 0 D.32 square units
4 5 3 3
Solution
A5 B10 C 15 D 18 E 20 This is a typical question of area between two curves: Shine
Exercise 31.20 your eyes and you will see y = 16 (linear equation) and the
Find the area of the finite region bounded by the curve curve equation which was mentioned y = x2.
y = x3 + x2 – 2x and the x-axis
506
Of course, if you err and think it is area under a curve, We only need any two points to sketch a straight line
the answer is there too as 64/3. Here let us take When x = 0 , y = 3  When x = 1 , y = 4
Here our limits were given in a ,b co-ordinate forms.
(iii) Co-ordinates of their points of intersection
x = 0 and x = 4;
5x – x2 = x + 3
Recall that the two functions are to be subtracted.
x – 4x + 3 = 0
2
The graph indicates f2 (x) to be y = 16
x2 – 3x– x + 3 = 0
 (16 − x )dx
4
2
(x – 3) (x – 1) = 0 Thus x = 3 or 1
0
And they are our upper and lower limits as well
= 16x - x3 4 From the sketch the curve is uppermost, hence
3 0 Area of the segment = 
3

1
(5x − x ) − (x + 3) dx
2

 (4 x − x )
= 16(4) – ( 4 )3 – 16( 0 ) – ( 0 )3 3
2
− 3 dx
=
3 3 1
= 64 – 64 2 3
3
3 = 4 x − x − 3x
2 3 1
= 192 – 64
3 4(3) 2 (3) 3 4 1 
= 128 square units (B) = − − 3(3) −  − − 3
2 3 2 3 
3
= 18 – 9 – 9 – ( – 11/3)
2000/11 (Nov) = 11/3square units
a ( i ) Sketch on the same axes the curve y = 5x – x2
and the line y = x + 3 2001/36 UME
( ii ) Find the co-ordinates of their points of Find the area bounded by the curves y = 4 – x2
intersection and y = 2x + 1.
b Find the area of the segment cut off from the A.201/3sq. units B.20 2/3sq. units
2
curve by the line. C.10 /3sq. units D.101/3sq. units
Solution Solution
y = 5x – x2 y = 4 – x2
dy = –2x
dy
= 5 – 2x dx
dx At turning point dy = 0 i.e –2x = 0
dy dx
At turning point =0 It will lead us to a dead end
dx
Thus, 5 – 2x = 0 Let us substitute using a simple range of – 3 to 3
5 = 2x y = 4 – x2 y = 2x + 1
x = 5/2 i.e 2.5 When x = -3 , y = -5 x = -3 , y = -5
x = -2 , y = 0 x= 1, y=3
Value of y at the turning point
x = -1 , y = 3
y = 5(2.5) – (2.5)2 x= 0, y=4
= 12.5 – 6.25) i.e 6.25 x= 1, y=3
Next we determine whether it is a max or min point x= 2, y= 0
d2y x = 3 , y = -5
= – 2  0 Hence a maximum point
dx 2 Sketching
y
Intercepts ( x – axis ) here y is 0 4
5x – x2 = 0
x(5 – x ) = 0
x = 0 or 5 – x = 0 1

x = 0 or 5
Intercept (y – axis) here x is 0 -3 -2 1
x
0 2
y = 5(0) – (02) i.e 0
Sketching the curve first then fit in the straight line
-5
y y
Curve Here our limits were not given; thus we equate the two
6.25 Curve+ line
6.25 equations and solve the resulting quadratic equation.
4 – x2 = 2x +1
3 4 – x – 2x – 1 = 0
2
or 2x +1 – 4 + x2 = 0
rearranging x2 + 2x – 3 = 0
x 3 – 2x – x = 0
2
x
0 2.5 5 0 2.5 5 multiply through by minus
Incorporating line y = x + 3
507
x2 +2x – 3 = 0 3x 2 x 3
Factorising = − 3
0
x2 + 3x – x – 3 = 0 2 3
x(x + 3) –1(x + 3) = 0 27 27
(x– 1)(x +3 ) = 0 = −
x = 1 or –3
2 3
The smallest value is –3 and that is the lower limit 81 − 54 27
= i.e. 4.5(B)
Note that from the sketch the curve is uppermost, 6 6
hence 2002/ 9
 (4 − x )− (2 x +1)dx
1
Area bounded =
2 A fixed point A(m, n) lies on the curve y = x2 + 1
−3 and the line y = x +7

 (4 − x )
1 (a) Find: ( i ) the two possible coordinates of A
=
2
− 2 x −1 dx ( ii ) the distances between the two locations of A
−3

 (3 − 2 x − x )dx
1
2
(b) Sketch the curve and draw the line
= (c) Find the area of the finite region enclosed by the line and
−3
the curve.
= 3x – 2x2 - x3 1
Solution
2 3 -3
a . i Points of intersection first
x2 +1 = x + 7
= 3x – x2 – x3 1
x –x–6=0
2
3 -3
x2 + 2x – 3x – 6 = 0
= 3(1) – (1) – ( 1 ) - 3(-3) – (-3) – ( -3 )3
2 3 2
x(x + 2) – 3(x + 2) = 0
3 3
(x + 2)(x – 3) = 0
= 3 – 1 – 1 – – 9 – 9 – (–27 )
x = - 2 or 3
3 3
Coordinate of intersection A
= 2 – 1/3 – (–9 )
When x = -2 , y = (-2)2 – (- 2) – 6 i.e. 0
= 2 – 1/ 3 + 9
When x = 3, y = (3)2 –3 – 6 i.e. 0
= 6 – 1 + 27
Thus coordinates of A are (-2, 0) and (3, 0)
3
= 32/3 i.e102/3 sq units (C)
(ii) Distances between two points of A
2000/40 PCE
Find the area bounded by the curve y = x2 and = (3 − (−2)2 + (0 − 0) 2
the straight line y = 3x. = 25 i.e. 5
A.22.5 B.4.5 C. – 4.5 D. – 22.5
Solution b. y = x2 +1
It is obvious y = x2 differential will play around zero. dy
Let us substitute using a simple range of – 3 to 3 = 2x
dx
y = x2 y =3x it is obvious ,everything here is about zero .Let us work by
When x = - 3, y = 9 When x = -3, y = -9 substitution of values x = - 2 to 3 say from x = -3 to 3
x = - 2, y = 4 When x = 0, y = 0 y = x2 +1 y=x+7
x = - 1, y =1 When x = - 3, y = 10 When x = 0, y = 7
x = 0, y = 0 x = - 2, y = 5 x = 3, y = 10
x = 1, y = 1 x = - 1, y = 2
x = 2, y = 4 x = 0, y = 1
x = 3, y = 9 x = 1, y=2
Sketching x = 2, y=5
y
9
x = 3, y = 10
Sketching y
4
10

1 7

-3 -2 -1 0 1 2 3 x

Equating the two curves to get limits of integral 1


x2 = 3x -3 -2 -1 0 1 2 3 x

x – 3x = 0
2
From the sketch, uppermost function is the straight line
x(x – 3) = 0
 x + 7 − ( x 
3
x = 0 or 3 c. Area enclosed =
2
+ 1) dx
−2
Note that from the sketch the line is uppermost, Thus
3

3
Area bounded =  0
(3x − x 2 ) dx =
−2
(x – x2 + 6) dx

508
 (4 x − x 
3
x2 x3 =
2
) − ( x 2 − 2 x) dx
= − + 6 x 3−2 0
2 3
3

=
3 2 33  (−2) 2 (−2) 3
− + 6(3) −  −

+ 6(−2)
=  0
(3x − x 2 ) dx
2 3  2 3 
3x 2 x 3 3
9  8  = − 0
= − 9 + 18 −  2 + − 12 2 3
2  3 
3(3) 2 (3) 3 27 9
= 27 + 22 = − = − 9 = square units.
2 3 2 3 2 2
= 125/6 square units. i.e 20.83 square units Example ABTC 1 Acid test for area between two curves
1994 / 9 x5
Find the area of the figure bounded by the curve y =
Given the curves, y = 4x – x2 and y = x2 – 2x 16
(a) Determine the coordinates of their points of and the line y = x and between x = -2 and 2
intersection. Solution
(b) Sketch, on the same axes, the two curves. 5

(c) Find the area of the finite region bounded by the two y=x y= x
16
curves. 4
Solution dy = 1 dy = 5 x
(a) Their points of intersection: dx dx 16
4x – x2 = x2 –2x At max or min dy/dx = 0 At max or min dy/dx = 0
4x – x – x2 + 2x = 0
2
It is obvious, everything here is about zero. Let us work by
4x – 2x2 + 2x = 0 substitution of values x = - 2 to 2
2x – x2 + x = 0 5

3x – x2 = 0 y= x y=x
16
x(3 – x) = 0 x = - 2, y=-2 x = -2, y = -2
x = 0 or 3 x = - 1.5, y = - 0.47 x=2 y=2
(b) y = 4x – x2 y = x2 – 2x x = - 1, y = -0.0625
dy = 4 – 2x dy = 2x – 2 x = - 0.5, y = -0.002
dx dx x = 0, y=0
At max or min dy/dx = 0 At max or min dy/dx = 0 x = 0.5, y = 0.002
4 – 2x = 0 2x – 2 = 0 x = 1, y = 0.0625
x=2 x=1 x = 1.5, y = 0.47
Values of y at turning points Values of y at turning points
x = 2, y=2
y = 4(2) –22 y = 12 – 2(1)
=4 =–1 Sketching
2 2 y
d y = -2 < 0 Max point d y = 2 > 0 Min point 2
dx 2 dx 2
1
Intercept at x-axis here y is 0 Intercept at x-axis here y is 0
4x – x2 = 0 x 2 – 2x = 0 -2 -1 0 1 2 x
x(4 – x) = 0 x(x – 2) = 0
x = 0 or 4 x = 0 or 2 -1

Intercept at y-axis here x is 0 Intercept at y-axis here x is 0 -2

y = 4(0) – (0)2 y = (0)2 – 2(0)


=0 =0 In the interval 0<x<2 the straight line y = x lies above the
curve y = x5/16 but in the interval -2<x<0 their roles are
Sketching reversed hence
y
Area of the figure bounded
4
0  x 
5
2  x5 
3
=
 −2  16 − x  dx +  0  x − 16  dx

6 2
0 2 6
2
2
= x − x + x − x
96 2 −2 2 96 0
1

= 0 –  64 − 4 
4 64 
-1 0 1 2 3 4 x +  −  –0
-1  96 2 2 96 
= 8/3 or 22/3 square units
Our limits are the points of intersection i.e 0 and 3.
From the sketch, uppermost function is 4x – x2 Example ABTC 2 exceptional case
(c) Area of the finite region bounded by the two Find the area of the figure bounded
curves. by the curve y = x2 and the line y = 2– x
and the x-axis for which x > 0, y ≥0
509
Solution Area of finite region bounded by a curve & the y-axis
y
It is obvious y = x2 differential will play around zero. y = f(x)
Let us substitute using a simple range but bear in mind
a
the condition x ≥ 0, y ≥ 0 i.e. positive axis only. Take 0
to 2
b

y = x2 y = 2– x x
x = 0, y=0 x = 0, y = 2
x = 0.5, y = 0.25 x = 2, y = 0 The diagram above shows a curve of a function f(x). The
x = 1, y=1 area of the segment cut of by the y-axis at points
x = 1.5, y = 2.25 a and b is denoted by
x = 2, y=4 a a
Sketching
y  b
f ( y )dy = F ( y )
b
= F(a) – F(b)

2 Here F is the integral result of f(y).

1 Example AII 1
Find the area of the finite region bounded by the curve
0 1 2 x y = 9x2 + 1 and the y-axis at y = 3 and y = 1
Solution
The curve and the line meeting points a
x2 = 2 – x Area =  b
f ( y )dy
x + x –2 = 0
2
First we make x the subject formula
Factorization y = 9x2 + 1 becomes
x2 + 2x – x – 2 = 0
y – 1 = 9x2
x(x + 2) – 1(x + 2) = 0 y – 1 = x2
x + 2 = 0 or x – 1 = 0 9
x = – 2 or 1
Since we are considering x ≥ 0 and y ≥ 0, we take 1 y −1 y −1
= x ; Thus, x =
Hence they meet at x = 1. 9 3
1 2
 x 2 dx +  (2 − x)dx ( y − 1) 2
1
The area of 3
0 1 Area =  dy
What happened is the region from x = 0 to x = 1 is
1 3
under the curve (x2) and from x = 1 to x = 2 that region 1 3
 ( y − 1) 2 dy
1
=
is under the straight line 3 1
3 2
x2  1  2( y − 1)
3
= x
1
+ 2x −
3 2

3 2 = 
0
1  3 3 1
= 1 4 1
3 + [ 4– 2 – (2– 2)] 2( y − 1)
3
3 2

=
= ⅓ + (2 – 1½) 9 1
=⅓+½ 3
2(2) 2
= 5/6 square unit. = −0
9
Exercise 31.21 2 8 4 2
Find the area of the region bounded by the curve = i.e square unit
9 9
y = 3 – x2 and the line y = – x + 1 and
between x = 0 and x = 2
Example AII 2
Find the area of the finite region bounded by the curve
2005/2 UME Exercise 31.22
y = 16x2 and the y-axis at y = 4 and y = 0
Find the area of the figure bounded by the given pair of
Solution
curves y = x2 – x + 3 and y = 3 a
A.7units(sq) B.1 units(sq) C.17units(sq) D.5
units(sq)
Area =  b
f ( y )dy

6 6 6 6 First we make x the subject formula


y = 16x2 becomes
y
= x2
16
1
y y y 2

= x Thus, x = i.e
16 4 4
510
1
4 y2 Solution
Thus, area =  dy Rotation around y – axis, volume is given by:
0 4 b
x
2
1 4 12 V = dy
4 0
= y dy a

We first of all make x the subject formula from y = x2 – 1


1 3 4 y + 1 = x2
=  y 
3
2
Taking the square root of both sides
4 2 0
3
( y + 1 )1/2 = x
y 2 4 substituting for x in the formula
  dy
= 2
 ( y +1)
3
6 0

1
V= 2

(4)
3 0
2
4
= −0 = square units
6 3 = y2 + y 3
2 0
VOLUMES
y
substituting for the upper and lower limits

=  32 + 3 –  ( 0 ) 2 – 0 =  ( 9/ 2 + 3 )
)
f(x

2 2 = 15/2  cubic units (B)


y=

2003/12a Exercise 31.23


( i ) Sketch the graph of y = x2 , showing the area in the
a b
x first quadrant enclosed by the curve and the two lines
If the area below the curve above is rotated: y = 1 and y = 3
( i ) completely about the x - axis or 3600 or four right ( ii ) If the area in a( i ) is rotated about the y – axis,
angles. Then the volume of solid it generates is find the volume of the solid generated

x
a b

b
y
2
With the formula V = dx
a

Note: y2 and dx

( ii ) Completely about the y – axis or 3600 or four right


angles Then the volume of solid it generates is
y

a x
b

With the formula


b
x
2
dy Note: x2 and dy
a
The solid figure generated by the two diagrams are called
solid of revolution, hence this subtopic is sometimes called
volumes of revolution

2000/37
A bowl is designed by revolving completely the area
enclosed by y = x2 – 1, y = 0 and x  0 around the
y – axis at y =0 and y = 3. What is the volume of this
bowl.?
A.7 cubic units B.15/2  cubic units C.8 cubic units
D.17/2  cubic units

511
512
513
514
515
516
517
518
519
0

520
521
522
523
1.1 37 2.25 SP = N 3,280.00 3.33 24a2x3y3
56 2.26 N3,200.00 3.34 1
/2(a + b + c)
2
1.2 /3 2.27 i. D4,000 ii. D4,940 iii. D940.00 3.35 –2(2x – y)
4
1.3 /9 2.28 N2,500.00 3.36 x−4
1.4 47 2x − 3
1.5 19/10 2.29 4% 3.37 6(3k –1)
1.6 5
/36 2.30 2yrs 3.38 a + 2b / a – 3b
1.7 2
/3 2.31 N2138.40 3.39 –m–n
1.8 16/7 2.32 71/2% 3.40 4 x − 13
1 ( x + 2)( 2 x − 3)
1.9 /40 2.33 162/3 %
3.41 x−6
1.10 77/30 2.34 N 189.00 12
1.11 32/39 2.35 2% 4x
1.12 21/2 2.36 N 225.00 3.42 –
1.13 17
/15 4 − x2
2.37 N 420.00 3.43 2 − 3x
1.14 – 19/5 2.38 10% 4 − x2
1.15Total profit N 28000
3.44 1
Ade N 9333.3 Tayo N 16333.3 2.39 N 420.00 x +1
Uche N 2333.3 3.45
2 2.40 N10,123.2 4y − x
1.16 /5
11 2.41 N 56.19 y
1.16b /60 3.46 − ( x + 4)
x 2 − 3x + 2
1.17 II and III only 3.47 4− x
1.18 0.0059 ( x − 1)( x + 2)
1.19 0.17 3.48 3x + 2
3.1 52/3
1.20 3.80 6
3.2 x = 1
1.21 0.0100 5a
3.3 3 3.49
1.22 0.00005 6
3.4 – 4
1.23 5.10 3.50 8/125
3.5 x = 8
1.24 0.98% 3.51 0
3.6 –5
1.25 31/8% 3.51b 1
3.7 151/2
1.26 55/9% 12 x + 5
3.8 5/24 3.52
1.27 2.6% 5 x − 12
3.9 5 = x
3.10 x = 7
3.11 5 3.53 x = –3
2.1 A 36yrs B 18yrs C 9yrs
3.12 x = 12 3.54 x = 3/2 or – 3/2
2.2 8:12:21
3.55 x = 4/3 or –3
2.3 2:3:4
3.56 x = 1 or –1
2.4 N 27,500
3.13 y+1 3.57 x = 2 or 7
2.5 N384.62, N 961.54, N1153.84
17 3.14 –10
2.6 3.15 –14
1000 mn
3.58 y=
2.7 N 4.00 3.16 5 x − z 2m
2.8 N3,200 and Ade N 900, Chi N1,100 3.17 (a + c)(b – d) m 
3.18 4xy(ab – ac + d) 3.59 (i) n = 5m − p  ii.  26.8 to
2.9 434  2 
2.9b N1,600 3.19 (3a + 4b)(2x – 3y)
3sf
2.10 Le1,800 at 7% and Le1,200 at 8% 3.20 (x – 3y)(x – 2)
2.11 2730 – 2000 = 730 3.21 (n + q) 9 y 2 N 2 − 4m
3.60 x =
2.12 £3,634.2 3.22 (x+b) 2
2.13 (a) GHC 560, (b) GHC 7.5 3.23 3(7x – y)(7x + y)
3.24 3(7x – y)(7x + y) 2 Eg
3.61 u = v −
2
2.14 N3,690.00 to 3 s.f
2.15 i. N 458,750 ii. N 5,734.375 3.25 (5a – 3b)(5a + 3b) m
iii. N 65,098.96 3.26 1133 H (T − 1)
2

2.16 GHC 6,352.94 3.27 (3r + 4s) 3.62 k =


3.27b 7x(x + 2) T2 − T
2.17 N500,000.00
2.18 N 450.00 3.28 (n – 3)(n + 27)  v 
2.19 30.4% 3.29 (1 – x) 3.63 r = 100  t −1
2.20 10% profit 3.30 (5 – x)(7 + x)  p 
2.21 10 plates 3.31 (2a – 3b)(a + 5b)
2.22 gain : 25% 3.32 –6 2 A − bh
2.23 2% 3.64 a=
h
2.24 G H C 38 (14%)

524
q (hr − 1) 4.40 7 5.11 3−20x +12
3.65 p=
ht − 1 4.41 x = 1 or 2/3 5.12 3/2
bx 4.42 x = 3/2, y = – 3/2 5.13 21/2
3.66 a = 5.14 x = 5/2 i.e 2.5
x− y 5.15 x = 5/4
2qr − rs 4.43 x  – 11 5.16 n = 3
3.67 p= 5.17 x = 1
6 4.44 x> 9
2s − r
17 5.18 x = –2
3.68 u= 4.45 x  – 47 5.19 x = 2
s − 2v 4.46 x < 2 1/ 4 5.20 x = –1
fu 4.47 x < – 1 /2 5.21 2
1

3.69 v = 3

u− f 4.48 x  11/2 5.22 a x = 2 and y = 1


dhp 4.49 x  – 12 5.22 b x = 2, y = - 1/2
3.70 m = 4.50 x  11/7 5.23 x = 1
t − dh
5.24 2, 1
m
3.71 n = 4.51 – 3  x < 4 5.25 x = –1 or 1
mt − 1
4.52 – 3  x  4 5.26 x = 1
4.53 x  3 or x  – 4 5.27 x = 1/5
4.54 y  -3 or y  4 5.28 x = 5/9
4.1 4
4.2 10 5.29 3
/4
4.3 30 4.55 –6 –5 – 4 –3 –2 –1 0 1
5.30 11/2
4.4 7 4.56 ( i ) x < –3 5.31 0 (log101= 0)
4.5 x = 3, y = 4 5.32 1
/2
4.6 x = 8, y = – 3 ( ii ) – 4 –3 –2 –1 0 1 5.33 9
4.7 y = –4 5.34 33/10 (33/10)
4.8 x = 5, y = – 1 4.57 ( i ) x  – 2 5.35 – 1/2 (– 2/4)
4.9 x = 1, y = –1 ( ii ) 1

4.10 x = 5, y = – 3 –3 –2 –1 0 1 2 5.36 33
4.11 22 5.37 –11/2
3
(– 2 )
4.58
4.12 4 -1 0 1 5.38 1.5441
4.13 4
4.59 x<3 5.39 p=8
4.14 g = –1
5.40 12
4.60 5.41 10
/8 = Q i.e 5/4 = 11/4
4.15 x2 – 2x – 15 = 0 -2 0 2
5.42 x = 33 i.e 27
4.16 x2 – 8x + 15 = 0 4.61 –4<x3
5.43 2 = x
4.17 2x2 – 5x + 2 = 0 4.62 – 2< x  3 6
5.44 /25
4.18 6x2 + x – 2 = 0 4.63 –5<x2 1
4.19 4x2 – 4x – 3 = 0 5.45 logy = 3log 2 + 2 log m
4.20 2x2 + 3x – 14 = 0 4.64 5.46 15/9
4.21 ( -1, 3 ) –3 –2 –1 0 1 2 3 4
5.47 y = –1 , x = –68 or y = 4, x
4.22 –1 4.65 –3< x  5 = 12
4.23 1 5.48 x = 16 , y = 4
4.24 5 or -2 4.66 5.49  10
–6 –5 – 4 –3 –2 –1 0 1 2 3
4.25 x = 12/3 or x = -1/2 5.50 m = 14 or 32
4.26 –3, 9/2 4.67 – 5  x <1
4.27 –9, 2/3 5.51 n = -1 or -5
4.28 2 or -5
4.29 – 3/2
4.30 1, –1/5 6.1 0.06425
5.1 7–2 6.2 5.02  10 – 4
4.31 x = 0 or 2 −
25

4.32 y = 0 or 9 5.2 3 6 6.3 1.6256  10– 2


4.33 0 5.3 4 6.3b 3.18  109
4.34 16 5.4 21/2 6.4 6.02 10 – 9
4.35 p = -6 and k = 36 5.5 1/3 6.5 5.4  10– 3
4.36 400 5.6 76 6.6 2.23  103
10
6 5.7 /9 6.7 9.687  102
4.37 2
2 5.8 8a– 2 6.8 1.055  102
1
3 5.9 /9 6.9 9.778  10– 2
4.38 x = 2 or 5 5.10 1 6.10 1.368  10 2
4.39 –3 and –1
525
6.11 10 6 8.9 III only 9.21
6.12 8.4  10 – 3 8.10 ii. 5 – 2 = 3 B26 N18
U40

6.13 7  104 8.11 26 - x x 18 - x


6.14 740.97 8.12 3
6.15 17.57 8.13 1.2 6

6.16 0.0369 to 3sf 8.14 1.5 i. x = 10 ii. 16 iii. 8


6.17 121.0 to 3sf 8.15 1.1
8.16 1.41 9.22 x=5
8.17 2.76 9.23 1
8.18 1.4 9.24 12
7.1 35ten = 100011ten
8.19 i. N12 ii.N14, N15, 3.55 9.25 i. 4 ii. 10 iii. 13 iv. 20
7.2 37ten = 100101two
8.20 43 9.26 i. 18 ii. 17 iii. 101
7.3 1111two
8.21 1 9.27 3
7.4 0.2758
8.22 i. 5.47 ii. 5 iii. 5 9.28 15
7.5 11201.100 three
8.23 54 is 7 55 is 5 56 is 7 9.29 7
7.6 0.011two 9.30 i. 8 ii. 18
7.7 2100.12 four 8.24 9
9.31
7.8 207/9 ten or 29.78ten C24
U60

7.9 1078 8.25 20 is 4.2 21 is 3 22 is 6 8-x


F25

7.10 7 8.26 28.75, 29.25, 26.75, 22.75 x

7.10b 8.27 28, 25 6-x 5-x

7.10c 5 M 22
7.11 seven b.i x = 3, b ii. 13 b. iii 15 b. iv
7.12 2 9.0 { Multiples of three }
9.1 Their intersection is an empty set 14
7.13 101101 two
7.14 n=6 9.2 {2, 5, 7, 8, 10, 11, 13, 14}
9.3 a. A i.e  b. {2, 3, 4, 6, 7, 8, 9} 9.32
7.15 x= 6
7.16 x=8 9.4 {0, 6}
9.5 P = {–5, –3, 2, 4} 10.1 1–x
7.17 y=1 10.2 3
/36 i.e 1/12
7.18 4 = y and x = 6 9.6 {s, p, m, e}
10.3 i. 6/36 i.e 1/6 ii.
26
/36 i.e
7.19 7 = y and x = 6 9.7 i. {3, 8} ii. {1, 2, 4, 5, 6, 7, 9, 10} 13
/18
7.20 200 9.8 i. {10, 12, 14} ii. n(X1Y) = 1
10.4 bi. 5/36 bii. 5/6 biii. 35/36
7.21 9.9 {3, 5, 7} 7
10.5 /20
7.22 108 9.10 Q 10.6 1
/3
7.23 141 five 9.11 {4, 6} 10.7 4
/13
7.24 34five, 21six, 22three 9.12 495
U 480
A 10.8 i. ii.
1 973 973
7.25 1+3 6 B
9
4
2, 3
5 10.9 /10
7.26 2 3 C 10.10 1
/20
6, 8
5 3 10.11 b 1/12 c 5/9
7.27 7, 9 10
3 10.12 58 is 13/15 59 is 11/12 60
7.28
ii. AC = { } iii. AB1 = (1, 4} is 2/15
3− 2
7.29 17 – 12 2
9.13 10.13 0
P Q
1 10.14 i. 11/36 ii. 5/9 iii. 1/6
7.30 3 10.15 3/5
3 R 10.16 4/25
7.31 10 2 9.14 A(B1 C) 10.17 2/5
7.32 2 9.15 R1S 10.18 1/3
7.33 11/3 9.16
......................... 10.19 1/6
.........................
.........................
U 10.20 5/36
.........................
G
.........................
.........................
.........................
.........................
E
......................... 11.1 p = 29/2 and q = 18
.........................
......................... 11.2 x = 1, y = 10
8.1 13
11.3 x = 8 and y = 13
8.2 9 9.17 (AB)1
11.4 41
8.3 6 9.18 (AC)B1 11.5 3
8.4 y=9 9.19 P1QR1 11.6 Un = 18 – 7n
8.5 4.0 9.20 36 boys 11.7 2(3n – 1)
8.6 13
11.8 3n + 2
8.7 7
11.9 73
8.8 mode
11.10 –44
526
11.11 i. a = -5 and d = 16 ii. 331 32 14.1 5 3m
11.12 2 12.22 i. w = 26 – ii. t = 2/3
t 2 14.2 14.5m = x
11.13 0 1 14.3 53.130
11.14 ( i ) 3 ( ii ) 0 12.23 i. c = 830 + n ii. GH₡ 930 14.4  = 36.870
(iii) n2 – 9n – 90 = 0, n = 15 5
12.24 Le1600.00 14.5 55 m
11.15 n(3n + 2)
11.16 12 n(n + 1) 12.25 45km/h, 14.6 48.3m to 1 d.p
14.7  38.16m to 2 d.p
11.17 6144 14.8 1000m
11.18 45 14.9 69.3m
11.19 81 2 14.10 y = 260m to 3 s.f
2− 6
11.20 2 13.1 14.11  373m
4
11.21 2 14.12 x = 7.15m to 2 d.p
2
11.22 1
/2 13.2 14.13  100 to the nearest degree
11.23 3 2
14.14 (alternate angles) 720
11.24 r = 3, a = 4/9 , S5 = 484/9 13.3 1
14.15 21 3 m
11.25 r = 1/3 , S5 = 242/81 13.4 5
13.5 1
/2 14.16 i. 150m ii.913m
11.26 r = 2 , T5 = 640/7 14.17 38.9m
11.27 8 13.6 – tan 600
13.7 3 14.18 9.60m
11.28 128
11.29 10 13.8 2100 and 3300
11.30 – 1/4 2
13.9 –
11.31 i. 268 ii a. n = 7 ii b.10 = n 2
11.32 50 13.10 –1 15.1 600
11.33 127 13.11 1500 and 2100 15.2 300
11.34 1 + (–2)n 13.12 ½ 15.3 1000
11.35 5n – 2 13.13 2500, 2900 15.4 2700
13.14  = 7.50 15.5 700
13.15 x = 400 15.6 1200
12.1 R = 2.84 ohms 13.16 169
/25 15.7 x = 490 , y = 3110
12.2 p = 12 13.17 7
/25 15.8 420
12.3 B=4 13.18 9
/41 15.9 600
12.4 216 15.10 170
b2
12.5 221/2km 13.19
a2 15.11 2160
12.6 80 31 15.12 700
12.7 0.05v 13.20 /20
13.21 12
/13 15.13 330
12.8 p=2 1 15.14 640
12.9 1
/6 13.22 /3
13.23 11 15.15 m = 500 , n = 600
12.10 Q = 150 15.16 700
12.11 Q=5 4 13
13.24 15.17 1220
12.12 z=4 13
31 15.18 700
12.13 18.9 13.25 /12
15
4B 13.26 /17
12.14 ( i ) A = ( ii ) A = 64 13.27 130
9C 2 15.19 1
/3
12.15 1
2 /2 kg/cm 3 13.28 x = 3 2 2
15.20 32.31km
C1 2 13.29 600 and 3000 15.21 3.78cm
12.16 = 13.30 600
C2 9 15.22 11.59cm
13.31 i. 1.3383 ii. 0.3839 15.23 1400
4R
12.17 E= a. 36 15.24 2300
d2
14 3 15.25 2200
b. 4.899 c. 9.27% 13.32 m
3 15.26 2300
12.18 b
P = aQ + 13.33 x = 21.65m to 2 d.p 15.27 1580
Q
13.34 x = 12.2 cm 15.28 25m
12.19 x is partly constant and 13.35 10m = x 15.29 073.320
partly varies as y 13.36 6 3 15.30 36.90
48 8 13.37 12m 15.31 17km
12.20 ( i ) y = + x2
5 5 13.38 10.95cm 15.32 Option B
( ii ) y = 35.2 13.39 290 15.33 25km
12.21 y = 4x + 3 13.40 10(3 + 3 )cm 15.34 781.02km
15.35 13m

527
15.36 2400 18.5 A =1000 B = 800 C = 500 18.16
15.37 8.7km D = 700 E = 600 18.17 y = ax2
15.38 1840 18.6 Japan = 360 U.S.S.R. = 900 18.18 x2 –3x–1 = 0
15.39 19.08km Canada = 620 U.K. = 740 18.19 (a) x = 1 (b) Slope = 8
15.40 ( i ) PR = 276.80km USA = 540 China = 440 (c) x = – 0.6 or 2.6 (d) 0.3 < x
Total distance covered 11
18.7 4 20 5 /20 < 1.7
= 200 + 250 + 276.80 18.8 Hint : 2x2 – 4x + 1< 0 is 2x2 – 4x + 3 < 2
 727.8km to 3s.f
(ii) 74.260  740 to the nearest degree 18.20
18.9 b 10.5 + 3 = 13.5 c 7/10 Intermediates: 1/2 , 3/2, 5/2, 7/2
15.41 i XZ = 49.29km  49.3km to 1 d.p 18.10 Hint : 0, 0.5, 1.5, 2, 2.5, 3, 3.5, 4
Interval Tally freq class boundaries
(ii) 239.60  2400 21 – 25 IIII II 7 20.5 – 25.5
(a) 72.2cm
15.42 13km 26 – 30 IIII IIII I 11 25.5 – 30.5 (b) Ball reaches ground
31 – 35 IIII IIII 9 30.5 – 35.5 when y = 0, t = 4s
15.43 a i: 6.93km a ii : 10km 36 – 40 IIII IIII 9 35.5 – 40.5
(b) 12.2km 41 – 45 IIII I 6 40 .5 – 45.5 (c) At t = 3, vel = 36m/s
46 – 50 IIII III 8 45.5 – 50.5
15.44 ( i ) 32km ( ii ) 1670 18.21
(c) 25.5 + 3.2 = 28.7 year (a) 0 and 4
(b) x = 2 and y = 12
16.1 7000km (c) x = 2
16.2 1400 18.11
Age (year) class boundaries frequency (d) – 1 and 3.3
16.3 3200km 10 – 14 9.5 – 14.5 7
18.22
R 15 – 19 14.5 – 19.5 18
16.4 cos150 20 – 24 19.5 – 24.5 25 (d)(i) – 1.55 and 2.55
2 25 – 29 24.5 – 29 .5 17 (ii) x = 1,
16.5 (a) 30 – 34 29.5 – 34.5 9
N 35 – 39 34.5 – 39.5 4 (iii) 0  x  2
15 0 E
65 0 E
(b) ii 19.5 + 2.2 = 21.7 18.23
5 (a) 3x2 – 5x + 4 = 0 has no roots
Q (c)
P
16 (b) x = – 0.3 or 2
00 E
18.12 (c) – 1.9 < x<3.5
W
Class int. Tally f cf C/boundaries Hint 3x2 – 5x < 20 is 3x2 – 5x + 4 <
T 21 – 30 II 2 2 20.5 – 30.5 24
31 – 40 III 3 5 30.5 – 40.5
41 – 50 IIII I 6 11 40.5 – 50.5 18.24 (c) i – 0.2 and 1.85 ii. y =
51 – 60 IIII IIII I 11 22 50.5 – 60.5
S 61 – 70 IIII IIII 9 31 60.5 – 70.5
0.8
(b) i Thus latitude of P is 340N 71 – 80 IIII IIII 10 41 70.5 – 80.5 18.25
81 – 90 IIII 5 46 80.5 – 90.5 18.25 b
(b) ii.  4632km to 4 s.f 91 – 100 IIII 4 50 90.5 – 100.5

16.6 (b) (i) 64.5 (ii) 76.5 – 52.5 = 24 18.26 y = 4 cos x


28 14 18.27 (c) 150 and 1650
( i ) x0 E  430 to nearest degree =
(c) Pr (> 60kg) = 18.28
(ii)   450 to the nearest degree 50 25
18.29
18.13 18.30 y = 2cos x
16.7 i. 6131km ii. 5587km Height Class boundaries f cf 18.31
45 – 49 44.5 – 49.5 10 10
50 – 54 49.5 – 54.5 36 46
18.32 y + x  3, x ≥ 0, y≥ 0
55 – 59 54.5 – 59.5 64 110 18.33
17.1 (a) Mean = 14
60 – 64 59.5 – 64.5 52 162
(b) M.D  4.67 to 2dp 65 – 69 64.5 – 69.5 28 190 19.1 The area of P is equal to the area of Q
(c) S.D = 5.26 70 – 74 69.5 – 74.5 10 200 19.2 Three angles
17.2 Mean = 59.05 19.3 TYS (AAS)
(iii) 65.5 – 55 = 10.5
17.3 mean = 78.75 S.D  7.95 19.4 12cm
17.4 mean  1.2 m 19.5 8cm
18.14
17.5 Mode 51.86  52kg Marks Class boundaries f cf
19.6 13.33cm
30 – 39 29.5 – 39.5 11 11 19.7 4.5cm
40 – 49 39.5 – 49.5 25 36 19.8 9cm
18.1 600 50 – 59 49.5 – 59.5 45 81 19.9 4.0cm
60 – 69 59.5 – 69.5 15 96
18.2 60 19.10 4cm
70 – 79 69.5 – 79.5 10 106
18.3 20 N60,000 21 N200,000 80 – 89 79.5 – 89.5 14 120 19.11
18.4 JSS 1 = 480 JS 2 = 560 19.12 Corresponding angles are equal
(b) (i) 54.5 (ii) 65.5
JS 3 = 960 19.13 4.6cm
SS 1 = 640 SS 2 = 520 SS 3 = 440 96 − 36 60 1
(c) Pr(50 and 60) = = = 19.14 20 = n
8 120 120 2 19.15 15 = n
(b)
45 19.16 6 = n
18.15 linear 19.17 19
528
19.18 Octagon 20.18 36.7cm 21.25 104cm2
19.19 900 20.19 1080 21.26 1.68litres
19.20 1300 20.20 45.71cm 21.26b depth = 0.95m,
19.21 1500 20.21 36.9cm 7800litres would be needed
19.22 1280 20.22 61cm 21.27 53.08m3
19.23 1240 20.23 101.33cm 21.28 336cm3
19.24 20.24 61cm 21.29 a. 12cm b. 670 c.192cm3
19.25 600 20.25 4.19cm2 21.30 92cm2
19.26 35 Ans 580 36 Ans 1300 20.26 18 6/7 cm2 21.31 a. 360cm2 b. 400cm3
19.27 250 20.27 25cm 21.32 84cm3
19.28 II and III only 20.28 7 3 cm 21.33 i. 13.82cm ii. 67.130
19.29 600 20.29 (a) 10.2848cm (b) 6.129cm iii. 276.4cm iv. 289.6cm
19.30 I & II only (s in same 20.30 4.2cm 21.34 16cm
segment) 20.31 8cm 21.35 21.67m
19.31 900 20.32 17.32 cm 21.36 8.44 litres
n 20.33 7.36cm 21.37 522.67cm2
19.32 m=
2 20.34 1.2cm 21.38 r = 7cm
19.33 190 20.35 4.0cm 21.39 33 cm2
19.34 450 20.36 a 9.78cm b 10.09cm2 21.40 3.16m
19.35 500 20.36b 10cm 21.41 3.32cm
19.36 1200 20.37 154cm2 21.42 680litres
19.37 650 20.38 34cm 2 21.43 61cm
19.38 1000 20.39 346.5m2 21.44 76.3cm
19.39 130 21.45 4.9cm
19.40 900 20.40 5425cm2 21.46 3,011pipes
19.41 1120 20.41 i = 30cm2 ii  103cm2 to 3s.f 21.47 a. 1440cm2 b. 4667cm3
19.42 380 iii  20.4cm to 3s.f
19.43 1070 20.42 (i)  58cm2 (ii) 30cm
19.44 720 20.43 8 : 1 22.1
19.45 560 20.44 (a) 135.3cm 2 (b) 47.5cm 22.2
19.46 530 20.45 707cm2 22.3
19.47 1220 22.4 4.8cm and 9.2cm
19.48 770 22.5 PM = 4.3cm
19.49 720 21.1 2, 816cm3 22.6 PC = 7.6cm
19.50 65 0 21.2 629cm3 22.7
19.51 16cm 21.3 11 litres 22.8
19.52 35 ans 12.5, 7.7 36 ans150 21.4 4400cm3 22.9
21.5 1484.7cm3 22.10
20.1 13cm 21.6 (i) 4cm (ii) 62.9cm3 22.11
20.2 233m2 21.7 643m3 22.12 4.3cm and 1.3cm
20.3 6cm  5cm 21.8 749πm2 22.13
20.4 x=1 21.9 7cm 22.14
20.4b L 100m and W 80m 21.10 3.5cm 22.15
20.5 242.00cm2 21.10 b 55cm3
20.6 2112cm 21.11 2cm
20.7 122 4/7 cm2 21.12 i. 4cm ii. h = 11.3cm vol 190cm3
21.13 462cm3 23.1
20.8 16 3 cm2 21.14 i.13cm ii. 204.29cm2 iii. 314.29cm3 23.2
20.9 54cm2 21.14b r = 3.5cm, h = 9.90cm (b) i. (6  5)  (3  2) = 2 6 = 1
20.10 h = 35.42cm Vol = 127cm3 ii. n = 5
20.10b 4cm 21.15 12cm3 23.3 (i) 6 2  5 = (6 2)  5
20.11 17 3.35cm 18 27cm2 21.16 38cm3 = 55=4
20.11b 482.96cm and 2.24×105 cm2 21.17 2425.5cm3 (ii) n = 5 (iii) n = 1
20.12 96cm2 21.18 7cm
20.13 i.Rhombus, ii. AB = 17cm, 21.19 3.50cm
iii. 240cm2 21.20 132cm2 24.1 john is not older than me
20.14 5.0 21.21 240cm3 24.2 option C
20.15 16.13cm 21.22 495cm3 24.3 pq
20.16 44cm 21.23 7cm
20.17 cm 21.24 96cm2

529
25.1 (–2, 2) 26.23 x = –2, y = –4 28.21
1− x
2x
25.2 (0, 2) 26.24 1
25.3 1/4 26.25 – 102 28.22 /2
3− x
25.4 – 2 /3 26.26 7 28.23 2
25.5 24 ans 3/2 25 ans 2 13 26.27 – 15 28.24 ii 3x +1/2–5x iii x = 2/5
25.6 13 Ans 2 14 Ans y = 8 28.25 3 x − 5
25.7 2x + 3y – 7 = 0
25.8 y = 2x + 5 27.1 28.26 i 2x/1–x ii 2/x2 –2
25.9 4y = –3x – 7  7  iii f(x) largest domain in R
27.2   is x ≠ ±1, g(x) ≠ – 2
25.10 y + 4x – 11 = 0  − 9
25.11 4x + y + 14 = 0  4
25.12 3y – x + 10 = 0 27.3   28.27 a bijective function
25.13 3x + 2y = 5 6 b i x +3/ 4–2x b ii 5–x/
25.14 t = 9 10 17 2x–2
27.4
17 b iii 1/18 b iv –17/2
27.5 13 170
170
 −6 4 −8  8
 
27.6
26.1 65 29.1 x2 + 2x + 1
 − 10 0 − 12 
 − 14 − 10 27.7 1350 29.2 1
 2 
29.3 – 10
26.2  9 5  27.8 p − q = 101 29.4 P = 4 and Q = – 2
 − 16 
− 10 
 27.9 21 29.5 1
26.3 13 10  1 2
  27.10 (3i + 4j)
 0 24  5
29.6 A= 3 B=2 C=–1
 9 13  2i − j 2 2
26.4   27.11
5 7 3
5 8  29.7 +
3 4 x −1 x + 2
26.5 T =  11 5  27.12 i− j
15 11 5 5 29.8 1 3 1
  + −
27.13 I and II x x + 2 x +1
14 8 
26.6   27.14 1, II and III are true 29.9 P = 3, Q = –2
 7 7 29.10 P = 41/92 and Q = 5/96
 3 16  27.15 p . q = p q only
26.7   P + Q = 17/64
10 1  29.11 A = -1, B = 1 , C = 3
 3 11 29.12 5
26.8   28.0
 − 11 4 
 3 −1 0
  30.1 – 3/2x –5/2
26.9  4 −1 1
− 4 1 1 30.2 12x2 – 4x + 1
  28.1 both 1 – 1 and onto 28.2 30.3 2 + 2x
26.10 constant 30.4 6x + 2
3 − 5 28.3 {-1, 5, 13, 49}
26.11   30.5 6x – 2
6 − 4  28.4 {5, 2, 1} x2
26.12 (– 2 , 3) 28.5 {-3, -2, -1, 0, 1, 2, 3} 30.6 sec x 2

26.13 –5 28.6 g only 30.7 – sin x


26.14 zero 28.7 a.i R, x ≠ – 3/5
−3 (b) Not onto (c) –2<x< 3/2 30.8 3sin   − 3x 
26.15 x = 4, 2 
28.8
2 30.9 (6x2 + 3) cos(2x3 + 3x – 4)
28.9
26.16 –3 30.10 6(2x + 2)2
28.10
26.17 3 30.11 – 3sin3x
28.11 i. –3 ii –61/2
26.18 22 30.12 2 + 2x
28.12 i. 9 ii. x + 3x iii. x = 1 or 2
2
26.19 – 11 1
28.13 sin x2 − 2
− 8 − 5 30.13 3(2x + 3) 2
26.20   28.14 – 49
 3 2  30.14 20x(2x2 + 3)4
28.15 2x2 –16x +29
 −3 30.15 16x(2x2 + 7)3
 2  28.16 4
26.21  2  1
30.16 cosx – xsinx
− 3 5  28.17 15 (5x+2) 30.17 1 – 6x
 
 2  28.18 – /4 5
30.18 6(x2 + 1)
26.22 x = 1, y = 1/3 28.19 4 x + 11 30.19 (2x + 5)(6x –11)
2x − 5
30.20 (–2x2 + 6x+10)/(x2 + 5)2
28.20 7–x 30.21 (6 – 2x2)/(x2 + 3)2
530
30.22 2 31.10 y = x2 + 7x - 18
(1 − x ) 2
31.11 1
2 31.12 18
30.23 + 4
x3 5 31.13 1.0
1 31.14 42/3
30.24 31.15 271/3 square units
( x + 1) 2
31.16 27square units
30.25 2x 2 − 6x − 8
31.17 11sq units
(1 + x )4 31.18 9
30.26 −
1
31.19 10
(
x −1
2 3/ 2
) 31.20 37/12 sq units
3 cos 3x + 2 sin 3x 31.21
30.27
e −2 x 31.22
30.28 (4x + 9x2)/(2x2 + 3)3/2
4 31.23
30.29 (1, 5)
30.30 (–2, – 6)
30.31 /52

30.32 -1/8
30.33 1
30.34 8
30.35 3
30.36 1
/3
30.37 –1.5
30.38 5
30.39 41
/16
30.40 –4
30.41 0
30.42 i. -24 ii.Min pt (5, -101)
Max pt (-1, 7)
30.43 ( -2, 10 )
30.44 (-2, 13 )
30.45 (1/2, 11
/4 )
30.46 (5, 75)
−( 2 x + y )
30.47
x
30.48 14
30.49 – 4/5
30.50 ( ax + hy )

hx + hy
30.51 3/2 i.e 11/2
30.52 5
30.53 0.88cm2s–1
30.54 6cm2/sec
30.55 0.2cm2/sec
30.56 0.08mm/s

x2
31.0 + sinx + c
2
x4 3
31.1 + 2x + c
4 3
31.2 1 cos6x + c

6
3
31.3 /2 sinx + 2cosx + c
31.4 Loge 3
31.5 7
/6ln(x2 – 3)
1
31.6 /8ln(x4 +1)
31.7 8
31.8 2y = 4x3 – x2 + 4x + 3
31.9 2x2 – 3x + 3
531

You might also like

pFad - Phonifier reborn

Pfad - The Proxy pFad of © 2024 Garber Painting. All rights reserved.

Note: This service is not intended for secure transactions such as banking, social media, email, or purchasing. Use at your own risk. We assume no liability whatsoever for broken pages.


Alternative Proxies:

Alternative Proxy

pFad Proxy

pFad v3 Proxy

pFad v4 Proxy